Download as docx, pdf, or txt
Download as docx, pdf, or txt
You are on page 1of 839

DAFTAR ISI

??? (2)

DES 2007 (25)


JAN 2010 (45)
DES 2008 (46)
??? (76)
DES 2010 (99)
DES 2011 (126) key
JUNI 2012 (143) key
DES 2012 (159) ??
JUNI 2013 (198) key
DES 2013 (177) key
JUNI 2014 (219) key
DES 2014 (241) key
MEI 2015 (263) ??
DES 2015 (285) ??
JULI 2016 (312) key
DES 2016 (334) key
JULI 2017 (365) key
JAN 2018 (388) key
FEB 2018 (423) key ?
JULI 2018 (456) ??
SEPT 2018 (490)??
NOV 2018 (523) ??
MAR 2019 (559) key ?
JUNI 2019 (592) key ( - 11 nomor)
CHOOSE THE MOST APPROPRIATE ANSWER

1. In the following condition Ultrasound has beneficial effect : C


A. Sudeck’ atrophy
B. Shoulder-hand syndrome grade III
C. Joint stiffness
D. Phantom pain

2. The appropriate position of SWD electrodes in an obese patient with hip joint pain Is : D
A. Coplanar method
B. Contraplanar method
C. Cable method
D. Cross-fire method.

3. All of the following are the gait determinants, EXCEPT : C


A. Vertical displacement of center of gravity
B. Horizontal dip of the pelvis
C. Extension of the knee during stance phase
D. Cadence

4. Cubital Tunnel syndrome is an entrapment neuropathy of the foilowing nerve: A


A. Ulnar
B. Median
C. Radial
D. Musculocutaneus

5. In lateral epicondylitis the overused of the following muscle is very important


for its development :B
A. Extensor carpi ulanaris
B. Extensor carpi radialis brevis
C. Extensor carpi radiali longus
D. Extensor indicis proprius

6. The nerve root(s) mostly affected in an Erb’s palsy is / are :B


A. C4,C5
B. C5,C6
C. C7, C8
D. C8, Tl

7. Exercise program are contraindicated in all of the following cardiovascular


conditions, EXCEPT :B
A. Unstable angina C. Active pericarditis/myocaritis
B. Old myocardial infarction D. Acute thrombophlebitis

8. All of the following are diagnostic criterias of RA according to ACR, EXCEPT: C


A. Morning stiffness
B. Arthritis more than 3 joints
2
C. Assymetrical arthritis
D. Rheumatoid factor

9. The benefits of postural drainage in patient with COPD includes all of the
following, EXCEPT : D
A. Prevent of sputum accumulation
B. Improve air circulation
C. Improve oxygenation
D. Increase breathing working

10. The following sensory modalities are sent to the brain through the Lateral
Spinothalamic tract, EXCEPT : D
A. Pain
B. Touch
C. Cold
D. Vibration

11. The purpose of bracing in CP patient is : D


A. To control movements that interfere functions
B. To prevent/ correct contractures
C. To promote better balance
D. All of the above

12. The management of an acute GBS includes all of the following, EXCEPT : C
A. Rest
B. Passive ROM exercise
C. Vigorous. active exercise
D. Proper positioning of the limbs

13. The earliest sign and symptom of Causalgia and Sympathetic Reflex Dystrophy
Is: D
A. Edema
B. Muscle atrophy
C. Joint contracture
D. Persistent burning pain

14. Consevative treatment of Acute LBP includes all of the following, EXCEPT :D
A. Rest
B. Analgesics
C. Gentle active exercise of the limbs
D. William’s flexion exercise

3
15. All of the following are the clinical manifestation of Lumbar radiculopathy,
EXCEPT :C
A. Radiating pain to lower extremity
B. Dermatomal pattern of sensory deficits
C. Increased DTRs ( deep tendon reflexes)
D. Pain on SLR test

16. The following statement is true regarding the innervation of the extrinsic
muscles of the hand :A
A. Innervated by Radial, Median and Ulnar nerves
B Innervated by Radial and Ulnar nerves
C. Innervated by Median and Ulnar nerves
D. Innervated by Radial and Median nerves

17. The following intrinsic muscle of the hand is innervated by the Median nerve:B
A. Adductor pollicis
B. Half of Flexor pollicis brevis
C. Palmar interossei
D. Lumbricals 3 and 4 .

18. The following intrinsic muscle of the hand has dual innervation :A
A. Flexor pollicis brevis
B. Abductor pollicis brevis
C. Opponens pollicis
D. Adductor pollicis

19. When the MCP joints are fixed in full flexion, the extension of PIP and DIP
joints are done by the following muscles : C
A. Extensor digitorum communis and.extensor indicis proprius
B. Extensor digitorum communis.and the Interossei
C. The Interossei and the Lumbricals
D. The Lumbricals and Extensor digitorum communis |

20. The function of all of the the following extrinsic muscle of the hand is to extend
the fingers, EXCEPT : A
A. Extensor‘carpi radialis longus
B, Extensor indicis proprius
C, Extensor digiti quinti
D. Extensor digitorum communis

21. All of the following statements are True regarding the carpal tunnel, EXCEPT: C
A. A very narrow tunnel
B. Covered by the tranverse carpal ligament
C. The base is made by the metacarpal bones
D. The Median nerve is the only nerve passing through this tunnel

4
22. All of the following extrinsic muscles of the hand are located very superficially
in the forearm, EXCEPT : A
A. Flexor pollicis longus
B. Flexor carpi ulnaris
C. Flexor carpi radialis
D. Palmaris longus

23. The Buchard’s nodes of an osteoarthritic hand arc typically located around the
following joints : B
A. DIPs
B. PIPs
C. MCPs
D. Carpometacarpals

24. The Boutonierre deformity in a Rheumatoid hand is typically located in the


following joint : D
A. Radiocarpal
B. Carpometacarpals
C. MCPs
D. PIPs

25. De Quevain’s disease is a stenozing tenosynovitis of the following muscles : D


A. Abductor pollicis brevis and Extensor pollicis brevis
B. Abductor pollicis brevis and Extensor pollicis longus
C. Abductor pollicis longus and Extensor pollicis longus
D. Abductor pollicis longus and Extensor pollicis brevis

26. A serious clinical syndrome consisting of Low Back Pain, lower limb weakness,
saddle anaesthesia with bowel and /or bladder incontinence most probably is
coming from : D
A. Lumbar hemiated disc
B. Lumbar stenosis
C. Lumbar spondylolisthesis
D. Cauda equina syndrome

27. A severe herniated disc between L4 — LS most probably leading to weakness of


the following muscle : B
A. Extensor digitorum longus
B. Extensor hallucis longus
C. Extensor digitorum brevis
D. Peroneus longus

28. The following hand grasp/grip requires very high precission component : D
A. Spherical grasp D. Pincer grasp
B. Cylindrical grasp
C. Scissors grasp

5
29. The following is not the early pathological feature of OA : A
A. Joint cartilage cracks
B. Swelling of the joint cartilage
C. Increased joint cartilage water content
D. Loosening of collagen framework

30. All of the following enzymes play important roles in thé degradation processes
in OA, EXCEPT : D
A. Proteinase
B. Stromelysin
C. Collagenase
D. Cyclooxygenase

31. In degenerative joint disease of the spine : A


A. Vertigo and visual disturbance may occur
B. Anterior spurs are the most common cause of symptoms
C. Radicular pain is rare
D. Ankylosing between the vertebrae never occurs

32. Which statement is TRUE regarding Calcitonin ? C


A. It is available in an oral form
B. It is the first line therapy for osteoporosis prevention and treatment
C. It has analgesic property
D. Its concomitant use with calcium is contraindicated

33. The following is the typical radiographic feature of an osteoathritic joint : A


A. Marginal osteophytes
B. Marginal erosions
C. Juxta-articular osteoporosis
D. Joint space preservation

34. Which of the following is a general treatment principle for knee osteoarthritis? B
A. Avoid extreme ROM exercise
B. Strengthen the quadriceps muscles
C. Encourage a high impact aerobic home exercise program
D. Avoid the use of modalities

35. According to the American College of Rheumatology guidelines, which agent is


recommended as first-line medication in OA of the hip ? D
A. Prednison
B. Viscosupplementation
C. Oral gold treatments
D. Acetaminophen

6
36. What is the primary role of Methotrexate in the treatment of Rheumatoid
Arthritis? D
A. To reserve already present joint deformity
B. To cure the disease
C. To provide analgesia and anti-inflammation .
D. To slow or prevent joint destruction and loss of function

37. The most successful treatment for relieving pain and swelling in an acute gout is
intra-articular injection of : A
A. Steroids
B. Colchicine
C. Methotrexate
D. Tumor Necrosing Factor

38. All of the following food contains high purine component, EXCEPT: D
A. Peking duck
B. Fried pigeon
C. Sardine
D. Fresh fish

39. The following test will be probably positive in the early stage of Ankylosing
spondylitis : B
A. Schober test .
B. Gaenslens test
C. Tinel test
D. Pattrick test

40. The following deformities are typical in an advance case of Rheumatoid


Arthritis, EXCEPT : D
A. Piano-key sign
B. Boutonierre
C. Ulnar deviation
D. DIP joint subluxation

41. Which of the foiiowing statements about diagnostic studies in OA is FALSE ? D


A. Radiographs often show joint space narrowing and periarticular
osteophyites
B. Joint fluid analysis should be pursues in an patient with known knee OA
who presents with a swollen, warm and painful knee
C. Synovial fluid analysis on OA typically shows leucocyte count less than
2000 / mm
D. There is good correlation between symptoms and degree of joint
destruction on radiographs

7
42. One of the following non-pharmacologic managements for Osteporosis is
WRONG : C
A. Calcium
B. Vitamin D
C. Hormon replacement therapy
D. Prevention to fall

43. The best measurement for assessing risk for future fractures in osteoporosis is: C
A. Weight
B. Age
C. T-score
D. Z- score

44. Thefollowing sport activities are recommended in an old osteoporotic lady,


EXCEPT: A
A. Jogging
B. Walking
C. Tai Chi
D. Light morning gymnastics

45. The following is gold standard for osteoporosis therapy: B


A. Biphosphonate
B. Biphosphonate + Calcium
C. SERM + Calcium
D. Calcitonin

46. Which of the following physical limitations is must common in patients with
peripheral vascular disease ? A
A. Localized muscular fatique
B. Low cardiac output
C. Chronotropic insufficiency
D. Exaggerated heart rate in response to exercise

47. All of the following are the absolute indication for terminating the exercise
testing, EXCEPT: D
A. Moderate to severe angina
B. The systolic blood pressure dropped more than 10 mmHg
C. Subject’s desire to stop
D. Increased blood pressure

48. All of the following are the components of exercise for the elderly person,
EXCEPT: B
A. Intensity
B. Load
C. Duration
D. Frequency and mode

8
49. The type of exercise mostly important for geriatrics patient is: A
A. Aerobic conditioning
B. Strengthening exercise
C. ROM exercise
D. Agility exercise

50. The most frequent fracture in geriatrics patient due to fall is : B


A. Femoral shaft
B. Femoral neck
C. 1/3 proximal femur
D. 1/3 distal femur

51. The following is the most important movements of the Ankle-Mortise: A


A. Plantar flexion — Dorsal flexion : .
B. Internal— External rotations
C. Abduction — Adduction
D. Inversion — Eversion

52. Schober’s test is routinely done if the following disease is suspected : C


A. Schuermann disease
B. Pott’s disease .
C. Marie-Strumpell disease
D. Werdnig — Hoffmann disease

53. Lesion of the following nerve roots will lead to a classical “winging of the
scapula” : B
A. C4, C5 and C6 roots
B. CS, C6 and C7 roots
C. C6, C7 andC8 roots
D. C7, C8 and T1 roots

54. If the following nerve is injured, the patient will not be able to ambulate with an
axillary crutches: D
A. Upper Subscapular
B. Lower subscapular
C. Axillary
D. Thoracodorsal

55. In Klumpke’s type of brachial plexus injury the following muscle will be spared: D
A. Flexor digitorum sublimis
B. Opponens digiti quinti
C. Palmaris brevis
D. Pronator quadratus

9
56. An injury affecting the Common peroneal nerve will lead to the following
pathologic gait : D
A. Waddling gait
B. Sailor gait
C. Equinus gait
D. Steppage gait

57. A severe pyriformis syndrome may lead to the following pathologic gait : B
A. Gluteus medius gait
B. Gluteus maximus gait
C. Quadriceps gait
D. Spastic gait

58. When you see a patient everytime he is walking always holding his thigh during
the stance phase, most probably he has lesion of the following nerve: D
A. Sciatic
B. Tibial
C. Common peroneal
D. Femoral

59. In a Brachial plexus injury involving the posterior cord the following muscle
will be paralyzed : C
A. Serratus anterior
B. Brachialis
C. Latissimus dorsi
D. Supraspinatus

60. The following statement regarding the Erb’s type of brachial plexus injury in a
newbom baby is WRONG : A
A. Mostly occurred in breech presentation labour
B. The Moro reflex is negative in the affected limb
C. The baby is sleeping in an assymetrical position of the upper extremities
D. The grasp reflex is positive in both hands

61. An indication of good prognosis for independent ambulation for CP patient if he


is able to do the following : D
A. Kneeling with assistance at 16 months old
B. Prone lying at the age of 15 months
C. Sitting with support at the age 24 months
D. Sitting without support at the age 24 months

62. All of the following primitive reflexes are positive in a healthy baby of 8
months old, EXCEPT: C
A. Placing reflex C. Palmar grasp
B. Parachute reflex D. Plantar grasp

10
63.A girl patient with Idiopathic scoliosis the following are the signs that the
Cobb’s angle might be increasing rapidly, EXCEPT: D
A. She has her menarch a month ago
B. Her breasts are growing bigger
C. Her pubic hairs are growing
D. The Risser’s sign is reaching to grade 4

64. The Milwaukee brace for scoliosis produces all of the following forces to.
correct the spinal curvature, EXCEPT: D
A. Upward-directed force
B. Dowriward-directed force
C. Medial-directed force
D. Lateral-directed force

65. One of the following statements is WRONG regarding the TLSO — F control
Spinal brace : A
A. It has the same name with Knight-Taylor brace
B. It has one Thoracic band, one Pelvic band and two lateral uprights
C. One of its indications is for thoraci¢ Pott’s disease
D. It will prevent the trunk flexion

66. All of the following statements are True regarding the Knucle-bender hand
splint; EXCEPT: B
A. It is catagorized as a dynamic splint
B. The indication is for a flexible Intrisic-plus hand deformity
C. It is frequently prescribed for the leprosy patients
D: It is quite simple and cheap technology

67. The following staterment is NOT TRUE regarding the “Cock-up hand splint”: C
A. It covers’ proximally up to the anterior part of the forearm
B. The distal end is not covering the MCP joints
C. The wrist is positioned at the angle of 15° dorsiflexion
D. It maintains the fingers movements optimally

68. All of the following are the requirements of good orthotics-prosthetics


materials; EXCEPT: C
A. Must be strong
B. Must be light
C.. Must be cheap
D. Must be durable

69. The part of the orthopedic shoe which stabilize the sub-talar joint is the: C
A. Shank
B. Quarter
C. Counter
D. Heel

11
70. The lace is the best closure system for a good orthopedic shoes because : D
A. Itis very simple and cheap
B. It is very simple and strong
C. It is very simple and durable
D. It is very good to follow the foot girth

71. The following level of amputations will lead to equinus gait, EXCEPT: A
A. Syme
B. Chopart
C. Lisfranc
D. Pirogoff

72. Which of the following part of the shoe maintains the longitudinal arch of the
foot: C
A. Inner sole
B. Outer sole
C. Shank
D. Heel

73. The most effective treatment for plantar fasciitis is : D


A. Steroid
B. NSAID
C. Surgery os
D. Custom orthosis

74. Flexion contracture of the hip may lead to all of the following, EXCEPT: A
A. Reduced lumbar lordosis
B. Oblique rotation of the pelvis
C. Tighthening of the iliotibial band
D. Secondary scoliosis

75. For chronic obstructive pulmonary disease the highesi risk to have exercise-
induced bronchospasin is due to : A
A. Low intensity continuous exercise
B. Moderate intensity ‘nterval exercise for long period
C. 24°-26° Celcius room temperature with high humidity
D. Continuous exercise with graded high intensity

76. The following is NOT the risk factor of COPD: D


A. Smoking
B. Air pollution
C. Low income group
D. Female

12
77. The following condition is NOT a contraindication for exercise program
prescribed to a diabetic patient with cardiovascular complications: C
A. Ketosis
B. Blood sugar : > 300mg %
C. Blood sugar : 180 — 250 mg %
D. Blood sugar : < 80 mg %

78. The maximal heart rate during exercise that is recommended for cardiac
insufficient patients is : B
A. 50-60% of MHR
B. 60 —80% of MHR
C. 80-90 %.0f MHR
D. 90- 100% of MHR

79. The risk of foot ulceration in diabetic patient is increasing by all of the
following, EXCEPT : A
A. Increased mobility of the subtalar joint
B. The presence of plantar foot callus
C. The loss of vibration sense
D. History of previous foot ulceration

80. A patient with DM has painful, swollen ankle, worst with weight bearing. X-ray
shows fragmentation of the articular surface, synovial fluid is clear. The choice
of orthosis is : B
A. AFO
B. Patellar tendon bearing orthosis
C. Orthopedic shoes with rocker bottom sole
D. Varus prevention strap

81. The muscles required for ambulation using cruthes are : A


A. Shoulder depressors, Triceps, Wrist extensors and Finger flexors
B. Shoulder depressors, Triceps, Wrist and Finger flexors
C. Shoulder depressors, Biceps, Wrist extensors
D. Biceps, Triceps, Wrist flexors

82. Exercise programs in adult patients with hereditary neuromuscular disorders : C


A. Are most effective when weakness Is rapidly progressive
B. Should use high resistance eccentric exercise
C. May increase maximal aerobic capacity and exercise endurance
D. Are ineffective if muscle strength is less than 75% of normal

83. The most common cause of work-related back injuries is: D


A. Prolonged sitting
B. Prolonged standing
C. Reaching overhead
D. Bending and lifting

13
84. Which of the following statements is TRUE regarding the blood supply to the
spinal cord: B
A. The great spinal artery of Adamkiewicks supplies the lumbar and sacral cord
B. The mid- thoracic region is most vulnerable to ischemia
C. The two anterior spinal arteries are branches of the vertebral artery
D. The watershed zone is at C6-7 which causes most cervical lesions to be complete

85. The highest risk factor leading to Stroke according to the Framingham study is: D
A. Diabetes mellitus
B. Dyslipidemia
C. Heavy smoker
D. Hypertension

86. In a stroke affecting the posterior cerebral artery may lead to the following
syndrome: D
A. Millard — Gubbler
B. Foville
C. Benedict
D. Dejerrine — Roussy

87. One of the following language problems is considered Fluent Aphasta : C


A. Global Aphasia
B. Broca’s aphasia
C. Conduction aphasia
D. Transcortical motor aphasia

88. The following clinical manifestation is NOT very frequent occurred in a stroke
affecting the Right brain hemisphere, : D
A. Superficial and deep sensory deficits
B. Left sided neglect
C. Visual memory deficits
D. Wemnicke’s aphasia

89. All of the following statistical results are TRUE regarding the functional
prognosis of stroke patients who underwent a good, vigorous and
comprehensive rehabilitation program, EXCEPT: D
A. 75% of patients will reach independent level of self-care or with
minimal help .
B. 75% will lead independent level of ambulation with canes / ambulation
device
C. Almost all patients will be able to control bladder and bowel
D. 30% of patients fall into severe disabilities and will be bedriddened

14
90. The following artery is NOT niginally coming trom the Vertebro-Basilar
system of the brain circulation : A
A. Middle cerebral artery
B. posterior cerebral artery
C. Superior Cerebellar artery
D. Anterior-Inferior Cerebellar artery

91. The Vertebro-Basilar arteries are the branch of the following arteries: A
A. Subclavians
B. Common carotids
C. Internal carotids
D. Extemal Carotids

92. All of the following are the cardinal signs of leprosy, EXCEPT : A
A. Claw hand
B. Anaesthetic maculas
C. Thickening of nerves
D. Positive Acid Fast bacilli

93. The eye blindness among the leprosy patients is caused by the damage of the
following cranial nerves : D
A. Facial and Abducens
B. Facial and Occulomotor
C. Facial and Optic
D. Facial and Trigeminal

94. The following measures are CORRECT regarding the management of the
insensitive feet of the leprosy patient, EXCEPT : D
A. Soaking the feet everyday
B. Apply oily substance after the soaking
C. Trim and smoothen the sharp edges of the soles
D. Ask the patient always to wear leather shoes when walking

95. The limb in the post amputation patient that is described by the patient as having
a definite shape and position, a tingling sensation in the absent limb which
maybe unpleasant but is usually not severe is called : B
A. Pre amputation pain
B. Phantom limb sensation
C. Phantom pain
D. Stump pain

96. The activity of climbing up the stairs requires the following amount of METs : C
A. 2 METs
B. 4 METs
C. 6 METs
D. 8 METs

15
97. The appropriate orthosis for a patient with Quadriceps gait caused by Polio is : B
A. AFO
B. KAFO
C. HKAFO
D. HKAFO + LS brace

98. The most. appropriate Wheelchair for a T1 Paraplegia AIS / Frankel A is : D


A. Mouth-operated wheelchair
B. Wheelchair with vertical bars on the handrims
C. Regular wheelchair
D. Regular wheelchair with higher backrest

99. The intermittent catheterization in Paraplegic patients is the method of choice


for bladder management because it Is : D
A. Very affordable now by most of the SCI patients
B. Very simple method now in modem medicine
C. Very much accepted by most of the patients
D. Very good to minimize the infections

100. All of the following conditions should be considered in the planning of


Bowel training in SCI patients, EXCEPT: D
A. The time of the bowel training
B. The patient bowel habits
C. The Gastro-Colic reflex
D. The age

101. The primary afferent fibers coming from the nuclear chain of the
muscle spindle have the following conduction velocity : B
A. 20-40 m/sec
B. 40-60 m/sec
C. 60-80 m/sec
D. 80- 100 m/sec

102. The afferent fiber coming from the GTO (Golgi Tendon Organ) belongs
to the following group of nerve fibers : B
A. A-α
B. A-β
C. A-ϒ
D. A-δ

103. The physiological process of muscle contraction is that every contraction


will be followed by relaxation. If the contraction is not followed by relaxation
the muscie will be continuously contracting. The following condition may be
Caused by : D

16
A. There are oversupply of ATP from mitochondria
B. There are oversupply of Ca from the endoplasmic reticulum
C. There are oversupply of Acetyl choline in the Neuro-Muscle junction
D. The Endoplasmic reticulum fails to reuptake the Ca from cytoplasm

104. Process of muscle contraction follows the following sequence of processes: C


A. Electrical -Mechanical — Biochemical
B. Mechanical - Electrical — Biochemical
C. Electrical — Biochemical — Mechanical
D. Biochemical _ Electrical _ Mechanical

105. The effect of Diazepam as a muscle relaxant is via the mechanism of : D


A. To inhibit Calcium ions release
B. To inhibit Acetyl choline
C. To stimulate Glutamate
D. To stimulate GABA

106. One of the effects of Baclofen as muscle relaxant is through its


mechanism of: C
A. Stimulating the Aspartate and Glutamate
B. Inhibiting, the Acetyl choline release
C. Inhibiting the Gamma motoneuron activities
D. Inhibiting the Alpha motorneuron activities

107. The first thing to do in the management of mild spasticity is : C


A. Give oral Diazepam
B. Give superficial heating such as hydrocollator warm compress
C. Remove all the possible of noxious stimuli
D. Give injection of simple nerve block such as phenol

108. From the research it shows that the best treatment to reduce severe
muscle spasticity in SCI patients is : D
A. Intavenous Diazepam
B. Intravenous Sodium dantrolene
C. Intrathecal Tizanidine
D. Intrathecal Baclofen

109. One of the following conditions has the highest possibility for successful
ambulation using HKAFO brace and bilateral axillary crutches : A
A. T12 Paraplegia AIS / Frankel A
B. T10 Paraplegia AIS / Frankel A
C. T8 Paraplegia AIS / FrankeI A
D. T6 Paraplegia AIS / Frankel A

17
110. All of the following Symptoms are the signs of the autonomic
dysreflexia in SCI Patient except : C
A. Blood pressure might increase up to 240/120
B. Sense of chest fullness
C. Heart rate might increase up to 140— 160 X/minute
D. Pounding headache

111. The absolute contraindication of Crede’s compression during the bladder


training in SCI patient is: B
A. Bladder infection
B. High intravesical pressure
C. Very high volume of residual urine
D. Bladder overdistension

112. The probable sexual functions of a male patient with T12 Paraplegia
AIS / Frankel A will be as follows: A
A. Good reflex erection, good emission, poor fertility
B. Good reflex erection. good emission, good fertility
C. Good reflex erection, good emission, no ejaculation
D. Poor erection, poor emission, no ejaculation

113. The most important thing that has to be done for SCI patient using an
indwelling catheter is : D
A. Irrigate the bladder every day
B. Change the catheter every week
C. Give the patient acidifying substance per oral (e.g.: Vitamin C) to
prevent stone formation
D. Ask the patient to drink water for 2 — 3 liters per day

114. One of the pitfalls of EMG examination is that with the EMG machine
we can get the following result: D
A. The level / location of motor unit pathology
B. The severity of pathology of motor unit
C. The motor and sensory conduction velocities
D. The etiology of motor unit abnormalities

115. All of the following factors will influence the results of EMG
examination, EXCEPT : D
A. The types of electrodes we are using
B. The patient body temperature
C. The age of the patient
D. The sex

18
116. In needle EMG examination, the minimal contraction is directed to get
all of the following data, EXCEPT: D
A. The shape of MUAP
B. The intensity of MUAP
C. The duration of MUAP
D. The recruitment of MUAP

117. All of the following are the characteristics of the fibrillation waves,
EXCEPT: D
A. The sound is typically like “diving bomber” "
B. They have regular appearance
C. The amplitude is very small
D. They appears spontaneously during muscle at rest

118. The presence of Nascent potentials in needle EMG is indicator of:B


A. Good prognosis
B. Reinnervation a
C. Partial denervation
D. Neuropraxia

119. All of the following are the characteristics of MUAP, EXCEPT: D


A. Under voluntary controls
B. The shape is mostly triphasic
C. The maximal amplitude is 3 mV :
D. it can be detected when the needle is inserted to the muscle

120. The following statements are all Correct regarding the H-reflex,
EXCEPT: A
A. It is a polysynaptic reflex
B. The ayerage value in adult is about 29 ms
C. This a typical test to see the abnormality of $1 root
D. The “H” is for Hoffmann

121. The following statements all are correct regarding the EMG machine,
EXCEPT: C
A. It is very expensive
B. The operator must be only the competence doctors
C. If this machine is not available its functions can be replaced by
Chronaxie meter machine
D. It needs a specially arranged room in order to get the best results

122. EMG examination of Duchene Muscular Dystrophy will show the


following findings regarding the MUAP: C
A. High amplitude, long duration D. Low amplitude, long duration
B. High amplitude, short duration
C. Low amplitude, short duration

19
123. One day after a nerve lesion, NCV test shows the following results:
Stimulation distal to the lesion gives a normal response while stimulation
proximal to the lesion gives no response. The conclusion is :A
A. Neuropraxia
B. Axon cachexia
C. Axonotmesis
D. Neurotmesis

124. All of the following are correct regarding the sensory NCV examination,
EXCEPT: D
A. Can be done with antidromic technique
B. Can be done with orthodromic technique
C. Can be done using ring electrodes
D. Can be done using needle electrodes

125. In a totally denervated of all muscles due to a severe Brachial plexus


injury, this can be categorized as pre-ganglionic lesions if we found the
following data in EMG examination: B
A. Motor NCV : no response, EMG : total denervation
B. Sensory NCV : good response, EMG : total denervation
C. Motor NCV : good response, EMG : total denervation
D. Sensory NCV : No response, EMG : total denervation

ANSWER A : if numbers 1, 2,3 are correct


B : if numbers 1 aad 3 are correct
C : if numbers 2 and 4 are correct
D : if only number 4 is correct
E : if all numbers are correct

126. The following is / are the gold standard(s) for diagnosis of osteoporosis :D
1. Ultrasound densitometry
2. CT scan
3. MRI
4. DEXA

127. All of the following methods of exercise are principally using the PNF -
approach:A
1. Kabat
2. Voss
3. Knott
4. Bobath

20
128. The following is / are the appropriate exercise prescribing for an arthritic patient :E
1. Isometric Strengthening exercise
2. Isotonic Strengthening exercise
3. Isokinetic Strengthening exercise
4. Aerobic and aquatic exercise program

129. The management of acute ACL (Anterior Cruciate Ligament) injury is /


are:B
1. Rest
2. Refer to orthopedic surgery
3. Cryotherapy
4. POP immobilization

130. Muscle tension is monitored primarily by :D


1. Extrafusal fibers
2. Muscle spindle
3. Motor endplate
4. Golgi Tendon Organ

131. The following are true about ACL:A


1. Prevents anterior displacement of tibia with respect to the
femur
2. Provides stability for lateral movements
3. Causes large effusion whwn tom
4. Acts in concert with the quadriceps

132. The following is / are correct regarding the Muscle Spindle :E


1. It will be stimulated if there is sudden elongation of the
muscle
2. Its main function is to monitor the muscle tone
3. The fastest primary afferent fibers are coming from the
nuclear bag
4. It is innervated by the Gamma motorneuron

133. The following is / are the test(s) to diagnose the knee meniscal tear:A
1. Mec Murray’s test
2. Pain toward the end range of tlexion when passively
flexing the knee
3. Apley’s compression test
4. Pivot shift test

134. The basic assessmeut of children patients with delayed growth and
development should include the following areas:A
1. Gross motor test 3. Language test
2. Fine motor test 4. 1Q test

21
135. What are the presdisposing factors for bursitis of the hip ?:E
1.Hemiparesis
2. LBP
3. Leg length discrepancy
4. Hip trauma

136. The following is / are the possible cause(s) of genu recurvatum at the
stance phase of gait :A
1. Quadriceps weakness
2. Plantar flexor spasticity
3. Quadriceps spasticty
4. Dorsiflexors contracture

137. Toddlers walk with :E


1. No heel strike
2. Little knee flexion during stance
3. A reduced stride with higher cadence
4. Absence of reciprocal arm swing

138. The following is / are the source(s) of shoulder pain in hemiplegic


patient:E
1. Subdeltoid bursitis
2. Rotator cuff tear
3. Bicipital tendinitis
4. Shoulder subluxation

139. The following is / are the contraindication(s) of postural drainage :A


1. Hemoptysis
2. Severe hypertension
3. Cerebral edema
4. Cystic fibrosis

140. The aim(s) of chest physical therapy in asthmatic patient is / are :E


1. To relieve brochospasm
2. To assist relaxation and gain control of breathing
3. To aid removal of secrets
4. To coordinate respiratory movements

141. The typical spasticity pattern In stroke is/ are :B


1. Retraction, depression and internal rotation of the shoulder
2. Protraction and external rotation of the shoulder
3. Flexion and adduction of the fingers
4. Forearm extension

22
142. In this following disease(s) the position sense could be abnormal:A
1. Brown-Sequard Syndrome
2. Tabes dorsalis
3. Subacute combined degeneration
4. Anterior cord syndrome

143. The general Principle(s) of exercise for osteoporosis include :E


1. Principle of specificity
2. Principle of progression
3. Principle of reversibility
4. Principle of initial values

144. Normal gait pattern :A


1. Consists of Stance and Swing phases
2. Stance phase is 60% and Swing phase 40%
3. There is a phase called “double support”
4. Needs normal ROM of the Hip, Knec and Ankle joints

145. The following syndromes are due to the disturbance of blood circulation
to the brain stem :B
1. Weber .
2. Dejerrine-Roussy
3. Foville
4. Parkinson

146. A stroke involving the Vertebro-Basilar system may give the following
manifestation(s):B
1. Ataxia
2. Global aphasia
3. Hemiplegia alternans facialis
4. Unhibited neurogenic bladder

147. The functional outcome of L2 Paraplegia AIS / Frankel A is / are :E


1. Independent bladder and bowel activities
2. Independent ambulation using bilateral axillary crutches
and KAFO
3. Independent in pressure sore relief
4. Requires no wheelchair for ambulation

148. The following is / are the appropriate management of chronic ankle


sprain in sport injury:A
1. Ultrasound diathermy
2. Laser therapy
3. Taping and straping
4. Icing

23
149. Early management of acute whiplash injury is / are :A
1. Rest
2. Analgesic and muscle relaxant
3. Soft cervical collar
4. Neck Cailliet’s exercise

150. The following statement(s) is / are correct regarding cervicai traction:C


1. The position must be in supine
2. Cervical spondylosis is one of the indications
3. Can be given to mild Rheumatoid Arthritis
4. The neck position is in mid-fiexion

GOOD LUCK

24
Soal Desember 2007

1.The following joint has two degrees of freedom of motion :


A. Shoulder
b. Elbow
c. Wrist
d. PIP

2. The following joint has one degree of freedom of motion :


a. Hip
b. Knee
c. Ankle mortise
d. Subtalar

3. The following joint is categorized as false joint :


a. Intercarpals
b. Subtalar
c. Ankle mortise
d. Coracoacromial

4. Branch of Kinesiology dealing with the geometry of movement is called :


a. Biomechanics
b. Kinematics
c. Kinetics
d. Dynamics

5. The following statement is WRONG regarding the center of gravity :


a. Located at the midline 1 inch in front of S2 vertebrae
b. Is the point where the body weight is concentrated on
c. The line of gravity normally passes this point
d. There are limitless cardinal planes that can be made through this point

6. The following ligament is categorized as long ligament :


a. Sitting
b. Standing
c. Standing and bending forward
d. Standing and bending backward

7. The following spinal ligament is categorized as long ligament :


a. Flavum
b. Intertransversum
c. Supraspinosum
d. Interspinosum

8. In the adult the length of spinal cord will reach up to the following lever of the spine :
a. L1-L2
b. L2-L3
c. L3-L4
25
d. L4-L5

9. The following is not the characteristic of the thoracic spine :


A. Has the joint surfaces with the ribs
B. Has relatively small intervertebral disc
C. Has no transversus foramina
D. Has the facet joints at the horizontal plane position

10. The number of plane(s) to analyze the body movement is / are :


a. One
b. Two
c. Three
d. Limitless

11. The formula of MECHANICAL. ADVANTAGE is :


a. EFFORT devided by WEIGHT
b. WEIGHT devided by EFFORT
c. WEIGHT ARM devided by EFFORT ARM
d. EFFORT ARM devided by EFFORT

12. The energy transfer of the following thermal modality is by CONVECTION method:
a. Cold Packs
b. Ice massage
c. Vapocoolant spray
d. Whirlpool bath

13. The cervical traction is contraindicated for the following condition:


A. Cervical spondylosis -
B. Cervical OA
C. Paracervical muscle spasm
D. Pott’s disease of the cervical spine

14. The following is not included as the major criteria for the diagnosis of Myofascial pain
syndrome:
A. Regional pain complaint
B. Taut band palpable in an accessible muscle
C. Some degree of restricted ROM
D. Pain elevated by elongating the muscle

15. The following IS NOT True regarding the electrotherapy:


A. Electrotherapy is applied transcutaneously to stimulate the nerve
B. To reeducate muscles
C. To treat condition due to circulatory impairment
D. For chronic muscle pain this given for 3 minutes per trigger point.

16. Electrical stimulation has the value primarily for the treatment of :
A. Spasticity
B. Rigidity
26
C. Denervated skeletal muscle
D. Flexion contracture

17. The following condition is Considered a Handicap :


a. Stroke patient ambulating with a walker
b. RA patient walking with platform chanes
c. A LBP patient walking with Canadian crutches
d. A SCI unable to climb the stairs with his wheelchair

18. A condition in which ultrasound therapy has beneficial-effect is:


a. Join contracture
b. Post mastectomy forearm lymphedema
c. Phantom pain
d. Stage 3 of shoulder hand syndrome

19. The most common cause of LBP in a community is.


a. Ankylosing spondylitis
b. Osteoporosis
c. Mechanical Origin
d. Spondylosis

20. The nociceptive system stars from the following receptor :


a. Free nerve endings
b. Muscle spindle
c. Ruffini
d. Krausse

21. The nociceptive system ends at the level of -


a. Sensory cortex
b. Midbrain
c. Pons
d. Medulla oblongata

22. The afferent fibers coming from the nuclear bag of the muscle spindle have the conduction
velocity of about :
A. 20-40 m/sec
B. 40 - 60 m/sec
C. 60 - 80 m/sec
D. 100-120 m/sec

23. The afferent fibers coming from GTO (Golgi Tendon Organ) are catagorized under the
following fiber type:
a.A-alpha fiber
b. A- betha fiber
c. A-delta fiber
d. C fiber

24. The following sensory modalities are brought to the brain via the tracts in the posterior
27
Column of the spinal cord, EXCEPT:
a.Light touch
b. Position
c. Vibration
d. Stereognosis

25. The lateral spinothalamic tract will be responsible to convey all of the following sensation
from the periphery to the brain, EXCEPT:
a.Superficial pain
b. Joint movements
c. Cold sensation
d. Touch sensation

26. In a brachial plexus injury if we found a “winging of the scapula” most probably the lesion is
located at the level of :
a.Roots
b. Trunks
c. Division
d. Cords

27. The long flexors of the hand have the innervation mostly coming from the following root :
a. C6
b. C7
c. C8
d. T1

28. The Latissimus dorsi muscle is innervated by the thoracodorsalis nerve which is a branch of
the following part of the brachial plexus :
a.Lateral cord
b. Medial cord
c. Anterior cord
d. Posterior cord

29. All of following muscles are innervated by musculocutaneus nerve, EXCEPT :


a. Brachioradialis
b. Biceps
c. Brachialis
d. Coracobrachialis

30. If in a brachial plexus injury we found a “wrist drop” this is due to the Iesion of the following
part of the brachial plexus :
a.Middle trunk
b. Medial cord
c. Posterior cord
d. Lateral cord

31. In Klumpke’s type of brachial plexus injury the following muscle is spared:
a. Flexor digitorum sublimis
28
b. Flexor digitorum profundus
c. Flexor pollicis longus
d. Extensor carpi radialis longus

32. In an advanced Carpal Tunnel Syndrome all of the following muscles will be very weak,
EXCEPT :
a.First Lumbrical
b. Flexor pollicis brevis (deep part)
c. Abductor pollicis brevis
D. Opponens pollicis

33. The Median nerve entrapment neuropathy may produce all of the following syndromes,
EXCEPT :
a.Posterior interosseous syndrome
b. Anterior interosseous syndrome
c. Pronator teres syndrome
d. Carpal tunnel syndrome

34. In a TOS (Thorasic Outlet Syndrome) the following condition will produce positive
hyperabduction test :
a.Hypertrophy of the Anterior Scalenus muscle
b. Hypertrophy of the Medial Scalenus muscle
c. Hypertrophy of the Pectoralis Major muscle
d. Hypertrophy of the Pectoralis Minor muscle

35. In an advanced “thick wallet syndrome” or “pyriformis syndrome” the following


pathological gait might be developed :
a.Quadriceps gait
b. Gluteus medius gait
c. Gluteus maximus gait
d. Steppage gait

36. The following muscle is spared in a Pronator teres syndrome :


a. Pronator teres
b. Pronator quadratus
c. Flexor digitorum sublimis
d. Palmaris longus

37. The most important step to do in geriatric rehabilitation is :


a. Assess the complete functional status
b. Recognize the daily activities of the patient
c. Set a complete cure as the goal of rehabilitation
d. Determine the medical status of the patient

38. The importance of geriatric rehabilitation :


a. Will increase the total helath care cost
b. Is an attempt to preserve the functional level for the rest of his/her life
c. There is no hope to restore function to the premorbid level
29
d. The geriatric patient can never increase his/her functional status

39. In the elderly most of the following functions will decrease, EXCEPT :
a. Muscle strength
b. Orthostatic tolerance
c. Systolic blood pressure
d. VO2 max

40. There is no routine physical examination in children. Each examination is individualized for
the Physiatrist, perhaps the most……………………. (soal tidak terbaca)

41. The following is the articulating muscle :


a. Diaphragm
b. Intercostal
c. Crycoarythenoid
d. Hypoglossus

42. Which maneuver is used to identify sacroiliac join pathology?


a. Lasegue
b. Gaenslen
c. Patrick
d. Thomas

43. Person with COPD typically experience exertional dyspnea when FEV 1 becomes Iess than :
750 ml
a.1500ml
b. 2200 ml
c. 3300 ml

44. A 50 year man with mild systemic hypertension has a marked increase in blood pressure
when he is performing his weight-lifting exercises. During active muscle contraction you
recommend :
a.Exhalation
b. Inhalation
c. Valsava mancuver (closed glottis) .
d. Shallow, rapid breathing

45. The primary focus of Rehabilitation for 2 year-old child newly diagnosed with cystic fibrosis
should be :
a.Anxiety reduction
b. Airway secresion management
c. Respiratory muscle excrcise
d. Respiratory muscle rest

46. All ofthe following charactcristics are correct regarding the Type I muscle fibers. EXCEPT:
a. The color is white
b. High myoglobin content
c. Low glycogen content
30
d. High mitochondria content

47. The following is the pathophysiology of the development of muscle spasticity :


a. The muscle spindle becomes less sensitive
b. The muscle spindle becomes very sensitive
c. The GTO becomes less sensitive
d. The GTO becomes very sensitive

48. All of the following are the complications of prolonged immobilization, EXCEPT:
a. Deep vien thrombosis
b. Orthostatic hypotension
c. Decreasing heart rate
d. Decreasing cardiac reserve

49. All of the following are the factors influencing the strength of muscle contraction, EXCEPT:
a. The increasing number of muscle fibers following strengthening exercise
b. The amount of ATPs available
c. The concentration of Calcium ions available
d. The number of motor units which are active during muscle contraction

50. The smallest unit of muscle contraction is the:


a. Skeletal muscle cell
b. Extrafusal fiber
c. Sarcomere
d. Actin and Myosin proicins

51. Descriptions of motor skills must take into account in all of the following factors, EXCEPT:
a. The direction of cach action ul cach joint
b. The force of joint actions
c. The timing, coordination and rhythm of joint actions
d. The non-integrated pattern of joint actions

52. All of the following statements are true regarding the TLSO, EXCEPT:
a. Extends from the sacrum to the inferior angle of the scapula
b. Used for support in pust spinal fusion
c. Stabilize the truncal paralysis
d. Stabilized in post scoliotic surgery

53. Volksman’s ischemic contracture is most often associated with fracture of the :
a. Clavicle
b. Humerus
c. Radius
d. Ulna

54. In injury of the sciatic nerve which of the following reflex in lost?
a. Patellar tendon reflex
b. Achilles tendon reflex
c. Cremasteric reflex
31
d. Bulbocavernosus reflex

55. In the syndrome of Erb-Duchene the following muscles are involved:


a. Suprasinatus, Infraspinutus and Opponces pollicis
b. Deltoid, Biceps and Brachialis
c. Deltoid, Biceps and Flexor digitorum sublimes
d. Subscapularis, Deltoid and Pronator teres

56. Children are expected to be able to differentiate between Right and Left is at about the
following age :
A. 2 years B.3 years C. 4 years D. 6 years

57. In disclocation of the hip, which of the following nerve is commonly injured ?
a. Femoral
b. Sciatic
C. Obturator
D. Superior gluteal

58. The Parkinsonian tremor has an average frequency of :


a. 1 per second
b. 6 per second
c. 20 per second
d. 40 per second

59. The following are absolute indications for terminating the exercise testing. EXCEPT:
a. Increased blood pressure
b. Subject desires to stop
c. Moderate to severe angina
d. The systolic blood pressure drops > 10 mmHg

60. Physical activities of the SCI patient are important in all of the following, EXCEPT:
a. Prevention of metabolic dysfunction
b. Psychological well-being
c. Decreasing risks of urinary stone
d. Stimulation of cord regeneration

61. The earliest sign and symptom of causalgia and sympathetic reflex dystrophy is :
a. Edema
b. Muscle atrophy
c. Joint contracture
d. Persistent burning pain

62. The conservative treatment of LBP in acute phase includes all of the following, EXCEPT:
a. Analgesics
b. Mild lumbar traction
c. William’s flexion exercise
d. Gentle active exercises of the limbs

32
63. All of the following are the signs of lumbar radiculopathy, EXCEPT:
a. LBP radiating to lower extremity
b. Diminished sensation in the affected dermatome
c. Exaggerated Achilles tendon reflex
d. Pain on SLR test

64. The limb in the post amputation that is described by the patient as having a definite shape
and position, a tingling sensation in the absent limb which maybe unpleasant but is usually
not severe, is called:
a.Phantom pain
b. Stump pain
c. Phantom limb sensation
d. Post amputation pain

65. All of the following are diseases in which the basic pathology is the degeneration/damage
the anterior horn cells of the spinal cord, EXCEPT :
a.Werdnig —Holtmann discasc
b. Jacob-Kreutzicld disease
c. Landry-Strohl-Guillain Barre disease
d. Heine-Medin disease

66. An example for limb deficiency in children included longitudinal intercalary is :


a. Phoeomelia
b. Radial aplasia
c. Below elbow amputation
d. Congenital absence of the tibia und medial pant of the foot

67. All of the following are the spinal primitive reflexes, EXCEPT:
a. Parachute
b. Flexor withdrawal
c. Extensor thrust
d. Crossed extension

68. Which of the following statements is NOT TRUE regarding calcaneal spur ?
a. Plantar faseiitis is clearly associated with plantar calcaneal spur
b. Plantar spur causes heel pain
c. The pull of plantar fascia on the medial calcaneal tuberosity that causes pain
d. By growing a heel spur, the body relieves tension on the fascia plantaris

69. The most elective treatment for plantar faseiitis is:


a. NSAID
b. Custom orthosis
c. Steroid
d. Surgery

70. The blindness occurred in leprosy patients is due to the damage of the following nerves:
a. Optic and Facial nerves
b. Trigeminal and Facial nerves
33
c. Trigeminal and Optic nerves
d. Optic and Occulomotor nerves

71. The characteristic signs of conus medullaris syndrome are all of the following, EXCEPT:
a. Lost of anal reflex
b. Perineal sensory deficit
c. Pyramidal Signs
d. Sphincter dysfunction

72. One of the following diseases will not Iead to gait problems due to deep sensory deficits :
a. Subacute combined degeneration
b. Tabes dorsalis
c. Syryngomyelia
d. Posterior cord syndrome

73. In the management of a young girl of 12-years-old with idiopathic scoliosis the following
procedures……………..(tidak terbaca), EXCEPT :
a.Check the …… (tidak terbaca) of the secondary sexual signs
b. Check if ….. (tidak tercaba) her first period and when (menarch)
c. Make X-ray study of the vertebral column every six months
d. Prescribe Milwakee brace inunediately

74. To reduce the severity of disability in patient with Rhemuatoid Arthritis the following have
To be done continuously, EXCEPT :
a.To avoid frequent exacerbation episodes
b. To give vigorous exercises every day
c. To prevent from any infections
d. To avoid severe physical and psychological stresses

75. A 40 years old woman has difficulty climbing the stairs after underwent a transvaginal
hysterectomy few days before. This is most probably due to :
a.Complication of the spinal anaesthesia
b. Psychogenic / hysterical weakness
c. Triceps surae muscles weakness
d. Neuropraxia of the femoral nerves

76. The pretibial group muscle is most active in the following phase of gait cycle:
a. Immediately following heel strike
b. At mid-stance
c. At toe-off
d. At mid-swing

77. Patient with cervical radiculopathy should avoid :


a. Cervical forward flexion
b. Cervical lateral bending
c. Heat modalities
d. Bed rest

34
78. 32 year old laborer with numbness and tingling in his right leg. SLR test is positive. This
means that :
a.Tension is transmitted to the nerve root immediately upon raising the heel off the table
b. Irritation of an affected nerve root has occurred
c. Raising the leg causes transient weakness of the affected Ieg
d. The patient is a malingerer

79. 4 year old girl with spastic quadriparetic CP is seeing you for wheelchair evaluation. She has
normal cognitive and language development, able to sit erect, marked increase tone in
bilateral lower extremities, truncal weakness but good head control, flexor posture of the
right upper extremity, poor fine motor control of the left upper extremity but with good
proximal control and placement of the letf hand is precise :
a.Hemi type wheelchair for self propulsion using left UE
b. Standard wheelchair since powered is not safe
c. No wheelchair since this may reduce her motivation to walk
d. Powered wheelchair

80. A 23 year old man got motorcycle accident shows weakness of the right shoulder. EMG
Shows fibrillation 4+ (++++) in the C5 dan C6 muscle. No voluntary motor units in C5 and C6
muscle. Nerve Coordination studies of the Median and Ulnar nerves are normal. Where is
the injury?
a.Neuropraxic injury of C5 and C6 with good prognosis
b. Root avulsion with poor prognosis
c. Severe brachial plexus injury involving the lateral cord
d. Severe brachial plexus injury involving the upper trunk

81. A patient has arm weakness 3 weeks after removal of a cast for proximal radius fracture.
Weakness is noted in pronation, wrist flexion, finer thumb flexion-abduction-opposition,
and decrease of sensation in lateral palmar aspect. EMG showed normal medial nerve
respons at the wrist but a 70% conduction block at the level of mid humerus. Spontaneous
activities (fibrillations and sharp waves) are noted in the intrinsic muscles. You will do the
following :
a.Referral for nerve exploration and decompression
b. MRI
c. Wrist splint and reassurance of a favorable prognosis
d. Electrical stimulation to ensure recovery

82. About the vascular supply of the spinal cord:


a. The great spinal artery of Adam Kiewicz supplies the lumbar and sacral cord
b. The two anterior spinal arteries are branches of the vertebral artery
c. The watershed zone is at C6-7, which causes most cervical lesions to be complete
d. The midthoracic region is most vulnerable to ischemia

83. The most likely symptoms of surgically proven spinal canal stenosis is:
a. LBP
b. Neurogenic intermittent claudication
c. Lower extremity weakness
d. Tingling in toes with prolonged sitting
35
84. The type of prehension provided by a wrist-driven tenodesis orthosis is :
a. Lateral
b. Three jawed chuck
c. Cylindrical
d. Hook

85. Which one is true ?


a. The two anterior spinal arteries are branches of the vertebral artery
b. There are 35 pairs of spinal nerve roots
c. The caudal spinal cord tapers to end at opposite to L1 vertebrae
d. The posterior spinal arteries originate from posterior cerebral artery

86. Idiopathic facial paralysis ( Bell's palsy):


a. Is usually bilateral
b. CIears completely in at least one half of cases
c. Rarely affects taste sensation
d. Is associated with decreased ipsilateral facial sensation

87. Which of the following is true of sex after Myocardial lnfarction (MI)?
There is… (tidak terbaca) difference between marital and extramarital sex
a.If you can climb two flight of stairs, you can return to sex after an MI
b. Typical energy expenditure is 2-1 MET's
c. You should wait 2-3 months before resuming sex

88. Muscle tension is monitored primarily by:


a. GTO (Golgi Tendon Organ)
b. Muscle spindle
c. Extrafusal muscle fibers
d. Pacinian corpuscles

89. In a child with muscular dystrophy with resultant respiratory dysfunction, a regular program
of exercise to Improve respiratory function will :
a.Improve maximal inspiratory pressure
b. Have clinical effects measurable by spirometry
c. Substantially reduce the incidence of pneumonia
d. Improve endurance of respiratory muscles

90. A patient with Diabetes Mellitus has paintful, swollen ankle, worse with weight bear, X-ray
reveals fragmentation of the articular surface. Synovial fluid is clear. The best orthosis ts :
a.Custom made shoc insert
b. AFO
c. PTB orthosis
d. Orthopedic shoe with rocker bottom sule

91. A patient with right C6 radiculopathy, the best traction is


a. 10 lbs with neck in flexion
b. 20 Ibs with neck in flexion
36
c. 20 Ibs with neck in extension
d. 30 Ibs with neck in extension

92. Which of the following exercise is a closed kinetic chain exercise :


a. Arms in the scapular plane in internal rotation with weights
b. Prone horizontal shoulder abduction in external rotation
c. Quadruped push-up
d. Arms in the scapular plane in external rotation with weights

93. Exercise in scoliosis :


a. Increases muscle strength and decreases muscle spasm/contraction
b. Prevents complications and decreases the hump
c. Increases muscle strength and decreases the hump
d. Decreases muscle spasm/contraction and decreases the hump

94. A 76-year old widow is hospitalized for surgical repair oa hip fracture sustain in u fall. She is
cleared by the orthopedic surgeon to begin full weight bearing ambulation training. In
therapy, however, she appears to be unable to walk. The most likely reason for her inability
to walk is :
a.Fear of falling
b. Surgical hardware failure
c. Disuse quadriceps weakness
d. Leg length discrepancy

95. Which of the following is most likely to enable you to determine the reason for a fall in an
older patient :
a.Performing….. (tidak terbaca) gait and balance evaluation
b. MRI of…….. (tidak terbaca)
c. ……. (tidak terbaca) the patient’s functional status
d. A careful history of the circumstances surrounding the fall

96. A stroke affecting the posterior inferior cerebellar artery will produce the following
syndrome:
a.Benedict’s:
b. Wallenberg’s
c. Millard-Gubbdler’s
d. Foville's

97. A 25-year old man with LS complete paraplegia is admitted to your rehabilitation service 2
weeks after his injury. On admission you note that he is tolerating an oral diet but has not
produce a bowel movement for 6 days. At this point you recommend :
a. Oxybutynin 3 times a day
b. A contact irritant suppository with digital stimulation daily
c. Manual removal of stool from the rectum 1-2 times daily
d. Nasogastric decompression for a presumed ileus

98. You are told by a physical therapist that your patient with acute C5 TETRAPLEGIA ASIA A is
having difficulty breathing, but only when sitting upright. Appropriate lab tests and
37
radiologic studies are unremarkable. ‘Io address the patient's breathing difficulty you
suggest :
a.Bilateral above-knee compression stockings
b. A tilt table program
c. Intermittent positive pressure breathing treatments
d. The use of an abdominal binder

99. The primary advantage of a 4-point crutch gait over a 2-point crutch gait is :
a. Stability
b. Speed
c. Weight-bearing relief
d. Eficiency of gait

100. A 28-year-old non-pregnant woman presents with persistent lancinating pain along her
right zygomatic arch. The pain is intermittent and often triggered by chewing. What is the
intial management ?
a.TENS
b. Carbamazepine (Tegretol)
c. Botulinum toxin A (Botox)
d. Cervical epidural injection

101. A stroke patient has recurrent aspiration pneumonia. In which phase of swallowing
dysfunction preducing aspiration ?
a.Oral
b. Lingual
c. Pharyngeal
d. Laryngeal

102. All of the following arteries are branches of the carat system of the brain circulation,
EXCEPT :
a.Aanterior cerebral artery
b. Middle cerebral arter
c. Posterior Cerebral artery
d. Anterior communicating artery

103. A stroke patient shows signs of weakness and sensory deficits especially in the Right
lower Extremity, while the Right Upper Extremity shows minimal disturbance. There are also
slight mental disturbance and aphasia. This condition most probably affecting the following
artery :
a.Anterior communicating artery
b. Anterior cerebral artery
c. Middle cerebral artery
d. Posterior cerebral artery

104. The following type of Aphasia is categorized as Fluent aphasia :


a. Global aphasia
b. Wemicke’s aphasia
c. Breca's aphasia
38
d. Transcortical motor aphasia

105. A stroke patient shows language problems in which he is fluent and has good
comprehension but poor in repetiting words. Most probably the stroke is affecting the
following :
a.Wernicke’s area
b. Prefrontal lobe
c. Angular gyrus
d. Arcuate fasciculus

106. The following statement is TRUE for shoulder pain after stroke :
a. Is usually due to shoulder subluxation and should initially be treated with mechanical
support/sling
b. Requires steroid injection for effective management
c. Is usually due to central pain / thalamic syndrome
d. Is usually due to reflex sympathetic dystrophy

107. The following statement is TRUE about EMG examination :


a. Able to replace MMT examination
b. Able to localize a peripheral nerve lesion
c. Able to make early detection of possible peripheral nerve lesion
d. Able to establish diagnosis of peripheral nervous system disease

108. The maximal muscle contraction during a needle EMG examination is directed to see the
following :
a.The spontaneous waves
b. The shape of MUAPs
c. The sound of MUAPs
d. The recruitment pattern of the motor units

109. All of the fellowin are the characteristics of normal MUAP, EXCEPT :
a. The shape mostly is triphasic
b. The duration is 4-12 msec
c. The amplitude is 100-2000 msec
d. The frequency is about 1-50 Hz

110. All of the following statement of H-reflex are true, EXCEPT :


a. Based on a monosynaptic reflex of S1 root
b. The normal value is about 40 msec in adult
c. To examine the possibilities of pathology at the level of nerve root
d. The stimulating electrodes must be placed at the popliteal area and the active electrode is
placed in proximal position

111. The following factor is not affecting the result of EMG examination :
a. Skin temperature
b. Patient cooperation
c. Sex
d. Age
39
112.One of the pitfalls of EMG examination is that this can decide the following result:
a. The etiology of motor unit pathology
b. The severity of motor unit pathology
c. The location of motor unit pathology
d. The degree of muscle denervation

113. The bladder dysfunction type in a stroke patient affecting the frontal lobe is :
a. Hyperreflexic detrussor — hyperreflexic sphincter
b. Hyperreflexic detrussor — hyporeflexic sphincter
c. Hyporreflexic detrussor — hyperreflexic sphincter
d. Hyporeflexic detrussor — hyporeflexic sphincter

114. All of the following are the possible signs and symptoms of an autonomic dysreflexia in
EXCEPT:
a T4 paraplegia ASIA A,
a. Pounding headache
b. Blood pressure : 190/110
c. Heart rate 120 — 140 / min
d. Feeling of chest fullness

115. About the sexual and reproductive functions in male with T10 paraplegia ASIA A, all of
the following statements are TRUE, EXCEPT :
a.Erection is still possible (reflex)
b. Emission is still possible
c. Retrograde ejaculation might develop
d. Reproductive function will still be normal

116. The absolute contraindication for performing Crede expression to a neurogenic bladder is:
a. High volume of residual urine
b. High intravesical pressure
c. Bladder overdistension
d. Presence of bladder stone

117. The most important measure for a SCI patient using indwelling catheter is
a. Bladder irrigation every 4 hours
b. Drink enough water so that the urine condition inside the tube is clean and transparent
c. Administration of acidifying substance to prevent stone formation
d. Change the catheter everyday

118. The most important part of n shoe to give calcaneal stabilization is the :
a. Heel
b. Shank
c. Counter
d. Inner sole

119. Lace is the closure of choice for en orthopedic shoe because :


a. It is very cheap
40
b. It is very popular and available everywhere
c. It is very strong and durable
d. It is able to adjust to different foot girths

120. A leprosy patient with flexible bilateral claw hands refused for surgical correction. The
splint of choice is :
a.Cock-up splint
b. Knucle-bender splint
c. Resting splint
d. Ulnar deviation splint

121. The main purpose of extending the shoe's quarter upward is :


a. To promote better foot movements
b. To give more stability to the ankle
c. To strengthen the ankle muscles
d. To give better adaptation to different surfaces

122. All of the following are the advantages of SACH foot, EXCEPT:
a. Simple
b. Durable
c. Easy for maintainance
d. Promote better ankle movements

123. The following level of Lower Extremity amputation is regarded as non accepted level
because it will lead to unfavourable foot position related to the production of the
appropriate prosthesis :
a.Lisfranc amputation
b. Syme amputation
c. Below knee amputation
d. Thru knee amputation

124. The following terminal device of an upper extremity prosthesis has the best function :
a. Cosmetic glove
b. Cosmetic hand
c. Cosmetic hand und movable thumb
d. Dorrance hook

125-131 tidak ada soalnya di file yang di foto

132. The indication(s) of treating muscle spasticity is/are : E


1. If the spasticity involving diffuse/systemic spasticity
2. If the spasticity leads to disturbance of functions
3. If the spasticity leads to difficulties in nursing care
4. If the spasticity leads to patient's discomfort

133. The immediate goal(s) of the spasticity treatment is / are : C


1. To apply orthosis early to correct spasticity
2. To reduce pain / dyscomfortness due to the spasticity
41
3. To refer immediately lo surgery for surgical correction
4. To make better joint movements for ADL

134. The following is / are the manifestation(s) of Brown-Sequard syndrome :A


1. Ipsilateral disturbance of exteroceptive sensation at the level of lesion.
2. Contralaterale disturbance of exteroceptive sensation at the level of lesion
3. Ipsilateral disturbance of proprioceptive sensation at the level of lesion
4. Contralateral disturbance of proprioceptive sensation at the level of lesion

135. The following is / are correct regarding the autonomic dysreflexia in SCI patient .A
1. Commonly triggered by the catheter block
2. Occurs in acute / paroxysmal condition
3. Should be treated immediately otherwise it could Iead to a stroke attack
4. May develop in T10 Paraplegia ASIA A

136. The following medication(s) is / are the choice(s) of the pharmacotherapy for OAB
(Overactive Bladder) in the elderly : A
1.Oxybutynin
2. Tolteredine
3. Solifenacin
4. Urfamycin

137. The following is / are the goal(s) of William's flexion exercise in LBP patient :B
1. To strengthen the abdominal muscles
2. To strengthen the Hamstring muscles
3. To strengthen the back extensor muscles
4. To strengthen the hip flexor muscles

138. The following muscle(s) is / are classified as the intrinsic muscle of the hand :A
1. Abductor pollicis brevis (superficial part)
2. The 4th lumbrical
3. Palmaris brevis
4. Extensor digiti minimi

139. The following muscle(s) is / arc innervated by the Median nerve :B


1. Palmaris longus
2. Flexor digitorum profundus (medial side)
3. Abductor pollicis brevis
4. Palmaris brevis

140. The classical claw hand deformity in leprosy patient is/are caused by the paralysis of
the following muscle(s) :B
1.Dorsal interosaei
2. Extensor digitorum communis
3. Lumbricals 1,2,3,4
4. Flexor digitorum sublimis

141. The upper extremity flexion synergy pattern in stroke patient consist of :E
42
1. Shoulder adduction
2. Shoulder internal rotation
3. Elbow flexion
4. Wrist flexion
142. The cardinal sign(s) of Leprosy is/are:A
1. Anaesthetic maculas
2. Thickening of the nerves
3. Positve skin/mucosal smear for AFB
4. Claw hand

143. The following muscle(s) is / are originated from lateral epicondyle of the humerus :E
1. Extensor carpi radialis brevis
2. Extensor carpi radialis longus
3. Extensor carpi ulnaris
4. Extensor digitorum communis

144. The common deformitie(s) in an advance stape of Rheumatoid Athritis is / are:A


1. Swan neck deformity
2. Boutoniere deformity
3. Ulnar drift deformity
4. Subluxation of the DIP joints

145. The main goal(s) of the management of RA is / are :E


1. Control the pain
2. Management of the altered joint biomechanics
3. Maintenance of muscle strength and endurance and joint’s ROM
4. Improvement of self-image and adjustment to disabilities

146. The ACR (American College of Rheumatology) critena(s) for OA of the knee includes :A
1. Knec pain
2. Osteophytes in X-ray
3. Crepitus
4. Age>40

147 The following statement(s) is / are correct regarding the physiology of the bladder :E
1.The filling phase is controlled by sympathetic system T11 - L2
2. The emptying phase is controlled by parasympathetic system S2 - S4
3. The external sphincter is controlled by somatic system via pudendal nerve
4. The function of the brain is to inhibit the Sacral Micturition Center (SMC)

148. The purpose of bracing in osteoporotic spine is fare :A


1. To diminish the incidence of kyphosis
2. To prevent compressive fracture
3. To support the weak erector spine muscles
4. To strengthen the trunk muscles

149. The normal gait : B


1. Consists of stance and swing phases
43
2. The stance phase is 40% and the swing phase is 60%
3. Needs normal ROM of the hip, knee and ankle joints
4. There is no double support period

150. The objective(s) of pursed-lips breathing is / are :E


1. To decrease respiration rate
2. To increase tidal volume
3. To increase oxygenation
4. To educate patient as how to control breathing effectively

44
Soal Januari 2010

51. A stroke patient with difficulty in repeating a sentence has the following language problem :
a. Aphasia Broca
b. Aphasia Wernicke
c. Anomic aphasia
d. Conduction aphasia

52. A stroke patient with Right hemiplegia and contralateral ophthalmoplegia is having the
following syndrome :
a.Benedict
b. Foville
c. Weber
d. Wallenberg

53. Amaurosis vugax is a sign of the disturbance of the following artery :


a. A. Carotis communis
b. A. Carotis interna
c. A. vertebro-basilaris
d. A. Cerebri posterior

54. One of the following IS NOT the predictor for death after a stroke :
a. History of smoker
b. Abnormality of ECG
c. Diabetes Mellitus
d. Haemorhagic stroke

55. Wallenberg syndrome is caused by the thrombosis affecting the following artery :
a. A. Cerebri media
b. A. Cerebri posterior
c. A. Cerebelli superior
d. A. Cerebelli inferior posterior

56. The following movement is GOOD in a case of “Upper Brachial Plexus Injury” :
a. Shoulder external rotation
b. Shoulder abduction
c. Elbow flexion
d. Wrist flexion

57. “Anterior Interosseous Syndrome” is an entrapment neuropathy of the following nerve :


a. Median nerve
b. Ulnar nerve
c. Radial nerve
d. Axillary nerve

45
NATIONAL BOARD EXAMINATION
PHYSICAI.'MEDICINE AND REHABILITATION
December 5' 2008

CHOOSE THE MOST APPROPRIATE ANSWER


1. The number of intrinsic muscles of the hand is :
A. l7
B. l8
C. l9
D. 20

2. The following statement is true about the intrinsic muscles of the hand :
A. Innervated by Median, Ulnar and Radial nerves
B. Innervated by Radial and Ulnar neryes
C. Innervated by Median and Ulnar nerves
D. lnnervated by Ulnar nerve only

3. The following intrinsic muscles are innervated by the Median nerve 'EXCEPT :
A. Lumbrical I
B. Lumbrical 2
C. Abductor Pollicis brevis
D. Adductor Pollicis

4.The following joint is not categorized as TRUE joint:


A. Gleno-humeraljoint
B. Humero-Uinarjoint
C. Calcaneo-Cuboid joint
D. Scapulo-thoracic joint

5. A Rheumatoid Arthritis (RA) will never affect the following joint :


A. Suprahumeral joint
B. PIP joint
C. DIP joint
46
D. Atlanto-Axial joint

6. The following movements are possible in Ankle-Mortise:


A. Entlorotation-Exorotation
B. Flexion-Extension
C. Abduction-Adduction
D. Eversion-lnversion

7. The following nerve is a branch of the Brachial plexus at Root level :


A. Upper subscapular nerve
B. Lower subscapular nerve
C. Musculocutaneusnerve
D. Longthoracic nerve

8. An entrapment neuropathy affecting the pathologic gait:


A. Quadriceps gait
B. Steppage gait
C. Gluteusmediusgait
D. Gluteus maximus gait

9. In a Lever System if the Mechanical Advantage is > 1, the following statements are true '
EXCEPT:
A. To lift a heavy object will need a lesser Effort
B. To lift a light object will need a bigger Effort
C. The Weight arm is always shorter than the Effort arm
D. The Effort could be located at the same side with the Weight from the fulcrum

10.The purpose of Patrick's test is to locate the possible pathology of the following joint :
A. Lumbosacraljoint
B. Sacro-iliacjoint
C. Hip joint
D. Knee joint

47
11. The autonomic component of the Facial nerve (N. VII) innervates the following structure:
A. Orbicularis occuli muscle
B. Levator Palpebrae muscle
C. Taste of-2/3 anterior part of the tongue
D. Lacrimal gland

12.Wallenberg syndrome in a stroke patient is caused by the thrombos is affecting the following artery:
A. A. CerebriPosterior
B. A. Cerebelli superior
C. A.Cerebelli inferior Posterior
D A. Cerebelii inferior anterior

13.Conduction aphasia is due to the pathology of the following area in the brain :
A. Broca's area
B. Wernicke's area
C. Angular gyrus
. D. Arcuate fasciculus

14. The following sport activities are suitable for osteoporosis, EXCEPT :
A. Weight lifting
B. Swimming
C. Jogging
D. Stair climbing

15.Electrcodiagnostic tests may include all of the following procedures' EXCEPT :


A. Erb's test
B. Klumpke's test
C. S-D / I-T curve test
D. Galvanic test

16.Part of the shoe which covers the dorsum pedis and the toe is called:
A. Vamp
B. Shank
48
C. Quarter
D. Counter

17. In all of the following conditions the orthoses should be removed, EXCEPT :
A. The function is deteriorating
B. The gait is worst
C. The muscle atrophy is increasing
D. The peripheral blood circulationis compromised

18.In PTB prosthesis the stump is in the following position:


A. Full extension
B. 150flexion
C. 300flexion
D. 450flexion
19. The main purpose of “immediate post-surgical fitting" procedure using a pylon leg is to :
A. Accelerate ambulation
B. Accelerate wound healing
C. Accelerate edema reducing
D. Preserve proprioceptive sensation

20. A 66-year old lady with advance stage of RA undergoes an arthroplasty surgery on the
left knee joint. she has severe bilateral MCP joint subluxation and ulnar deviation, which of
the following ambulatory aids most appropriate for her?
A. Tripod cane
B. Quadripod cane
C. Regular walker
D. Platform walker

21. A leprosy patient shows severe bilateral claw hands. However all of the hand and finger
joints have good ROM and all the extrinsic muscles of the hand have normal strength. He
refused a reconstructive surgery. The best orthotic device to maximize hand function is:
A. Resting hand splint
B. Cock-up splint
49
C. Knucle-bender splint
D. Posterior splint

22.The maximum knee flexion in a normal gait occurs in the following phase:
A. Heel strike
B. Push-off
C. Mid-stance
D. Mid-swing

23.The respiratory function of patient with Ankylosing spondylitis is characterized by :


A. Diminish chest expansion
B. lncrease use of the accessory muscles
C. Decrease in diaphragmatic excursion
D. Moderate dyspnea on exertion

24.Problems arising from OA of the knee are all of the following, EXCEPT :
A. Pain is the most important symptom
B. There is continuous process of joint cartilage destruction
C. There is always concomitant osteoporosis
D. There is malalignment of the knee joint

25.The most important substance/structure to hold water inside the joint cartilage is:
A. Collagen
B. Fibronectin
C. Hyaluronate
D. Proteoglycan

26.Which of the fo|lowing is the correct description of Yergason’s test?


A. The examiner provides resistance against supination of the forearm with the elbow flexed at
90 degrees
B. The examiner provides resistance against supination of the forearm with the elbow in
extension
C. The examiner provides resistance against pronation of the forearm with the elbow flexed
at 90 degrees
50
D. The examiner provides resistance against pronation of the forearm with the elbow extension

27.Which of the following is a general treatment principle for knee osteoarthritis? ( B )


A. Avoid extreme range of motion exercises
B. Strengthen the quadriceps muscles
C. Encourage high impact aerobic home exercise program
D. Avoid the use of modalities

THE FOLLOWING CASE IS FOR QUESTIONS NUMBER 28 TO 36


A young healthy man 30 years old suffered acute back pain after lifting a very heavy box.
Coughing/sneezing induced the pain which radiates to the left leg & foot, however no abnormality
detected in bladder or bowel function. He is still able to walk with an antalgic gait.

28.Clinically the patient is suffering from:


A. vertebral fracture.
B. osteoporosis.
C. osteoarthritis.
D. prolapsed intervertebral disc

29.The weakest spot in the posterior lumbar region which is not strengthened by posterior longitudinal
ligamentis :
A. L2-3
B. L3-4
C. L4-5
D. L5-Sl

31. On neurological examination the patient will reveal:


A.Diminished patellar tendon reflex
B. Diminished Achilles tendon reflex
C. Positive Babinski reflex.
D. Positive Chaddock reflex.

31. Sensory examination will show hypoaesthesia or anaesthesia in the dermatom of:
A. Ll-2
51
B. L2-3
C. L3-4
D. L5-Sl

32. The following examination will confirm the diagnosis of prolapsed intervertebral
disc:
A. lumbar puncture.
B. plain X-ray of the spine.
C MRI of the spine.
D. bones can.

33.Treatment of choice would be:


A. laminectomy.
B. bedrest
C. diathermy
D spinaltraction+ (B) +(C)

34.The patient will have difficulty on :


A. squatting.
B. lying.
C. sitting.
D. walking on tiptoe.

35.The proper way to pick up an object from the floor is by:


A. bending from the waist with horizontal trunk and knees extended to strengthened the spinal
extensors.
B. bending the knees keeping the trunk straight vertical.
C. bending the knees and inclining the body slightly forward.
D. kneeling.

36.Swimming will make the patient:


A. suffer more pain on his back because it requires strong action on spinal extensors.

52
B. feel less pain because it requires less effort on the spinal extensors due to non-weight
bearing.
C. complain more pain on his leg due to more leg movement.
D. feel better because of arms movement

37. Initial treatment for bicipital tendinitis may include all of the following, EXCEPT :
A. Anti-inftamatory medications
B. Therapeutic modalities
C. Avoidance of activities that provoke pain
D. Aggressive strengthening of the shoulder stabilizers

38.The phase of treatment for rotator cuff tendonitis include all of the following,
EXCEPT:
A. Pain control and decreased inflammation
B. Immobilization
C. Restoration of range of rnotion
D. Normalization of strength and dynamic muscle control

39.1n regards to movement of the scapula, the serratus anterior does which of the following:
A. Rotates the glenoid upward with shoulder abduction
B. Rotates the glenoid downward with shoulder abduction
C. Rotates the glenoid upward with the shoulder forward flexion
D. Elevates the vertebra borders of the scapula away from the chest wall

40.All of the following muscles will abduct the shoulder joint in the frontal plane together with the
deltoid muscle, EXCEPT :
A. Subscapular
B. Teres major
C. Infraspinatus
D. Supraspinatus

4l.During an acute exacerbation of hand rheumatoid arthritis, all of the following are appropriate
interventionts, EXCEPT :
A. Ice
53
B. Range of motion exorcises
C. Resistance training such as weightlifting
D. Anti inflammatory agents

42.Conservative treatment of Acute LBP includes all of following, EXCEPT :


A. Rest
B. Analgesics
C. Gentle active exercise of the limbs
D. William's flexion exercise

43.All of the following statement are TRUE regarding the carpal tunnel. EXCEPT
A A very narrow tunnel
B. Covered by the tranverse carpal ligament
C.The base is made by the metacarpal bones
D. The median nerve is, the only nerve passing through this tunnel

44.The Boutoniere deformity in a Rheumatoid hand is typically located in the following joint:
A. Radiocarpal
B. CarpometacarPals
C.MCPs
D. PIPs

45.The followingis not the early pathological feature of OA :


A. Joint cartilage cracks
B. Swelling of the joint cartilage
C. Increased joint cartilage water content
D. Loosening of collagen framework

46. In degenerative joint disease of the spine:


A. Vertigo and visual disturbance may occur
B. Anterior spurs are the most common cause of symptom
C. Radicular pain is rare
D. Ankylosing between the vertebrae never occurs
54
47. According to the American Collage of Rheumatology guidelines,which agent is recommended as
first-line medication in OA of the hip ?
A .Prednison
B. Viscosupplementation
C. Oral gold treatments
D. Acetaminophen

48. The following test will be probably positive in the early stage of Ankylosing spondylitis:
A. Schober test
B. Gaenslens test
C. Tinel test
D. Pattrick test

49. The following deformities are typical in an advance case of Rheumatoid Arthritis, EXCEPT :
A. Piano- sign
B. Boutonierre
C. Ulnar deviation
D. DIP joint subluxation

50. A girl patient with idiopathic scolios is the following are the signs that the Cobb's angle might be
increasing rapidly, EXCEPT :
A. She has her menarch a month ago
B. Her breast are growing bigger
C. Her pubic hairs are growing
D. The Risser's sign in reaching to grade 4

51.The Major Benefit of Exercise in musculoskeletal is :


A. Increasing Bone Mass
B Sustained weight loss
C. Improved balance and coordination
D. Improved blood lipids

55
52. Spinal traction is contraindicated in patients with what disorder?
A. Discitis
B. Herniated nucleus pulposus
C. Chronic low back pain
D. Fibromyalgia

53. Which is a risk factor for plantar fasciitis?


A. Obesity
B. Female gender
C. Age younger than 40 years
D. Increased subtalar motion

54. A 7O-year-old white man presents to your office with axial back pain that is worsen with
standing and walking. At times he also experiences pain radiating from the lumbar
spine into the legs when walking. What is the most likely diagnosis?
A. Lumbar canal stenosis
B. Sacroiliac joint pain
C. Piriformis syndrome
D. Ischial bursitis

55. About Thoracic Outlet Syndrome (TOS) below is true


A. The symptom of TOS is assume do be of neuro vascular origin
B. Because of compression of carotid artery, vein and brachial plexus
C. Adson's test is used to evaluate compression the bundless as it passes between first rib and
clavicle
D. The symptom involve shoulder and hand,but the elbow is normal

56. A patient complain pain over the radial side of the wrist when the patient does ulnar
deviation of the hand after a fist is made over the flexion thumb. We called is as:
A. Trigger finger
B. Mallet finger
C. Dupuytren contracture
D. De Quarvain's disease

56
57. The following statements below are true about scoliosis, EXCEPT :
A. The Risser sign uses the ossification of the iliac epiphysis
B. The indication for surgical intervention is curve >50 degreee
C. Scoliosis curvatures with an apex at T8 above, require Milwaukee brace
D. We call Juvenile scoliosis if spinal curvature developing during the first three years

58. The four cardinal features of Parkinson Disease are resting tremor, bradykinesia,
rigidity and postural instability. One of the statements below are not true :
A. Slowness of movement may result in a reduction of performing activities of daily living
B. Stage one of Hoehn and Yahr stage s on both side
C. Rigidity is not velocity dependent
D. Shuffling or dragging the leg is some gait characteristics in Parkinson

59.Shoulder pain are common after stroke episodes that bad to hemiparesis:
A. Is usually due to reflex sympathetic dystrophy
B. Is usually due to central pain/ thalamic syndrome
C. Requires injection with steroids for effective management
D. Is usually due to subluxation and should initially be treated with mechanical
supports/slings

60. One of the statements below is not the features of Wallenberg's syndrome :
A. Contralateral loss of pain and temperature
B. Ipsilateral ataxia
C. Horner's syndrome
D. Ipsilateral paralysis of soft palate

61 The usual sensory distribution of the median.nerves :


A. Medial two digits of the hand
B. Lateral 3 1/2 digits of the hand
C. Thumb, index finger, and long finger
D. Area over the carpal bones on the volar aspect of the hand

57
62. Erb's Palsy is : A
A. Lesion on C5-C6 brachial plexus related to delivery problems
B. Lesion on C5-C6 brachial plexus related to Brachial plexitis
C. Lesion on C5-C6 brachial plexus related to Thoraxis Outlet Syndrome
D. Lesion on C5-C6 brachial plexus related to any trauma

63. Sustained repetitive twisting movements of variable speed, anywhere in the body is :
A. Spasticity
B. Chorea
C. Balismus
D. Athetoid

64. Homonymus hemianopsia indicates a :


A. Pre chiastmal lesion
B. Chiasmal lesion
C. Postchiasmal lesion of the opposite side
D. Lesion of on optic tracton the sameside

65. A patient appears to have good strength in the hand and can use it for gross activities autonimically
but cannot perform skilled movements, This form of disability is called :
A. Constructional apraxia
B. Kinetic apraxia (motor apraxia)
C. Ideational apraxia
D. ideamotor apraxia

66. A C6 quadriplegic can be expected to be ablet o :


A. Ambulate with crutches and braces
B. Pushup while sitting in a whcel chair
C. Pick up a heavy object without a splint
D. Dress indepenciently

67. Internuclear opthalmoplegia is most commonly see in :


A. Multiple sclerosis

58
B. Cerebrovascular accident
C. Brain tumor
D. Guillain – Barre syndrome

68. Lesion of the following nerve roots will lead to a classical "winging of the scapula" : A
A. C4, C5 and C6 roots
B. C5, C6 and C7 roots
C. C6, C7 and C8 roots
D. C7, C8 and Tl roots

69.The following arteries NOT originally coming from theVertebro-Basilar system of the brain
circulation:
A. Middle cerebral artery
B. Posterior cerebral artery
C. Superior cerebral artery
D. Ariterior- Inferior cerebralartery

70. What is the greater risk factor for late post-traumatic seizures in patients with a traumatic brain injury:
A. Multiple subcortical contusions
B. Sudural hematoma with evacuation
C. Midline shift greater than 5 mm
D. Bilateral Parietalc ontusions

71.An individual with C7 ASIA D tetraplegia must have


A. A bulbocavemosus reflex and voluntary sphincter contraction
B. A muscle grade of 3 or greater in atleast half of the key muscles below C7
C. normal pinprick and light touch sensation trough the sacral dermatornes
D. normal strength (5/5) in the C7 myotome

72. A40-year-oldman sustained an injury to his 1eft arm, 3 weeks agowhen he lost his
Balance and crashed into a bookshelf .His complaints include left arm pain, weakness
with extension of his wrist and fingers,and decreased_hand grip. He denies any
nunbness but has odd sensation over the dorsum of the left hand. Prior to any testing,

59
which problem would you consider as the most likely
A. Posterior interosseous neuropathy
B. C7 radiculopathy
C. Posterior cord brachial plexopathy
D. Radial neuropathy

73. What is the definition of the 4th degree Sunderland's classification of peripheral nerve injury?
A. Transection of the entire trunk
B. Local conduction block with minimal structural disruption
C. Disruption of the axon endoneurium and perineurium 'The epineural tissue is spared
D. Disruption of axon only, leaving the endoneurium intact neuroma-in-continuity

74. You are evaluating a patient with a nerve palsy. Durng evaluation the patient
reveals positive Froment's sign. Which nerveis injured?
A. Median
B. Radial
C. Ulnar
D. Musculocutaneous

75. In what area is the cell body of the sensory neuron located?
A. Spinal cord
B. Dorsal root ganglion
C. Brain
D. Ventral root

76. Which of the following drugs can be given directly into the intra thecal space to treat
spasticity
A. Dantrolene
B. Diazepam
C. Baclofen
D. Tizanidine

77. The IV cranial nerve innervate the following muscles:


60
A m Lateral rectus
B. m Superior oblique
C. m Medial rectus
D. m Inferior oblique

78. These statements below are correct in Parkinson's disease, EXCEPT :


A. Tremor
B. Apraxia
C. Bradikinesia
D. Rigiditv

79. Pattern of LMN muscular weakness:


A. BabinskiPositif
B. Fasciculation
C. Increased muscle tone
D. Increased reflexes

80. The efferent of the corneal reflex is via the :


A. VII cranial nerve
B. V cranial nerve
C. VI cranial nerve
D. II cranial nerve

81. Posterior Interosseus nerve is a branch of


A. Radial nerve
R. Median nerve
C. Ulnar nerve
D. Musculocutaneus nerve

82. Cubital tunnel syndrome is an entrapment neuropathy of the following nerve :


A. Median nerve
B. Axillar nerve
C. Ulnar nerve
61
D. Radial nerve

83. A child with a congenital transverse radial limb deficiency should have in initial Prosthesis fit at what
developmental stage?
A. At the time of starting kindergarten
B. At soon as possible after birth
C. At the time of first sitting independently
D. At the time of initially walking

84. Palients with athetotic cerebral palsy are likely to have:


A. Communication disorders
B. More mild hearing deficits than patients with spastic cerebral palsy
C. immobility of the neck secondary to degenerative disease
D. Mental retardation

85. When treating patients with cerebral palsy, consider:


A. AFOs only in nonambulatory patients for positioning and contracture prevention
B. Bracing to prevent progressive deformity of scoliosis
C. vocational rehabilitation for job training and placement
D. Limited strengthening it has been shown to increase spasticity

86. Management of Pes Planus grade 1 at age less than I year old is done by :
A. Observation
B. Arches support
C. Thomas heel
D. Chukka type shoes

87. All of the following primitive reflexes are positive in a healthy baby of 8 months old,
EXCEPT:
A.Placing reflex
B.Parachute reflex
C. Palmar grasp
D. Plantargrasp

62
88. The purpose of bracing in CP patientis :
A. to control movements that interfere functions
B.To prevent / correct contractures
C.To promote better balance
D.All of the above

89. in Klumpke's type of branchial plexus injury the following muscle is spared:
A. Flexor digitorum sbulimis
B. Flexor digitorum Profundus
C. Flexor pollcis longus
D. Extensor carpi radialis longus

90. The tonic neck reflex is abnormal if :


A. the contralateral extremities flex
B. the ipsilateral extremities extend
C. it is variable in the same infant
D. it persists for more than six months of age

91. At six months, a normal baby can:


A. say " ma-ma"
B. pulls self-up
C. rolls over, may sit up
D. walks if helped

92. The following are the characteristics of Duchenne's Muscular Dystrophy, EXCEPT:
A. The disease affects males
B. It is transmitted by X-linked recessive mode of inheritance
C. The child's mother, his sisters and maternal aunts are potentially carriers
D. Electromyogram shows high amplitude, long duration motor unit action potentials with
decrease recruitment pattern on effort

93. The characteristic of cerebral palsy are as follow, EXCEPT:


A The underlying neurologic lesion must be static

63
B. The process which cause the cerebral palsy is still active
C. lt is caused by nonprogressive injury which affects the immature brain
D. The sign of cerebral palsy is a disorder of movement and posture

94. Devetopmental Evaluation and Screening Tests from Birth- 6 yr is :


A. Developmental Milesstone
B. Denver Developmental Screening Test
C. Bayle Scale of Infant Development
D. Gesell Developmental Schedule

95. The average cardiac output at maximal exercise is :


A. 5 L/minute
B. l0 L/minute
C. 15 L
D. 20L

96. For patient with Congestive Heart Disease, which of the following statements is
accurate?
Most of the important resulting.from regular exercise is within the myocardium.
These patient can never expect improved physical fitness.
C. Exercise capacity is improved because of peripheral adaptations.
D. Complete bed rest is prescribed for these patients.

97. The proper emergency response for a patient who has experienced a cardiac arrest,
but who is now breathing and hus a palpable pulse, includes:
A. Continuing the exercise test to determine why the patient had this response.
B. placing the patient in the recovery position with the head side to prevent an airway
obstruction.
C. Placing the patient in a comfortable seated position
D. Placing the patient head down position

98. the major goal of phase I cardiac rehabilitation is to:


A .stimulate the development of Coronary collaterals
B. stimulate of myocardial exercise tolerance
64
C. Evaluate the myocardial exercise tolerance
D. Prevent deconditioning

99. Which of the following risk factors is most associated with a higher incidence of Deep Vein
Thrombosis (DVT) in adult with a stroke ?
A. Smoking history
B. Increased motor weakness
C. Male sex
D. Female sex

100. For chronic lung disease various walking test with submaximal exercise have
All then benefit below, EXCEPT :
A. To detect exercise intolerance
B. To know and detect functional limitation
C. To detect limitation in lung function
D. To evaluate reconditioning exercice programme

101. Breathing control treatment that is important for obstructive pulmonary


Disease is :
A Diaphragma breathing
B. pursed lip sexpiration
C. Combination of diaphragmatic breathing and purse lip expiration
D. Combination of segmental breathing and purse lip Expiration

102. The risk of exercise induced bronchospasm in chronic obstructive pulmonary disease, mostly caused
by :
A. Low intensity continues exercise
B. Moderate intensity interval exercise for long period
C. 24-26 degree Celcius room temperature with high humidity
D. Continues exercise with graded high intensity

103. Maximal heart rate during exercise that is recommended for cardiac insufficient
Patient is :

65
A.50-60% of MHR
B. 60-80% of MHR
C. 80-90% of MHR
D. 90-100% of MHR

104. The Following is NOT the risk factor of COPD :


A. Smoking
B. Air Pollution
C. Low income group
D. Female

105. All of the following are components of exercise for the elderly person, EXCEPT:
A. lntensity
B. Load
C. Duration
D. Frequencyand mode
'
106. The most common cause of falls in the elderly is/are:
A. postural hypotension
B. weakness
C. environmental hazards
D. vertigo

107. Effects of exercises in the elderly, include:


A. Maintained or increased bone mass
B. Improved cardiopulmonary function
C. Improved mobility and function
D. All's true

108. The disorder is usually seen in pre adolescent athletes who participate in activities
Such as jumping or running. The disorder is a result of recurring micro trauma from the quadriceps
contracting :
A. Pes anserinus bursitis

66
B. Patellar tendonitis
C. Tibial plateau fracture
D. Osgood-Sctrlatter disease

109. Ankle sprain are ubiquitous in sports and are the most common athletic injury. The most
commonly injury for this disorder is:
A. the calcaneo fibular ligament
B. the anterior talofibular ligament
C. the posterior talofibular ligament
D. the strong medial deltoid ligament

110.A 20- year-old football player has repeated episodes of heat cramps during summer training sessions.
A deficiency of what electrolyte is most responsible for hoat cramps?
A. Potassium
B. Magnesium
C. Chloride
D. Sodium

111. A l5 rear-old jogger experiences new-onset a traumatic pain along the lateral aspect of the knee. Pain
is reproduced and accentuated when the examiner presses proximal to the joint
line and simultaneously brings the knee in and out oi 20 degree to 30 degree of knee flexion. The most
likely diagnosis is:
Lateral meniscus strain
Iliotihal band syndrome
Lateral plica syndrome
distal femur stressfracture

112. The most likely combination of musculoskeletat imbalances associated with the above diagnosis is:
A. weak gluteus medius /tight tensor fascia lata
B. weak lower abdominals / tight psoas
C. weak quadriceps/ tight hamstrings
D. weak gastroc-soleus/tight posterior tibialis

113. One of the pitfalls of EMG examination is that with the EMG machine we can
get the following result :

67
The level / location of motor unit pathology
.The severity of pathology
The motor and sensory conduction velocities
The etiology of motor unit abnormalities

114 All of the following are the characteristics of MUAP, EXCFPT :


A. Under voluntary controls
B..The shape is mostly triphasic
C. The maximal amplitude is 3 mV
D. It can be detected when the needle is inserted to the muscle

115. A 55-year-old man presents with a 2-month history of progressive weakness. On examination he has
mild proximal weakness in the upper and lower lirnbs. His muscle
Tone and bulk are normal and he has no facial weakness. Sensation is normal and deep tendon reflexes
are l+ and symnretrical. Which finding on electrodiagnostic
Testing is most consistent with this patient's presentation?
A. Prolonged or absent F waves
B. Decreased recruitment ratio
C. Motor unit potentials with amplitudes of l0 millivolts
D. Normal number of phases of the motor unit potentials

116. Electromyographic biofeedback for stroke patients is most beneficial when


A. proprioception is preserved
B. used in the upper limb
C. the patient is young
D. the patient has flaccid paralysis

117. Normal potensials in electromyography:


A. Positive sharp wave
B. Fibrillation potensial
C. Nascent potensial
D. Polyphasic potensial more than 20%

118. The purpose of minimal contraction during the EMG examination is to get all of the following data,
EXCEPT :

68
A. The amount of Motor Units which are active
B. The frequency of Motor Unit Action Potential
C. The duration of Motor Unit Action Potential
D. The intensity of Motor Unit Action Potential

119. Three months after a nerve lesion the characteristics of MUAPs will be :
A. The same prior to the lesion
B. High amplitude - long duration
C. Low amplitude - long duration
D. High amplitude - short duration

120, Prolonged or Negative of H-reflex shows the possibility of:


A. ALS
B. MS (multiple sclerosis)
C. Herniated disc between L4-L5
D. Herniated disc between L5 - Sl

121. Splints usage may help to do all of the following mechanism, EXCEPT:
A. Prevent shortening of the finger
B. Prevent extensor muscles from overstretching
C. Facilitate recovery of the radial nerve
D. Prevent joint contractures

122. The Milwaukee brace for scoliosis produces all of the following forces to correct the spinal
curvature, EXCEPT :
A. Upward-directed force
B. downward-directed force
C. Medial-directed force
D. Lateral-directed force

123.part of the orthopedic shoe which stabilize the sub-talar joint is called:
A. Shank
B. Quarter

69
C. Counter
D. Heel

124. If the following nerve is injured, the patient will not able to ambulate with an axillary crutches:
A. Upper subscapular
B. Lower subscapular
C Axillary
D.Thoracodorsal

125 The best example of a dynamic orthosis is:


A. Dorsal wrist hand orthosis with extension force in radial nerve injury.
B. Thermoplastic ankle foot orthosis in severe foot drop.
C. Wrist hand orthosis in l5 degree of extension for carpal tunnel syndrome.
D. A C-bar in median nerve injury

Please answer:
A. if numbers 1,2,3 are correct
B. if numbers l,3 are correct
C. if numbers 2,4 are correct
D. if only number 4 is correct
E. if all are correct

126. Reflection of ultrasound


1) Occurs mainly at interfaces of tissues with different acoustic impedances.
2) Is great at interface of soft tissue and bone.
3) Is great at the interface of soft tissue and metallic implants.
4) Is minimal at interfaces of layers of soft tissue.

127. The patient with painful back conditions often improve when lumbar traction is used
Continuously for many hours with distraction weights of 50 pounds or less.This is
because:
l) The bed rest the patient gets while in the traction apparatus is probably the actual
treatrnent benefit.
70
2) This technique actually does produce lumbar distraction.
3) The pull of the traction is usually adjusted such that it helps eliminate the lumbar
lordosis.
4) The benefit from the traction is strictly psychological.

128. The physiological effects of Heat is / are :


1. lncreasedblood flow
2. Decreased edema
3. lncreased nerve conduction velocity
4. Decrcased tendon extensibility.

129. The peripheral influences on motor neuron excitability usually used in proprioceptive
neuromuscular facilitation exercise techniques are:
1. Cerebellar and corticospinal input to the alphamotor neurons.
2. Muscle receptors such as the intrafusal muscle system.
3. Psychological influences such as emotion.
4. Afferent input to the alpha motor neurons from the skin and tendon receptors

130. The main difference between the therapeutic effects of quick and slow passive stretch
muscle is :
1) There is actually no significant therapeutic difference.
2) The pain experienced by the patient is less with slow stretch.
3) The slow stretch is used only when a muscle exhibits clonus.
4) A muscle inhibitory or relaxing effect is obtained by slow stretch.While fast stretch
is excitatory and facilitates muscle activity.

131. Proprioceptive neuromuscular facilitation (PNF) program typically include:


1) Deep cooling of muscles
2) Muscles stretching
3) Withdrawal reflexes
4) Superficial cooling

132. Tbe prescription of exercise for the patient with simple recurrent low back pain
71
should achieve at least the following:
1) Improved low back flexibility
2) Improved posture with minimized lumbar lordosis
3) Improved body mechanics in all activities and exercise
4) Improved strength of abdominal and hip extensor muscle.

133. In the left hemiplegic patient who his or her ieft hand is edematous and his or her left
shoulder is painful requires:
1. A short course of 30 mg prednisone daily
2. to elevate his / her L upper extremity
3. A low dose of Amiltritvline
4. TENS

134. The upper extremity flexion synergy pattrn in stroke patient consist of :
l. Scapular Protraction
2. Shoulder internal rotation
3. Elbow extension
4. Wrist flexion

135. Conributing Factor(s) to Risk of Fall in elderly people is/are :


1. Visual impairment
2. Vestibular changes
3. Cognitive changes
4. Inappropriate footwear

136. Goals of Rehabilitation Approach in Parkinson's Disease are:


1. Increase stride length
2. Improve arm swing
3 Improve posture
4. Increase cadence

137. Some of the gait characteristic(s) in normal aging include:


l) Slower walking speed
72
2) Shorter stride length
3) Wider base of gait
4) Relatively move time is spent in stance phase

138. About the ankle mortis.


l. Consist of tibia fibula and talus
2. The talus function as a hinge joint
3. Its motion is 20 degree dorsi flexion
4. Consist of talus and calcaneus

139. Laser therapy has been proven effective in the treatment of :


1. Rheumatoid arthritis
2. Joint contracture
3. Osteomalacia
4. Myofascial syndrome

140. The advantages of SWD is / are :


1. Easy of the application
2. Does not require contact between the body surface and the electrodes
3. Heat is relatively cover larger area than do other deep-heating modalities
4. Is very cheap

14l. The eflects of electrical stimulation is / are:


l. Muscle contraction
2. Maintains muscle nutrition
3. To give proprioceptive stimulation secondary to muscle contraction
4. Maintains muscle strength and endurance

l42.The following statement(s) is / are true:


1. The mechanism of heat transfer of paraffin bath is conduction
2. The mechanism of heat transfer of whirlpool is convection
3. The mechanism of heat transfer of SWD is conversion
4. The mechanism of heat transfer of USD is radiation
73
143.The technique(s) to control hand edema after surgery is / are:
1. Icing
2. Elevation
3. Compressive garments
4. Controlled passive movement

l44.The following statement(s) is / are true regarding Cervical traction:


l. Can be done in sitting position
2. The position of the neck is in extension
3. RA affecting cervical spine is contraindicated
4. The weight of traction is 50% of total body weight

145.The effect(s) of cervical traction is / are:


l. Separating the vertebralbodies
2. Tensing the spinal ligaments
3. Stretching the spinal musculature
4. Widening the intervertebral foramina

146.The effect(s) of continues lumbar traction is /.are:


l. Separating the vertebral bodies
2. Stretching the paralumbar muscles
3. Widening the intervertebral foramina
4. Immobilizing patient in bed (bedrest)

147.The following is / are the gold standard for diagnosis of osteoporosis:


l. Quantitative Computed Tomography
2. Peripheral Quantitative Computed Tomography
3. Ultrasonometry
4. Dual Energy X-rayAbsorbsiometry (DEXA)

148. Normal gait pattern:


l. Consists of the Stance and Swing phases
74
2. Stance phase is 40% and Swing phase 60%
3. Needs normal ROM of the Hip, Knee and Ankle joints
4. No phases of double support

149. The following statement(s) is/ are correct regarding the physiology of the Bladder:
1. The filling phase is under the control of the sympathetic system Tll-L2
2. The emptying phase is under the control of parasympathetic system S2-S4
3. External sphincter is under the control of somatic system S4-S5
4. The function of the frontal lobe of the brain is to inhibit the Sacral Micturition Center

150.Exercise in Diabetic patients may:


1. Lower the appetite
2. Improve insulin sensitivity
3. Improve diuresis
4. Lower blood sugar level

GOODLUCK

75
1.Which one of the knee bursitis condition is commonly seen in an overweight women?
a. Prepatellar
b. Infrapatellar
c. Anserine
d. Semimembranosus

2. Not included as “Red Flags” for urgent evaluation and management:


a. Soft tissue injury, fracture or internal derangement
b. Signs of fever, loss of weight and faigue
c. Neurogenic or radicular pain
d. Joint deformity

3. One of the following maneuvers is not for TOS (Thoracic Outlet Syndrome):
a. Adson test
b. Yergason test
c. Shoulder depression test
d. Hyperabduction test

4. The term of Herbeden’s node refers og the OA of the following joint:


a. MCP joint
b. PIP joint
c. DIP joint
d. Wrist joint

5. A Rheumatoid Arthritis (RA) will never affect the following joint:


a. Suprahumeral joint
b. PIP joint
c. Dip joint
d. Ankle joint

6. The following joint is not categorized as TRUE JOINT:


a. Gleno-humeral joint
b. Humero-Ulnar joint
c. Calcaneo-Cuboid joint
d. Scapulo-thoracic joint

7. The following movements are possible in Ankle-Mortise:


a. Endorotation-exorotation
b. Flexion-extention
c. Abduction-adduction
d. Eversion-inversion

8. One of the following statements about AS (Ankylosing Spondylitis) is TRUE:


a. Most common in females
b. The ealiest symptom is usually LBP
c. The etiology is thought to the infections
d. The onset of symptom is often abrupt

9. De Quirvain disease is a stenozing tenosynovitis of the following muscle:


a. Abductor pollicis longus – Extensor pollicis longus
b. Abductor pollicis brevis – Extensor pollicis brevis
76
c. Abductor pollicis longus – extensor pollicis brevis
d. Abductor pollicis brevis – Extensor pollicis longus

10. The purpose of Patrick test is to locate the possible pathology of the following joint:
a. Lumbosacral joint
b. Sacro-iliac joint
c. Hip joint
d. Knee joint

11. Ortolani test is to examine the possibility of the following pathology:


a. Congenital hip dislocation
b. Congenital Talipes equinovarus
c. Congenital patella dislocation
d. Congenital limb deficiency

12. The presence of a Scalenus Anticus Syndrome can be detected through this following
sequence of manouvers:
a. Deep inspiration – Turn the head backward and lateralward
b. Deep inspiration – Turn the head forward and lateralward
c. Deep inspiration – turn the head in circumduction
d. Deep exhalation – Turn the head backward and lateralward

13. If Trendelenburg test is positive during one-legged standing this is caused by the
weakness of the following muscle:
a. M. Gluteus minimus
b. M. Gluteus medius
c. M. Gluteus Maximus
d. M. iliopsoas

14. Asking a patient to hop (jump) on one leg is mainly directed to examine the following
muscle:
a. M. tibialis anterior
b. M. tibialis posterior
c. M. Gastrosoleus (Triceps surae)
d. M. Quadriceps femoris

15. The knee flexion is in the biggest position in the following normal gait phase:
a. Heel strike
b. Heel off
c. Mid stance
d. Mid swing

16. The smallest unit of muscle contraction is:


a. Muscle fiber
b. Actin and myosin
c. Itrafusal fibers
d. Sarcomere

17. The mechanism of muscle spasticity is due to the following factor:


a. Oversensitivity of the muscle spindle.
b. Decreased sensitivity of muscle spindle
77
c. Oversensitivity of golgo Tendon Organ
d. Decreased sensitivity of Golgi Tendon Organ

18. One of the following manifestations IS NOT occurred in Disused Syndrome in to


prolonged immobilization:
a. Increasing of the cardiac reserve
b. Decreasing of the lung vital capacity
c. Deep venous thrombosis
d. Increasing of the heart rate

19. The conduction velocity of the nerve fiber coming from the Nuclear bag of the muscle
spindle is:
a. 20-40 m/ sec
b. 40-60 m/sec
c. 60-80 m/sec
d. >100 m /sec

20. The afferent nerve fibers coming from GTO (Golgi Tendon Organ) belong to the
following group of nerve fiber:
a. A- alpha
b. A – betha
c. A – gamma
d. A – delta

21. There is exquisite localized tenderness over the lateral humeral epicondyle on palpation.
This is reproduced by resisted extension of the wrist joint. This a sign of strong
possibility of:
a. Student’s elbow
b. Golfer’s elbow
c. Tennis elbow
d. Osteoarthritis elbow

22. When the patient feels pain in the shoulder on flexing and supinating the forearm on
resitance, this implies to:
a. Suprascapular tendinitis
b. Infraspinatus tendinitis
c. Bicipital tendinitis
d. Supraspinatus tendonitis

23. Hyperextension of the Metatarso-phalangeal joints and distal interphalangeal joint and
flexion of the proximal interphalangeal joints is called:
a. Claw toes
b. Hammer toes
c. Mallet toes
d. Rigid toes

24. With osteoarthritis, restriction of shoulder motion is usually most significant in the
direction of:
a. Flexion & internal rotation
b. External rotation & abduction
c. Internal rotation & external rotation
78
d. Flexion & abduction

25. The success of improving flexibility in OA shoulder is determined mostly by:


a. Flexibility of the pectoralis mayor & minor
b. Strength of the rotator cuff
c. The extent of mechanical bony blockade
d. Regular compliance with their home exercise program?

26. The simple parameter to check the aerobic performance is the :


a. Blood pressure
b. Respiratory rate
c. Heart rate
d. Muscle endurance

27. The pathology of Rheumatoid Arthritis is primarily located in the following tissue :
a. Joint capsule
b. Joint cartilage
c. Subchondral bone
d. Synovial membrane

28. Pain around the medial aspect of the knee joint is very often caused by inflamation of the Pes
Anserinus. This is a conjoint tendon of the following muscles:
a. Sartorius-Gracillis-Semitendinosus
b. Gracillis-Sartorius-Biceps femoris
c. Sartorius-Biceps femoris-Semitendinosus
d. Semimembranosus-semitendinosus-Sartorius

29. One of the following criterias IS NOT TRUE regarding the diagnosis of RA according to
ACR (American College of Rheumatology) :
a. Morning stiffness
b. Artrhritis more than 3 joints
c. Asymmetrical artrhritis
d. Rheumatoid factor

30. The main function of muscle spindle is :


a. To maintain muscle volume
b. To maintain muscle endurance
c. To maintain muscle flexibility
d. To maintain muscle tonus

31. A 70 years old patient comes to your PM&R outpatient clinic with axial back pain that is
worse with standing and walking. At times he also experiences intermittent claudications from
the lumbar spine into the legs when walking. What is the most likely diagnosis?
a. Sacroilitis
b. Lumbar stenosis
c. Piriformis syndrome
d. L4-L5 Herniated disc

79
32. A 35 years old man suffered of pain in his right knee for this last 5 days, mostly after he
finished jogging in the morning. The pain located along the lateral side of his thigh through the
knee and there was a tender point at just above the lateral epicondyle of his right knee.
This man probably might suffered of :
a.Osgood-Schlater syndrome
b. Quadriceps muscle strain
c. Ilio tibial band syndrome
d. Hamstring tightness

33. The treatment of that condition (number 32) is:


a. Stretching the Hamstring
b. RICE
c. Surgery
d. Taping

A thirty-one year old female suffered of spinal fracture after a traffic accident. She was not able
to move her arms and legs and there was numbness feeling through her body from the papilla
mamae downward.

34. According to the Standards for Neurological Classification of Spinal Cord Injury published
by the American Spinal Injury Association (ASIA):
a. Shoulder abductors are 1 of the 5 key upper-extremity mucscle groups
b. The zone of partial preservation is only pertinen in complete spinal cord injury
c. If half the key muscle below the neurologic level of injury have a muscle grade less than 3,
the ASIA grade is D
d. The patient should be examined in a seated position

35. This following step should be followed first on classifying a SCI patient:
a. Perform anal examinations to classify as complete or incomplete
b. Determine zone of partial preservation
c. Perform sensory examination bilaterally for pinprick and light touch including test
for anal sensation
d. Perform motor examination in the 10 key muscle groups including anal contraction

36. An “anterior cord syndrome” should be suspected if :


a. Greater weakness in the upper limbs than in the limbs
b. Greater ipsilateral proprioseptive and motor loss and contralateral loss of sensitivity to
pain and temperature
c. Injury that resulting a reflexic bladder, bowel and lower limbs
d. Variable loss of motor function and pain and temperature while preserving
proprioception

37. Which statement is TRUE concerning bladder management considerations in acute SCI?
a. Increased urine volumes occur with immobilitation
b. Intermitent catheterization may begin once intravenous fluids are discontinued and
strict urine outputs are not needed
c. Intermittent catheterization volumes should be greater than 250ml
d. Prophylactic antibiotics for prevention of urinary tract infections are recommended

38. In the Brown-Sequard syndrome of spinal cord hemisection, a spastic paresis develops in the
muscle innervated by nerves derived from spinal roots:
80
a. Bilaterally at the level of injury
b. Ipsilaterally at the level of injury
c. Contralaterally at the level of injury
d. Ipsilaterally below the level of injury

39. Autonomic dysreflexia usually was found in spinal cord injury patients with:
a. Lesions above T10
b. Lesions above T8
c. Lesions above T6
d. Lesions above T4

40. ASIA Impairment scale B is :


a. Motor and sensory incomplete with the strength of most muscles below the lesion at grade
3 or less
b. Incompletes sensory but no motor function preserved through S4-5
c. Motor and sensory incomplete without functional motor recovery
d. Motor and sensory incomplete (motor functional) with most muscle 3/5 or greater in
strength

41. The following wheelchair is the most appropriate choice for a C8 complete tetraplegia
patient:
a. Regular wheelcair
b. Mouth-operated electrical wheelchair
c. Chin-operated electrical wheelchair
d. Hand-operated electrical wheelchair

42. An individual with C7 ASIA D tetraplegia must have


a. A bulbocavernosus reflex and voluntary sphincter contraction
b. A muscle grade of 3 or greater in at least half of the key muscle below C7
c. Normal pinprick and light touch sensation through the sacral dermatomes
d. Normal strength (5/5) in the C7 myotome

43. According to ASIA (American Spinal Injury Association), the area to evaluate pinprick
sensation for determining sensory level of T4 is :
a. The acromion
b. Lateral side of elbow
c. The nipple line
d. Apex of axilla

44. The following condition of a SCI patient has the highest possibility of successful ambulation
using HKAFO brace and bilateral axillary cruthes:
a. T8 paraplegia ASIA A
b. T9 paraplegia ASIA A
c. T10 paraplegia ASIA A
d. T12 paraplegia ASIA A

45. The bowel training in SCI patient has to consider all of the following factors, except :
a. The use of laxatives
b. He previous bowel habits
c. The toilets conditions
d. The physiological Gastro-Colic reflex
81
46. The bladder type of patient with T8 paraplegia ASIA A is:
a. Detrussor flaccid – Sphincter flaccid
b. Detrussor flaccid – Sphincter spastic
c. Detrussor spastic – Sphincter flaccid
d. Detrussor spastic – Sphincter spastic

47. The absolute contraindication for performing Crede expression to a neurogenic bladder is:
a. Presence of bladder stone
b. High intravesical pressure
c. Bladder overdistension
d. High volume of residual urine

48. One of the following signs IS NOT part of the Autonomidysreflexia in SCI Patient :
a. Heart rate 120-140 / minute
b. Blood pressure 220/120
c. Sense of chest fullness
d. Pounding headache

49. A male patient with T10 paraplegia ASIA a will have high possibilities to get the following
situations regarding the sexual functions:
a. Chance to get erection but poor penetration
b. Chance to get erection, good penetration but no ejaculation
c. Chance to get erection, good penetration, poor fertility
d. Chance to get erection, poor penetration, good fertility

50. The most important measure for a SCI using Indwelling catheter is :
a. Insert the catheter by Non-Touch technique
b. Change the catheter every other day
c. To drink enough volume of water
d. Bladder irrigation every 4 hours

51. A stroke patient with difficulty in repeating a sentence has the following language problem :
a. Aphasia Broca
b. Aphasia Wernicke
c. Anomic aphasia
d. Conduction aphasia

52. A stroke patient with Right hemiplegia and contralateral ophthalmoplegia is having the
following syndrome :
a.Benedict
b. Foville
c. Weber
d. Wallenberg

53. Amaurosis vigax is a sign of the disturbance of the following artery :


a. A. Carotis communis
b. A. Carotis interna
c. A. vertebro-basilaris
d. A. Cerebri posterior

82
54. One of the following IS NOT the predictor for death after a stroke :
a. History of smoker
b. Abnormality of ECG
c. Diabetes Mellitus
d. Haemorhagic stroke

55. Wallenberg syndrome is caused by the thrombosis affecting the following artery :
a. A. Cerebri media
b. A. Cerebri posterior
c. A. Cerebelli superior
d. A. Cerebelli inferior posterior

56. The following movement is GOOD in a case of “Upper Brachial Plexus Injury” :
a. Shoulder external rotation
b. Shoulder abduction
c. Elbow flexion
d. Wrist flexion

57. “Anterior Interosseous Syndrome” is an entrapment neuropathy of the following nerve :


a. Median nerve
b. Ulnar nerve
c. Radial nerve
d. Axillary nerve

58. One of the following statement IS NOT TRUE regarding the Carpal Tunnel :
A. Median nerve is the only nerve inside it
B. The tunnel is very narrow
C. There are 8 tendons inside it
D. The basis of the tunel are the carpal bone

59.The best treatment of spasticity in SCI is :


A. Diazepam oral
B. Diazepam i.v
C. Baclofen oral
D. Baclofen intratecal

60. The effect of Diazepam in reducing muscle spasticity is through the following mechanism :
A. To stimulate GABA
B. To stimulate Aspartate
C. To inhibit Calcium ion release
D. To inhibit acetyl choline
83
61.One of the following statements IS NOT correct regarding the H-reflex :
A. It is a monolysynaptic reflex
B. The average value in adult us about 50ms
C. This is a typical test to see the abnormality of S1 root
D. The “H” is for Hoffmann

62. One of the following statements is TRUE about EMG examination :


A. Able to establish diagnosis of Central nervous system disease
B. Able to establish diagnosis of Peripheral nervous system disease
C. Able to make early detection of possible peripheral nerve lesion
D. Able to localize a peripheral nerve lesion

63. One of the following characteristics IS NOT Correct regarding the F waves:
A. The F wave can be elicited by placing the recording electrode over the muscle of the
extremities while supra maximally stimulating the appropriate nerve
B. It variably appears at approximately similar location as H reflex
C. The amplitude of F waves is higher then M response
D. The stimulus ascends the motor fibers antidromically and impacts on the anterior horn
cells to return orthodromically to cause muscle contraction

64. The following factor is not affecting the results of EMG examination :
A. The skin temperature
B. Age of patient
C. Sex
D. Patient cooperation

65. In minimal contraction during the needle EMG examination IS NOT to get the following
finding :
A. The total number of motor unit which are active
B. The duration of Motor Unit Action Potential
C. The amplitude of Motor Unit Action Potential

84
D. The frequency of Motor Unit Action Potential
66. The presence of Nascent potentials in EMG examination shows the following process of :
A. Poor prognosis
B. Good prognosis
C. Reinnervation
D. Partial denervation

67. Three months after a nerve lesion the characteristics of MUAPs are :
A. Low amplitude – short duration
B. High amplitude – short duration
C. Low amplitude – long duration
D. High amplitude – long duration

68. The following is true statement about Fasciculation waves :


A. Will only occur in nerve lesion
B. The rate / frequency is irregular
C. The amplitude is less than 1000 microvolts
D. The duration is less than 4 msec

69.Electromyographic biofeedback for stroke patients is most beneficial when :


A. Proprioception is preserved
B. Used in the upper limb
C. The patient is young
D. The patient has flaccid paralysis

70. What is the definition of the 4 th degree Sunderland’s classification of peripheral nerve
injury
A. Transection of the entire trunk
B. Local conduction block with minimal structural disruption
C. Disruption of the axon, endoneurium, and perineurium. The epinueral tissue is
spared
D. Disruption of the axon only, leaving the endoneurium inytack, a neoroma in
continuity

85
71. What is the estimated rate of nerve regeneration?
A. 1 mm/day
B. 1 mm/week
C. 1 mm/month
D. 1 mm/year

72. You are evaluating a patient with a nerve palsy. During evaluation, the patient reveals a
positive Froment’s sign. Which nerve is probably injured?
A. Median
B. Radial
C. Ulnar
D. Musculocutaneous

73. The test which used to diagnose meniscal tears and especially useful for diagnosing
posterior meniscal tears is :
A. Mc Murray Test
B. Appley’s Grinding Test
C. Appley’s Distraction Test
D. Drawer’s Test

74. Which of the following is the most important lifestyle modification for prevention of
osteoporosis?
A. Decreasing the intake of alcohol
B. Eating a diet high in protein and phosphorus
C. Avoiding cigarette smoking and high intake of caffeine
D. Minimizing the use of nonsteroidal anti-inflammatory mediations

75. A 40 year old woman is evaluated in the ER after a motor vehicle accident resulting in the
left facial injuries. Examination after she recovers acutey demonstrates that her left VII and VIII
cranial nerves remain dysfunctional. Which of the following skull structures may have been
affected by her injury :

86
A. Cibriform plate
B. Optic canal
C. Superior orbital fissure
D. Internal auditory canal

76. Findings in Duchenne’s muscular dystrophy DO NOT include :


A. CPK elevation
B. EMG shows long duration and high amplitude MUAPs
C. Non specific muscular fiber changes microscopically
D. Positive Gower’s sign

77. One of the following characteristics IS NOT TRUE regarding Duchene’s Muscular
Dystrophy :
A. The disease affects males
B. It is transmitted by X- linked mode of inheritance
C. The child’s mother, his sisters and maternal aunts are potential carriers
D.The motor NCV studies show significantly reduced (Normal )

78. One of the following characteristics IS NOT TRUE regarding Becker’s Muscular
Dystrophy :
A. This form is similar to Duchenne’s dystrophy in terms of distribution of weakness
B. It has a later onset than Duchenne’s muscular dystrophy
C. Intellectual impairment is the most common clinical findings
D. It is more benign than Duchenne’s muscular dystrophy

79. The trainer for People With Disabllity (PWD) In CBR area ls:
A. Family
B. Cadres
C. Formal leader
D. Community

80. One of the statements below in CBR is true, that Is for self survey, It should be
Done after :
A. Training CBR cadres
B. Village mini workshop
C. Dissemination information at provincial level
D. Implementation of CBR Manuals

81. When treating patients with cerebral palsy, consider :


A. AFO Is only in nonambulatory patien for positioning and contracture prevention
B. Bracing to prevent progressive deformity of scollosis
C. Vocational rehabilitation for job training and placement
D. Limited strengthening as it has been shown to increase spasticity

82. Congenital Talipes Equinovarus is a deformity of the foot :


A. Disorder of the posterior part of the foot
B. Disorder of the medial part of the foot
C. Disorder of the anterior part of the foot

87
D. Disorder of the posterior, medial and anterior parts of the foot

83. Which of the statement below is correct ?


A. Dyskinetic cerebral palsy is further Specified by body region involved (Monoplegia,
diplegia, triplegia, quadriplegia, and hemiplegia)
B. Deterioration of function can occur if the lesion causing cerebral palsy Progresses
C. Most patients exhibit lower motor neuron Signs.
D. Lack of basic functional skills is the key predictor that reduced life expectancy

84. One of the following statements IS NOT CORRECT regarding CP :


A. The underlying neurologic lesion must be static
B. The process which cause the cerebral palsy is still active
C. It is caused by non progressive injury which affects the immature brain
D. The sign of cerebral palsy is a disorder of movement and posture

85. One of the following positions Is NOT TRUE regarding the Erb’s paralysis :
A. Shoulder abduction
B. Endorotation of the upper arm.
C. Ptonation of forearm
D. Elbow extension

86. Neurodevelopmental treatment to normalize tone inhibit abnormal primitive


reflex patterns and facilitate automatic reaction and subsequent normal
development is :
A. Kabat
B. Bobath
C. Rood
D. Voyta

87. In Klumpke’s type of Brachial plexus injury the following muscle is mostly
affected :
A. Lumbricals I, II, III, IV
B. Extensor carpi radialis longus
C. Extensor carpi radialis brevis
D. Extensor carpi ulnaris

88. Your 5-year-old Patient with spastic tetraplegic cerebral palsy needs a wheelchair
prescription. He is dependent for transfers, but cognitively normal. He is able to feed
himself and uses a communication device. His family transports him in their car in an
adapted car seat. On examination, he is unableto sit unsupported, but sirs well with
minimal support; he has no scoliosis, and his passive range of motion is full. Which
elements would be best to include in his wheelchair prescription?
A. Folding frame, sling seating
B. Adaptive stroller, linear seating
C. Tilt in space frame, custom seating
D. Rigid frame, countered seating

89. On examination, a 3-month-old girl still has a Moro reflex, asymmetric tonic neck
reflex, and plantar grasp reflex. She does not have any protective extention. You
advise her parents that :
A. These reflexes are normal reflexes
88
B. She needs a neurology evaluation
C. She requires a physical therapy evaluation
D. Further diagnostic evaluation is indicated

90. An infant is born at term with normal weight and Apgar. She has hypotonia and
poor suck, temted mouth and unable to fully close her eyes. This is the first child of a
20 year old maternal uncle with cataracts, clumsiness of the hands, and recent onset of
DM. It is expected that
A. The baby’s symptoms will be rapidly progressive
B. The baby’s symptoms will resolve with minimal sequalae
C. The baby’s symptoms are consistent with congenital myotonic dystrophy

D. The baby’s symptoms are consistent with Wernig Hoffmann disease

91. A 9-year-old boy with primary muscle disease is being evaluated to correct toe
Walking. Reasons for toe-walking might include :
A. Weakness of plantarflexors
B. Hip flexor weakness
C. Hamstring hyperextensibility
D. Quadriceps weakness

92. A 7 year-old girl with a spastic diplegia pattern of motor function due to
cerebral palsy. She was integrated into a normal elementary school. She is
inteligent and alert and enjoys all aspect of her school routine. Her gait slow and
labored, making it difficult to walk in distant locations. She has a large part of the
reces activity. The large patient’s problem is :
A. Limited ROM
B. Social and emotional
C. Tone, control and strength
D.Unable to ambulate independently

93. With the problems above, the most appropriate program is :


A. Night Splints
B. Daily ROM exercise
C. Improve strength, control of trun musculature and gait training
D. Educate and support patient in way to cope with and compensate for
handicap

94. When a child is diagnosed as cerebral palsied, the question asked by the parents is always
“will he walk?”. To answer these question, we have to deal with probability. These following
tests were done and this point gave a GOOD PROGNOSIS :
A. Parachute reaction
B. Extensor thrust
C. Moro reflex
D. ATNR

89
95. SMA type I (severe) from was defined by The International SMA Consortium
as follows:
A. Onset from birth to 6 months, no achievement of sitting without support, and
death usually before age 2 years.
B. Onset from birth to 6 months, sitting is usually obtained but death usually
before age 2 years
C. Onset from birth to 6 months, usually can live longer than 2 years, although no
achievement of sitting without support
D. Onset from birth to 6 months, sitting is usually obtained, but standing and
ambulation are never obtained, and death usually before 2 years

96. Arthrogryposis multiplex congenital most commonly is:


A. Neurogenic in origin
B. Myopathic in origin
C. Associated with early joint fusion
D. Asscociated with muscle hypertrophy

97. One of the following statements IS NOT the characteristic of Juvenile


Rheumatoid Arthritis :
A. Joints are swollen, stiff, sometimes warm and tender
B. A chronic connective tissue disease
C. Occurs in individual <20 years
D. No specific diagnostic test for RA

98. A 3 year old girl with a history of swelling of the knees bilaterally and left ankle
with restricted ROM. She has no fevers, no history of trauma. The pupils show
slight irregularity in shape when they constrict to light. The diagnosis is
A. pauciartieular juvenile RA C. SLE
B. polyarticular juvenile RA D. Still's disease

99. Six years old boy with Syndrome who develops torticollis, hyperrelexia,
lower extremity hypertonicity after open heart surgery. You should
A. EMG of the Lower extremity.
B. Obtain cervical spine x-ray
C. MRI Of lumbal spine
D. Ophthalmologic evaluation

100. A 2-month-old infant presents to you for evaluation of delayed development.


He was the product of a normal term pregnancy, labor, and delivery. Birth
weight was 3500 grams. He has had difficulty feeding since birth. Family
history is negative for developmental problems. On physical examination, he
is awake, but not alert. Weight is 3600 grams, Respiration is unlabored. He
has poor head control and decreased tone throughout. Deep tendon reflexes
are absent. What is the most likely diagnosis?
A. Kugelderg Welander Syndrome
B. Duchenne Muscular Dystrophy
C. Infantile Botulism
D. Tetraplegic Cerebral Palsy

90
Answer:
A. if numbers 1,2,3 are correct
B. if numbers l,3 are correct
C. if numbers 2,4 are correct
D. if only number 4 is correct
E. if all are correct

101. The following methods of exercise are principally using the PNF approach :
1. Kabat
2. Voss
3. Knott
4. Bobath

102. Bowel incontinence management for spina bifida is / are:


1. Increase fiber and bulk in diet
2. Reflex emptying techniques
3. Maintain a consistent daily time for bowel movement ,
4. Strengthening of accessory muscles for relaxation or expulsion

103. congenital muscular torticollis caused by


1. Deformation from intrauterine positioning vo.
2. Hemivertebrae ;
3. Fibrotic contracture of the sternocleidomastoid wd
4. Ipsilateral plagiocephaly

104. For the child with cerebral palsy, the orthosis should be used on the lower extremities when
the child :
1. Begins to pull to a stand on toes
2. Stands flat but walks on the toes extension and the feet in
3. Non ambulatory but tends to keep the leg in extension and the feet in Plantar
flexion ankle
4. Has fixed contractures of the foot and

91
105. Leg length discrepancy may result from:
1. Undergrowth
2. Overgrowth
3. Hip dislocation 4
4. Neuromuscular abnormalities

106. The condition of mental retardation in children with cerebral palsy is / are as follow:
1. Frequently important factor influencing rehabilitation.
2. Seen in 50% of children with CP
3. Most common in spastic quadriplegia
4. The child needs IQ test to know that the child is educable or just trainable.

107. Communication disorder in cerebral palsy:


1. Occurs in 40% of children with CP (especially quadriplegia and extra pyramidal
a type)
2. Due to oral motor problems
3. Sometimes causes misdiagnosis of mental retardation.
4. Begin speech therapy early and continue throughout childhood

108. Seizures in children with cerebral palsy:


1. Seen in 50% of children with CP
2. Associated with spastic (pyramidal) CP
3. Most frequently in spastic hemiplegia and quadriplegia
4. No need anti seizure medication

109. The peripheral influences on motor neuron excitability usually used a,


proprioceptive neuromuscular facilitation exercise techniques is / are:
1.Cerebellar and corticospinal input to the alpha motor neurons.
2.Muscle receptors such as the intrafusal muscle system.
3. Psychological influences such as emotion.
4. Afferent input to the alpha motor neurons from the skin and tendon receptor

92
110. Proprioceptive neuromuscular facilitation (PNF) program typically include
1. Deep cooling of muscles
2. Muscles stretching
3. Withdrawal reflexes
4. Superficial cooling

111. The following is / are the early treatment(s) of CTEV:


1. Dennis-Brown splint
2. Outflare Orthopedic shoes
3. Surgical soft tissue release
4. Stretching and Serial plaster correction

112. The basic Assessment of children patients with delayed growth and development Should
include the following areas
1. Gross motor test
2. Fine motor test ,
3. Language test
4. IQ test

113, The respiratory function of a patient with ankylosing spondylitis is characterized by :


1. moderate dyspnea on exertion
2. decrease in diaphragmatic excursion
3. Increase use of accessory muscles
4. Diminished chest expansion

114,The folowing conditions is/are the contraindication(s) for postural drainage approach: .;
1. Hemoptysis ,
2. Severe hypertension
3. Cerebral edema
4. cystic fibrosis

115. The objective(s) of giving pursed-lips breathing Is / are :

93
1. To decrease respiration rate. a
2. Toincrease Tidal Volume.
3. To increase oxygenation. a
4. Introduce the patient of how to control respiration

116. Respiratory management for COPD comprise of the following :


1. Aerosol therapy
2. Oxygen therapy
3. Chest physical therapy
4. General Reconditioning exercise

117 The absolute contraindication(s) of exercise in Chronic Heart Failure is/are :|


Unstable angina
Aortic stenosis
Active myocarditis
Old age

118. The goal(s) of rehabilitation program for COPD is/are:


1. To gain independence ambulation
2. To optimize oxygen needs
3. To improve endurance
4. To improve muscle strength

119. The common feature(s) of Parkinson’s disease is /are:


1. Tremor
2. Rigidity
3. Bradykinesia
4. Hyperactive of physiological reflexes

120. The following exercise(s) is / are important for Parkinson patient ;


1. Breathing exercise
2. Agility exercise
3. Gait training exercise
4. Increasing speed exercise
94
121. The following statement(s) is / are Important to be done in every geriatric patient : ,
1. Careful clinical geriatric assesment (CGA)
2. Careful & competent functional assessment
3. Management to identify remediable problems
4. Management to minimize functional status

122. The following is / are the ADL measurement(s) for the elderly :
1. Barthel’s Index ,
2. FIM
3. Alzheimer Disease Assessment Scale
4. Katz Index

123. The benefit(s) of Aquatic therapy for geriatric patient is / are:


1. Buoyancy enables to perform ROM activities
2. Aquatic exercise has been shown to reduce pain in arthritis
3. Buoyancy provides early reaction of individuals who have been injured
4. To promote recovery from inactivity

124. Risk factor(s) of FALL among elderly peoples is / are :


1. Muscular weakness
2. Lack of outdoor activities
3. Poor balance
4. Living with family

125. The Fall-related assessment in the elderly is / are :


1. Visual assessment
2. Neuro-musculo-skeletal assessment
3. Mental status assessment
4. Postural blood pressure assessment

Choose the most appropriate answer :

95
126. Gait changes that predispose to falls and injuries in old men :
1. Waddling gait
2. Small stepped gait
3. Narrow walking and standing base
4. Wide walking and atanding base

127. The advantages of SWD is / are :


1. Easy of the application
2. Does not require contact between the body surface and the electrodes
3. Heat is relatively cover large area than do other deep-heating modalities
4. Is very cheap

128. The effects of electrical stimulation is /are :


1. Muscle contraction
2. Maintains muscle nutrition
3. To give proprioceptive stimulation secondary to muscle contraction
4. Maintains muscle strength and endurance

129. The following statement(s) is / are true :


1. The mechanism of heat transfer of paraffin bath is conduction
2. The mechanism of heat transfer of whirlpool is convection
3. The mechanism of heat transfer of SWD is conversion
4. The mechanism of heat transfer of USD is radiation

130. OA is / are not likely to involve the following joint(s) :


1. Elbow
2. Suprahumeral
3. PIP
4. Temporo-occipitalis

131. The following deformity(s) is / are classical in advance RA :


1. Swan Neck Deformity
2. Piano-Key sign
3. Boutoniere deformity
4. Buchard’s nodes

132. The ACR (American College of Rheumatology) criteria for OA of the knee is / are :
1. Knee pain
2. Normal X-ray of the knee
3. Crepitation of the knee on palpation
4. Age>40

133. The following condition(s) has / have good prognosis :


1. Axonotmesis
2. Partial denevation
3. Neurotmesis
4. Neuropraxia

134. The following wave is / are considered normal in needle EMG examination :
1. Fasciculation waves
2. Triphasic waves
96
3. Positive sharp waves
4. Biphasic waves
135. The summary of an EMG examination should include the following :
1. The synopsis of all of the data found in EMG examination
2. The disease entity leading to the abnormality finding in EMG
3. The clinical impression(s) of the motor unit pathology
4. The prognosis of the disease

136. The cardinal sign(s) of leprosy is / are :


1. Claw hand deformity
2. Anesthetic maculae
3. Insensitive feet
4. Thickening of superficially-located nerves

137. The following statement(s) is / are true regarding cervical traction :


1. Can be done in sitting position
2. The position of the neck is in extension
3. RA affecting cervical spine is contraindicated
4. The weight of traction is 50% of total body weight

138. The transfemoral (above knee) prosthesis consist of :


1. Foot-ankle assembly
2. Shank
3. Knee unit
4. Suspension device

139. Ankle-foot orthoses are more commonly prescribed for :


1. Muscle weakness affecting the ankle
2. Prevention or correction of deformities of the foot ankle
3. Muscle weakness affecting the subtalar joints
4. Increasing of weight bearing forces

140. The following(s) is/are indication of biofeedback therapeutic :


1. To inhibit spasticity
2. Muscle reeducation for volitional movement of muscle with upper motor neuron
dysfunction
3. Reeducation of pelvic floor musculature in woman with incontinence or pelvic pain
4. To strengthen muscle

141. Ultrasound is effective in :


1. Increasing the range of motion of the heel cord
2. Periarthritic shoulders
3. Contracted hips
4. Dupuytren contracture

142. The following statements is / are the sign(s) of cerebellar dysfunction :


1. Hipotonia
2. Asynergy
3. Nystagmus
4. Dysarthria

97
143. The objectives in medical rehabilitation management for dysphagia is/are ?
1. To prevent aspiration
2. To maintain an adequate nutritional intake
3. To reestablish oral eating to the safest optimum level
4. To determine surgery indication

144. The following is/are the source(s) of shoulder pain in hemiplegic patient :
1. Subdeltoid bursitis
2. Rotator cuff tear
3. Bicipital tendonitis
4. Shoulder subluxation

145. The typical flexion synergic spasticity pattern in post stroke patient with one sided
hemiparetic is/are :
1. Retraction, depression and internal rotation of the shoulder
2. Protraction and external rotation of the shoulder
3. Flexion and adduction of the fingers
4. Forearm extension

146. The following statements below are specific regarding thoracic outlet syndrome :
1. May caused by a cervical by a cervical rib that compresses the medial cord of the
brachial plexus
2. Tingling and numbness in the medial ospect of the arm
3. Decreased extremity pulses
4. Produced upper motor neuron deficits

147. The following is / are the characteristic(s) of neuropatic potentials :


1. Duration : 20 msec
2. Amplitude : 0,5 mV
3. Shape : polyphasic
4. Sound : dive bomber

148. The following wave(s) is / are considered normal in needle EMG examination :
1. Triphasic waves
2. Biphasic waves
3. Polyphasic waves < 10%
4. Sound : sharp

149. The following syndrome(s) may occur in vertebro-Basisar stroke :


1. Weber
2. Millard-Gubler
3. Wallenberg
4. Dejerrine-Roussy

150. The information of nociceptive system from the receptor to the brainstem is / are through
the following process(s) :
1. Transduction
2. Transformation
3. Conduction
4. transmision

98
99
Board Examination (Desember 2010)

1. Which of the following statement is true?

a. The most common cause of thoracic radiculopathy is diabetes melitus

b. Common level affected by herniated disc at cervical are C4-5, C5-6 and C6-7

c. L4-5 disc nerve most commonly herniates than L5-S1

d. The nerve root involved most commonly in thoracic outlet syndrome are C8 and T1

2. An 80 year old man presents with a 2 year of history of low back pain radiating down from both legs to his
ankles. He also feel weakness in his left foot. The pain is increased with walking and relieved within
seconds of cessations of activity. Leaning forward eases his pain and lying supine relieves it. Which of the
following is most likely diagnosis?

a. HNP

b. Facet arthopathy

c. Muscle spasm

d. Spinal stenosis

3. Phantom pain refers to :

a. Any sensation of the missing limb, except pain.

b. Painful sensation referred to the missing limb

c. Spontaneous movement of the visible contraction

d. Pain referred to the amputation stump

4. Stump pain and phantom pain are often confused. There are however, notable differences. One of the
following statements is wrong :

a. Unlike phantom pain, stump pain occurs in the body part that actually exist in the stump that remains

b. Stump pain typically is describe as a ‘sharp’, ‘burning’, ‘electric like’, or ‘skin sensitive’ pain

c. Surgical revision of the stump or removal of the neuroma is sometimes considered when treating stump
pain

d. Stump pain is the precursor of phantom pain

5. One of the following signs and symptoms is not consistently found with cervical radiculopathy

a. Gait disturbance

b. Normal muscle tone

c. Negative babinsky test

d. Positive axial compression test (spurling maneuver)

100
6. Which of the following statements is true regarding arthritis?

a. OA often cause malaise, fatigue, weight loss and fever

b. AS is a chronic disease that primarily affect the spine and may lead to stiffnes of the back

c. Diagnosis for gouty arthritis is made through blood test for HLA-B27 genu

d. RA actually have acute onset of symptoms that develop over days

7. A 42 years old female patient came to the out-patient clinic with the complaint of back pain. What is the
first examination you will do?

a. Straight leg raising test

b. Flexion extension of the trunk

c. Observation of gait pattern

d. Six minute walking test

8. From your examination you find that above patient has a thoraco-lumbar scoliosis with a 45-degree Cobb’s
angle. What will be your choice of treatment for this patient?

a. Exercise program

b. Treatment with modalities

c. Bracing program

d. Treatment with modalities and exercise program

9. Which one is a cause of postural/functional back pain?

a. Scoliosis

b. Spondylosis

c. Hyperlordosis

d. Spondylotisthesis

10. You have a female patient, 15 years old with the diagnosis of idiopathic scoliosis, major curve in the
thoracal vertebra, with 45 degree Cobb’s angle. When and/or what is the indication to refer her to
orthopedic surgeon?

a. She complain of back pain

b. She complain of shortness of breath

c. She has problems with her appearance

d. She complains of the asymmetry of her breast

11. Patient who suffer from trochanteric bursitis, will have a special pathologic gait :

101
a. Antalgic gait

b. Spastic gait

c. Limping gait

d. Swinging gait

12. One of the most effective methods of decreasing the load on the hip joint is :

a. Reduce weight

b. Use of external support

c. Use of HKAFO

d. Immobilized the hip

13. Classification of pain by type of origin/organ affected are :

a. Nociceptive, neuropathic, visceral, mixed type

b. Musculoskeletal, neuropatic, visceral, psycogenic

c. Nociceptive and neuropatic

d. Nociceptive, neuropatic, psycogenic

14. The purpose of using pressure garment for burned patient in recovery phase is to:

a. Reduce capillary perfusion

b. Increase tissue oxygenation

c. Increased cellular activity

d. Increase collagen synthesis

15. Physical exercise component for trunk osteoporotic patient that should be avoided is:

a. Forward bending

b. Backward bending

c. Lateral bending

d. Rotation

16. One of the following is not true regarding OA :

a. Pain is the most important symptoms

b. There is continuous process of joint cartilage destruction

c. There is always concomitant osteoporosis

d. There is malalignment of the knee joint

102
17. A 70 years old white man presents to your office with axial back pain that is worse with standing and
walking. At times he also experiences pain radiating from the lumbar into the leg when walking. What is
the most likely diagnosis?

a. Lumbar canal stenosis

b. Sacroiliac joint pain

c. Piriformis syndrome

d. Ischial bursitis

18. XXX

19. XXX

20. A test for foraminal intervertebralis closure in which rotates the head to one side, side bends to the same
side and extend is :

a. Cervical spine compression test

b. Cervical spine distraction test

c. Spurling’s test

d. Lhermitte test

21. A common source of shoulder pain in shoulder impingement syndrome is :

a. Extra-articular non subacromial

b. Intra articular

c. Impingement of the rotator cuff tendon in the subacromial space

d. Biceps injury

22. The test used in the examination and evaluation of suspected cubital tunnel syndrome, which the wrist is
full extension is :

a. Tinnel’s test

b. Roos test

c. Phalen test

d. Elbow flexion test

23. Mr. ali, a 56 years old man suffer a R hemisphere stroke with a consequence of L hemiplegia. L neglect
and L homonymous hernianopsia. He had trouble with swallowing. He was completely dependent for
grooming, eating or dreassing, however he is able to ambulate with help of 1 person and a quad cane. The
persistent neglect and L homonymous hernianopsia make him unable to drive and therefor unable to return
to work the following is not classified as his activity limitation or disabilities:

a. Difficulty with grooming

b. Difficulty with eating


103
c. Difficulty with socializing

d. Difficulty with dressing

24. The following is not his participation limitation or handicap:

a. Inability to drive

b. Inability to work

c. Difficulty in bathing

d. Inability to socialize

25. One of the following statements for spasticity post stroke is not true:

a. Usually seen days to weeks post ischemic stroke

b. Usually follows classic upper extremity flexor and lower extremity extensor pattern during ambulation

c. Velocity dependent resistance to passive movement of affected muscle at rest

d. Pill rolling tremor of the fingers is very common

26. The positive effect of spasticity on a stroke patient:

a. Allows hemiplegic patient to weight bear on affected leg during stance phase

b. Better result in movement of the affected leg with electrical stimulation

c. Easier to apply back splint on the affected leg

d. Improve coordination of the affected leg

27. The following statements is not true for botulinum toxin injection:

a. Works by weakening spastic muscle by blocking the neuromuscular junction

b. Works for up to 18 month

c. In lower extremity, most treatment is focused on spastic equinovarus deformity

d. Primary goals of treatment are improvement in gait velocity

28. To reduce tone in a spastic muscle, the following is not appropriate:

a. Performing a sustained stretch of the spastic muscle

b. Placing pressure on the tendon of a spastic muscle

c. Performing a quick stretch to the spastic muscle

d. Gentle massage of the spastic muscle

29. Splinting, serial casting and seating and positioning are all therapeutic technique used to reduce tone in
hypertonic muscle group. These techniques are based on the following principle:

104
a. A quick stretch of a spastic muscle group facilitates the golgi tendon organ which inhibit the spastic muscle

b. Placing pressure on the muscle belly of a spastic muscle group facilitates the golgi tendon organ which
inhibit the spastic muscle

c. A sustained stretch of a spastic muscle group facilitates the golgi tendon organ which inhibit the
spastic muscle

d. None of the above

30. The correct statements of autonomic dysreflexia is:

a. A noxious stimulus below the lesion level in person with injuries generally above T10-T6

b. Results in a parasympathetic discharge from the splanchnic outflow

c. Because of the ability of supraspinal inhibitory signal to modulate the exaggerated sympathetic
response that is brought on by a noxious stimuli

d. The most common cause are bladder and bowel distension

31. Treatment of detrusor hyperreflexia is:

a. Cholinergics

b. Crede or Valsava technique

c. Indwelling catheter

d. Scheduled voiding program

32. Less of a sensibility on plantar pedis in MH patient cause by neuropathy of:

a. N tib post

b. N peroneus

c. N femoralis

d. N ischiadicus

33. In Barthel AD2 index there is no component:

a. Eating

b. Toileting

c. Sensory component

d. Defecatie control

34. One of the statements below are poor prognostic indicators associated with stroke outcome:

a. Prolonged flaccid period

b. Twitchell described a synergic patterns that will be followed by volitional individual component

c. In the middle cerebral territory infarct, lower limb shows more recovery that upper limb
105
d. Proprioception is a key factor in executing and re learning motor function

35. A stroke affecting the PICA will produce the following syndrome:

a. Benedict

b. Millard Grubber

c. Wallenberg

d. Foville

36. Idiopathic facial paralysis (Bell’s palsy):

a. Is usually bilateral

b. Clear completely in at least one half of cases

c. Rarely affect taste sensation

d. Is associated with decreased ipsilateral facial sensation

37. A stroke patient shows sign of weakness and sensory deficits especially in the right lower extremity, while
the right upper extremity shows minimal disturbance. There are also slight mental disturbace and aphasia.
This condition most probably affecting the following artery:

a. Anterior communicating artery

b. Anterior cerebral artery

c. Middle cerebral artery

d. Posterior cerebral artery

38. Which of following is TRUE regarding the characteristic of spasticity:

a. Decreased resistance to passive movement

b. Presence of clonus

c. Presence of fasciculation

d. Presence of atrophy

39. A six years old girl with poliomyelitis. She has an abnormal gait pattern, she show lateral trunk flexion to
the left during stance phase between foot flat and heel off, Swing phase is normal. The muscle strength of
gluteus is 4-/5 the tibialis anterior 4-/5. ROM is normal in all joint. Which gait abnormalities occurring:

a. Waddling gait

b. Stepage gait

c. Trendelenburg gait

d. Circumducted gait

106
40. A patient with a Trendelenburg gait, you suspect an injury of the following serve:

a. Femoral nerve

b. Superior gluteal nerve

c. Obturator nerve

d. Sciatic nerve

41. Your 15 years old patient with DMD complains of new morning headaches. What is the most likely cause?

a. Neck extensor tightness

b. Migraine

c. Hypercarbia

d. Vision change

42. Which insult is the most likely cause of spastic diplegic cerebral palsy?

a. Intrauterine stroke

b. Hyperbilirubinemia in the neonatal period

c. Post intraventricular haemorrhage

d. Perinatal asphyxia

43. The patient with DMD, decline in vital capacity tends to tocolacide with the onset of:

a. Limb contracture

b. Use of orthotic/assistive device for ambulation

c. Wheel chair dependence

d. Dysphagia

44. Dewi, 6 years old from CBR area, she can not play with her friends of the same age. She was born
abnormal except with low birth weight. She is unable raising the arm and she walk indenpendently with
tiptoeing gait, no other impairments. What kind of assistive device does the patient need for the lower
extremity:

a. AFO

b. Forearm cane

c. Walker

d. Bilateral axillary crutches

45. Commonly used criteria for selection of infants with high risk for cerebral palsy is:

a. Persistence of primetitive reflexes

b. Poor head control head


107
c. Feeding difficulties

d. Asphyxia at birth

46. The abnormality in oral motor control that is not impair nutrition in children with cerebral palsy is:

a. Hypoactive gag reflex

b. Tongue weakness

c. Persistent infantile reflexes

d. Poor suck reflexes

47. Which aspect has to have priority in rehabilitation management of a disables child?

a. Ambulation skills

b. Communication

c. Activities of daily living

d. Transfer sklill

48. Which reflex shows maturation of the midbrain is an infant?

a. Prone tonic labyrinthine reflex

b. Associated reaction

c. Simian posture

d. Labyrinthine righting acting on the head

49. Automatic movement reaction are reactions produced by chages in the position of the head, involving the
semi circular canals of labyrinths, or neck proprioception. Which one of the following is included in the group of
reflexes?

a. Stepping reflex

b. Simian posture

c. Parachute reaction

d. Amphibian reaction

50. In examining an infant, a sign of generalized weakness is:

a. Frog position

b. Scissoring of the legs

c. Amphibian movement

d. Asymmetry of the face

108
51. CP athetoid type most probably has perinatal history of:

a. Icteric

b. Hypoxic

c. Fell down

d. Fits/seizure

52. A girl 2 years old, was diagnosed spina bifida with bladder problem need catheterization, your prescription
for the catheter will be:

a. Nelator catheter size 15 non latex

b. Nalaton catheter size 15 latex

c. Nelaton catheter size 12 non latex

d. Nelaton catheter size 12 latex

53. At what the most early age you may prescribe bilateral axillary crutches for a child?

a. 12 months

b. 18 months

c. 24 months

d. 30 months

54. One of the following associated condition in the cerebral palsy child should not be treated by medication:

a. Seizure

b. Severe scissoring

c. Severe spasticity

d. Hyperactivity

55. For which of the following symptoms of cerebral palsy will bracing be most beneficial?

a. Ataxia

b. Spasticity

c. Athetosis

d. Contracture

56. The tonic neck reflex is abnormal if:

a. It is variable in the same infant

b. The ipsilateral extremities extend

c. If persist more the six month of age

109
d. The contralateral extremities flex

57. The following condition is positive response asymmetrical tonic neck reflex:

a. When the head is turned the trunk and limb turn to the same side

b. When the head is turned the limb flex in the skull side(occipital) and extend on the face side

c. Flexion side head causes flexion of forelimbs and extension of the hind limbs

d. When the head is turned the limbs extend on the skull side(occipital) and flex on the face side

58. The most important and reliable examination to indentify potensial carrier of ducheane’s muscular
dystrophy is:

a. Muscle biopsy

b. Measurment of serum creatinine kinase

c. Measurement of serum pyrufate kinase

d. Measurement of serum lactic dehydrogenase

59. The most common cause of dead of ducheane’s muscular dystrophy is:

a. Cardiac failure

b. Respiratory failure

c. Marasmus

d. Ulcus decubitus

60. Absence of radius alone, according to classification of congenital limb deficiency include:

a. Horizontal terminal deficit

b. Longitudinal terminal deficit

c. Intercalary ransvense deficit

d. Longitudinal deficit

61. Waiter’s tip position is characteristic condition which involve nerve roots of: A

a. C5 C6

b. C7

c. C8 T1

d. C5 C6 C7 C8 T1

62. The following statement is not true for SMA type 1:

a. Clinical signs under 3 years of age

110
b. Medium age of dead exceed 10 year

c. The synonym is kuguelberg-welander syndrome

d. Autosomal recesive

63. ATNR reflex appreances parallel with level brain development of:

a. Spinal

b. Brainterm

c. Midbrain

d. Cortical

64. The following is NOT the beneficial effect of walking test, submaximal exercise test for the chronic
pulmonary patients:

a. To assist detection of exercise intolerance

b. To know and determine functional limitatons

c. To measure limitation of pulmonary function

d. To evaluate benefit and effect of medication to functional capacity

65. Which one of the most important and the most beneficial of breathing control training for COPD:

a. Diaphragm breating

b. Expiratory pursed lip

c. Combination of diaphragm brating and expiratory pursed lip

d. Combination of relaxed deep breathing and expiratory pursed lip with 3-5 second holding inspiration

66. Which statement the most important thing to perform exercise test :

a. Symptom limited test is the choice for COPD in all stage

b. 6 MWD test and 12 MWD test are highly recommended for COPD in all stage

c. Monitoring of heart rate and blood pressure before, during and after exercise test is an absolute thing to do
in performing exercise test

d. Monitoring of dyspnoe scale, muscle fatique and Sa02 is absolute and heart rate and blood pressure
as well

67. One thing should be considered in performing 6 MWDT in COPD patient:

a. It is important to measure pulmonary function

b. It is conduct in the way we conduct 6 MWDT in cardiac patient

c. It is not a maximal test describing real functional capacity but can be a parameter of training to measure
functional capacity of patient

111
d. It is just submaximal test to measure but can describe the real functional capacity of patient

68. Using spirometry to get the right number of lung volume, the maneuver could be:

a. Breating as tidal breathing

b. Fast blowing

c. Fast and prolong expiration

d. Prolong expiration

69. The benefit physical exercise for diabetic patient is:

a. Increase glucose blood level

b. Increase insulin sensivity

c. Increase insulin resistance

d. Increase lipid blood level

70. The erderly obesity patient came to the obesity clinic and hopefully to reduce her body weight. She had the
complication such as hypertensi, DM type 2 and waist circumference 90 cm. the major goal for this patient
was:

a. Muscle mass maintenance

b. Reduce risk factor

c. Good quality of life

d. Physical function improvement

71. For patient with congestive heart disease, which of the following statement is accurate:

a. Most of the important resulting from regular exercise is within the myocardium

b. The patient can never expect improve physical fitness

c. Exercise capacity is improve because of peripheral adaptations

d. Complete bed rest is prescribed for these patients

72. The statement below is NOT general recommendations exercise training for post CABG patient:

a. Begin inpatient exercise rehabilitation sooner

b. Progress at amore accelerated rate

c. More attention to upper extremity ROM exercise

d. Resistive exercise should be done at inpatient phase program

73. Exercise testing is most commonly use to evaluate person suspected of having:

a. Coronary artery disease

112
b. Valve disease

c. Cardiomyopathy

d. Pulmonary edema

74. The event of exercise induced asthma is reduced in pool therapy because of the:

a. The buoyancy effect

b. High humidity of air

c. Hydrostatic pressure

d. Viscosity of water

75. Three month after myocardial infraction a patient is interested in starting a reconditioning program. Which
of the following should probably?

a. Progressive walking and jogging

b. Swimming

c. Isometric execise

d. Calisthenics and streaching exercise

76. The forced expiratory volume in one second (FEV1) and the force vital capacity (FVC) are measure using
spirometer. In the restrictive lung disease:

a. Both of FEV1 and FVC are reduced, often in proportion to each other

b. FEV1:FVC ratio reduced (<80%)

c. FEV1 is reduced much more than FVC

d. Flow rate is low in relation to lung volume

77. Following a myocardial infarction a patient asked when he might having sex with his wife, a reasonable
answer would be:

a. Take it easy for three month and ask me about it

b. Having sex after heart attack is dangerous

c. Wait at last six months

d. As soon as you can climb up steps of stair without symtoms you are probably ready

78. The vital capacity is the sum total of:

a. Inspiratory capacity, tidal volume and expiratoty capacity

b. Inspiratory capacity, functional residual capacity

c. Inspiratory reserve volume, tidal volume and expiratory reserve volume

d. Inspiratory reserve volume, functional residual capacity and tidal volume


113
79. In changing from the supine to sitting position, what will happen to the capacity?

a. It will increase up to 1000cc

b. It will increase up to 200cc

c. It will not change

d. It will decrease up to 200cc

80. In patient with obstructive airway who get air trapping would breath:

a. Forcefully inhale for as long as possible

b. Slowly inhale until the lungs are fully expanded

c. Rapidly alternate forcefull inspiration with forcefull expiration

d. Exhale with the lips pursed

81. This factor significantly associated with hip fracture in the elderly:

a. Hip muscle weakness

b. Poor balance

c. Osteoporosis

d. Osteoarthritis

82. Ballistic streaching may be performed provided the following criteria are met:

a. The athlete should be throughtly cooled up

b. It should be performed slowly and in controlled manner, gradually increasing intensity

c. It should be preceded by strengthening

d. It should only be introduced in the advance stages of streaching program

83. The management of most sport injuries is NOT base on:

a. Minimize the extent of initial damage

b. Reduce associated pain and inflammation

c. Limit healing of damaged tissue

d. Restore flexibility, strength, proprioception and overall fitness during the healing phase

84. Squatting is the example of:

a. Open kinetic chain exercise

b. Close kinetic chain exercise

c. Mixed
114
d. None of the above

85. The following is not sign and symptom of ligamentum sprain first degree:

a. Minimal, localized tenderness

b. Subtle structural instability

c. No ligamentum instability

d. Pain and swelling

86. Which of the following activities would be most difficult to return following an ACL injury:

a. Baseball pitching

b. Squestrian activities

c. Cross country skiing

d. Marathon running

87. This disorder usually seen in preadolescent athletes who participate in activities such as jumping or
running. The disorder is a result of recurring micro trauma the quadriceps contracting namely:

a. Tibial plateu fracture

b. Osgood Schlatter disease

c. Patellar tendonitis

d. Pes anserinus bursitis

88. XXX

89. The stance phase of gait include:

a. Acceleration, swing through, deceleration

b. Heel strike, mid stance, push off

c. Push off, deceleration strike, heel strike

d. Acceleration, heel strike, push off

90. Mechanical advantage less than one:

a. Weight arm is shorter than effort arm

b. Weight arm is longer than effort arm

c. Effort is bigger than weight

d. Effort move with the speed faster then weight

115
91. Tilting of pelvis occurs maximally:

a. At mid stance

b. At push off

c. During swing trough

d. At heel strike

92. At heel strike of the ipsilateral limb, the most active muscle is:

a. Gluteus minimus

b. Gluteus medius

c. Gluteus maximus

d. None of the above

93. The 2 major of crutch walking muscle of the shoulder are:

a. Lower pectoralis major and teres minor

b. Pectoralis minor and latissimus dorsi

c. Lower pectoralis major and latissimus dorsi

d. Pectoralis minor and teres minor

94. The muscle hypertrophy and increase in strength is dependent on biochemical factor that is stimulated by
conditioning:

a. Recruitment of neighboring motor units

b. Decreased vascularitation

c. Decreased number of myofibrils

d. Decreased glycogen and protein storage in muscle

95. This statement is not correct statement of changes in biologic functions in response to aging:

a. Increased VO2max

b. Decrease cardiac output

c. Increased systolic blood pressure

d. Increased serum lipid

96. Three months after a nerve lesion the characteristics of MUAPs wil be:

a. The same prior to the lesion

b. High amplitude-long duration

c. Low amplitude-long duration

116
d. High amplitude-short duration

97. EMG biofeedback measures which of the following:

a. Muscle contraction directly

b. Electrical activity associated with muscle contraction

c. The movement ions across the cell membranes

d. The magnitude of a motor nerve depolarization

98. Some factors that influence strength, power and endurance:

a. Length of muscle : longer muscle has more tension

b. Position of muscle insertion to the fulcrum : longer distance to insertion give more force

c. The amount of training : the longer of exercise increase more strength

d. Age : peak performance at the age of 20-25 yo and decline 1% after 25 yo

99. Which type of current does shortwave diathermy use :

a. Alternating

b. Direct

c. Indirect

d. Modified direct

100. Which of the following is NOT a contraindication to microwave diathermy:

a. Brushing

b. Electrical implant

c. Infection skin disease

d. Old scar tissue

101. Which is the best modality for tendon healing:

a. Continuous ultrasound diathermy

b. Pulse ultrasound diathermy

c. Microwave (pulsed) diathermy

d. Shortwave diathermy

102. Modalities could facilitate lymphatic flows is:

a. Ultrasound

b. Electrical stimulation
117
c. Intermittent pneumatic compression

d. Laser

103. Performing seat leg extension is an example of:

a. Open kinetic chain exercise

b. Closed kinetic chain exercise

c. Static kinetic chain exercise

d. Mixed kinetic chain exercise

104. The effect of treadmill training in Parkinson patient is NOT in the following:

a. Decrease fall risk

b. Improve OOL

c. Reduces tremor

d. Improve gait impairment

105. The following statement about DeLorme technique is NOT correct:

a. Use to strengthen muscle

b. Also called as progression resistance exercise

c. Object test to determine his/her 10 RM

d. Object test to determine his/her 1 RM

106. Which type of muscle contraction produces the greatest amount of force generation (torque):

a. Slow contraction

b. Fast contraction

c. Fast eccentric

d. Isometric

107. Neurodevelopmental treatment to normalize tone, inhibit abnormal primitive reflex pattern and facilitate
automatic reaction and subsequent normal development is:

a. Bobath

b. Rood

c. Voyta

d. Ayres

108. Prescription for diabetic shoes from medical rehabilitation specialist SHOULD NOT perform in this
component:

118
a. Diagnostic and goal

b. Correction or modification

c. Material for archieve the goal

d. Shoes number/size

109. Special insole for diabetic shoes should be:

a. Distribution plantar pedis pressure

b. Soft insole without distribution plantar pedis pressure

c. Hard insole

d. Without thermoplastic effect

110. A leprosy wt shows bilateral claw hands. However, all of the hand and finger joints have good ROM and
all the extrinsic muscle of the hand have normal strength. He refered reconstructive surgery. The best
orthotic device to maximize hand function is:

a. Resting hand splint

b. Cock-up splint

c. Knuckle-bender splint

d. Posterior splint

111. Foot wear for MH pts should have NOT this criteria:

a. Wide toe box

b. Rocker sole

c. In sole must distribute pressure

d. High of the heel must no more than 2 inches

112. T-strap is required for CP to control:

a. Genu varum

b. Genu valgum

c. Calcaneus

d. Pes valgus or varum

113. Bracing of LE is NOT useful in the following condition:

a. Weak dorsiflexion of the foot

b. Instability of the knee

c. Athetosis

119
d. Lack of hip control

114. LE bracing will not:

a. Decrease pain

b. Support weight

c. Prevent contracture

d. Correct a scoliosis curve

115. Which of the following is most important characteristics of a LE porsthesis:

a. Good cosmetic

b. Ability to be used for stair climbing

c. Stability and security

d. Minimal energy comsumption

116. What is the purpose of terminal device in functional UE prosthesis?

a. To make a better appearance cosmetically

b. To provide prehension

c. To get done and doff more easily

d. To make a confortable usage

117. The Gaenslen’s test might be positive in the early stage of:

a. OA

b. Ankylosing spondylitis

c. RA

d. Psoriatic arthritis

118. Positif McMurray test shows the probability of rupture of the following structure:

a. Lateral or medial meniscus of knee

b. Medial collateral ligament of knee

c. Anterior cruciate ligament of knee

d. Posterior cruciated ligament of knee

119. A 45 yo pt coming to you because pain of 2 weeks duration on right knee joint. No history of injury, heavy
working or sport activities, except that this last 10 days he frequently has to sit down on the floor (lesehan)
for hours to follow cultural-spiritual ceremonies. On exam of the knee are no abN except pain on palpation

120
and pressure at medial surface of tibial condyle. The most likely of the diagnosis is an inflammation of the
following tendons:

a. Biceps femoris – Gracilis – Semitendinosus

b. Semitendinosus – Semimembran – Gracilis

c. Semimenbran – Gracilis – Sartorius

d. Semitendinosus – Sartorius – Gracilis

120. The steinbroker grading system is used to evaluate severity of:

a. Shoulder-arm syndrome

b. Shoulder-hand syndrome

c. CTS

d. Thoracic Outlet syndrome

121. In the quervain, the pain will increase with: B

1. Grasping

2. Radial deviation of wrist

3. Ulnar deviation of wrist

4. Abduction of thumb

122. Suggested equipment for C7 complete tetraplegia is/are: A

1. Resting hand splint

2. Short opponens splint

3. Utensil cuffs

4. Wrist-driven tenodesis splint

123. The following statement(s) are correct regarding program for a child with haemophilic arthritis:A

1. Prevention of joint deformities and preservation of muscle strength are two equally important aims of
rehab

2. Regardless whether the hemarthrosis is acute, sub acute or chronic, therapeutic exv must follow a slowly
progressive couse of intensity determined to subjective symptoms and clinical findings

3. The child is taught isometric exc to muscle surrounding the affected joints

4. Passive ROM exc should be used in haemophilic arthritis since it doesn’t cause minor trauma with
bleeding

124. The following condition(s) is/are the variation of muscular dystrophy: C

1. Congenital myotonia

121
2. Limb-girdle dystrophy

3. Spinal muscular atrophy

4. Fascioscapulohumeral dystrophy

125. Club foot consist of associated deformity(s): B

1. Forefoot varus

2. Valgus or eversion deformity of heel

3. Equinus or plantar flexion of ankle

4. Forefoot valgus

126. Rehab goal after ACL reconstructon – week 0-4 are: E

1. Control pain to maximize effort in rehab

2. Rapidly reduce effusion

3. Activate quadriceps mechanism

4. Begin closed-kinetic chain exc

127. The main feature(s) of gait in children that differ from adult gait pattern is/are: E

1. The cadence is greater

2. The base is wider

3. The stride length is less

4. The velocity is less

128. An ideal gait cycle will be: A

1. Symmetrical and rhythmic

2. Asymptomatic

3. Metabolically efficient using minimum energy

4. Wider stride length

129. Improvement of muscle performace including strength can be increased by: A

1. Increasing the amount of weight lifted

2. Increasing the contraction velocity

3. Increasing the amount of repetitions

4. Increasing the muscle tone

130. In classical PNF technique the therapist might stretch a muscle to: E
122
1. Facilitate voluntary activity in the muscle

2. Increase the firing rate of afferent from muscle spindle

3. Inhibit voluntary activity in antagonist muscle

4. Focus the pts attention on movement of muscle

131. Which of the following statements about SWD is/are correct: E

1. It is a deep heat modality

2. It is the therapeutic application of radiofrequency electrical current

3. The tissue temperature should be elevated to a range of 40-45 ‘C

4. The treatment time is usually 20-30 minutes

132. Which of the following statement about iontophoresis is/are correct: E

1. The introduction of ion into body tissue by means of a electrical current

2. Iontophoresis used a dicrect electrical current

3. The treatment technique is painless and sterile

4. Much of the penetration probably occurs at sweat glands and site of skin breakdown

133. Which of the following about TENS and opiate pain control theory is/are correct: A

1. Electrical stimulation of sensory nerves may stimulates the release of enkephalin

2. Analgesic effect should last for several hours

3. Electrical stimulation of sensory nerves may stimulate B-endorphine from pituitary gland

4. The current intensity of ES should not be high

134. The pattern of relative heating in US is such that: B

1. A large joint must be heated from all sides

2. US penetrates bone better than soft tissue

3. US heats the deep tissue with relatively little heating of the skin and subcutaneous tissues

4. US heats tissue no deeper the SWD

135. The functions of suprapatellar and supracondylar sockets are: B

1. Suspension device

2. The role at stance phase

3. The role at swing phase

4. Prevent piston phenomena

136. The indication pylon leg of post amputation LE are: E


123
1. Reduce stump pain

2. Make early stump mature

3. Psychologic effect

4. Early mobilization

137. The indication of figure of 8 dressing are: E

1. Prevent stump flabby

2. Make stump conus shape

3. Prevent stump edema

4. Reduce stump pain

138. The potential complication(s) of spinal orthoses are: E

1. Weakening of axial muscles

2. Soft tissue contractures

3. Osteopenia

4. Increased movement of ends of immobilized segments

139. The common deviation(s) seen when observing the AK amputees from the rear is/are: A

1. Lateral trunk bend

2. Abducted gait

3. Circumduction

4. A sudden-heel strike

140. LBP in pregnant woman is/are caused by: E

1. Postural changes

2. Increased in lumbar lordosis

3. Weight gain

4. Ligamentous laxity

141. The following is/are major clinical feature of parkinson’s disease: A

1. Resting tremor

2. Bradykinesia

3. Rigidity

4. Masked face

142. The following factor(s) has/have influence to results of conduction velocity studies: A
124
1. Age

2. Sex

3. Room temperature

4. Skin color

143. The following statement(s) is/are correct regarding the characteristic(s) of normal MUAP: C

1. The shape is mostly polyphasic

2. The amplitude is about 0,5-1,5mVV

3. The frequency is about 100-200Hz

4. The duration is about 5-12msec

144. The following wave(s) can be detected in needle EMG during resting stage: A

1. Fibrillations

2. Positive sharp waves

3. Fascucullations

4. Nascent potential

145. The following statements is/are true: E

1. The mechanism of heat transfer of paraffin bath is conduction

2. The mechanism of heat transfer of whirlpool is convection

3. The mechanism of heat transfer of SWD is conversion

4. The mechanism of heat transfer of US is radiation

146. Which the following statement(s) about infrared is/are correct: B

1. Radiant energy for superficial heating

2. Non luminous type is deeper penetration than luminous type

3. 2 types near infrared and far infrared

4. The wave length are 770-120

147. Which the following statement(s) about TENS is/are correct: E

1. TENS will evoke the gate control mechanism and diminish awareness of painful stimuli

2. As long as the stimulation is causing firing of the sensory nerve, the gate to pain should be closed

3. Current intensity should be adjusted to tolerance but should not cause a muscular contraction

4. Pulse per second should be 8 125 or as high as possible on the machine

148. PNF program typically include: E


125
1. Deep cooling of muscle muscle

2. Muscle stretching

3. Whitdrawal reflexes

4. Superficial cooling

149. Which of the following about TENS and opiate pain control theory is/are correct: A

1. Electrical stimulation of sensory nerves may stimulates the release of enkephalin

2. Analgesic effect should last for several hours

3. Electrical stimulation of sensory nerves may stimulate B-endorphine from pituitary gland

4. The current intensity of electrical stimulation should not be high

150. The pattern of relative heating in US is such that: A

1. A large joint must be heated from all sides

2. US penetrates bone better than soft tissue

3. US heats the deep tissue with relatively little heating of the skin and subcutaneous tissues

4. US heats tissue no deeper the depth achieved with SWD

126
NATIONAL BOARD EXAMINATION PHYSICAL MEDICINE AND REHABILITATION

DECEMBER 2011
CHOOSE THE MOST APPROPIATE ANSWER
1. The primary pathology of Rheumatoid Arthritis is in the following structure: D
a. Joint capsule
b. Joint cartilage
c. Joint ligament
d. Synovial membrane
2. The muscle/tendons most commonly involved in rotator cuff tendinitis is: C
a. Subscapularis
b. Infraspinatus
c. Supraspinatus
d. Biceps brachii
3. In the early stage of a patient with Ankylosing Spondylitis the following test will be positive A
a. Gaenslen
b. Laseque
c. Thomas
d. Ober
4. What are the most reliable physical signs of Myofascial Trigger Point Syndrome? A
a. A rope-like nodulary in the muscle and pattern of referred pain
b. Focal tenderness and pain recognition
c. Vasomotor and temperature changes
d. Limited of ROM
5. Using a computer mouse that is located too far to the side or too high on the table will lead to: C
a. Anterior shoulder subluxation
b. Supraspinatus nerve palsy
c. Rotator cuff tendinitis
d. Low back pain
6. Which of the following is a general treatment principle for knee osteoarthritis? C
a. Encourage a high impact aerobic home exercise program
b. Avoid extreme range of motion exercises
c. Strengthen the quadriceps muscle
d. Avoid use of modalities
7. When you see a patient every time he is walking always holding his thigh during the stance phase, most
probably he has lesion of the following nerve: C
a. Tibial
b. Sciatic
c. Femoral
d. Common peroneal
8. The eye blindness among the leprosy patients is caused by the damage of the following cranial nerve: C
a. Facial and optic
b. Facial and abduscens
c. Facial and trigeminal
d. Facial and Occulomotor
9. The following measure is NOT correct regarding the management of the insensitive feet of the leprosy
patient: D
a. Soaking the feet everyday
b. Apply oily substance after the soaking
c. Trim and smoothen the sharp edges of the soles
d. Ask the patient always to wear leather shoes when walking

127
10. If somebody has difficulties in chewing the food most probably he has problem with the following nerve:
C
a. Facial nerve
b. Trigeminal nerve
c. Hypoglossus nerve (KUNCI)
d. Glossopharyngeal nerve
11. Lateral winging of the scapula is due to the lesion of the following nerve D
a. Accessory nerve
b. Subscapular nerve
c. Suprascapular nerve
d. Long thoracic nerve (KUNCI)
12. Which of the following physical examination findings would be most consistent with a C6 radiculopathy D
a. Triceps weakness
b. Deltoid weakness
c. Medial brachial sensory loss
d. Decreased brachioradialis reflex
13. Management of pes planus grade I at age less than 1 years old is done by A
a. Observation
b. Thomas heel
c. Arches support
d. Chukka type shoes
14. In Klumke’s type of brachial plexus injury the following muscle is spared D
a. Flexor pollicis longus
b. Flexor digitorum sublimis
c. Flexor digitorum profundus
d. Extensor carpi radialis longus
15. You are examining a child of 7 to 8 months of age. Persistence of which one of the following primitive
reflexes or postural responses would you be concerned about? D
a. Placing
b. Landau
c. Parachute
d. Asymmetric tonic neck
16. Arthogryposis multiplex congenital most commonly is: A
a. Neurogenic in origin
b. Myopathic in origin
c. Associated with early joint fusion
d. Associated with mental retardation
17. The ortolani test will be positive in the following condition: D
a. Congenital lower limb deficiency
b. Congenital talipes equinovarus
c. Congenital hemisacralization
d. Congenital hip dislocation
18. An indication of good prognosis for independent ambulation for cerebral palsy patient if he/she is able to
do the following: C (cucur 789)
a. Prone lying at the age of 15 months
b. Sitting with support at the age 24 month
c. Sitting without support at the age 24 months
d. Kneeling with assistance at the age 16 months
19. The major goal of phasa I Cardiac Rehabilitation is to: C
a. Stimulate the development of coronary collateral
b. Stimulate of myocardial exercise tolerance
c. Prevent deconditioning
d. Increase cardiac output
128
20. Target heart rate that is recommended for cardiac insufficient patients during exercise is: C
a. 50% of maximal heart rate
b. 50-70% of maximal heart rate
c. 70-85% of maximal heart rate (KUNCI)  SEHAT ?
d. >85% of maximal heart rate
21. The activity of climbing up the stairs requires the following amount of METs: C
a. 2 Mets
b. 4 Mets
c. 6 Mets
d. 8 Mets
22. Preload of the left ventricles refers to: B
a. Stroke volume
b. Diastolic filling
c. Ejection fraction
d. Ventricular outflow
23. The respiratory function of patient with Ankylosing Spondylitis is characterized by: A
a. Diminished chest expansion
b. Moderate dyspnea on exertion
c. Increase use of accessory muscle
d. Decrease in diaphragmatic excursion
24. For chronic lung disease various walking test with sub maximal exercise have all the benefit below,
EXCEPT: B
a. To detect exercise intolerance
b. To detect limitation in lung function
c. To know and detect functional limitation
d. To evaluate reconditioning exercise programme
25. The following condition is NOT a contraindication for exercise program prescribed to a diabetic patient
with cardiovascular disease: B
a. Blood sugar more than 300 mg%
b. Blood sugar about 180-250 mg%
c. Blood sugar less than 80 mg%
d. Ketosis
26. Which type of exercise training is widely accepted as the optimal way to enhanced and maintain function
in older adults? D
a. Endurance
b. Resistance
c. Flexibility
d. Balance
27. The following sport activity is NOT recommended in an old osteoporosis women: B
a. Tai Chi
b. Jogging
c. Walking
d. Light gymnastics
28. A good initial treatment plan for neurogenic claudication secondary to spinal stenosis would include: A
a. Gabapentin
b. Acetaminophen
c. Lumbar bracing
d. Narcotic pain medication
29. Which activity is NOT associated with a reduced risk of dementia? B
a. Dancing
b. Bowling
c. Playing board games
d. Playing musical instrument
129
30. This disorder is usually seen in preadolescent athletes who participate in activities such as jumping or
running. The disorder is a result of recurring micro trauma from the quadriceps contacting: D
a. Ligament sprain
b. Pes anserinus bursitis
c. Tibial plateau fracture
d. Osgood-schlatter disease
31. Ankle sprains are ubiquitous is sports and are the most common athletic injury. The most commonly injury
for this disorder is: B
a. The calcaneo fibular ligament
b. The anterior talofibular ligament
c. The posterior talofibular ligament
d. The strong medial deltoid ligament
32. Which of the following activities would be the most difficult to return to following an anterior cruciate
ligament (ACL) injury? B
a. Speed cycling
b. Baseball pitching
c. Marathon running
d. Cross-country skiing
33. Which the physiologic factor in the elderly exacerbates orthostasis? D
a. Decreased in arterial stiffness
b. Decreased creatinine clearance
c. Decreased peripheral resistance
d. Decreased baroreceptor response
34. The knee joint is a: A
a. Trochogynglymus joint
b. Gynglimus joint
c. Trochoid joint
d. Pivot joint
35. A patients is always complaining of difficulty in climbing the stairs. Most probably this is caused by the
weakness of the following muscle: D
a. Pyriformis
b. Gluteus medius
c. Gluteus maximus
d. Quadriceps femoris
36. In regards to movement of the scapula, the serratus anterior does which of the following: C
a. Rotates the glenoid upward with shoulder abduction
b. Rotates the glenoid doenward with shoulder abduction
c. Rotates the glenoid upward with the shoulder forward flexion (KUNCI)
d. Elevates the vertebral borders of the scapula away from the chest wall
37. The following is NOT true regarding patient with COPD: C
a. Pulmonary rehabilitation program is a central therapeutic regimen for these patient
b. COPD patient need cardiac risk evaluation before beginning an exercise program
c. Patient with COPD require higher levels of exercise training to gain benefit because of the
severity of their disease
d. Exercise tolerance improve from exercise because of gains in aerobic fitness, respiratory muscle
function, and breathing pattern
38. Children with Tetralogy of Fallot with assume the squatting position to relieve exercise-induced dyspnea C
a. Increase inspiratory capacity
b. Decrease pulmonary artery pressure
c. Increase peripheral vascular resistance and there by decreases right to left shunt
d. Reduces the energy requirement of the activity by lowering the center of gravity
39. The following intrinsic muscle of the hand that has dual innervation B
a. Abductor pollicis brevis
130
b. Flexor pollicis brevis
c. Opponens pollicis
d. Adductor pollicis
40. Maximal flexion of the knee in the gait cycle is reached: C
a. In mid stance
b. Immediately after the heel strike
c. In the middle of swing phase
d. During acceleration in swing phase
41. Double limb stance is what percent of the entire gait cycle? C
a. 5%
b. 10%
c. 20%
d. 30%
42. The most common method of exercise based on the neuro development approach for cerebral palsy patient
is the: D
a. Voyta
b. Kabat
c. Phelps
d. Bobath
43. The following is NOT the characteristic of cerebral palsy: A cucur 782
a. The underlying pathologic process is still active
b. The spastic type is the most common clinical picture
c. It is caused by non progressive patology affecting the immature brain
d. The clinical manifestation include the abnormalities in movement and posture
44. Conditioning exercise program is NOT result in the following: A
a. Decrease in metabolic activity of muscle
b. Increase in stroke volume
c. Increase in venous return
d. Decrease in pulse rate
45. The following is NOT used for lymphedema: D
a. Compression garment
b. Elevation
c. Massage
d. Icing
46. The following statement is NOT correct regarding with the coordination exercise: D
a. To apply sensory input
b. To accelerate the speed as tolerable
c. To perform constant repetitive movements
d. To strengthen progressively the particular muscles
47. As the left hemiplegic patient shows marked spasticity, semi voluntary and volitional initiated movement
of involved limbs, resulting as well as flexion synergy in arm and extension synergy in legs, it means that
the patient now is in Brunnstrorm stage: B
a. 2
b. 3
c. 4
d. 5
48. The mechanism of diazepam to reduce the muscle spasticity is by: A
a. Stimulating the GABA mediated inhibition in CNS
b. Inhibiting the extrafusal fiber inhibition in CNS
c. Inhibiting Glutanmate and aspartate in CNS
d. Inhibiting the motor cortex
49. The following is NOT the characteristic of the Barthel Index: C
a. Prediction of rehabilitation outcome
131
b. Good interrater reliability for all subcategories
c. Assessment of progress in language and cognitive skills
d. Assignment of numerical scores and relative weights to ADL variables
50. A young healthy man 30 years old suffered acute back pain after lifting a very heavy box.
Coughing/sneezing induce the pain which radiates to the left leg & foot, however no abnormality detected
in bladder or bowel function. He is still able to walk with an antalgic gait.
Clinically the patient is suspect suffering from: A
a. Herniated intervertebral disc
b. Tumor of the spine
c. Osteoarthritis
d. Osteoporosis
51. D
52. B
53. D
54. B
55. D
56. B
57. A patient with DM has painful, swollen ankle, worst with weight bearing, X-ray shows fragmentation of
the articular surface, synovial fluid is clear. The choice of orthosis is: C
a. AFO
b. HKAFO
c. PTB orthosis
d. Orthopedic shoes
58. In PTB socket the patient’s stump in held in how many degree knee flexion? A
a. 15 degree
b. 30 degree
c. 45 degree
d. 60 degree
59. A girl patient with idiopathic scoliosis the following is NOT the signs that the Cobb’s angle might be
increasing rapidly: D
a. Her public hairs are growing
b. Her breasts are growing bigger
c. She has her menarch a month ago
d. The Risser’s sign is reaching to grade 4
60. Rehabilitation guidelines following total knee arthroplasty include: C
a. Use of a continues passive motion device to improve passive knee extension
b. Joint immobilization until sutures are removed and anterior incision is healed
c. Patients are usually allowed to resume driving 6 weeks after surgery
d. Non-weight-bearing status for 1 week following surgery
61. According to the American Collage of Rheumatology guidelines, which agent is recommended as first-line
medication in OA of the hip? C
a. Viscosupplementation
b. Oral gold treatments
c. Acetaminophen
d. Prednisone
62. Intermittent compression pumps is contraindicated in which of the following condition: C
a. Pstmastectomy lymphedema
b. Venous insufficiency
c. Arterial insufficiency
d. Amputation
63. When treating supraspinatus tendonitis with USD, which shoulder position is most beneficial? A
a. Arm abducted and internally rotated
b. Arm abducted and externally rotated
132
c. Arm adducted and externally rotated
d. Position of comfort
64. The original gate theory of Milzack and Wall explained the pain modulation phenomenon in which of the
following ways? D
a. Postsynaptic inhibition of pain transmission occurs via posterior column fibers
b. Stimulation of thermoreceptors causes a decrease in the conduction rate of unmyelinated C fibers
c. Large A myelinated fibers enter the spinal cord through a circuitous anterior route to cause presynaptic
inhibition of pain neurons
d. Neurons within the substantia gelatinosa cause presynaptic inhibition of first-order neurons to
second-order pain afferents in the dorsal horn
65. ASIA Impairment scale B is:C
a. Incomplete, motor function is preserved below the neurological level, and more than half of key
muscle below the neurological level have a muscle grade less than 3
b. Incomplete, motor function is preserver below the neurological level, and at least half of key muscle
below the neurological level have a muscle grade greater than or equal to 3 less
c. Incomplete, sensory but no motor function is preserved below the neurological level and includes
the sacral segments S4-5
d. Complete, no sensory or motor function is preserved in the sacral segments S4-S5
66. A 21 year old man is evaluated in your spinal cord injury clinic 12 months after a C2 complete spinal cord
injury requiring full-time mechanical ventilation. You recommend D
a. Agresive diaphragmatic strengthening exercise
b. Avoiding a breath control system for his power wheelchair
c. Initiating a weaning protocol by slowly decreasing tidal volume
d. An electrodiagnostic study to evaluate for a phrenic nerve pacemaker
67. What is the highest neurologic level at which a person with complete spinal cord injury might be capable
of unassisted, independent transfer skills with or without equipment? C
a. C2
b. C4
c. C6
d. C8
68. The smallest unit of muscle contraction is C
a. Motor unit
b. Sarcomere
c. Actin, myosin
d. Intrafusal fibers
69. The following is NOT correct regarding the purpose of EMG/NCV examination: D
a. To know the function of motor unit
b. To know the nerve conduction velocity
c. To know the level of motor unit pathology
d. To know the etiology of motor unit pathology
70. C6 quadriplegic can be expected to be able to : D
a.. pick up the heavy object without a splint
b. Push up while sitting in a wheelchair
c. Ambulate with crutches and braces
d. dress independently
71. The intermittent catheterization in paraplegic patients is the method of choice for bladder management
because it is: D
a. Very affordable now by most of the SCI patients
b. Very simple method now in modern medicine
c. Very much accepted by most of the patients
d. Very good to minimize the infection
72. Which of the following statements is TRUE regarding the blood supply to the spinal cord? A
a. The mind-thoracic region is most vulnerable to ischemia
133
b. The two anterior spinal arteries are branches of the vertebral artery
c. The great spinal artery of Adamkiewick supplies the lumbar and sacral cord
d. The watershed zone is at C6-7 which causes most cervical lesion to be complete
73. The annulospinal ending is the primary afferent fiber coming from the: A
a. Nuclear bag
b. Nuclear chain
c. Intrafusal fibers
d. Extrafusal fibers
74. The following factors is NOT influence the strength of muscle contraction: A
a. The muscle volume
b. The amount of calcium and ATP
c. The amount of action and myosin
d. The number of motor units which are active
75. A child with congenital transverse radial limb deficiency should have in initial prosthesis fit at what
development stage? A
a. At the time of first sitting independently
b. At the time of starting kinder garden
c. At soon as possible after birth
d. At the time of initially walking
76. The following is NOT the characteristic of Duchenne’s Muscular Dystrophy D
a. It is transmitted by X linked mode of inheritance
b. Measurement of serum creatinine kinase is the most reliable test
c. The child’s mother, his sisters and maternal aunts are potential carriers
d. Electromyogram shows high amplitude, long duration motor unit action potentials with decrease
recruitment pattern on effort
77. The following primitive reflex is NOT positive in a healthy baby of 8 months old: A
a. Palmar grasp
b. Plantar grasp
c. Placing reflex
d. Parachute reflex
78. An indication of good prognosis for independent ambulation for Cerebral Palsy patient if he/she is able to
do the following : A
a. Sitting without support at the age 24 month
b. Kneeling wit assistance at 16 month old
c. Sitting with support at the age 24 months
d. Prone lying at the age of 15 months
79. The definition of chronaxy is: D
a. Half the rheobase
b. Time a twice the rheobase
c. The minimal deviation of current to cause a minimal contraction
d. The minimum time requires to obtain a minimal muscle contraction by a stimulus
80. In SWD treatment if the electrodes are place side by side on the same aspect of the body, provided they
have adequate distance, is called A
a. Coplanar method
b. Cross-fire method
c. Monoplanar method
d. Contraplanar method
81. The following is NOT indication of biofeedback therapeutic: B
a. To inhibit spasticity
b. To strengthen muscle
c. Muscle reduction in upper motor neuron dysfunction
d. Pelvic floor muscle reduction in women with incontinence or pelvic pain
82. For patient with Congestive Heart Disease, which of the following statements is accurate? B
134
a. Most of the important resulting from regular exercise is within the myocardium
b. Exercise capacity is improved because of peripheral adaptation
c. These patient can never expect improved physical fitness
d. Complete bed rest is prescribed for these patients
83. The proper emergency response for patient who has experienced a cardiac arrest, but who is now breathing
and has a palpable pulse, includes: A
a. Placing the patient in the recovery position with the head side to prevent airway obstruction
b. Continuing the exercise test to determine why the patient had this response
c. Placing the patient in a comfortable seated position
d. Placing the patient in a head down position
84. Which of the following risk factors is most associated with a higher incidence of Deep Vein Thrombosis
(DVT) in adult with a stroke? D
a. Male sex
b. Increased age
c. Smoking history
d. Increased motor weakness
85. Dressing the upper part of the body in a patient with one-side non-functional upper extremity implies that:
A
a. The involved extremity should be dressed first
b. The garment should be pulled over the head first
c. The uninvolved extremity should be dressed first
d. Only garments with complete opening in front should be used
86. Exercise program in adult patients with hereditary neuromuscular disorders: D
a. Should use high resistance eccentric exercise
b. Are most effective when weakness is rapidly progressive
c. Are ineffective if muscle strength is less than 75% of normal
d. May increase maximal aerobic capacity and exercise endurance
87. Which of the following is true regarding pain in the elderly person? C
a. Pain can commonly be localized to a single site
b. Elderly persons do not feel pain-as-much as younger people
c. Approximately one third of elderly individuals have chronic joint pain and arthritis
d. Pain result in less functional impairment in the elderly compared with the younger population
88. The interdisciplinary approach to geriatric patient care emphasizes D
a. Team communication when problem occur
b. Concentration on specific clinical problem
c. The physician as the leader and director of the team
d. Common patient and team goals rather than discipline-specific goal
89. A 55-year old woman with breast cancer has a solitary metastatic lesion at the left rib. Among the
following, the first-line pain medication in this setting is: A
a. Ibuprofen (KUNCI)
b. Amiptriptyline
c. Acetaminophen
d. Morphinesulfate
90. A C7 quadriplegic patient should NOT be expected to perform the following activities independently: C
a. Sliding board transfer
b. Wheelchair-to-car transfer
c. Floor-to-wheelchair transfer
d. Wheelchair-to-tub bench transfer
91. A 35 year-old jogger experiences new-onset a traumatic pain along the lateral aspect of the knee. Pain is
reproduced and accentuated when the examiner presses proximal on the joint line and simultaneously bring
the knee in and out of 20 to 30 knee flexion. The most likely diagnosis is: C
a. Lateral plica syndrome
b. Lateral meniscus strain
135
c. Illiotibial band syndrome
d. Distal femur stress fracture
92. The most likely combination of musculoskeletal imbalance associated with the above diagnosis is: A
a. Weak gluteus medius/tight tensor fascia lata
b. Weak gastroc-soleus/tight posterior tibialis
c. Weak lower abdominals/tight psoas
d. Weak quadriceps/tight hamstring
93. The mechanism of Botulinium A toxin (Botox) in the treatment of spasticity is by C
a. Blocking muscle spindle
b. Blocking the nerve fibers
c. Blocking the release of Ach
d. Blocking the release of Ca ions
94. A pulmonary embolism which may occur in a prolonged immobilization patient mostly due to: D
a. Development of atrial fibrillation
b. Decreasing the total lung capacity
c. Decreasing of the lung vital capacity
d. Development of deep venous thrombosis
95. A 12-year-old baseball pitcher presents with new-onset medial epicondylar pain A typical physical exam
finding on the throwing arm side in early ”little league elbow” is: A
a. Loss of terminal elbow extension
b. Decreased biceps brachii strength
c. Increased shoulder internal rotation
d. Decreased sensation to light touch in an ulnar nerve distribution
96. A 29-year-old cross-country runner experiences new-onset pain in the region of the first metatarsal. Exam
reveals tenderness medial to the first metatarsal head worsened by passive great toe extension. The LEAST
likely cause of her sesamoid injury is: C
a. Early hallux rigidus
b. Excessive pronation
c. Excessive supination
d. Gastroc-soleus tightness
97. A 55-year-old runner present with presents with pain in the region of the Achilles tendon. This is the third
such episode over the past 4 years. The Achilles tendon appears swollen. Appropriate initial management
would include: C
a. Cortisone injection of the Achilles tendon
b. Local anesthetic injection of the Achilles tendon sheath
c. Use of a small heel lift with initiation of gastro-soleus stretches as tolerated
d. Immobilization in an ankle plantar-flexed position with limited weight bearing
98. In hemiplegic patient, the function of a double upright AFO with posterior stop, dual action adjustable
ankle joint and extended steel shank is for: D
a. To help the weak plantar flexors
b. To correct the knee stability during heel strike
c. To improve the medial and lateral stability of the ankle
d. To overcome the moderate spasticity of the plantar flexors
99. A patient with complete C5 injury with only 2/5 (grade 2 from MMT) deltoid strength, and nothing else,
will benefit more or best from: B
a. Wrist driven flexor hinge wrist hand orthosis
b. Balanced forearm orthosis and wrist hand orthosis
c. Cable driven flexor hinge orthosis with figure of eight harness
d. Spring operated ratches passive prehension wrist hand orthosis
100.The most appropriate wheelchair for a T 1 Paraplegia AIS/Frankel A is: C
a. Regular wheelchair
b. Mouth-operated wheelchair
c. Regular wheelchair with higher backrest
136
d. Wheelchair with vertical bars on the handrims

ANSWER: A. IF NUMBER 1,2,3 ARE CORRECT


B. IF NUMBER 1,3 ARE CORRECT
C. IF NUMBER 2,4 ARE CORRECT
D. IF ONLY NUMBER 4 IS CORRECT
E. IF ALL NUMBERS ARE CORRECT
101.The following deformities are typical in an advance case of rheumatoid arthritis: A
1. Boutonierre
2. Piano-key sign
3. Ulnar deviation
4. DIP joint subluxation
102.The following condition could lead to Thoracic Outlet Syndrome: A
1. Cervical rib
2. Fracture of the clavicle
3. Hypertrophy of the M Scalenus anterior
4. Hypertrophy of the M Pectoralis mayor
103.Which is/are predisposing factors for bursitis of the hip? E
1. Hip trauma
2. Hemiparesis
3. Low back pain
4. Leg length discrepancy
104.Low Back Pain which is common among pregnant women is/are caused by: E
1. Increased in lumbar lordosis
2. Ligamentous laxity
3. Postural changes
4. Weight gain
105.The following clinical manifestation is very frequent occurred in a stroke affecting in the right brain
hemisphere C
1. Right sides neglect
2. Visual memory deficits
3. Wernicke’s aphasia
4. Superficial and deep sensory deficits
106.A stroke involving the vertebra-basilar system may give the following manifestation (s): B
1. Ataxia
2. Cortical blindness
3. Hemiplegia alternans facialis
4. Unhibited neurogenic bladder
107.The most common behavioral and personality changes seen following Traumatic Brain Injury are: E
1. Attention deficit rigidity
2. Visual perceptual deficit

137
3. Spatial dysfunction
4. Memory deficit
108.The purpose of bracing in CP patient is: E
1. To control movements that interfere function
2. To promote better balance
3. To prevent contractures
4. To correct contractures
109.Congenital muscular torticollis caused by: B
1. Deformation from intrauterine positioning
2. Hemivertebrae
3. Fibrotic contracture of the stenocleidomastoid
4. Ipsilateral plagiocephaly
110.The following is/are the common cause for”floppiness”at birt E
1. Cerebral dysfunction
2. Prader Willi syndrome
3. Spinal muscular atrophy type I
4. Congenital myotonic dystrophy
111.Exercise for patient with cardiac disease should be discontinued in the following condition if: A
1. Systolic BP decreased 20 mm Hg
2. Chest pain
3. Dyspnea
4. Joint stiffness
112.The following of the indication of postural draining as a form of chest physical therapy to enhance the flow
of mucus out of airways, except E
1. Emphysema
2. Bronchiectasis
3. Chronic Bronchitis
4. Polycystic lung disease
113.Among the following which activity might be especially stressful for the patient with cardiac disease? D
1. Leg raises
2. Walking (2.0 mph)
3. Watching tv
4. Bridging exercise on a mat
114.Effect of exercise in the elderly, include: A
1. Improved cardiopulmonary function
2. Maintained or increased bone mass
3. Improved mobility and function
4. Decrease orthostatic tolerance
115. The following is/are the gold standard for diagnosis of osteoporosis: D
1. US densitometry
2. CT scan
138
3. MRI
4. DEXA
116.The following is/are the management in the acute state of sport injury: A
1. Icing
2. Elevation
3. Bandaging
4. Passive ROM
117.Some of the gait characteristics of normal aging include: E
1. Relatively move time is spent in stance phase
2. Slower walking speed
3. Shorter stride length
4. Wider base of gait
118. The ankle mortis: A
1. Consist of tibia, fibula and talus
2. The talus function as a hinge joint
3. Its motion is 200 dorsiflexion
4. Consist of talus and calcaneus
119.Muscle tension is/are monitored by D
1. Muscle spindle
2. Motor endplate
3. Extrafusal fibers
4. Golgi tendon organ
120.The following is/are correct regarding the muscle spindle: E
1. It is innervated by the gamma motorneuron
2. Its main function is to monitor the muscle tone
3. It will be stimulated if there s sudden elongation of the muscle
4. The fastest primary afferent are coming from the nuclear bag
121.The effect of UV radiation are: B
1. Biological
2. Reduce of muscle spasm
3. Photochemical
4. Increase the pain threshold
122.The following is/are good characteristic(s) of an orthopedic shoes: C
1. Soft shank
2. High toe box
3. Very high heel
4. Material : genuine leather
123.The SACH foot has the following characteristic(s): C
1. No “toe break”
2. Solid ankle joint
3. Very expensive price
139
4. Quite good functionally
124. The prescription of exercise for the patient with simple recurrent low back pain should achieve at least the
following: E
1. Improved low back flexibility
2. Improved posture with minimized lumbar lordosis
3. Improved body mechanics in all activities and exercise
4. Improved strength of abdominal and hip extensor muscle
125.A circumducted gait pattern in a patient with an above-knee amputation is/are likely caused by: D ??
1. Madequate friction in the prosthetic knee unit
2. An externally rotated prosthetic knee unit
3. A silesian suspension belt that too tight
4. In adequate socket suspension
126.The following is/are the part of William’s exercise for patient with LPB: C
1. Hip flexor muscles strengthening
2. Abdominal muscles strengthening
3. Hamstring muscles strengthening
4. Hamstring muscles stretching
127.The benefit(s) of physical activies in the elderly include(s) the following: A
1. Slowing of disease progression
2. Promotion of the person’s health
3. Prologation of functional independence
4. Prevention of aortic aneurysm rupture
128.Reflection of ultrasound E
1. Occurs mainly at interfaces of tissues of with different acoustic implants
2. Is great at the interface of soft tissue and metallic implants
3. Is minimal at interfaces of layers of soft tissue
4. Is great at interfaces of soft tissue and bone
129.The following(is) is/are true about iontophoresis: A
1. Medications used for iontophoresis must be soluble in water
2. Iontopherosis should not be used in patient with pacemaker
3. Hot and cold packs should not be applied during treatment
4. Tissues penetration of medication may reach up to 2 cm
130.Difficulty in performing opposition of the thumb against the index and third fingers can caused the
weakness of the following muscle A
1. Abductor pollicis brevis
2. The first lumbrical
3. Opponens pollicis
4. Adductor pollicis
131.The following waves is/are considered abnormal in needle EMG examination: A
1. Positive sharp waves
2. Fasciculation
140
3. Fibrillation
4. Biphasic waves
A basketball athlete had injured after he jump and landed on his right foot in the internally rotated position. He
heard ”pop” and felt painful on his right knee. He ceased from this game because he cannot walk and got an edema
suddenly.
132. What is/are possible diagnoses for his condition? B
1. ACL rupture
2. PCL rupture
3. Meniscal torn
4. Quadriceps muscle rupture

133.What is special test/sign will positive for his condition? C


1. Human test
2. Mc Murray test
3. Sagging knee
4. Anterior drawer test
134.The following is/are the test(s) to diagnose the knee meniscal tear: C
1. Lachman test
2. Mc Murray’s test
3. Pivot shift test
4. Apley’s comprehension test
135.The management of acute anterior cruciate ligament injury is/are? B
1. Rest
2. Massage
3. Cryotherapy
4. POP immobilization
136.Risk factor for falls in elderly people is/are: E
1. Environment related
2. Use of sedative hypnotics
3. Inappropiate use of gait devices
4. Weakness, balance, or joint problem
137. Changes in biological functions in response to aging is/are: B
1. Decrease VO2max
2. Decrease serum lipid
3. Increase thrombosis
4. Increase cardiac output
138.In prescribing pain treatment for elderly patient, it is best to use the following approach(s): B
1. Keep on guard against the side effect
2. Start with aspirin, which is safe and effective
3. Start low and go sow for all medicine
4. Use high does short-acting narcotic initially
141
139. The benefit of postural drainage for patient with COPD is/are of the following: B
1. Decrease work of breathing
2. Increase work of breathing
3. Improve ventilation
4. Decrease ventilation
140.The aim(s) of chest physical therapy in asthmatic patient is/are: E
1. To relieve bronchospasm
2. To aids removal of secrets
3. To coordinate respiratory movement
4. To assist relaxation and gain control of breathing
141.The following is/are the contraindication(s) of postural drainage: B
1. Severe hypertention
2. Cystic fibrosis
3. Hemoptysis
4. Fixed contractures of the foot
142. For the child with cerebal palsy, the orthosis should be used on the lower extremities when the child : A
1. Non ambulatory but tends to keep the leg in extension and the feet in plantar plexion
2. Begins to pull to a stand on toes
3. Stands flat but walks on the toes
4. Fixed contractures of the foot
143.The following is/are the early treatment(s) of CTEV D
1. Dennis-Brown splint
2. Outflare orthopedic shoes
3. Surgical soft tissue release
4. Stretching and serial plaster correction
144.Leg length discrepancy in children can result from several disorder. A disorder that would result in a
discrepancy by lengthening the extremity would be: A
1. Poliomyelitis
2. Legg-Perthes disease
3. A growth plate injury
4. A vascular malformation
145.In the left hemiplegic patient who his or her left hand is edematous and his or her left shoulder is painful
requires: E
1. A short course of 30 mg prednisone daily
2. To elevate his/her left upper extremity
3. A low dose of amitriptyline
4. TENS
146. The statements below is/are the features of Wallenberg’s syndrome : A
1. Ipsilateral ataxia
2. Horner’s syndrome
3. Contralateral loss of pain and temperature
142
4. Ipsilateral loss of pain and temperature
147.Some physical findings below must be assessed in patients with neurogenic bladder E
1. Sacral sensation
2. Cognitive impairements are an important test
3. Hand function for SCI patients who are the perform self catherization
4. Bulbocavernosus reflex to test in the integrity of the pundendal nerve and S2-S4 segments
148. The following is/are predictor(s) for poor prognosis of Bell’s Palsy: E
1. Dryness of the eyes
2. Diabetes mellitus
3. Hypertension
4. Pregnancy
149.What is the primary role for methotrexate in the treatment of rheumatoid arthritis? D
1. To cure the disease
2. To reserve already present joint deformity
3. To provide analgesia and anti inflammation
4. To slow or prevent joint destruction and loss of function
150.The following is/are the first line treatments of plantar fasciitis: A
1. Heel cup to cushion and lift the heel
2. Anti-inflammatory agents
3. Stretching program
4. Lidocaine injection

143
NATIONAL BOARD EXAMINATION
PHYSICAL MEDICINE AND REHABILITATION

JUNE 22, 2012

CHOOSE THE MOST APPROCIATE ANSWER


1. The following statement is NOT the appropiate characteristic of spastic diplegia Cerebral Palsy C
A. Children who did not sit by 4 year have not achieved ambulation
B. Sitting by 2 year was a good predictive sign of eventual ambulation
C. Children with spastic diplegia have not adequate movement control of the arms and hands
D. Persistent obligatory infantile reflex activity beyond 18 months implied poor prognosis for ambulation
2. Communication is very important in human and takes many forms especially in children. The approciate skills
of communication in a child 2 year old: D
A. A look
B. A gesture
C. The first word
D. Two-words phrases
3. What kind of foot deformities in children with S1-S2 lesion: D
A. Imbalance between intrinsic and extrinsic muscles
B. Plantigrade ulceration
C. Calaneal foot
D. Cavus foot
4. A 12 year-old boy with haemophilia had recurrent hemarthorosis of the knee in the past but now has full range
of motion and equal sized quadriceps muscles measurement. His parents ask advice regarding approciate
athletic activities. You advice for sports but advice against: A
A. Competitive basketball
B. Competitive swimming
C. Competitive golf
D. Biking
5. Protective and postural reflexes (advanced pastural reactions) is: C
A. Plantar grasp
B. Foot hand placement
C. Head and body righting
D. Symetric tonic neck reflex
6. Which of the following describes “scapulo humeral rhythm” A
A. 2/3 glenohumeral and 1/3 scapulothorasic
B. 1/3 glenohumeral and 2/3 scapulothorasic
C. 1/2 glenohumeral and 1/2 scapulothorasic
D. 1/4 glenohumeral and 3/4 scapulothorasic
7. The most function gluteus maximus during gait when A
A. Loading response
B. Terminal stance
C. Mid stance
D. Pre swing
8. The normal degere of femoral neck inclination in adult is: B
A. 100 degrees
B. 125 degrees
C. 150 degrees
D. 175 degrees
9. A 45 year-old woman present with pain in the region of the Achilles tendon. This is the third episode over the
past 2 years. The Achilles tendon appears swollen. Approciate initial management would include: D

144
A. Cortison onjection of the Achilles sheath
B. Prescription of a custom-molded AFO to reduce pronation
C. Immobilization in an ankle plantar-flexed position
D. Use of a small heel lift with initiation of gastroc-soleus stretches as tolerated
10. A male patient, 30 year-old, no history of regular sport activities, comes with a chief complaint of pain on the
right knee joint around the patellar tendon insertion after play football. This worst when he runs or climbs the
stairs. The most probable diagnosis is: A
A. Osgood-Schlatter disease
B. Chondromalacia patella
C. Ligament sprain
D. Meniscal tear
11. American College of Rheumatology Classification criteria for Systemic Lupus Erythematosus, is NOT include
in the following: A
A. A Raynaud’s phenomenon
B. Photosensitivity
C. Malar rash
D. Arthritis
12. The following test is a common test for cervical root sydrome: B
A. Adson’s test
B. Spurling’s test
C. Finkelstein’s test
D. Trendelenburg’s test
13. Pain around the medical aspect of the knee joint very often caused by inflammation of the Pes Anserinus. This
is conjoin tendon of the folloring muscles: C
A. Gracilis-Sartorius-Bicep femoris
B. Gracilis-Sartorius-Vastus medialis
C. Semitendinosis-Sartorius-Gracilis
D. Semimembranosus-Satorius-Gracilis
OP
14. The function of medial T strap in AFO: A
A. Pes valgus control
B. Pes planus control
C. Pes varus control
D. CTEV control
15. The muscles required for ambulation using crutches are: D
A. Shoulder depressors and biceps
B. Biceps, triceps and wrist felxions
C. Shoulder depressor, biceps, wrist extensors
D. Shoulder depressors, triceps, wrist extensors and finger flexors
16. A 60 year-old woman with right medial knee pain has a genu varum deformity that is observed while she is
standing and walking. What shoe modification can help her pain? C
A. Rocker bottom
B. Medial wedge
C. Lateral wedge
D. Arch support
17. A prerequisite to glossopharyngeal breathing is B
A. Patent trancheostomy
B. Good tongue strength
C. Good abdominal support
D. Need some diaphragmatic activity to assist
18. An individual with emphysema: A
A. A would show decreased flow on forced expiration largely due to loss of elastic recoil of the lung
B. Would have a smaller (more negative) pleural pressure than normal at TLC

145
C. Would have increased bronchial smooth muscle tone
D. Would have an elevated FEV/FVC ratio
19. The activity of climbing up stairs requires the following amount of METs: C
A. 2 METs
B. 4 METs
C. 6 METs
D. 8 METs
20. Which of the following is the effects of aerobic training on autonomic modulation: D
A. A lower vagal tone
B. Increase of systolic
C. Increase of sympathetic activity
D. Decrease of cathecolamine concentration
21. Reconditioning exercise in COPD patient can lead: B
A. Strengthen diaphragm
B. Increase exercise tolerance
C. Markedly increase VO2 max
D. Increase FEV in spirometry test
22. Raindrop in tin roof is classical description given to which of the following spontaneous activity found while
doing EMG? B
A. Fasciculations
B. Fibrillations
C. Myokimia
D. Mytonia
23. The annulospiral ending is the primawy afferent fiber coming from the: A
A. Nuclear bag
B. Nuclear chain
C. Intafusal fibers
D. Extrafusal fiber
24. Which of the protein is not involved in muscle contraction? A
A. Dystrophin
B. Troponin
C. Myosin
D. Actin
25. The neurotransmitter that is released into the synapses by all preganglionic sympathetic and parasympathetic
fiber is: B
A. Norepinephrine
B. Acetylcholine
C. Serotonin
D. Glycine
26. What is the estimated rate of nerve regeneration? C
A. 0.1 mm/day
B. 1 mm/week
C. 1 mm/day
D. 1 cm/day
27. What is the definitions of 4th degree Sunderland’s classifications of peripheral nerve injury? A
A. Distruption of the axon, endonerium and perineurium, the epineural tissue is spared
B. Distruption of axon only, leaving the endoneurium intact; neuroma-in-continuity
C. An intact perineurium surrounding a distruption of axon and endoneurium
D. Local conduction block with the minimal structural distruption
28. The following statement is NOT the characteristic signs of conus medullaris syndrome: D
A. Perineal sensory deficir
B. Sphinter dysfunction
C. Loss of anal reflex
146
D. Pyramidal sign
29. The mechanism of diazepam to reduce the muscle spasticity is by: A
A. Stimulating the GABA-mediated inhibition in CNS
B. Inhibiting glutamate and aspartate in CNS
C. Inhibiting the extrafusal fiber
D. Inhibiting the motor cortex
30. The benefits of physical activity in the elderly include all of the following EXCEPT C
A. Showing of disease progression
B. Promotion of the person’s health
C. Prevention of aortic aneurysm rupture
D. Prolongation of function independence
31. What is the most common cause of dementia in geriatric population? D
A. Toxic-Metabolic dementia
B. Lewy body dementia
C. Vascular dementia
D. Alzheimer disease
32. The primary analgesics to control cancer pain: A
A. Non opioid and opioid
B. Opioid and muscle relaxants
C. Cirticosteroid and non opioid
D. Non opioid and antidepressant
33. What is the best lower limb exercise for the elderly with osteoarthritis C
A. Low resistence isotonic
B. Low intensity aerobic
C. Isometric
D. Isotonic
34. Delayed Onset of Muscle Soreness (DOMS) usually develop after: C
A. Isometric exercise
B. Isokinetic exercise
C. Eccentric exercise
D. Concentric exercise
35. Performing a seated leg exercise is an example of: B
A. Closed kinetic chain exercise
B. Open kinetic chain exercise
C. Mixed kinetic chain exercise
D. Static kinetic chain exercise
36. Which are emphasized to increase muscle strength? D
A. Low to moderate resistance and fewer repetition
B. Low to moderate resistance and more repetition
C. More resistance and more repetition
D. More resistance and fewer repetition
37. A 40 year-old experience an acute a traumatic pain along the lateral aspect of the knee. Pain is reproduced and
accentuated when when the examiner presses proximal to the joint line and simultaneously brings the knee in
and out of 20° to 30° knee flexion. The most likely diagnosis is: B
A. Distal femur stress fracture
B. Iliotibial band syndrome
C. Lateral meniscus strain
D. Lateral plica syndrome
38. An athlete sustains an injury to the extensor mechanism at the proximal interphalangeal joint of his index
finger. Appropiate management includes: D
A. Maintening active motion to prevent collateral ligament shortening
B. Splinting with a volar splint in 25° flexion
C. Early surgical reconstruction
147
D. Splinting in extension
39. Ankle sprains are ubiquitos in sport and are the most common athletic injury. The most commonly injured for
this disorder is: C
A. The medial deltoid ligament
B. The calcaneo fibular ligament
C. The anterior talofibular ligament
D. The posterior talofibular ligament
40. Which of the following activites would be most diffilcut to return to following an anterior cruciate ligament
(ACL) injury A
A. Baseball pitching
B. Marathon running
C. Squestrian activites
D. Cross-country skiing
41. The intensity of ultrasound prescription for tendonitis/bursitis is: B
A. 0.2-0.3 W/cm2
B. 0.5-2.0 W/cm2
C. 2.5-3.0 W/cm2
D. 3.5-4.0 W/cm2
42. Patients with diagnosis Bell’s Palsy of left side, six days after onset. Modalities that ber given were SWD of
area retro auricular. Effect that expected is: A
A. Decreased of stylomastoideus foraminal edema
B. Increased of left facial muscle strength
C. Make facial massage more easy
D. Reduced facial pain
43. The initial treatment for osteoarthritis is: B
A. Knee orthosis
B. Therapy to relieve joint sympotoms
C. Medication to reverse articular damage
D. Immobilization of the joint to prevent deformity
44. A common source of shoulder pain in shoulder impingement syndrome is D
A. Extra-articular
B. Short head of biceps
C. Long head of biceps
D. The rotator cuff tendon
45. For pain to considered chronic it must have been present for at least: B
A. 1-2 months
B. 3-6 months
C. 7-12 months
D. 12-18 months
46. In thoracic Outlet Syndrome, if Adson test is positive, this suggested to the following condition: D
A. Hypertrophy of the m scalenus posterior
B. Hyperthrophy of the m pectroalos mayor
C. Fracture of the clavicle
D. Cervical rib
47. Radiological changes in degenerative joint disease do NOT include: A
A. Ankylosis of the large joint
B. Subchonral sclerosis
C. Joint space narrowing
D. Osteophytes
48. ATNR reflex apperance pararel with development of: A
A. Brainstem
B. Mildbrain
C. Cortical
148
D. Spinal
49. The period when intense motor learning and basic language development occur at: D
A. Birth to sixth month
B. Birth to one year of age
C. Birth to two years of age
D. Birth to three years of age
50. Child with a congential transvere radial limb deficiency should have in initial prosthesis fit at what
developmental stage? A
A. At the time of first sitting independently
B. At the time of starting kinder garden
C. At the time of initially walking
D. At soon as possible after birth
51. Six years old boy with Down Syndrome who develops torticollis, hyperreflexia, lower extremity hypertonicity.
You should: B
A. Ophthalmologic evaluation
B. Obtain cervical spine x-ray
C. EMG lower extremity
D. MRI of the brain
52. Arthrogryposis multiplex congential most commonly: B
A. Myophatic in origin
B. Neurogenic in origin
C. Associated with mental retardation
D. Associated with muscle hyperthrophy
53. Mrs V, 59 y.o. with diagnosis right cervical pain, radiating to the right arm and hand, radiologic examination
revealed narrowing of foramen intervertebrale and uncovertebra area of C5 and C6. The rational treatment
consist of: A
A. Diathermy, TENS, Cervical traction and Neck Cailliet Exercise
B. Diathermy, TENS and Neck Cailliet Exercise
C. Diathermy, TENS and low level laser therapy
D. Diathermy, TENS and Cervical traction
54. A patient present to your office with knee pain from a flare of rheumatoid arthritis. She has a moderate
efflusion and warmth at her knee, the therapist wants to use ultrasound to her knee for treatment. The
consideration about using USD: A
A. It s contraindicated
B. It accelerates healing
C. It helps with pain control
D. It will help prevent deformity
55. Which therapeutic modality delivers medication to the site of pathology by promoting the movement of
changed particles through the skin under an imposed electrical field? A
A. Iontophoresis
B. Phonophoresis
C. Low energy laser
D. Ultraviolet radiation
56. Which of the following is characteristic of the radial nerve lesion? B
A. Ape hand
B. Wrist drop
C. Claw hand
D. Intrinsic minus hand
57. The gold standard for diagnostic of Duchene muscular dystrophy is: B
A. to check blood creatine kinase level
B. to have muscle biopsy
C. MRI of leg muscles
D. to perfor EMG
149
58. The cause of Myasthenia gravis is: D
A. nerve cells are destroyed
B. lesion in the cerebral cortex
C. lesion inthe anterior horn of the spinal cord
D. acetylcholine receptor in muscle cells is destroyed
59. The most important measure for a SCI patient using indwelling catheter B
A. Bladder irrigation every 4 hours
B. To drink enough volume of water
C. Change the catheter every other day
D. Administration of acidifying substance
60. The patient may complain of numbness, paresthesis or dysethesis radiating to the first second, third, and lateral
digits. Symptoms may exacerbated during sleep and relieved with wrist shaking. This condition found in: B
A. Cubital Tunnel Syndrome
B. Carpal Tunnel syndrome
C. Pronator teres syndrome
D. Guyon syndrome
61. The Barthel index is NOT characterized by the following statement: B
A. Assigment of numerical scores and relative weights to ADL variables
B. Assesment of progress in language and cognitive skills
C. Utilizationfor interinstitutional comparisons
D. Prediction of rehabilitation outcome
62. Massage techniques used in sport medicine for the purpose of relieving soft tissue from the abnormal grip of
tight fascia is: A
A. Friction
B. Kneading
C. Efflorage
D. Vibration
63. The following condition is NOT aa contraindication for exercise program to diabetic patient with
cardiovascular complications: C
A. Ketosis
B. Blood sugar: <80 mg%
C. Blood sugar: 180-250 mg%
D. Blood sugar: >300 mg%
64. Which of the following cardiovascular factors contributes orthostatic hypotension in the immobilized patient: D
A. Normal stroke volume is maintaned despite lowered blood pressure
B. Rapid heart rate prevents optimal systolic filling
C. Increased venous return causes prolonged systole
D. Sympathetic response is inadequate
65. During vigorous exercise, while participating in a stress test, it is noted that the T wave increases by 10%. No
other changes are noted, ho should this be interpreted D
A. Definitely abnormal-highly suspicious but not absolutely diagnostic of ischemic heart disease
B. Midly abnormal-would recommend that maximum effort stress test performed
C. Definitely abnormal and diagnostic for ischemic heart disease
D. Normal
66. How might the patient with obstructive pulmonary disease help overcome air trapping? C
A. Slowly inhale until the lungs are fully expanded
B. Forcefully inhale for as long as possible
C. Exhale with the lips pursed
D. Forcefully exhale
67. Sign of improvement from pulmonary rehabilitation program: A
A. Decreased of dyspnea and increase in exercise tolerance
B. Increase strength of respiratory muscle
C. Increase pulmonary physiology test
150
D. Increase work of breathing
68. Which cervical is the most restrictive? A
A. Halo
B. Four-poster brace
C. Philadelphia collar
D. Stemo-occipitial mandibular immobilizer (SOMI)
69. To maintain a level pelvis and a normal gait while using an above knee prosthesis, the muscle of the hip that
must be adequately contracted are the: B
A. Adductors
B. Abductors
C. Extensors
D. Flexors
70. A female volleyball athlete has been diagnosed with a grade 2 quadriceps strain after lunging for a ball. The
athlete trainer has determined that the rectus femoris in involved and there is loss of 45 degrees of knee flexion
while lying prone. What do you recommend to help the athlete return to play? A
A. Hamstring stretch
B. Quadriceps stretch
C. Hip adductor stretch
D. Hip abductor stretch
71. To prevent sport injury to the lower extremity, the following core should be strength: D
A. Abdominal muscle, hip extensor, knee extensor
B. Abdominal muscle, hip internal rotator, hip extensor
C. Abdominal muscle, hip internal rotator, hip extensor, knee extensor
D. Abdominal muscle, hip abductor, back extensor, hip external rotator
72. 24 year-old marathin runner report lateral knee pain after hill training. Examination revelas no effusion; and
result of Ober’s test are positive, what is the most likely diagnosis? D
A. Lateral meniscal tear
B. Popliteus tenosynovitis
C. Peroneal nerve entrapment
D. Iliotibial band friction syndrome
73. Which is the most important risk factor for an ankle sprain? D
A. Rear foot valgus
B. Short Achilles tendon
C. Generalised joint laxity
D. A history of a previous sprain
74. Parkinsonian feature which impaired automatic motor task performance is: A
A. Bardykinesa
B. Hypotonia
C. Spasticity
D. Weakness
75. This type of activity and exercise is NOT recommended is osteoporotic to prevent developing compression
fractures of spine: D
A. Prevent falls
B. Keep heavy objects close to chest when lifting\
C. Flex hips and knees to bend down to lift objects
D. Flexion exercises for the spine and progressive resistive exercise
76. This is NOT the characteristic of geriatric patient: C
A. Multipathologic
B. Non specific clinical symptoms
C. No changes of functional status
D. Limited physiologic reserved capacity
77. Which of the following is the most important lifestyle modification for preventation of osteoporosis? C
A. Increasing the intake of alcohol
151
B. Eating a diet high in protein and phosphorus
C. Avoiding cigarette smoking and high intake of caffeine
D. Minimizing the use of nonsteroidal anti-inflammatory medications
78. The following below is NOT cause of the chronic ankle instability: A
A. Achilles tendinitis
B. Recurrent ankle sprain
C. Peroneal muscle weakness
D. Ankle propioceptive deficit
79. Conservative rehabilitation for rotator cuff tear on recovery phase (up to 6 months): C
A. Increased flexibility
B. Reduced pain and inflammation
C. Improve upper extremity range of motion and propioception
D. Reestablish nonpainful and scapulohumeral range of motion
80. Special problem associated with humeral shaft fracture is: B
A. Ulnar nerve injury
B. Radial nerve injury
C. Axiallary nerve injury
D. Medianus nerve injury
81. A 54 year-old man has complete foot drop for 15 days. EMG showed fibrillation potentials & reduced
interference pattern of anterior tibial, peroneus longus, extensor halucius longus and lumbosacral paraspinal
muscles but not in tne gastrocnemius. Nerve conduction velicity of peroneal nerve is normal the interpretation
is: C
A. L3 radiculopathy
B. L4 radiculopathy
C. L5 radiculopathy
D. S1 radiculopathy
82. The vital capacity is the sum of: D
A. Inspiratory reserve volume plus tidal volume
B. Inspiratory capacity and functional residual capacity
C. Inspiratory capacity, tidal volume and expiratory reserve volume
D. Inspiratory reserve volume, tidal volume and expiratory reserve volume
83. Component of comprehensive pulmonary rehabilitation: D
A. Decrease of symptom, normal pulmonary capacity test
B. Muscle strength exercise, repair of pulmonary radiography examination
C. Endurance exercise, intervention of psychological and life style, normal respiratory function test
D. Exercise training, nutritional therapy, education for management of psycological and life style,
promotion for long term adherence
84. Which of the following statement is NOT appropiate for Borg scale for rating perceived exertion (RPE Scale):
A
A. The objective rating of the intensity of exertion perceived
B. Rather than using heart rate alone to clinically determine intensity of exercise to another
C. Borg scale can’t assist the clinician in judging degree of fatigue reached one test to another
D. Borg scale can assit to correlate the level of fatigue during testing with the experienced during daily
activity
85. Exercise programming in patients PTCA have more benefit: A
A. Increase aerobic capacity
B. Increase myocardial oxygen demand
C. Decrease ejection fraction of the left ventricle
D. Decrease stroke volume response to submaximal exercise
86. The most common cause for “floppiness” at birth is: A
A. Cerebral dysfunction
B. Central core myopathy
C. Pradder-Willy syndrome
152
D. Spinal muscular atrophy type 1
87. The main motor characteristic of CP Ataxia: A
A. Disturbance of balance
B. Writhing movement
C. Jerky movement
D. Spasticity
88. The statement below is NOT charachteristic of cerebral palsy: B
A. The underlying neurologic lesion must be static
B. The process which cause the cerebral palsy is still active
C. The sign of cerebral palsy is a disorder of movement and posture
D. It is caused by non progressive injury which affects the immature brain
89. Test used to assess the contracture of the tensor fascia lata is: A
A. Ober
B. Fabere
C. Thomas
D. Trendelenberg
90. A 13 year-old girl is found to have scoliosis om routine physical examination. Spine radiographs demonstrate a
Cobb angle of 25° . The optimal treatment is: B
A. Radiologic follow-up every 4 to 6 months
B. Use of a Milwaukee brace for 23 hours a day
C. Lateral surface electrical stimulation with bracing for 6 hours a day
D. Lateral surface electrical stimulation with trunk strengthening exercises
91. The rehabilitaiton program for spondylolisthesis: A
A. Back flexion exercise program is more effective
B. Anterior pelvic tilt exercise program is more effective
C. The patient shoul not restrict heavy lifting activity
D. For grades 1, surgical treatment is more recomended
92. The most commong cause pf significant hemarthrosis after traumatic injury is: A
A. Anterior cruciate ligament rupture
B. Lateral collateral ligament tear
C. Osteochondral fracture
D. Patellar dislocation
93. One of the muscles is NOT a biarticular muscle: D
A. Iliopsoas
B. Hamstring
C. Biceps brachii
D. Gluteus maximus
94. In pronator teres syndrome all of the following muscles could be affected, EXCEPT: A
A. Pronator teres
B. Pronator quadratus
C. Flexor digitorum sublimis
D. Flexor digitorum profundus
95. A disorder of neuromuscular transmision due to an autoimmune response against Ach receptors on the
portsynaptic membrane, associated with thymic disosder or thymc tumor with the clinical presentation are
proximal fatigue and weakness is: B
A. Lambert-Eatin Syndrome
B. Myasthenic Syndrome
C. Myasthenia Gravis
D. Botulism
96. Individuals with SCI are considered to be at risk for autonomic dysreflexia and orthostatic hypotension of
lesion on: A
A. T6 or above
B. T7
153
C. T8
D. T9
97. Repetition in preserved in A
A. Transcortical sensory aphasia
B. Wernickle aphasia
C. Global aphsia
D. Borca aphasia
98. Two months ago a 70 year-old male got hemorrhagic stroke with right hemiparesis. Until discharged from
hospital, his family has difficulty when communicating with him. He has difficulty when speaking and cannot
repeat when he was tolde to. This 70 year-old male may have a: A
A. Lesion in the inferio frontal gyrus
B. The lesionis in the right hemispher
C. His impairment does not socially incapacitatiing
D. An incomplete lesion in the posterior pareto-occipital region
99. The statement below is NOT the correct answer for visual and body perception: C
A. Neglect
B. Asteriognosis
C. Propioception
D. Agraphaesthesia
100.A stroke patient shows signs of weakness and sensory deficit upper extremity more than lower extremity. This
condition most probably affecting the following artery: A
A. Middle cerebral artery
B. Anterior cerebral artery
C. Superior cerebral artery
D. Posterior cerebral artery
ANSWER:
A. IF NUMBERS 1,2,3 ARE CORRECT
B. IF NUMBERS 1,3 ARE CORRECT
C. IF NUMBER 2,4 ARE CORRECT
D. IF ONLY 4 IS CORRECT
E. IF ALL NUMBERS ARE CORRECT
101.For patient with osteoporosis, the exercise precription may be modified to AVOID: D
1. Exercise frequency three to five days per week
2. Weight bearing aerobic activities
3. Exercise duration 30-60 minutes
4. High intensity resistance exercise
102.Which activity(is) is(are) include in energy conservation method below? C
1. Work with antigravity assisting
2. Sit to work when possible
3. Rest after fatidue
4. Plan ahead
103.About Frenkel’s exercises for ataxic conditions: E
1. Design primary for condition
2. They are not intended for strengthening
3. Commands should be given in an even, slow, monotonous voice
4. Ability of patient to interpret deep muscle and joint sensibility may be checked by having the patient
perform the exercises with eye closed
104.The purposes of an initial comprehensive assesment of sensory integrative problem are: A
1. To indentify the impact sensory processing and praxis problems
2. To provide information assist with treatment planning
3. To identify specific sensory integrative problems
4. To idenitfy the behavior problem
105.Physical examination of someone with biceps tendinitis will typically reveal: D

154
1. Absent biceps tenden reflex
2. Decreased ROM of the elbow
3. Tenderness over the lateral aspect of the arm
4. Significant palpale tenderness at the bicipital groove
106.Rheumatoid arthritis: A
1. Evidence if inflammatory by history, examination and laboratory test
2. Symmetric, polyarticular inflammatory arthritis
3. A characteristic of joint involvement
4. History suggest mechanical pain
107.Genu valgum: the mechanical axis for the lower limbs is displaced laterally and this might be increasing
rapidity B
1. Foot pronation
2. Femoral anteversion
3. Compression of the medial knee joint
4. Shortened medial knee joint structure
108.A girl patient A
1. Her pubic hairs are growing
2. Her breast are growing bigger
3. She has her menarche a month ago
4. The Risser’s sign in reaching to grade 4
109.The inflammation of the thumb extensor pollicis brevis and abductor pollicis longus, is known as: B
1. De Quervain’s syndrome
2. Carpal tunner syndrome
3. Tenosynovities of the thumb
4. Trigger finger of the thumb
110.Fucntionally, a trans metatarsal amputation is superior to Lisfranc’s (tarsometatarsal) amputation, because it
provides: E
1. An intact tibialis anterior insertion to oppose the pull of the triceps surae
2. A stump that is easier to fit
3. A broader base of support
4. A longer lever arm
111.Phantom pain: C
1. Is an expression of wish fulfillment
2. Does no occur with congenital absence of a limb
3. Indicates an emotional maladjustment
4. Is accentuated and perpetuated by delayed wound healing
112.The indication(s) for using afo an ankle-foot orthosis (AFO) to improve gaits is/are: E
1. Weak push-off at late stance
2. Mediolateral instability at the ankle
3. Passive plantarflexion in swing phase
4. Foot drop at heel strike due to weak ankle dorsiflexor
113.The manifestation of circumduction gait of amputee who is wearing a lower limb prosthesis usually caused by:
A
1. Socket to small, the residual limb cannot enter fully
2. Inadequate suspension, socket slips down during swing
3. Amputee is reluctant to flex the knee during swing because of poor balance
4. Amputee does not bother to flex the knee because of prosthesis is too short
114.A 50 year-old woman reportedly had a right anterior cerebral arteri stroke. In your evaluation, you expect to
find: C
1. Distal greater than proximal upper-extremity weakness
2. Lower limb greater than upper-limb weakness
3. Upper and lower facial weakness
4. Language is typically not disturbed
155
115.A C7 quadriplegic patient should be expected to perform the activities below independently, which are (is): A
1. Sitting pivot transfer
2. Sliding board transfer
3. Wheelchair to-car transfer
4. Floor to wheelchair transfer
116.The goal of medical treatment on Parkinson’s Disease are: C
1. Decreased dopamine action
2. Increased dopamine action
3. Increased cholinergic effect
4. Decreased cholinergic effect
117.Claw hand deformity and loss sensibility on first until fifth in Morbus Hansen disease caused by lesion on: B
1. N ulnaris
2. N radialis
3. N medianus
4. N musculoscutaneous
118.The most common behavioral and personality changes seen following Traumatic Brain Injury is/are: E
1. Attention deficit rigidity
2. Visual perceptual deficit
3. Spatial dysfunction
4. Memory deficit
119.The difference between Type I in contrast to Type IIB skeletal muscle fibers is: B
1. Type I are predominantly aerobic
2. Type I are predominantly anaerobic
3. Type IIB are predominantly anaerobic
4. Type IIB fibers are high hemoglobin content
120.The following statement(s) about type I muscles fiber is/are correct: A
1. Contain a large number of mithocondria and myoglobin
2. Dominant in much of marathon runner
3. Also called as slow twitch fiber
4. Generate larger force
121.The following is/are the characteristic (s) of fibrillation waves: A
1. Have very samll amplitude
2. Have very short duration
3. The rhytm is irregular
4. Diving bomber sound
122.With the EMG machine we can get the following result: A
1. The severity of pathology
2. The level/location of morot unit pathology
3. The motor and sensosry conduction velocities
4. The etiology of motor unit abnormalities
123.The following is/are the clinical characteristic of Down Syndrome: E
1. The head usually larger than normal because of development of facial bone and muscle
2. The toes are usually short and in the majority; there is wide space between the first and second toes
3. Ligamentum laxity causing deficiency in grip strength and ankle strength that develop on scholl age
4. Most significant musculoskeletal impairment are osing to hypotonia an ligamentum laxity caused by
deformity of the joint
124.Characterisric of normal locomotor development: B
1. Wide to narrow base
2. Lower limb more functional initially
3. Proximal before distal
4. Extension patterns initially then flexion followed by isolated movement
125.The following are the characteristics of Duchene’s Muscular Dystrophy: E
1. The disease affects males
156
2. It is transmitted by X linked mode on inheritance
3. Measurement of serum creatinine kinase is the most reliable test
4. The chil’s mother, his sister and maternal aunts are potential carries
126.You are examining a child of 7 to 8 months of age. Peristence of which the following primitive reflexes of
postural responses would you be concern about? D
1. Placing
2. Landau
3. Parachute
4. Asymmetric tonic neck
127.Consideration for bracing in children with Spinal Muscular Atrophy type II: E
1. Light weight braces
2. To provide functional
3. Psychological benefit
4. Fitting for these brace is best done under 2 year of age
128.Man, 55 year-old, weight 85 kg, height 165 cm. He got myocard infarct I month ago. The fasting glucose level
250, total cholesterol 400, no hypertension. Factor that should be considered to prescribe cardiac rehabilitation
program: C
1. Avoid all exercise
2. Assessed the stress test
3. Limitation of daily activity
4. Reduce the risk factor for 2nd cardiac disease attack
129.Indication for exercise test termination in Cardiac Rehabilitation are: E
1. Moderately severe angina (+3 to +4)
2. Signs of poor perfusion (palor, cynosis)
3. Central nervous system symptoms (dizziness)
4. Drop in SBP >10 mmHg from resting value
130.Improve aerobic capacity and increased oxygen transport also result from exercising: C
1. Decreased cardiac output
2. Decreased resting heart rate
3. Decreased ejection fraction
4. Decreased systolic blood pressure
131.Goals of pulmonary rehabilitation program in the patient with chronic obstructive pulmonary disease (COPD)
ia are/: A
1. Facilitating drainage of exercise bronchial secrection
2. Improvefing the efficiency of the breathing patterns
3. Overall physical reconditioning and well being
4. Reversting the result of pulmonary function test
132.For chronic lung disease various walking test with sub maximal exercise have the benefit below: A
1. To detect exercise intolerance
2. To detect functional limitation
3. To evalute reconditing program
4. To detect limitation in lung function
133.The following assesment is assess gait and balance: B
1. Romberg test
2. 6 minute walking test
3. The Get up and Go test
4. 12 minute walking test
134.Older adults are the least physically active group, the essential healthy aging to maintaned regular physical
activities is/are from exercise type: E
1. Balance exercise
2. Flexibility exercise
3. Endurance exercise
4. Muscle strenghtenig exercises
157
135.Assesment for geriatric patient with falls: E
1. Visual system
2. Polypharmacy
3. Foots problems
4. Environment factors
136.Effects of aging on function, for strength and endurance: A
1. Increases in fat and connective tissue within older muscle
2. Reduction in the number of functioning motor units
3. Reduced of type II fast-twitch muscle fiber
4. Increased of type I slow-twich muscle
137.The American Geriatrics’s goal of diabetic control in the elderly includes: D
1. Tight glycemic control
2. Hemoglobin A1c below 7%
3. Easting plasma glucose below 120 mg/Dl
4. Reduction of cardiovascular complication
138.Where is the ground reaction force vector located at mid stance? B
1. Anterior to ankle
2. Posterior to ankle
3. Anterior to knee
4. Posterior to knee
139.Plantar flexon muscles are inactive by the time of: D
1. Foot flat
2. Heel strike
3. Mid stance
4. Toe off
140.Hamstring function as: E
1. Protectors of posterior structure from over stretching in combined movement of hip flexion and knee
extension
2. Good extensors, rotators and weak adductors of the hip
3. Rotator of the knee with knee flexion
4. Good flexor of the knee
141.A lesion at medial side of the Sciatic nerve will lead to weakness of the following muscle(s): A
1. Semimembranosus
2. Semintendinosus
3. Gastrocnemius
4. Biceps femoris
142.What is/are the treatment for progress proprioception, coordination and agility during the chronic phase of soft
tissue healing in sport injury rehabilitation are: B
1. Balance activities
2. Isotonic strengthening exercise
3. Surface modification
4. Isokinetics strenghtening exercise
143.How does the anterior cruciate ligament (ACL) injury become torn? B
1. Deceleration of the leg via quadriceps contraction combined with valgus and external rotation forces
upon a slightly flexed knee
2. Sudden external rotation of hyperflexed knee
3. Sudden hyperextension of the knee
4. Direct blows to the ankle
144.Medial stress of injuries of the elbow in throwing athlete, includes: A
1. Ulnar nerve traction
2. Avulsion of medial epicondyle
3. Flexor muscle strain or tear
4. Olecranon osteophyte formation
158
A basketball athlete had injured after he jump and landed on his right foot in the internally rotate position. He heard
“pop and felt painful on his right knee. He ceased form this game because he cannot walk and got an edema
suddenly.
145.What is/are possible diagnoses for his condition? C
1. Quadriceps muscle rupture
2. ACL rupture
3. PCL rupture
4. Meniscal tron
146.What is special test/sign will positive for his condition? C
1. Sagging knee
2. Mc Murray test
3. Homan test
4. Anterior Drawer test
147.A variety of therapeutic gains by electrical stimulation a muscle contraction is/are: B
1. Muscle strengthening
2. Recovery of neuropraxia
3. Muscle pump contraction
4. Prevents from Wallerian degeneration process
148.A 53 year-old woman with chronic venous disease has lower extremity edema, skin hyperpigmentation,
dermatitis, and venous varicosities. She does not have any ulceration. The mainstay of treatment for this patient
is:D
1. Intermittent external pneumatic compression
2. Hydrotherapy with warm water (100 degree F)
3. Topical steroid cream
4. Gradient elastic stocking
149.The effects of ultra violet radiating is/are: B
1. Photochemical effects
2. Relief of muscle spasm
3. Have biological effects
4. Increases the pain threshold
150.The following statement(s) is/are TRUE about shortwave diatherny: A
1. Inductive applicators produce more heat on deeper tissue
2. Capacitive plates produces more heat in the skin and superficial tissue
3. Inductive coil applicators produce the most heat in tissues that have high electrical conductivity
4. To avoid burns during application, the patients’s skin must be keep moist by wrapping with wet towels.

159
INDONESIAN COLLEGE
OF
PHYSICAL MEDICINE AND REHABILITATION

NATIONAL BOARD EXAMINATION

DECEMBER 07, 2012

160
1. The primary afferent fibers coming from the nuclear chain of the muscle spindle have the following conduction
velocity :
A. 20-40 m/sec
B. 40-60 m/sec
C. 60-80 m/sec
D. 80-100 m/sec
2. One the efects of baclofen as muscle relaxant is via the mechainism of:
A. To inhibit calcium ions release
B. To inhibit acetylcholine
C. To stimulate glutamate
D. To stimulate GABA
3. In neuropraxia the following finding can be deteced during EMG/NCV studies:
A. Positive shrap waves
B. Fasciculations waves
C. Fibrillation potentials
D. Conduction block at certain level of nerve
4. Which statement below is true according patellofemoral pain syndrome?
A. The pain often be felt while running of jogging ??
B. Also refferend to as posteromedial knee pain syndrome
C. Should always be associated with laxity of ligament around the knee
D. Is characterised by pain front of middle edge of the knee or under the knee cap
5. Injury to the cuff muscles and tendons of the shoulder is often occurs in?
A. Tennis player
B. Discus Thrower
C. Boxing Athletes
D. Rowing athetes
A young basketball athele, aged 18 years old, injured his right ankle while playing basketball in a competition I
week ago,. His x-ray films showed no fracture. His physician diagnosed the injuy as lateral ankle sprain. Visual
Analog scale score has in intensity from 8/10 to 6/10, but the pain increases with weight bearing and certain
demonstrated movements. No warmth of the skin of anterolateral aspect of the right ankle, slight acchymosis and
sweling are noted. His anterior drawer test is positive, and his talar tilt is negative (number 6 until number 8)
6. What is classification of the injury due to its severity, and what stage of healing is this patients in?
A. Grade I ankle sprain and inflammatory phase
B. Grade I ankle sprain and proliferative phase
C. Grade II ankle sprain and inflammatory phase
D. Grade II ankle sprain and proliferalive phase
7. In this case, what structure has been injured?
A. Anterior talofibular ligament
B. Posterior talobulator ligament
C. Calcancofibular ligament
D. Syndesmotic ligament
8. What progams are suited for this case?
A. Best and fixation ??
B. Stetching and endurance exercise
C. Proprioceptive and balance exercise
D. Weight bearing as tolerated and ROM exercise
9. Which flexor muscle below located profoundly?
A. Pronator teres
B. Pronator longus
C. Pronator quadratus
D. Flexor carpi ulnaris
10. A severe herniated disc between L4-L5 most probably leading to weakness of the following muscle :
A. Extensor digitorum longus

161
B. Extensor digitorum brevis
C. Extensor hallucis longus
D. Peroneus longus
11. The quadriceps femoris is an important knee extensor muscle. Which portion of this muscle also causes hip
flexion?
A. Vastus intermedius
B. Vastus lateralis
C. Astus medialis
D. Rectus femoris
12. The most important change in the aging musculoskeletal system is
A. Increase in bone density
B. Decrease in lumbar lordosis
C. Increase in thoracic kyphosis
D. Increase in type to muscle fiber
13. Which of the following is most likely to enable you to determine the reason for a fall in an older patient?
A. A completed neurologic examined
B. Magnetic resonance imaging of th brain
C. Determination of the patient’s functional status
D. A careful history of the circumstances surrounding the fall
14. Functional assessment of instability in the older faller is :
A. Vision
B. Vestibuler
C. Mobility skills
D. Proprioception
15. In aging people, which lung function increases :
A. Forced vital capacity
B. Total lung capacity
C. Residual volume
D. Vital capacity
16. This is NOT the characteristic of geriatric patient :
A. Multipathologic
B. Non spesific clinical symptoms
C. No change of functional status
D. Limited physiologic reserved capacity
17. Which of the following is NOT a contraindicators to microwave diathermy ?
A. Bruising
B. Old scar tissue
C. Eletrical implant
D. Infectious skin disease
18. The high voltage pulsed current (HVPC) is
A. A direct current and biphasic
B. A monophasic pulsed current
C. A diadynamic pulsed current
D. Symmrtric biphasic pulsed current
19. Which one of the following CPM (Continuous Passive Motion) is contraindicated ?
A. Burn patients
B. Capsulotomies
C. Knee arthroplasty
D. Unstable fractures
20. Which of the following parameters most closely correlates with oxygen consumption ?
A. rectal temperature
B. oral temperature
C. respiratory rate
162
D. pulse rate
21. how right the patient with obstructive airway disease help overcome air trapping ?
A. slowly inhale until the lungs are fully expanded
B. force fully inhale for as long as possible
C. exhale with the hips pursed
D. forcefully exhale
22. During exercise, there is an increase in a person’s
A. Stroke volume
B. Diastolic pressure
C. Total peripheral resistance
D. Pulmonary arterial resistance
23. In a resting, healthy man, the ejection fraction is approxinately?
A. 0.1
B. 0.2
C. 0.3
D. 0.6
24. The best device to unload a painful arthritis of the hip is :
A. Knee Hip Ankle orthoses
B. Single point straight cane
C. Hip derotation orthoses
D. Forearm erutches
25. When should upper extremity prosthesis fitting be initiated in the adult ?
A. When residual limb strength is full
B. When the patient requests a prosthesis
C. Within the first month after amputation
D. When residual limb volume has stabilized
26. The most commonly used prosthesis for patients with hip disarticulation :
A. Hip disarticulation prosthesis with wooden socket
B. Canadian hip disarticulation prosthesis
C. Modular hip disarticulation prosthesis
D. Hemipelvectomy prosthesis
27. Signs of Leg. Calve Perthes disease in the child is :
A. Limping and restriction in hip motion, especially hip abduction.
B. Limping and restriction in hip motion, especially hip adduction
C. Severe hip pain that is exacerbated with any motion
D. Groin pain that increases in weight bearing
28. The child has chief complaint of bowlegs that are gradually progressing in severity and not improving
spontaneusly. The determine is called :
A. Clubfoot
B. Tibia vara
C. Knock knees
D. Genu recurvatum
29. Above case is caused by growth retardation of the medial and posterior pat of the proximal tibial epiphysis due
to abnormal weight bearing stress and compression force on the knees. This disease is known as :
A. Genu varum
B. Blount’s disease
C. Bicket’s disease
D. Ollier’s multiple enchondromatosis
30. What is seen when the Landau reflex is elicited and when does it appear ?
A. In the vertical position, the baby will support weight on his feet. Appears at 3 – 4 months of age
B. In ventral suspension, baby will extend their hand & lower extremities. Appears at 4 – 5 months of
age

163
C. In supine position, the baby will extend the arm and leg on the side that the head is turned toward. Appears
a 5 – 6 months of age
D. In sitting position, the baby will extend the arm and hand to catch himself and prevent falling to one side.
Appears at 5 – 7 months of age
31. A 55 year old man has severe pain on gentle touching of the arm. Six months ago the median nerve was
damaged during creation of arteriovenous fistula for dialysis. Which of the following terms best describes the
phenomenon?
A. Allodynia
B. Hyperpathia
C. Hyperalgesia
D. Hypersensitivity
32. A 27-year old woman complaints of pain in the shoulder and suprascapular region 1 month after undergoing
modified radical neck dissection for carcinoma of the thyroid. Examination shows decereased sensation to light
touch over the superior aspect of the right trapezius and scapular winging with resisted shoulder abduction.
Which of the following nerve was most likely transected during the procedure?
A. Axillary
B. Long thoracic
C. Dorsal scapular
D. Spinal accesory
33. A woman riding a mountain bike on a rough trail hits a rut and is thrown from the bike. Her upper arm hits a
tree, fracturing the humerus just above the teres major muscle. Later, during examination, it is noted that she
cannot extend her forearm at the elbow or hand at the wrist. Which nerve must have been injured?
A. Musculocutaneous
B. Axillary
C. Median
D. Radial
34. The most cranial nerve involved in Miller Fisher syndrome ( variant of GBS ) is :
A. Facialis nerve
B. Abducent nerve
C. Trigeminal nerve
D. Oculomotor nerve
35. A 33 yo male foot ball player came with trauma of his shoulder during a foot ball game. A few week later
showed weakness of internal rotation and adduction the left limb, the forearm was held in extension and
pronation because of elbow flexion weakness. These findings are suggestive of :
A. Klumke palsy
B. Erb –duchenne palsy
C. Thoracle outlet syndrome
D. A lesion of cervical plexus
36. These are the effects of treadmill training in Parkinson patients EXCEPT :
A. It reduces termor
B. It decrease fall risk
C. It impoves quality of life
D. It improves gait impairment
37. Maximal force generation by a muscle occurs during
A. Fast eccentric contraction
B. Slow concentric contraction
C. Fast concentric contraction
D. Slow concentric contraction
38. Which are emphasised to increase muscle endurance ?
A. More resistance, more repetition
B. More resistance, fewer repetition
C. Low in moderate resistance, more repetition
D. Low ro moderate resistance, fewer repetition
164
39. The following is the test to evaluate the posterior cruciate ligament tear (PCL) :
A. Varus test
B. Valgus test
C. Posterior drawer test
D. Anterior drawer test
40. Characteristic of joint pain and stiffness. In knee osteoarthritis:
A. In the morning but typically last less than one hour
B. In the morning but last longer than one hour
C. Through the whole day
D. Just at night
41. Stage 1 of rotator cuff injury:
A. Inflammation and edema in the rotator cuff
B. Progressed to fibrosis and tendonitis
C. There is a complete rotator cuff tear
D. Pain but no inflammation
42. Tearing of the extensor hood causing hyperextension of the MCP, flexion of PIP, hyperextension of DIP is
called:
A. Boutenniere
B. Swan neck deformity
C. Heberden’s node
D. Bouchard’s node
43. Which of the following is a general treatment principle for knee osteoarthritis?
A. Encourage a high impact aerobic home exercise program
B. Strengthen the quadriceps muscle
C. Avoid extreme range of motion
D. Avoid the use of modalities
44. The following statements regarding F-wave are ture, EXCEPT:
A. The amplitude of F-wave are such lower than the amplitude of the M-wave
B. The pathway is from motor fiber to cell body back to motor fiber
C. It requires supramaximal stimulation
D. It requires submaximal stimulation
45. The most commong clinical application of the H-reflex is to test:
A. Polyneuropathy
B. S1 radiculopathy
C. Cervical radiculopathy
D. Guillain Barre Syndrome
46. The following is the characteristic of a MUAP (Motor Unit Action Potential:
A. Irregular in nature
B. The shape is always trisphasic
C. Appear under voluntary control
D. Appear under unvoluntary control
47. A discus thrower complains of pain in the upper part of shoulder. One of the finding on examination is cross
adduction chest positive sign, pain arch at 110o. What structure most likely to be involved
A. Suprahumeral joint
B. Glenohumeral joint
C. Sternoclavicular joint
D. Acromioclavicular joint
48. A martial arts player complaint of back knee pain after kicks his opponent. Examination of knee demonstrates
pain during varus stress test. What is the most likely diagnosis?
A. Strain popliteus muscle
B. Medial collateralligament injury
C. Lateral collateral ligament injury
D. Posterios cruciate ligament injury
165
49. Which bony wrist injury related to forceful extension of the hand is commonly seen in gymnasts?
A. Ulnar fracture
B. Radius fracture
C. Lunate fracture
D. Scaphoid fracture
50. What should be the first treatment considered for chronic lateral epicondylitis?
A. Surgical release
B. Botulinium toxin
C. Corticosteroid injections
D. Wrist extensor stretching
51. What is the internal moment of the ankle when heel strike?
A. Concentric contraction of the gastrocnemius
B. Eccentic contraction of the gastrocnemius
C. Concentric contraction of the dorsiflexor
D. Eccentic contraction of the dorsiflexor
52. What muscle is the most powerfullat internal rotation of the shoulder?
A. Pectoralis mayor
B. Supraspinatus
C. Subscapularis
D. Teres mayor
53. Atlanto-odontoid classified as
A. Gynglimus joint
B. Ellipsoidal joint
C. Trochoid joint
D. Saddle joint
54. A good initial treatment plan for neurogenic claudication secondary to spinal stenosis would include:
A. narcotic pain medication
B. lumbar bracing
C. acetaminophen
D. gabepentin
55. Which physiologic factor in the elderly exacerbates orthostasis?
A. Decreased baroreceptor response
B. Decreased peripheral resestance
C. Decreased in arterial stiffness
D. Decreassed creatine clearance
56. A 70 year-old woman complains of acute localized mid back pain. She ha non-focal neurologic examination.
An anteroposterior and lateral thoracic spine x-ray confirms your clinical suspicion of an acute T8 compression
fracture. Which recommendation would best help her to reduce her risk of future fracture?
A. A weight reduction diet
B. Avoidance of tobacco use
C. Swimming laps 20-30 minutes daily
D. Isotonic abdominal strengthening program
57. A 65 year-old woman with a history of insulin dependent diabetes mellitus, gastroparesis, hypertension and
obesity had a right cortical ischemie infarct 7 days ago. The nurses note that she is having frequent small
urinary voids with a weak voiding stream. What bladder mechanism is most characteristic for this presentation?
A. Hyperactive detrusor with large volumebladder
B. Spastic detrusor activiry with normal sphincter
C. Small volume bladder with sphincter flaccidity
D. Flaccid detrusor with large volume bladder
58. A 50 year-old woman with secondary progress multiple sclerosis has mild cognitive deficits and severe lower-
limb spasticity. What the most effective treatment for this patient?
A. Range ot motion exercise
B. Warm water exercise
166
C. Intrathecal baclofen
D. Diazepam injection
59. A 45 y.o.woman develop thermoanestesia in a cape like distribution involving both upper extremities, with
preservvation of light touch sensation and propioception. These findings are suggestive of:
A. Syringomyelia
B. B 12 deficiensy
C. Amyotropic lateral sceloris
D. Occlusion of the anterior spinal artery
60. The following statements below are true for stroke syndromes of the middle cerebral arterycirculation
EXCEPT:
A. Pure sensory without weakness
B. Kontralateral hemiparesis and sensory loss
C. olved compare with lower extremity
D. Aphasia, if it affect superficial branch of middle cerebral artery on the left side
61. This clinical syndromes that effect the dorsal collum medialmeniscus system causing propioceptive disturbance
that affect position sense and balance is caused by:
A. Tabes dorsalis
B. Anterior cord syndromes
C. Idiopathic plexus brachialis lesion
D. Idipathic carpal tunnel syndrome
62. In stage four of Parkinson Disease the patient can still walk only on limited extent, because:
A. The sympptom are mild
B. Severe rigidity and bradikinesia
C. Mild impairment on equilibrium
D. Usually present with tremor on one side
63. The potential adverse effects of corticosteroid injection in EXCEPT:
A. Glucocorticoid-induced osteoprosis
B. Subcutaneous hypertrophy
C. Latrogenic joint infection
D. Tendon/ligament rupture
64. The following is the indication for pneumatic pumping:
A. Postmastectomy lymphedema
B. Acute deep venous thrombosis
C. Severely impaired sensation
D. Cellulitis
65. The following statement is WRONG about MWD:
A. The main ofenergytransfer is conversion
B. Can be used in fluid cavities (eye, bullae, effusion)
C. The frequencies for therapeutic use are 915 MHZ and 2456 MHZ
D. The average temperatures at the depth of 1-3 cm is about 41 degree Celcius
66. During exercise, cardiac output is augmented by
A. Sympathetic stimulation of resistance vessels
B. Decreased mean systemic arterial pressure
C. Increased ventricular contractility
D. Decreased e-diastolic volume
67. Which of the following statements regarding the rehabilitation programs of hospitalization for cardiovascular
illness operation is true?
A. After initial treatent of CAD syndrome is completed and the patient stabilized, having the patient sit
in an armchair,to minimize the loss of postural reflexes resulting from bed rest
B. Walking, at first with assistance shoul be encouraged even the patient is moderately symptomatic
C. Limited range ofmotion exercise are usually saveat this point, even in very unstable patients
D. The period of hospitalization is the optimal time to improve the patient’srisk factor profile

167
68. What exercise are recommended for persons with nonprogressive mild to moderate restrictive lung disease (VC
of 60% predicted)?
A. Pursed lip breathing
B. Rapid shallow breathing
C. Glossopharyngeal breathing
D. Inspiratory resistive exercise
69. The activity of climbing up stairs requires the following amount of METs:
A. 3 METs
B. 4 METs
C. 5 METs
D. 6 METs
70. The typical gait abnormality find in Spastic Diplegia Cerebral Palsy :
A. Weak hip flexion and ankle dorsiflexion
B. Supinated foot in stance phase
C. Overactive posterior tibialis
D. Scissoring gait pattern
71. A baby of 6 months old shows a symmetrical LMN weakness, no sensory deficits, proximal muscles weaker
than the distal, abdominal breathing. The strong possibility of diagnosis is:
A. Duchene’s Muscular Dystrophy
B. Kugelberg-Wellander disease
C. Werdnig-Hoffman disease
D. Poliomyelitis
72. One of your 4 year-old patients exhibits the following characteristic: distress over minor changes in
environment, echolalia, lack of awareness of the existence of feelings in others, nonparticiaption in simple
games. The most likely diagnosis is:
A. Autism
B. Cerebral Palsy
C. Hearing impairment
D. Mental Retardation
73. A 9 year-old boy has recently been diagnosed as having spinal muscular atrophy type III (Kugelberg Welander
Disease). You would expect:
A. Electromyographic changes consistent with a myopathic process
B. Rapid progression of the disease with death by age 20 years
C. Early involvement of distal upper extremity musculature
D. Ambulation to be preserved until age 25-30 years
74. The best rational choice of wheelchair for a C8 Tetraplegia ASIA A is:
A. Regular wheelchair with solid tyre
B. Mouth-operated electrical wheelchair
C. Regular wheelchair with aired/inflated tyre
D. Regular wheelchair with vertical bars on its handrims
75. In a hemiplegic, the function of a posterior stop in a double upright AFO with dual action adjustable ankle joint
extended steel shank to:
A. Improve knee stability at heel strike when the posterior stop is set in 15 degrees of dorsiflexior rather than
plantar flexion
B. Cause the ground reaction force line to fall posterior to the knee joint in mid stance
C. Assist in control of moderate to severe plantar flexor spasticity
D. Assist weak plantar flexors
76. 75 year-old woman falls and fractures her distal humerus. In the process, the ulnar nerve is damaged. She
presents 6 months later with weakness in the fourth and fifth digits, claw deformity, and loss of grip power.
Which of the following orthotic components could benefit this patient and, therefore, should be included in the
orthotic prescription?
A. Dorsal outrigger
B. Lumbrical bar
168
C. Opponens bar
D. C bar
77. A 23year-old man complaint of localized low back pain that is worse in the morning and improves as the day
progresses with activities. He has no radicular symptoms and nonfocal neurologic examination. Which finding
unequivocally supports the diagnosis of Ankylosing spondylitis?
A. A positive human leukocte antigen B27
B. An elevated esythrocyte sedimentation rate
C. Bilateral sacroiliitis on radiograph of the pelvis
D. Atlantoaxial subluxation on a radiograph of the cervical spine
78. Mechanical low back pain refers to:
A. Nondiscogenic pain
B. Pain which is discogenic in origin
C. Pain related specifically to soft tissue
D. Pain related to the posterior elements
79. Which medication is associated with osteoporosis?
A. Hydroclorothiazide
B. Coumadin
C. Phenytoin  penyerapan vitamin D
D. Diltiazem
80. Which statement is TRUE regarding calcitonin?
A. It is first line therapy for osteoporosis prevention and treatment
B. Its concomitant use with calcium is contraindicated
C. It is available in an oral form
D. It has analgesic properties
Ms. S, 21 year-old reffered to the rehabilitation ward with paraplegia due to T5 burst fraktur. The PPSW
stabilization was done 2 weeks ago and now was she medically stable. On physical examination you found that the
vital signs and general health status were within normal limit (number 81)
81. What should you examine when you have to determine wether the patient has a complete or incomplete lesion?
A. Anal sensation and voluntary anal contraction
B. Anocutaneal and bulbocavernous reflexes
C. Myotome and dermatome
D. Pathological reflexes
82. A 72 year-old man presented less ofmuscle bulk on volume aspect forearm and compaired flexion of the
distalinterphalangeal joints ofthe finger. Which muscle is responsible for these symptoms?
A. Flexor digitorum superficialis
B. Flexor digitorum profundus
C. Flexor pollicis longus
D. Flexor carpi radialis
83. Which disosrder is least likely to cause foot drop ?
A. Superficial peroneal neuropathy
B. Lower lumbar radiculopathy
C. Lumbar plexopathy
D. Sciatic neuropathy
84. Common clinical symptoms and signs of myasthenia gravis include:
A. Diplopia and/ptosis
B. Absent deep tendon reflexes
C. Abnormal sensory examination
D. Predominantly distal weakness
85. Some factors that influence muscle strength, power and endurance.
A. Length of muscle: longer has more tension
B. The amount of training: the longer the exercise increase more strength
C. Position of muscle insertion to the fulcrum: longer distance to insertion give more force
D. Age: peak performance at the age of 20 – 25 years old and decline 1% year after 25 years old

169
86. Which swallowing phase is primarily affected in dysphagia associated with Parkinson’s Disease.
A. Oral
B. Gastric
C. Pharyngeal
D. Esophageal
87. A 60 year-old malehas a right hemispheric infarction. He has muscle strength with grade 3 at the
shoulder,elbow and hand. He also has mildly decreased response to pinprick and propioception throughout the
left side. He use right hand for reading but always leaves some food untouched on the left side of his plate. The
most likely reason for this patient’s difficulty in eating is:
A. Motor apraxia
B. Feeding apraxia
C. Left unilateral neglect
D. Left hemisensory deficits
88. The initial exercise for s short of breath and frightened patient is:
A. Assessed tolerance test
B. Doing endurance exercise
C. Start with breathing control to achieve relaxation
D. Strengthening exercise for upper and lower extremities to avoid muscle atrophy
89. The beneficial of leg exercise in the training of patients with lung disease in pulmonary rehabilitation:
A. Decreased FVC
B. Decreased peak oxygen uptake VO2
C. No improvement in dyspnea and cough
D. Reduction in exercise lactic acidosis and ventilation after training
90. For patient with Congestive Heart Disease, which of following statements in accurate?
A. Most of the important resulting from regular exercise is within the myocardium
B. Exercise capacity is improved because of peripheral adaptations
C. These patient can never expect improved physical fitness
D. Complete bed rest is prescribed for these patients
91. Cardiac rehabilitation program encountera variety of heart including in below, EXCEPT:
A. 3rd degree AV Block
B. Post myocardial infarction
C. Post Coronary Bypass Graft
D. Chronic Congetive Heart Failure
92. The bestpredictor of community ambulation beyond chilhood in patients with myelomeningocele is:
A. Body mass index
B. Quadriceps strength
C. Early surgical closure of meningocele
D. Bowel and bladder continence
93. Arthrogryposis multiplex congetial most commonly is:
A. Myopathic in origin
B. Neurogenic in origin
C. Associated with early joint fustion
D. Associated with mental retardation
94. A child with a congenital tranverse radial limb deficiensy should have in inital prosthesis fit at what
developmental stage?
A. At the time of first sitting independently
B. At the time of starting kinder garden
C. At the time of initially walking
D. At soon as possible after birth
95. Management of pes planus grade 1 at age less than 1 years old done by
A. Obervation
B. Thomas heel
C. Arches support
170
D. Chukka type shoes
96. Early symptoms of ankylosing spondylitis in a young adult is:
A. Cervical pain
B. Low back pain
C. Upper thoracal pain
D. Lower extremity pain
97. The test for acromioclavicular (AC) joint and labial abnormalities is:
A. O’Brian Test
B. Speed’s Test
C. Yergason’s Test
D. Horizontal adduction Test
98. American College of Rheumatology Classification criteria for systemic lupus erythematosus include all the
following EXCEPT:
A. Raynaud’s phenomenon
B. Photosensitivity
C. Malar rash
D. Arthritis
99. Plantar fascitis.
A. Is common in patients whose occupation requires long period of sitting
B. There is deep tenderness of the anterolateral aspect of the calcaneus
C. The condition is more prevalent in people with supinated feet
D. The treatment is stretching of the heel cord
100.Treatment for rotator cuff tendonitis include all of the following, EXCEPT:
A. Immobilization
B. Restoration of range of motion
C. Control of pain and inflammation
D. Normalization of strength and dynamic muscle control

171
ANSWER:
A. IF NUMBERS 1,2,3 ARE CORRECT
B. IF NUMBERS 1,3 ARE CORRECT
C. IF NUMBERS 2,4 ARE CORRECT
D. IF ONLY NUMBER 4 IS CORRECT
E. IF ALL NYMBERS ARE CORRECT
101.The moderate to severe Traumatic Brain injury patients will have move evident deficits, usually from the
beginning stress. These include: C/E
1. Memory
2. Attention
3. Concentration
4. Language processing
102.Rating of perceived exertion which is used to monitor exercise responses is (are) strongly associated with: A
1. Oxygen comsumption
2. Lactic acid level
3. Breathing rate
4. Blood pressure
103.The precription of exercise for patient with recurrent low back pain should achieve at least: E
1. Improve low back flexibility
2. Improved body mechanics in all activities
3. Improved posture with minimized lumbar lordosis
4. Improved strength of abdominal and hip extensor muscles
104.Common cause of shin splints is (are): A
1. Sudden increases in distance or speed running
2. Running on slopes or uneven surfaces
3. Inappropiate footwear
4. Flat arches of the feet
105.The following factor(s) contributing towards the occurence of muscle rupture if the muscle may have been: B / E
1. Inadequate training or lack of warm-up
2. Weakened by previous injury
3. Overstrained or fatiqued
4. Scar muscle formation
106.The goal or orthosis precription in arthritis is (are)? C
1. to prevent sclerotic process
2. to reduce joint load
3. to reduce joint stifness
4. to reduce pain
107.Vaulting is one common deviations in tranfemoral prosthetic. It appears as rising up on thesound forefoot during
mid stand in an effort to enhance prosthetic swing limb clrearance. It may result from: A
1. Long prosthesis
2. Excessive knee friction
3. Fear of letting the knee flex
4. Excessively abducted socket
108.Lower extremity orthosis for pressure distribution are: A
1. Patellar Tendon Bearing (PTB) Orthosis
2. Ischial Weight – Bearing Orthosis
3. Pattern Bottom Orthosis
4. Tone-Inhibitory Orthosis
109.Effects of aging on function, for strength and endurance: C
1. Increased in muscle mass
2. Decreased number of motor neurons
3. Decreased accumulation of oxidative stress
4. Selective atrophy of type II fast-twitch muscle fiber

172
110.A healthy elderly person may display the following gait deviations: B
1. smaller step length
2. increased arm swing
3. slower ambulation velocity
4. axtension of the hips and knees
111.Which the following statement(s) about shortwave diathermy (SWD) is/are correct? E
1. Is the therapeutic application of high radiofrequency electrical current
2. The tissue temperature should be elevated to range of 40 – 45o C
3. The treatment time is usually 20 -30 minutes
4. Is deep heaat modality
112.Clinical usesof NMES (Neuromuscular Electrical Stimulation) are: E
1. Strengthening muscles and maintaning muscle mass after immobilization
2. Shoulder subluxation in hemiplegic limb
3. Provides cardiovascular conditioning
4. Spasticity management
113.The general goals of cryotherapy is (are): A
1. analgesia
2. muscle relaxation
3. decreased spasticity
4. increased collagen extensibility
114.The exercise stress test may be valuable in: A
1. Making the diagnosis of the ischemic heart disease
2. Establishing an individualized exercise program for the patient with cardiac disease
3. Following the progress of patient involved in cardiovascular rehabilitation programs
4. Determining how much exercise to precribe for the hospitalized patient with an acutemyocardial infarction
115.Goals of the physical therapy program in the patient with chronicobstructive airway disease include: A
1. Overall physical reconditioning
2. Improving the eficiency of the breathing patterns
3. Facilitating drainage of excessivebronchial secretions
4. Reverse the pulmonary function with spesific breathing exercise
116.The patient with restrictive pulmonary disease may be seen by the physiatrist. What right of exercise be useful for
this patient? A / B
1. Slowly inhale until the lungs are fully expanded
2. Intermmitent positive pressure breathing  ppok ?
3. Deep breathing exercise
4. Forcefully exhale
117.In quite standing, the ground reaction force (GRF) are located in: C
1. Posterior to knee
2. Anterior to ankle
3. Anterior to hip
4. Posterior to hip
118.The function of the following extrinsic muscle of the hand is (are) to extend the finger: A
1. Extensor digititorum
2. Extensor indicis propius
3. Extensor digititorum communis
4. Extensor carpi radialis longus
119.The probable cause(s) of steppage gait is (are): A
1. Severely weak dorsiflexors
2. Plantar flexors spasticity
3. Equinus deformity
4. Weak plantarflexors
120.When a child is diagnosed as cerebral palsied, the first question asked by the parents is always “will he walk?”. To
answer these question the following test were done: E
173
1. Asymmetrical tonic neck reflex
2. Symmetrical tonic neck reflex
3. Foot palcement reaction
4. Parachute reaction
121.Children in this spastic subgroup typically manifest signs of upper motor neuron involment. This may include: E
1. Hyperreflexia
2. Persitent primitive reflexes
3. Over flow reflexes such as crossed adduction
4. Extensor Babinski response/abnormalafter 2 years of age
122.Cardnial clinical sign of Werdnig Hoffman Disease (severe SMA) are: E
1. Severe limb and axial weakness
2. Leg in frog position
3. Poor head control
4. Weak cry
123.The advantages of Standard Concentric (Coaxial) needle electrode is: A
1. Fixed location from reference
2. Standardized exposed area
3. Used for quantitive EMG
4. Less artefact
124.Muscle tension is monitored primarly by: D
1. Muscle spindle
2. Motor endplate
3. Exrtrafusal fibers
4. Golgi tendon organ
125.In a totally denervated of all musles due to a severe Brachial plexus injury, this can be categorized aspre-ganglionic
lesions if we found the following data in EMG examination: B
1. Sensory NCV: good response
2. Motor NCV: good response
3. EMG: total denervation
4. EMG: partial denervation
126.These are the rehabilitation programs for leprosy: E
1. Oil massage
2. Active exercise
3. Soaking the feet in water
4. Protection of the hands and feet
127.The first thing to do in the management of mild spasticity is: D
1. Give oral diazepam
2. Give superficial heating
3. Give nerve block injection
4. Remove all the possible of noxious stimuli
128.The damage to the superior branch of the facial nerve in leprosy will increase the chances of A
1. blindness
2. exposure keraltitis
3. damage to the cornea
4. sensory lossof the face
129.Patients with carpal tunnel syndrome often complain of: D
1. Hand weakness more frequently than pain
2. Numbness over the dorsal aspect of the fingers
3. Sensory changes the dorsal aspect of the finger
4. Nocturnal symptoms of numbness and tingling
130.Which following criteris(s) must be present for at least 6 weeks for diagnosis of Rheumatoid Arthritis? A
1. Swelling in three or more joint areas
2. Symmetric joint swelling
174
3. Swelling in hand joint
4. Rheumatoid nodule
131.Diagnosis of myofascial trigger point: E
1. Exquisite spot ternderness
2. Local twitch response
3. Pain recognition
4. Taut band
132.The following is gold standard for osteoporosis therapy: B
1. Calcium
2. Calcitonin
3. Biphosphonate
4. Selective estrogen receptor modulator
133.Adhesive capsulitis E
1. Associated with diabetes mellitus, inflammatory arthritis, prolonged immobilization
2. More common in women than men
3. Usually an idiophatic condition
4. Divided into 3 stages
A woman, aged 32 years oldhad ACL (anterior cruciatum ligament) reconstruction 1 week ago. She arrives at her first
outpatient visit with moderate edema and ankle. She states that she has been compliant with weight
bearing and uses her crutches and brace as instructed. (Number 134 and 135)
134.To reduce her edema, what kind of exercise program you give for this patient? A
1. Elevate leg above heart level 20 minutes four times a day
2. Ankle pumps 10 repetitions every minute while elevating her leg
3. Quadriceps and hamstring isometric contraction using closed chain enviroment
4. Prone position with the involved leg hanging over the edge of the table or bed
135.What are your goal for this patient in this phase (1 – 4 weeks post surgery)? B
1. Full knee extension
2. Regain cardiovascular capacity
3. Increased active and passive ROM
4. Ambulation without an assistive device
136.The characteristic changes postur in elderly: B
1. Stooped posture
2. Backward head posture
3. Increased postural sway
4. Decreased postural instability
137.Independent of bone mineral density, in elderly women, which factor(s) contributes to an increased risk of fracture
secondary to fall? D
1. Morbid obesity
2. Use of Coumadin
3. Cafein comsumption
4. Poor visual acuity
138.Man, 55year-old, weight 85 kg, height 165 cm. He got myocard 1 month ago. The fasting glucose level
250,total cholesterol 400,no hypertension. Factor that should be considered to prescribe cardiac rehabilitation
program: C
1. Avoid any kind of exercise
2. Assessed functional capacity with stress test
3. Limitation of activity daily living and walking
4. Reduce the risk factor for 2nd cardiac disease attack
139.Common treadmill protocols for cardiovascular graded exercise tolerance testing D
1. Naughton-Balke
2. Astrand
3. Cornell
4. Bruce

175
140.Among the following, which activity wight be especially stressful for patient with cardiac disease? D
1. Lower extremity cycle ergometry
2. Upper extermity arm ergometry
3. Leg raises exercise
4. Bridging exercise
141.The following is/are the characteristic(s) of Becker’s Muscular Dystrophy: E
1. This form is similar ti Duchene’s dystrophy in terms of distribution of weakness
2. It has later onset than Duchene’s muscular dystrophy
3. It is more benign than Duchene’s muscular dystrophy
4. It is also an X linked inherited disorder
A patient 5 years old was diagnoses Spastic Cerebral palsy Diplegic type e.c. prenatal, come to the Rehabilitation Clinic
with compaints could not walk and straight, both legs crossed and tiptoe n standing position (number 142)
142.What assessment is/are needed to determine spasticity of the hip: A / E
1. Phelps Gracilis Test
2. Duncan Elly Test
3. Thomas Test
4. Tardieu Test
143.The following factor(s) is/are early treatment(s) of CTEV: D
1. Dennis-Brown splint
2. Outflare orthopedic shoes
3. Surgical soft tissue release
4. Stretching and serial plaster correction
144.The following are the signs of autonomic dysreflexia in a T4 Paraplegia ASIA: A
1. Pounding headache
2. Nasal congestion
3. Hypertension
4. Tachycardia
145.The functional outcome of L2 Paraplegia AIS/Frankel A is/are: E
1. Independent ambulation using bilateral axillary crutches and KAFO
2. Independent bladder and bowel activities
3. Require no wheelchair for ambulation
4. Independent in pressure sore relief
146.Early management of acute whilplash injury is/are: A
1. Rest
2. Soft cervical collar
3. Analgesic & muscle relaxant
4. Range of motion and Neck Calliet’s exercise
147.The following inclueded seronegative spondyloarthropathies: B
1. Ankylosing spondylitis
2. Rheumatoid arthritis
3. Psoriatic arthritis
4. Spondylitis
148.Indication/s of using orthotic in rheumatoid arthritis is/are: A
1. Decreased joint motion-stabilization
2. Decrease pain and inflammation
3. Reduced weight through joint
4. Stop the deformities
149.De Quervan’s disease is a stenozing tenosynovitis of the following muscles:
1. Abductor pollicis brevis
2. Abductor pollicis longus
3. Extensor pollicis longus
4. Extensor pollicis brevis
150.The principle(s) of Burn Rehabilitation is/are:

176
1. Splints are useful for passive positioning
2. Hydrotherapy
3. Lubrication
4. USD

177
NATIONAL BOARD EXAMINATION
PHYSICAL MEDICINE AND REHABILITATION
DECEMBER 13, 2013
CHOOSE THE MOST APPROPIATE ANSWER

1. Which of the following muscles is the lateral rotator of the joint? E


A. m. Tensor fascia latae
B. m gluteus minimus
C. m gluteus medius
D. posoas major
E. m priformis
2. The muscle that has the important role in push-off is D
A. Iliopsoas
B. Hamstring
C. Quadriceps
D. Triceps
E. Tibialis anterior
3. The most characteristic of hemiplegic gait: E
A. Flexion of the knee
B. Adduction of the hip
C. Abduction of the hip
D. Dorsi flexion of the anke’circumduction of the leg
E.
4. Return to sport phase focused to the component: C
A. Strengthening all muscles
B. Stretching exercise
C. Sport specific skill
D. Balance training
E. ROM exercise

5. Which biomechanical factors has NOT implicated in athlete risk for recurrent stress fracture? C
A. Excessive supination of the foot
B. Leg length discrepancy
C. Excessive hip rotation
D. Forefoot valgus
E. Flat foot
6. What is the most proper modality should be given in partial thickness tear of supraspinatus tendon. E
A. TENS
B. Electrostimulation
C. Ultrasound Diathermy
D. Short Wave Diathermy
E. Low Level LASER Therapy
7. The one following is NOT of the prevention programs for Rotator Cuff tendinopathy: C
A. Maintain the dynamic control ratio of shoulder external and internal rotators
B. Suggestion to coach to make proper training periodization
C. Hotpack before and after training
D. Warming up and cooling down
E. Faciliate proper resting period
178
8. Ruptur of the profundus tendon (flexor digitorum) which most commonly due to trauma as seen on athlete (foot
ball,. Wrestling) is said: E
A. Bennet’s fracture
B. Rolando’s fracture
C. Boxer’s Fracture
D. Mallet Finger
E. Jersey Finger

9. The most common method of exercise based on neuro developmental approach for CP patient is: E
A. Kabat method
B. Voyta method
C. Phelps method
D. Deaver method
E. Bobath method
10. The following is true about DeLorme technique, EXCEPT: E
A. Used to strengthen the muscle
B. Is also called progessive resistive exercise
C. The subject is tested to determine the “10 repetition maximum”
D. The weights would progressively increase from week to week
E. The person lift at various percentages of the 10 RM, starting at 100%
11. The following is true about plyometric exercise: B
A. It is have no risk to injury
B. Functional type of exercise
C. The principle is sloe stretching
D. Is useful to many neurologic impairment
E. Exercise using this principle including balance training
12. The Scoliosis Research Society (SRS) has defined scoliosis as: A
A. Any curve which is greater or equal to 10 degrees with or without a rotatory component in frontal plane
B. Any curve which is greater or equal to 15 degrees with or without a rotatory component in frontal plane
C. Any curve which is greater or equal to 15 degrees with or without a rotatory component in sagital plane
D. Any curve which is great or equal to degrees with component in frontal plane
E. Any curve which is great or equal to 15 degrees with a rotatory component in frontal plane

13. The following is medical rehabilitation program after hip replacement (THR): D
A. Prevent hip abduction
B. Prevent hip flexion above 100o
C. Ambulation training after 4 weeks
D. Early ambulation PWB (partial weght bearing) with walker
E. Ambulation NWB (Non weight bearing) with double crutches
14. All of the following statement are true regarding the carpal tunnel. EXCEPT C
A. A very narrow tunnel
B. Covered by the transverse carpal ligament
C. The base is made by the metacarpal bones
D. Osteoarthritis will be naarowed the carpal tunnel
E. The median nerve is the only nerve passing through this tunnel
15. Test for the tensor fascia lata or iliotibial band tightness is: A
A. Ober test

179
B. Ely’s test
C. Thomas’s test
D. Patrick’s test
E. Tredenlenburg’s test
16. Patient with acute neck sprain usually is treated with: C
A. Strengthening neck muscles
B. Tricyclic antidepressan
C. Neck soft collar
D. Deep heating
E. Bed rest
17. The following statement are true in chronic low back pain, EXCEPT: E
A. Another common type is caused by myofascial pain
B. It is often caused by poor muscle tine and postural strain
C. Lumbarization and sacralization can caused chronic LBP
D. Patient with Multiple Myeloma is often presenting chronic LBP
E. Spondyloysis is a unilateral of bilateral defect in the vertebral isthmus with vertebral slippage is another
etiologic factor of chronic LBP

18. The following is the risk factor of secondary osteoporosis, EXCEPT: C


A. Immobilization
B. Organ transplant
C. Hyparathyroidism
D. Proton pump inhibitors
E. Glucocorticoid treatment
19. The following is true about osteoarthritis (OA): E
A. Osteoarthritis is a systemic bone disorder
B. Genetic predispositon is not the risk of OA
C. The prevelence of OA is higher in men than women
D. Characterized by non progressive articular cartilage damage
E. Knee OA is heterogeneous disease. involving cartilage. subchondral bone. menisci and periarticular soft
tissues
20. Medical rehabilitation after Total Knee Replacement: B
A. Isotonic quadriceps exercise in the first week after surgery
B. The goal after 2 weeks is 0-90o ROM of the affected knee
C. Early mobilization is not recommended
D. The use of CPM until 4 weeks
E. Gait training after 3 weeks
21. Bouncing or exaggerated plantar flexion on midstance through toe off can caused by: E
A. Metatarsalgia
B. Hallus rigidus
C. Achilles tendon rupture
D. Gastroc-soleus weakness
E. Achilles tendon contracture
22. For short very short transradial amputation level A
A. Limit elbow flexion strength and elbow range of motion
B. Optimal for body powered prosthetic restoration
C. Major advantage due to cosmetic problems

180
D. Limit action abduction shoulder range of motion
E. Ideal for elbow suspension
23. Spinal orthotic which is choice for maximum immobilization of the spine is : A
A. Plastic body jacket
B. Flexion Control Orthosis (Jewell Brace)
C. Flexion Extension Control Orthosis (Taylor Brace)
D. Flexion Extension Lateral- Rotary Orthosis (Cowhorn Orthosis)
E. Flexion Extension-Lateral Control Orthosis (Kinght- Taylor Brace)
24. According to the standard of the American Spinal Injury Association, what is the key area for testing pin-prick
sensation to determine the sensory level after T4 spinal cord injury ? C
A. Umbilikus
B. Acromion
C. Nipple line
D. Apex of axilla
E. Lateral aspect of the elbow
25. Mrs Siti 69-year-old suddenly has trouble in knowing how far or near an object is to her body and also very poor in
decision making. It is possible that she suffers: A
A. Right brain lesion
B. Mid nrain lesion
C. Left brain lesion
D. Cerebellar brain lesion
E. Pons lesion
26. A hemiplegic patient who has trouble in using the correct word for objects, places and events might be suffering
from: D
A. Primary progressive aphasia
B. Expensive aphasia
C. Receptive aphasia
D. Anomic aphasia
E. Global aphasia

27. A patient appears to have good strength in the hand and can use it for gross activities automatically but cannot
perform skilled movement. This form of disability is called : E
A. Ataxia
B. Agraphia
C. Ideational apraxia
D. Constructional apraxia
E. Kinetic (motor) apraxia
28. Predictors of hand movement and function after stroke : E
A. Arm pronation
B. Arm supination
C. Lower arm spasticity
D. Paresthesia of upper arm
E. Bhoulder shrug and synergistic hand movement
29. Which of the following is NOT the characteristic of nerve radial lesion E
A. Weakness of extensor carpi radialis
B. Weakness of extensor carpi ulnaris
C. Weakness of finger extensor

181
D. Drop hand
E. Ape hand
30. Traumatic SCI with neurological deficit ipsilateral propioceptive and a spastic paralysis develops in the muscle
innervated ipsilaterally below the level of injury, with contralateral loss of sensitivity to pin and temperature is
indicated as : D
A. Central Cord Syndrome
B. Anterior Cord Syndrome
C. Posterior Cord Syndrome
D. Brown-Sequad Syndrome
E. Conus Medullaris Syndrome

31. What is the definition of the 4th degree Sunderland’s classification of the peripheral nerve injury? E
A. Disruption of the axon
B. Transection of the entire trunk
C. Disruption of the axon and endonerium
D. Local conduction block with minimal structural disruption
E. Disruption of the axon, endoneurium, and perineurium, the epineural is
32. All of the following factors influence the strength of muscle contraction, EXCEPT: A
A. The muscle volume
B. The amount of calcium and ATP
C. The amount of actin and myosin
D. The number of motor units which are active
E. The starting point point in which muscle start to contract
33. Loss of the Achilles tendon reflex results from lesion of the : C
A. L4 segment of the spinal cord
B. L5 segment of the spinal cord
C. S1 segment of the spinal cord
D. S2 segment of the spinal cord
E. S3 segment of the spinal cord
34. The annulospiral ending is the afferent fibers coming from: A
A. Nuclear bag
B. Nuclear chain
C. Intrafusal fibers
D. Extrafusal fibers
E. Golgi tendon organ

35. A 35-year-old man paraplegic who develops swelling and erythema in the left lower extremity. Non invasive
venous studies reveals a deep venous thrombosis extending into the thigh. He receive therapeutic anticoagulation
with intravenous heparin. You would be give treatment program to the affected extremity : A
A. Bed rest
B. Cold pack
C. Superficial
D. Ankle stretching exercise
E. Intermittent pneumatic compression devices
36. The following is the contraindication for treatment with deep heating D
A. Myofascial pain
B. Tendinopathy
C. Trigger finger
182
D. Hemarthros
E. Synovitis
37. Miss A 35-year-old came to Rehabilitation Clinic with the chief complain acute neck pain. No history of injury, no
neurologic impairment. There are paraspinal muscle spasm and pain on palpation, restriction of cervical ROM. You
plan to make cervical x-ray. What modality will you give ? D
A. USD
B. SWD
C. Hotpack
D. Coldpack
E. Low Power Therapy
38. Which is closed to sensory integration test? B
A. Decreasing adaption to environment
B. Sequential finger test
C. Turn to look behind
D. Same with praxis
E. Sit to stand

39. The initial for CTEV is : A


A. Stretching and serial plaster correction
B. Use of outflare orthopedic
C. Use of dennis brown splint
D. Use ulthrasound diathermy
E. Surgery
40. Spina bifida is considered a primary failure of neurulation and the spinal cord defect associated with the anatomic
level of the lesion and neurologic level of functional involvement. A boy suffered from spina bifida L4-L5
segment, the preserved muscle function is : A
A. Iliopsoas muscles
B. Hamstring muscles
C. Quadriceps muscles
D. Foot intrinsic muscles
E. Gluteus maximus muscle
41. Children two years of age have stereotypic and ritualistic behavior with impairment in social interaction,
communication and imaginative play. The diagnose is : E
A. Fragile x syndrome
B. Mental reterdation
C. Down syndrome
D. ADHD
E. Autism
42. That above cases have also clinical features : E
A. Flat feet
B. In attention
C. Cognitive impairment
D. Executive ysfunction
E. May have a lack of eyes contact and dificulty playing with others

183
43. This reflex is the best elicited by a sudden dropping of the baby’s head in relation to its trunk E
A. Vertical Suspension
B. Righting Reflex
C. Extensor thrust
D. Landau Reflex
E. Moro Reflex
44. The methode frequently use on CP which bases treatment on activation of postural development and equilibrium
reaction to guide normal development is : C
A. Bobath Methode
B. Deaver methode
C. Vojta methode
D. Phelp methode
E. PNF methode
45. A 56 yo female suffered from obesity (BMI 31) and hypertension (BP 140/85). What the best exercise regimen for
this woman? E
A. Continue walking exercise while blood pressure exceed 220/100, because the exercise will decreased the blood
pressure
B. Arm curl with dumbbell for biceps strengthening, with intensity 80% of 1RM, 8 repetitions, 3 bouts/session, 2-
3x/week
C. Leg extension with quadrriceps bench, with intensity 80% of 1RM, 8 repetitions, 3 bouts/session, 2-3x/week
D. Fast walking exercise, with intensity 40-70% of heart rate reserve, 30-60 minutes/day, 3-5x/week
E. Static cycle exercise, with intensity 40-70% of heart rate reserve, 30-60 minutes/day, 3-5x/week
46. The following is causes of restrictive pulmonary disease, EXCEPT : B
A. Guillian Barre Syndrome
B. Chronic bronchitis
C. Myasthenia gravis
D. Pleural effusion
E. Obesity
47. For chronic obstructive pulmonary disease, six minutes walking test with submaximal exercise have all the benefit
below, EXCEPT: C
A. To detect exercise intolerance
B. To know the functional limitation
C. To detect the improvment of lung limitation
D. To to evaluate reconditioning exercise progam
E. To know how distance the patient can walk in 6 minutes
48. The risk exercise induced bronchospasm in COPD, mostly caused by: E
A. Graded high intensity exercise
B. Low intensity interval
C. Low intensity continuous exercise
D. Moderate intensity interval exercise
E. 24-26%Celcius room temperature with high humidity
49. What is the relative oxygen consumption (ml/kg/minute) equivalent to 2.5 METs? D
A. 3.50 ml/kg/min
B. 5.25 ml/kg/min
C. 7.00 ml/kg/min
D. 8.75 ml/kg/min
E. 10.3 ml/kg/min

184
50. If the body weight 50 kg, what absolute oxygen consumption (ml/minte) equivalent to 2.5 METs? D
A. 30.625 ml/min
B. 43.750 ml/min
C. 306.25 ml/min
D. 437.50 ml/min
E. 4375.o ml/min
51. Exercise training for patient with peripheral artery disease in the erderly : B
A. Walking with intensity, progression from 50% of peak exercise
B. Intermittent walking to near miimal, claudication pain
C. Isotonic strengthening exercise of lower extremity
D. Swimming one hour
E. Bicycling 30 minutes
52. Functional assessment of instability in the order faller is? E
A. Vision
B. Strength
C. Flexibility
D. Propioception
E. Standing reach
53. The most common cause of falls in the erderly is/are: E
A. Vertigo
B. Weakness
C. Hearing impairment
D. Postural hypotension
E. Environmental hazard
54. The following is/are the gold standard(s) for diagnosis of osteoporosis: B
A. Quantitative Computed Tomography
B. Dual Energy X ray Absorbsiometry
C. Ultrasound densitometry
D. CT scan
E. X-ray
55. One of the common low back injuries among Golfers is disc herniation. The mechanism of the incidence is: D
A. Overuse
B. Muscle cramp
C. Muscle imbalance
D. Rotation with axial loading
E. Axial load with forward flexion

56. This disorder is usually seen in preadolescent athletes who participate in activities such as jumping or running.this
disorders a result of reccuring micro trauma from the quadriceps contracting: E
A. Patella fracture
B. Patellar tendinosis
C. Tibial plateu fracture
D. Pes anserinus bursitis
E. Osgood-Schaltter disease
57. A 18 y.o. basketball player suffered from chronic ankle instability. The statement below is EXCEPT: E
A. Mechanical ankle instability can be caused by anterior talofibular ligament laxity
B. Chronic ankle instability consist of mechanical and functional ankle instability
185
C. This condition needs structural and comprehensive rehabilitation program
D. Functional ankle instability reflects the propriceptive incompetence
E. This condition always needs surgery approach
58. The management of acute ACL injury is: D
A. Isotonic exercise
B. Balance training
C. Bicycle exercise
D. Cryotherapy
E. Knee brace
59. Which of the following is true regarding in the geriatric population? D
A. Most falls are considered accidental,rather than related to underlying diseases or functional impairments
B. The timed “Get up & Go” test is an appropiate way to assess someone who presents with frequent falls
C. Dynamic balance training activities such as Tai Chi have no effect on fall or fear of failing
D. Fall risk assessment should be evaluate for both intrinsic and extrinsic risk factors
E. Joint problem is the most cause of fall in elderly
60. The main goal of exercise program for geriatric patient is to: E
A. Reeduce depression
B. Increase endurance
C. Increase flexibility
D. Increase agility
E. Reduce falls
61. A 75 year-old sedentary man with a history of stable hypertension, type II diabetes mellitus, and mild sensory
neuropathy who is currently a nonsmoker wishes to begin an exercise program. What would be the safest initial
program? D
A. Walking with no competition
B. Treadmill walking at 1.5mph on level surface
C. Stair stopper for 15minutes without resistance
D. Lifting 1- to 3-pound weights overhead while seated
E. Machine-based quadriceps extensions at 80% maximum weight
62. Which factor improves bowel transit time in persons with chronic constipation? B
A. Fluid restriction
B. Bulking agents
C. Low fiber diet
D. Drink water
E. Bedrest
63. A pain is considered chronic if it has been present for at least: B
A. 1-2 months
B. 3-6 months
C. 7-9 months
D. 10-11 months
E. 12-24 months
64. The mechanism of Botulinum Toxin in the treatment of spasticity is by: E
A. Blocking the nerve fibers
B. Blocking the release of ATP
C. Blocking the muscle spindle
D. Blocking the release of calcium ions
E. Blocking the release of acety choline
186
65. The following statement is NOT true about endurance exercise response in adult human: B
A. Steady-state exercise reflects a balance between the energy required by working the muscles and the rate of
ATP production through aerobic metabolism
B. The greatest improvement in VO2max occurs when exercise involves the use of small muscle groups
C. A linear increase in systolic blood pressure occurs with increasing exercise intensity
D. Long-term exercise training increased MVO2 during rest and submaximal exercise
E. Arteriovenous oxygen difference is greater at maximal exercise
66. The nociceptive system starts from the following organ: E
A. Golgi
B. Ruffini
C. Paccini
D. Krause
E. Free nerve endings
67. Following surgery a patient is asked to perform bottle blowing exercises to maintain pulmonary function. Which of
the following is the most common side effect of the bottle blowing exercise E
A. A V Block
B. Atrial fibrillation
C. Rupture of the trachea
D. Pulmonary embolism
E. Premature ventricular beats
68. The vital capacity is the sum total of the: D
A. Tidal volume plus rasidual volume
B. Inspiratory capacity and functional reduce capacity
C. Inspiratory capacity,tidalvolum and expiratory reserve volume
D. Inspiratory reserve volume, tidal volume and expiratory reserve volume
E. Inspiratory reserve volume,functional residual capacity and tidal volume
69. What is the best test of cardiovascular function for patient who is obese, has claudication in his legs, has limited
mobility from diabetic neuropathy? B
A. Resting echocardiogram
B. Dipyridamol or dobutamine testing
C. Continuos submaximal cycle ergometer
D. Continuos submaximal treadmill exercise test
E. Discontinuous treadmill exercise test
70. A good measure of lung obstruction in COPD patient is: D
A. Residual volume
B. Forced vital capacity
C. Maximal voluntary ventilation
D. Forced expiratory volume at one second
E. Forced expiratory volume at one minute
71. COPD patient who has a measured forced vital capacity of 3.5 L and was able to expel 3.1 L within 1 second has
FEVI of: C
A. FEVI of 68 %
B. FEVI of 98%
C. FEVI of 89%
D. A restrictive lung disease
E. An obstructive lung disease
72. When the respiratory muscles are relaxed the lungs are at: E

187
A. Residual Volume (RV)
B. Total capacity (TLC)
C. Inspiratory reserve volume (IRV)
D. Expiratory reserve volume (ERV)
E. Functional residual capacity (FRC)

73. The gait in spastic diplegia cerebral palsy, with increasing muscle tone of hip adductor and knee extensor, feet
inverted position is : D
A. Hemiplegic
B. Lumpig gait
C. Spastic gait
D. Scissor gait
E. Ataxic gait

74. All of the following muscle will abduct the shoulder joint in the frontal plane together with deltoid muscle,
EXCEPT:B
A. Teres minor
B. Teres mayor
C. Subscapular
D. Infraspinatus
E. Supraspinatus
75. The normal cadence (steps per minutes) of the adults male ranges between : C
A. 70-140
B. 70-150
C. 70-130
D. 60-130
E. 50-130
76. The child has chief complain of bowlegs that are gradually intercalary transverse deficit which is the: C
A. CETV
B. Clubfoot
C. Tibia vara
D. Knock knees
E. Genu recurvarium

77. One of the following congenital limb deficiencies in children intercalary transive deficit, which is the: C
A. Phocomella
B. Absence of the hand
C. Absence of the radius
D. Congenital below elbow amputation
78. A reliable and valid outcome measure designed to evaluate movement patterns and hand function in children with
cerebral palsy is : C
A. Assisting Hand Assessment (AHA)
B. The Gross Motor Function Measure (GMFM)
C. The Quality of Upper Extremity Skills Test (QUEST)
D. The pediatric Evaluation of Disability Inventory (PEDI)
E. The Functional Independence Measure for Children (WeeFIM)
79. Erb’s Palsy is lesion on: C
A. C3-4 brachial plexus

188
B. C4-5 brachial plexus
C. C5-6 brachial plexus
D. C6-7 brachial plexus
E. C7-Th1 brachial plexus
80. The following are the characteristic of Werdnig-Hoffmann Disease, EXCEPT: A
A. The prognosis is good
B. Affected in the motor neuron
C. Is an autosomal recessive inheritance disease
D. The infants have a floppy, limp, froglike posture
E. A muscle biospy specimen shows findings typical of neurogenic injury
81. The effect continuous lumbar traction : D
A. Stretching the nerve root
B. Mobilizing patient in bcd
C. Separaitng the vertebral bodies
D. Stretching the paralumbar
E. Widening the intervertebral foramina
82. Heat therapy through convention method could be generated by the following modality: B
A. USD
B. Whirpool
C. Low laser therapy
D. Shockwave therapy
E. Ultraviolet therapy
83. The following modality has electrical stimulation is patient with: A
A. Parkinson
B. Cerebral Palsy
C. Spinal Cord Injury
D. Infant with spina bifida
E. Duchenne Muscular Dystrophia
84. The indication of functional Electrical Stimulation is patient with: C
A. Parkinson
B. Cerebral Palsy
C. Spinal Cord Injury
D. Infant with spina bifida
E. Duchene Muscular Dystrophia
85. Assessment of the cognitive functioning on patient with Traumatic Brain Injury should include the following areas,
EXCEPT: D
A. Insight and awareness
B. Perception
C. Attention
D. Initation
E. Memory
86. Exercise than doesn’t use neuroplasticity principle is: C
A. Body weight-supported treadmill training
B. Constrain induced movemnet therapy
C. Traditional therapy method
D. Virtual reality training
E. Robotic training

189
87. A male 23 years old suffered traffic accident 3 months ago. He has stabilized fracture at vertebrae Th XII. The
motor impaired at LA level both side with the MMT l/l and no spasticity. To increase the walking ability. What
should you do? E
A. Body weight-support partial training
B. Strengthening the impaired muscles
C. Functional electrical stimulation
D. Constraint-induced stimulation
E. Give double AFO
88. Benefit of moderate spasticity in stroke patient, EXCEPT: D
A. Help for transfer or ambulation
B. Prevent of disuse atrophy
C. Prevent of osteoporosis
D. Prevent of contracture
E. Prevent of DVT
89. The most effective treatment or severe generalized spasticity in persons with SCI is: D
A. Diazepam
B. ROM exercises
C. Botulinum toxin
D. Baclofen intrathecally
E. Stretching of spasticity muscles
90. Suction sockets can best be used by: C
A. Pediatric amputees
B. Geriatric amputees
C. Young traumatic amputees
D. Heart failure with new amputation
E. Any ages group amputated for diabetic gangrene

91. For short and very short transradial amputation level, the statement below is true A
A. Limit elbow flexion strength and elbow range of motion
B. Optimal for body powered prosthetic restoration
C. Major advantage shoulder range of motion
D. Limit abduction shoulder range of motion
E. Ideal for elbow suspension
92. The right answer about Chopart amputation D
A. At the tarso-metatarsal junction
B. At the talo-navicular,cuboid, talus adn calcaneous amputation
C. A horizontal calcaneal amputation, all tarsals removed except calcaneus/talus
D. At midtarsal, talo-navicular, calcaneo-cuboid amputatuion, talus and calcaneus bones remain
E. A vertical calcaneal amputation, the lower articular surfaces of the tibia/fibula are sawn through
93. These are the effect of treadmill training in Parkinson patients, EXCEPT: A
A. It reduces tremor
B. It decreased rigidity
C. It decreased fall risk
D. It improves quality of life
E. It improves gait impairment
94. Which are emphasized to increase muscle strength? C
A. Low resistance, high repetition
190
B. High resistancem high repetition
C. High resistance, fewer repetition
D. Low resistance, fewer repetition
E. Moderate resistance, high repetition
95. In a dynamic exercise a lengthening of the muscle fiber is called D
A. Concentric contraction
B. Isokinetic contraction
C. Isometric contraction
D. Eccentric contraction
E. Isotonic contraction
96. The following statement about Rheumatoid Arthritis (RA) is true: B
A. Female : male ratio = 1 : 3
B. Affects the synovial membrane of multiple joints
C. The joints mostly affected are shoulders and hips
D. DIP subluxation is the hand deformity in the late stage of RA
E. Systemic inflammation is not the risk factor for osteoporosis
97. This is the special test for total tear rotator cuff tendon: E
A. Hawkins impingement test
B. Neer impingement test
C. The joints mostly affected are shoulders and hips
D. DIP subluxation is the hand deformity in the late stage of RA
E. Systemic inflammation is not the risk factor for osteoporosis in RA
98. A 66 year-old lady with advance stage of Rheumatoid arthritis undergoes an arthroplasty surgery on the left knee
joint. She has severe bilateral MCP joint subluxation and ulnar deviation. Which of the following ambulatory aids
most appropiate for her? E
A. Cane
B. Tripod cane
C. Regular walker
D. Quadripod cane
E. Platform walker
99. A 85 year-old man present to your office with axial backpain that is worse with tanding and walking. At times he
also experience pain radiating from the lumbar spine into the legs when walking. What is the most likely diagnosis?
B
A. Claudiocatio intermittent
B. Lumbar canal stenosis
C. Piriformis syndrome
D. Sacroiliac joint pain
E. Ischial bursitis

100.A young healthy man 40 years old suffered acute low back pain after lifting a very heavy box. Coughing/sneezing
induce the pain which radiates to the left leg and foot, however no abnormality detected in bladder and bowel
function. He still able to walk with antalgic gait. Clinically, the patient suffering from: B
A. Grade 1 spondylolysthesis Lumbal vertebrae 4-5
B. Prolapsed intervertebral disc Lumbal 4-5
C. Spondylosis lumbalis
D. Lumbarization
E. Sacralization

191
ANSWER:
A. IF NUMBERS 1,2,3 ARE CORRECT
B. IF NUMBERS 1,3 ARE CORRECT
C. IF NUMBERS 2,4 ARE CORRECT
D. IF ONLY NUMBER 4 IS CORRECT
E. IF ALL NUMBERS ARE CORRECT

101.Deconditioning effects in elderly people, include A


1. Decreased VO2 max
2. Decreased muscle strength
3. Decreased reaction time, balance and flexibility
4. Increased time to fatigue during submaximal work
102. The benefit(s) of physical activities in the elderly include(s) the following: A
1. Slowing of disease progression
2. Promotion of the person’s health
3. Promotion of functional independence
4. Precention of aortic aneurysm rupture
103.An exercise program for elderly persons generally should emphasize increased: D
1. Intensity and frequency
2. Duration
3. Intensity and duration
4. Prevention od aortic aneurysm rupture
104.Elderly patient with hypoglycemia may experiencing: E
1. Tachycardia
2. Diaphoresis
3. Lighthadedness
4. Visual disturbances
105.The following statement(s) above type 1 muscle fiber is/are true: C
1. Generate larger force
2. Also called as slow twitch fibers
3. Also called as fast twitch fibers
4. Contain a large numer nitochondria and myoglobin

106. The types of flexibility exercise: E


1. PNF
2. Static stretching
3. Paasive stretching
4. Ballistic stretching
107.Adaptive equipment can be provided when deficit in upper extremity control limit efficient and safe
function are E
1. Swivel fork and spoons
2. Plate quards or specialized dishes
3. Weighed or large handledcups and utensils
4. Velcro or zipper closures instead of buttons may improve dressing
108.Complication associated with autonomic dysreflexia could include: E
1. Severe hypertension

192
2. Pulmonary edema
3. Cerecral bleeding
4. Retinal bleeding
109.According to ICE (International Classification of Functioning, Disability and Health) in stroke patient, the
following is (are) include in the loss of body functions and structures dimension: B
1. Cognitive important
2. Difficulty in ADL
3. Decubiti ulcer
4. Comorbidites
110.The following is (are) true about method is stroke rehabilitation: A
1. The principle of constaint induce movement therapy to avoid the learned nonuse of the paretic
side
2. The principle of mirror theraphy is that movement of the affected limb can be stimulated via
visual cues originating from the opposite site of the body
3. Functional Electrical Stimulation (FES) of the paretic limb enhances plasticity
4. Propioceptive Neuromuscular Facilitation (PNF) emphasized synergistic patterns of movement that
develop during recovery

111.The clinical sign(s) of Alzheimer is/are: E


1. Recent memory deficit
2. Language impairment
3. Psikomotor impairment
4. Cognitive dysfuntion
112.A man have very short below knee amputation proximal to the tibial tubercle: B
1. Knee extension strength is lost
2. Knee flexion strength is lost
3. The added length creates difficulties in prosthetic fit
4. Prostethic fits more simples compared to knee disarticulation
113.When positioning one or more joints with an orthosis, take into consideration: E
1. Joints biomechanics
2. Tissue tension
3. The pathology
4. Function
114.The SACH footh as the following characteristic(s) C
1. No toe break
2. Solid ankle joint
3. Very expensive price
4. Quite good functionally
115.The following is/are the test for diagnostic Achilles tendon rupture: C
1. Anterior drawer test
2. Hiperdorsoflexion sign
3. Posterior drawer test
4. Thompsn (Simmond test)
116.Joint protection techniques in patients with osteoarthritis include the following principles: E
1. Use appropiate assistive devies
2. Avoid activities that hurt affected joints

193
3. Avoid staying in one position for too long
4. Use largest and strongest joint and muscles
117.The principle(s) of burn rehabilitation is/are: A
1. Lubrication
2. Hydrotherapy
3. Splints are useful forpassive positioning
4. USDusing stationary tranducer with intensity >3watt/cm 2
118.What should be the first treatment considered for chronic lateral epicondylitis? D
1. Surgical release
2. Botuliniumtoxin
3. Corticosteroid injection
4. Wrist extensor stretching
119.The following is/are true about Parkinson disease (PD): E
1. Involving primarly a degeneration of nerve cell in the basal ganglia
2. Tremor at rest is the characteristic feature of PD
3. The gait with slow, small step and festination
4. Decrease in fine motor coordination
120.The blindness occurred in leprosy patients is/are caused by the involvement of the following nerve(s): B
1. Facial nerve
2. Optic nerve
3. Trigeminal nerve
4. Occulomotor nerve
121.The following is/are signs of autonomic dysreflexia: A
1. Pounding headache
2. Sense of chest fullnes
3. Blood pressure 220/120
4. Heart rate 120-140/minute
122.Which of the following are suggested order of muscle tests on sidelying? A
1. Hip adductor
2. Gluteus medius
3. Gluteus minimus
4. Gluteus maximus
123.The two major “crutch walking” muscles of the shoulders are: C
1. Teres minor
2. Latissimus dorsi
3. Pectoralis minor
4. Lower pectoralis major
124.The following is/are the possible cause(s) of genu recurvatum at stance phase of gait: A
1. Quadriceps spasticity
2. Quadriceps weakness
3. Plantar flexor spasticity
4. Dorsiflexors contracture
125.The examination is the initial step in rehabilitation management for the child with Muscular Dystrophy: E
1. History with family concern

194
2. Asistive and adaptive device
3. Aerobic capacity and endrance
4. Community and school/play integration
126.The symptoms comparison of pyramidal and extrapyramidal cerebral palsy is: D
1. Involuntary movement is often present in pyramidal type
2. The tone of extrapyramidal type often decreased
3. Joint contracture in pyramidal type is usually
4. Clasp knife phenomenon occus in pyramidal type
127.Club foot consist of associated deformities B
1. Forefoot varus
2. Forefoot valgus
3. Equinus of the foot at the ankle
4. Eversion deformity of the heel

128.A sedentary life style has: C


1. Decreases HDL cholesterol
2. A risk of hypertension
3. Reduce mortality rates
4. Decreases the sensitivity to insulin
129.A 60 year-old man with COPD in undergoing exercise training during ambulation training, he complains of
fatigue and dyspneu. His blood pressure 135/88, pulse is 100/minute and regular, spO 2 reveals 86%
saturation. At this time you would be: D
1. Start nerbulizer
2. Initiate incentive spirometer
3. Increased the duration of exercise
4. Add oxygen via nasal cannule during ambulation
130.Goals of the rehabilitation program in the patient with chronic obstructive airway disease include: A
1. Faciliating drainage of exercise brochial secretions
2. Improving the efficiency of the breathing patterns
3. Overall physical reconditioning
4. Reversing the pulmonary disease
A 19 y.o. female wusu athlete suffered from right knee pain after pivoting activity during jumping drill exercise.
She stopped her exercise, and after several hours her right knee became swollen and could not be flexed. Her
coach suggested do RICE (rest, icing, compression and elevation) on her right knee within the first three days.
Swelling reduced after 2 weeks and atrophy of her thigh muscle became marked. The painwa reduced, but
stillfelt pain on right lateral knee joint line during landing froma jumping activity. She also felt giving way and
locking while do pivoting and kicking activites on the right knee
131.From the history,what the possible anatomical structure/s could be injured? A
1. Ligament/s
2. Meniscus
3. Cartilage
4. Tendon
132.If Lachman, anterior drawer and pivot shift test were positive in this case, what specific ligament/s will be
involved? D
1. Posterior cruciate ligament
2. Lateral collateral ligament

195
3. Medial collateral ligament
4. Anterior cruciate ligament
133.From knee MRI, the one of findings was a longitudinal tear on the red zone of lateral meniscus with bucket
handle appearance C
1. Valgus test
2. McMurray test
3. Sagging knee
4. Apley’s compression test
134.What the possible management/s can be done in this patient? E
1. Conservatice with programmed rehabilitation
2. Arthroscopic assisted ligament reconstruction
3. Meniscal repair
4. Menisectomy
135.The effectiveness of cervical traction is dependent on many factors, including the amount of force applied.
Other factors include: E
1. Angle of pull
2. Position of the patient
3. Position of the cervical spine
4. Continuous or intermitten traction
136.Indications for the use Spinal Traction are: A
1. Spasm paraspinal muscles
2. Disc bulge or herniation
3. Nerve root impingement
4. Ankylosing spondilitis

137.The effect of ultra violet radiating is/are: B


1. Photochemical
2. Relief of muscle spasm
3. Bilogical effects
4. Increases the pain threshold
138.Human obesity is associated with an increased risk on knee OA specifically at: D
1. Patello tibial compartment
2. Patello femoral compartment
3. Lateral tibio femoral compartment
4. Medial tibio femoral compartment
139. The following is/are about idopathic scoliosis: C
1. Juvenile idiopathic scoliosis occurs between ages 2 and 8
2. The curves have strong tendency to progress rapidly during the adolescent growth spurt
3. A sex-linked trait can be transmitted by father, to either a son and daugther
4. Genetic factor is a sex-linked trait with incomplete penetrance and variable expressivity
140.Management of chronic back pain associated with osteoporotic vertebral fracture should include: E
1. A program of strengthening paravertebal, abdominal and gluteal muscles
2. Relief of stress on the spine through use proper body mechanic
3. Sfae program to improve flexibility and balance
4. Spinal orthosis

196
141.Rehabilitation management of osteoporosis patient depend on: E
1. The risk factors for osteoporosis
2. The degree of frailty and propensity to fall
3. Accurate determination of the degrees of bone loss
4. The capasity for participation in ADLs and safe exercise

142.The following is true about Duchenne’s Muscular dystrophy A


1. Sex linked recessive inheritance
2. Intellectual impairment has been documented
3. Symptoms may first appear when the child begins walks
4. Symptoms may first increasing numbers of high-amplitude, long-duration motor unit recruited
response to minimal patient effort
143.When a child is diagnosed as cerebral palsy, the first question asked by the parents is always “will he
walk?”. To answer these question the following test were done: E
1. Asymetrical tonic neck reflex
2. Symmetrical tonic neck reflex
3. Foot placement reaction
4. Parachute reaction
144.The following is/are the rehabilitation program of poliomyelitis patient in the convalescence phase: E
1. ROM exercise
2. Orthosis prescription
3. Strengthening exercise
4. Evaluate the muscle strength every 3 months
145.Needle EMG assesses nerve and muscle function.a recording needle electrode is placed into a muscle to
evaluate the following parameters: A
1. Voluntary recruitment
2. Insertional activity
3. Resting activity
4. Exertional activity
146.Basic function of the circulatory system include: E
1. Delivery of nutrient
2. Transportation of oxygen
3. Conveyance of hormones
4. Removal of waste products

147.The following condition(s) can be detected with EMG examination: E


1. The level/location of a peripheral nerve injury
2. Distrubance of the peripheral nervous system
3. Distrubance in neuro-muscular junction
4. Disturbance in muscle function
148.The following is/are the risk factors of deep vein thrombosis (DVT): E
1. Venous stasis in spinal cord injury
2. Increased coagulability such as diabetes mellitus
3. Increased fibrinogen during acute inflammatory response
4. Endothelial injury to the vessel wall during total hip or knee replacement

197
149.Aquatic-exercise program in cardiac patients: A
1. The recommended water temperature for ranges from 20 to 33 Celcius
2. Target heart rate reserve (THRR) is preferred for estimating intensity
3. The intensity are similar to those for aerobic exercise
4. The Borg RPE scale may be accurate with water exercise
150.Benefits of Cardiac Rehabilitation after bypass surgery are: B
1. Increased dischemic threshold
2. Increased serum cathcolamine
3. Increased coronary collaterals
4. Increased platelet aggregration

198
NATIONAL BOARD EXAMINATION

PHYSICAL MEDICINE AND REHABILITATION

JUNE 27, 2013

CHOOSE THE MOST APPROPRIATE ANSWER

1. The presence of nascent potentials in EMG examination shows the following process of : A
A. Reinnervation
B. Poor prognosis
C. Good prognosis
D. Total denervation
E. Partial denervation
2. The following is the characteristic of a MUAP (Motor Unit Action Potential) : C
A. Irregular in nature
B. The shape is always triphasic
C. Appears under voluntary control
D. Is an electrical activity coming from a muscle fiber
E. Appears when a needle electrode is coming into the muscle
3. The term is used to designate a mild degree of neuro insults in blockage of impulse conduction across
affected segment A
A. Neuropraxia
B. Neurometsis
C. Axonotmesisln neuropraxia, the following finding can be defected during the EMG/NCV studies tesis
D. Type 2 sunderland’s classification
E. Focal conduction block (Linberg)
4. Plantar fascitis is caused by which of the following ? A
A. Chronic traction on the plantar aponeurosis
B. Chronic inflamation on the flexor tendons
C. Chronic traction on tne flexor tendons
D. Acute plantar flexion
E. Acute dorsiflexion
5. The Boutonniere deformity in a Rheumatoid hand is typically located in the following joint: A
A. PIPs
B. MCPs
C. Radiocarpal
D. Ulnacarpal
E. Carpometacarpals
6. Problems arising from OA of the knee are all of the following, EXCEPT : C
A. There is continuous process of joint cartilage destruction
B. The in is the most active joint in human life
C. There is always concomitant osteoporosis

199
D. There is mataligment of the knee joint
E. Pain is the most important symptom

7. The following is NOT the typical musculoskeletal and systemic manifestations of SLE D
A. Arthralgias
B. Ligamentous laxity
C. Muscle pain and weakness
D. Osteoarthritis with joint deformities
E. Psychological and neurologic disorders
8. An adolescent disorder of the vertebral end plates and apophysis is called E
A. Spondylosis
B. Thoracic londosis
C. Facet syndromes
D. Spondylolisthesis
E. Schuermann’s disease

9. Adapted feeding utensils used by people with stroke is A


A. Rocker knife
B. Button hooks
C. Standard walker
D. Stand up mirror
E. Long handled sponge
10. Therapeutic exercise in CP uses a series of set patterns repeated many times during the day attemping to
train cerebral dominance and normalization of function is A
A. Doman and Delacate
B. Bobath
C. Deaver
D. Phelps
E. Voyta
11. Which type of massage “kneads” tissue by lifting, rolling, or pressing intermittently? C
A. Friction
B. Vibration
C. Petrissage
D. Effleurage
E. Tapotement

12. Reproduction of radicular symptoms with cervical spine extension, rotation, and lateral flexion of the seated
patients is C
A. Compression test
B. Sitting Roof test
C. Spurling’s test
D. Bowstring test
E. Slump test

13. Test with full flexion of the thumb into the pain followed by ulnar deviation of the wrist will produce pain
to diagnosis for de Quervain tenosynovitis is : E
A. Prayer test
B. Tinnel test

200
C. Phallen test
D. Froment’s test
E. Finkelstein’s test
14. Provocative test for acromioclavicular joint impingement is : A
A. Cross-chest test
B. Relocation test
C. Apprehension test
D. Anterior drawer test
E. Anterior load and shift test
A man 40 years old, had a motorcycle accident 3 months ago. On physical examination a normal sensation was
found at the area of medical malleoli right and left. The MMT grades were found on the lower extremities
iliopsoas normal, quadriceps fair tibialis anterior poor, extensor hallucis longus and gastrocnemius zero
(Question no 15-16)
15. The sensory level in this cases would be: E
A. T12
B. L1
C. L2
D. L3
E. L4

16. The classification according to ASIA Impairment Scale (AIS) is AIS: C


A. A
B. B
C. C
D. D
E. E

17. A patient is always complaining of difficulty in climbing the stairs. Most probably this is caused by
weakness of the following muscle : E
A. Pyriformis
B. Hamstring
C. Gluteus medius
D. Gluteus maximus
E. Quadriceps femoris
18. The following joint is NOT categorized as TRUE JOINT : A
A. Scapulo-thoracic joint
B. Calcaneo-cuboid joint
C. Gleno-humeral joint
D. Humero-ulnar joint
E. Atlanto-axial joint
19. Which of the following muscles is the lateral rotator of the hip joint? E
A. Tensor fascia latae
B. Gluteus minimus
C. Gluteus medius
D. Psoas major
E. Piriformis

201
20. The patient lies supine with the examiner holding the knee in 15 degree of flexion. The femur is stabilized
with one hand while pressure is applied to the posterior aspect of proximal tibia in an attempt to translate it
anteriorly. A positive test occurs when excessive anterior translation of tibia occurs with a soft endpoint.
This test named : A
A. Lachman test
B. Mc Murray test
C. Posterior drawer test
D. Patellofemoral grind test
E. Valgus and varus stress test
21. When the adolescent patient’s with pain at the tibial tuberosity that is exacerbated with activities and direct
contact. The radiographic is irregularly and fragmentation of the tibial tuberosity. The diagnosis is : C
A. Tibial stress fracture
B. Compartment syndrome
C. Osgood – Schlater disease
D. Medial tibial stress syndrome
E. Sinding – Larsen – Johanssen disease
22. The most commonly employed diagnostic maneuvers for TOS are all the following, EXCEPT: ?

A. Three minutes arm elevation


B. Shoulder depression test
C. Hyperabduction test
D. Yegarson test
E. Adson test
23. The most important substance/structure to hold water inside the joint cartilage is E
A. Collagen
B. Fibroblast
C. Fibronectin
D. Hyaluronate
E. Protenglycan
24. The following is hip abductor muscle EXCEPT D
A. Gluteus minimus
B. Tensor fascia lata
C. Gluteus medius
D. Psoas minor
E. Sartorius

25. The following is TRUE during gait cycle: D


A. About 60% of the normal gait cycle is spent in swing phase
B. As the speed of gait increases, the period of double support increased
C. The lower extremity tends to rotate externally through the swing phase
D. In the middle of stance phase and at push-off, the entire lower extremity begins to rotate
externally
E. As the external rotation in the lower extremity occurs, a degree of increased instability of the medical
aspect of the hip, knee and ankle
26. The following is TRUE about femoral neck : C
A. The normal angle of infliction of the femoral neck to the shaft in the frontal plane is 140 degree
B. The normal angle of infliction of the femoral neck to the shaft in the frontal plane is 100 degree

202
C. The normal angle of infliction of the femoral neck to the shaft in the frontal plane is 125 degree
D. It is called coxa valga if the angle 150 degree
E. It is called coxa vara if the angle 90 degree
27. A 45 year-old man with multiple sclerosis (MS) comes to the Neurology clinic complaining of the urinary
incontinence. He indicates that he experience increased urgency and frequency of urination. The most likely
urodynamic finding in this patient is : A
A. Spastic bladder
B. An atonic bladder
C. Stress incontinence
D. Overflow incontinence
E. Absence of abnormalities

28. A 35-year-old woman presents symptom in a few of progressive ascending muscle weakness. She had a
viral infection a few weeks earlier. On examination you find weakness and areflexia. The most likely
finding in the CSF is : B
A. High protein-high cell count
B. High protein-low cell count
C. Low protein-high cell count
D. Low protein-low cell count
E. Normal CSF
29. A patient complains of difficulty chewing. On examination he is found to have decreased strength of his
muscles of mastication. Which of the following cranial nerves is responsible for this motor function? B
A. Facial
B. Trigeminal
C. Oculomotor
D. Hypoglossal
E. Glossopharyngeal
30. Homonymus hemianopsia indicates a : D
A. Retinal bleeding
B. Chiasmal lesion
C. Prechiasmal lesion
D. Postchiasmal lesion of the opposite side
E. Lesion of on optic tract on the same side

31. You are asked to evaluate a 33-year-old construction worker who is complaining of paresthesias in the first
and second digits of his right hand. You physical examination shows no weakness but a mild decrease in
light touch over the thumb. You request a nerve conduction study to rule out carpal tunnel syndrome and it
turns to be normal. On repeated history, the patient indicates that on occasion, he gets a sharp “electric”
pain travelling from his neck to the right hand. What are you missing? E
A. A neuromuscular junction disorder aflecting distal hand muscles
B. A median neuropathy at the wrist
C. A lower trunk brachial plexopathy
D. A C8-TI radiculopathy
E. A C6-7 radiculopathy
32. The minimum current amplitude, with a long pulse duration, required to produce an action potential is A
A. Rheobase
B. Chronaxie
C. Cuilination

203
D. Monochromaticity
E. Minimal divergence of photon
33. Which of the following is NOT a property of lasers ? B
A. Coherence
B. Divergence
C. Collination
D. Monochromaticity
E. Minimal divergence of photon
34. In what condition would thermotherapy be indicated C
A. Acute muscoluskeletal
B. Impaired circulation
C. Chronic tendinitis
D. Skin anesthesia
E. Septic arthritis
35. The following statement is WRONG regarding the physiologic effects of spinal traction: D
A. Reduction of pain
B. Reduction of compression
C. Reduction of muscle spasm
D. Prevention and loosening of adhesion within nerve root and adjacent vertebral body
E. Prevention and loosening of adhesion within nerve root and adjacent capsul structures
36. All of the following are the thermal effects of USD, EXCEPT
A. Increased peripheral arterial blood flow
B. Increased permeability of membrane
C. Increase of tendon extensibility
D. Increase in tissue metabolism
E. Increase the pain threshold
37. A stroke patient signs of weakness and sensory deficits especially in the right lower extremity, while the
right upper extremity shows minimal disturbance. There are also slight mental disturbance and aphasia.
This condition most probably affecting the following artery:
A. Posterior inferior celbellar artery
B. Superior cerebellar artery
C. Posterior cerebral artery
D. Anterior cerebral artery
E. Middle cerebral artery
38. All of the following conditions may lead a Bell’s palsy patient has a poor prognosis, EXCEPT :
A. Severe inner ear pain
B. Hyperlacrimation
C. Diabetes millitus
D. Hypertension
E. Pregnancy
39. To reduce the incidence of infection in indwelling catheterization in SCI patients, the following is the best :
A. Flush the bladder with water everyday
B. Flush the bladder with saline everyday
C. Consume 1000mg Vitamin C everyday
D. Drink water 2-2.5 liters everyday
E. Clamp the catheter everyday 4 hours

204
40. The patient may complain of weakness in abduction (Supraspinatus) and external rotation (Infraspinatus) of
the glenohumeral joint is a clinical presentation of:
A. Musculacutancus neuropathy
B. Long thoracic neuropathy
C. Suprascapular neuropathy
D. Axillary neuropathy
E. Radial neuropathy
41. Shoulder pain are common after stroke episodes that bad to hemiparesis
A. Is usually due to Frozen shoulder
B. Is usually due to Theamic syndrome
C. Is usually due to Reflex sympathctic dystrophy
D. Is usually subluxation of the shoulder joint
E. Requires injection with steroid for effective management
42. All of the following are the purpose of prescribing an orthoses, EXCEPT
A. To replace exercise therapy
B. To correct deformity
C. To immobilize joint
D. To restore function
E. To reduce pain

43. In a hemiplegic, the function of a posterior stop in a double tipright AFO with action adjustable ankle joint
and external steel shank is to :
A. Improve knee stability at heel strike when the posterior stop is set in 5 degrees of dorsiflexion
B. Cause the ground reaction force line to fall posterior to the knee joint in mid stance
C. Assist in control of moderate to severe plantar flexor spasticity
D. Improve medial and lateral stability at the ankle
E. Assist weak plantar flexors

44. A lumbrical bar on a wrist-hand orthoses


A. Is primarily used in radial nerve injuries
B. Is used to replace the absent of MCP flexion
C. Is used with tenodesis flexor hinge orthoses
D. Prevents MCP hyperextension with finger extension
E. Is positioned on the volar aspect of the proximal phalanges
45. Disorder of skilled movement in the absence of motor sensory, or cognitive impaired instroke are called
A. Aprosody
B. Agraphia
C. Anomia
D. Apraxia
E. Alexia
46. A male 50 years old have a clinical features clumbars, buttock, perianal discomforrt, and lower limb
weakness Bowel and bladder abnormalities,sexual dysfunction, saddle anesthesia including the back of the
legs, buttocks, and soles of the feet. The diagnosis is :
A. Cauda equina syndrome
B. Spondylolysthesis L5-S1
C. Pyriformis syndrome
D. Spondylolysis
E. Myelopathy

205
47. Which of the following is TRUE :
A. A bilateral lesion at C1 will result in cessation of brething
B. A complete transection of the spinal cord at C2 never results in urinary retention
C. A complete transection of the spinal cord at S2-S4 never results in urinary retention
D. Large lesion of the cauda equina will result in spastic bladder both acutely & chronically
E. A complete lesion of the spinal cord at C2 results in a spastic bladder immediately after the injury
48. A central cord syndrome is characterized by:
A. Hyperflexion injury
B. Pain below the level of injury
C. Loss of posterior column function
D. Disproportionately greater motor impairment of the upper than lower estremities with sacrl
sparing
E. Disproportionately greater motor impairment of the lower than lower estremities without sacrl sparing
49. The knee orthosis is used to control minor to moderate genu recurvatum is
A. The generation II knee orthosis
B. Osteoarthritis knee orthosis
C. Reciprocating gait orthosis
D. Swedish knee cage
E. HKAFO
50. A leprosy patient shows severe bilateral claw hand. However, all of the hand and finger joint have good
ROM and all the extrinsic muscles of the hand have normal strength. He refused a reconstructive surgency.
The best orthosis device to maximaze hand function :
A. Knucle-bender splint
B. Resting hand splint
C. Posterior splint
D. Cock-up splint
E. Thumb splint
51. The following terminal device of an upper extremity has the best function
A. Cosmetic hand with moveable thumb
B. Myoelectric hand
C. Cosmetic glove
D. Dorrance hook
E. Cosmetic hand
52. A 2-month-ld infant presents to you for evaluation of delayed development. He was the product of a normal
term pregnancy, labor, and delivery. Bith weight was 3500 grams. He has had difficulty feading since birth.
Family history is negative for developmental problems. On physical examination,he is awake, but NOT
alert. Weight is 3600 grams. Respiration is unlabored. He has poor head control and decreased tone
throughout. Deep tendon reflexes are absent. What is the most likely diagnosis?
A. Kugelberg Welander syndrome
B. Duchenne muscular distrophy
C. Werding-Hoffman Disease
D. Tetraplegic cerebral palsy
E. Infantile botulism
53. Patient with athetoid cerebral palsy are likely to have what impairment?
A. Disuse atrophy
B. Mental retardation
C. Communication disorders

206
D. Secondary degenerative disease
E. More hearing deficits than patients with spastic diplegia CP

54. Physical examination findings that are consistent with hypotonia in a term infant include all of the
following, EXCEPT :
A. Atrophy
B. Weak suck
C. Frog leg position of the legs
D. Decrease spontaneous movements
E. Head lag when pulled tositting position
55. Pediatric limb deficiencies with missing hand and foot is called
A. Amelia
B. Adactily
C. Acheiria
D. Aphalangia
E. Phocomelia
56. Typical gait abnormalities of spastic diplegia in CP children is :
A. Hip hiking
B. Scissoring gait
C. Hip circumduction
D. Supinated foot in stance phase
E. Weak hip flexion and dorsiflexion
57. This reflex is best elicited by a sudden dropping of the baby’s head in relation to its trunk
A. Verical Suspension
B. Ridghting Reflex
C. Extensor thrust
D. Landau Reflex
E. Moro Reflex

58. This statements is WRONG in specifictherapeutic excercise for scoliosis


A. Done in and out of the brace
B. Stretching exercise of the convex side
C. EDLF exercise is done in the kneeling position
D. Muscle strengthening exercise of the convex side
E. Abdominal muscle strengthening excercise incude
59. Goal for children with developing language in syntax area is:
A. Increase production of closed syllables
B. Increase consonant repertoite
C. Increase sentence lenght
D. Increase vocabulary size
E. Increase intelligibility
60. Therapeutic approaches to the upper limb in stroke which proposed a technique that incorporates cutaneous
stimuli to facilitate movements is :
A. PNF
B. Brunnstrorm
C. Rood method
D. Phelps method

207
E. Bobath method
61. The propioceptive Neuromuscular Fascilitation (PNF) is a neurophysiologic therapeutic approach for CP,
developed by
A. Fay
B. Rood
C. Bobath
D. Brunnstorm
E. Kabat and knort

62. Arnold chiari malformation is associated finding on :


A. Spina Bifilda Ocuta
B. Myeloningocel
C. Syriangornyelia
D. Meningocele
E. Myelocele
63. You are called to see your 3 years old inpatient with a CS ASIA Aspinal cord injury. She has a headache
and complains of NOT feeling well. Vital signs are pulse 60, respirations 20, blood pressure 120/80.
Weight 33 lbs (15kg). Physical examination is unchanged from previously. You order :
A. Administer acetaminophen (Tylenol) orally
B. Obtain computed tomography of the head
C. Place the patient in the supine position
D. Empty the bladder
E. All of the above
64. Congenital dislocation of the hip in older infant does NOT have clinical signs at dislocation side with
maneuver :
A. Galeazzi
B. Thomas
C. Ortolani
D. Barlow
E. Allis
65. The most common type of cerebral Palsy is :
A. Spastic Diplegia
B. Spastic Triplegia
C. Spastic Tetraplegia
D. Spastic Hemiplegia
E. Spastic Monoplegia

66. The child has chief complaint of bowlegs that are gradually progressing in severity and NOT improving
spontaneously. The deformity is called :
A. Genu recurvation
B. Knock knees
C. Tibia vara
D. Clubfoot
E. CTEV

208
67. One of the most catastrophic injuries in all of sport medicine is
A. Muscle cramps
B. Rotator cuff tears
C. Suden cardiac arrest
D. Exercise associated collapse
E. Cervical spinal cord injuries
68. A softball player suffered from shoulder pain on the night side. The pain increased when he made repetitive
rotation on his right shoulder,no radiating pain, no tingling sensation. Shoulder ROM within normal limit.
Neer and hawkin. Kennedy tests were positive. Drop arm, yergason’s and apprehention tests were negative.
What is possible diagnosis?
A. Rotator cuffimpingement syndrome
B. Shoulder subluxation
C. Supraspinates tear
D. Bicipital tendinitis
E. Frozen shoulder
69. Ankle sprains are ubiquitous in sports and are the most common athletic injury. The most commonly injury
for this disorder is
A. The calcaneo tibia ligament
B. The calcaneo fibular ligament
C. The anterior talofibular ligament
D. The posterior talofibular ligament
E. The strong deltoid ligament
70. Thich one of the following is TRUE about inhalation theraphy for airway clearance
A. Nebulizer therapy will be giving a better result in airway clearance when administered in a crying child
B. Inhalation therapy delivers modication effectively throughout the whole respiratory tract
C. Nebulizer therapy is the most important part of airway clearance techniques
D. Nebulizer therapy is most effectivedone after chest physical therapy
E. Nebulizer therapy can deliver short acting beta agonist drug
71. Mode of exercise training hat is proven safe, and effective enough with minimal fatigue level in all degrees
of COPD is :
A. Endurance training with high intensity strength training
B. Endurance exercise with continuous training
C. Endurance exercise with interval training
D. Anaerobic exercise with strength training
E. Circuit weight training
72. The following is the moderate risk stratification criteria for cardiac patient
A. Angina to recovery
B. Absnce of clinical depresion
C. Functional around >7 METs
D. Complex vetricular arrhythmias at rest
E. Abnormal Bemodynanies with exercise
73. The proper emergency response for a patient who has experiented a cardiac arrest, but who is now breathing
and has a palpable pulse, includes ;
A. Placing the patient in the recovery position with the head side to prevent airways obstruction
B. Continuing the exercise test to determine why the patient had this response
C. Placing the patient in a comfortable seated position

209
D. Placing the patient in a side lying position
E. Placing the patient in a head-up position
74. Absolute indication for terminating. Exercise stress testing is
A. Drop in systolic BP of > 10 mmHg from baseline
B. Supraventricular tachycardia
C. Fatique, shortness of breath
D. Moderate to severe angina
E. Hypertensive response
75. The main goal of exercise progam for the geriatric patient is to :
A. Reduce falls
B. Increase agility
C. Reduce depression
D. Increase flexibility
E. Increase endurance
76. Which statement regarding the proven effects of Tai Chi in persons with osteoporosis is TRUE?
A. It helps retard bone loss in both trabecular and cortical weight bearing bones
B. It helps to increase muscle strength but has no effect on retarding bone loss
C. It helps to increase balance but has no effect on retarding bone loss
D. It helps retard bone loss only in trabecular bone
E. It helps retard bone loss only in cortical bone
77. The most common causes of dementia in the elderly is
A. Untreated or undertreated pain
B. Poor functional status
C. Alzheimer disease
D. Immobilization
E. Hydrocephalus

78. During exercise there is an increase in a person’s


A. Stroke volume
B. Diastolic pressure
C. Venous compliance
D. Total peripheralresistance
E. Pulmonary arterial resistance
79. The majir goalofphase 1 Cardiac Rehabilitation is to:
A. Stimulate the development of coronary collaterals
B. Increased the strength of upper and lower limb
C. Evaluatethe myocardial exercise tolerance
D. Prevent deconditioning
E. Prevent heart attack
80. Woman,24 year-old consulted from ward surgery with diagnose post ASD closure 1 st day, rehabilitaiton
program when she awake is:
A. Do sitting,deep breathing effective coughing with monitoring vital sign
B. Gradual sitting with support breathing exercise, effective coughing with monitoring vital sign
C. Active sitting, breathing exercise, effective coughing, ankle pumping, ROM with monitoring viral
D. Active sitting, breathing xercise, effective coughing, ankle ROM, chest physical therapy when
indicated

210
E. Gradual sitting, breathing exercise, effective caoughing active ROM, ankle pumping exercise,
chest physical therapy when indicated

81. Man, 60 years old, came to emergency ward with short of breath since two days before. He was heavy
smoker for 20 years and has been stopped 10 years ago because of short and recurrently hospitalized 2 years
recenly. History of past illness: no diabetic, no heart disease. After acute state was overcome, he was
refered to pulmonary ward and consulted to rehabilitation department. What the propper progam for this
patient ?
A. Breathing control and relaxation
B. Relaxation teqhnique, gradualmobilization
C. Breathing control, relazation teqhnique, gradual mobilization
D. Breathing control, pursed lip breathing, progress to diaphragmatic brething, chest physical therapy
E. Breathing control, pursed lip breathing, progress to diaphragmatic brething, chest physical
therapy, gradual mobilization
82. Ventricular tachycardia is marked by
A. One PVC on every other beat repeatdly
B. Two PVCs on every other beat repeatdly
C. Two premature ventricular contractions (PVCs) in row
D. Three PVCs (or more) in a row without any beats
E. The rate about 150-500 beats/minute the absence of a distinct QRS complex
83. The most commonly injured knee ligment in sport is
A. Medial Collateral Ligament
B. Anterior Collateral Ligament
C. Posterior Collateral Ligament
D. Lateral Collateral Ligament
E. Patellofemoral Ligament

84. Turf toe is metatarsophalangael sprain occurs with hyperextension injuries of the great toe. Which sports
that generally occurs ?
A. Table Tennis
B. Basket Ball
C. Gymnastic
D. Jumping
E. Soccer
85. Medical emergency of exercise – associated collapse in atletic is
A. Benign exercise associated collapse
B. Hear exhauston
C. Muscle cramps
D. Hypoglicemia
E. Hypothermia
86. Common injuries in the deceleration phase of pitching is
A. Bicipital
B. Anterior Instability
C. Posterior instability
D. Internal impingemente
E. Anterior surface rotator cuff tears
87. Non contact ACL injuries of the knee more frequently with the knee in

211
A. Full flexion
B. Less flexion
C. Full extension
D. Less extension
E. Less hip external rotation

88. Pain around medical aspect of the knee joint is very often caused by inflammation of the pes anserius. This
is a conjoin tendon of the following muscles :
A. Semimembranous-ssemitendinosus-sartorius
B. Semitendinosus-semimembranosus-gracilis
C. Sartorius-gracilis-semimbranosus
D. Semitendinousus-sartorius-gracilis
E. Gracilis-sartorius-bicep femoris
89. Ambulatory assistive device the elderly if ther are limitations with grip strengthor painful joints in the hand
is:
A. Cane
B. Walker
C. Srtches
D. Wheelchair
E. Forearm crutches
90. The following factor decreased risk factor for osteoporosis is:
A. Advanced age
B. Thin habitus
C. Smoking
D. Obesity
E. Female
91. A 80 year-old man presents to oyur office with axialoain that worse with standing and walking. At times he
also experiencres pain radiating from the lumber spine into legs when walking. What the most likely
diagnosis?
A. Sacrolitis
B. Ischias syndrome
C. Piriformis syndrome
D. Spondylosis lumbalis
E. Lumbal canalis stenosis

92. What is the equivalent to8.75 ml/kg/min?


A. 1.0 MET
B. 1.5 METs
C. 2.0 METs
D. 2.5 METs
E. 3.0 METs
93. Non pharmacologic management of exercise induced bronchospasm is:
A. Decreasing physical conditioning
B. Exercise for at least 2 hours following a meal
C. During cold weather don’t cover the mouth and throat
D. Warm up period for at least 10 minutes prior to exercise
E. Exercise shouldn’t be performed in a humidified enviroment

212
94. Detrusor overactivity from damage to upper motor neuron impaired function and cognitionis effectof
continence in:
A. Stroke
B. Delirium
C. Dementia
D. Multiple sclerosis
E. Parkinson’s disease
95. The following is NOT the problem associated with rehabilitationof the elderly:
A. Loses approximately 20% of muscle mass by age 65
B. Increased the level of hydration of connective tissue
C. Increased collagen cross-linkages
D. Peripheral loss of sensation
E. Urinary incontinence

96. The following is the change in specialsenses with aging, EXCEPT:


A. Lens protein fibers undergo oxidative damage
B. Loss of higher frequency sound
C. Loss of salt perception
D. Visual cortical changes
E. Hyperosmia
97. This statement is true about exercise precription in elderly:
A. At least 60 minutes of lowintensity in all days of weeks
B. At least 30 minutes ofhigh intensity in all days of the weeks
C. At least 60 minutes of moderate intensity in all days of the weeks
D. At leart 30 minutes of moderate intensity in all days of the weeks
E. Substituting low intensity with moderate intensity exercise in all dyas of the weeks
98. In neuropracia, the following finding can be detected during the EMG/NCV studies:
A. Fibrilation potentials
B. Positive shar waves
C. Fasciculations waves
D. All the NCV studies are “no response”
E. Conduction block at certain level of nerve
99. The most common clinical application of the H-reflex is to test:
A. Polyneuropathy
B. S1 radiculopathy
C. Cervical radiculopathy
D. Guillain barce syndrome
E. Carpal tunnel syndrome

100.EMG is the recording and study of the electrical activity of muscle. Procedure typically performed during
an EMG evaluation is te study of
A. Insertional activity
B. Spontaneous activity

213
C. Nerve conduction velocity
D. Voluntary activity – minimal contraction
E. Voluntary activity – maximal contraction

ANSWER:

A. IF NUMBERS 1,2,3 ARE CORRECT


B. IF NUMBERS 1,3 ARE CORRECT
C. IF NUMBERS 2,4 ARE CORRECT
D. IF ONL NUMBER 4 IS CORRECT
E. IF ALL NUMBERS ARE CORRECT

101.The following is (are) the advantage of SACH foot:


1. Very simple
2. Quite cheap
3. Easy maintenance
4. Good ankle movemnets
A rigid dressing may be attached apylon and ffot to allow ambulation soon after surgery.the combination of
rigid dressing, pylon and foot is calledan immediate postoperative prosthesis (IPOP). (Number 2-3)
102.The advantage(s) of IPOP is/are:
1. Improved healing time
2. Proctection of the surgical site
3. Reduction of contracture development
4. Control and shaping of the residual limb

103.The contraindication(s) to the use of an IPOP include:


1. Excessive postoperative edema
2. History of slow healing
3. Extreme obesity
4. Diabetic patients
104.About Rheumatoid Arthritis:
1. Is an auto immune disease
2. Erotion of cartilages
3. Chronic synovities
4. Erotion of bones
105.The following are the other name of frozen shoulder
1. Duplay disease
2. Tendinitis long rotator
3. Periarthritis adhesive
4. Adhesive burstitis scapulohumeral
106.In the case of low back pain, when we suspect Paget’s disease, the laboratory test that must be order to
conform the diagnosis is/are:
1. CPK
2. Amilase
3. Rheumatoid factor
4. Alkaline phosphatase
107.The following muscle is/are the primary movers at scapulothoracic interface:

214
1. Subscapularis
2. Latissimus dorse
3. Infraspinatus
4. Seratus anterior

108.The following statement(s) is/are TRUE about physical modality:


1. Iontophoresis uses charged electrodes (positve of negative) to drive medically active
2. TENS is applies small electrical signals to the body with superficial skin electodes
3. In SWD water and metal are excellent electrical conductors
4. MWD will produce catarract
109.The following statement(s) is/are true
1. The mechanism of heat transfer of paraffin bath is conduction
2. The mechanismof heat transfer of whilpool is convection
3. The mechanism of heat transfer of SWD is conversion
4. The mechanism of heat transfer of USD is radiation conductors
110.The following is/are true about paraffin bath:
1. It is maintained at about 52 degress Celcius
2. The indications is contractures particularly in the hands
3. Paraffin baths consist of 1-7 mixture of mineral oil and paraffin
4. Three forms application of paraffin bat: dipping, immersion, and uses abrush to paint paraffin
onto portions of the body
111.The propiocaptive disturbance would be found in the following disease(s):
1. Stroke
2. Tabes dorsalis
3. Brown-sequard syndrome
4. Funiculus anterior lesions
112.The functional outcome of complete L3 paraplegia is/are:
1. Independent in bowel and bladder routines
2. Independent in wheelchair propulsion
3. Independent in pressure relief
4. Standing without orthosis

113.Important factors that influence walking ability after stroke include are
1. Balance
2. Weakness
3. Coordination
4. Cognitive function
114.About ankle mortis:
1. Its motion is 20 degress dorsiflexion
2. The talus function as a hinge joint
3. Consist of tibia,fibula and talus
4. Consist of talus and calcaneus
115.The two major “crutch walking” muscles of the shoulders are
1. Teres minor
2. Latissimus dorsi
3. Pactoralis minor

215
4. Lower pectoralis major
116.The following is/are true about hip joint:
1. Is a ball and socket joint
2. Hip flexion at least 120 degrees
3. The use of a brace in the leg can alter the magnitude of the hip joint
4. The magnitude of the hip joint reaction force is influenced by the ration of the abdcutor muscle
force and gravitational force lever arm
117.The characteristic of cerebral palsy are as follow
1. The underlying neurologic lesion must be static
2. The process which causethe cerebral palsy is stillactive
3. The sign of cerebral palsy is a disorderof movement and posture
4. It is caused by progressive injury which affects the mature brain

118.The characteristic spastic dyplegia of cerebral palsy are as follow:


1. 18 months implied signs poor prognosis for ambulation
2. Children who did not sit by 4 year have not achieved ambulation
3. Sitting by 2 year was a good predictive sign of eventual ambulation
4. Children with diplegia have not adequate movement control of the arm and hands
119.General heading for each level for Gross Motor Function Claasification System for cerebral palsy is
1. Weak cry
2. Tremor of hands
3. Frog posture
4. Weakness of legs, prodominantly proximal
120.General heading for each level Gross Motor Function Classification System for cerebral palsy is
1. Level I: walks with limititation
2. Level II: walks without limitation
3. Level IV: transported in a manual wheelchair
4. Level III: walks using a hand held mobility device
121.Predictor for ambulation is spina bifida occulta are:
1. Motor level
2. Sitting balance
3. Training in wheelchair use
4. Deformites of the spine
122.The following are developmental on primitive reflexes:
1. Grasp reflexes
2. Rooting reflexes
3. Gross extension reflexes
4. Asymmetrical tonic neck reflex

123.A 45 year-old woman present chronic pain in the region of the Achilles tendon appears swollen. Appropiate
initial management would include:
1. Cortison injection of the Gastrocnemius muscle
2. Cortison injection of the Achilles tendon sheath
3. Immobilization in ankle plantar-flexed position with limited weight bearing
4. Use of a small heel lift with initiationof gastroc-soleus stretches as tolerated
124.The rehabilitaion of patient with AMP

216
1. Do not flex the hip joint more than 90 degrees
2. Weight bearing can be done after 3 months
3. Weight bearing can be done immediately
4. Do not abduct the hip joint
125.What kind of exercise can be done as early rehabilitation approach(es) is ICU setting?
1. Neuromuscular electrical stimulation
2. Activity daily living training
3. Speech language therapy
4. Chest physical therapy
126. The most common locations of DVT include the calf, tight, arms, and pelvis. The symptooms and signs
are:
1. Pain in the calf with walking
2. Pain in the calf or thigh
3. Positive Homans sign
4. Swelling of the calf
127.The objective(s) of giving pursed-lips breathing respiration is/are:
1. Introduce the patient of how to control respiration
2. To decrease the respiratory rate
3. To increase tidal volume
4. To increase oxygenation

128.These are the rehabilitation programs for leprosy


1. Oil massage
2. Active exercise
3. Soaking the feet in water
4. Protection of the hands and feet
129.The mode of action(s) of diazepam to reduce muscle spasticity is/are through the following mechanism(s):
1. To stimulate P substance
2. To stimulate Glutamate
3. To stimulate Aspartate
4. To stimulate GABA
130.Rehabilitation treatment for Parkinson’s disease are
1. Gait evalaution should be done, including speed and distance
2. Assesment of fine motor tasks,such aswriting
3. Coordination exercise
4. Endurance exercise
131.A recreational tennis player hadreight wlbow pain since 3 weeks ago. The pain increased gradually with the
time especially after she played tennis and extended the wrist against resistance on dorsal aspect af the
hand. What is/are the proper management(s) for this particular patient?
1. Avoid wrist extension
2. Checkand adjust the grip of racquet
3. Wrist extensor eccentric strengthening exercise
4. Ultrasound diathermy on lateral epicondylus area
132.In sport activities, tendons are most vulnerable to injury is/are to
1. Tension is applied quickly and sustained without adequate warm up
2. Tension is applied quickly and stopped rapidly

217
3. Tension is applied obliquetly direction
4. Tension is applied as tensite forces
133.The objective(s) in training muscles, tendons, and joints after injury is/are to:
1. Stretch the connective tissue of tendons and muscles to an optimal length
2. Increased the strength of muscle and tendon attachments
3. Improve coordination and propioception
4. Regain good mobility of the joints
134.Preoperative exercises for total hip replacement include:
1. Progressive ressitive exercise for hip triceps, latissimus dorsi and other crutch walking muscle
2. Foor-ankle exercise to diminish venous stasis to prevent the occurence of thrombophlebitis
3. Breathing exercise, including instruction on proper cough technique and postural drainage
4. Isometric exercise dor quadriceps, gluteus medius and gluteus maximus
135.The prescription of exercise for the patient with simple recurrent low back pain achieve at least the
following
1. Improve low back flexibility
2. Improve posture with minimize lumbar lordosis
3. Improved body mechanics in all activities and exercise
4. Improved strength of abdominal and hip extensor muscle
136.This statement is the contraindication for prenatal exercise:
1. At risk for premature labour
2. Severe hypertensive disease
3. Poor obstetric history
4. No prenatal care

137.14 year-old boy with hemophilia has had recurrent hemarthrosis of the knee in the past, but presently, has
full ROM and normal quadriceps muscle strength. His parents ask advice regarding appropiate athletic
activities. You storngly support an activelife style, but advise against
1. Biking
2. Competitive golf
3. Competitive swimming
4. Comppetitive basketball
138.During exercise for pulmonary rehabilitation in the patient with COPD, the goals to keep the SaO2 above
what value?
1. 80%
2. 85%
3. 88%
4. 90%

139.What is the purpose of a warm-up period?


1. Gradual reduction of cardiac work
2. Promotes continuous dissipation of heat
3. Prevent pooling of blood in the lower extremity
4. Redistribution of blood from muscle to internal organs
140.The following below is/are the condition causing ventilatory impairment:
1. Spinal muscular atrophy
2. Muscular dystrophia

218
3. Kyphoscoliosis
4. Poliomyelitis
141.The role of golgi tendon organs in voluntary motor activity is/are
1. Detects relative muscle tension
2. In paralled with the musclefibers
3. Has dynamic and static responses
4. Increase in muscle length during passive movement
142.Electrodiagnostic technique which most useful for identifying Myastenia Gravis are
1. Needle EMG
2. Single fiber EMG
3. F-waves H-reflexes
4. Repetitive nerve stimulation
143.The following statement(s) is/are correct regarding the motor unit
1. Its electrical activity is called MUAP
2. The targeted system of EMG examination
3. The member of its muscle fibers depends on the function of the particular muscle
4. Consisting of one motor call in the anterior horn of the medulla spinalis and all of its muscle
fibers innervated
144. A strategy to enhance the stability of a wheelchair is/are
1. Lessening the seat depth
2. Elevating the seat height
3. Moving the rear axle forward
4. Tilting the wheel chamber outward top to bottom
145.A 20 year-old football player has episodes of heat cramps during summer training sessions. A deficiency of
what electrolyte is/are responsible for heat cramps?
1. Magnesium
2. Potasium
3. Chloride
4. Sodium

146.Which of the following statements is true regarding pain in the old person?
1. Pain can commonly be located to a single site
2. Elderly persons do not feed pain a much as younger people
3. Pain leads to less functional impaiment in the elderly than in the young people
4. Approximately one third of elderly persons have chronic joint pain and arthritis
147.The following is/are the indication for measurment of bone density:
1. Adults taking medications such as phenytoin
2. To monitor treatment effect on osteoporosis
3. Women discontinuing estrogen therapy
4. Women aged 65 and other
148. In prescribing pain treatment for elderly patient, the following is/are the proper approach?
1. Use high-dose, short-acting narcotics initially to get the pain control
2. Avoid use of acetaminophen due to liver toxicity
3. Start with aspirin, which is safe and effective
4. Start low and go slow for all medications

219
149.The following statement(s) is/are correct regarding the physiology of the bladder
1. The external spincter is under the control of somatic system S4-5
2. The emptying phase is under the control of parasympathetic system S2-4
3. The filling phase is under the control of the sympathetic system Th11-L2
4. The function of the frontal lobe of the brain is to inhibit the sacral micturition center
150.Which physiologic factor(s) in elderly exacerbate(s) orthostatic hypotension?
1. Decrease in arterial stiffness
2. Decreased creatinin clearance
3. Decreased peripheral resistance
4. Decreased baroreceptor response

INDONESIAN COLLEGE
OF
PHYSICAL MENDICINE AND REHABILITATION

NATIONAL BOARD EXAMINATION

JUNE 13, 2014


NATIONAL BOARD EXAMINATION
PHYSICAL MEDICINE AND REHABILITATION
June 13, 2014

220
CHOOSE THE MOST APPROPIATE ANSWER
1. In the Brown-Sequard syndrome of spinal cord hemisection. A spastic paresis develops in the muscle
innervated by nerves derived from spinal roots: D
A. Bilaterally at the level of injury
B. Ipsilaterally at the level of injury
C. Contralaterally at the level of injury
D. Ipsilaterally below the level of injury
E. Contralaterally bellow the level of injury
2. This assessment should NOT be done for a person with certain characteristic and it must be emphasized in
rehabilitation evaluation: A
A. Analyze equilibrium by tandem walking and let the patient fall
B. Assess the degree of rigidity and bradykinesia
C. Measure and record restriction in joint mobility
D. Assess fine motor task such as writing
E. Asses the respiratory
3. A 40 year-old man, 4 weeks after traumatic brain injury with left hemiplegia. He now develops severe
spasticity, particularly affecting the left shoulder and arm muscles. Which one of the following treatments
in NOT validated? D
A. Injection of Botulinum A toxin in the spastic muscles
B. Alcohol injection in the spastic motor points
C. Gaba-type medication
D. Low level laser therapy
E. Stroking with ice
4. A 50 year-old man who had been living independently suffers a left frontotemporal infarction. Neurologic
examination reveals nonfluent aphasia and right hemiparesis, wheelchair dependent ambulation. After 2

221
weeks in the rehabilitation clinic, he is noted to be progressing very poorly functional viewpoint despite
improving neurologic examination. The most likely reason for poor progress is: E
A. Progresive dementia
B. Recurrent infarction
C. Subdural hematome
D. Seizure disorder
E. Depression
5. Mr. A 50 year-old with ischemic stroke. In the physical examination found left hemineglect. The following
sign can be present EXCEPT: A
A. Aphasia
B. Denial of his hemiplegia
C. Mistakes in the localization sound
D. The absence of the left part of his drawings
E. A decrease of the visual left at the opposite side of the lesion
6. The most common cause for floppiness at birth is: E
A. Congenitial myotonic dystrophy
B. Spinal muscular atrophy type I
C. Prader-Willi Syndrome
D. Central core myopathy
E. Cerebral dysfunction
7. What are handly language millestones to remember? C
A. Bubbling: 1 year
B. Single word: 2 years
C. Body parts: 18 months
D. Short sentences: 12 months
E. Long sentences: 18 months
8. A 9 year-old girl with C5 ASIA A spinal cord injury sustained 2 years ago is evaluated for upper extremity
splinting. Which statement regarding this scenario is TRUE? D
A. A short hand splint should be prescribed to strengthen wrist extensor muscles
B. A resting hand splint should be prescribed for daytime use to preserve fuction
C. A long short hand splint should be prescribed to strengthen wrist extensor muscles
D. A mobile arm support or balanced forearm orthosis could be prescribed to make self-feeding
possible
E. Wrist extension splint would be contraindicated for daytime use because it would interfere with
function
9. The parents of a 4 year-old boy with Duchenne Muscular Dystrophy (DMD) ask your opinion about the
future of their son. Which of the following statements is NOT correct? B
A. DMD typically becomes clinically evident at approximately 3 to 5 years of age
B. Weakness will beginning in the shoulder girdle muscles and later in the pelvis
C. Early difficulties noted are clumsiness, poor walking and frequent falls
D. Death is usually due to respiratory
E. Intellect may be affected
10. The following is a finding that has been document to predict independent ambulation in cerebral palsy: C
A. Cruising by age 24 months
B. Pulling to stand by age 24 months
C. Sitting unsupported by age 24 months
D. Kneeling supported by age 15 months

222
E. Rolling supine to prone by age 15 months

11. Nerve conduction is slow in: E


A. Myopathy
B. Upper motor neuron lesion
C. Axonal sensory neuropathy
D. Primary axonal degeranation
E. Demyelinated peripheral nerve
12. The structure below is NOT a member of the motor unit : D
A. Skeletal muscle
B. Motor end plate
C. Neuromuscular junction
D. Neuron of the dorsal horn of the spinal cord
E. Neuron of the anterior horn spinal cord
13. The myoneuronal junction main neurotransmitter is: D
A. Dopamine
B. Serotonine
C. Substance P
D. Acetyl choline
E. N-Methyl-D-A spartic acid (NMDA)
14. When disk protudes, usually it does so in the posterolaterally direction, because D
A. The anulus is thicker posteriorly
B. The longitudinal ligament support the spine in trunk position
C. The longitudinal ligaments support the spine in bending position
D. The posterolateral aspect is not covered by the longitudinal ligaments
E. The forces of weight bearing in upright position are move on the posterior side of the vertebral body
15. Pes anserinus in a conjoint tendon of the followong muscles: B
A. Gracilis-Sartorius-Biceps femoris
B. Semitendinosus-Sartorius-Gracilis
C. Sartorius-Gracilis-Semitendinosus
D. Sartorius-Biceps femoris-Semitendinosus
E. Semimembranosus-Semitendinosus-Sartorius
16. The following muscle DOES NOT belong to the intrinsic muscle group of the hand: E
A. 4th lumbricalis
B. Palmaris brevis
C. Flexor pollicis brevis
D. Abductor pollicis brevis
E. Extensor carpi radialis brevis
17. Which one of the knee bursitis condition is commonly seen in an overweight women? A
A. Anserine
B. Prapatellar
C. Infrapaterllar
D. Superior patellar
E. Semimembranosus
18. A 50 year-old female dentist with right carpal tunnel syndrome. She left tingling on the right hand for 4
weeks. No thenar atrophy and weakness. The most approciate treatment is: E

223
A. Surgery
B. Wrist splint
C. Ultrasonic diathermy
D. A local corticosteroid injection
E. Untrasound diathermy and wrist spling
19. Which of the following is considered a handicap? D
A. Inability to lift a 20 kg box
B. Presence of balance disturbance
C. Presence of a unilateral above knee amputation
D. Inability to enter a courtroom via the entry stairs
E. Persistence of sensory loss in peripheral neuropathy
20. In patient with third-degree burns, which of the following positions would most effectivelly minimize
contractures? E
A. Elbow extended and pronated
B. Shoulder abducted 180 degrees
C. Hips adducted and internally rotated
D. Hips abducted 20 degrees and externally rotated
E. Shoulder abducted 90 degrees and externally rotated
21. Which of the following hand splint is utilized in the acute phase management of burns of the dorsum of the
hand? C
A. Knuckle-bender splint
B. Dorsal extension splint
C. Volar positioning splint
D. Cook up with metacarpophalangeal cuff
E. Radial wrist conformer with thumb and index
22. The bisphosponates alendronate and residronate: D
A. Have exellent gut absorption
B. Have no gastrointestinal side effects
C. Act primarily by stimulating bone formation
D. Have been shown to reduce vertebral and nonvertebral fractures
E. Are no more effective on fractures reduction than hormone-replacement theraphy, selective estrogen
receptor modulators adn calcitonin
23. Complications from Parkinson’s disease and its treatment include all of the followong EXCEPT: E
A. Dysphagia
B. Depression
C. Dyskinesis
D. Constipation
E. Auditory halllucination

24. In older patient undergoing a rehabilitation program after a hip fracture, benefit from weight-bearing
exercises can: C
A. Improve pain control
B. Improve gait pattern
C. Improve walking velocity
D. Decrease risk of prosthetic
E. Decrease incidence of hip dislocation
25. The most common cause of myelopathy in cancer patients is: E

224
A. Radiation myelitis
B. Spinal cord infarct
C. Primary spinal cord tumor
D. Cord compression from intradural
E. Extrinsic cord compression from epidural metastatis
26. Which of the following chemotherapy agents is most likely to cause an axonal neuropathy? C
A. Etoposide
B. Prednisone
C. Vineristine
D. Doxorubicin
E. Methotrexate
27. Foot orthosis most commonly benefit ther runner with leg and foot problems by A
A. Reducing pronation
B. Limiting supination
C. Amplifying push off
D. Everting the calcaneous
E. Decreasing knee flexion
28. A 55 year-old runner present with pain in the region of the Achilles tendon. This is the third such episode
over the past 4 years. The Achilles tendon appears swollen. Appropiate initial management would include:
D

A. Deep heating of the Achilles tendon


B. Cortisone injection of the Achilles tendon
C. Local anesthetic injection of the Achilles tendon Sheath
D. Use of small heel lift with initiation of gastro-soleus stretches as tolerated
E. Immobilization in an inkle plantar-flexed position with limited weight bearing
29. Among the following, the tool redesign most likely reduce repetitive trauma is:
A. Designing a control that requires continuous forces rather than episodic activation D
B. Removing the plastic slevee in the handle
C. Switching from a foot to a hand control
D. Changing from a pinch to a power grip
E. Elimination fixtures holding parts
30. Competitive female athletes are at high risk of stress fractures if they have the female athlete triad of: A
A. Amenorrhea, anorexia, osteoporosis
B. Amenorrhea, bulimia, poor flexiblity
C. Bulimia, hypocalcemia, poor flexibility
D. Amenorrhea, hypocalcemia, osteoporosis
E. Amenorrhea, hypercalcemia, osteoporosis
31. Which bony wrist injury related to forceful extension of the hand is commonly seen in gymnast? A
A. Scaphoid fracture
B. Pisiform fracture
C. Lunate fracture
D. Radius fracture
E. Ulnar fracture

32. Exercises for cardiovascular conditioning should involve the following EXCEPT: A
A. Isometric training

225
B. Sircuit training
C. Swimming
D. Jogging
E. Cycling
33. Exercises for diabetic patients is beneficial because: B
A. Increase blod glucose level
B. Increase insulin sensitivity
C. Increase insulin resistance
D. Decrease HDL
E. Increase LDL
34. Physical activities advised to patient with CHF functional class III (NYHA) is: B
A. 1-2 METs
B. 3-4 METs
C. 5-6 METs
D. 7-8 METs
E. 9-10 METs
35. The respiratory function of a patient with ankylosing spondylitis is characterized by: E
A. Decrease in diaphragmatic excursion
B. Increase in diaphragmatic excursion
C. Obstrutive respiratory dysfunction
D. Increase use of accessory muscles
E. Diminished chest expansion
36. A 60 year-old man with COPD had recurrent respiratory failure, requiring tracheostomy and has been on
bed rest for 8 weeks. You would expect to see: D
A. Decrease nutriuresis
B. Elevated serum calcium
C. Decreased albumin level
D. Carbohydrate intolerance
E. Decreased excretion of potasium
37. One of the statement below are NOT contraindication on using traction: D
A. Vertebrobasilar artherosclerosis disease
B. Metastatic bone disease
C. Ligamentous instability
D. Radicular pain
E. Osteoporosis
38. The therapeutic target temperature in deep heating is generally considered to be: C
A. 30° to 35° C
B. 35° to 40° C
C. 40° to 45° C
D. 45° to 50° C
E. 50° to 55° C

39. Therapeutic ultrasound: B


A. Is generally used for more superficial heating
B. Is classified as acoustic rather than electromagnetic energy
C. Is the modality of choice for minimizing the joint flexibility
D. Is the modality of choice for minimizing increases in cellular metabolism
E. Is capable of enchaning healing at the cellular level as a result of its thermal physiologic

226
40. A patient with DM has painful, swollen ankle, that become worst with weight bearing. X-ray shows
fragmentation of the articular surface, synovial fluid is clear. The choice of orthosis is: E
A. AFO
B. HKAFO
C. Diabetic shoe
D. KAFO with medial pad
E. Patellar tendon bearing orthosis
41. In amputee with a patellar tendon bearing prosthesis and a SACH foot, incoordinately delayed knee flexion
at heel strike may be to: C
A. Anterior socket tilt
B. Poor hip extension
C. To soft a heel cushion
D. Excessive dorsiflexion of the foot
E. Discomfort over the anterior
42. A plastic AFO with a rigid ankle is indicated in patients with: B
A. Weak or absent dorsiflexors without severe weakness of the plantar flexors
B. Weak or absent dorsiflexors and plantar flexors
C. Weak or absent knee extensors
D. Absent to moderate spasticity
E. Plantar flexion contractures
43. A 66 year-old woman with rheumatoid arthritis 21 years duration for rehabilitation program following a left
total knee arthroplasty. On examination, you note that she has marked ulnar devination and subluxation of
the metacapophalangeal joint in both hands. Which one of the following is the best assitive ambulation
device for she initially rehabilitation program? D
A. Quad cane
B. Rolling walkier
C. Double crutch
D. Platform walker
E. Lofstrand crutches
44. Activates postural development and equilibrium reaction to guide normal development in CP patients is C
A. Rood approach
B. Ayres approach
C. Voyta approach
D. Bobath approach
E. Doman-Delacato approach
45. The activity of climbing up stairs requires the following amount of METs: C
A. 2 METs
B. 3 METs
C. 6 METs
D. 8 METs
E. 10 METs
46. A 60 year-old male with generalized painful sensory neuropathy is to be started on amitripyline for pain
control. This treatment should be avoide if patient has: A
A. Second-degree atrioventicular block
B. Congestive heart failure
C. Renal insufficiency
D. Stress incontinence

227
E. Diabetes mellitus
47. What is the mechanism of action of an intramuscular injection botulinum type A toxin in reducing
spasticity? A
A. Blocks the release of presynaptic acetycholine release at the neuromuscular junction and end
plate
B. Inhibits afferent fibers in the dorsal root of the spinal nerve
C. Competitively blocks the acetycholine receptor
D. Destroy the neuromuscular junction
E. Acts as a GABA agonist
48. You are evaluating a patient with a nerve palsy. During evaluation, the patient reveals a positive froment’s
sign. Which nerve is injured? E
A. Ulnar and median
B. Ulnar and radial
C. Median
D. Radial
E. Ulnar

49. Which of the following symptoms is rare in amyotrophic lateral sclerosis? E


A. Respiratory insufficiency
B. Muscle wasting
C. muscle cramp
D. Dysartria
E. Pain
50. Which of the following is the common sources of afferent stimulation that result in autonomic dysreflexia:
B
A. Bladder irritablity
B. Rectal distension
C. Skin irritability
D. Stomachache
E. Headache
51. A baby of 7 months old shows a symmetical LMN weakness, no sensory deficits, proximal muscles weaker
than the distal, abdonimal breathing. The strong possibility of diagnosis is: C
A. Duchne’s Muscular Dystrophy
B. Kugelberg-Wellander disease
C. Werding-Hoffmann disease
D. Guillan Bare Syndrome
E. Poliomyelitis
52. The period which intense motor learning and basic language development occur is at: D
A. Birth to sixth month
B. Birth to one year of age
C. Birth to two years of age
D. Birth to three years of age
E. Birth to four years of age
53. What type of Cerebral Palsy (CP) with unsteady gait, dysmetria, intention tremor in upper extremities and
truncal titubation? E cucur 785
A. Hypotonic type
B. Athetoid type

228
C. Spastic type
D. Mixed type
E. Ataxia type
54. One of your 4 year-old patients exhibits the following characteristic: distress over minor changer in
enviroment, echolalia, lack of awarness of the exixtence of feelings in others, nonparticipation in simple
games. The most likey diagnosis is: A
A. Autism
B. Cerebral Palsy
C. Mental retardation
D. Hearing impairment
E. Minimal brain damage
55. The symptoms and signs of internal tibial torsion are more prominent, because of: A
A. Obesity
B. Back knees
C. Retroversions of the hip
D. Foor progression angle positive 10 degrees
E. External rotation of the hip more than 90 degrees
56. The structure that sensors of change of skeletal muscle tension is: D
A. Troponin
B. Tropomyosin
C. Muscle spindle
D. Golgi tendon organ
E. Myotendineal junction
57. Factors involved in the formation of contractures include: D
A. A increaser in the number of sarcomeres
B. A change from type I to type II muscle fibers
C. Inadegquate cross-linkage of collagen molecules
D. Disturbed balance between synthesis and degradation of collagen
E. A decrease in the number of mitochondria present in muscle fibers

58. Which one of the following is TRUE regarding energy sources for contraction C
A. Energy for muscle contraction is derived directly from AMP
B. Energy for muscle contraction is derived directly from ADP
C. Creatine phosphate provides a substrate for rapid resynthesis of ATP
D. The energy bonds creatine phosphate are equal with concentration of resting muscle ATP
E. Muscle glycogen provides a source of the energy required for first muscle contraction
59. Atrophy of the first dorsal interosseous muscle may indicate damage of the following roots: D
A. C5-C6
B. C6-C7
C. C7-C8
D. C8-T1
E. T1-T2
60. The knee joint is a: A

A. Trochogynglimus joint
B. Gynglimus joint
C. Trochoid joint
D. Pivot joint

229
E. False joint
61. A patient is always complaining of difficulty in climbing the stairs. Most propably this is caused by the
weakness of the following muscle: E
A. Pyriformis
B. Hamstring
C. Gluteus medius
D. Gluteus maximus
E. Quadriceps femoris
62. Following a skin graft for a burn, active and passive ROM of the temporarily immobilized recipien t site
should be reinstated after: B
A. 2 days
B. 5 days
C. 10 days
D. 15 days
E. 20 days
63. A 30 year-old male quadriplegic which develops swelling and erythema in the right lower extremity. Non
invasive venous studies are consistent with a deep venous thrombosis extending into the thigh. Appropiate
treatment would include therapeutic anticoagulantuon with intravenous heparin and: A
A. Bed rest
B. Icing to the affected extremity
C. Warm and soaks to the affected extremity
D. Intermittent pneumatic compression devices
E. Contiuned ROM exercises to the extremity
64. A foraminal closure by rotates the head to one side, side bends to the same side and extends is: C
A. Valsava Test
B. Lhermite test
C. Spurling’s test
D. Cervical spine distraction test
E. Cervical spine compression test
65. Which of the following may be associated with a subacrominal corticosteroid injection? D
A. Dermal keratinification
B. Localized osteopenia
C. Muscle rupture
D. Tendon rupture
E. Osteoporisis

66. A 70 year-old man comes to your office complaining of bilateral shoulder pain worse in the morning.
Radiographs of his shoulder reveal glenohumeral osteoarthritis, and his erythrocyte sedimentation rate is
70. Your initial management is to B
A. Prescribe Codman exercises
B. Prescribe prednisone 20mg per day
C. Prescribe stretching for the glenuhumeral joint
D. Perform bilateral glenuhumeral corticosteroid injections
E. Prescribe strenthening exercises for the deltoid muscles
67. Among supported-ventilation techniques, the most pratial way to allow adequate oxygenation is: E
A. A chest shell
B. A rocking bed

230
C. A wrap-style ventilator
D. An intermittent abdominal pressure ventilator
E. An intermittent positive pressure ventilator (IPPV)
68. Which physiological change occurs in the cardiovascular system with aging? D
A. Increased VO2
B. Increased stroke volume
C. Increased resting heart rate
D. Decreases ejection fraction
E. Increased resting cardiac output
69. The define of impairment is: A
A. Dysfunction or abnormalities in body system
B. Difficulties in performing instrumental ADL
C. Difficulties in performing basic ADL
D. A restriction in physical activities
E. A restriction in participation
70. An elderly woman with severe arthritic pain, has a poor appetite, insomnia and anxiety. You recommended
aggressive treatment of her pain. What is likely secondary consequence of her pain in this setting that
should also be treated? E
A. Dementia
B. Disuse atrophy
C. Deconditioned
D. Acute delirium
E. Clinical depression
71. The 30 year-old football player who present with knee pain, immediate swelling and inability to bear
weight after a fall, should undergo: A
A. Ice, elevation, compression and crutches as indicated
B. Ice, elevation, return to football playing next day
C. Heat, NSAID, Ace wrap, return to football playingthe next day
D. Cold compresses, acetaminophen, 5 days off from football playing
E. Heat wrap, knee immobilization, advantage weight bearing as tolerated
72. The initial phase of treatment of iliotibial band friction syndrome include the following EXCEPT: A
A. Introduce upgrades sport-activity training
B. Give anti-inflammatory medications
C. Introduces therapeutic modalities
D. Consider steroid injection
E. Apply friction massage
73. The general treatment for low back pain in athletes in maintenance phase (functional adaptation) include the
following, EXCEPT: E
A. Core stabilization
B. Sacral mobilization
C. McKnzie program
D. Stretching piriformis
E. Muscle relaxation medication

74. The following patterns are the predispose factor to anterior instability for long distance swimmer,
EXCEPT: D
A. Rounded

231
B. Thoracic kyposis
C. Foreward head attitude
D. Retracted shoulder with external-rotated arm
E. Protracted shoulders with internal-rotated arm
75. The initial rehabilitation program for a short of breath and frightened COPD patient is: D
A. Low intensity endurance exercise
B. Modified protocol of exercises stress test
C. Massage of secondary respiratory muscle
D. Start with breathing control to achieve relaxation
E. Stretching exercise for upper and lower extremities
76. What exercises are recommended for persons with nonprogressive mild to moderate restrictive lung disease
(VC 0f 60% predicted)? D
A. Pursed lip breathing
B. Rapid shallow breathing
C. Glossophayngeal breathing
D. Inspiratory resistive exercise
E. Expiratory resistive exercise
77. The gold standard measure for cardiorespiratory fitness is B
A. Measurement of METS
B. Measurement of VO2max
C. Measurement of heart rate
D. Measurement blood pressure
E. Measurement of cardiac output
78. Cardiac rehabilitation program encounter a variety of heart condition including in below EXCEPT: B

A. Post PTCA
B. 3rd degree AV block
C. Post myocardial infarction
D. Post Coronary Bypass Graft
E. Chronic Congestive Heart Failure
79. In prescribing modalities, it is important to take consideration various patient spesific factors such as in the
following, EXCEPT B
A. Sensory deficit
B. Functional status
C. Cognitive impairment
D. Underlying malignancy
E. Decreased vascular supply
80. In SWD treatment if the electrodes are places side on the same aspect of the body, provide they have
adequate distance, is called: B
A. Biplanar method
B. Coplanar method
C. Cross-fire method
D. Monoplanar method
E. Contraplanar method
81. The following is NOT indication of biofeedback therapuetic: B
A. To inhibit spasticity
B. To strengthen muscle

232
C. Muscle reeducation in UMN dysfunction
D. Muscle reeducation in LMN dysfunction
E. Pelvic floor reeducation in women with incontinence
82. Subjective dyspnea in congestive heart failure patients can be improve with B
A. Chest physical therapy
B. Repiratory muscle training
C. Limiting the use of inotropic agents
D. Treatments that decrease filling pressures
E. Biofeedback to accessory respiratory muscles
83. For a patient with coronary artery disease, good left ventricular function and no history of myocardial
infarction, exercise intensity should be approximately B
A. To the angina threshold
B. 10 beats/min below angina threshold
C. 20 beats/min below angina threshold
D. 30 beats/min below angina threshold
E. 40 beats/min below angina threshold
84. Among the following which activity might be especially stressful for patient with cardiac disease? D
A. Lying
B. Leg raising
C. Outdoor walking
D. Bridging exercise on mat
E. Lower extremity cycle ergometry
85. Which of the following symptoms is rare in amyotrophic lateral sclerosis? E
A. Respiratory insufficiencey
B. Muscle wasting
C. Muscle cramp
D. Dysarthria
E. Pain
86. Which of the following is the common sources of afferent stimulation that result in autonomic dysreflexia B
A. Bladder irritability
B. Rectal distension
C. Skin irritability
D. Stomachache
E. Headache
87. Management of GBS are as follow, EXCEPT C
A. Pool exercise
B. Breathing exercise
C. Treadmill exercise
D. Occupational therapy
E. Provision of walking aid
88. The final station of the nociceptive system is: A
A. Thalamus
B. Hypothalamus
C. Limbic sytem
D. Pre-central gyrus of the brain
E. Post-central gyrus of the brain

233
89. Common disabling condition in childhood that is not progressive is: A cucur 782
A. Cerebral Palsy
B. Cystic Fibrosis
C. Muscular Dystrophy
D. Spinal Muscular Atrophy
E. Juvenile Theumatoid Arthritis
90. The following statements are true congenital bracial plexopathy, EXCEPT A
A. Affect males and females differently
B. Usually unilateral; could be bilateral
C. Most common etiologies are big baby ad shoulder dystocia
D. Treat with splinting ROM exercises, strengthening, and encourage use of affected arm
E. Overall 80-90 percent favourable prognosis with most improvement in the first 3 months, poor
prognosis if there is root avulsion
91. The following is NOT indication of biofeedback therapeutic: B
A. To inhibit spasticity
B. To strengthen muscle
C. Muscle reeducation in UMN dysfunction
D. Muscle reeducation in LMN dysfunction
E. Pelvic floor reeducation in women with incontinence
92. Which of the following components of a KAFO would stabilize the knee, while applying force to pressure
tolerant tissue in the best way: D
A. A knee cap strap
B. A single suprapatellar strap
C. A single patellar tendon strap
D. Combined suprapatellar and patellar tendon straps
E. A lower thigh band closure in combination with a calf banf closure
93. Which of the following prosthetics knee mechanisms provide good swing-phase control at variable
cadences and low energy expenditure? D
A. Single axis with manual lock
B. Constant friction
C. Polycentric
D. Hydraulic
E. Safety
94. In the management of the juvenile upper limb amputee, which of the following in corret? B
A. Children are fifted and trainde with prosthese according to their chronologic age
B. A prosthesisis fifted when the child can sit independently at about 6 months of age
C. The terminal device is activated by cable control when the child is ready to grasp at about 12 months
D. Acceptance and use of a prosthesis are most succesful if the child is fifted and trained by a age 6
(years)
E. In above-elbow amputee, elbow use can be expected if the elbow-locking mechanism is activated at 16
to 12 months
95. Target heart rate that is recommended for cardiac insufficient patients during exercise is: C
A. 40-50% of MHR
B. 51-69% of MHR
C. 70-85% of MHR
D. 86-90% of MHR

234
E. 91-95% of MHR
96. The following statement in NOT correct regarding with the coordination exercise: E
A. To apply sensory input
B. To perform appropiate placement
C. To accelerate the speed as tolerable
D. To perform constan repetitive movements
E. To strengthen progressively the particular muscles
97. The aim of rehabilitation program in asthmatic patients are the following, EXCEPT: D
A. To relieve bronchospasm
B. To aids removal of secrets
C. To coodinate respiratory movements
D. To increase the strength of inspiratory muscles
E. To assist relaxation and gain control breathing
98. Extraskeletal manifestations of ankylosing spondylitis include: A
A. Achilles enthesopathy
B. Tricuspid regurgitation
C. Nail pitting
D. Cataracts
E. Colitis
99. The examination for Achilles tendon rupture is: E
A. Tinel’s test
B. Suction test
C. Talar tilt test
D. Homan’s test
E. Thompson’s test
100.After a mastectomy, full active assisted shoulder ROM exercise should be done: E
A. At 1 day
B. At 3 days
C. At 1 week
D. To patient tolerance
E. After removal of surgical drain

ANSWER:
A. IF NUMBERS 1,2,3 ARE CORRECT
B. IF NUMBERS 1 AND 3 ARE CORRECT
C. IF NUMBERS 2 AND 4 ARE CORRECT
D. IF ONLY NUMBER 4 IS CORRECT
E. IF ALL NUMBERS ARE CORRECT

101.The following statements below are spesific regarding thoracic outlet syndrome? E
1. Usually caused by an extracervical rib that compresses the medial cord of the brachial plexus
2. Tingling and numbness in the medial aspect of the arm
3. Produced lower motor neuron deficits
4. Decreased extremity
102.The prescription of exercise for the patient with simple recurrent low back pain should achieve at the
following: E
1. Improved low back flexibility
2. Improved posture with minimize lumber lordosis

235
3. Improved body mechanics in all activities and exercise
4. Improved strength of abdominal and hip extensor muscle

103.Which factor is/are NOT a consideration in diagnosing lateral elbow pain? D


1. C7-7 radiculopathy
2. Osteochonditis dissecans
3. Lateral collateral ligament injury
4. Tear of the flexor digitorum superficialis
104.The bowel trainning in SCI patient should consider the following factor(s): E
1. The previous bowel habits
2. The gastro-colic reflex
3. The toilet condition
4. The nutrition diet

105.Bowel incontinence management for spina bifida is/are E


1. Increase fiber & bulk in diet
2. Reflex emptying techniques
3. Maintain a consistent daily time for bowel movement
4. Strengthening of accessory muscles for relaxation or expulsion
106.The statement below describe the types of aphasia that may occur after stroke: E
1. Wernicke aphasia which lead to reductions in comprehension with speech that is fluent but often
nonsensical
2. Wernicke aphasia may show a loss of repetition ability with preservation of expression
3. Global aphasia show that it has loss its expression ability and comprehension
4. Speech in nonfluent with a reduction in expression in Broca aphasia
107.The following statement is/are the sign(s) of cerebellar dysfunction: E
1. Hypotonia
2. Nystagmus
3. Dysarthria
4. Asynergy
108.The objectives in rehabilitation management for dysphagia is/are? A
1. To reestablish oral eating to the safest optimum level
2. To maintain an adequate nutritional intake
3. To prevent aspiration pneumonia
4. To determine surgery indication
109.The following are the characteristics of Becker’s Muscular Dystrophy E
1. This form in similar to Duchene’s Muscular Dystrophy in terms of distribution of weakness
2. Intellectual impairment is not common in Becker’s Muscular Dystrophy
3. It has a later onset that Duchene’s Muscular Dystrophy
4. It is more benign than Duchene’s Muscular Dystrophy
110.The goal of neuro development treatment by Bobath approach to cerebral palsy is/are: B cucur 792
1. To normalizze tone
2. Activating muscles through sensory receptors
3. To inhibit primitive reflexes
4. Activates Postural development and equilibrium reactions

236
111.At IQ level 54-4- (moderate retardation), the speech and language ability are: A
1. Able to learn functional communication
2. Poor receptive language skills
3. Articulation disorders
4. Minimal speech
112.The absolute contraindication for entry exercise stress testing: E
1. Unstable Angina
2. Thromboplebitis
3. Moderate to severe aortic stenosis
4. Symptomatic congestive heart failure
113.Which of the following is/are appropiate exercise for diabetic patients with loss of protective sensation in
the extremities? D
1. Jogging with treadmill
2. Prolonged walking
3. Step class exercise
4. Swimming
114.Which of the following is/are including as restrictive lung disease? C
1. Asthma
2. Tubercolosis
3. Emphysema
4. Pulmonary edema
115.The characteristic changes posture in elderly: A
1. Slightly flexed of knees and hips
2. Increasing thoracic kyphosis
3. Forward of head posture
4. Narrowed base of support
116.Rehabilitation strategic(s) for reducing risk of falls in elderly: C
1. Strengthen lower limb
2. Home safety assesment
3. Stretching ankle joint
4. Balance exercise
117.Lympedema treatment consist of all of the following: A
1. Use of compression garments
2. Manual lymphatic drainage
3. Proper skin care and diet
4. Use diuretic
118.The following is/are the causes of incontinence in elderly people: A
1. Dementia
2. Immobilization
3. Urological disorder
4. Vascular disosder
119.What injury are associated with running? C
1. Ankle sprain
2. Achilles tendinitis
3. Stress fracture if tibia

237
4. Patellafemoral pain syndrome
120.How does the anterior cruciate ligament (ACL) injury become torn? B
1. Deceleration of the leg via quadriceps contraction combined with valgus and external rotation
forces upon slightly flexed knee
2. Sudden external rotation of hyperflexed knee
3. Sudden hyperextension of the knee
4. Direct blows to the anklE

238
A basketball athlete had injured after he jump and landed on his right foot in the internally rotated
position. He heard “pop” and felt painful on his right knee. He ceased from this game because he
cannot walk and got an edema suddenly (no 121 & 122)

121.What is/are possible diagnosis for his condition? B


1. ACL rupture
2. Patellafemoral syndrome
3. Meniscal torn
4. Quadriceps muscle rupture
122.What special test(s)/sign(s) will positive for his condition? C
1. Grinding test
2. Mc Murray test
3. Sagging test
4. Anterior drawer test
123.The following statement(s) is/are correct regarding the physiology of bladder: E
1. External sphincter in under the control of somatic system S4-S5
2. The filling phase is under control og symphatetic system T11-L2
3. The emptying phase is under the control parasymphatetic system S2-S4
4. The function of frontal lobe of the brain is to inhibit the sacral micturiton center
124.The following factor(s) influence the strength of muscles contraction: A
1. The starting point/position in which muscle start to contract
2. The number of motor units which are active
3. The amount of actin and myosin
4. The muscle volume
125.In the “sliding theory” of muscle fiber contraction: C
1. The actin and myosin filaments are arranged in such to maintain the “double helix”
configuration
2. Actin is the thin filament and myosin is the thick filament
3. Actin is the thick filament and myosin is the thin filament
4. Myosin is found in the H band
126.In gait analysis what is/are the possible causes of equinus in swing phase? E
1. Spasticity of the solues and gastrocnmeus
2. Spasticity of posteroir tibialis muscles
3. Heel cord contracture
4. Weak dorsiflexor
127.The following is/are the possible cause(s) of genu recurvatum at the stance phase of gait: A
1. Quadriceps spasticity
2. Quadriceps weakness
3. Plantar flexor spasticity
4. Dorsiflexor contracture

239
128.Plantar flexor muscles are inactive at time of: D
1. Mid stance
2. Heel strike
3. Foot flat
4. Toe off
129.Intermittent compression pumps is/are indicated in which of the following condition: A
1. Postmastectomy hymphedema
2. Venous insufficiency
3. Post traumatic edema
4. Arterial insufficiency
130.These are about ultrasound diathermy: E
1. The intensities of therapeutic area in the range of 0.5 to 2.00 W/cm2
2. The most common technique applicative is the storking
3. One of the precaution is near laminectomy site
4. The non-thermal effect include cavitation
131.The following statement(s) is/are correct regarding cervical traction: C
1. The position must be in supine
2. Spondyloarthrosis is indication
3. Can be given to rheumatoid arthritits
4. The neck position is in mid-flexion
132.The readiness for lower limb prosthetic fitting is/are concern with: E
1. Stable medical status
2. Adequate extensor strength
3. Ability to stand on the intact leg
4. Healing incision with no open areas longer than 1 cm
133.A 6 year-old girl with spastic quadriparesis needs a new wheelchair prescription. Patient
factors that influence the prescription include: E
1. Presence of lower limb spasticity
2. Degree of trunk control present
3. Presence of thoracic kyphosis
4. Patient’s weight and height
134.Tha SACH foot has the following characteristic(s): C
1. No “toe break”
2. Solid ankle joint
3. Very expensive price
4. Quite good functionally
135.The following conditions is/are the contraindication(s) for postural drainage approach: A
1. Hemoptysis
2. Cerebral edema
3. Severe hypertension
4. Policystic lung disease

240
136.About Frenkel’s exercises for ataxic conditions: E
1. Design primarily for coordination
2. They are not intended for strengthening
3. Commands should be given in an even, monotonous voice
4. Ability of patients to interpret deep muscle and joint sensability may be checked by
having the patient perform the exercise with eye closed
137.The following is/are the part of William’s exercise for patient with low back pain” C
1. Hip flexor muscle strengthening
2. Abdominal muslces strengthenng
3. Hamstring muscle strengthening
4. Hamstring muscle stretching
138.The functional outcome of complete L3 paraplegia is/are: A
1. Independent in pressure relief
2. Independence in wheelchair propulsion
3. Independence in bowel and bladder routines
4. Requires wheelcahir ambulation in a short distance
139.The general principles of shoulder rehabilitation is/are: B
1. Start pain reduction early and restore pain-free ROM of shoulder joint
2. Initiate open chain with hand moving in the early phase
3. Start pain-free exercise and avoid impingement
4. Achive 180 degree of abduction early
140.A 45 year-old woman complains of severe heel pain with ambulation, especially with her first
few step each morning, which condition is/are likely cause of her complaints? D
1. Calaneal stress fracture
2. Achilles tendonosis
3. Chronic metatarsalgia
4. Plantar fasciitis

141.The purpose of bracing in osteoporosis of the spine is/are: A


1. To compensate for the weak erector spinae muscles
2. To prevent vertebra compression fracture
3. To diminish the incidence of kyphosis
4. To strengthen the paraspinal muscle
142.Rehabilitation program in Parkinson’s Disease is/are: E
1. Swallowing evaluation
2. Assesment of fine motor task
3. Assesment of degree of rigidity
4. Adaptive equipment to improve feeding
143.The typical communication deficits seen in patients with right cerebral hemisphere damage
is/are: E
1. Difficulty organizing information in a meaningful way
2. Difficulty reading facial expression and gestures

241
3. Inability to understand figurative
4. Left neglect
144.A male patient is still under rehabilitation program. The family noted that the patients dit not
find his coffe-cup at the left side of his plate, but he already asked for more food. This
phenomena is called: C
1. Apraxia
2. Neglect
3. Agnosia
4. Hemispatial inatention
145.Monitoring in children with bow leg with evaluation of: E
1. thigh foot angle
2. tibio femoral angle
3. foot progression angle
4. the position of lateral and medial malleoli
146.The etiology congetial muscular torticollis may caused by: A
1. Muscle injury during delivery
2. Compartment syndrome
3. Intrauterine crowding
4. plagiocephaly

242
INDONESIAN COLLEGE
OF
PHYSICAL MEDICINE AND REHABILITATION

NATIONAL BOARD EXAMINATION

243
DECEMBER 12, 2014
NATIONAL BOARD EXAMINATION
PHYSICAL MEDICINE AND REHABILITATION
December 12, 2014

CHOOSE THE MOST APPROPIATE ANSWER

1. Where is the lesion site in ataxic dysarthria, as found in Friedreich’s ataxia? A


A. Cerebellum
B. Multiple sites
C. Basal ganglia
D. Extrapyramidal system
E. Bilateral upper motor neuron
2. An infarct in the lower division of the left middle cerebral artery division would be associated
with which type of aphasia: C
A. Broca
B. Global
C. Wernicke
D. Transcortical motoric
E. Trancortical sensoric
3. Which finding os associated with a more favorable prognosis in Amyotrophic Lateral
Sclerosis (ASL) B
A. Female sex
B. Young age and male sex
C. Bulbar involvement presentation
D. Short time period from symptoms to diagnosis
E. Predominance of lower motor neuron findings on EMG studies
4. Which of the following physical exam findings would be most consotent with a C6
radiculopathy? A
A. Decreased brachioradialis reflex
B. Medial brachial sensory loss
C. Decreased biceps reflex
D. Deltoid weakness
E. Tricep weakness
5. The following luscles are usually affected by a spiral groove lesion, EXCEPT for the: A
A. Triceps
B. Brachioradials
C. Extensor carpi radials
D. Extensor carpi ulnaris
E. Extensor digitorum communis
6. Which type of stroke typically has the best outcome? B
A. Hemorrhagic pontine

244
B. Internal capsule lacunar
C. Embolic left cortical infarction
D. Rupture of anterior cerebral artery aneurysm
E. Ischemic occlusion of the right internal carotid artery
7. Anosognosia commonly occurs in patients with stroke and refers to a patient’s A
A. Lack of recognition of illnes
B. Loss of two-point discrimination
C. Inability to orient objects on three dimension
D. Inability to distinguish colored visual stimuli
E. Increased risk of seizures from olfactory stimulation
8. Treatment of aquired dysphagia in a 67 year-old man with a left hemiparesis might include B
A. Chin tucking anf reclining feeding
B. Chin tucking and head turning to the affected side
C. Changing diet to one of soter and thinner consistency
D. Upright feeding and head turning to the nonaffected side
E. Use of long-term thermal stimulation and upright feeding

9. What is the most reliable physical signs of Myofascial Trigger Point Syndrome? A
A. A rope-like nodular in the muscle and pattern of reffered pain
B. Focal tenderness and pain recognition
C. Vasomotor and temperature changes
D. Limited of ROM
E. Muscle spasm
10. Using a computer mouse that is too far to the side or too high in the table will lead to: C
A. Anteroir shoulder subluxation
B. Thoracic outlet syndrome
C. Rotator cuff tendinitis
D. Bicipitalis tendinitis
E. Adhesive capsulitis
11. A 35 year-old package-delivery truck driver was present with the insidious onset of right
buttock and posterior thigh pain that aggravated sitting and using foot pedals. Examination
was reveal external rotation of the right shortening, but no weakness or sensory or reflex loss.
Treatment should include: E
A. Pelvic traction
B. A lumboscral corset
C. A right shoe insert and heel lift
D. Infrared to gluteal muscles
E. Stretching of the buttocks and his muscles
12. When burn wound is around the prenium and both lower extremities, the preferred position
are the following, EXCEPT: D
A. Hip abduction
B. Knee extension
C. Ankle dorsiflexion

245
D. Pillow under the knees
E. Hip in neutral position
13. Para Articular Heterotropic Ossification in SCI patient most commonly occurs at her
following joint: A
A. Hip
B. Knee
C. Ankle
D. Elbow
E. Shoulder
14. In patient with chronic lateral epicondylitis, the important consideration in therapy would be
on: E
A. Infrared
B. Immediate icing
C. Injection of corticosteroids
D. Use appropiate wrist strap
E. Strengthening and stretching of the wrist extensors
15. The most commonly employed diagnostic maneuvers for TOS are all the following EXCEPT:
B
A. Adson test
B. Yergason test
C. Hyperabduction tesst
D. Shoulder depression test
E. Three minutes arm elevation
16. The following are the characteristic of Duchene’s Muscular Dystrophy, EXCEPT: A
A. EMG shows high amplitude, long duration MUAP with decrease recruitment
pattern on effort
B. The child’s mother, his sisters and maternal aunts are potential carriers
C. Measurement of serum creatinin kinase is the most reliable test
D. It is tranmitted by x-linked recessive mode in inheritance
E. The disease affects males

17. What is the strongest single predictor of morality in aduls with pediatric onset disabilities? C
A. Inability to walk
B. Feeding problems
C. Intellectual disability
D. Head circumference
E. Presence of epilepsy
18. The maneuver to detect hip dislocation in infant, where begin with flexed to 90 degress the
hips is abduted, and the examiner’s index finger gently psushes up on the greater thorchanter,
is called C
A. Barlow maneuver
B. Galleazi maneuver
C. Ortolani maneuver

246
D. Hip click maneuver
E. Hip reduction maneuver
19. Which statement below is correct about spastic diplegia? B
A. Involves all four extremities
B. Have good walking prognosis
C. Significant mental retardation
D. Significant oromotor disfunction
E. Involves upper extremities more than lower extremities
20. The period which intense motor learning and basic language development occurs is at: E
A. Birth to sixth month
B. Birth to tenth month
C. Birth to one year of age
D. Birth to two years of age
E. Birth to three years of age
21. A 13 year-old girl is found to have scoliosis on routine physical examination. Spine
radiographs demonstrate a Cob angle 25 degree. The optimal treatment is: E
A. Lateral surface electrical stimulation woth trunk-strengthening exercise
B. Lateral surface electrical stimulation woth bracing for 6 hours a day
C. Radiologic follow-up every 4 to 6 months
D. Surgical instrumentation and bony fusion
E. Use Milwaukee brace for 23 hours
22. A child 13 years old has clinical sign : asymmetric face, shortening of sternocleidomastoid
muscles without tumor, deficit of head rotation > 30. The appropriate management : D
A. Deep heating
B. Stretching exercise
C. Aymmetric collar
D. Refer to orthopedic surgeon
E. Stimulation of the head movement
23. A good initial treatment plan for neurogenic claudication secondary to spinal stenosis would
include: A
A. Gabapentin
B. Acetminophen
C. Neurotropic drug
D. Narcotic pain medication
E. Nonsteroid anti-inflammation
24. Patient with pronator teres syndrome will have the following clinical signs, EXCEPT: B
A. Intrinsic muscle atrophy
B. Pain in the lateral dorsal forearm
C. Normal strength of the pronator teres muscle
D. Weakness of the median-innervated hand intrinsic
E. Decrease sensation in the first three digits and over the thenar area

25. The following nerve is a brach of the posterior cord of the brachial plexus: B

247
A. Long scapular nerve
B. Subscapular nerve
C. Suprascapular nerve
D. Levator scapular nerve
E. Musculocutaneous nerve
26. Pronator Teres syndrome is caused by entrapment of the following nerve: C
A. Ulnar nerve
B. Radial nerve
C. Median nerve
D. Anterior interosseous nerve
E. Posterior interosseous nerve
27. Tic doloreaux is caused bt pathology of the following cranial nerve: D
A. II
B. III
C. IV
D. V
E. VI
28. The best treatment of spasticity in SCI is: E
A. Botox im
B. Baclofein iv
C. Baclofen oral
D. Diazepam iv
E. Baclofen intrathecal
29. Which of the following risk factors os the most associated with a higher incidence of Deep
Vein Thrombosis (DVT) in adult with a stroke? E
A. Male sex
B. Female sex
C. Increased age
D. Smoking history
E. Increased motor weakness
30. Which symptom is characteristic of vascular claudication rather than neurogenic claudication?
A
A. Spasm and pain hamstring muscle
B. Calf pain in evoked with uphill walking
C. Inclined treadmill walking decrease the pain
D. Thigh cramping is noted with prolonged standing
E. A change in position is neede to decrease the pain
31. Regarding exercise in congestive heart failure, which of the followng is true? A
A. Rapid hemodynamic changes many occur during warm-water aquatic therapy
B. Exercise heart rate should be at 80% to estimated maximum heart rate
C. Warm-up and cool-down periodes should be shortened
D. Strengthening should be done isommetrically
E. Blood pressure monitoring is not necesary

248
32. The Center for Disease Control classifies adult HIV infection by both clinical and CD4
parameter as below: A
A. Asymptomatic, CD4 >500 cells/mm3
B. Asymptomatic, CD4 <500 cells/ mm3
C. Symptomatic, CD4 <200 cells/ mm3
D. Symptomatic with AIDS, CD4 <400 cells/ mm3
E. Symptomatic without AIDS, CD4 400-500 cells/ mm3
33. The clinical signs of COPD patient as below A
A. Exertional dyspneu often occurs when the FEV1 is less than 1500 ml
B. Arterial blood gases may normal during exercise
C. FEV1/FVC is more than 70% predicted normal
D. TLC are generally decreased
E. Residual volume decreased

34. For pulmonary patients, clinical exercises testing have all the benefit below, EXCEPT: A
A. To determine function of the lung
B. To monitor the result of rehabilitation program
C. To prescribe the intensity of exercise programme
D. To rule out the presence of significant cardiac disease
E. To rule out the presence exercise-induced bronchospasm
35. You got referral a 60 year-old cancer patient with an acute deep vein thrombosis (DVT) in the
right upper limb, secondary to a long-standing central venous catheter. What therapy
restriction would you recomended for the patients? D
A. No restriction
B. Bed rest for 10-12 days to allow for clot maturation
C. Bagin resitive exercise 12-24 hours after the patient is therapeutic on an anticoagulant
D. Limit therapy to ambulation, balance and ADL training if anticogulant is medically
contraindicated
E. No activity restriction, since upper limb DVT’s have a low likelihood of causing a
pulmonary embolus
36. A 6 year-old boy with hemophulia has suffered a reccurent hemarthrosis of the knee. After
providing adequate factor VIII replacement, the most important therapy on the first 48 hurs is
A
A. Immobilization
B. Active assistive ROM exercise
C. Icing and passive ROM exercise
D. Ambulation with full weight bearing
E. Ambulation with partial
37. Children with Tetralogy of Fallot with assume the squatting position to relieve exercise
induced dyspnea D
A. Decrease heart rate
B. Increase inspiratory capacity
C. Decrease pulmonary artery pressure
D. Increase peripheral vascular resistance and there by decreases right to left shunt

249
E. Reduces the energy requirement of the capacity by lowering the center of gravity
38. A child with a congenital transverse radial limb deficiency should have in initial prosthesis fit
at what development stage? A
A. At the time of first sitting independently
B. At the time of starting kinder garden
C. At soon as possible after birth
D. At the time of initially walking
E. At the time of initially standing
39. Acquired subluxation or dislocation of the hips in spastic cerebral palsy is usually due to
muscular imbalance and pull of the: C
A. Knee extensor and hip abductors
B. Hip flexors and tensor fascia lata
C. Hip flexor and hip abductors
D. Hip extensors and hip abductors
E. Knee flexor and hip abductors
40. Physical examination findings that are consistent with hypotonia in a term infant include all of
the following EXCEPT: D
A. Head lag when pulled to sitting position
B. Decrease spontaneous movements
C. Frog leg position of the legs
D. Episthotonus
E. Weak suck

41. A 65 year-old woman with breast cancer has a solitary metastatic lesion at the left rib cage.
Among the following, the first-line pain medication in this setting is: C
A. Ibuprofein
B. Amitriptyline
C. Acetaminophen
D. Morphin sulfate
E. Local corticosteroid injection
42. The following are the contraindicated physical activities in the osteoporotic patients, EXCEPT
D
A. Golf swing
B. Running
C. Jumping
D. Walking
E. Jogging
43. The following are the physiologic changes in the aging process, EXCEPT B
A. Lower flexibility
B. Higher flexibility
C. Lower reaction time
D. Higer residual volume
E. Higher resting blood pressure

250
44. The use of opiod analgesics for chronic pain: E
A. Is contraindicated
B. Demonstrates a threshold, but not a ceiling effect
C. May allowed only by intramuscular administration
D. Is most effective when given on as-needed dosing regimen
E. May be no more effective than nonsteroid anti-inflammatory drugs
45. The activity of climbing up the stairs requires the following amount of METs: C
A. 10 METs
B. 8 METs
C. 6 METs
D. 4 METs
E. 2 METs
46. The respiratory function of patient with Ankylosing Spondylitis in characterized by: A
A. Diminished chest expansion
B. Increase airways obstruction
C. Moderate dyspnea on exertion
D. Increase use of accessory muscle
E. Decrease in diaphragmatic excursion
47. The following condition is NOT a contraindication for exercise program prescribe to a
diabetic patient with cardiovascular disease A
A. Blood sugar: 180-250 mg%
B. Blood sugar: >300 mg%
C. Blood sugar: <89 mg%
D. Acidosis
E. Ketosis
48. For chronic lung disease various walking test with sub maximal exercise have all the benefit
below, EXCEPT: B
A. To detect exercise intolerance
B. To detect limitation in lung function
C. To know and detect functional limitation
D. To evaluate reconditioning exercise programme
E. To prescribe rhe intensity of exercise programme
49. For patient with Congetive Heart Desease, which of the following statements is accurate? B
A. Most of the important resulting from regular exercise is within the myocardium
B. Exercise capacity is improved because of peripheral adaptations
C. These patient can never expect improved physical fitness
D. Strengthening exercise is prescribe for these patients
E. Complete bed rest is prescribed for these patients
50. A 29 year-old cross-caountry runner experiences new-onset paint in the region of the first
metatarsal. Exam revelas tenderness to the first metatarsal head worsened by passive great toe
extension. The cause of her sesamoid injury include of the following, EXCEPT: D
A. Pes planovalgus

251
B. Early hallux rigidu
C. Excessive pronation
D. Excessive supination
E. Gastroc-soleus tighness
51. A 20 year-old football player reports anterior shoulder pain during a gam. He completes the
game, but radiographs after the game revealed a type 2 acromioclavicular joint sprain. How is
a type 2 acromioclavicular joint injury define? C
A. Dislocation of corococlavicular joint
B. Acromoclavicular and corococlavicular ligaments are both intact
C. Acromoclavicular and corococlavicular ligaments are both distrupted
D. Acromoclavicular ligament is distrupted, but the corococlavicular ligament is intact
E. Acromoclavicular ligament is intact, and the corococlavicular ligaments are both
distrupted
52. A 55 year-old runner present with pain in the reion of the Achilles tendon. The third such
episode over the past 4 years. The Achilles tendon appears swollen. Appropiate initial
management would include: D
A. Surgery
B. Cortisone injection of the Achilles tendon
C. Cortisone injection of the Achilles tendo sheath
D. Use of a small heel with initiation of gastro-soleus stretches as tolerated
E. Immobilization in an ankle plantar-flexed position with limited weight bearing
53. The following are true statements about ankle sprain, EXCEPT: D
A. The most common injured ligament in anteriois talofibular ligament (ATFL)
B. The provocative tests are anteroir drawer test and talar tilt test
C. The mechanism of injury is inversion on a plantar flexed foot
D. The mechanism of injury is eversion on a plantar flexed foot
E. History of rolling over the ankle
54. Which the physiologic factor in the elderly exacerbates orthostasis? D
A. Decreased in arterial stiffness
B. Decreased creatinine clearance
C. Decreased peripheral resistance
D. Decreased baroreceptor response
E. Decreasedlower limb muscle strength
55. The American Geriatrics Society’s goal of diabetic control in elderly includes: D
A. Tight hypoglycemic control
B. Tight Hemoglobin A1c below 7 %
C. Fasting plasmaglucose below 120mg/dL
D. Reduction of cardiovascular complications
E. Reduction of neuromuscular complications
56. In prescribing pain treatment for elderly patients, it is best to use which approach? B
A. Start with NSAID
B. Start low and go slow for all medications
C. Start with aspirin, which os safe and effective

252
D. Avoid use of acetaminophen, due to licer toxicity
E. Use high-dose, short acting narcotics initially to get the pain under control
57. The following are the risk factors developing NSAID-related gastropathy, EXCEPT: E
A. Previous history of uncomplicated ulcer
B. Concurrent use of corticosteroid
C. High-dose NSAID therapy
D. Concurrent use of aspirin
E. Age under 40 years
58. A 69 year-old woman wishes to reduce her risk of compression fractures and is interested in
taking alendronate. You inform her that D
A. Alendronate increase BMD by increasing osteoblastic activity
B. Gastrointestinal absortion of alendronate is facilitated by calcium
C. Although fracture risk is reduced with the use of alendronate, BMD is relatively
unaffected
D. Use alendronate by woman with existing vertebral fractures decreases the risk of
further fracture
E. The most common side effects associated with use of alendronate involve the
cardiovascular and neurologic systems
59. Which electrodiagnosis findings is associated with a poor prognosis in Guillain-Barre
Syndrome? A
A. Compound MUAP less than 10% normal
B. Significant delayed conduction velocity
C. Abscence of late response F or H wave
D. Large fibrillation potentials
E. Nascent potential
60. Typical median nerve conduction changes in patients with mild carpal tunnel syndrome show:
A
A. Prolonged sensory distal latency
B. Prolonged motor distal latency
C. Low amplitude motor response
D. Abnormal F waves
E. Abnormal H waves
61. The primary neurotransmitter of afferent pain neurons in the spinal cord appears to be D
A. GABA
B. Endorphin
C. Enkephalin
D. Substance P
E. Beta-lipoprotein
62. This disorder is usually seen in preadolescent athletes who participate in activities such as
jumping or running. The disorder is a result of recurring microtrauma from the quadriceps
contracting: E
A. Pes anserinus
B. Ligamen sprain

253
C. Pes anserinus bursitis
D. Tibial plateau fracture
E. Osgood-schlatter disease
63. Ankle sprains are ubiquitous in sport and are the most common athletic injury. The most
commonly injury for this disorder is the: C
A. Achilles tendon
B. Calcaneo fibular ligament
C. Anterior talofibular ligament
D. The posterior talofibular ligament
E. The strong medial deltoid ligament
64. Which of the following activities would be the most difficult to return to following an anterior
cruciate ligament (ACL) injury? C
A. Swimming
B. Speed cycling
C. Baseball pitching
D. Marathon running
E. Cross-country skiing
65. A 22 year-old female gymnast present to your clinic after a patellar dislocation during
practice. She was treated in the emergency room with reduction of the patella and
immobilization. Radiographs and MRI of the knee are negative fracture or evidence of
osteochondral lesions. You choose to treat her with immobilization for 2 weeks and the begin
physical therapy. The most appropiate therapy recommendation is to focus on improving: D
A. Strength of the iliopsoas
B. Strength of the hamstring
C. Felxibility of the biceps femoris
D. Strength of the vastus medials
E. Flexibility of gastrocnemius-soleus complex
66. A 35 year-old jogger experinces new-onset a traumatic pain along the lateral aspect of the
knee. Pain is reproduced and accentuated when the examiner presses proximal to the joint line
and simultaneously brings the knee in and out of 20 to 30 of knee flexion. The most likely
diagnosis is: C
A. Lateral plica syndrome
B. Lateral meniscus strain
C. Iliotibal band syndrome
D. Distal femur stress fracture
E. Sprain of collateral lateral ligament
67. The weakest spot in the posterior lumbar region which is not strengthened by posterior
longitudinal ligament is: E
A. L1-2
B. L2-3
C. L3-4
D. L4-5
E. L5-S1

254
68. Maximal flexion of the knee in the gait cycle is reched: C
A. In mid stance
B. In heer strike
C. In the middle of swing phase
D. Immediately after the heel strike
E. During acceleration in swing phase
69. The following joint is classified as trochoginglimus joint: C
A. Hip joint
B. Wrist joint
C. Knee joint
D. Ankle joint
E. Shoulder joint
70. The smallest unit of muscle contraction is: D
A. Motor unit
B. Sarcomere
C. Sarcolemna
D. Actin, myosin
E. Intrafusal fibers
71. What is the most common abnormal electrodiagnostic finding in lumbar spinal stenosis? E
A. Normal H-reflex
B. Abnormal sensory nerve conduction test
C. Decreased chronodispersion of F-wave test
D. Presence of fasciculation and cramp potentials
E. Fibrillation potentials in multiple and bilateral myotomes
72. Which information is useful in the electrodiagnostic evaluation of a 9 year-old child with
spinal cord injury who has no radiologic abnormality but has sensory abnormalities? B
A. Repetitive stimulation recording
B. Somatosensory evoked potential
C. Sensory nerve conduction study
D. Motor nerve conduction study
E. Magnetic evoked potential
73. Heat therapy through convection method could be generated by the following modality: D
A. USD
B. Infra red
C. Cold laser
D. Whirlpool
E. Ultraviolet treatment
74. When standing upraight in water with surface level at the level of papilia mammae, the
relative body weight is: C
A. 5% weight bearing
B. 15% weight bearing
C. 25% weight bearing
D. 35% weight bearing

255
E. 45% weight bearing
75. Which finding is a relative contraindication to cyrotherapy: D
A. Pain
B. Edema
C. Acute heamtoma
D. Impaired sensation
E. Acute inflammation
76. What the role of the gluteus maximus in providing spine stability? D
A. It controls stance phase
B. It primarily functions as a hip flexor
C. It controls repetitive lumbar deceleration
D. It controls acceleration and deceleration
E. It seves as a primary muscle to fascilitate lumbar flexion
77. The following muscle is classified as monoarticular type: A
A. Soleus
B. Gastrocnemius
C. Flexor carpi radialis
D. Flexor pollicus longus
E. Flexor digitorum superficialis
78. When you see a patient every time he is walking always holding his thigh during the stance
phase, most probaly he has lesion of the following nerve: C
A. Tibial
B. Sciatic
C. Femoral
D. Common peroneal
E. Tibial and peroneal
79. The best rational choice of wheelchair for a C8 tetraplegia ASIA A is: E
A. Regular wheelchair with solid type
B. Hand-operated electrical wheelchair
C. Mouth-operated electrical wheelchair
D. Regular wheelchair with aired/inflated tyre
E. Regular wheelchair with vertical bars on its handrims
80. The immediate post-surgical fitting with pylon leg is especially directed to: E
A. Shape a good stump
B. Accelerated the stump healing
C. Avoid the development of phantom pain
D. Accelerate the cessation of phantom sensation
E. Promote early return of propioceptive function
81. Part of shoe which hold firmly the calceneous is the: A
A. Counter
B. Quarter
C. Shank

256
D. Vamp
E. Heel

82. The following is NOT used for lymphedema: B


A. Compression garment
B. Superficial heating
C. Elevation
D. Stroking
E. Massage
83. A therapeutic modality theorixed to exert its main effect by the “gate control theory” of pain
is: B
A. USD
B. TENS
C. Contrast bath
D. Laser therapy
E. Magnetotherapy
84. Biofeedback has been effective for treating strroke patients with, EXCEPT: D
A. Spasticity
B. Visual neglect
C. Impaired balance
D. Impaired hand function
E. Oropharyngeal incoodination
85. Spesific therapeutic exercise for scoliosis are all the following, EXCEPT: B
A. Done in and out of the brace
B. Stretching exercise of the convex side
C. EDLF exercise is done in the kneeling position
D. Muscle strengthening exercise of the convex side
E. Abdominal muscle strengthening exercise is include
86. The following are guidelines for exercise during pregnancy, EXCEPT: D
A. Avoid supine exercise after the first trimester
B. Regular exercise is preferable to intermittent activity
C. Stop exercising when fatigued and avoid exhaustive exercise
D. Prolonged periods of motionless standing will minimize the risk of injury
E. Pregnancy should maintain adequate intake for basa, exercise and pregnancy needs
87. The following is guide lines for exercise for persons with HIV injections: A
A. Aerobic exercise that produces a negative energy balance is contraindicated
B. Aerobic exercise for person with estabilished clinical weght loss in indicated
C. Aerobic exercise for person with a lack cardiopulmonary reserve is indicated
D. Person with symptomatic HIV-seropositive are restricted activity and competition
E. Person symptoms prior to development of AIDS are encouraged to exercise and
competition

257
88. A patient with DM has painful, swollen ankle, worst with weght bearing, X-ray shows
fragmentation of the articular surface, synovial fluid is clear. The choice of orthosis is: D
A. AFO
B. KAFO
C. HKAFO
D. PTB orthosis
E. Orthopedic shoes
89. In PTB socket the patient’s stump is held in how many degree knee flexion? A
A. 15 degree
B. 20 degree
C. 25 degree
D. 30 degree
E. 35 degree

90. A boy 6 year-old with Prader-Willi syndrome, associated hypotonia, has hypermobile flat feet
requiring treatment. These are best managed by: B
A. Double-upright short leg brace and medial T strap
B. Molded plastic shoe insert for the talus support
C. Othopedic surgery for a triple arthrodesis
D. High-top shoes with Thomas heel
E. Soft plastizole shoe insert
91. The propioceptive Neuromuscular Fascilitation (PNF) is a neurophysical therapeutic approach
for CP, develop by: E
A. Fay
B. Rood
C. Bobath
D. Brunnstrom
E. Kabat and knott
92. Parkinson disease patients may improve in walking by the following exercise: E
A. Voyta
B. Knott
C. kabat
D. Bobath
E. Frenkel
93. Diabetic patient with peripheral neuropathy SHOULD NOT perform the following exercise:
A
A. Jogging
B. Rowing
C. Bicycling
D. Swimming
E. Light weight lifting
94. What is the best predictor of fracture in a person with osteoporosis C
A. Onset

258
B. Recent falls
C. Prior fractures
D. Low body weight
E. Low physical activity
95. A 27 year-old manual laborer was present with a 6-month history of right shoulder pain. He
had a past medical history of right shoulder dislocation after a water-skiing accident 3 years
ago. On physical examination, he had normal strength and sensation with symmetric reflexes.
The shoulder apprehension test was positive. Impingement tests and the O’Brien active
compression test were negative. Which diagnosis was most consistent with this presentation.
D
A. Rotator cuff tendinitis
B. Glenohumeral osteoarthritis
C. Dislocation of biceps tendon
D. Anterior-inferior labrum tear
E. Superior labral anterior to posterior (SLAP)
96. A 25 year-old man camoes to your office for evaluation of low back pain. As part of the pain
physical examination, you mark a point at he L5 vertebral body and another point midline 10
cm above. You ask him to flex forward maximally while keeping his knees extended and
measure the distance between the two points. This distance is 13.5 cm. You suspect he may
have what diagnosis? C
A. Scheuermann disease
B. Lumbar herbiated disc
C. Ankylosing spondilitis
D. Spondilitis teubercolosis
E. Lumbar spondylolisthesis
97. Spurling’s maneuver: D
A. Assese for upper motor neuron involvement
B. Applies tension on the nerve root
C. Is not sensitive for radiculopathy
D. Reproduces radicular symptom
E. Distract the neuroforamen
98. With adhesive capsulitis of the shoulder, the physical examination is most consistent with
limitations in: C
A. Active range of motion only
B. Passive range of motion only
C. Both active and passive range of motion
D. Reduced strength and loss of active range of motion
E. No loss of range of motion but reduced strength of the muscles of the shoulder girdle
99. Which of the followng is the correct description of Yergason’s test? B
A. The examiner provides resistance against pronation of the forearm with the elbow in
extension
B. The examiner provides resistance against supination of the forearm with the elbow
flexed at 90 degree

259
C. The examiner provides resistance against supination of the forearm with the elbow in
extension
D. The examiner provides resistance against pronation of the forearm with the elbow flexed
at 90 degree
E. The examiner provides resistance against flexion of the elbow at 90 degree and the
forearm in complete supination
100.The muscle that has the important role in push-off is D
A. Iliopsoas
B. Hamstring
C. Quadriceps
D. Triceps
E. Tibialis anterior

ANSWER :
A. IF NUMBERS 1,2,3 ARE CORRECT
B. IF NUMBER 1,3 ARE CORRECT
C. IF NUMBER 2,4 ARE CORRECT
D. IF ONLY NUMBER 4 IS CORRECT
E. IF ALL NUMBERS ARE CORRECT

101.In patients with de Quervain’s tenosynovitis, which tendons are involved? C


1. Abductor pollicis brevis
2. Abductor pollicis longus
3. Extensor pollicis longus
4. Extensor pollicis brevis
102.In rheumatoid arthritis, the body releases a lytic substance that destroy the following: E
1. Bone
2. Joint capsule
3. Tendon sheath
4. Articular cartilage
103.Trigger finger occurs near the A1 pulley of the B
1. Deep flexor tendon
2. Deep extensor tendons
3. Superficial flexor tendon
4. Superficial extensor tendons
104.The laboratory test most commonly associated with the diagnosis of ankylosing spondylitis
is/are: D
1. Rheumatoid factor
2. Antinuclear antibody
3. Antimitochondrial antibody
4. HLA-B27 histocompatibility antigen
105.If somebady has difficulties in chewing the food most probably he has problem with the
following cranial nerve: B

260
1. XII
2. IX
3. V
4. VII
106.The characteristic of Parkinson’s disease is/are: E
1. Rigidity
2. Mask face
3. Bradykinesia
4. Pill-rolling tremor
107.According to ICF (International Claasification of Functioning, Disability and Health) in stroke
patient, the following is (are) include in the loss of body functions and structures dimension: B
1. Cognitive impairment
2. Difficulty in ADL
3. Decubiti ulcer
4. Comordibities
108.The following is(Are) true about method is stroke rehabilitation: A
1. The principle of constraint induce movement therapy to avoid the learned nonuse of
the paretic side
2. The principle of mirror therapy is that movement of the affected limb can be
stimulated via visual cues originationg from the opposite side of the body
3. Functional Electrical Stimulation (FES) of the paretic limb enhances plasticity
4. Propioceptive Neuromuscular Facilitation (PNF) emphasized synergistic patterns of
movement that develop during recovery

109.The muscle imbalance that cause cruch gait is/are: A


1. Tight hip flexor
2. Tight hamstring
3. Weak quadriceps
4. Excessive plantarflexion
110.The goals of rehabilitation management for brachial plexus injuriers in children are to: A
1. Prevent contractures, which can develop because of muscle imbalance
2. Increase the child’s awareness of the paretic limb
3. Assist normal development and function
4. Nerve continuity
111.Both the hard and the soft palate (the velum) are important structures to consider for
evaluating the mechanics of sucking and feeding of infant. The role(s) of the hard palate
include the following: D
1. To form the anterior seal
2. To maintain the shape of the tongue
3. Provide lateral boundaries for food in the tongue
4. Assist positioning and stability of the nipple
112.Absolute contraindication(s) for inpatient and outpatient exercise program according to
ACSM include: E

261
1. Ustable angina
2. Uncontrolled arrhythmia
3. Exacerbation of Congestive Heart failure
4. Resting blood pressure >200/110 mmHg
113.The benefit(s) of exercise on diabetes includes: E
1. A decrease in morbidity and mortality
2. An improvement in blood presssure
3. An increases in insulin sensitivity
4. A decreases in hyperlipiderma
114.Which is/are the following including in classification of Cyanotic Congenital Heart Disease?
D
1. Patent ductus arterious Botalli
2. Ventricular septal defect
3. Atrial septal defect
4. Tetralogy of Fallot
115.Adeverse effect(s) of hospitalization in the elderly include: A
1. Insomnia
2. Disorientasion
3. Emotional sequele
4. Decreased incidence of iatrogenic complication
116.Rehabilitations of chronic back pain in patients with osteoporotic vertebral compresion
fracture is/are: E
1. Practice good body mechanics
2. Medications for pain control and bone enhancement
3. Avoid activities such as forward bending that increase compression on vertebrae
4. Strengthening exercise for the trunk, pelvis, thighs muscles and lower extremities
117.The following of exercise(s) is/are physioball exercise(s) for the Core: E
1. Pelvic bridging
2. Modified push-up
3. Abdominal crunch
4. Balancing exercise while seated
118.The following resistance training guideline(s) is/are recommended for apparently healthy
adult: C
1. Perform these exercise every day
2. Perform every exercise through a full ROM
3. Perform a minimum of 3 separate exercise that train the minor muscle groups
4. Perform concentric phase and eccentric phase portion of the resistance exercise in a
controlled manner.
119.Muscle proteins involved in muscle contraction does include: A
1. Actin
2. Myosin
3. Troponin

262
4. Dystrophin
120.Fibrilation potentials: D
1. Always indicate denervation
2. Always present with disease
3. Indicate abnormal muscle
4. Indicate membrane instability
121.From the nerve conduction studies we can get an impression of: A
1. The continuity of the peripheral nerve
2. The condition of the myelin sheeth
3. The location of a lesion or compression
4. The condition of central nervous system
122.A previously active 63 year-old woman was undergo an uncomplicated comented total hip
replacement. The following orders(s) were appropiate: A
1. Practice swing and stance in parallel bar
2. Progress to crutch walking when safe in parallel bars
3. Supervised active-assistive ROM exercise of the hip
4. Hamstring stretch by forward bending over extended legs
123.The following is/are about idiopathic scoliosis: B
1. Genetic factor is a sex-linked trait with incomplete penetrance and variable
expressivity
2. A sex-linked trait can be transmitted by a father, to either a son daughter
3. The curves have strong tendency to progress rapidly during the adolescent growth
spurt
4. Juvenile idiopathic scoliosis occurs between ages 2 and 8
124.The following is/are the test for diagnosis Achilles tendon rupture: C
1. Anterior drawer test
2. Hiperdorsoflexion sign
3. Posterior drawer test
4. Thompson (Simmond test)
125.Rehabilitation management of osteoporosis patients depend in: E
1. The risk factors for osteoporosis
2. The degree of frailty and propensity to fall
3. Accurate determination of the degrees of bone loss
4. The capacity for participation in ADLs and safe exercise
126.Which ergonomic recommendation for computer station is/are appropiate for an injured
worker with upper extremity cumulative trauma disorder A
1. Forearm position horizontal to the floor
2. Mid-line positoning of the keyboard
3. Support neutral position of the wrist
4. Elevated positioning of the mouse
127.Osteoarthritis may occur in the following joint: A

263
1. Ankle joint
2. Subtalar joint
3. Facet joint of the vertebra
4. Uncovertebral joint of luschka
128.A 60 year-old woman had a left total hip arthroplasty 4 weeks ago. During her gait evaluation,
she is noted to have a left lateral trunk lean during left stance phase. This gait deviation is/are
likely a result of weakness in which left lower limb muscle(s)? D
1. Gluteus maximus
2. Gluteus minimus
3. Tensor fascia lata
4. Gluteus medius
129.You are about to examinte a child who sustained a fracture during a fall from a swing set. The
serious complication following supracondylar fracture of the humerus in a child is/are: D
1. Gunstock deformity
2. Tardy ulnar nerve palsy
3. Nonunion of the fracture
4. Volkmann’s eschemic contracture
130.The examination is the initial step in rehabilitation management for the child with Muscular
Dystrophy: E
1. Aerobic capacity
2. History with family concern
3. Asistive and adaptive device
4. Community, school and play integration
131.The following statement of Brachial plexus palsy in infant and child is/are true: E
1. Injury to the upper plexus occurs in 90% of cases
2. Must enrolled in a closely supervised rehabilitation program
3. Neurosurgery may benefit infants who have severe proximal weakness but a good
hand
4. The presence of sensory nerve potentials in a clinically arm is heplful in delineating
root avulsion

132.The following is/are true statement(s) about SWD: E


1. In condenser field heating, tissues that offers the greatest resistance to current flow
(i.e fat and skin) tends to overheat
2. In induction field heating, tissues that are high electrlutic content (i.e muscle)
respond best to this method
3. In condenser field heating, the tissues between the two electrodes is in the series
4. In induction field heating, the tissues are in parallel circuit
133.The following is/are the conditon(s) of contraindication for SWD: E
1. Metal implant
2. Contact lens
3. Pace maker

264
4. Gonad
134.The obejctive(s) for using cervical traction include: A
1. Relief of compressive effects of normal pressure
2. Enlargement of the foramina intervertebralis
3. Strech of the paracervical muscle
4. Decreases in joint propioception
135.The following(s) is/are the contraindication for participation in physical exercise of patients
with congenitaal heart disease: E
1. Aortic stenosis
2. Severe cyanosis
3. Complex arrhythmia
4. Severe pulmonary hypertension
136.The following(s) is/are the indication of postural drainage as a from of chest physical therapy
to enhance the flow of mucus out of airways A
1. Polycystic lung disease
2. Chronic Bronchitis
3. Brochiectasis
4. Emphysema
137.A patient develops angina with low level exercise. Nitroglycerin gives symptomatic relief
which allows exercise to continue for alaonger period time. Which the following is/are true: A
1. Nitroglycerin can be used as a reguler prophylaxis to increase work tolerance
2. In evaluating exercise stress tests, any nitroglycerin should be used identically for
each test
3. Nitroglycerin increases the exercise tolerance by up to 300 kilopond meters/min
before symptoms of electrocardiographic occur
4. Nitroglycerinshould not be used before exercising. It may mask the presence of angina
and increase the chance of sudden death
138.A circumducted gait pattern in a patient with an above-knee amputation is/are likely caused by
D
1. Inadequate friction on the prosthetic knee unit
2. An externally rotated prosthetic knee unit
3. A Silesian suspension belt that too tight
4. Inadequate socket suspension
139.An orthosis is a device to achieve one or more of the following: A
1. Prevent deformity
2. Improved function
3. Reduced axial load
4. Mobilize musculoskeletal segment
140.The following features of running shoe may diminish symptom associated with a pronated
foot: A
1. Board and straight last construction
2. Motion control heel counter

265
3. High-density insole medially
4. Lateral wedges

141.The characteristic changes postur in elderly: A


1. Slighly flexed of knees and hips
2. Increasing thoracic kyphosis
3. Forward head posture
4. Narrowed base of support
142.The following is(are) the prescription guideline(s) for elderly people E
1. Walking is an exellent mode of exercise for many elderly people
2. The exercise modality should be does not impose excessive orthopedic stress
3. The activity should be accessible, convenient and enjoyable to the participant
4. Stationary cycle exercise may be advantageous for those with reduced ability to
tolerate weight bearing activity
143.Contraindication(s) to manipulation of the cervical spine include of: A
1. Osteoporosis
2. Herniated cervical disc
3. Acute soft tissue injury
4. Strain paracervical muscles
144.Mr. A 60 year-old with COPD is undergoing exercise training in your outpatient clinic.
During ambulation training he complains fatigue and dyspnea. Oxymetry reveals 80% O2
saturation. For the next exercise session you would: D
1. Start postural drainage and percussion
2. Add digoxin to prevent arrhythmias
3. Initiative incentive spirometry
4. Add oxygen via nasal cannule
145.Pursed lip breathing is often helpful in the management of the patient with COPD because it:
B
1. Preventing air trapping due collapse of small airways during expiration
2. Decrease gas exchange in alveoli
3. Maintaning high airways pressures
4. Increase vital capacity
146.The following technique(s) is/are used to measure the body composition: E
1. Bioelectrical impedance analysis
2. Dual energy x-ray absorptiometry
3. Near-infrared interactance
4. Skinfold measurement
147.The following of initial program of postoperative posterior cruciate ligament is/are: E
1. Initial bracing in full extension
2. Quadriceps isometric exercises
3. The day after surgery pwb with crutches as tolerated
4. In the early phase of rehabilitation closed chain exercises

266
148.The clinical sign(s) of Alzheimer is/are: E
1. Recent memory deficit
2. Language impairment
3. Psikomotor impairment
4. Cognitive dysfunction
149.The blindness occured in leprosy patients is/are caused by the involvement of the following
nerve(s) B
1. Facial nerve
2. Optic nerve
3. Trigeminal nerve
4. Occulomotor nerve
150.The following is/are the signs of autonomic dysreflexis: A
1. Pounding headache
2. Sense of chest fullnes
3. Blood pressure 220/120
4. Heart rate 120-140/minute

267
MEI 2015

1. The function of medial T-strap in AFO …


a. Equinus varus control
b. Pes valgus control
c. Pes planus control
d. Pes varus control
e. Drop foot control

2. When Should upper extremity prosthesis fitting be initiated in the adult?


a. When residual limb strength is full
b. When the patients request prosthetis
c. Within the first month after amputation
d. When residual volume has stabilized
e. When there is no phantom pain

3. The common “knuckle-bender orthosis” is used to:…


a. Stretch flexion contracture at the DIP joint
b. Stretch flexion contracture at the PIP joint
c. Stretch flexion contracture at the MCP joint
d. Stretch extension contracture at the DIP joint
e. Stretch extension contracture at the MCP joint

4. Regarding diabetic foot care , which of the following statements is FALSE?


a. In the precense of foot deformities the physician should consider customs shoe insert
and/ or footwear
to distribute pressure evenly over the foot
b. It is advisable that patients minimize or eliminate weight bearing during the healing
period of skin
infections
c. Patients should be encouraged to routinely visit a podiatrist for nail and general foot
care
d. Tennis or running shoes should be avoided in diabetic patients
e. In the precense of sensory disturbance, SWD is contraindicated

5. A 25- years old man has had level pain for 1 month. He runs approximately 40 km weekly. The
pain is
most severe when he first stand up and walks after sitting or lying down. Palpation on the medial
palmar
surface of the heel increase the pain. He walks with a limp. Which of the following is the most
likely cause of
pain?
a. Heel spur
b. Achiles tendinosis
c. Impingement of the posterior sacral nerve root
d. Inflammation of the origin of the plantar fascia

268
e. Articular cartilage degeneration of the calcaneocuboid joint

6. A 26- years old mother of 18 month-old twins presents with a complain of pain in her right
shoulder,
which started several weeks ago. The pain progressed significantly, and she finds it hard to
perform activities that involved reaching over her head. Physical examinations abnormalities
include pain in anterior aspects of the shoulder with palpation as well as pain resisted elbow
flexion and supination of the arm. What is the most likely diagnosis for this patients?
a. Osteoarthritis
b. Bursitis calcanea
c. Bicipitalis tendinitis
d. Incomplete rotator cuff tear
e. Degenerative disease of the acromioclavicular joint

7. If the drop arm test is positive, this suggested to an injury of all the following muscle , EXEPT:
a. Subscapularis
b. Supraspinatus
c. Infraspinatus
d. Teres mayor
e. Teres minor

8. Which of the following is felt to contribute to the onset of lateral epycondylitis ?


a. Exessive extension of the elbow
b. Synovitis of the radiocapitellar joint
c. Entraptment of the radial sensory nerve
d. Mechanical breakdown of the extensor digitorum
e. Compressive overload from the rotating radial head

9. A pulmonary embolism which may occur in prolonged immobilization mostly due to:
a. Development of artherosclerosis
b. Developmentatrial fibrillation
c. Decreasing the total lung capacity
d. Decreasing of the lung vital capacity
e. Development deep vein thrombosis

10. What is the estimated rate of nerve regeneration ?


a. 0,1 mm/day
b. 1 mm/week
c. 1cm/week
d. 1 mm/day
e. 1 cm/ day

11. The primary afferent fibers coming from the nuclear chain of the muscle spindle have the
following conduction
velocity:
a. <40 m/sec
b. 40-60 m/sec
c. 61-70 m/sec

269
d. 71-80 m/ sec
e. 81-90 m/sec

12. In aging people which lung function increases?


a. Forced expiartory volume in 1 second
b. Forced vital capacity
c. Total lung capacity
d. Residual volume
e. Vital capacity

13. In elderly people there are such several physiological changes, which of the following is
TRUE?
a. Increase fat free body mass
b. Increase glucose tolerance
c. Decrease recovery time
d. Increase vital capacity
e. Increase reaction time

14. Regarding prescription of endurance exercise of elderly people , which of the following is
TRUE ?
a. Exercise 1-2 days in a week
b. At least 30 minutes of moderate - intensity exercise
c. Initially increase exercise intensity than exercise duration
d. Walking is not an excellent mode of exercise for many elderly with ability to tolerate
weight bearing
activity
e. Use the heart rate reserve method to calculate a target heart rate to provide a more
accurate estimate of
% of VO2max

15. The one of the following guideline resistance training for elderly people is NOT the
appropriate
statement: the one of the following guidelines resistance training for elderly people is NOT
the
appropriate statement:
a. Each set should involve 20-30 repetitions
b. Start with resistance of 50% of previous training intensity
c. Resistance training should performed at least twice a week
d. The first several resistance training sessions should be closely supervised
e. Perform at least 1 set of 8 to 10 exercises thas use all the major muscle groups

16. Lifestyle factors associated with bone density and increased fracture risk is:
a. Caffeine intake 50 mg/day
b. History of hypothyroidism
c. 2 alcoholic drinks per week
d. history of anticonvulsant drug
e. spending 6 hours a day on their feet

270
17. De quervain’s tenosynovitis affects which two tendons?
a. Abductor pollicis brevis and extensor pollicis brevis
b. Abductor pollicis brevis and extensor pollicis longus
c. Abductor pollicis brevis and abductor pollicis longus
d. Abductor pollicis longus and extensor pollicis brevis
e. Abductor pollicis longus and extensor carpi radialis

18. A 24 year old marathon runner reports lateral knee pain after hill training. Examination was no
effusion, and results of Ober’s test are positive. What is the most likely diagnosis?
a. Biceps tendinitis
b. Lateral meniscal tears
c. Popliteus synovitis
d. Peroneal nerve entrapment
e. Iliotibial band friction syndrome

19. Prior to returning a 54 year old man to participation in sports 1 year after total hip arthroplasty,
you would
recommend :
a. progressive resistive exercises for hip extensors
b. stretching of hip flexors and rotators
c. administration of calcitonin
d. myofascial release therapy
e. prophylaxis with NSAID

20. ACR kriteria of SLE, Except...


a. arthritis
b. malar rash
c. photosensitivity
d. raynaud’s phenomenon
e. persistent proteinuria > 500 mg/day

21. Most common symptom of cervical subluxation in Rheumatoid Arthritis


a. hyperreflexic bladder
b. pain radiating up into the occiput
c. asymmetric upper extremity weakness
d. numbness in the lateral aspect of the arm
e. dizziness when going from supine to standing

22. Anatomical Principles, wrist should immobilized in...


a. slight supination position
b. Slight extension position
c. Slight pronation position
d. normal flexion position
e. slight flexion position

23. Mechanical orthosis or hybrid system with lowest energy expenditure for ambulation for
persons with paraplegia ?
a. hip guidance orthoses

271
b. bilateral HKAFO
c. FES without orthoses
d. bilateral KAFO and forearm crutches
e. Reciprocating Gait Orthosis with Functional Electrical Stimulation (FES)

24. In providing foot orthosis (FO) for the rheumatoid patients


a. Modifications of positive model obtained are not indicated
b. Inversion eversion should be balanced during impression taking
c. The negative impression of the foot should be obatained with the patient standing
d. The durability of plastizote makes it the most suitable material for FO construction
e. The degree of metatarsal arch modification diminished with the severity of
metatarsophalangeal (MTP)
subluxation

25. The following statement is NOT the characteristic signs of conus medularis syndrome:
a. Perineal sensory deficit
b. Sphincter dysfunction
c. Loss of anal reflex
d. Pyramidal sign
e. Incontinentia

26. The mechanism of diazepam to reduce the muscle spasticity is by:


a. Stimulating the GABA- mediated inhibition in CNS
b. Inhibiting glutamate and aspartate in CNS
c. Inhibiting the extrafusal fiber
d. Inhibiting the pyramidal tract
e. Inhibiting the motor cortex

27. The cause of myasthenia gravis is:


a. Nerve cells are destroyed
b. Lesion in the muscle fibers
c. Lesion in the cerebral cortex
d. Lesion in the anterior horn of the spinal cord
e. Acethylcholine receptor in muscle cell is destroyed

28. The patient may complain of numbness, paresthetias or dysesthesis radiating to the first,
second, third and lateral fourth digits. Symptoms may exacerbated during sleep and relieved with
shaking. This condition found in:
a. Cubital tunnel syndrome
b. Carpal tunnel syndrome
c. Pronator teres syndrome
d. Cervical root syndrome
e. Guyon syndrome

29. Individuals with SCI are considered to be at risk for autonomic dysreflexia and orthotastic
hypotension if lesion on:
a. T6 or above

272
b. T7
c. T8
d. T9
e. T10

30. One of the effects of baclofen as muscle relaxant is via the mechanism of:
a. To inhibit calcium ions release
b. To inhibit acetylcholine
c. To stimulate glutamate
d. To stimulate GABA
e. To inhibit GABA

31. In a resting healthy man, the ejection fraction is approximately?


A. 0,1
B. 0,2
C. 0,3
D. 0,4
E. 0,6

32. What is innervated by alpha motor neurons?


A. Intrafusal muscle fibers
B. Extrafusal muscle fibers
C. Primary spindle ending
D. Golgi tendon organ
E. Actin-myosin

33. The one of the following characteristic spastic diplegia of cerebral palsy is NOT the
appropriate statement :
A. Persistent obligatory infantile reflex activity beyond 18 months implied poor prognosis
for ambulation
B. Children with spastic diplegia have not adequate movement control of the arms
and hands
C. Children with spastic diplegia who did not sit by 4 year have not achieved ambulation
D. Sitting by 2 year was a good predictive sign of eventual ambulation
E. Children wih spastic diplegia have good prognosis for ambulation

34. Protective and postural reflexes (advanced postural reaction) is :


A. Plantar grasp
B. Foot hand placement
C. Head and body righting
D. Symetric tonic neck reflex
E. Asymetric tonic neck reflex

35. A-15-year-old girl is referred with her polyarthritis which has resulted in severe joint pains
despite appropriate medication. You are consulted for a physical medicine program. On
examination she is a cooperative child. No warm ankles, wrists, and digits, but there are limitation
in range of motion at all of these joints. Your prescription should include :
A. SWD for the ankles

273
B. Strengthen the grip in both hand
C. Paraffin baths for the hands, wrists, and ankles
D. Isotonic strengthening for the biceps and triceps
E. Ultrasound applied in water over the ankles, wrists, and the hands

36. The child has chief complaint of bowlegs that gradually progressing in severity and not
improving spontaneously. The deformity is called :
A. Clubfoot
B. Tibia vara
C. Knock knees
D. Genu recurvatum
E. Anteversion deformity

37. A common source of shoulder pain in shoulder impingement syndrome is :


A. Bursitis
B. Osteoarthritis
C. Short head of biceps
D. Long head of biceps
E. The rotator cuff tendon

38. In Thoracic Outlet Syndrome, if the Adson test is positive, this suggestion to the following
condition :
A. Hypertrophy of the splenius cervicis muscle
B. Hypertrophy of the m scalenus posterior
C. Hypertrophy of the m pectoralis mayor
D. Fracture of the clavicle
E. Cervical rib

39. The following below is NOT cause of the chronic ankle instability :
A. Ligamen laxity
B. Achilles tendinitis
C. Recurrent ankle sprain
D. Peroncal muscle weakness
E. Ankle propioceptive deficit

40. Special problem associated with humeral shaft fracture is :


A. Ulnar nerve injury
B. Radial nerve injury
C. Axillary nerve injury
D. Medianus nerve injury
E. Radial and axillary nerve injury

41. The most common cause of significant knee hemarthrosis after traumatic injury is :
A. Anterior cruciate ligament rupture
B. Lateral collateral ligament tear
C. Osteochondral fracture
D. Patellar dislocation
E. Meniscal tear

274
42. Parkinsonian feature which impaired automatic motor task performance is :
A. Bradykinesia
B. Hypotonia
C. Spasticity
D. Weakness
E. Dystonia

43. The most cranial nerve involved in Miller Fisher syndrome (variant of GBS) is :
A. Fascialis nerve
B. Abducent nerve
C. Trigeminal nerve
D. Oculomotor nerve
E. Hipoglosus nerve

44. The nerve most commonly injured with posterior or posterolateral approach for total hip
replacement is the
A. Superior Gluteal.
B. Obturator
C. Peroneus
D. Femoral
E. Sciatic

45. You are evaluating a patient with a nerve palsy. During evaluation, the patient recals a positive
Froment’s sign. Which nerve is injured?
A. Brachioradialis
B. Axillaris
C. Median
D. Radial
E. Ulnar

46. The major effect of the tendon lenghtening on spasticity is to


A. Weaken the muscle
B. Strengthen the antagonist
C. Alter the gamma bias of muscle
D. Increase the output of Golgi tendon organ
E. Increase the inhibitory influences on the alpha motor neuron

47. Atlanto-odontoid is classified as


A. Gynglimus joint
B. Ellipsoidal joint
C. Trochoid joint
D. Saddle joint
E. False joint

48. The normal gait pattern, the ankle is held in a neural position at
A. Toe off
B. Foot flat

275
C. Heel strike
D. midstance
E. midswing

49. How to exame the strength of 1st lumbrical-interosseous muscle?


A. Abducting the index finger againt resistence
B. Adducting the index finger againt resistance
C. Touching the base of the little finger with the thumb against resistance
D. Flexing the distal phalanx of the thumb against resistance while the proximal phalanx
is fixed
E. Extending the index finger at the pip point againt resistance with the mcp joint
hyperextended
and fixed

50. When patients with post traumatic brain injury (TBI) ask about returning to drive, you tell
them :
A. All persons with post TBI can drive
B. Persons with brain injury should not drive
C. Only an on-road evaluation is necessary to determine fitness for driving
D. Persons with TBI must wait at least 1 year post injury before resuming driving
E. Clinical assessment should include sensorimotor impairment, vision, and cognitive
function

51. Shoulder pain are common after stroke episode :


A. Is usually due bursitis calcarea
B. Is usually due to reflex sympathetic dystrophy
C. Is usually due to central pain/thalamic syndrome
D. Is usually due to subluxation the head of humerus
E. Requires injection with steroids for effective management

52. A 50-year old man has had gradually progressive hand weakness. He has atrophy of the
forearm muscles, fasciculations of the muscles of the chest and arms, hyperreflexia of the lower
extremities and positive Babinsky signs. Sensation is not impaired. Which of the following is the
most likely diagnosis?
A. Amyotrophic Lateral sclerosis
B. Dementia Alzheimer type
C. Spinal Muscular Atrophy
D. Guillain Barre Syndrome
E. Multiple sclerosis

53. 45-year old man is evaluated in your spinal cord injury clinic 12 months after a C2 complete
spinal cord injury requiring full-time mechanical ventilation. You recommend
A. Progressive frog breathing exercise
B. Progressive diaphragmatic strengthening exercise
C. Progressive breathing control exercise for his power wheelchair
D. Initiating a weaning protocol by slowly decreasing tidal volume
E. electrodiagnostic study to evaluate for a phrenic nerve pacemaker

276
54. When treating supraspinatus tendonitis with USD, which shoulder position is most beneficial?
A. Arm abducted and internally rotated
B. Arm abducted and externally rotated
C. Arm adducted and externally rotated
D. Arm adducted and internally rotated
E. Position of comfort

55. The following is NOT indication of biofeedback therapeutic :


A. To inhibit spasticity
B. To strengthen muscle
C. Muscle reeducation in upper motor neuron dysfunction
D. Pelvic floor muscle reeducation in women with pelvic pain
E. Pelvic floor muscle reeducation in women with incontinence

56. Which of the following is NOT a contraindication to microwave diathermy?


A. Malignancy
B. Old scar tissue
C. Electrical implant
D. Acute inflammation
E. Sensory impairment

57. A 6-month-old baby was noted to be hypotonic at birth. On examination, head circumference
is normal, there is a pronounced head lag, reflexes are diminished. The baby doesn’t roll, alert,
aware and attempts to uses her arms to reach for toys. The cranial nerves are normal except for the
fasciculations of the tongue. This disease is known as :
A. Cerebral palsy
B. Infantile bolutism
C. Spinal muscular atrophy
D. Duchenne muscular dystrophy
E. Myotonic muscular dystrophy

58. What type of Cerebral Palsy (CP) with unsteady gait, dysmetria, intention tremor in upper
extremities and truncal titubation?
A. Ataxia type
B. .mixed type
C. spastic type
D. athetoid type
E. hypotonic type

59. Management of Pes Planus grade 1 at age less than 1 year old is done by :
A. Observation
B. Thomas heel
C. Arches support
D. Boot type shoe
E. Chukka type shoes

60. Congenital muscular torticollis caused by


A. Hermivertebra

277
B. Ipsilateral plagiocepahly
C. Deformation form intrauterine positioning
D. Fibrotic contracture of the paraspinal muscle
E. Fibrotic contracture of the upper trapezius muscle

61. Sign of improvement from pulmonary rehabilitation program:


a. Decrease of dyspnea and increase in exercise tolerance
b. Increase strength of respiratory muscle
c. Increase pulmonary function test
d. Increase work of breathing
e. Increase chest expansion

62. Strategies for decreasing dyspnea during eating in patient with pulmonary disease that include
the following, EXCEPT:
a. Using O2 during meal
b. Using pursed lip breathing
c. Using liquid supplements between meals
d. Eat high-carbohydrate and gas forming foods
e. Assuming a body position that minimizes perceived work of breathing

63. During vigorous exercise, while participating in a stress test, it is noted that the T wave
increases by 10%. No other changes are noted. How should this be interpreted
a. Definitely abnormal-highly suspicious but not absolutely diagnostic of ischemic heart
disease
b. Mildly abnormal-would recommend that maximum effort stress test be performed
c. Definitely abnormal and diagnostic for ischemic heart disease
d. Definitely abnormal and diagnostic for myocardial infarction
e. Normal

64. How might the patient with obstructive pulmonary disease help overcome air trapping ?
a. Slowly inhale until the lungs are fully expanded
b. Forcefully inhale for as long as possible
c. Exhale with the lips pursed
d. Forcefully exhale
e. Breathing control

65. During exercise, there is an increase in a person’s


a. Stroke volume
b. Residual volume
c. Diastolic pressure
d. Total peripheral resistance
e. Pulmonary arterial resistance

66. These are the effects of treadmill training in Parkinson patients EXCEPT:
a. It reduces tremor
b. It decrease fall risk
c. It decrease endurance
d. It improves quality of life

278
e. It improves gait impairment

67. You are asked to see a 66 year old woman with rheumatoid arthritis 21 years duration for
rehabilitation program following a left total knee arthroplasty. On examination, you note that she
has marked ulnar deviation and subluxation of the metacarpophalangeal joint in both hands. Which
one of the following is the best assistive ambulation device for he initially rehabilitation program?
a. Quad cane
b. Single crutch
c. Rolling walker
d. Platform walker
e. Lofstrand crutches

68. Following statement is the purpose of bracing in CP patient, EXCEPT:


a. To control movements that interfere functions
b. To prevent/ correct contractures
c. To promote better balance
d. To decrease spasticity
e. To promote walking

69. A child with a congenital transverse radial limb deficiency should have in initial prosthesis fit
at what developmental stage?
a. At the time of first sitting independently
b. At the time of starting kinder garden
c. At the time standing independently
d. At the time of initially walking
e. At soon as possible after birth

70. A 3 year-old girl with Juvenile Rheumatoid Arthritis (JRA) has involvement limited to right
knee and left ankle. Her referring physician noted a positive antinuclear antibody. You explain to
her mother that type of JRA is associated with:
a. Diurnal temperature variations
b. A high incidence of iridocyclitis
c. Spondylitis of the thoracic spine
d. Lymphadenopathy and hepatosplenomegaly
e. Severe erosive changes and residual disability

71. A 16 year old basketball player is referred to you with leg pain. On examination the boy has
point tenderness over the proximal tibia. Treatment with ultrasound had exacerbated the pain.
Your clinical impression is:
a. Tendinitis
b. Shin splints
c. Stress fracture
d. Compartments’s syndrome
e. Osteocondritis dissecans

72 The most common method of exercise based on the neurodevelopmental approach for cerebral
palsy patient is the:
a. Rood

279
b. Voyta
c. Kabat
d. Phelps
e. Bobath

73.The following treatment encourage to decrease edema, EXCEPT


a. Kneading
b. Elevation
c. Elastic compression
d. Intermittent pressure device
e. Rhythmic muscle contraction

74. The following is NOT including in rehabilitation program for SLE patient:
a. Orthosis and adaptive equipment
b. Reeducation in selfcare skills
c. High intensity strength exercise
d. Antispasticity agents
e. Endurance exercise

75. The following exercise prescriptions for diabetic is true, EXCEPT:


a. Avoid exercise at peak insulin times
b. Advice that insulin be injected in muscle to be exercised
c. Encourage adequate hydration to avoid effects of fluid shifts on blood sugar levels
d. Caution patient that blood sugar may continue to drop for 24-48 hours after exercise
e. If blood sugar value is <100 mg/dl, delay exercise, provide patient 15 g of
carbohydrate, retest in 15 minutes

76. Exercise training for patient with coronary vascular disease


a. Risk stratify the patient using the risk stratification schema (the AHA and the
AACVPR) is not recommended
b. Symptom-limited exercise testing prior to participation is strongly recommended
c. For resistance exercise the intensity 20-25 repetitions per set to fatigue
d. Exercise training is based on comorbidities only
e. Calisthenic is modalities for aerobic exercise

77. Regarding exercise in heart failure, which of the following is TRUE?


a. Telemetry is never necessary
b. Strengthening should be done isometrically
c. Duration of exercise should be prolonged until fatigue
d. Exercise heart rate should be at 80-90% of estimate maximum heart rate
e. The exercise heart rate be kept at least 10 beats per menit below the arrhythmia/
severe dyspnea level

78. Absolute contraindications to exercise tolerance testing:


a. Unstable angina
b. Controlled DM
c. FEV1 >60% of predicted value
d. Chronic compensated heart failure

280
e. Resting BP 140/90 mmHg

79. A 50 y.o woman with secondary progressive multiple sclerosis has mild cognitive deficits and
severe lower-limb spasticity. What the most effective treatment for this patient?
a. ROM exercise
b. Warm water exercise
c. Intrathecal baclofen
d. Diazepam injection
e. Cold therapy

80. The following statement in true about laser therapy, EXCEPT:


a. It is better to overexposure than underexposure
b. Laser therapy should not be used over cancerous growths
c. HeNe and GaAs lasers are two low power lasers therapeutic application
d. The categories of lasers include solid state, gas, semiconductor, dye and chemical
e. The proposed therapeutic application of lasers include increase of collagen synthesis,
vascularization, and reduction of pain and inflammation

81. The following is NOT contraindication of pelvic traction


a. Disciitis
b. Acute sprain
c. Osteoporosis
d. Joint instability
e. Grade 1 spondylolisthesis

82. A male patient, 30 y.o, no history of regular sport activities, comes with a chief complaint of
pain on the right knee joint around the patellar tendon insertion after play football. This worst
when he runs or climbs the stairs. The most probable diagnosis is:
a. Osgood Schlatter disease
b. Chondromalacia patella
c. Ligamental sprain
d. Meniscar tear
e. ACL tear

83. The most likely combination of musculoskeletal imbalances associated with iliotibial band
syndrome is:
a. Weak quadriceps/tight hamstring
b. Weak lower abdomen/tight psoas
c. Weak gastro-soleus/tight posterior tibialis
d. Weak gluteus medius/tight tensor fascia lata
e. Weak gluteus maximus /tight adductor magnus

84. To prevent sport injury especially for lower extremity, should be strengthen following core
muscle:
a. Abdominal muscle, hip extensor, knee extensor
b. Abdominal muscle, hip internal rotator, hip extensor
c. Abdominal muscle, hip internal rotator, hip extensor, knee extensor
d. Abdominal muscle, hip abductor, back extensor, hip external rotator

281
e. Abdominal muscle, hip external rotator, back extensor, knee extensor

85. Injury to the rotator cuff muscle and tendons of the shoulder is often occurs in?
a. Golf player
b. Tennis player
c. Discus thrower
d. Boxing athletes
e. Rowing athletes

86. What is the most common cause of dementia in geriatric population?


a. Toxic-metabolic dementia
b. Traumatic brain injury
c. Lewy body dementia
d. Vascular dementia
e. Alzheimer disease

87. This type of activity and exercise in NOT recommended in osteoporotic patient to prevent
developing compression fractures of the spine:
a. Prevent falls
b. Back exercise
c. Flexion exercise for the spine
d. Keep heavy objects close to chest when lifting
e. Flex hips and knees to bend down to lift objects

88. Which of the following is most likely to enable you to determine the reason for a fall in an
older patient?
a. Muscle strength examination
b. A completed neurologic examination
c. MRI of the brain
d. Determination of the patients’s functional status
e. A careful history of the circumstances surrounding the fall

89. A 70-year-old man with diffuse degeneration joint disease who report the gradual onset of
anterior-lateral shoulder pain, worse at night, particularly when lying on the affected side. There is
tenderness over the deltoid and pain with resisted abduction. Passive and active shoulder motions
are normal. Neer’s maneuver and the “empty can test” negative. Your initial recommendation is:
a. Steroid injection into subdeltoid bursae
b. Isotonic deltoid strengthening exercise
c. Stretching of the shoulder joint
d. Superficial heating
e. TENS application

90. A discus thrower complains of pain in the upper part of shoulder. One of the finding on
examination is cross adduction chest positive sign, pain arch at 110. What structure most likely to
be involved?
a. Scapulocostal
b. Suprahumeral joint
c. Glenohumeral joint

282
d. Sternoclavicular joint
e. Acromioclavicular joint

91. A martial arts player complaint of back knee pain after kicks his opponent. Examination of
knee demonstrates pain during varus stress test. What is the mostly likely diagnosis?
a. Medial collateral ligament injury
b. Lateral collateral ligament injury
c. posterior cruciate ligament injury
d. Anterior cruciate ligament injury
e. Strain popliteus muscle

92. a 42-years old tennis player complain of severe right elbow pain for 6 months. He has tried
heat, ice and compression wrap without relief. He denies numbness, but does report weakness in
his grip, especially with his backhand. On examination, the patient has severe pain with palpation
just inferior to the lateral epicondyle. Which finding will most likely be present on further
examination
a. resisted wrist extension with a straightened elbow will reproduce the patient’s pain
b. pain will be increased with ulnar deviation of the wrist with resisted flexion
c. an audble click will be heard with active supination of the forearm
d. the tricep tendon reflex will be significantly diminished
e. the brachoradialis reflex will be significantly diminished

93. which statement below is TRUE according rupture of the Achilles tendon in jumpers
a. thompson’s test gives a negative result
b. impaired ability to bend the foot plantar flexion
c. the injured athlete can walk normally on the foot or on tiptoe
d. after surgery it is usually necessary to allow 2 months break competition
e. the tendons begin to show degenerative changes at the early age of 25 to 30 years

94. the most function of gluteus maximus during gait when


a. loading response
b. terminal stance
c.double support
d. mid stance
e. pres wing

95. which flexor muscle below located profoundly?


a. pronator teres
b. palmaris longus
c. pronator quadratus
d. flexor carpi ulnaris
e.flexor carpi radialis

96. what is the characteristics of swan neck deformity?


a. flexion of MCP and DIP, hyperextension of PIP
b. flexion of MCP, hyperextension of PIP and FIP
c. extension of MCP, flexion of DIP and PIP
d. extension of MCP and DIP, flexion of PIP

283
e. extension of MCP, PIP, and DIP

97. this serum enzyme from myocardial damage or necrosis appear at 4 – 6 hours ; peaks at 12 –
24 hours; returns to normal within 72 hours
a. SGOT
b. troponin I
c. C reactive protein (CRP)
d. CK Myocardial Band (CKMB)
e. creatinine phospo kinase (CPK or CK)

98. Estimated energy requirements of swimming slowly activity is


a. 3.0 METs
b. 3.5 METs
c. 4.0 METs
d. 4.5 METs
e. 5.0 METs

99. regarding for high risk stratification of cardiac patients, which of the following is true?
a. ejection fraction 50%
b. exercise induced 1 mm ST segmen depression
c. reversible ischemic defect (echocardiography)
d. functional capacity METs onn gradual exercise test
e. failure to rise systolic blood pressure with increasing exercise workload

100. the following lists some of the rationale for outcome assessment in pulmonary rehabilitation
except:
a. pulmonary function test
b. steady state endurance test
c. the 6-minute walking test
d. st. George’s respiraory questionnaire
e. the medical research council dyspneas scale

101. regarding the clinical signs of Rheumatoid arthritis, which of the following is/are true: A
1. evidence of inflamatory by history, examination and laboratory test
2. symmetric, polyarticular inflammatory arthritis
3. a characteristic of joint involvement
4. history suggest mechanical pain

102. a 57 yo woman with steroid-dependent severe chronic obstructive pulmonary disease,


diabetes mellitus, and gastroesophageal reflux disease wishes to be on prophylactic therapy should
include: D
1. glucosamin
2. alendronate
3. sodium fluoride
4.Estrogen/progerstin

103. functionally, a transmetatarsal amputation is superior to a lisfranc’s (tarsometatarsal)


amputation because it provides : E

284
1. an intact tibialis anterior insertion to oppose the pull of the tricep surae
2. a stump that is easier to fit
3. a broader base of support
4. a longer lever arm

104. what is a primary role of methotrexate in the treatment of Rheumatoid Arthritis? D


1. to cure the disease
2. to reserve already present joint deformity
3. to provide analgesia and anti inflammation
4. to slow or prevent joint destruction an Loss of function

105. Contraindications for therapeutic muscle stretching include the following : E


1. inflamation
2. excessive pain
3. lack of stability
4. severe osteoporosis

106. the change(s) that result from stress deprivation from disuse or immobilization of the joint
is/are: E
1. atrophy of cartilage
2. adhesion between synovial fold
3. declines collagen mass by approximately 10%
4. excss deposition of restricting connective tissue in muscle

107. the following(s) below is/are the pharmacokinetic interactions of NSAIDs with other
medications : E
1. interactions with diuretics will reduce diuretic effect
2. interaction with antidiabetics will intensified antidiabetic effect
3. interaction with anticoagulants will intensified effect of anticoagulant
4. interaction with antihypertensive will inhibition of antihypertensive action

108. for patient with osteoporosis, the exercise prescription may be modified to avoid : D
1. exercise frequency 3 – 5 days/week
2. weight bearing aerobic activities
3. exercise duration 30-60 minutes
4. high intensity resistance exercise

109. assessment for geriatric patient with falls include the folllowing: E
1. visual system
2. polypharmacy
3. foot problem
4. enviroment factor

110. what is/are the effect(s) of aging on muscle composition, for strength and endurance E
1. increase in fat and connective tissue within older muscle
2. reduction in the number of functioning motor units
3. reduced of type II fast twitch muscle fiber
4. reduced protein synthesis

285
111. the American Geriatrics Society’s goal of diabetic control in the elderly includes: D
1. tight glicemic control
2. hemoglobin A1c below 7%
3. fasting plasma glucose below 120 mg/dL
4. reduction of cardiovascular complication
112. which of the following classes of patient(s) would be appropriate for osteoporosis
investigation? E
1. women over age 65
2. men with hypogonadism
3. person with hyperparatiroidism
4. women with menopause before age 40

113. the following conditions could lead to Thoracic Outlet Syndrome : A


1. cervical rib
2. fracture of the clavicule
3. hypertrophy of the m. scalenus anterior
4. hypertrophy of the m. pectoralis minor

114. Systemic Lupus Erythematosus may affect: E


1. renal
2. musculoskeletal
3. mucocutaneus
4. central nervous system

115. the following is/are the musculoskeletal system sign(s) in SLE: E


1. myalgia
2. painful joint
3. muscle weakness
4. jaccoud-like arthropathy

116. A 50 years old woman who reportedly had a right anterior cerebral artery stroke. In your
evaluation you expect to find: C
1. distal greater than proximal upper-extremity weakness
2. lower limb greater than upper limb weakness
3. upper and lower facial weakness
4. language is typically not disturbed

117. the goal of medical treatment of parkinson’s disease is/are: C


1. decreased dopamin action
2. increased dopamin action
3. increased cholinergic effect
4. decreased cholinergic effect

118. the most common behavioral and personality changes seen following traumatic brain injury
is/are: E
1. attention deficit rigidity
2. visual perceptual deficit

286
3. spatial dysfunction
4. memory deficit

119. a patient appears to have good strength in the hand and can use it for gross activities
automatically but cannot perform skilled movements. This form disability is called: D
1. kinetic apraxia (motor apraxia)
2. constructional apraxia
3. ideational apraxia
4. ideamotor apraxia

120. the indication(s) for using of an ankle-foot orthosis (AFO) to improve gait is/are: E
1. weak push-off at late stance
2. mediolateral instability at the ankle
3. passive plantarflexion in swing phase
4. foot drop at heel strike due to weak ankle dorsifleksor

121. The manifestation of circumduction gait of amputee who is wearing a lower limb prosthesis
usually caused by: A
1. Socket too small, the residual limb cannot enter fully
2. Inadequate suspension, socket slips downs during swing
3. Amputee is reluctant to flex the knee during swing because of poor balance
4. Amputee does not bother to flex the knee because the prosthesis is to short

122. Which of the following(s) is/are TRUE regarding skeletal design of a lower extremity
prosthesis? B
1. Endoskeletal tends to weight less
2. Exoskeletal is less hardy
3. Endoskeletal is demand more maintenance
4. Exoskeletal is easily adjusted after fabrication

123. What is/are the treatment for progress proprioception, coordination and agility during the
chronic phase of soft tissue healing in sport injury rehabilitation are: B
1. Balance activities
2. Isotonic strengthening exercise
3. Surface modification
4. Isokinetics strengthening exercise

124. What is/are special test(s) / sign(s) will positive for ACL rupture and meniscal tore? C
1. Sagging knee
2. Mc murray test
3. Homan Test
4. Anterior Drawer test

125. Treatment for patient with runner’s knee or the ilio-tibial band syndrome include of: A
1. shoe insertions in cases of increased pronation
2. Anti-inflammatory medications
3. Apply ice in the acute phase
4. Avoid running uphill

287
126. Symptom(s) of tear the anterior talofibular ligaments is/are: A
1. Pain when ankle joint is loaded or moved
2. Positive anterior drawer test in total tear of ligament
3. Swelling and tenderness in front of the lateral malleolus
4. Increased supination compared with the undamaged ankle joint

127. A stroke involving the vertebra-basilar system may give the following manifestation(s): B
1. Ataxia
2. Cortical blindness
3. Hemiplegia alternans facialis
4. Unhibited neurogenic bladder

128. In the left hemiplegic patient who his or her left hand is edematous and his or her left
shoulder is painful requires: E
1. A short course of 30mg prednisone daily
2. to elevate his/her left upper extremity
3. A low dose of amitriptyline
4. TENS

129. Some physical findings below must be assessed in patients with neurogenic bladder E
1. Sacral sensation
2. cognitive impairments are an important test
3. Hand function for SCI patients who are the perform self catheterization
4. Bulbocavernosus reflex to test in the interity of the pundendal nerve and S2-S4 segments

130. Where is the ground reaction force vector located at mid stance? B
1. Anterior to ankle
2. Posterior to ankle
3. Anterior to knee
4. Posterior to knee

131. Age-related changes in the spine include: E


1. degeneration of disk
2. bulging of the annulus fibrous
3. tension on the posterior longitudinal ligament
4. shifting of load bearing form anterior to posterior structures

132. The ankle mortis A


1. The talus function as a hinge joint
2. consist of tibia, fibula and talus
3. Its motion is 20º dorsiflexion
4. its motion is 90º plantarflexion

133. The following(s) is/are the clinical characteristic of Down Syndrome : E


1. The head usually larger than normal because of over development of facial bone and
muscle

288
2. The toes are usually short and in the majority; there is wide space between the first and
second toes
3. Ligamentum laxity causing deficiency in grip strength and ankle strength that develop on
school age
4. most significant musculoskeletal impairment are owing to hypotonia and ligamentum
laxity caused by deformity of the joint

134. In Klumpke’s type of brachial plexus injury the following muscle is/are spared D
1. Flexor pollicis longus
2. Flexor digitorum sublimis
3. Flexor digitorum profundus
4. Extensor carpi radialis longus

135. The following is/are the early treatment(s) of CTEV D


1. Dennis-Brown splint
2. outflare orthopedic shoes
3. Surgical soft tissue release
4. Stretching and serial plaster correction

136. Man, 55-yo, weight 85kg, height 165cm. He got myocard infarct 1 month ago. The fasting
glucose level 250, total cholesterol 400, no hypertension. Factor that should be considered to
prescribe cardiac rehabilitation program: C
1. Avoid all of exercise
2. Assessed the stress test
3. Limitation of daily activity
4. Reduce the risk factor for 2nd cardiac disease attack

137. Improve aerobic capacity and increased oxygen transport is the effect of exercise in cardiac
patient. This will result(s): C
1. Decreased cardiac output
2. Decreased resting heart rate
3. Decreased ejection fraction
4. Decreased systolic blood pressure

138. Exercise for patient with cardiac disease should be discontinued in the following condition, if:
A
1. Systolic BP decrease 20 mmHg
2. Chest pain
3. Dyspneu
4. Joint stiffness

139. A 53-yo woman with chronic venous disease has lower extremity edema, skin
hyperpigmentation, dermatitis, and venous varicosities. She does not have any ulceration. The
mainstay of treatment for this patient is: D
1. intermittent external pneumatic compression
2. hydrotherapy with warm water (100 oF)
3. Topical steroid cream
4. gradient elastic stocking

289
140. Reflection of ultrasound E
1. Occurs mainly at interfaces of tissues with different acoustic impedances
2. is great at the interface of soft tissue and metallic implants
3. is minimal at the interfaces of layers of soft tissue
4. is great at the interface of soft tissue and bone

141. A 15-yo boy has Juvenile rheumatoid arthritis, manifested hip and knee contracture. The
following treatment is/are appropriate: A
1. prone position in bed
2. stretching exercise twice daily
3. low-impact and low intensity endurance exercise
4. ultrasound diathermy treatment of both hips and knees

142. The risk of foot ulceration in patients with diabetes is/are increased by all of the following: E
1. The loss of vibration
2. The presence of plantar foot callus
3. A history of previous foot ulceration
4. Metatarsal subluxation anf pad fat pad migration

143. With the EMG machine we can get following result: A


1. The severity of pathology
2. The level/location of motor unit pathology
3. The motor and sensory conduction velocities
4. The etiology of motor unit abnormalities

144. Reduced nerve conduction velocity indicates: C


1. Axonal degeneration
2. Demyelinization
3. Myopathy
4. Neuropathy

145. The following is/are the common cause for “floppiness” at birth A
1. Congenital myotonic dystrophy
2. Spinal muscular atrophy type I
3. Cerebral dysfunction
4. Spina bifida

146. The following statement(s) is/are the special precautions that should be applied when
designing exercise program for children: E
1. Children with certain illness may require adjustments in their exercise prescription
2. Children may experience a higher incidence of damage the ephiphyseal growth plate
3. In a hot environtment, children show less heat dissipitation than adult
4. In a cold environtment, increasing their risk of hypothermia

147. Some of the benefit(s) obtainable from regular exercise in children is/are: E
1. skill development
2. Social interaction

290
3. Enhanced bone formation
4. Improve self-esteem and self-efficacy

148. The aim(s) of chest physical therapy in asthmatic patient is/are: E


1. To relieve bronchospasm
2. to aids removal of secrets
3. to coordinate respiratory movements
4. To assist relaxation and gain control of breathing

149. what exercises are recommended for persons with nonprogressive mild to moderate
restrictive lung disease (VC of 60% predicted)? D
1. pursed lip breathing
2. Rapid shallow breathing
3. Glossopharyngeal breathing
4. inspiratory resistive exercise

150. In a child with muscular dystrophy with resultant respiratory dysfunction, a regular program
of exercise to improve respiratory function will D
1. require more time then is practical
2. improve maximal inspiratory pressure
3. have clinical effects measurable by spirometry
4. improve endurance of respiratory muscles

291
jawaban soal UTUL juni 2015
1-10 BCEDDCDEED
11-20 BDEBADDEED
21-30 BBEBDAEBAD
31-40 EBBCCBEEBB
41-50 AADEEACCEE
51-60 DAEABBCAAC
61-70 ADECAADDAB
71-80 CEACBBEACA
81-90 EADDBECEAE
91-100 BAEACADDEA
101-110 ADEDEEEDEE
111-120 DEAEECCEDE
121-130 ABBCAABEEB
131-140 EAEDDCCADE
141-150 AEACAEEEDD

NATIONAL BOARD EXAMINATION


292
11 DECEMBER 2015

1. Yergason test is a test to detect inflammation of the following muscle: D


a. Medial head of the triceps
b. Long head of the triceps
c. Short head of the triceps
d. Long head of the biceps
e. Short head of the biceps

2. Treatment for chronic rotator cuff tendonitis include all of the following, EXCEPT: E
a. Normalization of dynamic muscle control
b. Control of pain and inflammation
c. Restoration of range of motion
d. Normalization of strength
e. Immobilization

3. Early symptoms of Ankylosing Spondylitis in a young adult is: B


a. Cervical pain
b. Low back pain
c. Upper thoracal pain
d. Lower thoracal pain
e. Lower extremity pain

4. Early rehabilitation program for Adhesive Capsulities includes: C


a. Aggresive ROM exercise
b. Intrarticular injection
c. Passive joint glides
d. Infra red radiation
e. Hydrocollator pack

5. The following is true about plantar fascitis: E


a. Is common in patients whose occupation requires long period of sitting
b. There is deep tenderness of the posterolateral aspect of the calcaneus
c. The condition is more prevalent in people with supinated foot
d. Use of a accomodation shoes to improve the pain
e. The treatment is stretching of the heel cord

6. The following general guidelines and principles of strength exercise for children : C
a. Balistic movement
b. Practice power lifting
c. Should be supervised
d. Breath-holding techniques
e. Perform single joint exercise

7. Crouched gait in cerebral palsy patients is cause by : cucur 789 E


a. Abductor weakness
b. Increased adductor tone
c. Increased iliopsoas tone
d. Spasticity of the gastrocnemius muscles
e. Hamstring, psoas, rectus femoris and gastrocnemius contracture

293
8. The following drug can cause sedation, EXCEPT : E
a. Baclofen
b. Diazepam
c. Clonidine
d. Tizanidine
e. Dantrolene sodium

9. Based on acquisition of speech and language skills, in how old child have ability to first
words emerge and responds to simple requests? D
a. 3 months
b. 6 months
c. 9 months
d. 12 months
e. 15 months

10. What exercises are recommended for persons with nonprogressive mild to moderate
restrictive lung disease (VC of 60% predicted)? A
a. Inspiratory resistive exercise à obstructive
b. Glossopharyngeal breathing à severe restrictive/ventilator
c. Rapid shallow breathing
d. Diaphragmatic breathing
e. Pursed lip breathing & obstuctive

11. Which of the following statements regarding the rehabilitation programs of


hospitalization for cardiovascular illnesses or operation is true? E
a. Begin an isometric and isotonic exercises after the patient stabilized
b. Limited range of motion exercise are usually save at this point, even in very unstable
patients
c. Walking, at first with assistance should be encouraged even the patient is moderately
symptomatic
d. The period of hospitalization is the optimal time to improve the patient’s muscle strength
e. After the patient stabilized, having the patient sit in an armchair, to minimize the
loss of postural reflexes resulting from bed rest

12. The major goal of phase II cardiac rehabilitation is: B


a. Assessment of cardiovascular function
b. To achieve cardiovascular conditioning
c. Alleviation of anxiety and depression
d. ADL independency
e. Early ambulation

13. Exercise and activity modification prescription in type 2 Diabetes Mellitus with
retinopathy complication is: D
a. High-intensity aerobic exercises
b. Resistance physical activity
c. Head-down activities
d. Low-intensity exercise
e. Jumping activities

14. Posterior interosseous nerve syndrome or resistant tennis-elbow C


a. Pain reproduction with resisted wrist extension while elbow extended
294
b. Pain with resisted long finger extension is a specific finding
c. Cause by nerve compression at the arcade of Frohse
d. Weakness of digital flexor can occur
e. X-rays are usually helpful

A young basketball athlete, 8-year-old, injured his right ankle while playing basketball in a
competition 1 week ago. His x-ray films showed no fracture. His physician diagnosed the
injury as lateral ankle sprain. Visual Analog Scale has decrease in intensity from 6/10 to
4/10, but the pain increases with weight bearing and certain demonstrated movements.
Physical examination reveals no warmth of the skin of anterolateral aspect of the right ankle,
slight ecchymosis and swelling are noted. His anterior drawer test and talar test is negative
(number 15 and 16).

15. What is the most likely diagnosis? B


a. Grade I ankle sprain, a minor tear with instability
b. Grade I ankle sprain, a minor tear with no instability
c. Grade II ankle sprain, a partial tear with instability
d. Grade II ankle sprain, a partial tear with no instability
e. Grade III ankle sprain, a complete tear with gross instability

16. In this case, what structure has been injured? A


a. Anterior talofibular ligament
b. Posterior talofibular ligament
c. Calcaneofibular ligament
d. Syndesmotic ligament
e. Tibiotalar ligament

17. Gluteus maximus muscles are innervated by B


a. Gluteus superior nerve
b. Gluteus inferior nerve
c. Pudendus nerve
d. Obturator nerve
e. Sciatic nerve

18. The real or true leg length is the distance between E


a. Umbilicus to medial malleolus
b. Umbilicus to lateral malleolus
c. Femoral neck to lateral malleolus
d. Greater trochanter to lateral malleolus
e. Anterior superior iliac spine to medial malleolus

19. What is the characteristics of swan neck deformity? B


a. Ulnar drift at the MCP joint
b. Hyperextention of PIP and flexion of DIP
c. Hyperextension at the interphalangeal joint
d. Extension of MCP, flexion of DIP and PIP
e. Fixed flexion at the PIP joint and hyperextension at the DIP joint

20. The following are the indication for use of an AFO to improve a patient’s gait,
EXCEPT E
a. Foot drop at heal strike due to weak ankle dorsiflexors
295
b. Passive plantar flexion in swing phase
c. Mediolateral instability at the ankle
d. Weak push-off at late stance phase
e. Pes planovalgus

21. A logical approach to the initial management of phantom pain 1 week postoperatively
is A
a. TENS
b. Epidural steroid injection
c. Local ice pack treatment
d. Lumbar sympathetic block
e. Surgical neuroma excision

22. The most likely cause of verrucous hyperplasia in a patient with below-knee
amputation is C
a. The residual limb setting to deeply into the prosthetic socket
b. An insufficient number of prosthetic socks
c. Lack of total contact in the distal socket
d. An alergic reaction to wool socks
e. A distal end pad that is too firm

23. Word finding difficulty without other serious linguistic deficits, it is mean: A
a. Anomia
b. Global aphasia
c. Broca’s aphasia
d. Wernicke’s aphasia
e. Transcortical aphasia

24. You are seeing a 56 years old male in consultation 1 day after severe stroke. He is
medically stable and has flaccid hemiplegia with poor sitting balance. He is sitting up in a
chair for 3 hours twice daily and just started bedside physical therapy and occupational
therapy. You would recommend: D
a. Strength the flaccid arm
b. Improve with less intensive physical therapy
c. Focusing on sitting balance, followed by transfer
d. Start aggressive physical and occupational therapy
e. Start electrical stimulation to the flaccid arm to enhance neurologic recovery

25. Basic principles of weight resistance training should be followed to derived the
maximum strength including the following, EXCEPT: E
a. Forced to contract at maximum (overload)
b. Forced to contract near the maximum tension (overload)
c. The type of exercise produce a variety of specific muscle adaptation (specificity)
d. The intensity of the load must be progressively increase (progressive resistance)
e. Allow 12 hours between strength exercise for complete physiology recovery
(allowing adequate recovery)

26. A 60-year old man who had a left cortical stroke 6 months ago wishes to improve his
hand function. He has good cognition. Sensation was only mildly decreased to light touch.
Muscle strength of shoulder flexor 4/5, elbow flexor 3/5, elbow extensor 3/5, wrist extensor
3/5, finger flexor 2/5, finger extensor 2/5. Which technique is most likely to result in
296
functional improvement in this patient? B
a. ROM exercises
b. Constraint induced movement
c. Electrical stimulation to finger flexors
d. Proprioceptive neuromuscular fascilitation
e. Electromyographic biofeedback to wrist extensor muscle

27. A woman riding a mountain bike on a rough trail hits a rut and is thrown from the bike.
Her upper arm hits a tree, fracturing the humerus just above the insertion of the teres mayor
muscle. Later, during examination, it is noted that she cannot extend her forearm at the
elbow or hand at the wrist. Which nerve must have been injured? B
a. Ulnar
b. Radial
c. Axillary
d. Median
e. Musculocutaneous

28. The correct statement for stage 4 of brunnstrom recovery in stroke patients is: D
a. More complex movement combinations are learned as the basic synergies lose their
dominance over motor acts
b. Basic synergy patterns appear, minimal voluntary movements may be present
c. Patient gains voluntary control over synergies, increase in spasticity
d. Synergy patterns still predominate, decrese in spasticity
e. Increase in spasticity and synergy patterns

29. Ms. S, 21 year-old referred to the rehabilitation ward with paraplegia due to T5 burst
fraktur. The spine stabilization was done 2 weeks ago and now was she medically stable. On
physical examination you found that the vital signs and general health status were within
normal limit. What should you examine when you have to determine whether the patient has
a complete or incomplete lesion? E
a. Myotome
b. Dermatome
c. Pathological reflexes
d. Anocutaneal and bulbocavernous reflexes
e. Anal sensation and voluntary anal contraction

30. A 45-year-old woman had a modified radical mastectomy with axillary dissection for
breast cancer and now has severe lymphedema. Evaluation reveals no evidence of recurrent
tumor, infection, or venous thrombosis. To maintain edema control, you recommend : E
a. Diuretic medication
b. Codman’s exercises
c. Anti-inflammatory medication
d. Low-dose antibiotic prophylaxis
e. Use of a graded compression garment

31. The mainstay of treatment of moderately severe visceral pain due to cancer is B
a. TENS
b. Opiates
c. Exercise
d. Nerve blocks
e. Anti-inflammatory agents
297
32. Which of the following would most optimally be included in an initial exercise program
for healthy elderly person with moderate degenerative osteoarthritis? E
a. Tennis
b. Treadmill
c. Volleyball
d. Step aerobic
e. Stationary cycling

33. In a resting, healthy man, the ejection fraction is approximately? E


a. 0.1
b. 0.2
c. 0.3
d. 0.4
e. 0.6

34. Primary muscle of inspiration D


a. Sternocleidomastoid
b. Rectus abdominalis
c. Pectoralis major
d. Diaphragm
e. Trapezius

35. The following physiology of structural protein in the muscle fiber: D


a. Actin is the thick filaments
b. Myosin is the thin filaments
c. Tropomyosin and troponin are located in the thick filaments
d. Myosin, actin, tropomyosin and troponin are the major structural proteins
e. Action potentials in the muscles cause release of calcium from the thick filaments

36. The contraindication of TENS is: C


a. Ischemic pain
b. Neurogenic pain
c. Over carotid sinus
d. Rheumatoid arthritis
e. Myofascial trigger point syndrome

37. The good conductor for electric current is: C


a. Ligament
b. Tendon
c. Muscle
d. Nerve
e. Bone

38. The following is the contraindication for massage, EXCEPT E


a. Recent trauma
b. Skin disease
c. Haemorrhage
d. Inflammation
e. Fibromyalgia

298
39. The language characteristic in Boston Classification of Aphasia, which fluent
neologistic language, poor comprehension and intact repetition is A
a. Transcortical sensory type
b. Transcortical motor type
c. Conduction type
d. Broca’s type
e. Global type

40. Key elements of the physical examination for communication, cognitive and
swallowing including the following, EXCEPT: C
a. Mini mental status examination
b. Hearing and vision testing
c. Lower airway assessment
d. Cognitive level
e. Swallow screen

41. A 62-year-old woman complains of right knee pain and stiffness. On physical
examination she has a genu varum deformity with unstable joint. A physical therapy
prescription should include: E
a. Isokinetic hamstring strengthening
b. Isokinetic quadriceps strengthening
c. Isometric gastrocsoleus strengthening
d. Open kinetic chain quadriceps strengthening
e. Closed kinetic chain quadriceps strengthening

42. A person presents with complaints of right wrist and hand pain. She attributes the
problem to prolonged use of the computer mouse. Which intervention should be included in
the initial treatment plan? D
a. Biweekly steroid injections to minimize symptoms
b. Eccentric strengthening of the wrist extensors
c. Moving the mouse away from the keyboard
d. Wrist splint to minimize symptoms
e. Stretching of the wrist joint
43. A 45-year-old has been hospitalized for an acute exacerbation of rheumatoid arthritis. In
order to prevent additional loss of strength you would prescribe: B
a. Isotonic exercises
b. Isometric exercises
c. Isokinetic exercises
d. Stretching exercises
e. Passive ROM exercises

44. A 35-year-old male presents with a 3-week history of right-sided interscapular pain,
which began approximately 48 hours after she painted a ceiling. The pain occasionally
radiates into the right upper extremity, and is associated with paresthesia in the index and
middle fingers of the right hand. Based on this history alone, the most likely diagnosis is: C
a. Fibromyalgia
b. Brachial plexitis
c. Cervical radiculopathy
d. Carpal tunnel syndrome
e. Thoracic outlet syndrome
299
45. Ulnar deviation of the fingers in rheumatoid arthritis has many causes of the following
factors, EXCEPT : B
a. Synovitis and capsulitis in the MCP joint
b. Extensive tenosynovitis in the flexor tendons
c. The contraction of the long flexor tendons of the fourth and fifth fingers
d. Any external stress by movements performed in the course of one’s daily life
e. Sliding down from the back of the metacarpal capitulum in the direction of the ulna

46. Rehabilitation guidelines following total knee arthroplasty include B


a. Non-weight bearing status for 1 week following surgery
b. Patients are usually allowed to resume driving 6 weeks after surgery
c. Isotonic strength exercises of the lower extremity 3 days following surgery
d. Use of a continuous passive motion device to improve passive knee extension
e. Joint immobilization until sutures are removed and anterior incision is healed

47. Child with a congenital transverse radial limb deficiency should have in initial
prosthetic fit at what developmental stage? A
a. At the time of first sitting independently
b. At the time of starting kinder garden
c. At the time of iniatially walking
d. At soon as possible after birth
e. At 3-year-old

48. A child with C5 ASIA A spinal cord injury should eventually become independent in
which activity? D
a. Ambulation with lower leg brace
b. Intermittent catheterization
c. Transfer to level surface
d. Feeding
e. Bathing

49. To maximize pediatric patient safety, we may need consider when writing a prescription
for rehabilitation, including the following, EXCEPT: B
a. Impaired communication and cognition can pose a risk if modalities are being considered
b. Risk atlantoaxial instability for patient with congenital lower limb amputee
c. Weight bearing and ROM restriction after surgery
d. Fatigue precaution in neuromuscular disease
e. Seizure precaution

50. The exercise regimen for children with myopathy include C


a. Anerobic exercise
b. Eccentric exercises
c. Concentric exercises
d. Maximal aerobic activity
e. High resistance exercises

51. For the post-stroke patients with history of coronary artery disease and fibrillation, an
aerobic exercise session should be terminated if: D
a. Heart rate does not increases
b. Heart rate increases to 20% over baseline
300
c. Respiratory rate increases 10% over baseline
d. Diastolic blood pressure increases to 120 mmHg
e. Systolic blood pressure increases to 160 mmHg

52. The following are the benefit effect of pursed lip breathing, EXCEPT: D
a. Improving oxygen stauration
b. Prevent airways collapse
c. Reduce respiratory rate
d. Reduce tidal volume
e. Reduce dyspnea

53. The 6MWT is one of the exercise clinical testing, has been used for the following
EXCEPT:A
a. Provide the mechanism of exercise limitation
b. Monitor response to therapeutic intervention
c. Preoperative and postoperative evaluation
d. Assessment of functional capacity
e. Predict morbidity and mortality ATS guidelines for the 6MWT

54. Among cardiovascular disease patients with congestive heart failure who participate in a
cardiac rehabilitation program, the primary means by which improved fitness is attained : E
a. Improve diastolis function
b. Improved ejection fraction
c. Improved blood pressure
d. Decreased arrhythmias
e. Peripheral adaptations

55. A national football player twists his knee when he attempts to tackle an oncoming
players. Examination reveals no medial lateral laxity or joint line tenderness. The posterior
drawer test and pivot shift results are negative, but the Lachman test result is positive. What
is the most likely diagnosis? C
a. MCL injury
b. LCL injury
c. ACL injury
d. PCL injury
e. ACL-MCL injury

56. How does the anterior cruciate ligament (ACL) injury become torn? A
a. Deceleration of the leg via quadriceps contraction combined with valgus and
external rotation forces upon a slightly flexed knee
b. Sudden external rotation of hyperflexed knee
c. Sudden internal rotation of hyperextend knee
d. Sudden hyperflexion of the knee
e. Direct blows to the ankle

57. The following statement is TRUE about tennis elbow, the ... of lateral elbow pain in the
athlete: D
a. Return to sport after the patient is pain free and the ... 50% of ... on the contralateral side
b. Tenderness is localized in ... to ... of proximal and ..
c. Symptoms are reproduced with ...
d. Acute on chronic pain is suggestive of a frank rupture of the extensor origin
301
e. The extensor carpi ulnaris is ...

58. The subtalar joint of the foot is an articulation between : C


a. Tibia and talus bone
b. Fibula and talus bone
c. Calcaneus and talus bone
d. Talus and navicular bone
e. Calcaneus and cuboid bone

59. Winging of the scapula during arm trunk elevation is a ... sign of weakness of the : E
a. Trapezius muscle
b. Rhomboid muscle
c. Supraspinatus muscle
d. Levator scapula muscle
e. Serratus anterior muscle

60. Atlanto-odontoid is classified as: B


a. Gynglimus joint
b. Trochoid joint
c. Ellipsoidal joint
d. Condyloid joint
e. Ball and socket joint

61. Quadrilateral transfemoral socket: C


a. The ischial tuberosity is contained inside the socket providing a ... ischium and greater
trochanter
b. Weight bearing is consentrated in the medial aspect of the ischium and the ... ramus
c. There are reliefs for rectus femoris, adductor longus, hamstring and gluteus
maximus
d. Narrow mediolateral design was developed to provide a ...
e. Narrow mediolateral socket can decrease neuromuscular bundle compression

62. Which of the following knee type (qAK prostheses) provides good stability in early
stance phase and facilitate flexion while weight bearing during the pre-swing phase
(maximal stance phase) of the gait cycle: B
a. Single axis
b. Polycentric
c. Manual locking
d. Stance phase control
e. None the above

63. In a hemiplegic, the function of a posterior stop in a double upright AFO with dual
action adjustable ankle joint and extended steel shank is to: C
a. Improve knee stability at heel strike when the posterior stop is set in 5 degrees of
dorsiflexor rather than plantar flexion
b. Cause the ground reaction force line to fall posterior to the knee joint in mid stance
c. Assist in control of moderate to severe plantar flexor spasticity
d. Assist weak plantar flexor
e. Assist weak hamstring

64. A 72 years-old man presented with bilateral asymmetrical weakness of hand grip. On
302
examination there was loss of muscle bulk on the volar aspect of the forearm and impaired
flexion of the distal interphalangeal joints of the fingers. Impaired function of which muscle
is chiefly contributing to the weakness seen? D
a. Flexor carpi ulnaris
b. Flexor carpi radialis
c. Flexor pollicis longus
d. Flexor digitorum profundus
e. Flexor digitorum superficialis à proximal interphalangeal joint

65. If the patients can perform a particular movement automatically or spontaneously, but
cannot repeat the movement when asked, he/she has impairment of: A
a. Motor apraxia
b. Apraxia of gait
c. Dressing apraxia
d. Ideational apraxia
e. Apraxia of speech

66. A 4 years-old boy was brought into your office because his mother has noticed that he
has difficulty getting up from a seated position on the floor while playing his toys . On
physical exam, there was increased gastrocnemius calf circumference bilaterally. You think
the child may have dystrophic myopathy. The maneuver the child performs to assist him in
standing was caused by :C
a. Distal leg weakness
b. Distal arm weakness
c. Proximal leg weakness
d. Proximal arm weakness
e. Spinal Muscle weakness

67. Which type of aphasia would you expect to see in a patient who is noted to have fluent
speech, with sign of comprehension, but unable to repeat : C
a. Broca aphasia
b. Anomis aphasia
c. Conduction aphasia
d. Transcortical motor aphasia
e. Transcortical sensory aphasia

68. Locked-in syndrome (tetraparesis in a completely conscious patient with only the ability
to move eyes vertically and blink) is suggestive of an occlusion in which artery(ies)? C
a. Vertebral artery
b. Posterior cerebral artery
c. Bilateral basilar arteries
d. Vertebral and basilar arteries
e. Posterior inferior cerebellar artery

69. Gait examination results below is common findings in gait analysis in the elderly,
EXCEPT :C
a. Slower velocity
b. Wider base of support
c. Increased stride length
d. Longer double support time
e. Decreased initial swing height
303
70. Functional assessment of instability in the older faller is : D
a. Vision
b. Vestibular
c. Proprioception
d. Standing reach test
e. Activities at time of fall

71. The following structures may changes with age, EXCEPT : E


a. Atrophy
b. Elasticity
c. Sarcopenia
d. Demyelinization
e. Increased fast twitch fiber size

72. Surface electrodes for recording antidromic sural nerve conduction studies are best
placed at :B
a. Posterior to the medial malleolus
b. Posterior to lateral malleolus
c. Anterior to the medial malleolus
d. Anterior to the lateral malleolus
e. Posterior gastrocnemius

73. The most common clinical application of the H-reflex is to test : C


a. Guillain Barre Syndrome
b. Cervical radiculopathy
c. S1 radiculopathy
d. Polyneuropathy
e. Neuritis

74. The following statement about type I muscles fiber is NOT correct : E
a. Contain a large number of mitochondria and myoglobin
b. Dominant in much of marathon runner
c. Also called as slow twitch fibers
d. Predominantly aerobic
e. Generate large force

75. Produces deep heating through the conversion of multiwave electromagnetic energy to
thermal energy is: C
a. Radial shockwave therapy
b. Ultrasound diathermy
c. Shortwave diathermy
d. Microwave diathermy
e. Ultraviolet radiation

76. A transdermal delivery system to which a substance bearing a change is propelled


through the skin by a low electrical current is : A
a. Iontophoresis
b. Phonophoresis
c. Functional neuromuscular stimulation
d. Transcranial electromagnet stimulation
304
e. Transcutaneus electro nerve stimulation

77. Which one of the following CPM (Continous Passive Motion) is contraindicated? C
a. Burn patients
b. Capsulotomies
c. Unstable fracture
d. Total knee replacement
e. Fractures with open reduction internal fixation

78. A 27-year-old woman complains of pain in the shoulder and suprascapular region 1
month after undergoing modified radical neck dissection for carcinoma of the thyroid.
Examination shows decreased sensation to light touch over the superior aspect of the right
trapezius and scapular winging with resisted shoulder abduction. Which of the following
nerve was most likely transected during the procedure? B
a. Subclavius
b. Long thoracic
c. Suprascapular
d. Dorsal scapular
e. Spinal accessory

79. The following are TRUE about individual with SCI, EXCEPT : D
a. Neuropathic pain is the major problem in more tha 80% of patients with SCI
b. More than 60% of individuals with chronic SCI develop symptoms of spasticity
c. Recovery of motor activity and residual muscle strength is a major area of interest in
rehabilitation
d. Recovery of motor function after SCI is not depend on plasticity of the motor cortex
and the corticospinal tract as seen in the stroke patients
e. Recovery mechanisms after SCIs may include sensory motor cortex reorganization, nerve
root recovery, axonal sprouting, changes in gray matter at the level of SCI

80. Gradually increasing pain and weakness and numbness in the hand that radiates up the
arm suggest : B
a. Amyotropic lateral sclerosis
b. Carpal tunnel syndrome
c. Cervical root syndrome
d. Brachial plexus palsy
e. Radial nerve palsy

81. The following are the clinical signs of Parkinson patient, EXCEPT : B
a. Difficult to initiate walking because of the bradykinesia
b. Consistent resistance exhibited only in flexion
c. Tremor involve on lower parts of the limbs
d. Patient tend to flexed trunk
e. Arm swing is decreased

82. A 60-year-old man without any significant pas medical history presents ton your
outpatient office with asymmetric atrophy, weakness, and fasciculations. He also complains
of some difficulty swallowing his meals and complains of a strained and strangled quality in
his speech. He describes normal bowel and bladder function. Which of the following was
most likely his diagnosis? A
a. Amyotropic lateral sclerosis
305
b. Primary lateral sclerosis
c. Spinal muscular atrophy
d. Multiple sclerosis
e. Poliomyelitis

83. The following factors is the body function and structures of ICF core set of risk factors
for falls in acute rehabilitation setting, EXCEPT: A
a. Previous falls
b. Vestibular function
c. Proprioceptive function
d. Blood pressure function
e. Visuospatial perception

84. Preventive measure in impending pathologic fracture of cancer patients: A


a. Monitor patient for pain in limb and spine
b. Perform chest physical therapy routinely
c. Progressive resistive exercises in a limb
d. Excessive flexion exercises of spine
e. Stretching lower limb joints

85. Global decline in cognitive capacity in a full conscious geriatric patients is commonly
caused by : C
a. Anxiety
b. Delirium
c. Dementia
d. Major depression
e. Couping behavior

86. Injury to the rotator cuff muscles and tendons of the shoulder is often occurs in? D
a. Boxer (Boxing athletes)
b. Rowing athletes
c. Football players
d. Tennis players
e. Runner

87. A discus thrower complains of pain in the upper part of shoulder. One of the finding on
examination is cross adduction chest positive sign, pain arch at 110°. What structure most
likely to be involved? E
a. Sternocostal joint
b. Suprahumeral joint
c. Glenohumeral joint
d. Sternoclavicular joint
e. Acromioclavicular joint

88. Martial arts player complains of back knee pain after kicks his opponent. Examination
of knee demonstrates pain during varus stress test. What is the most likely diagnosis? C
a. Strain popliteus muscle
b. Medial collateral ligament injury
c. Lateral collateral ligament injury
d. Posterior cruciate ligament injury
e. Anterior cruciate ligament injury
306
89. The following is TRUE about Myofascial Pain Syndrome (MPS), EXCEPT: C
a. Is one of the most common case of musculoskeletal pain
b. Is characterized by myofascial trigger points (TrPS) in skeletal muscle
c. The tender areas are generalized, diffuse, involving bilateral and symmetrical
muscles
d. TrP is defined as a hyperirritable spot associated with a hypersensitive palpable nodule in
a taut band
e. Electromyographic studies show that the trigger points within the taut band show
distinctive spontaneous electrical activity

90. A patient ambulates with a Trendelenburg gait. You suspect an injury to the: E
a. Sciatic nerve
b. Femoral nerve
c. Obturator nerve
d. Inferior gluteal nerve
e. Superior gluteal nerve

91. Anterior knee pain syndrome is associated with the following signs, EXCEPT: A
a. Positive anterior drawer test
b. Tight lateral thigh structures
c. Atrophy of the quadriceps
d. Q angle more than 15o
e. Lateral patellar tilt

92. A 42-year-old factory worker you have diagnosed with carpal tunnel syndrome. The
most important objective testing that confirms your diagnosis include one of the following
test: A
a. Nerve conduction studies and electromyography
b. Tinel’s test at the wrist
c. Sensory examination
d. X-rays of the wrist
e. Prayer test

93. Which maneuver is used to identify sacroiliac joint pathology? E


a. Ober
b. Thomas
c. Schoeber
d. Lasegue
e. Gaenslen

94. All of the following statements are correct regarding cerebral palsy , EXCEPT: C
a. Children with CP who do not gain independent sitting balance by 4 years of age have a
poor prognosis for motor skill development
b. The neurologic deficit is stable, but clinical manifestations vary as the nervous system
matures
c. Sitting by the age of 4 years is a good prognostic sign for walking
d. Children with GMFCS V had limited trunk control
e. Spastic type of CP prone to develop contracture

95. When hemophilic child has significant bleeding in the knee joint. The initial treatment
307
of choice is: A
a. Application of cold and pressure
b. Factor V should be given later
c. Application skin traction
d. Passive ROM exercise
e. Aspiration of joint

96. All of the following statements are correct regarding hemophilia, EXCEPT: D
a. After repeated hemarthrosis, irritating hemosiderin deposits occurs in the synovial lining
cells, the synovium becomes largerly by fibrous tissue
b. Most patients with hemophilia in whom joint problems develop have factor VIII or IX
deficiency
c. The prevalence of joint contracture in the severe from the disease between 50-95%
d. Treatment for acute hemarthrosis with ROM, traction, casts and exercise
e. The knees, elbows and ankles are the joints most often affected

97. The main differential diagnosis of gait abnormality in the age group 4 to 10 years is: D
a. Trauma
b. Osteogenic sarcoma
c. Juvenile rheumatoid arthritis
d. Legg-perthes aseptic necrosis
e. Slipped capital femoral epiphysis

98. Dyspnea, tachycardia and sudden decrease of SaO2 to less than 90% in phase I cardiac
rehabilitation may cause by the following, EXCEPT: C
a. Atelectasis
b. Pneumonia
c. Emphysematous
d. Pulmonary embolism
e. Exacerbation of CHF

99. The following phase I cardiac rehabilitation prescription is TRUE following CABG: A
a. The target heart rate should be at resting heart rate plus 20 bpm
b. Heart rate should be maintained at 120 bpm
c. Raising legs above the heart level
d. Lifting weight more than 4.5 kg
e. Isometric exercises

100. A 50-year-old woman with a history of insulin dependent diabetes mellitus,


gastroparesis, hypertension, and obesity. The nurses note that she is having frequent small
urinary voids with a weak voiding stream. What bladder mechanism is most characteristic
for this presentation? A
a. Flaccid detrusor with large volume bladder
b. Spastic detrusor activity with normal sphincter
c. Small volume bladder with sphincter flaccidity
d. Hyperactive detrusor with large volume bladder
e. Hyperactive detrusor with small volume bladder

Answer : A. IF NUMBERUS 1,2,3 ARE CORRECT B. IF NUMBERUS 1,3 ARE


CORRECT C. IF NUMBERUS 2,4 ARE CORRECT D. IF ONLY NUMBER 4 IS
CORRECT E. ALL NUMBER ARE CORRECT
308
101.The following is/are the positive effects of singing in group music therapy; E
1.Can improve mood
2.Improve social participation
3.Improve respiratory function
4.In quadriplegia would increase recruitment of accessory respiratory muscle

102.Flaccid dysarthria is common find in which disease below : A


1.Bell’s Palsy
2.Myasthenia Gravis
3.Guillain Barre Syndrome
4.Bilateral cortical Stroke

103.The Purpose of initial comprehensive assessment of sensory integrative problem are : A


1.To identify the impact of sensory processing and praxia problem
2.To provide information to assist with treatment planning
3.To identify specific sensory intergrative problem
4.To Identify the behavior problem

104.The following is/are the factor(s) have/has been reported to affect the tendon healing of the
hand E
1.Age
2.Scar formation
3.Pulley intergrity
4.The anatomic zone of injury

105.Secondary Frozen shoulder result from a known precipitating event such as : E


1.Cervical spondylosis
2.Tendonitis of Rotator Cuff
3.Fracture of shoulder region
4.Degenerative of arthritis of glenohumeral joint

106.Early management of whiplash syndrome involve(s): A


1.Bed rest
2.Semi rigid cervical collar
3.Analgesik and mild sedatives
4.Low weight intermitten cervical traction

107.NSAIDs must be used with caution in the elderly because have adverse drug reaction such
as : E
1.Gastrointestinal ulceration
2.Inhibition of platelet aggregation
3.Inhibition of antidiuretic hormone
4.Increased renal reabsorption of chloride and K+

108.In Checking out lower extremity prosthesis attention is directed to : E


1.Comfort
2.Stability
3.Alignment
4.Appearance

309
109.The functions of upper limb movements that must be considered in the orthotic prescription
is/are E
1.Prehension pattern
2.Release
3.Reach
4.Carry

110.When prescribed shoes for patient with rheumatoid arthritis, you should consider : B
1.Rubber and buffer heel for better contact and additional shock absorptions
2.Sufficiently wide heel, between 10 cm high
3.Broad forefoot section
4.Hard forefoot sole

111.The common complication in individuals with SCI is/are : E


1.Neurogenic bladder/bowels
2.Deep Venous thrombosis
3.Pressure ulcers
4.Pneumonia

112.The goal of medical treatment on Parkinson’s Disease are : C


1.Decrease dopamine action
2.Increased dopamine action
3.Increase cholinergic effect
4.Decrease cholinergic effect

113.What is post-polio syndrome E


1.A confirmed history of paralytic poliomyelitis
2.Partial to fairly complete neurologic and functional recovery
3.A period of neurological and functional stability of at least 15 years duration
4.Onset of 2 or more of the following problem since achieving period of stability

114.Ataxic Dysarthria is found in : B


1.Multiple sclerosis
2.Myasthenia Gravis
3.Friedrich’s Ataxia
4.Dystrophy progressive

115.Transcranial Magnet Stimulation : A


1.It can depolarize neuron and create evoked respon or change neural plasticity
2.It can study the activity of motor cortices via electromagnetic induction
3.It can denote interhemispheric differences in corticospinal input
4.is one of invasive methods

116.The following statement is/are TRUE about low power LASER E


1.The FDA considers low power laser as low-risk investigational device
2.The proposed theraupetic application include reductions pain and inflammation
3.GaAs LASER has a direct penetration of 1 to 2 cm and an indirect penetration of 5
cm
4.During the first trimester of pregnancy is contraindication of therapeutic application

117.The effects of ultraviolet radiating is/are : B (CUCUR 631)


310
1.Photochemical effects
2.Relief of muscle spasm
3.Have biological effects
4.Increase the pain threshold

118.The following is /are the characteristic(s) of Duchenne’s Muscular Dystrophy E (CUCUR


811)
1.The disease affects males
2.It is transmitted by X linked mode of inheritance
3.Measurement of serum creatinine kinase is the most reliable test
4.The Child’s mother, his sisters and martenal aunts are potential carriers

119.The following is/are the clinical characteristic(s) of Down Syndrome : E


1.The head usually larger than normal because of overdevelopment of facial bone and
muscle
2.The toes are usually short and in the majority there is wide space between the first
and second toes
3.Ligamentum laxity causing deficiency in grip strength and ankle strength that
developed on school age
4.Most significant musculoskeletal impairment are owing to hypotonia and ligamentum
laxity caused by deformity of the joint

120.The following is/are TRUE about Legg-Calve –Perthes Disease : E


1.Predispose to pain and late degeneration
2.Is classified as one of the osteochondrosis
3.Is a disorder affecting the capital femoral epiphysis
4.Is one of the most common hip disorder of childhood

121.Man, 55 yo, weight 85, height 165 cm. He got myocard infarct 1 month ago. The fasting
glucose level 250, total cholesterol 400, no hypertension. Factor that should be considered
to prescribe cardiac rehabilitation programs : C
1.Avoid all of exercise
2.Assessed the stress test
3.Limitation of daily activity
4.Reduced the risk factor for 2nd cardiac disease attack

122.For chronic lung disease various walking test with sub-maximal exercise have the benefit
below: A
1.To detect exercise intolerance
2.To detect functional limitation
3.To evaluate reconditioning
4.To detect etiology of limitation in lung function

123.Which of the following statements about static/isometric exercise is accurate ? D


1.It is inappropriate for all patient with heart disease
2.It result in reduced systolic blood pressure
3.It result in reduced pressure on the heart
4.It is result in increase heart rate

124.The Following is/are the synovial joint in the shoulder girdle A


1.acromioclavicular joint
311
2.sternoclaviculo joint
3.glenohumeral joint
4.scapula thoracic joint

125.Plantar flexor muscle are inactive by the time of : D (CUCUR 475)


1.Heel strike
2.Mild Stance
3.Foot-flat
4.Toe off

126.To stabilize the shoulder against the upper reaction force from a cane , patient need the
good function of these muscle : B
1.Pectoralis Major
2.Pectoralis Minor
3.Lattisimus dorsi
4.Subclavius

127.Risk of falling in cancer patient is/are due to : E


1.Hemiplegia secondary to a brain tumor
2.Chemotheraphy-related neuropathy
3.Steroid-induced myopathy
4.Orthosis hypotension

128.The common condition (s) seen in cancer patients is/are : A


1.DVT
2.Pneumonia
3.Pathologic fracture
4.Severe hypertension

129.Older adults are the least physically active group, the essential for healthy aging to
maintained regular physical activities is/are from exercise type : E
1.Balance exercise
2.Flexibility exercise
3.Endurance exercise
4.Muscle strengthening exercise

130. Assessment for geriatric patient with falls E


1. Visual system
2.polypharmacy
3.foots problems
4.environment factors

131.What rehabilitation program(s) is /are suited for case after --?- -week grade I ankle sprain :
D
1.Rest and fixation
2.Ankle Stretching Exercise
3.Lower Limb strengthening exercise
4.Weight Bearing as tolerated and ROM exercise

132.The following is/are TRUE about pyriformis syndrome : A


312
1.Examination reveals buttock tenderness, pain over the pyriformis fossa
2.Pain with sustained hip flexion, adduction and internal rotation
3.Pain is exarcebated by sitting and athletic activity
4.Initial treatment with pelvis traction

133.What is/are possible diagnose for this condition ? C (cucur 245)


1.Quadriceps Muscle injury
2.ACL rupture
3.PCL rupture
4.Meniscus torn

134.What is /are special test(s) positive for this condition : C


1.Patellar tilt test
2.Mc Murray test
3.Posterior drawer test
4.Anterior drawer test

135.The risk factors for disability after stroke is/are : E


1.Global afasia
2.Bilateral lesions
3.Impaired cognition
4.Sensory and visual deficit

136.The clinical predictor of aspiration on bedside examination in stroke patient include the
following : E
1.Tongue lateralization and tongue strength
2.Vocal “ wetness” When talking
3.Ineffective tongue
4.Altered cough mechanism

137.The following is/are action of the phenol injection for spasticity management ; A
1.Side effect of risk destruction of surrounding tissues
2.Effect on motor power is minimal weakening
3.Side effect is risk of dysesthesias
4.The duration of effect is 15 days

138.The rehabilitation program for delayed development is stimulation of E


1.Fine motor
2.Gross motor
3.Speech development
4.Cognitive and behavior development

139.The following factor(s) is /are the early treatment of CTEV D


1.Dennis-Brown Splint
2.Outflare orthopedic shoes
3.Surgical soft tissue release
4.Stretching and serial plaster correction

140.The following is/are the secondary complication of children with Brachial Plexus Injury E
1.Torticollis
2.The affected arm is shorter
313
3.Lack of awareness of the arm
4.Muscle atropy and joint contracture

141.An important goal(s) of cardiac rehabilitation is/are to E


1.Optimize the participation in society
2.Improved health related quality of life
3.Optimize domestic, occupational and recreational activity
4.Improving the condition for participant in physical capacity and mental status

142.The benefit(s) of pulmonary rehabilitation is/ are A


1.Improvement in the quality of life
2.Improvement in the exercise tolerance
3.Reduction in number of hospitalization
4.Able to complete simple activities with increasing heart rate more than 120 bpm

143.The toxic effect(s) of cigarette smoking is/are : C


1.Coronary artery relaxation
2.Elevated the carbon monoxide levels
3.Decreased platelet activation
4.Pregnant women has risk a low-birth-weight-baby

144.Man, 55 yo, weight 85, height 165 cm. He got myocard infarct 1 month ago. The fasting
glucose level 250, total cholesterol 400, no hypertension. Factor that should be considered
to prescribe cardiac rehabilitation programs : C
1.Avoid all of exercise
2.Assessed the stress test
3.Limitation of daily activity
4.Reduced the risk factor for 2nd cardiac disease attack

145.With the EMG Machine we can get the following result : A (cucur 341)
1.The severity of pathology
2.The level / location of motor unit pathology
3.The motor and sensory conduction velocity
4.The etiology of motor unit abnormality

146.The different between type I to contrast the Type II B skeletal muscle fiber is B
1.Type I are predominantly aerobic
2.Type I are predominantly anaerobic
3.Type II B are predominantly anaerobic
4.Type II B fibers are high hemoglobin content

147.The following is/are absolute contraindication of intraarticular steroid injection : E


1.Critical Weight bearing tendon
2.Lack of informed consent
3.History of True allergy
4.Infected tissue

148.Physical examination of someone with biceps tendinitis will typically reveal : D


1.Absent of tendon biceps reflex
2.Decreased of ROM of the elbow
3.Tenderness over the lateral aspect of the arm
314
4.Significant palpable tenderness at the bicipital groove

149.A girl patient with idiopathic scoliosis of the following is/are the sign(s) that the cobb’s
angle might be increasing rapidly : A
1.Her pubic hair is growing
2.Her breast are growing bigger
3.She has her menarche a month ago
4.The Risser’s sign is reaching grade 4

150.The inflammation of the thumb extensor pollicis brevis and abductor pollicis longus is
known as : B
1.de Quervain’s syndrome
2.Carpal Tunnel syndrome
3.Tenosynovitis of the thumb
4.Trigger finger of the thumb

NATIONAL BOARD EXAMINATION


PHYSICAL MEDICINE AND REHABILITATION
JULY 22, 2016

Choose the most appropriate answer

1. The Boutomierre deformity in a Rheumatoid hand is typically located in the following joint A
A. PIPs
B. MCPs
C. Radiocarpal
D. Ulnarcarpal
E. Carpometacarpals

2. The following is NOT the typical musculoskeletal and systemic manifestations of SLE D
A. ArthraIgias
B. Ligamentous Laxity
C. Male pain and weakness
D. Osteoarthritis with joint deformities
E. Psychological and neurologic disorders

3. Provocative test for acromioclavicular joint impingement is A


315
A. Cross-chest test
B. Relocation test
C. Apprehension test
D. Anterior drawer test
E. Anterior load and shift test

4. What is the highest neurologic level at which a person with complete spinal cord injury might
be capable of unassisted, independent transfer skills with or without equipment: C
A. C2
B. C4
C. C6
D. C8
E. T6

5. Which statement is TRUE concerning autonomic dysfunction? A


A. Bradycardia occurs in response to blood pressure elevations
B. Hypothermia is common because of poor thermoregulation
C. It is associated with injury below T9
D. Mostly it is happened in children
E. It is common in the acute period

6. A 45 year-old man with multiple sclerosis (MS) came to the outpatient clink complaining of
urinary incontinence. He was experience increased urgency and frequency of urination The
most likely urodynamic finding in this patient is B
A. Atonic bladder
B. Spastic bladder
C. Urge incontinence
D. Stress incontinence
E. Overflow incontinence

7. A 35-year-old woman presents a few days of progressive ascending muscle weakness. She had
a viral infection a few weeks earlier. On examination, you found diffuse weakness and
areflexia. The most likely finding in the CSF is B
A. High protein-high cell count
B. High protein-low cell count
C. Low protein-high cell count
D. Low protein-low cell count
E. Normal CSF

8. A patient complains of difficulty chewing. On examination he is found to have decrease:


strength of his muscles of mastication. Which of the following cranial nerves is responsible for
this motor function? B
A. Facial
B. Trigeminal
C. Oculomotor
D. Hypoglossal
E. Glossopharyngeal

9. In which of the following disorder would the highest elevation of creatine kinase (CK) be? C
A. Poliemyelitis
316
B. Myotonic dystrophy
C. Duchenne muscular dystrophy
D. Limb-girdle muscular dystrophy
E. Lmbert-Eaton mystathenic syndrome

10. You are asked to evaluate a 33-year old construction worker who is complaining of paresthesias
in the first and second digits of his right hand. Your physical examination shows no weakness
but a mild decrease in light touch over the thumb. You request a nerve conduction study to rule
out carpal tunnel syndrome and it turns out to be normal. On repeated history, the patient
indicates that on occasion, he gets a sharp, 'electric" pain travelling from his neck the right
hand. What are you missing? E
A. A Neuromuscular junction disorder affecting distal hand muscles
B. A median neuropathy at the wrist
C. A lower trunk tracheal plexopathy
D. A C8-T1 radiculopathy
E. A C6-7 radiculopathy

11. A 9-year-old boy with primary muscle disease is being evaluated to correct toe-walking.
Reasons for the toe-walking might include C
A. Hip-flexor weakness
B. Poor proprioception
C. Quadriceps weakness
D. Weakness of Plantiflexors
E. Hamstring hyperextensibility

12. A child with spastic hemiparesis is observed to have excessive plantar flexion and inversion of
the ankle during gait training, impairing his safety. An initial therapy to improve his ambulation
is A
A. Ice packs to the gastrocnemius muscle for 30 minutes prior to ambulation
B. Stimulation of the tibialis anterior at 3-5 Hz for 1 hour a day
C. An AFO with anterior channel spring
D. Phenol block of the peroneal nerve
E. An AFO with medial T-strap
13. The 3-year-old child has a normal gross motor development with characteristic D
A. Legs externally rotated
B. Immature balance
C. A faster cadence
D. Runs smoothly
E. Wide base

14. Congenital dislocation of the hip in older infant who cannot stand DOES NOT have clinical
signs at dislocation side with maneuver A
A. Trendelenberg
B. Ortolani
C. Galeazzi
a.
D. Allis

15. In phase I of the cardiac rehabilitation program, the patient is susceptible to the following major
complications, EXCEPT D
A. DVT
317
B. Stroke
C. Bleeding
D. Decubital ulcer
E. Electrolyte disturbance

16. Exercise has to be stopped if the patient has sign : B


A. 1 mm segment displacement
B. Frequent multifocal
C. 1st degree AV-block
D. Sinus rhythm
E. HR 60 bpm

17. The following statement is NOT correct for therapeutic prescription for patient with COPD D
A. The goals is to optimize oxygen needs and control of secretions
B. Instruct patient in diaphragmatic and pursed lip breathing
C. Instruct the use of metered-dose inhaler before exercise
D. Assess baseline endurance with Harvard stress testing
E. Train the patient energy conservation

18. Which one of the statement about exercise testing below is NOT correct ? C
A. Harvard step test is the symptom limited test
B. Monitoring should continue for at least 5 minutes after exercise stress test or until ECG changes
return to baseline
C. The absolute indications for terminating exercise testing is the increasing in systolic blood
pressure of ≥10 mmHg from baseline
D. The ventilator threshold (VT) that obtained from expired gases reflects a balance between
lactate production and removal in the blood
E. Exercise beyond the ventilator threshold is associated with metabolic acidosis hyperventilation,
and reduced capacity to perform work

19. The most common symptom experience of cancer patients related to their disease is : C
A. Pain
B. Nausea
C. Fatigue
D. Depression
E. Sleep disorder

20. A bone marrow transplant patient is receiving high-dose chemotherapy in preparation for stem
cell rescue. She asks you what she can safely do to preserve her of physical conditioning. You
recommend. B
A. Beginning ambulation 10 days after stem cell infusion
B. Beginning intent aerobic training after stem cell infusion
C. Daily living activities until blood cell counts have normalized
D. Avoiding functioning activities throughout the bone marrow acceptance
E. Using only a stationary bicycle to minimize the risk of bleeding related to thrombocytopenia

21. Manual lymphatic drainage is a gentle massage technique, achieves its effects through all the
following, EXCEPT : E
A. Stimulation of lymphangion motoricity
B. Accelerated sequestration of interstitial fluid
C. Acceleration of lymph filtration through nodes
318
D. Development of potential lymphatic anastomoses
E. Enhanced ultrafiltration of fluid out of the vascular space

22. A softball player suffered from shoulder pain on the right side. The pain increased when he
made repetitive rotation on his right shoulder. No radiating pain, no tingling sensation.
Shoulder ROM within normal limit. Neer and Hawkin-Kennedy tests were positive. Drop arm,
Yergason's and apprehension tests were negative. What is possibly diagnosis? A
A. Rotator cuff impingement syndrome
B. Shoulder subluxation
C. Bicipital tendinitis
D. Supraspinates tear
E. Frozen shoulder

23. A 24 years old far jump athlete suffered from shin pain. On the physical examination, found a
promotion on mid I/3 of shin area, which was pain on pressure palpation. Plain examintaion
showed a thickening of the bone cortex on mid 1/3 of tibia, with ossification. What is the
WRONG statement? B
A. March fracture
B. Should be operant
C. Proper test for promote balance modeling state of the bone
D. Possibility of already healed fracture of the mid 1/3 of tibia
E. The fracture can be occurred spontaneously as a consequence of repetitive injury

24. The Unhappy triad of Donohue is : C


A. ACL, lateral collateral ligament and posterolateral complex tears
B. PCL, medial collateral ligament and posterolateral complex tears
C. ACL, medial collateral ligament and meniscus tears
D. PCL, lateral collateral ligament and meniscus tears
E. ACL, PCL and cartilage tears

25. The following muscles arc an important contributor to lumbar stability, EXCEPT: C
A. Gluteus maximus
B. Rectus femoris
C. Gastrocnemius
D. Hamstring
E. Illiopsoas

26. What is the role of gluteus maximus in providing spine stability E


A. It primarily functions as a hip flexor
B. It saves to facilitate lumbar flexion
C. It controls repetitive lumbar rotation
D. It primarily functions as a hip abductor
E. It control acceleration and desceleration

27. Patellofemoral contact area is between : C


A. the last 20 degree of extension
B. 90-135 degree of flexion
C. 20-90 degree of flexion
D. 10-20 degree of flexion
E. 0-10 degree of flexion

319
28. The following statement is NOT correct regarding age-related psychological change in
cardiovascular system D
A. Maximal heart rate with exercise does decrease progressively
B. Decreased in the rate of early diastolic filling
C. Decreased myocardial contractility
D. Increased baroreceptor sensitivity
E. Decreased ejection fraction

29. The following statement is NOT correct regarding the neurophysiology of the bowel function:
E
A. Somatic efferents are under sympathetic efferent and voluntary (cortical) control
B. Afferent fibers originate from sensory receptors of the Gl tract
C. The intestine and rectal mocosa are sensitive only to distension
D. The external anal sphincter is innervated by the S2-S54 roots
E. Parasympathetic efferents inhibit GI tract motility

30. The earliest electro diagnostic abnormality in Guillain Barre syndrome is: A
A. Slowing or absence of F waves
B. Slowing of the sensory distal latencies
C. Slowing of cranial nerve distal latencies
D. Increased temporal dispersion of the CAMP
E. Slowing of lower extremity motor distal latencies

31. The minimum current amplitude, with a long pulse duration, required to produce an action
potential is A
A. Rheobase
B. Chroniaxie
C. Propagation
D. Depolarization
E. Relative refractory pencil

32. Which of the following is NOT a property of lasers? B


A. Coherence
B. Divergence
C. Collation
D. Monochromaticity
E. Minimal divergence of photon

33. The following statement is true regarding cryotherapy. EXCEPT B


A. The contraindications for criotherapy include cryoglobulinemia and cold allergy
B. Prolong cooling increases nerve conduction and metabolism
C. Cryotherapy may produce longer-lasting effects than heat
D. Shorter-term cooling may improve local circulation
E. Prolong cooling reduces spasticity

34. The following is the basic principle in pediatric prosthetic management, EXCEPT E
A. The prosthesis should be simple, durable, and light-weight
B. An upper limb prosthesis is fitted when the child is attempting to sit
C. A lower limb prosthesis is fitted when the child begins to pull up to stand
D. Most pediatric prostheses require replacement an average of 12 to 18 months
E. An articulated knee prosthesis is prescribed for child with transfemoral amputation at the
320
age 1 year

35. The key muscles for movements in operating a manual-opening terminal device for body-
powered prosthesis include all of the following, EXCEPT: A
A. Stemocleidomastoid
B. Serratus anterior
C. Pectoralis major
D. Latissimus dorsi
E. Deltoid

36. The quadrilateral socket has been recommended for person with what condition" E
A. High-activity sport
B. Unstable residual limb
C. Shorter residual limb
D. Atrophy adductor musculature
E. Geriatric and debilitated persons who walk with walker

37. Every patient with spinal cord injury should have a functional goal established from him. The
goals are established from the findings of the evaluations in accordance with C
A. The level of injury and vertebral fracture
B. The level of injury, age, sex, culture, family
C. The level of injury, age, sex, culture, motivation, family
D. The level of vertebral fracture, age, sex, family, motivation
E. The level of vertebral fracture, age, sex, culture. Motivation

38. This statement is WRONG in specific therapeutic exercise for scoliosis B


A. Done in and out of the brace
B. Stretching exercise of the convex side
C. EDLF exercise is done in the kneeling position
D. Muscle strengthening exercise of the convex side
E. Abdominal muscle strengthening exercise is included

39. The following statement are commonly used techniques of therapeutic massage E
A. Acupressure is often done to improving circulation
B. Deep friction manage is applied parallel the muscle fibers
C. A manage medium is used to loosen or stretch scar tissue
D. Stroking can help to break up adhesions in chronic injuries
E. Tapotement is primarily used for chest therapy in conjunction with postural drainage

40. Deformations can be found in advance stage of rheumatoid arthritis. EXCEPT: B


A. SubIuxation of the mcp joints
B. SubIuxation of the dip joints
C. Bautonierre deformity
D. Swan neck deformity
E. Ulnar drift

41. A 45-year-old dock worker wants to prevent a back injury Among the following, the most
important preventive measure would be A
A. Back muscle exercises
B. Use lumbosacral corset
C. Hip flexor muscle stretching
321
D. Quidriceps strengthening exercises
E. Use prophylactic NSAID medication

42. The bone mineral density results of a 48-year-old woman with chronic active hepatitis B but no
history of hormone-sensitive malignancy is highly suggestive of osteoporosis of the left femur.
Your treatment choice could include all of the following. EXCEPT: B
A. Calcium supplementation 1500 mg
B. Hormone replacement therapy
C. Fluoride supplementation
D. Bisphosphomites
E. Vitamin D

43. Which of the following is a complication that could result from the use of digital-rectal
stimulation as part of bowel care for a patient with C6 tetraplegia? C
A. Incontinence of mucus I hours after bowel care
B. Laxative-dependent colon
C. Autonomic dysreflexia
D. Mucosal laceration
E. Melanosis coli

44. You are asked to begin a bowel program on a spinal cord-injured patient with recess asset of a
complete lower motor neuron lesion. You recommend the following bowel care procedure A
A. Digital-rectal stimulation with manual removal of stool
B. Bowel care to be carried out twice weekly
C. Stimulant laxative every other day
D. Use of Valsava maneuver
E. Saline enemas

45. In foot equinovarus spastcity, the following muscles involved that can be injected with botox,
EXCEPT: C
A. Flexor digitorum lumgus and brevis
B. Tibialias anterior and posterior
C. Extensor hallucis longus
D. Lateral gastrocnemius
E. Medial gastrocnemius

46. What is the MOST frequent initial symptom of multiple sclerosis'? A


A. Sensory symptoms
B. Gait abnormality
C. Incoordination
D. Visual loss
E. Vertigo

47. A 47 year-old woman with secondary progressive multiple scleroses has mild cognitive ….
And severe lower-limb spasticity that limits her ability to transfer independently and to perform
toilet hygiene. What is the most effective treatment for this patient? B
A. Range of motion exercise
B. Inrathecal baclofen
C. Superficial hearing
D. Stretching exercise
E. Ice application
322
48. The following factor is NOT associated with an increase in falls in Parkinson's disease : A
A. Tremor
B. Dementia
C. Depression
D. Loss of arm swing
E. History of prior falls

49. The following factor care and rehabilitation phase after spinal cord injury, which area is the
most common site of presence pressure ulcer development? C
A. Scapular
B. Ischium
C. Sacrum
D. Occiput
E. Heel

50. With regard so acquired amputations in children, which of the following is TRUE? B
A. Dissarticulation-level amputations are preferred to help prevent neuroma formation
B. Dissarticulation -level amputations are preferred to avoid terminal bony overgrowth
C. Disarticulation-level amputations are preferred to maximize soft tissue wound healing
D. Terminal overgrowth is rare under age 12 after a diaphyseal or metaphyseal amputation
E. Transdiaphyseal amputations are preferred to facilitate maximal growth of the residual limb

51. Which of the following statements is TRUE regarding Erb's palsy in infants? B
A. The primary goal of surgical interventions is cosmesis
B. Proximal muscles have better functional recovery than distal muscles
C. The most common goal of tendon transfer is to provide finger FlexIon
D. Implantation of avulsed nerve roots has a high success rate in this age group
E. Diffuse fibrillations with decreased motor unit recruitment in a 1-month-old infant confirms the
need for surgical repair

52. The most appropriate exercise program for 54-year-woman with lower extremity weakness
from childhood polio is : C
A. Open-chain kinetic eccentric exercise training.
B. No exercise program, as exercise is known to increase weakness
C. Submaximal isometric and weight training with frequent rest breaks
D. Functional electrical stimulation of weak muscles to avoid central fatigue
E. Progressive resistance exercise for muscles with less than grade 3 strength

53. The most common cause of hypotonia in term infants is : E


A. Congenital myopathy
B. Electrolyte abnormality
C. Congenital myasthenia gravis
D. Infantile motor neuron disorder
E. Central nervous system disorder

54. The potential change due to exercise training for patients with chronic bean failure is increases
inE
A. Sympathetic nervous system activity
B. Submaxinial exercise ventilation
C. Resting exercise heart rate
323
D. Blood lactate level
E. Leg blood flow

55. In men, the energy cost of sexual activity with married partner typically does not exceed how
many metabolic equivalents? B
A. 2 METs
B. 5 METs
C. 7 METs
D. 10 METs
E. 12 METs

56. The following management of healed foot ulcers is true, EXCEPT: B


A. Proper care of nails
B. Foot immobilization
C. Not walking barefoot
D. Accommodative footwear with adequate toe box
E. Rigid rocker sole shoes for preventing recurrent fore foot plantar ulcers

57. Postural drainage been used as a form of chest physical therapy to enhance the flow of airways.
However, one should AVOID this modality on patients E
A. With sputum production greater than 30 ml per day
B. Using inhalant bronchodilator
C. Who have difficulty coughing
D. Using nebulizer therapy
E. With pulmonary edema

58. Which of the following drugs can cause symmetrical distal motor sensory neuropathy? D
A. Piroxicam
B. Methotrexate
C. Ciprofloaxin
D. Nitrofurantoin
E. Carbamazepin

59. The most common causes of dementia in the elderly is C


A. Untreated or undertreated pain
B. Poor functional status
C. Alzheimer disease
D. Immobilization
E. Hydrocephalus

60. 80 year old man presents to your office with axial back pain that worse with standing and
walking. At times he also experiences pain radiating from the lumbar spine into the legs when
walking. What the most likely diagnosis? E
A. Sacroilitis
B. Ischias syndrome
C. Piriformis syndrome
D. Spondylosis lumbalis
E. Lumbal canalis stenosis

61. In endurance athlete with gluteal pain, which diagnosis is NOT likely to apply? B
A. Femoral neck stress fracture
324
B. Iliopsoas muscle strain
C. Lumbar radiculopathy
D. Spondyloarthropathy
E. Sacroiliac joint pain

62. The following statements is TRUE about Mallet finger C


A. The injury results from direct blow to the flexed DIP joint of the long, ring or small lingers
B. Is not common athletic injury in sport such as soft-ball, baseball and basketball
C. The injury leading to disruption of the extensor tendon over the joint
D. The injury lending to disruption of the flexor tendon over the joint
E. Untreated mallet fingers may develop a Boutonniere deformity

63. Turf toe is a metatarsophalangeal sprain occurs with hyperextension injuries of the great toe.
Which sports that generally occurs ? E
A. Table tennis
B. Basket ball
C. Gymnastic
D. Jumping
E. Soccer

64. A patient complains of difficulty walking down steps, you would expect to find weakness of B
A. Iliopsoas
B. Quadriceps
C. Hip adductors
D. Gastrocnemius
E. Gluteus medius

65. The following is TRUE about femoral neck C


A. The normal Angle of inclination of the femoral neck to the shaft in the frontal plane is 140
degree
B. The normal angle of inclination of the femoral neck to the shaft in the frontal plane is 100
degree
C. The normal angle of inclination of the femoral neck to the shaft in the frontal plane is 125
degree
D. It is called coxa valgus if the angle 150 degree
E. It is called coxa valgus if the angle 90 degree

66. The fallowing is TRUE during gait cycle D


A. About 40% of the normal gait cycle is spent in swing phase
B. As the speed of pit increases, the period of double support increased
C. The Ions extremity tends to rotate externally through the swing phase
D. In the middle of stance phase and at push-off, the entire lower extremity begins to route
externally
E. As the external rotation in the lower extremity occurs, a degree of increased instability of the
medial aspect of the hip, knee and ankle

67. The protein found in greater quantity in the thick filament of myofibrils is A
A. Actin
B. Myosin
C. Desmin
D. Troponin
325
E. tropomyosin

68. Eccentric contraction's is distinguished from concentric muscle contraction by eccentric


contraction's greater A
A. Maximal force production
B. Use in abnormal gait patterns
C. Force with slower contraction
D. Electrical activity at a given tension
E. Oxygen consumption at a given work load

69. The most important substance/structure to hold water inside the joint cartilage is E
A. Collagen
B. Fibroblast
C. Fibronectin
D. Hyaluronate
E. Proteoglycan

70. What is interferential current (IFC)? A


A. Electrical waves that differ slightly in frequency can interact with each other and produce
product waveforms with frequencies equal to the sum or different of the original waves
B. Electrical waves that some in frequency can interact with each other and produce product
waveforms with frequencies equal to the each of the original waves
C. Pairs of electrodes associated with each wave arc placed so that their states interfere at out site
of treatment
D. The waves can penetrate tissue with discomfort
E. IFC can be used only to produced TENS effects

71. The following are the contraindications to pneumatic pumping, EXCEPT B


A. Cellulitis
B. Poststernatecsomy edema
C. Compromaind perfusion
D. Severely impaired sensation
E. Acute deep venous thrombosis

72. When prescribing traction, the following parameters should be specified, EXCEPT B
A. Duration
B. Repetition
C. Positioning
D. Amount of pull
E. Intermittent or continuous administration

73. Compared to a single-axis knee, polycentric axes knee unit has advantages as the following,
EXCEPT B
A. Stable
B. Heavier
C. Improved sitting
D. Greater toe clearance in swing
E. Is indicated in amputee with weak hip extensor

74. Which of the following is NOT a demonstrated advantage of immediate postsurgical fitting for
the upper limb amputee? E
326
A. Decrease incidence of edema
B. Increased prosthetic wear and use
C. Decreased incidence of panthom limb pain
D. Enhanced psychology adaptation to amputation
E. Decreased need for skilled therapy for prosthetic training

75. Ambulatory assistive device for the elderly if there are limitations with painful joints in the
hand is E
A. Cane
B. Walker
C. Crutches
D. Wheelchair
E. Forearm crutches

76. The following is NOT the contraindication to manipulation : A


A. Neck and back pain
B. Vertebral malignancy
C. Ligamentous instability
D. Active spinal infection
E. Active inflammatory arthropathy

77. The capability of the brain to reorganize by forming new neural connections throughout life
(brain plasticity) base on the following principles. EXCEPT : E
A. There is a need for the brain to use experience to initiate a new synaptic connection between
neurons
B. If hemisphere is damaged, the intact hemisphere may take over some of its functions
C. Pharmacologic agents to reduce infarct site and promote repair can enhance plasticity
D. Sensory stimulation enhances the sensory representation of the body part
E. Treat post-stroke depression, use drugs that induce drowsiness

78. The following is an appropriate exercise program for a patient with ankylosing spondylitis D
A. Lying supine for 10-20 minutes twice a day, sleeping on a firm mattress
B. A twice-daily ROM and stretching should be done for wrist joints
C. Sports such bowling should be recommended
D. Aerobic exercise in the pool
E. Spinal flexion exercise

79. Intervention for the patient with rheumatoid arthritis of the hands which slow the progression of
the joint erosion include B
A. Naproxen
B. Metotrexate
C. Stretching of intrinsic hand muscle
D. Strengthening of hand intrinsic muscle
E. Superficial application of heat to small joint

80. A woman 75-year-old with midthoracic back pain from T6 acute spinal compression fracture
sustained 1 week earlier. Images show no evidence of malignancy. You recommend : E
A. Gentle progressive flexion exercise
B. Functional electrical simulation to the thoracic spine
C. Bed for 14 days follow by progressive mobilization
D. High lumbosacral corset, cold laser theraphy, end extension exercise
327
E. Thoracolumbar corset, narcotic analgesic, and extension exercise

81. What would the earliest and and most common symptom of cervical subluxation in rheumatoid
arthritis? B
A. Hyperflexic bladder
B. Pain radiating up into the occiput
C. Asymmetry upper extremity weakness
D. Numbness in the lateral aspect of the arm
E. Dizziness when going from spine to standing

82. A 68-year old man with degenerative joint disease who reports the gradual onset of anterior-
lateral, MSS pain, worsened by raising his arm above shoulder level. The pain is worse at night,
particularly when lying on the affected side. There is tenderness over the deltoid and pain with
resisted abduction. Passive and active shoulder motions are normal. Neer’s maneuver and
empty can tell VC negative. Your initial recommendation is : E
A. A shoulder x-ray
B. MRI of the shoulder
C. Passive ROM ROM exercises
D. Isotonic deltoid strengthening with arm above shoulder height
E. Betamethasone and lidocaine injection into subdeltoid bursae

83. Predictor of functional recovery in stroke patient includes the following factors. EXCEPT. B
A. ADL score at rehabilitation admission
B. Educational level anti intelligence
C. Consciousness at onset
D. Previous stroke
E. Age

84. The Brown-Sequard syndrome result in E


A. Sparing of posterior column function
B. Paralysis of the arms more than the legs
C. Paralysis of one arm and contralateral leg with crossed hypalgesia
D. Paralysis of the arm and leg on one side with loss of position sense and pain sensation
E. Paralysis of the arm and leg on one side, with loss of ipsilateral position sense and
contralateral pain sensation

85. The following statement is true about virtual reality training for stroke patient. EXCEPT : D
A. Virtual reality stimulate "real life"
B. Repetitive stimuli can be introduced
C. Gives immediate performance feedback
D. Stimulates potentially hazardous situation with risk
E. Allow users to "forget" they are in a testing environment

86. The lesion location of Lacunar infarct with clinical sign pure sensory deficit is at B
A. Pons
B. Thalamus
C. Cerebellum
D. Corona radiate
E. Cerebral peduncle

87. Which of the following disorders is characterized by muscle hypertrophy? D


328
A. Distal myopathy
B. Limb-girdle dystrophy
C. Scapulo.. dystrophy
D. Becker-type muscular dystrophy
E. Fascioscapulohumeral dystrophy

88. The following are the key points in the safe inpatient rehabilitation of children with a Traumatic
Brain Injury (TBI), EXCEPT A
A. A history of ADHD is not important in children with TBI
B. Review medications for potential drug-drug interaction
C. Rehabilitation prescription with safety precautions
D. Monitoring of missed fracture and agitation
E. Spasticity management

89. The following measurement is true regarding leg-knish discrepancy in children, EXCEPT C
A. Apparent leg-length and actual leg-length discrepancy may differ from functional leg-length
discrepancy
B. Serial mammograms or computed tomography measures leg-length more precisely
C. Differences of 0.5 cm can be corrected with shoe lift
D. Result from septic arthritis
E. Result from hip dislocation

90. The clinical sign of congenital muscular torticolis include of the following. EXCEPT: E
A. Soft enlargement in sternocieidomastoid (SCM) muscle
B. Olive-sign in mobile within the belly of SCM muscle
C. Secondary deformity including orbital asymmetry
D. Between 10-20% develop persistent torticolis
E. Typically there are neurological deficits

91. A 45 year old administrator comes to your office for advice before starting moderately
strenuous exercise program. He has led a relatively sedentary life-style but has been healthy
and is asymptomatic. Physical examination and laboratory finding were normal. Before
embarking on his exercise program he should have C
A. No further testing
B. Thallium stress testing
C. Exercise stress testing
D. Coronary angiography
E. Pulmonary function testing

92. During exercise, there is an increase in a person's A


A. Stroke volume
B. Diastolic pressure
C. Venous compliance
D. Total peripheral resistance
E. Pulmonary arterial resistance

93. Ventricular tachycardia is marked by D


A. One PVC on every other beat repeatedly
B. Two PVCs on every other beat repeatedly
C. Two premature ventricular contractions (PVCs) in row
D. Three PVCs (or more) PVCs in a row without any normal beats
329
E. The rate about 150-500 beats/minute and the absence of a distinct QRS complex

94. To optimize the benefits an exercise program for patient with peripheral vascular disease,
patient should receive exercise: B
A. Walk until claudication occurs, rest until subsides, continuous and repeating the cycle for 1
hour in every one week over period of 12 weeks
B. Walk until claudication occurs, rest until subsides, continuous and repeating the cycle for
1 hour each day over period of 12 weeks
C. At least 1 sessions weekly exercise program over period of 12 weeks
D. At least sessions weekly exercise program over period of 2 weeks
E. Intermittent walking to maximal pain without rest

95. Characteristics of aging include : E


A. Increased ability to adapt in response to different environments
B. Increased mime capacity of organ system
C. Increased internal homeostatic
D. Increased capacity to respond to stress
E. Increased vulnerability to frailty

96. The multiple factors of constipation in older people include the following factors, EXCEPT: A
A. Medications such parasympathomimetics
B. Prolonged use of stimulation laxative
C. Low dietary fiber and fluid intake
D. Impaired rectal sensation
E. Sedentary habits

97. Medical causes falls in old age include the following. EXCEPT B
A. Carotid sinus syndrome
B. Valgus deformities of the knees
C. Syncope attacks due to arrhythmia
D. Muscle weakness due to neurologic disease
E. Syncope attack due to hypotension

98. Medical emergency of exercise - associated collapse in athletic is D


A. Benign exercise associated collapse
B. Heat exhaustion
C. Muscle cramps
D. Hypoglycemia
E. Hypothermia

99. The following is the classification of grade II lateral ankle sprain C


A. Normal drawer sign and talar tilt
B. Tom calcaneofibular ligament (CFL)
C. A partial tear with moderate instability
D. Drawer sign > 10 mm and talar tilt > 15 degree
E. Partial tear of anterior talofibular ligament (ATFL)

100. The following statement is true regarding Pes anserinus syndrome, EXCEPT E
A. Is found commonly in long-distance runners
B. It is usually cause by overuse friction or by a direct contusion
C. It is also common in conjunction with osteoarthritis in the elderly
330
D. The injuries result from pivoting, cutting, jumping and deceleration
E. Other factors of injury include varus knees and excessive internal rotation

101. Differential diagnosis of male osteoporosis is/are the following : B


1. Hypogonadism
2. Hypothyroidism
3. Multiple myeloma
4. Hypoparathyroidism

102. For the process of biomechanical job analysis, the following work parameter(s) must be
individually assesed : E
1. Work station, hand tools and equipment
2. Parts and materials needed
3. Environmental conditions
4. Repetition of job tasks

103. The differential diagnosis of buttock pain include : E


1. Greater trochanter bursitis
2. Posttraumatic arthritis
3. Myofascial pain
4. Malignancy

104. The following statement is true regarding Pes anserinus syndrome, EXCEPT E
1. Serum positive for rheumatoid factor
2. Bilateral symmetrical joint disease
3. Erosive changes radiographically
4. Chronic proliferative synovities

105. Thumb-in-palm is typically spasticity in stroke patient. The following musce(s) involved that
can be treatable with botox is/are : E
1. Flexor pollicis longus
2. Flexor pollicis brevis
3. Adductor pollicis
4. Opponens

106. The following is/are the advantage(s) to partially supporting body weight on the treadmill for
gait training in stroke patient: A
1. It facilitates a reciprocal stepping pattern
2. It allows upright posture
3. It minimize fall risk
4. It is not task specific

107. The predictor(s) of functional outcome in stroke patients include the following factor(s) E
1. Level of social support
2. Urinary incontinence
3. Severity of paralysis
4. Sitting balance

108. The oral medications to managing spasticity may be limited by side effects such as A
1. Ataxia
2. Sedation
331
3. Hypotonia
4. Polyneuropathy

109. A 6 year old girl with spastic quadriparesis needs a new wheelchair prescription. Patient
factors)that influence the prescription include all of the following A
1. Presence of lower limb spasticity
2. Patient's weight and height
3. Degree of trunk control present
4. Presence of Moro reflex

110. The following is/are the cause(s) of ischemic stroke in young adults E
1. Rheumatic heart disease
2. Infective endocarditis
3. Patent foramen ovale
4. Vasculitis

111. The following is/are the level Gross Motor Function Classification System is used to classify
the mobility of people with CP : C
1. Level II: ambulatory with no assistive device, indoors and out
2. level III: standing ineffective for distance ambulation, requires wheelchair in home and in
community
3. Level IV: upright ambulatory in home but has impaired balance for outdoors
4. Level V: very limited movement, even with the use of an adaptively controlled power wheel
chair

112. Individuals at risk for Deep Venous Thrombosis (DVT) include those who have sustained
local trauma to a vessel, have a hypercoagulable disorder, or have been immobilized by bed or
casts. The most common locations of DVT include the calf, tight, arms, and pelvis. The
symptoms and signs are: E
1. Pain in the calf with walking
2. Pain in the calf thigh
3. Positive Homan sign
4. Swelling of the calf

113. Changed plasma levels of this substance(s) have been determined to be a risk factor for
atherosclerosis D
1. Reduced levels of LDL
2. Reduced levels of vit E
3. Elevated plasma of HDL
4. Elevated plasma of homocystein

114. Risk factor(s) for foot ulceration in diabetics including E


1. Amputation of contralateral limb
2. Loss of protective sensation
3. Leg length discrepancy
4. Improper foot wear

115. The following is/are the indication for measurement of bone density E
1. Adults taking medications such as phenytoin
2. To monitor treatment effect of osteoporosis
3. Women discontinuing estrogen therapy
332
4. Woman aged 65 and other

116. In prescribing pain treatment for elderly patients, the following is/are the proper approach D
1. Use high-dose, short acting narcotics initially to get the pain control
2. Avoid use of acetaminophen, due to liver toxicity
3. Start with aspirin, which is safe and efficient
4. Start low and go slow for all medication

117. A recreational tennis player had right elbow pain since 3 weeks ago. The pain increased
gradually with the time, especially after she played tennis, and extended the wrist against
resistance on dorsal aspect of the hand. What is/are the proper management(s) for this particular
patient? E
1. Avoid wrist extension
2. Check and adjust the grip of racquet
3. Wris extensor eccentric strengthening exercise
4. Ultrasound diathermy on lateral epicondylus area

118. A gymnastic girl, felt pain on her left ankle, after suspinated-inverted left ankle position when
she landing from somersault. She felt pain on anterolateral area. Ankle anterior drawer test
positive on the left side, pain ankle radiographic on stress varus position showed the talar tilt
15th on left ankle. What is/are the possible diagnosis? B
1. ATFL tear
2. Deltoid ligament tear
3. CFL tear
4. Planta fascia tear

119. The two major “crutch walking’muscles of the shoulders are C


1. Teres minor
2. Latissimus dorsi
3. Pectoralis minor
4. Lower pectoralis major

120. The characteristic (s) of anatomy and biomechanic on the ankle joint is//are the following A
1. The subtalar joint is the key to the biomechanic of foot and ankle
2. It has 15 to 20 degrees of dorsiflexion and 35 to 40 degree of plantar flexion
3. It must invert at toe-off to lock to the joints for rigidity and efficient transfer of power
from the Achilles mechanism through the ankle forefoot
4. The medial structure around the ankle are the peroneus brevis and longus tendons and sural
nerve

121. Which statement(s) below is CORRECT regarding the rotator cuff ? C


1. The subscapularis muscle act as an external rotator
2. The infraspinatus muscles act as an external rotators
3. The suprapinatus active in shoulder depression
4. The teres minor muscles act as external rotators

122. The mode of action(s) of to reduce muscle spasticity is …through the following mechanism D
1. To stimulate P substance
2. To stimulate Glutamate
3. To stimulate Aspartate
4. To stimulate GABA
333
123. The following technique(s) is/are used to measure the body composition: E
1. Bioelectrical impedance analysis
2. Dual energy x-ray absorptiometry
3. Near-infrared intractance
4. Skinfold measurement

124. Which of the following statement(s) is/are TRUE? E


1. Potassium play an essential role in muscular contraction and nerve conduction
2. Fat is aconcentrated source of energy and act as carrier of fat-soluble vitamin
3. Potassium helps in the transport of glucose across cell membranes
4. Body water constitutes 65% to 75% of the weight of muscle

125. The preposthetic phase program(s) for upper limb prosthesis include the following E
1. Desentization
2. Joint mobilization
3. Muscle strengthening
4. Shaping and shrinking

126. The disadvantage of single axis foot is/are A


1. Heavy weight
2. Poor at energy storage
3. Poor on uneven terrains
4. Reached foot flat slowly

127. Which of the following knee types provides good stability in early stance phase and facilitates
flexion while weight bearing during the pre-swing phase (terminal stance) of the gait cycle? D
1. Stance-phase control
2. Manual locking
3. Single axis
4. Polycentric

128. This statement(s) is/are the contraindication for prenatal exercise E


1. At risk for premature labor
2. Severe hypertensive disease
3. Poor obstetric history
4. No prenatal care

129. Which of the following statement(s) is/are TRUE regarding circuit weight training (CWT)? A
1. The level of resistance 30-60% of 10 RM with 10-15 repetitions per station
2. It is usually doen after aerobic phase, on alternate days, 3 times per week
3. The number of sets varies from 1 to 3 and the number of stations varies from 5 to 18
4. Each CWT session should last from 20 to 30 minutes with rest intervals of 10 minutes between
stations

130. The following statements is/are TRUE regarding aquatic-exercise programs B


1. The target heart rate is preferred for estimating intensity
2. The recommended water temperature for cardiac patients ranges from20-22 degree Celsius
3. The exercise enhancing venous return and augmenting stroke volume
4. The exercises are more stressful on bones and joints than land-based weight bearing exercise

334
131. The following is estimating burn area according rule of nine A
1. Each lower extremity is 18%, 9% on each surface
2. Each upper extremity is 9%, 4.5% on each surface
3. The head is 9%, 4.5 % on each surface
4. The genital areas is 2%

132. The following is/are the differential diagnose(s) of lateral elbow pain A
1. Extensor –suspinator strain
2. Cervical radiculopathy
3. Lateral epicondylitis
4. Flexor-pronator strain

133. A-45-year old woman presents chronic pain in the region of the Achilles tendon. The Achilles
tendon appears swollen. Appropriate initial management would include D
1. Cortison injection of the Soelus muscle
2. Cortison injection of the Achilles tendon sheath
3. Immobilization in ankle plantar flexed position with limited weight bearing
4. Use of a small heel lift with initiation of gastro-soleus stretches as tolerated

134. In the case of low back pain, when we suspect Paget’s disease, the laboratory test that must be
order to confirm the diagnosis is/are D
1. CPK
2. Amilase
3. Rheumatoid factor
4. Alkaline phosphatase

135. The following is/are clinical indicator(s) of possible swallowing impairment E


1. If the patient have difficulty swallowing his/her secretions
2. If the patient have an open mouth posture with drooling
3. If the patient’s voice wet-gurgly
4. If the patients’s speech slurred

136. The functional outcome of complete T3 paraplegia is/are A


1. Independent in bowel and bladder routines
2. Independent in wheelchair propulsion
3. Independent in pressure relief
4. Standing without orthosis

137. The propioceptive disturbance would be found in the following disease A


1. Stroke
2. Tabes dorsalis
3. Brown-sequard syndrome
4. Funiculus anterior lesion

138. The characteristics of cerebral palsy is/are as follow B


1. The underlying neurologic lesion must be static
2. The process which cause the cerebral palsy is still active
3. The sign of cerebral palsy is a disorder of movement and posture
4. It is caused by progressive injury which affects the mature brain

139. The following statement(s) is/are TRUE about myelodysplasia A


335
1. Folic acid supplementation, taken prenatally and during the first trismester, reduce the
incidence of myelodysplasia by 70%
2. Neurologic bladder and bowel dysfunction affects more than 80% children with
myeninomyelecele
3. Myeninomyelecele is herniation of spinal cord and the meningen through the vertebral
defect
4. Spina bifida occulta is a bony defect, the spinal cors and the meninges

140. The cardinal clinical sign(s) for SMA Type I Werdnig Hoffman Disease B
1. Weak cry
2. Tremor of hands
3. Frog posture
4. Weakness of legs, predominantly proximal

141. The appropriate candidates of chronic heart failure patients for exercise training if A
1. Have no more than moderate fatigue
2. Able to speak without symptoms of dyspnea
3. Have rales present in less than half of the lungs
4. A central venous pressure of more than 12 mmHg

142. What kind of exercise can be done as early rehabilitation approach(es) in ICU setting? E
1. Neuromuscular electrical simulation
2. Speech language therapy
3. Chest physical therapy
4. Transfer out of bed

143. The objective(s) of giving pursed-lips breathing is/are A


1. Introduce the patient of how to control respiration
2. To decrease the respiratory rate
3. To increase oxygenation
4. To decrease tidal volume

144. A number of factor(s) that place stroke patients at risk for all include E
1. Impulsity
2. Bilateral stroke
3. Use of deuretics
4. Visuospatial deficit

145. Contributory factor(s) in cancer-related fatigue including E


1. Immobility
2. Malnutrition
3. Chemotherapy related
4. Mood disorder (depression)

146. The following statement(s) is/are true about biceps tendinitis E


1. Yergason’s may reduce pain at the shoulder
2. Patients may note pain with forward elevation of the arm and elbow flexion
3. Local injection of lidocaine in the bicipital groove is of value for diagnostic purpose
4. Return to sports when the athlete is pain-free, usually 4 to 6 weeks when there is no
surgery and 3 to 6 months when there is surgical intervention

336
147. The following statement(s) is/are true about pain syndrome A
1. Calcaneal spur maybe seen on lateral x-ray of the heel at the origin of flexor digitorum
brevis
2. Stretching of the heel cord and plantar fascia 2 to 3 times a day is recommended
3. Results from repetitive heel impact along with hyperextension of the toes
4. A single steroid injection at the plantar facsia origin is not recommended

148. The psychological effect(s) of spinal traction is/are A


1. Reduction of pain
2. Reduction of compression
3. Reduction of muscle spasm
4. Prevention and loosening of adhesion within nerve root and adjacent vertebral body

149. The following statement(s) is/are true regarding the Neuromuscular Electrical Stimulators
(NMES) E
1. When NMES is used to activate multiple muscles to attain specific functional goals, it is
called Functional electrical stimulation
2. NMES used for the stimulation of muscle with intact peripheral nervous system or muscle
that are decentralized
3. It is usually modulated to provide interrupted current for stronger motor excitation
4. It can also convert type II fast muscle fibers into type I slow fiber

150. Biofeedback is used in patients in what condition? A


1. Urinary and focal incontinence
2. Chronic neck and back pain
3. Peripheral nerve denervaton
4. Receptive aphasis

NATIONAL BOARD EXAMINATION

337
PHYSICAL MEDICINE AND REHABILITATION
DES 2016

CHOOSE THE MOST APPROPRIATE

1. (36)For short and very short transradial amputation level: cucurulo 479
A. Limit elbow flexion strength and elbow range of motion
B. Ideal for elbow suspension
C. Major advantage due to cosmetic problems
D. Optimal for body powered prosthetic restoration
E. Limit abduction shoulder range of motion

2. (37) Spinal orthotic which is choice for maximum immobilitation of the spine is:
A. Flexion Extension-Lateral Control Orthosis (Knight-Taylor Brace)
B. Plastic Body Jacket
C. Flexion-Control Orthosis (Jewett Brace)
D. Flexion-Extension Control Orthosis (Taylor Brace)
E. Flexion-Extension-Lateral-Rotary Orthosis (Cowhorn Orthosis)

3. (38) Exercise that doesn’t use neuroplasticity principle is:


A. Robotic Training
B. Virtual reality training
C. Constrain induced movement therapy
D. Body weight-supported treadmill training
E. Traditional therapy method

4. (39) The following statement is NOT true about endurance exercise response in adult
human:
A. Steady-state exercise reflects a balance between the energy required by working
the muscles and the rate of ATP production through aerobic metabolism
B. The greatest improvement in VO2max occurs when exercise involves the
use of large muscle groups
C. A linear increase in systolic blood presssure occurs with increasing exercise
intensity
D. Long-term exercise in systolic blood pressure occurs with increasing exercise
intensity
E. Arteriovenous oxygen difference is greater at maximal exercise

5. (40) The methode frequently are use on CP which bases treatment on activation of
postural development and equilibrium reaction to guide normal development is:
A. Bobath methode
B. Vojta methode  postural
C. Deaver methode  extensive bracing limit
D. Phelp methode
E. PNF methode

6. (1) A 50-year-old man has a right hemispheric infarct. He has mildly increased tone on
the left side with grade 3/5 movement at the shoulder, elbow and hand. He has mildly
338
decrease response to pinprick and propioception throughout the left side. He is noted to
use his right hand exclusively for feeding tasks, and he leaves some food untouched of
the left side of his plate. The most likely reason for this patient’s difficulty with eating
is: C
A. Feeding apraxia
B. Ideomotor apraxia
C. Left unilateral neglect
D. Left hemisensory deficit
E. Motor planning impairment

7. (2) A 65-year-old patient with a severe resting tremor of approximately 4 to 6 Hz,


affecting primarily the left distal upper extremity. On physical exam, you also note a flat
facial affect and slow shuffing gait. If you were to prescribe medication to address this
patient’s symptoms, which drug be most appropiate? D
A. Benzodiazepines
B. Carbamazepine
C. Propanolol
D. Levodopa
E. Diazepam

8. (3) In assesing a C5 quadriplegic person, which of the following would be considered


an impairment? B
A. Inability to use a balanced forearm orthosis
B. Insensate skin below the elbow
C. Loss of a job
D. Inability to do sport
E. Inability to ambulate

9. (6) The following statement about Rheumatoid Arthritis (RA) is true: B


A. Female : male ratio = 1 : 3 (2:1)
B. Affects the synovial membrane of multiple joints
C. The joints mostly affected are shoulders and hips (hand, PIP, bilateral, MCF,
wrist, elbow, knee, ankle)
D. DIP subluxation is the hand deformity in the late stage of RA
E. Systemic inflammation is not the risk factor for osteoporosis in RA

10. (4) Electromyography studies for back and leg pain may helpful in differentiating all the
following condition EXCEPT: D
A. Neuropathic pain
B. Lumbar plexopathy
C. Lumbar spinal stenosis
D. Metastatic prostate cancer
E. Spinal nerve root dysfunction

11. (5) Compensatory strategies for treating dysphagia could be chin tuck, head rotation,
head tilt, and etc. Chin tuck strategy is: A. /chin down (layanan KFR hal 89)
A. Decreases the space between the base of the tongue and the posterior
pharyngeal wall and increases pharyngeal pressure to move the bolus
through the pharyngeal region
B. Patient voluntarily holds the larynx at its maximal height to lengthen duration of
crycopharyngeal opening  mandelsohn swallow
339
C. Closes ipsilateral pharynx, forces bolus into contralateral pharynx, and decrease
cricopharyngeal pressures  head rotation
D. Concomitant breath holding and swallowing closes the vocal folds to protect the
trachea  supraglottic swallow
E. Uses gravity to guide bolus into ipsilateral pharynx  head tilt

12. (7) Mr. M, 78-year old, suffered from low back pain after falling down 2 month ago.
His both legs are numbness and weak. The physical examination are found kyphosis at
thoracal area and gibbus at lower thoracal, parathorco-lumba tenderness, vertebrae
percussion pain (+). MMT C5-T1 = 5/5, L2-S1 = 3/3, BHR +/+. Deficit sensory at left
and right L5 and S1 area. DTR within normal limits. Sensory and voluntary anal
contractions are preserved. According to ASIA classifications, this man suffered of: D
A. L1 Paraplegia ASIA C, ZPP sensory L2-L4
B. Cauda Equina Synddrome
C. T2 paraplegia ASIA D
D. L1 paraplegia ASIA C
E. L4 paraplegia ASIA C

13. (8) Medical rehabilitation after Total Knee Replacement: B


A. Isotonic quadriceps exercise in the first week after surgery
B. The goal after 2 weeks is 0-90° ROM of the affected knee
C. Early mobilization is not recommended
D. The use of CPM until 6 weeks (6 hari)
E. Gait training after 3 weeks (langsung)

14. (10)Which statement concerning patient with osteoarthritis is TRUE?


A. Exercise contributes to pain reduction
B. Group exercise programs are more effective than individual programs in
reducing disability
C. Reduced aerobic capacity is difficult to reserve
D. Knee orthosis and walking aids are not effective to reduce pain
E. High intensity progressive resistance exercises causes the disease to progress

15. (9) The following is the risk factor of secondary osteoporosis, EXCEPT: C
A. Immobilization
B. Organ transplant
C. Hypoparathyroidsm
D. Proton pump inhibitors
E. Glucocorticoid treatment

16. (28) The following is the changes due to cardiac deconditioning:


A. The cardiac output is increased
B. Increased resting HR by one bpm every 2 days for the first 4 weeks
C. Increased resting systolic BP
D. Insceased resting stroke volume up to 15% after 2 weeks
E. Weakness of finger extensor

17. (35) A male 23 years old suffered traffic accident 3 month ago. He has stabilized
fracture at vertebrae Th XII. The motor impaired at L4 (dorsoflexor ankle) level both
side with the MMT 1/1 and no spasticity. To increase the walking ability, what should
you do?
340
A. HKAFO with pelvic band
B. Strengthening the impaired muscles
C. Body weight-support partial training
D. Functional electrical stimulation
E. Give double AFO

18. (32) Which of the following is NOT the characteristic of radial nerve lesion?
A. Weakness of extensor carpi radialis
B. Ape hand
C. Drop hand
D. Weaknees of finger extensor
E. Weakness of extensor carpi ulnaris

19. (34) Which of the following muscles is the lateral rotator of the hip joint? (deep rotator
muscle : piriformis, gamelus sup & inf, obturator internus & ext, femoris quadratus
A. m. tensor fascia latae
B. m. piriformis
C. m. gluteus minimus
D. m. piriformis
E. m. gluteus medius

20. (33) The muscle that has the important role in push-off is:
A. Iliopsoas
B. Hamstring
C. Quadriceps
D. Triceps surae (gastrocnemius + soleus)
E. Tibialis anterior

21. (11) Spina bifida is considered a primary failure of neurulation and the spinal cord
defect associated with the anatomic level of the lession and neurologic level of
functional involvement. A boy suffered from spina bifida L4-L5 segment, the preserved
muscle function is:
A. Iliopsoas muscles
B. Hamstring muscles
C. Quadriceps muscles  n. femoralis L2-L4
D. Gluteus maximus musce
E. Foot intrinsic muscles

22. (12) Childrens two years of age have stereotypic and ritualistic behavior with
impairment in social interaction, communication and imaginative play. The diagnose is:
(question number 22 and 23)
A. Fragile x syndrome
B. Autism
C. ADHD
D. Mental retardation
E. Down syndrome

23. (13) That above cases have also clinical features:


A. Flat feet
B. May have a lack of eyes contact and difficulty playing with others
C. Executive dysfunction
341
D. Cognitive impairment
E. In attention

24. (14) The following are the characterictic of Wernic-Hoffmann (SMA I) Disease,
EXCEPT: (onset 0-6 bulan) (die 2 y.o)
A. The prognosis is good
B. A muscle biopsy specimen shows findings typical of neurogenic injury
C. Affected in the motor neuron
D. Is an autosomal recessive inheritance disease
E. The infants have a floppy, limp, froglike posture
SMA Werdnig-Hoffman II  intermediate
III  Lugelberg-Welander

25. (27) The increase in strength noted 1 week after beginning progresive resistance
exercises in a spesific group of muscles is most likely due to:
A. Increased muscle protein
B. Increased recruitment of motor units
C. Inreased in muscle oxidative capacity
D. Decreased activity of Golgi tendon organs
E. Conversion of fast to slow twitch fibers

26. (31) The following is the changes due to cardiac deconditioning:


A. Increasing resting HR by one bpm every 2 days for the first 4 weeks
B. Increased cardiac size by 11%
C. Increased resting stroke volume up to 15% after 2 weeks
D. Incresed resting systolic BP
E. The cardiac output is increased

27. (29) Dantrolene, a drug used to treat spasticity,


A. Is a central alpha-adrenergic agonist
B. Decreases the release of calcium from the sarcoplasmic retinaculum
C. Is a GABA analogue worling on the spinal cord
D. Decreases the rate of depolarization in the basal ganglia
E. Stabilizes the membranes of anterior horn cells

28. (30) The increase in strength noted 1 week after beginning progresive resistance
exercises in a spesific group of muscles is most likely due to:
A. Increased muscle protein (hipertrofi
B. Increased recruitment of motor units
C. Inreased in muscle oxidative capacity
D. Decreased activity of Golgi tendon organs
E. Conversion of fast to slow twitch fibers

29. (15) A 56-year old female suffered from obesity (BMI 31) and hypertension (BP
140/85). What the best exercise regimen for this woman?
A. Continue walking exercise while blood pressure exceeds 220/100, because teh
exercise will decreased the blood pressure
B. Arm curl with dumbbell for biceps strengthening, with intensity 80% of IRM, 8
repetitions, 3 bouts/session, 2-3x/week
C. Leg extension exercise with quadriceps bench, with intensity 80 % of 1 RM, 8
repetitions, 3 bouts/session, 2-3x/week
342
D. Fast walking exercise, with intensity 40-70% of heart rate reserve, 30-60
minutes/day, 3-5x/week
E. Static cycle exercise, with intensity 40-70% (moderate) of heart rate
reserve, 30-60 minutes/day, 3-5x/week

30. (16) The following is causes of restrictive pulmonary disease, EXCEPT:


A. Guillian Barre Syndrome
B. Chronic bronchitis  obstruction
C. Myasthenia gravis
D. Pleural effusion
E. Obesity

31. (17) If the body weight 50 kg, what is the absolute oxygen consumption (ml/minute)
equivalent to 2.5 METs?
A. 30.625 ml/min
B. 43.750 ml/min
C. 306.25 ml/min
D. 437.50 ml/min
E. 4375.0 ml/min

32. (18) Which of the following is true regarding falls in the geriatric population?
A. Most falls are considered accidental, rather than related to underlying diseases or
functional impairments
B. The timed “Get up & Go” tetst is an appropriate way to assess someone who
presents with frequent falls
C. Dynamic balance training activities such as Tai Chi have no effect on fall or fear
of falling
D. Fall risk assessment should be evaluate for both intrinsic and extrinsic risk
factors
E. Joint prblem is the most cause of fall in elderly

33. (23) The one of the following is NOT of the prevention programs for Rotator Cuff
tendinopathy:
A. Maintain the dynamic control of shoulder external and internal rotators
B. Suggestion to coach to make proper training periodization
C. Hotpack before and after training
D. Warming up and cooling down
E. Facilitate proper resting period

34. (24) A disc thrower athlete has complained pain in his right shoulder after weight
training using over head barbell. Physical examination found Jobe’s test (empty can
test) negative. Neer test negative, pain full arch positive at 120-150 degree, Scarf test
positive, cross-body adduction manuever positive. His complain related to:
A. Anterior shoulder disloctions
B. Partial tear of tendon Subscapular
C. Acromio-clavicular joint pathology
D. Partial tear of tendon Supraspinatus  jobe (+)
E. Subacromial impingement syndrome  Neer (+)

35. (26) Dantrolene, a drug used to treat spasticity,


A. Is a central alpha-adrenergic agonist (tizonidine, ionidine)
343
B. Decreases the release of calcium from the sarcoplasmic retinaculum
C. Is a GABA analogue worling on the spinal cord (baclofen, dimedula spinalis)
D. Decreases the rate of depolarization in the basal ganglia
E. Stabilizes the membranes of anterior horn cells

36. (25) A field professional tennis athlete during rehabilitation program phase sport
spesific skill. Her previous complain was pain in right medial elbow side. Which
rehabilitation program appropriate for this athlete:
A. Cryotherapy on medial side elbow
B. Ultrasound diathermy at area lateral epicondylus
C. Eccentric strengthening extensor group muscle
D. Isometric strengthening exercise extensor group muscle
E. Active range of motion flexion, extension and supination elbow

37. (19) A 75-year-old sedentary man with a history of stable hypertension, type II diabetes
mellitus,and mild sensory neuropathy who is currently a nonsmoker wishes to begin am
exercise program. What would be the safest initial program?
A. Severe deprassion
B. Walking with no competition
C. Treadmill walking at .5 mph on level surface
D. Lifting 1- to 3-pound weights overhead while seated (jawaban kunci)
E. Machine-based quadriceps extension at 80% maximum weight

38. (20) Which condition is common in elderly may lead to inaccurate assessments when
formal measurement scales are used?
A. Severe depression
B. Cultural and social factors
C. Unwillingness to cooperate
D. Vision and hearing disturbances (jawaban kunci)
E. Dementia causing to not understand

39. (21) The following conditions are the contraindications to exercise for patients with
cancer, EXCEPT:
A. Significant decline in cognitive status
B. Peripheral sensory neuropathy
C. Resting SBP,85 mmHg
D. Disorientation
E. Chest pain

40. (22) Return to sport phase focused to the component:


A. Strengthening all muscles
B. Stretching exercise
C. Sport spesific skill
D. Balance training
E. ROM exercise

41. (41) A 40-year-old woman presents with history of progressive pain over the proximal
medial aspects of tibia. The pain is said to be increased with running up hills. The most
likely diagnosis is:

344
a. Prepatellar bursitis
b. Pattelar tendon synovitis
c. Cartilage damage to the medial aspect of the knee
d. Pes anserinus
e. Tibial-collateral ligament strain

42. (42) After a spinal cord injury, heterotopic ossification most commonly occur the : cucur
605
a. Hips
b. Hands
c. Knee
d. Elbow
e. Shoulder

43. (44) A Jefferson Fracture is :


a. A teardrop fracture  body vertebral, fleksi spinal
b. A burst fracture of CI
c. A type II odontoid fracture
d. A synonym for a Hangman’s fracture
e. A fracture dislocation of C5 or C6 with greater than 50% dislocation and locked facets

44. (43) Sacroilitis always seen in :


a. Juvenille chronic arthropathy
b. Ankylosing spondylitis
c. Psoriatic arthropathy
d. Intestinal arthropathy
e. Reiter’s syndrome

45. (46) The Root syndrome that result from hernia nucleus pulposus between vertebrae
lumbalis 4 and 5 is
a. Root L3
b. Root L4
c. Root L5  posterolateral
d. Root S1

345
e. Root S2

46. (45) Which statement is correct regarding the iliotibial band friction syndrome?
a. Mc Murray test has been found
b. The Ober test has not been found
c. Treatment consist of stretching and massage of the ITB
d. Excessive supination secondary to soft sole on shoe sole is causative
e. The most severe pain is directly over medial joint line of the knee

47. (47) Locked-in syndrome (tetraparesis in a completely conscious patient with only the
ability to move eyes vertically and blink) is suggestive of an occlusion in which
artery(ies)?
a. Posterior inferior cerebellar artery
b. Vertebral and basilar arteries
c. Bilateral basilar arteries
d. Posterior cerebral artery
e. Vertebral artery

48. (48) Which type of aphasia would you expect to see in a patient who is noted to have fluent
speech, with signs of comprehension, but unable to repeat?
a. Broca aphasia
b. Anomic aphasia
c. Conduction aphasia
d. Transcortical sensory aphasia
e. Transcortical motor aphasia

49. (73) The use of USD is indicated for the following condition :
a. Over the stellate ganglia
b. Myositis ossificans
c. Metal implants
d. Thromboplebitis  thromboplhebitis
e. Acute ankle sprain  pulsed USD

50. (74) The following statements regarding cryotherapy, except :

346
a. After 12 to 15 minutes a reflex deep tissue vasodilatation or hunting response occur
b. It is indicated in Raynaud’s phenomenon
c. A minimum of 15 minutes is necessary to achieve extreme analgesic effects
d. Cryotherapy is the use of cold in the treatment of acute trauma
e. Cryotherapy is contraindicated in patients with cold allergies

51. (75) The following conditions is indication of MWD treatment, except :


a. Myofascial trigger point
b. Chronic tenosynovitis
c. Muscle spasm
d. Tendinitis
e. Malignancy

52. (76) A male factory worker has a traumatic on his right carpometacarpal. The orthopaedic
surgeon decide to ampute his right forearm, and ask your opinion regarding management to
obtain the BEST functional level. You suggest :
a. Below elbow amputation in the proximal one-third of the forearm
b. Below elbow amputation 6cm proximal to the radial styloid
c. Wrist disarticulation
d. Skin graft and salvage of the carpals
e. Preservation of styloid processes for better suspension

53. (72) The following joint is NOT categorized as true joint :


a. Intercarpal joint
b. Humero-ulnar joint
c. Gleno-humeral joint
d. Scapula-thoracic joint
e. Calcaneo-cuboid joint

54. (71) An entrapment neuropathy affecting the inferior gluteal nerve will lead to the
following pathologic gait :
a. Limping gait
b. Steppage gait  common peroneal (fibular) neuropathy
c. Quadriceps gait

347
d. Gluteus maximus
e. Gluteus medius gait

55. ( 70) Muscles that, if shortened, predispose a person to slouched sitting posture with
posterior tilted pelvis are :
a. Iliopsoas muscles
b. Hamstring muscles
c. Quadriceps muscles
d. Abdominal muscles
e. Semispinalis muscles

56. (69) Axonal damage (with Wallerian degeneration) would present with :
a. Slowing of conduction velocity across the lesion
b. Slowing of conduction velocity distal to the lesion
c. Decrease CMAP amplitude distally but not proximally
d. Decrease CMAP amplitude with proximal stimulation but not distal stimulation
e. Decrease compound motor action potential (CMAP) amplitude with proximal
stimulation and distal stimulation

57. (53) Children with cleft palate have complex feeding difficulty due to :
a. Inability of lip seal
b. Normal tongue movement
c. Ability to create the intraoral negative pressure
d. Produce rhythmic sucking pattern
e. Nasal regurgitation and aspiration

58. (52) Infantile tibia vara characterized by :


a. Unilateral/bilateral bowing
b. Tender bony protuberance can be palpated at the medial aspect of proximal tibia
c. Bowing at the middle portion of tibia
d. Results of the symmetry of disordered physeal growth of proximal tibia
e. Always needs lateral view of proximal tibia

59. (51) The statement is NOT associated with the role of exercise in obesity children :

348
a. Increasing of active behaviours
b. Protect against chronic disease
c. Engagement in physical activity
d. Reducing the risk of obesity in adulthood
e. Prevention of becoming overweight and obese

60. (50) Atrophy of the first dorsal interosseous muscle may indicate change to spinal roots :
a. C3 and C4
b. C5 and C6
c. C6 and C7
d. C7 and C8
e. C8 and T1

61. (49) Posterior interosseous nerve is a branch of :


a. Femoral nerve
b. Tibial nerve
c. Median nerve
d. Ulnar nerve
e. Radial nerve

62. (65) The following statements are the goals of the acute phase of rehabilitation in sport
injuries, EXCEPT :
a. Retard muscle atrophy
b. Establish neuromuscular control of the joint in the neutral positions
c. Reduce pain and inflammation
d. Re-establish full range of motion around the injury
e. Create conditions for tissue healing

63. (68) The following are the risk and etiological risk factors of pressure ulcers, EXCEPT :
a. Fecal and urinary incontinence
b. Elderly
c. Hypoalbuminemia
d. Pressure, shear, and friction
e. Decrease skin temperature

349
64. (67) The following structure is NOT important for the process of muscle contraction :
a. Calcium ions
b. Mitochondria
c. Actin – myosin
d. Sarcoplasmic reticulum
e. Adenosine diphosphate

65. (66) The following is TRUE regarding stress fracture of metatarsal :


a. More distal fractures tend to occur in the dancer
b. In athlete with recurrent stress fractures, custom-molded orthotics is not indicated
c. The fifth metatarsal is the most commonly affected
d. The treatment include rest and soft-soled shoes are mandatory
e. If symptoms have been present for less than 1 month, a technetium bone scan is
indicated

66. (54) Common characteristics of infants with Erb’s birth pasy include a :
a. Birth weight of less than 2500 g
b. Birth history of a precipitious delivery
c. Flexor synergy pattern of the upper limb
d. Prenatal history of decrease fetal movement
e. “waiter’s tip” posture of the affected extremity

67. (57) For which stable chronic disease would progressive resistance training 2 to 3 times
weekly at an intensity 12 to 16 on the Borg Scale when performing 2 to 3 sets of 8 to 10
repetitions NOT be appropriate?
a. Stroke
b. Osteoporosis
c. Respiratory disease
d. Diabetes Mellitus  F: 3-7 hr/ mng
I : mild moderate RPE 11-13
Time : 150x/ min/ minggu
Type : long muscle
e. Cardiovascular disease

350
68. (56) Patients with CAD may be considered for activity progression when activity response
include :
a. ST displacement over 3mm for rest
b. There is excessive fatigue
c. There is palpitation
d. Systolic blood pressure rise to with 10 – 40 mmHg for rest
e. Heart rate increase to with more than 40 beats above rest

69. (55) A COPD patient who has a measured forced vital capacity of 3.5 L and was able to
expel 3.1 L within 1 second has how many % of FEV1 :
a. FEV1 of 68%
b. FEV1 of 98%
c. FEV1 of 89%
d. A restrictive lung disease
e. An obstructive lung disease

70. (64) The following statements are true regarding myositis ossificans (MO) after sport
injuries, EXCEPT :
a. A previous quadriceps injury is a risk factor of MO
b. Pathologic findings have revealed the most of tissue damage occurs superficial
within the muscles
c. Bone scanning may demonstrate the lesions several weeks prior the plain radiographs
d. By 3 or 6 months, the mass matures into cancellous bone
e. Early treatment to limit swlling and haemorrhage minimizes the incidence of MO

71. (63) A rock climbing athlete during return to sport phase after injured in right A2 pulley
second finger and third finger. To prevent reinjure during on field training, the athlete need
:
a. Wrist support with rigid side on palmar side
b. Rigid tape on second and third fingers palmar side
c. Rigid finger support phalanges second and third fingers
d. Longitudinal elastic tape second and third fingers dorsal side
e. Circular rigid tape on mid and proximal phalanges second and third fingers

351
72. (62) In elderly people, antihypertensive are chosen with attention to concurrent morbidities
and medications’ side effect profiles such below EXCEPT :
a. ACE Inhibitor, thiazide or loop diuretic associated with risk of hyperkalemia, reduction
of renal function, cardiac arrest and ventricular arrhythmias
b. Thiazide diuretics increased risk of osteoporosis  reduce urinary calcium
excretion (kunci)
c. Alpha blockers may cause orthostatic hypotensive
d. Calcium channel blockers may exacerbate constipation and cause lower extremity
edema
e. Contraindications beta-blockers include cardiac conduction disease, reactive airway
disease and depression

73. (60) Which physiologic factor in the elderly exacerbates orthostasis?


a. Decrease baroreceptor response
b. Decrease peripheral resistance
c. Decrease creatinine clearance
d. Decrease in arterial stiffness
e. Decrease heart rate

74. (59) Patients with cancer may experience symptoms and signs that may limit their ability to
exercise such as the following, EXCEPT :
a. Fluctuating periods of sickness
b. Low risk of microbial infection
c. Cachexia
d. Bone metastasis
e. Cancer-related fatigue

75. (58) Children with Tetralogy of Fallot will assume the knee-chest or squatting position to
relieve exercise-indiced dyspnea. This position :
a. Decrease pulmonary artery pressure
b. Inscrease inspiratory capacity
c. Increase vagal tone
d. Reduce the energy requirement of the activity by lowering the center of gravity

352
e. Increases peripheral vascular resistance and thereby decreases right-to-left shunt

76. ( 61) The following statement is NOT TRUE regarding diabetes in elderly persons :
a. Minimizing hypoglicemia in decreasing diabetes-associated morbidity and mortality
caused by nonadherence to a prescribed nutritional, exercise and drug regimen
b. The primary goal of diabetes control is to reduce the severity of comorbid conditions
c. Long-term goals include tight glycemic control to prevent retinopathy or nephropathy
d. For frail older adults, maintenance of HbA1c below 7%, and fasting plasma
glucose below 150mg/dL
e. Several comorbidities include depression, cognitive impairment, urinary incontinence,
falls, and neuropathic pain

77. (80) The general principles of exercise for osteoporosis include the following, EXCEPT :
A. Principle of specificity
B. Principle of progression
C. Principle of reversibility
D. Principle of initial values
E. Principle of overload
78. (79) The indication of manipulation treatment including the following conditions :
A. Rheumatoid arthritis
B. Osteomyelitis
C. Osteoporosis
D. Disciitis
E. Postural Low Back Pain

79. (78) At normal walking speeds of transtibial amputees, energy-storing prosthetic feet such
as the Flex Foot and Sattle Foot :
A. Demonstrate increased energy release during push-off as compared to SACH
feet
B. Primarily absorb energy during heel strike and release energy during pusg-off
C. Reduce metabolic energy consumption as compared to SACH feet
D. Neither stire nor release energy during gait
E. Create the abnormal muscle work characteristics of the hip and knee during
prosthetic stance phase

353
80. ( 77) Which of the following is not an advantage of knee disarticulation amputation as
compared to a transfemoral above-knee amputation?
A. Distal residual limb weight-bearing surface
B. Improved suspension and rotational stability
C. Tehnically less involved surgical procedure
D. Greater selection of prosthetic knee components
E. Improved balance and gait

81. (83) A 25-year-old man at work presents to you with 2 days of knee pain and swelling after
falling of ladder. The swelling began immediately after the fall. His neurologic examination is
normal. He is unable to fully extend or flex the knee because of pain and swelling. He is
ambulating with antalgic gait limp. Your recommendations include :
A. NSAID drugs, x-rays, return to work, recheck in 6 weeks
B. Icing, crutches, x-rays, recheck in 2 weeks
C. Referal to an orthopaedic surgeon
D. Icing, NSAIDS drug, knee immobilization, crutches, activity modification, x-
rays, recheck in 5 days
E. Icing, narcotics, TENS, referral to an orthopaedic surgeon

82. (82) A 60-year-old female pesents with foot pain for the last 3 weeks. She reports severe
pain on the bottom of her foot, which is worse with the first few steps in the morning after
getting out of bed. She has no history of trauma. Her ADL has been limited since this pain
began. What is the most likely diagnosis?
A. Plantar fasciitis
B. Stress fracture
C. Osteoporosis
D. Tarsal tunnel syndrome
E. Morton’s neuroma

83. (81) All of the following statements are correct regarding the coordination exercise,
EXCEPT :
A. One of methods is Frankel exercise  slow repetitious exc to treat ataxia
B. To perform constant repetitive movements  smooth accurate control motor
response
C. To strengthen progressively the particular muscles

354
D. To apply sensory inputs
E. To accelerate the speed is tolerable

84. (87 )When patients with traumatic brain injury ask about returning to driving, you tell
them :
A. Persons with TBI must wait at least 1 year post injury before resuming driving
B. Clinical assessment should include sensorimotor impairment, vision, and
cognitive function
C. Should not drive
D. Clinical assessment should include only motor function
E. Only on-road evaluation is necessary to determine fitness for driving

85. (86) The most common level of lumbar disc herniation occurs at :
A. L1-2
B. L2-3
C. L3-4
D. L4-5
E. L5-S1

86. (85) In which case is a corticosteroid injection contraindicated?


A. A woman with chronic shoulder pain who has had 1 prior injection 6 months ago in
good results
B. A diabetic woman with subacute medial knee pain that is warm to touch
compared to the other leg
C. A woman with acute de Quervain’s tenosynovitis
D. Tennis player with acute elbow pain, 1 week before a tournament
E. A man with plantar fasciitis, will return to his job 2 days after treatment

87. (84) Mr John 60-year-old comes to your clinic complaining of a hot, painful, swollen left
foot. He denies any history of trauma and states that he forgot to take his allopurinol the past
several days. X-rays of his foot may reveal :
A. Pencil in cup deformity  psoriatic arthritis
B. Osteoporosis
C. Bony erosions with an an overhanging edge  punch out lesion
D. Chondrocalcinosis of articular cartilage
355
E. Severe juxta-articular osteopenia  RA

88. (91) An 18-year-old student sustained multiple injuries, including a proximal left radius
fracture, in an automobile accident. He unable to extend the digits of his left hand. You would
expect to find fibrillation potentials in which of the following muscles if radial nerve was
injured near the fracture
A. Anconeus, brachioradialis, extensor digitorum communis
B. Brachioradialis, extensor carpi radialis longus, extensor indicis
C. Anconeus, extensor digitorum communis, extensor indicis, extensor pollicis longus
D. Extensor digitorum communis, extensor indicis, extensor pollicis longus
E. Triceps, supinator, extensor indicis

89. (90) In the treatment of Myasthenia gravis, pyridostigmine (Mestinon) acts to :


A. Impair antibody blocking of post ganglionic receptor sites
B. Decrease the amount of calcium available in the axon terminal
C. Increase the amount of calcium available in the axon terminal
D. Increase the number of acetylcholine molecules in each quantum
E. Inhibit acetylcholine esterase activity

90. (89) In planning a rehabilitation program for a patient with amyotropic lateral sclerosis, you
need to consider that :
A. Cognitive deficits will be a major limiting factor
B. Congestive heart failure is usual cause of death
C. Restrictive lung disease may limit exercise tolerance
D. Abnormalities of swallowing are confined to the oral phase
E. Bladder incontinence may be seen early

91. (88) The patient begin to activate muscles selectively outside the flexor and extensor
synergies is characteristic for Brunnstrom stages of motor recovery in :
A. Stage 1
B. Stage 2
C. Stage 3
D. Stage 4
E, Stage 5

356
92. (95) The best predictor of community ambulation beyond childhood in patients with
myelomeningocele is :
A. Early surgical closure of the myelomeningocele
B. Joint flexibility
C. Body Mass Index
D. Quadriceps strength
E. Bowel and bladder continence

93. (94) A 3-year-old child has a high thoracic spinal cord injury. When he reaches the age 10
years, which complication is the child most likely to have?
A. Deep vein thrombosis  RF: older age
B. Isolated lumbar lordosis
C. Thoracolumbar scoliosis
D, Heterotopic ossification  RF : young age
E.Hip-knee contracture

94. (93) A nonambulatory 15-year-old boy with spinal muscular atropy is requesting a new
power wheelchair after growth spurt. An important feature of the wheelchair prescription will
be :
A. Short room to provide pelvic adequate pelvic support
B. A solid seat with foam padding
C. Back-slanted seat with pommel
D. Seat back below the scapular ridge
E. Extra room to each side to allow for growth

95. (92) In planning a rehabilitation program for children with congenital complete tibial defect
(unilateral), our goal is for them to be :
A. Independent on crutches without prostetic fitting by 12 to 15 months
B. Standing in parapodium by 12 to 15 months
C. Standing with a prosthesis by 12 to 15 months
D. Sitting alone in a corner chair by 12 to 15 months
E. Independent with a manual wheelchair by 12 to 15 months

96.(99) The following is possible indication to resistance training in cardiac patients :


357
A. systolic blood pressure ≥ 160 mmHg
B. Two weeks after Acute MI
C. Severe valvular disease
D. Unstable angina
E. diastolic blood pressure 90 mmHg
97. (98) A 60-year-old men has a long history of exercise-induced angina effectively, treated
with one nitroglycerine tablet. Over the last week, her angina has been occurring with less
physical activity, and his pain has started radiating to his left arm. His pain has completely
relieved with nitroglycerine, but he now requires two tablets, and relief is for a shorter time
than before. This patient most likely has :
A. Acute myocardial infarction
B. Variable-treshold angina
C. Unstable angina
D. Stable angina
E. New-onset congestive heart failure

98. (97) Exercise training programs on a symptom-limited basis for persons with HIV
infections include the following :
A. Resistance training
B. Anaerobic programs
C. Aerobic programs
D. Competition training
E. Circuit weight training

99. (96) The following statement is TRUE about HIV :


A. As CD4 count decline, symptoms gradually decrease
B. Peripheral neurophaties are a rare disorder seen in HIV
C. HIV-related arthritis tends to affect the upper extremities
D. HIV exerts is primary pathological effect by destroying CD4 T lymphocytes and
thereby paralyzing cell-mediated immunity
E. Pulmonary disease in HIV infection is uncommon

100. (100) Inpatients exercise program for patient post CABG, the activity require :
A. 2 METs
B. 3 METs
358
C. 4 METs
D. 5 METs
E. 6 METs

ANSWER A : IF NUMBERS 1,2,3 ARE CORRECT


ANSWER B : IF NUMBERS 1 AND 3 ARE CORRECT
ANSWER C : IF NUMBERS 2 AND 4 ARE CORRECT
ANSWER D : IF ONLY NUMBER 4 IS CORRECT
ANSWER E : IF ALL NUMBERS ARE CORRECT

101. (104) The following is/are NOT appropriate interventions during an acute exacerbation of
hand rheumatoid arthritis
1. Range of motion exercises
2. Stretching
3. Resistance training
4. Icing

102. (103)Rehabilitation management of osteoporosis patients depend on :


1. The capacity for participation in ADLs and safe exercise
2. Accurate determination of the degrees of bone loss
3. The degree of frailty and propensity to fall
4. The risk factors for osteoporosis

103. (102) The following is/are characteristic(s) for the injuries to the menisci of knee joint :
1. Sometimes there is clicking within the knee
2. Mc Murray test is positive
3. A sudden or forceful twisting motion on a planned foot is the most
4. Physical examination include medial or lateral joint line tenderness & effusion

104. (101) Tender point or trigger point injections may be used in fibromyalgia to relieve local
regions of muscle pain. The following agents and method may be used for injections :
1. Dry needling

359
2. 1% lidocaine
3. Saline
4. Botulinium toxin 3 mng  3 bln

105. (110)The most common associated problem(s) in Cerebral Palsy is/are :


1. Strabismus
2. Cortical blindness
3. Mental retardation
4. Flaccidity

106. (109) The main signs and symptoms of acute respiratory distress syndrome (ARDS) :
1. Abnormal heart rhythms
2. Rapid breathing
3. Chest pain
4. Low oxygen level in blood

107. (108) The location of the entrapment ulnar nerve is/are in the :
1. Guyon’s canal
2. Circumflex of midportion humerus  n. radialis
3. Cubital tunnel
4. Arcade of Frohse  nervus radialis / pin syndrome/ Supinator syndrome

108. (107) Factors predicting poor activity daily aoutcome in stroke patient are :
1. Mental changes
2. Visuospatial deficits
3. Incontinence
4. Poor sitting balance

109. (106) Sign(s) and symptom(s) of Weber Syndrome is/are


1. Contralateral hemiplegia
2. Nystagmus vertigo, cerebellar sign, hiccups, diplopia
3. Ipsilateral cranial nerve III palsy

360
4. Ipsilateral Horner’s syndrome  lateral medullary syndrome / Wallenberg

110. (105) Horner Syndrome are consist of :


1. Ptosis
2. Anhydrosis
3. Myosis
4. Enopthalmus

111. (114) Breathing strategy that has benefit for COPD patients :
1. Diaphragmatic breathing combination with purse lip breathing allows the
patient to remain in control of their breathing
2. Segmental breathing to prevent unbalance of thoracal movement
3. Pursed lip breathing that could prevent airway collapse and to reduce
respiratory rate and dyspnea while improving tidal volume and oxygen saturation
4. Glossopharyngeal breathing to help lack of breathing

112. (113) The following statement(s) is/are TRUE regarding exercise prescription for patient
with renal disease :
1. During Periods of rejection, the intensity and duration should be reduced
2. Should be done immediately postdialysis
3. Aerobic exercise, in moderate intensity (40% to < 60% VO2R), RPE 11-13 on
scale of 6-20
4. Resistance exercise 75% to 90% 1 RM, 2-3 days/week

113. (112) The indications of secretion mobilization techniques :


1. Post-thoracic surgery
2. Moderate sputum production in debilated patients
3. Sputum production > 30 ml/day
4. Atelectasis

114. (111) In patients with Duchene muscular dystrophy, decline in vital capacity tends to
coincide with the onset of :
1. Use orthotic for ambulation
2. Limb contractures

361
3. Dysphagia
4. Use wheelchair for ambulation

115. (117) In elderly patients, which condition(s) is/are a contraindication to starting exercise
program?
1. expanding aortic aneurysm
2. malignant or unstable arrhythmias
3. severe dementia or behavioral disturbance
4. sarcopenia

116. (119) Factors that influence rehabilitation after rotator cuff surgery in athlete:
1. Physician approach
2. Surgery approach
3. Size of repair
4. Gender type

117. (118) Rehabilitation goals after an athlete got rotator cuff injury are:
1. Strengthen the rotator cuff muscle and surrounding muscles.
2.Back to functional activity
3. modification in activity
4. Prevention to reinjured

118. (115) The following is/are intervention(s) to reduce risk of fall and injury for geriatric
patients:
1. treat pain
2. provide cardiac pacing
3. initiate proper footwear
4. initiate gait assistive devices

119. (116) The following medication(s) include as an intrinsic risk factors for falls in the older
adult:
1. Thiazides
2. antihistamine
3. antihypertensive
362
4. sedative and hypnotics

120. (122) From the nerve conduction studies we can get an impression of:
1. The location of avb lesion or compression.
2. The continuity of the peripheral nerve.
3. The condition of the myelin sheeth.
4. The central nervous system.

121.(120) “Female Athlete Triad” is syndrome consist of:


1. Disordered eating.
2. Amenorrhea.
3. Bone loss.
4. Gastritis.

122. (121) The difference between type I in contrast to type IIB skeletal muscle fibers is:
1. Type I are predominantly aerobic.
2. Type IIB are predominantly anaerobic.
3. Type I are more active in the gastrocnemius during standing.
4. Type IIB fibers are red because its high hemoglobin content.

123.(124) A way to reduce the amount of force required of the back extensor musle while
lifting is:
1. Reduce the speed of lifting.
2. Reduce the magnitude of the external load.
3. Reduce the length of the external moment arm.
4. Reduce the length of the internal moment arm.

124.(123) The important electrodiagnostic test for carpal tunnel syndrome is:
1. Abnormal F wave latency.
2. Abnormal median nerve distal latency.
3. Normal amplitudes of all segments of the median nerves.
4. Normal NCV of the proximal part of the median nerve.

363
125.(127) The regimen treatment of postmastectomy lymphedema including the following
application(s):
1. Compression garments and bandaging to increase interstitial pressure.
2. Massage and manual lymph drainage to increase interstitial pressure.
3. Pneumatic compression, to foster lymph clearance.
4. Elevation to facilitate drainage.

126. (125) First class lever is/are:


1. The resistance is in the middle, with the axis at one end and the force at the other end.
2. The example of first class lever is head moving on neck.
3. The example of first class is plantar flexor lifting body weight.
4. The axis is located between the force and the resistance.

127. (126) Prime mover muscle that make the scapula moving up is/are:
1. m. infraspinatus
2. m. supraspinatus
3. m. rhomboideus
4. m. trapezius

128. (129) The dosage of laser depend on:


1. The beam surface area of the laser in cm2
2. The time of exposure in seconds.
3. Output of the laser in mW.
4. Output of the laser in Watt.

129. (128) The mechanical benefit of intermittent traction to the structures surrounding
vertebrae is:
1. Relieve pressure on nerve roots in the intervertebral foramina.
2. Decrease the concentration of noxious chemical irritants.
3. Relieve pressure on the dura and blodd vessel.
4. Reduce circulatory congestion.

130. (132)Muscles that need strengthening in preparation for crutch walking:

364
1. Latissimus dorsi.
2. Pectoralis major.
3. Hip extensors.
4. Triceps.

131. (130) In below knee amputees, excess knee flexion on the prosthetic side during heel
strike may be caused by:
1. Heel cushion too stiff.
2. Foot set with insufficient plantar flexion.
3. Socket set too far anterior over foot.
4. Insufficient flexion provided within the socket.

132. (131) Above knee suction socket generally is NOT prescribed for an older patient for the
following reason(s):
1. It prevents desired rotation of the hip.
2. It is more difficult and energy consuming to put on.
3. It is heavier than an above knee prosthesis with hip joint and pelvic band.
4. It requires more training of stump musculature to maintain the suction.

133. (134) The principles of stress testing include:


1. An initial workload that is low in terms of the individual’s anticipated aerobic
threshold.
2. Changing the workload by increasing the speed and/or grade or resistance.
3. Maintenance of each workload for 1 minute or longer.
4. Termination of the test at the onset of symptoms.

134. (133) The goal of neurodevelopmental treatment of Bobath to Cerebral Palsy is/are:
1. To normalize tone.
2. To activate postural response.
3. To inhibit primitive reflexes.
4. To improve motor coordination.

135. (137) Indications of using orthotics in the treatment of Rheumatoid Arthritis is/are:
1. Decrease pain and inflamation.
365
2. Reduce weight through joint.
3. Decrease joint motion.
4. Stabilization.

136. (135) The specific recommendations for balance training for elderly people is/are:
1. tandem-walk and circle turns.
2. one-legged stand.
3. heels stands.
4. tai-chi

137. (138) A 60 year old woman is reffered by her internist for evaluation of her gait
abnormality and treatment program recommendation. Observation of her gait reveals an
excessive lateral trunk shift to the right during right single leg support. Your differential
diagnosis of this problem should include a;; the following:
1. Short right leg.
2. Painful right hip.
3. Weak right hip abductor.
4. Weak right knee extensor.

138. (139) The Miller Fisher syndrome is a relatively benign variant of acute inflammatory
demyelinating polyneuropathy (AIDP) cases. It is characterized by:
1. Ataxia.
2. Dysphagia.
3. Opthalmoplegia.
4. Hyperreflexia.

139.(136) Yellow flags of low back pain is/are as follow(s):


1. Lose of anal sphincter tone.
2. Pyschological burden.
3. Weight loss.
4. High BMI.

140. (142) Club foot consist of associated deformity:


1. Forefoot varus
366
2. Forefoot valgus
3. Equinus deformity at the ankle
4. Valgus or eversion deformity of the heel

141.(140) The following is/are trigger factor(s) os spasticity:


1. Urinary tract infection.
2. Ulcus decubitus.
3. Constipation.
4. Contracture.

142.(143) The typical gait abnormalities of spastic diplegia CP:


1. Scissoring gait pattern.
2. Flexion upper limb posturing.
3. Knees flexed with valgus deformities.
4. Knees extended with ankle plantar flexion.

143. (144) You are treating a 30-year-old quadriplegic who developed swelling and erythema in
the right lower extremity. Noninvasive venous studies are consistent with a deep venous
thrombosis extending to the thigh. Appropiate treatment would include therapeutic
anticoagulation with intravenous heparin and:
1. Intermittent pneumatic compression devices.
2. ROM exercises to prevent contracture.
3. Warm soaks to the affected leg.
4. Bed rest.

144. (141) Children with major burn injuries should be placed in positions that tend to prevent
contractures. These include:
1. No head pillow.
2. Feet at neutral dorsiflexion.
3. Shoulder at 90 degree abduction.
4. Elbows, wrist, finger interphalangeal joints extention.

145. (147) The reasons for drug problems in older adults is/are:
1. Polypharmacy
367
2. Altered receptor sensitivity.
3. Medications not taken as prescribed.
4. Increased susceptibility to adverse reactions.

146. (149) The following condition(s) is/are the element factor in overload injuries:
1. Sleep pattern.
2. Recent equipment changes.
3. Current strength and flexibility programs.
4. Other sports with overlapping schedules.

147. (145) Exercise stress testing may be expected to disclose in what condition(s)?
1. Myocardial ischemia.
2. Left ventricular dysfunction.
3. Ventricular ectopic activity.
4. Chronotropic incompetence.

148. ( 148) Clinically frail older adult, characterized by the presence of the following
condition(s):
1. Weakness.
2. Weight loss.
3. Poor endurance.
4. Low physical activity.

149. (150) The following statement(s) is/are TRUE regarding Iliotibial band syndrome:
1. Caused by excessive friction between the iliotibial band and the lateral femoral
epicondylar eminence.
2. It occurs most often in long-distance runners, cyclist, repetitive knee flexion.
3. Pain can be produced with single limb stance knee-bend test.
4.. Rehabilitaion emphasizing strengthening of iliotibial band. (stretching ITB)

150. (146) The following statement(s) is TRUE regarding exercise prescription for patient with
pulmonary disease:
1. The intensity also be based on dyspnea ratings determined from graded exercise testing
with rating between 3 to 5 on scale 0-10.
368
2. Training of the inspiratory muscles will reduces dyspnea and improve exercise
tolerance.
3. Resistance training of skeletal muscle should be an integral part of exercise
prescription.
4. Supplement O2 is indicated for patients with SaO2 96%. (92% / 88%)

INDONESIAN COLLEGE OF PHYSICAL MEDICINE AND REHABILITATION JULY, 2017

CHOOSE THE MOST APPROPIATE ANSWER

1. What is the most reliable physical signs of Myofascial Trigger Point Syndrome? A
A. A rope-like nodular in the muscle and pattern of reffered pain
B. Focal tenderness and pain recognition
C. Vasomotor and temperature changes
D. Limited of ROM
E. Muscle spasme
2. A 25-year-old man comes to your office for evaluation of low back pain. As part of the physical
examination, you mark point at the L5 vertebral body and another point midline 10 cm above. You ask
him to flex forward maximally while keeping his knees extended and measure the distance between the
two points, this distance is 13.5 cm. You suspect he may have what diagnosis? C
A. Scheurmann disease
B. Lumbar herniated disc
C. Ankylosing spondylitis
D. Spondylitis tuberculosis
E. Lumbar spondylolisthesis
3. The Boutinierre deformity in a Rheumatoid hand is typically located in the following joint A
A. PIPs
B. MCPs
C. Radiocarpal
D. Ulnarcarpal
E. Carpometacarpals
4. Para Articular Heterotropic Ossification in SCI patient most commonly occurs at the following joint: A
A. Hip
B. Knee

369
C. Ankle
D. Elbow
E. Shoulder
5. A common source of shoulder pain inshoulder impingement syndrome is: E
A. Bursitis
B. Osteoarthritis
C. Short head of biceps
D. Long head of biceps
E. The rotator cuff tendon
6. A 72-year old man presented loss of muscle bulk on volar aspect of forearms and impaired flexion of the
distal interphalangeal joint of the finger. Which muscle is responsible for these symptoms? B
A. Flexor digitorum superficialis
B. Flexor digitorum profundus
C. Flexor pollicis longus
D. Flexor carpi radialis
E. Flexor carpi ulnaris
7. A 55-year old man has severe pain on gentle touching of the arm. Six months ago the median nerve was
damaged during creation of arteriovenous fistula for dialysis. Which of the following terms best describes
this phenomenon? A
A. Allodynia
B. Hyperpathia
C. Hyperalgesia
D. Hyperesthesia
E. Hypersensitivity
8. This clinical syndromes that affect the dorsal collum medial lemniscus system causing proprioceptive
disturbance that affect position sense and balance is caused by: B
A. Syringomyeli
B. Tabes doralis
C. Central cord syndrome
D. Anterior cord syndromes
E. Brown-sequard syndrome
9. Ms.S,21 year-old referred to the rehabilitation ward with paraplegia due to T5 burst fraktur. The PPSW
stabilization was done 2 weeks ago and now was she medically stable. On physical examination you
found that the vital signs and general health status within normal limit. What should you examine when
you have to determine wether the patient has a complete or incomplete lesion? A
A. Anal sensation and voluntary anal sphincter contraction
B. Anocutaneal and bulbocavernous reflexes
C. Myotome and dermatome
D. Physiological reflexes
E. Pathological reflexes

370
10. A male 50 years old have a clinical features: lumbars, buttock, perianal discomfort, and lower limb
weakness. Bowel and bladder abnormalities, sexual dysfunction, saddle anesthesia including the back of
the legs, buttocks, and soles of the feet. The diagnosis is A
A. Cauda Equina Syndrome
B. Spondylolisthesis L5-S1
C. Pyriformis syndrome
D. Spondylolysis
E. Myelopathy
11. A case is caused by growth retardation of the medical and posterior part of the proximal tibial epiphysis
due to abnormal weight bearing stress and compression force on the knee. This disease is known as: A
A. Blount’s disease
B. Ricket’s disease
C. Internal tibial torsin
D. Chondromalacea patellae
E. Ollier’s multiple enchondromatosis
12. The typical gait abnormality find is Spastic Diplegia CP: D
A. Weak hip flextion and ankle dorsiflextion
B. Supinated foot in stance phase
C. Overactive posterior tibialis
D. Scissoring gait pattern
E. Crouched gait
13. One of your 4 year-old patients exhibits the following characteristic: distress over minor changes in
environment, echolalia, lack of awareness of the existence of feelings in others, nonparticipation in simple
games. The most likely diagnosis is: A
A. Autism
B. Celebral palsy
C. Hearing Impairment
D. Mental Retardation
E. Attention deficit hyperactivity disorder
14. Physical examination findings that are consisten with hypotonia in a term infant include all of the
following, EXCEPT A
A. Atrophy
B. Weak suck
C. Frog leg position of the legs
D. Decreased spontaneous movements
E. Head lag when pulled to sitting position
15. During exercise, cardiac output is augmented by C
A. Sympathetic stimulation of resistance vessels
B. Decreased mean systemic arterial pressure
C. Increased ventricular contractility
D. Decreased end-diastolic volume
371
E. Decreased end-systolic volume
16. The activity of climbing up stairs requires the following amount of METs: E
A. 2 METs
B. 3 METs
C. 4 METs
D. 5 METs
E. 6 METs
17. For patient with chronic heart failure, which following statement is accurate? B
A. Most of the important resulting from regular exercise is within the myocardium
B. Exercise capacity is improved because of peripheral adaptation
C. These patient can never expect improved physical fitness
D. High intensity exercises is prescribed to these patient
E. Complete bed rest is prescribed for these patients
18. Cardiac rehabilitation program encounter a variety of heart including in below, EXCEPT: B
A. Stable angina
B. 3rd degree AV Block
C. Resting heart rate 90
D. Oxygen saturation 95%
E. Resting diastolic blood pressure 90 mmHg
19. The most important change in the aging musculoskeletal system is C
A. Increase in bone density
B. Decrease in lumbar lordosis
C. Increase in thoracic kyphosis
D. Increase in threshold for pain
E. Increase in type II muscle fiber
20. A good initial treatment plant for neurogenic claudication secondary to spinal stenosis would include: D
A. Narcotic pain medication
B. Lumbar bracing
C. Acetaminophen
D. Gabapentin
E. NSAID
21. A 70 year-old woman complains the acute localized mid back pain. She has non-focal neurologic
examination. An anteroposterior and lateral thoracic spin x-ray confirm your clinical suspicion of an acute
T8 compression fracture. Which recommendation would best help her to reduce her risk of future
fracture? B
A. A weight redaction diet
B. Avoidance of tobacco use
C. Use a lumboscaral orthoses
D. Swimming lap 20-30 minute daily
E. Isotonic abdominal strengthening program

372
22. A 80 year-old man present to your office with axial back pain that worse with standing and walking. At
times he also experiences pain radiating from the lumbar spine into the legs when walking. What the most
likely diagnosis?E
A. Sacroilitis
B. Ischias syndrome
C. Spondylosis syndrome
D. Spondylosis lumbalis
E. Lumbal canalis stenosis
23. Detrusor overactivity from damage to upper motor neuron, impaired function and cognition is effect of
continence in C
A. Stroke
B. Delirium
C. Dementia
D. Multiple sclerosis
E. Parkinson’s disease
24. Injury to the rotator cuff muscles and tendons of the shoulder is often occurs in? A
A. Tennis player
B. Bowling player
C. Discus thrower
D. Boxing athletes
E. Rowing athletes
25. A softball player suffered from shoulder pain on the right side. The pain increased when he made
repetitive rotation on his right shoulder. No radiating pain, no tingling sensation. Shoulder ROM within
normal limit. Neer and Hawkin-Kennedy test were positive. Drop arm, Yergason’s and apprehention test
were negative. What is possible diagnosis? A
A. Rotator cuff impigment syndrome
B. Shoulder subluxation
C. Supraspinates tear
D. Bicipital tendinitis
E. Frozen shoulder
26. What should be the first treatment considered for chronic lateral epicondylitis? E
A. Wrist orthoses
B. Surgical release
C. Botulinium toxin
D. Corticosteroid injection
E. Wrist extensor stretching
27. The general treatment for low back pain in athletes in maintenance phase (functional adaptation) include
the following, EXCEPT: E
A. Core stabilization
B. Sacral mobilization
C. McKenzie program
373
D. Stretching piriformis
E. Muscle relaxation medication
28. Common shoulder injuries in the decelaration phase of pitching is: C
A. Bicipital tears
B. Anterior instability
C. Posterior instability
D. Internal impingement
E. Anterior surface rotator cuff tears
29. The primary afferent fibers coming from the nuclear chain of the muscle spindle have the following
conduction velocity: C
A. 10-20 m/sec
B. 20-40 m/sec
C. 40-60 m/sec
D. 60-80 m/sec
E. 80-100 m/sec
30. The most common clinical application of the H-reflex is to test: C
A. Canalis stenosis
B. Polyneuropathy
C. SI radiculopathy
D. Cervical radiculopathy
E. Guillain Barre Syndrome
31. In neuropraxia, the following finding can be detected during the EMG/NCV studies: E
A. Fibrillation potentials
B. Positive sharp waves
C. Fasciculation waves
D. All the NCV studies are “no response”
E. Conduction block at certain level of nerve
32. The following is hip abductor muscle EXCEPT: D
A. Gluteus minimus
B. Tensor fascia lata
C. Gluteus medius
D. Psoas minor
E. Sartorius
33. What the role of the gluteus maximus in providing spine stability? D
A. It controls stance phase
B. It primarily function as a hip flexor
C. It control repetitive lumbar rotation
D. It control acceleration and deceleration
E. It seves as a primary muscle to facilitate lumbar flexion
34. The following muscle is classified as monoarticular type: A
A. Soleus
374
B. Gastrocnemius
C. Flexor carpi radialis
D. Flexor pollicis longus
E. Flexor digitorum superficialis
35. Which one of the following CPM (Continuous Passive Motion) is contraindicated? E
A. Burn patient
B. Capsulotomies
C. Frozen shoulder
D. Knee arthroplasty
E. Unstable fractures
36. The following statements are the potential adverse effects of corticosteroid injection, EXCEPT: B
A. Glucocorticoid-induced osteoporosis
B. Subcutaneous hypertrophy
C. Iatrogenic joint infection
D. Tendon/ligament rupture
E. Muscle atrophy
37. The minimum current amplitude, with a long pulse duration, required to produce an action potential is: A
A. Rheobase
B. Chronaxie
C. Propagation
D. Depolarization
E. Relative refractory period
38. In PTB socket the patient’s stump is held in how many degree knee flexion? A
A. 15 degree
B. 20 degree
C. 25 degree
D. 30 degree
E. 35 degree
39. In a hemiplegic, the function of a posterior stop in a double upright AFO with dual action adjustable ankle
joint and extended steel shank is to: C
A. Improve knee stability at heel strike when the posterior stop is set in 5 degrees op dorsiflexor rather
than plantar flexion
B. Cause the ground reaction force line to fall posterior to the knee joint in mid stance
C. Assist in control of moderate to severe plantar flexor spasticity
D. Assist weak plantar flexor
E. Assist weak dorsi flexor
40. The following terminal device of an upper extremity prosthesis has the best function D
A. Cosmetic hand with movable thumb
B. Myoelec tric hand
C. Cosmetic glove
D. Dorrance hook
375
E. Cosmetic hand
41. A 60 year-old male has a right hemispheric infarction. He has muscle strength with grade 3 at the
shoulder, elbow and hand. He also has mildly decrease response to pinprick and proprioception
throughout the left side. He use the right hand for feeding but always leaves some food untouched on the
left side on his plate. The most likely reason for this patient’s difficulty in eating is: D
A. Hemianopsia
B. Motor apraxia
C. Feeding apraxia
D. Left unilateral neglect
E. Left hemisensory dificits
42. Goal for children with developing language in syntax area is: C
A. Increase production of closed syllables
B. Increased consonant repertoire
C. Increased sentence length
D. Increased vocabulary size
E. Increased intelligibility
43. Therapeutic approaches to the upper limb in stroke which proposed a technique that incorporates
cutaneous stimuli to facilitate movement is: C
A. PNF
B. Brunnstrorm
C. Rood method
D. Phelps method
E. Bobath method
44. A 38 year-old with rheumatoid arthritis complains of neck pain with occipital headaches. MRI of the
cervical spine is obtained which demonstrates atlantoaxial subluxation. Your treatment at this time before
you recommend surgical treatment includes: C
A. TENS
B. Deep heating
C. Use Thomas collar
D. Use of a Hallo brace
E. Use Philadelphia collar
45. Rehabilitation guidelines following total knee arthroplasty include: C
A. Non-weight- bearing status for 1 week following surgery
B. Partial-weight-bearing status for 3 month following surgery
C. Patient are usually allowed to resume driving 6 weeks after surgery
D. Use of a continuous passive motion device to improve passive knee extension
E. Joint immobilization until sutures are removed and anterior incision is healed
46. The risk of foot ulceration in diabetic patients is increasing by all of the following, EXCEPT B
A. Metatarsal subluxation and fat pad migration
B. Increase mobility of the subtalar joint
C. A history of previous foot callus
376
D. The loss of vibration sensation
47. Provocative Test for AC (acromioclavicular) joint impingement is: A
A. Cross-chest Test
B. Relocation Test
C. Apprehension Test
D. Anterior Drawer Test
E. Anterior Load and Shift Test
48. The patient lies supine with the examiner holding the knee in 15 degrees of flexion. The femur is
stabilized with one hand while pressure is applied to the posterior aspect of the proximal tibia in an
attempt to translate it anteriorly. A positive test occurs when excessive anterior translation of tibia occurs
with a soft endpoint. This test named: A
A. Lachman test
B. Mc Murray test
C. Posterior drawer test
D. Pattelofemoral grind test
E. Valgus and varus stress test
49. The following factors associated with a poor prognosis in multiple sclerosis, EXCEPT: A
A. Female
B. Progressive course at onset
C. Cerebellar involvement at onset
D. Age at onset greater than 40 years
E. Multiple system involvement at onset
50. A 35-year-old woman present to the Emergency Department reporting a few days of progressive
ascending muscle weakness. She had a viral infection a few weeks earlier. On examination, you find
diffuse weakness and areflexia. The most likely finding in the CSF is: B
A. High protein-high cell count
B. High protein-low cell count
C. Low protein-high cell count
D. Low protein-low cell count
E. Normal CSF
51. You are asked to evaluate a 33 year-old construction worker who is complaining of paresthesias in the
first and second digits of his right hand. Your physical examination shows no weakness but a mild
decreased in light touch over the thumb. You request a nerve conduction study to rule out carpal tunnel
syndrome and it turns out to be normal. On repeated history, the patient indicated that on occasion, he
gets a sharp, “electric”pain travelling from his neck to the right hand. What are you missing? E
A. A neuromuscular junction disordet affecting distal hand muscles
B. A lower trunk brachial plexopathy
C. A median neuropathy at the wrist
D. A C8-T1 radiculopathy
E. A C6-7 radiculopathy

377
52. A woman riding a mountain bike on a rough trail hits a rut and is thrown from the bike. Her upper arm
hits a tree, fracturing the humerus just above the insertion of the teres major muscle. Later, during
examination, it is noted that she cannot extend her forearm at the elbow or hand at the wrist. Which nerve
must have been injured? D
A. Musculocutaneous
B. Axillary
C. Median
D. Radial
E. Ulnar
53. Patient with pronator teres syndrome will have the following clinical signs, EXCEPT: B
A. Intrinsic muscle atrophy
B. Pain in the lateral dorsal forearm
C. Normal strength of the pronator teres muscle
D. Weakeness of the median-innervated hand intrinsic
E. Decrease sensation in the first three digits and over the thenar area
54. Pediatric limb deficiencies with missing hand and foot is called C
A. Amelia
B. Adactily
C. Acheiria
D. Aphalangia
E. Phocomelia
55. This reflex is best elicited by a sudden dropping of the baby’s head in relation to its trunk E
A. Vertical Suspension
B. Righting Reflex
C. Extensor thrust
D. Landau Reflex
E. Moro Reflex
56. Management of pes planus grade 1 at age less than 1 years old is done by: A
A. Observation
B. Thomas heel
C. Arches support
D. Chukka type shoes
E. Ankle foot orthoses
57. A 6 year-old boy with hemophilia has suffered a recurrent hemartrosis of the knee. After providing
adequate factor VIII replacement, the most important therapy in the first 48 hours is: A
A. Immobilization
B. Active assistive OM exercise
C. Icing and passive ROM exercise
D. Ambulation with full weight bearing
E. Ambulation with partial weight bearing
58. Ventricular tachycardia is marked by D
378
A. One PVC on every other beat repeatedly
B. Two PVCs on every other beat repeatedly
C. Two premature ventricular contractions(PVCs) in row
D. Three PVCs (or more) PVCs in a row without any normal beats
E. The rate about 150-500 beats/minute and the absence of a distinct QRS comple
59. What is the equivalent to 8.75 ml/kg/min? D
A. 1.0 MET
B. 1.5 METs
C. 2.0 METs
D. 2.5 METs
E. 3.0 METs
60. Sign of improvement from pulmonary rehabilitation program A
A. Decrease of dyspnea and increase in exercise tolerance
B. Increase the oxygen consumption of an activity
C. Increase the energy expenditure of an activity
D. Increase pulmonary function test
E. Increase work of breathing
61. The respiratory function of patient with Ankylosing Spondylitis is characterized by: A
A. Diminised chest expantion
B. Increase airways obstruction
C. Moderate dyspnea on exertion
D. Increase use of accessory muscle
E. Decrease in diaphragmatic excurtion
62. The following is the change in special senses with aging, EXCEPT: E
A. Lens protein fibers undergo oxidative damage
B. Loss of higher frequency sound
C. Los of salt perception
D. Visual cortical changes
E. Hyperosmia
63. Which the physiologic factor in the elderly exacerbates orthostasis? D
A. Decrease in arterial stiffness
B. Decrease creatinine clearance
C. Decrease peripheral resistance
D. Decrease baroreceptor response
E. Decrease lower limb muscle strength
64. In prescribing treatment for elderly patient, it is best to use which approach? B
A. Start with NSAID
B. Start low and go slow for all medication
C. Start with aspirin, which is safe and effective
D. Avoid use of acetaminophen, due to licer toxicity
E. Use high-dose, short acting narcotics initially to get the pain under control
379
65. In aging people, which lung function increases: D
A. Forced expiratory volume in 1 second
B. Forced vital capacity
C. Total lung capacity
D. Residual volume
E. Vital capacity
66. This disorder is usually seen in preadolescent athletes who participate in activities such as jumping or
running. The disorder is a result of recurring microtrauma from the quadriceps contracting: E
A. Pes anserinus
B. Ligament sprain
C. Pes anserinus bursitis
D. Tibial plateau fracture
E. Osgood-schatter disease
67. Ankle sprains are ubiquitous in sports and are the most common athletic injury. The most commonly
injury for this disorder is the: C
A. Achilles tendon
B. Calcaneo fibular ligament
C. Anterior talofibular ligament
D. The posterior talofibular ligament
E. The strong medial deltoid ligament
68. Which of the following activities would be the most difficult to return to following an anterior cruciate
ligament (ACL) injury? C
A. Swimming
B. Speed cycling
C. Baseball pitching
D. Marathon running
E. Cross-country skiing
69. A discus thrower complains of pain in the upper part of shoulder. One of the finding on examination is
cross adduction chest positive sign, pain arch at 110o . what structure most likely to be involved? E
A. Scapulocostal
B. Suprahumeral joint
C. Glenohumeral joint
D. Sternoclavicular joint
E. Acromioclavicular joint
70. The presence of nascent potentials in EMG examination shows the following process of: C
A. Neuropraxia
B. Axonotmesis
C. Reinnervation
D. Total denervation
E. Partial denervation
71. The smallest unit of muscle contraction is: D
380
A. Motor unit
B. Sarcomere
C. Sarcolemma
D. Actin, myosin
E. Intrafusal fibers
72. A pulmonary embolism which may occur in a prolonged immobilization patient mostly due to: E
A. Development of atherosclerosis
B. Development of atrial fibrillation
C. Decreasing the total lung capacity
D. Decreasing of the lung vital capacity
E. Development of deep venous thrombosis
73. Atlanto-odontoid is classified as C
A. Gynglimus joint
B. Ellipsoida joint
C. Trochoid joint
D. Saddle joint
E. False joint
74. In the normal gait pattern, the ankle is held in a neutral position at C
A. Toe off
B. Foot flat
C. Heel strike
D. Mid stance
E. Mid swing
75. When a disk protrudes, it does so in the posterolaterally direction, because C
A. The anulus is thicker posteriorly
B. The longitudinal ligament support the spine in bending position
C. The posterolateral aspect is not covered by the longitudinal ligaments
D. The forces of weight bearing in upright position are more on the posterior side of the vertebral body
E. The forces of weight bearing in bending position are more on the posterior side of the vertebral body
76. The following is NOT used for lymphedema: B
A. Compression garment
B. Superficial heating
C. Elevation
D. Stroking
E. Massage
77. Which finding is a relative contraindication to cryotherapy: D
A. Pain
B. Edema
C. Acute hematoma
D. Impaired sensation
E. Acute inflammation
381
78. The following is NOT contraindication of pelvic traction: E
A. Discitis
B. Acute sprain
C. Osteoporosis
D. Join instability
E. Grade 1 spondylolisthesis
79. A patient with DM has painful, swollen ankle, worst with weight bearing, X-ray shows fragmentation of
the articular surface, synovial fluid is clear. The choice of orthosis is: D
A. AFO
B. KAFO
C. HKAFO
D. PTB orthosis
E. Orthopedic shoes
80. The best rational choice of wheelchair for C8 Tetraplegia ASIA A is: E
A. One-arm drive wheelchair
B. Fully-reclining wheelchair
C. High performance wheelchair
D. Mouth-operated electrical wheelchair
E. Regular wheelchair with vertical bars on its handrims
81. The common “knuckle-bender orthosis”is used to: E
A. Stretch flexion contracture at DIP joint
B. Stretch flexion contracture at PIP joint
C. Stretch flexion contracture at MCP joint
D. Stretch extension contracture at the DIP joint
E. Stretch extension contracture at the MCP joint
82. Parkinson disease patients may improve in walking by the following exercise: E
A. Voyta
B. Knott
C. Kabat
D. Bobath
E. Frenkel
83. The following treatment encourage to decrease edema, EXCEPT: A
A. Kneading
B. Elevation
C. Elastic compression
D. Intermittent pressure device
E. Rhythmic muscle contraction
84. The following is NOT including in rehabilitation program for SLE patient: C
A. Orthosis and adaptive equipment
B. Reeducation in self-care skills
C. High intensity strength exercise
382
D. Antispasticity agents
E. Endurance exercise
85. The most common cause of significant knee hemarthrosis after traumatic injury is: A
A. Anterior cruiciate ligament rupture
B. Lateral collateral ligament tear
C. Osteochondral fracture
D. Patellar dislocation
E. Meniscal tear
86. A 25 year-old man has had heel pain for 1 month. He runs approximately 40 kilometers weekly. The pain
is most severe when he first stand up and walks after sitting or lying down. Palpation of the medial
plantar surface of the heel increases the pain. He walks with a limp. Which of the following is the most
likely cause of the pain? D
A. Heel spur
B. Achilles tendinosis
C. Impingement of the posterior sacral nerve root
D. Inflammation of the origin of the plantar fascia
E. Articular cartilage degeneration of the calcaneo cuboid joint
87. A 24-year-old marathon runner reports lateral knee pain after hill training. Examination reveals no
effusion; and result of Ober’s test are positive. What is the most likely diagnosis? E
A. Biceps tendinitis
B. Lateral meniscal tear
C. Popliteus tenosynovitis
D. Peroneal nerve entrapment
E. Illiotibia band friction syndrome
88. American college of rheumatology classification criteria for systemic lupus erythematosus include all of
the following EXCEPT: D
A. Arthritis
B. Malar rash
C. Photosensitivity
D. Raynaud’s phenomeon
E. Persistent proteinuria>500 mg/day
89. Tic doloreaux is caused by pathology of the following cranial nerve: D
A. II
B. III
C. IV
D. V
E. VI
90. The mechanism of diazepam to reduce the muscle spasticity is by: A
A. Stimulating the GABA-mediated inhibition in CNS
B. Inhibiting glutamate and aspartate in CNS
C. Inhibiting the extrafusal fiber
383
D. Inhibiting the pyramidal tract
E. Inhibiting motor cortex
91. The cause of Myasthenia gravis is: E
A. Nerve cells are destroyed
B. Lesion in the muscle fibers
C. Lesion in the cerebral cortex
D. Lesion in the anterior horn of the spinal cord
E. Acetycholine receptor in muscle cells is destroyed
92. A 50-year-old man with a stroke and spastic hemiparesis. The occupational therapist asks your opinion on
the use of botulinum toxin versus phenol injection to decrease finger flexion tightness. An advantage of
botulinum toxin is: B
A. Lower cost
B. Lack of dysesthesias
C. Shorter onset of action
D. Longer duration of efficacy
E. Direct effect on the myelin surrounding the nerve
93. Children with tetralogy of Fallot with assume the squatting position to relieve exercise induced dyspnea D
A. Decrease heart rate
B. Increase inspiratory capacity
C. Decrease pulmonary artery pressure
D. Increase peripheral vascular resistance and there by decreases right to left shunt
E. Reduces the energy requirement of the activity by lowering the center of gravity
94. A child with a congential transverse radial limb deficiency should have in initial prosthesis fit at what
developmental stage? A
A. At the time of first sitting independently
B. At the time of starting kinder garden
C. At soon as possible after birth
D. At the time of initially walking
E. At the time of initially standing
95. A 7-year-old girl is referred with an acute flare of her polyarthritis which has resulted in severe joint pains
despite appropriate medications. You are consulted for a physical medicine program. On examination she
is cooperative child but in obvious pain from swollen. Warm knees, ankles, wrist, and digits, with
limitation in range of motion at all of these joints. Your prescription should include: C
A. SWD for the knees
B. Strengthen the grip in both hand
C. Paraffin baths for the hands, wrist, and ankles
D. Isotonic strengthening for the quadriceps and hamstrings
E. Ultrasound applied in water over the ankles, wrists, and the knees
96. What type of Cerebral Palsy (CP) with unsteady gait, dysmetria, intention tremor in upper extremities and
truncal titubation? A
A. Ataxia type
384
B. Mixed type
C. Spastic type
D. Athetoid type
E. Hypotonic type
97. The following condition is NOT a contraindication for exercise program prescribed to a diabetic patient
with cardiovascular disease A
A. Blood sugar : 180-250 mg%
B. Blood sugar: >300 mg%
C. Blood sugar: <80 mg%
D. Acidosis
E. Ketosis
98. For chronic lung disease various walking test with sub maximal exercise have all the benefit below,
EXCEPT:B
A. To detect exercise intolerance
B. To detect limitation in lung function
C. To know and detect functional limitation
D. To evaluate reconditioning exercise programme
E. To prescribe the intensity of exercise programme
99. The following exercise prescription for diabetic is true, EXCEPT: B
A. Avoid exercise at peak insulin times
B. Advice that insulin be injected in muscle to be exercise
C. Encourage adequate hydration to avoid effects of fluid shifts on blood sugar levels
D. Caution patient that blood sugar may continue to drop for 24-48 hours after exercise
E. If blood sugar value is <100 mg/dl, delay exercise, provide patients 15 g of carbohydrate, retest in
100 minutes
100.Regarding exercise in heart failure, which of the following is true? E
1. Telemetry is never necessary
2. Strengthening should be done isometrically
3. Duration of exercise should be prolonged until fatigue
4. Exercise heart rate should be at 80-90% of estimate maximum heart rate
5. The exercise heart rate be kept at least 10 beats per minute below the arrhythmia/severe
dyspnea level

ANSWER: A. IF NUMBER 1,2,3 ARE CORRECT


B. IF NUMBER 1,3 ARE CORRECT
C. IF NUMBER 2,4 ARE CORRECT
D. IF ONLY NUMBER 4 IS CORRECT
E. IF ALL NUMBERS ARE CORRECT

101. Treatment of myofascial trigger point: E


1. Dry needling
385
2. Intermitten cold and stretch
3. Deep pressure soft tissue massage
4. Myofascial tigger point injection
102. Indication/s of using orthotic in rheumatoid arthritis is/are: A
1. Reduced weight through joint
2. Decrease pain and inflammation
3. Decrease joint motion-stabilization
4. To stop the deformities of the joints
103.A 45-year-old woman presents chronic pain in the region of the Achilles tendon. The Achilles tendon appears
swollen. Appropriate initial management would include D
1. Cortison injection of the Gastrocnemius muscle
2. Cortison injection of the Achilles tendon sheath
3. Immobilization in ankle plantar-flexed position with limited weight bearing
4. Use a small heel lift with initiation of gastroc-soleus as tolerated
104. The rehabilitation of a patient with AMP: B
1. Do not flex the hip joint more than 90 degrees
2. Weight bearing can be done after 3 months
3. Weight bearing can be done immediately
4. Do not abduct the hip joint
105. These are the rehabilitation programs for leprosy: E
1. Oil massage
2. Active exercise
3. Soaking the feet in water
4. Protection of the hands and feet
106. The proprioceptive disturbances would be found in the following disease(s) Stroke A
1. Stroke
2. Tabes dorsalis
3. Brown-sequard syndrome
4. Funiculus anterior lesions
107. The function outcome of complete L3 paraplegia is / are: A
1. Independent in bowel and bladder routines
2. Independent in wheelchair propulsion
3. Independent in pressure relief
4. Standing without orthosis
108. Important factors that influence walking ability after stroke include are E
1. Balance
2. Weakness
3. Coordination
4. Cognitive function

386
109. A patient 5 years old was diagnosed Spastic Cerebral Palsy Diplegic type e.c prenatal, come to the
Rehabilitation Clinic with complaints could not walk and stand straight, both legs crossed and tiptoe in
standing position. What assessment are needed to determine spasticity of the hip: E
1. Phelps Gracilis Test
2. Duncan Elly Test
3. Thomas Test
4. Tardieu Test
110. Cardinal Clinical signs for SMA Type 1 Werdnig Hoffman Disease B
1. Weak cry
2. Tremor of hands
3. Frog posture
4. Weakness of legs, predominantly proximal
111. Both the hard and the soft palate (the velum) are important structures to consider for evaluating the mechanics
of sucking and feeding of infant. The role(s) of the hard palate include the following D
1. To form the anterior seal
2. To maintain the shape of the tongue
3. Provide lateral boundaries for food on the tongue
4. Assist positioning and stability of the nipple
112. Man, 55 year old, weight 85 kg, height 165 cm. He got myocard infact 1 month ago. The fasting glucose level
250, total cholesterol 400, no hypertension. Factor that should be considered to prescribe cardiac rehabilitation
program: C
1. Avoid endurance exercise
2. Assessed the exercise stress test
3. Limitation any kind of daily activity
4. Reduce the risk factor for 2nd cardiac disease attack
113. The most common location of DVT include the calf, tight, arms, and pelvis. The symptoms and signs are: E
1. Pain in the calf with walking
2. Pain in the calf or thigh
3. Positive Hormans sign
4. Swelling of the calf
114. Absolute contraindication(s) for inpatient and outpatient exercise program according to the ACSM include: E
1. Unstable angina
2. Uncontrolled arrhythmia
3. Exacerbation of Congestive Heart failure
4. Resting blood pressure >200/110 mmHg
115.Factors to consider in establishing an aerobic exercise prescription in elderly people: B
1. The exercise does not impose orthopedic stress
2. The recommended intensity is 70-80% of HR reserve
3. Initially increase exercise duration rather than intensity
4. The intensity is better to use a measured age-predicted HRmax
116. The following is/are the indication for measurement of bone density: E
387
1. Adults taking medications such as phenytoin
2. To monitor treatment effect on osteoporosis
3. Women discontinuing estrogen therapy
4. Women aged 65 and other
117. Which medication is/are associated with osteoporosis? D
1. Hydroclorothiazide
2. Coumadin
3. Diltiazem
4. phenytoin
118. what is/are the effect(s) of aging on muscle composition, for strength and endurance: E
1. Increased in fat and connective tissue within older muscle
2. Reduction in the number of function motor units
3. Reduced of type II fast-twitch muscle fiber
4. Reduced protein synthesis
119. Common cause of shin splints is (are): E
1. Sudden increases in distance or speed running
2. Running on slopes or uneven surfaces
3. Inappropriate footwear
4. Flat arches of the feet
120. A woman, aged 25 years old had ACL reconstruction 1 week ago. She arrives at her first outpatient visit with
moderate edema about knee and ankle. She states that she has been compliant with weight bearing and uses
her crutches and brace as instructed edema, what kind of exercise program you give for this patient? A
1. Elevate leg above heart level 20 minutes four times a day
2. Ankle pumps 10 repetitions every minute while elevating her leg
3. Quadriceps and hamstring isometric contraction using closed chain environment
4. Prone position with the involved leg hanging over the edge of the table or bed
121. The objective(s) in training muscle, tendons and joints after injury is/are to: E
1. Stretch the connective tissue of tendons and muscle to an optimal length
2. Increased the strength of muscle and tendon attachments
3. Improve coordination and proprioception
4. Regain good mobility of the joints
122. What are the rehabilitation goal for 1-4 weeks post ACL reconstruction? E
1. Full knee extension
2. Regain cardiovascular capacity
3. Increase active and passive ROM
4. Ambulation without an assistive device
123. Genu valgum: the mechanical axis for the lower limbs is displaced laterally and this cause: B
1. Foot pronation
2. Femoral anteversion
3. Compression of the medial knee joint
4. Shortened medial knee joint structure
388
124. The two major “crutch walking” muscles of the shoulders are C
1. Teres minor
2. Latissimus dorsi
3. Pectoralis minor
4. Lower pectoralis major
125. Osteoarthritis may occur in the following joints: A
1. Ankle joints
2. Subtalar joint
3. Facet joint of the vertebra
4. Uncovertebral joint of Luschka
126. In quite standing, the ground reaction force (GRF) re located in: C
1. Posterior to knee
2. Anterior to ankle
3. Anterior to hip
4. Posterior to hip
127. The function of the following extrinsic muscle of the hand is(are) to extend the finger: A
1. Extensor digiti quinti
2. Extensor indicis propius
3. Extensor digititorum communis
4. Extensor carpi radialis longus
128. A healthy elderly person may display the following gait deviations: B
1. Smaller step length
2. Increased arm swing
3. Slower ambulation velocity
4. Extension of the hips and knees
129. The mechanism of blood flow reduction caused by vasoconstriction following cryotherapy is/are: A
1. Decrease of histamine and prostaglandin release
2. Increase of sympathetic adrenergic activation
3. Smooth muscle contraction
4. Increase blood viscosity
130. A 53- year-old woman with chronic venous disease has lower extremity edema, skin hyperpigmentation,
dermatitis, and venous varicosities. She does not have any ulceration. The mainstay of treatment for this
patient is:B
1. Intermittent external pneumatic compression
2. Hydrotherapy with warm water (100 degree F)
3. Topical steroid cream
4. Gradient elastic stocking
131. Reflection of ultrasound E
1. Occurs mainly at interfaces of tissues with different acoustic impedances
2. Is great at the interface of soft tissue and metallic implants
3. Is minimal at the interface of layers of soft tissue
389
4. Is great at the interface of soft tissue and bone
132. A rigid dressing may be attached to pylon and foot to allow ambulation soon after surgery. The combination
of rigid dressing, pylon and foot is called an postoperative prosthesis (IPOP). The advantage(s) of IPOP is/are:
E
1. Improved healing time
2. Protection of the surgical site
3. Reduction of contracture development
4. Control and shaping of the residual limb
133.The following features of running shoe may diminish symptom associated with a pronated foot: A
1. Board and straight last construction
2. Motion control heel counter
3. High-density insole medially
4. Lateral wedges
134. The manifestation of circumduction gait of amputee who is wearing a lower limb prosthesis usually caused
by: A
1. Socket small, the residual limb cannot enter fully
2. Inadequate suspension, socket slips down during swing
3. Amputee is reluctant to flex the knee during swing because of poor balance
4. Amputee does not bother to flex the knee because the prosthesis is too short
135. Indication to use of aquatic therapy: E
1. Decreased range of motion
2. Weight bearing restrictions
3. Peripheral edema
4. Gait deviation
136. The occupational methods in the treatment of cancer patient is/are by: A
1. Use music to promoting relaxation
2. Use of craft activities to maintain self esteem
3. Use of humor in changing the personal outlook on the situation
4. Exercise program with progressive resistive exercise of upper and lower extremity
137. Contraindication(s) to manipulation of the cervical spine include of: E
1. Osteoporosis
2. Herniated cervical disc
3. Acute soft tissue injury
4. Spondylitis tuberculosa
138. The laboratory test most commonly associated with the diagnosis of anklylosing spondylitis is/are: D
1. Rheumatoid factor
2. Antinuclear antibody
3. Antimitochondrial antibody
4. HLA-B27 histocompatibility antigen
139. The following is/are about idiopathic scoliosis: B
1. Genetic factor is a sex-linked trait with incomplete penetrance and variable expressivity
390
2. A sex-linked trait can be transmitted by a father, to either a son and daughter
3. The curves have strong tendency to progress rapidly during the adolescent growth spurt
4. Juvenile idiopathic scoliosis occurs between ages 2 and 8
140. What is the primary role for methotrexate in the treatment of rheumatoid arthritis? D
1. To cure the disease
2. To reserve already present joint deformity
3. To provide analgesia and inflammation
4. To slow or prevent joint destruction and loss of function
141. The following condition could lead to Thoracic Outlet Syndrome: A
1. Cervical rib
2. Fracture of the clavicle
3. Hypertrophy of the M Scalenus anterior
4. Hypertrophy of the M Pectoralis minor
142.The clinical sign(s) of Alzheimer is/are: E
1. Recent memory deficit
2. Language impairment
3. Psikomotor impairment
4. Cognitive dysfunction
143. The most common behavioral and personality changes seen following Traumatic Brain Injury is/are: E
1. Attention deficit rigidity
2. Visual perceptual deficit
3. Spatial dysfunction
4. Memory deficit
144. 107 Some physical findings below must be assessed in patients with neurogenic bladder E
1. Sacral sensation
2. Cognitive impairments are an important test
3. Hand function for SCI patients who are the perform self catheterization
4. Bulbocavernosus reflex to test in the integrity of the pundendal nerve and S2-S4 segments
145. You are about examine a child who sustained a fracture during a fall from a swing set. The serious
complication following supracondylar fracture of the humerus in a child is/are: D
1. Gunstock deformity
2. Tardy ulnar nerve palsy
3. Nonunion of the fracture
4. Volkmann’s ischemic contracture
146. The following statement of Brachial plexus palsy in infant and child is/are true: E
1. Injury to the upper plexus occurs in 90% of cases
2. Must enrolled in a closely supervised rehabilitation program
3. Neurosurgery may benefit infants who have severe proximal weakness but a good hand
4. The presence of sensory nerve potentials in a clinically insensitive arm is helpful in delineating root
avulsion
147. The following(s) is/are the clinical characteristic of Down Syndrome: E
391
1. The head usually larger than normal because of overdevelopment of facial bone and muscle
2. The toes are usually short and in the majority, there is wide space between the first and second toes
3. Ligamentum laxity causing deficiency in grip strength and ankle strength that develop on school
age
4. Most significant musculoskeletal impairment are owing to hypotonia and ligamentum laxity caused
by deformityof the joint
148. Which is/are the following including in classification of Cyanotic Congenital Heart Disease? D
1. Patent ductus arteriosus Botalli
2. Ventricular septal defect
3. Atrial septal defect
4. Tetralogy of Fallot
149. The following(s) is/are the indication of postural drainage as a form of chest physical therapy to enhance the
flow of mucus out of airways E
1. Polycystic lung disease
2. Bronchopneumonia
3. Chronic Bronchitis
4. bronchiectasis
150. What exercise are recommended for persons with nonprogressive mild to moderate restrictive lung disease
(VC of 60% predicted)? D
1. Pursed lip breathing
2. Rapid shallow breathing
3. Glossopharyngeal breathing
4. Inspiratory resistive exercise

392
Januari 2018

1. The clinical features of osteoarthritis may includes … A


a. Herbeden’s nodes : Bouochard
b. Swan-neck deformitu : RA
c. Boutonniere deformity : RA
d. Ulnar deviation at MCP joints : RA
e. Spindle-shaped swelling of PIP joints : RA

2. One of the following is not appropriate intervention during an acute exacerbation of


hand rheumatoid arthritis … E
a. Icing
b. Analgesic agents
c. Anti-inflammatory agents
d. Active range of motion exercises
e. Resistance training such as weight lifting

3. There are two tendons that are involved in patient with De Quervain’s tenosynovitis,
they are: D
a. Opponens pollicis and flexor pollicis brevis
b. Abductor pollicis brevis and opponens pollicis
c. Flexor pollicis brevis and abductor pollicis brevis
d. Abductor pollicis longus and extensor pollicis brevis
e. Abductor pollicis brevis and extensor pollicis longus

4. Adson maneuver for diagnosis of the following condition : A


a. Scalenus-anticus and cervical rib syndrome
b. Costoclavicular syndrome : eden test  TOS III
c. Hyperabduction syndrome
d. Cervical root syndrome
e. Brachial plexus injury

393
5. The following structural changes are the predisposising factors of costoclavicular
syndrome , EXCEPT … B  TOS I, II, III tdk ada hub dgn m.subclavius
a. Cervicodorsal scoliosis
b. Atrophy subclavius muscle
c. Congenital malformation of first rib
d. Abnormality of the costocoracoid ligament
e. Congenital abnormality of anterior curvature of the clavicle

6. The nerve most commonly injured with posterior or posterolateral approach for total hip
replacement is the …A : anterior  femoral
a. Sciatic
b. Femoral
c. Obturator
d. Inferior gluteal
e. Superior gluteal

7. A 40-yers-old man sustained an injury to his left arm 3 weeks ago, when he lost his
balance and crashed into a bookshelf. His complaints include left arm pain, weakness with
extension of his wrist and fingers and decreased hand grips. He denied any numbness but had
odd sensations over the dorsum of the left hand. Prior to any testing, which problem would you
consider as the most likely? D
a. Posterior interosseous neuropathy
b. Posterior cord brachial plexopathy
c. Median neuropathy
d. Radial neuropathy
e. C7 radiculopathy

8. Sustained, repetitive twisting movements of variable speed, anywhere in the body is …


B
a. Chorea : movement disorder that causes involuntary, unpredictable body movement
b. Athetoid
c. Balismus : very rare movement disorder, seluruh tbh
d. Spasticity

394
e. Titubation : staggering gait of a swaying or shaking of the trunk

9. Homonymus hemianopsia indicates a … D


a. Cortical lesion
b. Chiasmal lesion : hemianopsia bitemporal
c. Prechiasmal lesion
d. Postchiasmal lesion of the opposite side
e. .lesion of on optic tract on the same side

10. A patient appears to have good strength in the hand and can use it for gross activities
automatically but cannot perform skilled movements. This form of disability is called : C
a. Motor apraxia : impaired of skilled movement that is not explained by weakness
b. Kinetic apraxia
c. Ideational apraxia : can’t plan specific movement
d. Ideamotor apraxia : inability to carry out
e. Constructional apraxia : affects the person ability to draw or copy stuple diagram

11. When treating patients with cerebral palsy consider : B


a. AFOs only for not nonambulatory children : ambulatory & non-ambulatory
b. Computers are important for school and recreation
c. Bracing to prevent progressive deformity of scoliosis : scoliosis CP  no need brace
d. Limited strengthening as it has been shown to increase spasticity : reduce spasticity
e. Using constraint-induced therapy techniques to improve upper extremity function in
children with spastic tetraplegia : CIMT >3

12. Management of Pes Planus grade I at age less than 1 years old is done by : E
a. Chukka type shoes
b. Arches support
c. Lateral wedge
d. Thomas heel
e. Observation : s/d 2 thn

395
13. The following is NOT the characteristic of Becker’s Mucular Dystrophy (BMD) … D
a. It is unusual for patients with BMD to be a wheelchair dependent before late
adolescence
b. Is similar to Duchenne’s dystrophy in terms of distribution of weakness
c. Generally presents with a later onset and slower rate of progression : BMD 
adolescent
d. Intellectual impairment is the most common in BMD : - DMD
e. It is also an X-linked inherited disorder

14. The following statement is NOT characteristic of spastic diplegia CP … E


a. Persistent obligatory infantile reflex activity beyond 18 months implied poor prognosis
for ambulation
b. Children with diplegia have adequate movement control of the arms and hands
c. Sitting by 2 year was a good predictive sign of eventual ambulation
d. Children who did not sit by 4 year have not achieved ambulation
e. Mental retardation is the most common in spastic diplegia : quadriplegia

15. The statement below is ….. effect of exercise training cardiac rehabilitation … C
a. Reducing plasma glucose
b. Lowering resting heart rate
c. Increasing LDL cholesterol
d. Increasing HDL cholesterol
e. Lowering resting blood pressure

16. The following below is NOT absolute indication for terminating exercise testing : D
a. Dislodged ECG leads
b. Subject’s desire to stop
c. Onset of 2nd or 3rd degree AV block
d. Increased systolic blood pressure until 180 mmHg
e. Increased diastolic blood pressure more than 120 mmHg

17. Effect of prolonged bed rest include : B


396
a. Increased VO2 max
b. Decreased of stroke volume
c. Increase of plasma volume
d. Decreased resting heart rate
e. Increased left ventricular end diastolic volume

18. The proper emergency response for a patient who has experienced a cardiac arrest but
still who breathing and has a palpable pulse, includes … A
a. Placing the patient in the recovery position with the head side to prevent airway
obstruction
b. Continuing the exercise test to determine why the patient had this response
c. Placing the patient in a comfortable seated position
d. Placing the patient in a head up 30 degree position
e. Placing the patient in a head down position

19. What is the appropriate initial treatment of an older patient with a degenerative meniscal
tear?A
a. Trial of nonsteroidal anti-inflammatory drugs (NSAIDS) and of by immobilisation
b. Up to three cortisone injections over a 3-week period as symptoms indicate
c. Corticosteroid intra-articular injection to control the pain
d. Immediate arthroscopy with partial meniscectomy
e. Prolonged non-weight-bearing

20. The primary analgesics to control chest pain … A


a. Non opioid and opioid
b. Opioid and muscle relaxants
c. Corticosteroid and non opioid
d. Non opioid and antidepressants
e. Antidepressants and muscle relaxants

21. The following drug is not causing weakness in elderly people … D


a. Diuretic
397
b. Antihistamine
c. Corticosteroid
d. Bisphosphonate : osteoporosis
e. Antidepressant

22. Drug that NOT give effect of orthostatic hypotension is … C


a. Diuretics
b. Hipnotics
c. Beta blocker : propanolol
d. Tranquilizers : anti psikotik, ES : bradikardi
e. Antidepressants

23. The anterior cruciate ligament (ACL) generally injured after what type of injury ? A
a. Flexion, valgus, external rotation
b. Hyperextension with valgus
c. Flexion, varus
d. Direct trauma
e. Pure varus

24. An example of a closed kinetic chain ankle group strengthening exercise that can be
used during rehabilitation of chronic ankle … is one of the following : A
a. Tiptoe : jinjit
b. Kickboard use in a pool
c. Walking use treadmill with elevation
d. Ankle eversion exercises with resistance tubing
e. Seated knee extension exercises with a sandbag weight on the ankle

25. The following statement is TRUE regarding delayed-onset muscle soreness … : A


a. Is generalized muscle pain that occurs 1 to 2 days after intense eccentric exercise
b. Clinically muscle swelling and weakness may persist for one day
c. Strength loss may up to 100% immediately post exercise

398
d. The structural abnormalities include myofibril disruption
e. Initial treatment local injection of steroids

26. Ankle sprain is ubiquitous in sport and is the most common in atlet injury. The most
commonly injury for this disorder is … B
a. The posterior taofibular ligament
b. The anterior talofibular ligament
c. The anterior talotibial ligament
d. The posterior talotibial bands
e. The calcaneotibial ligament

27. The following ….. muscle of the hand that has …… : C


a. Adductor pollicis
b. Opponens pollicis
c. Flexor pollicis brevis : ½ ulnar & ½ median
d. Extensor pollicis brevis
e. Abductor pollicis brevis

28. Weakest spot in the posterior lumbar region which that not strengthened by posterior
longitudinal ligament is : E
a. L1-2
b. L2-3
c. L3-4
d. L4-5
e. L5-S1

29. The function of rotator cuff is : A


a. To counter compressive force generated by deltoid during active shoulder
abduction
b. To prevent the head of humerus from impinging on the transverse scapular notch
c. To counter rotary forces generated by deltoid during active shoulder abduction
d. To depress the shoulder girdle
399
e. To flexion the shoulder girdle

30. The following is normal cardiorespiratory responses to exercise EXCEPT … D


a. At exercise intensities up to 50% VO2 max, the increase in cardiac output is facilitated
by increases in HR and SV
b. Heart rate (HR) increases in a linear fashion with the work rate and oxygen uptake
c. Myocardial blood flow may increase 4-5 times with exercise
d. Stroke volume (SV) decreases with the work rate
e. Ejection fraction may increse

31. All of the following are the characteristics of the fibrillation waves EXCEPT … B
a. They appears spontaneously during muscle at rest
b. The sound is typically like “diving bomber” : myotonic discharge
c. They have regular appearance
d. The amplitude is very small
e. Denervated potential

32. Which one of the following is TRUE regarding energy sources for contraction … B
a. Energy for muscle contraction is derived directly from ADP : ATP
b. Creatinine phosphate provides a substrate for rapid resynthesis of ATP
c. Rapid glycolysis is capable of producing ATP with involvement of oxygen : ADP
d. Muscle glycogen provides a source of the energy required for first muscle contraction
e. The energy bonds of creatine phosphate are equal with concentration of resting muscle
ATP

33. A male factory worker has a traumatic on his right carpometacarpal. The orthopaedic
surgeon decide to amputee his right fore-arm, and asks your opinion regarding management to
obtain the BEST functional level. You suggest : D
a. Wrist disarticulation
b. Skin graft and salvage of the carpals
c. Preservation of styloid processes for better suspension
d. Below-elbow amputation 6 cm proximal to the radial styloid

400
e. Below-elbow amputation in the proximal one-third of the forearm

34. Three months ago, a 24 years old man suffered complete avulsion of all motor &
sensory roots of the brachial plexus. The MOST appropriate orthosis is … E
a. A molded shoulder cap and harness for a functional arm brace
b. An Engen externally powered elbow-flexion unit
c. An airplane sjhoulder orthosis (splint)
d. A myoelectric unit
e. An arm sling

35. Following a cervical spine fracture, you are ordering a cervical orthosis for a C6-spared
quadriplegic pattern. He has been in a halo for 2 months, and the ortopedic surgeon request 3
additional months of neck immobilization. Which cervical orthosis BEST provides effective
limitation of flexion as well as comfort and ease of application? E
a. Camp plastic collar with adjustable occipital and mandibular pieces
b. Soft cervical collar with plastic reinforcement
c. Plastizote Philadelphia collar
d. Fom-poster orthosis
e. SOMI

36. Among the following electrical modalities, which one may burn or cause tissue necrosis
around an internal bony fixation plate? A
a. High frequency short wave currents : kontraindikasi
b. Galvanic current
c. Ultra sound
d. Cold laser
e. TENS

37. When treating supraspinatus tendinitis with USD, which shoulder position is the most
beneficial? A : abd & int rot
a. Arm abducted and internally rotated
b. Arm abducted and externally rotated
c. Arm adducted and external rotated

401
d. Arm adducted and internal rotated
e. Position of comfort

38. The following statement(s) is true regarding ultrasound diathermy as a therapeutic


modality, EXCEPT … C
a. Ultrasound diathermy have thermal and mechanic effect
b. Most ultrasound have duty cycles present at either 20% or 50%
c. Ultrasound is contraindication for people with metal implants
d. The transducer converts electrical energy to acoustic energy
e. Pulsed ultrasound is most often used with the stationary technique

39. This type of exercises is NOT regarded as strengthening exercise … E


a. Hettinger and Muller technique : isometric exercise
b. Zinovieff’s Oxford technique
c. DeLorme technique : utk ACL
d. Knight technique
e. Frenkel exercise : lat koordinasi balance  utk Parkinson

40. Which of the following is TRUE regarding core conditioning? C


a. Core conditioning focuses on the abdominal muscle
b. Core conditioning focuses on the spine intrinsic musculature
c. Core conditioning focuses on the abdominal, paraspinal and gluteal musculature
d. Core conditioning has been demonstrated to minimize injury to the low back in athletes
e. Core conditioning refers to strengthening of the major (core) muscle groups of the upper
and lower extremities

41. For which of the following is CPM (Continuous Passive Motion) contraindicated? C
a. Burn patients
b. Capsulotomies
c. Unstable fractures
d. Surgical repair of ligament
e. Fractures with internal fixation
402
42. Which is a risk factor for plantar fascitis? A
a. Bigger runner
b. Female gender
c. Younger than 40 years
d. Increased subtalar motion
e. Limited foot plantar flexion

43. American College of Rheumatology classification criteria for systemic lupus


erythematous include all of the following, EXCEPT : E
a. Malar rash
b. Photosensitivity
c. Hemolytic anemia
d. Immunologic disorder
e. Raynaud’s phenomenon : gang pembuluh darah perifer (PAD)  skleroderma

44. The most common symptom musculoskeletal manifestations of SLE is : D


a. Myofascial pain
b. Neck pain
c. Weakness
d. Athralgia
e. Fatigue

45. The nerve most commonly injured with posterior or posterolateral approach for total hip
replacement is : A
a. Sciatica
b. Femoral
c. Obturator
d. Inferior gluteal
e. Superior gluteal

46. The clinical signs of biceps tendinitis may include all the following, EXCEPT : B
403
a. Speed’s test is often positive
b. The impingement sign always present : blm tentu
c. Frequently found association with cuff disease
d. Yergason’s test may produced pain at the shoulder
e. Biceps degeneration is seen with compression from the anterior acromion

47. Lesion of the following nerve roots will lead to classical “winging of the scapula” : B
a. C4, C5 and C6 roots
b. C5, C6 and C7 roots : erb’s palsy
c. C6, C7 and C8 roots
d. C7, C8 and T1 roots
e. C8, T1 and T2 roots

48. On examination the hemiplegic patient showing that he is fluent and able to repetite the
examiner but cannot comprehend the examiner. It means that the patient has : E
a. Broca aphasia : F (+), C(+), R(-)
b. Global aphasia : F(-), C(-), R(-)
c. Wernicke aphasia : F(+), C(-), R(-)
d. Transcortical motor aphasia : F(-), C (+), R(+)
e. Transcortical sensory aphasia : F(+), C (-), R (+)

49. The most appropriate wheelchair for a T1 paraplegia AIS/Frankel A is : B


a. Wheelchair with vertical bars on the handrims
b. Regular wheelchair with higher backrest
c. High performance wheelchair
d. Mouth-operated wheelchair
e. Regular wheelchair

50. A 50-year old man has had gradually progressive hand weakness. He has atrophy of the
forearm muscles fasciculations of the muscles of the chest and arms, hyper reflex of the lower
extremities and positive Babinsky signs. Sensation is not impaired. Which of the following
most likely diagnosis?A

404
a. Amyotrophic lateral sclerosis
b. Dementia Alzheimer type
c. Gullain Bare syndrome
d. Transverse myelitis
e. Multiple sclerosis

51. Atrophy of the first dorsal interosseous muscle may indicate damage to spinal roots : E
a. C4 and C5
b. C5 and C6
c. C6 and C7
d. C7 and C8
e. C8 and T1

52. The most common Arthrogryposis multiplex congenital is : B


a. Myopathic in origin
b. Neurogenic in origin
c. Associated with cardiac diseases
d. Associated with early joint fusion
e. Associated with mental retardation

53. A 3-months old girl with congenital torticollis does not have a palpable fibrosis muscle.
Her chin to the left and her head tilts significantly to the right. You should : D
a. Instruct the parents in gentle stretches to the left sternocleidomastoideus muscles
b. Prescribe Dantrolene sodium to control muscular spasms
c. Advice surgical consultation for operative intervention
d. Examine the hips to rule out an associated hip dysplasia
e. Prescribe semirigid cervical collar

54. A 9 year-old boy has recently been diagnosed had spinal muscular atrophy type III
(Kugelberg Welander Disease). You would expect : D
a. Electromyographic changes consistent with a myopathic process
b. Rapid progression of the disease with death by age 20 years
405
c. Early involvement of distal upper extremity musculature
d. Ambulation to be preserved until age 25-30 years
e. Affects children under 1 year of age

55. The appropriate orthoses to improve ambulation in myelomeningocele is : A


a. For mid thoracic level prescribe parapodium for both standing and sitting
b. For low thoracic or high lumbar level, prescribe HKAFO for ambulation
c. For low lumbar level or sacral level, prescribe KAFO for ambulation
d. For mid lumbar level prescribe reciprocal gait orthosis
e. For mid lumbar level prescribe shoe modification

56. Which of the following risk factors is the most associated with a higher incidence of
Deep Vein Thrombosis (DVT) in adult with a stroke? E
a. Male
b. Elderly
c. Obesity
d. Smoking history
e. Increased motor weakness

57. For a patient with CAD, good left ventricular function, and no history of myocardial
infarction, exercise intensity should be approximately … C
a. To anginal threshold
b. 5 bpm below angina threshold
c. 10 bpm below angina threshold
d. 15 bpm below angina threshold
e. 20 bpm below angina threshold

58. The following is NOT the indication of postural drainage as a form of chest physical
therapy to enhance the flow of mucus out of airway … C
a. Chronic bronchitis
b. Bronchopneumonia
c. Pneumothorax
406
d. Bronchiectasis
e. Atelektasis

59. The potential benefit of exercise training program in severe COPD patients is … C
a. Increased maximum voluntary ventilation
b. Increased ratio of FEV1 to FVC
c. Increase exercise tolerance
d. Improve vital capacity
e. Increased FEV1

60. Balance in the elderly can NOT be measured by this : E


a. Environmental hazards
b. Postural hypotension
c. Cardiac arrhythmias
d. Fall risk index designed by Tinneti
e. Katz Index

61. The most common cause of falls in the elderly is … A


a. Environmental hazards
b. Postural hypotension
c. Cardiac arrhythmias
d. Weakness
e. Vertigo

62. The best therapy to prevent postmenopausal osteoporosis in a 45 year-old menopausal


women is … E
a. Hormone therapy
b. Calcitonin therapy
c. Bisphosphonate therapy
d. Increase calcium intake only
e. Regular exercise and increased calcium intake

407
63. Rehabilitation effort for cancer patients with poor prognosis including the following,
EXCEPT… C
a. Skin protection
b. Transfer training
c. Increased life span
d. Equipment assessment
e. Improved quality of life

64. A national swimmer athlete complains of shoulder pain. Pain will increase in overhead
and external rotation of shoulder. Physical examination showed apprehension and relocation
test is positive. What is the diagnosis? C
a. Superior labrum anterior posterior tear
b. Posterior glenohumeral instability
c. Anterior glenohumeral instability
d. Supraspinatus tendinitis
e. Subscapularis tendinitis

65. A junior basketball athelete 14-year-old presents with a three-month history of knee
pain. The pain is most marked in the area of the tibial tubercles bilaterally and it will increase
with activity. Physical examination is unremarkable. What is the likely diagnosis? A
a. Osgood Schlatter disease
b. Early knee osteoarthritis
c. Patellofemoral arthritis
d. Rheumatoid arthritis
e. Patellar tendinitis

66. A gymnastic athlete present ankle pain after the competition. Physical examination
showed Achilles tendon rupture. What is the most common mechanism for Achilles tendon
rupture ? B
a. Passive dorsiflexion
b. Eccentric dorsiflexion
c. Eccentric plantarflexion

408
d. Concentric dorsoflexion
e. Concentric plantarflexion

67. The most likely combination musculoskeletal imbalances associated with iliotibial
syndrome are …A
a. Weak gluteus medius, tight tensor fascia lata
b. Weak gastric-soleus, tight tensor fascia lata
c. Weak gluteus maximus, tight psoas
d. Weak quadriceps, tight hamstrings
e. Weak gastric soleus, tight psoas

68. The rhomboids cause : A


a. Downward rotation, adduction and elevation of the scapula
b. Downward rotation, abduction and depression of the scapula
c. Downward rotation, adduction and depression of the scapula
d. Upward rotation, abduction and elevation of the scapula
e. Upward rotation, adduction, and elevation of the scapula

69. Double limb stance is in what percent of the entire gait cycle? C
a. 5%
b. 10%
c. 20%
d. 30%
e. 40%

70. The knee joint is a : A


a. Trochogynglimus joint
b. Gynglimus joint
c. Trochoid joint
d. Hinge joint
e. Pivot joint

409
71. EMG examination of Duchene Muscular Dystrophy will show the following findings
regarding the MUAP : C
a. High amplitude, long duration
b. High amplitude, short duration
c. Low amplitude, short duration
d. Low amplitude, long duration
e. Long duration biphasic potential

72. Low amplitude in conduction study most likely indicate … E


a. Damage at the myoneural junction
b. Damage to the myelin sheeth
c. Loss of schwann sell
d. Neuropraxia
e. Axonal loss

73. This is characterize of type I muscle fiber … A


a. High myoglobin, low glycogen, many mitochondria, slower contraction
b. High myoglobin, high glycogen, many mitochondria, fast contraction
c. Low myoglobin, high glycogen, many mitochondria, slow sonraction
d. High myoglobin, high glycogen, few mitochondria, fast contraction
e. Low myoglobin, high glycogen, few mitochondria, fast contraction

74. For an individual who has C5 tetraplegia, orthotic splinting attempts to maintain the
functional position of the hand. This usually includes … A
a. Supporting the wrist in 20˚ to 30˚ of extension
b. 30˚ to 40˚ of metacarpo-phalangeal flexion
c. Promoting flattening of the palmar arch
d. Supporting the wrist in 10˚ of flexion
e. Closing the thumb web space

75. Resection through the metatarsal and tarsal joint .. D


410
a. Syme amputation
b. Boyd amputation
c. Chopart amputation
d. Lisfranc amputation
e. Pirigoff amputation

76. In an amputee with a patellar tendon-bearing prosthesis and a SACH foot, show delayed
knee flexion at heel strike may due to … B
a. Excessive dorsiflexion of the foot
b. Too soft a heel cushion
c. Posterior socket tilt
d. Poor knee flexion
e. Poor hip flexion

77. Intermittent compression pumps is contraindicated in which of the following condition :


C
a. Post mastectomy lymphedema
b. Venous insufficiency
c. Arterial insufficiency
d. Traumatic edema
e. Amputations

78. The following mechanism of pain control may explain the analgesic effects of physical
agents, EXCEPT … A
a. Stimulate small diameter afferents fibers with TENS
b. Decrease pain fiber transmission velocity with cold or ultrasound
c. Stimulate small-diameter afferent fibers and desending pain control mechanisms with
TENS
d. Stimulate a release of β-endorphin and other endogenous opioids through prolonged
fibers stimulation with TENS
e. TENS with long pulsed widths (<200ꭒ) and low pulse rates (1-5 pulse/sec) will β-
endorphin release

411
79. The following statement of low power laser is TRUE … A
a. The dose dependent on is the output, the time of exposure and the beam surface
area
b. HeNe laser has an indirect effect on tissues up to 2 to 5 cm deep
c. GaAs laser has an indirect effect on tissues up to 1 to 2 cm deep
d. The primary action of laser application for bioinhibition
e. The scanning technique should be utilize over close wounds

80. Classical peripheral neuromuscular facilitation (PNF) using this technique … E


a. Uses cardinal component movement
b. The goal to strengthen muscle group
c. Emphasize inhibition of abnormal reflex activities
d. It uses stretching during movements in cardinal planes’
e. The goal to fascilitating irradiation of impulses to other parts of the body

81. The main difference between the therapeutic effects of quick and slow passive stretch of
a muscle is … A
a. Fast stretch is excitatory and facilitates muscle activity
b. There is actually no significant therapeutic difference
c. The pain experienced by the patient is less with slow stretch
d. The slow stretch is used only when a muscle exhibits clonus
e. A muscle inhibitory or relaxing effect is obtained by fast stretch

82. Circuit weight training including the following techniques, EXCEPT … E


a. Exercise in bouts
b. Exercises interspersed with short rest periods
c. Using light to moderate work loads with frequent repetitions
d. Exercises can be done using free weights or weight training units
e. Exercises could include 10 to 12 RMs with 5 minutes rest periode between each
both of exercise

83. The statement below is NOT TRUE regarding vertebroplasty … D


412
a. The complication include fat embolization
b. No more than 2 or 3 levels should be done at one time
c. The injection of polymethylmethacrylate into vertebral body
d. The percutaneous placement of inflatable bone tamp into a fracture vertebral body
e. It is indicated for the treatment of painful vertebral compression fracture caused by
osteoporosis

84. According to the American College of Rheumatology guideines, which agent is


recommended as first-line medication in OA of the hip? D
a. Oral gold treatments
b. Corticosteroids
c. Methotrexate
d. Paracetamol
e. Diclofenac

85. Mechanical low back pain refers to ? D


a. Pain related to spondylolisthesis
b. Pain related to sacralization of lumbar vertebrae
c. Pain related specifically to acute lumbar strain
d. Non discogenic pain that is often provoked by physical activity and relieved by rest
e. Pain related to the posterior elements (including the facet joints and pars interarticularis)

86. The following test will be probably positive in the early stage of Ankylosing spondylitis
…A
a. Gaenslens test
b. Lasseque test
c. Schober test
d. Thomas test
e. Elly test

87. The following statements is TRUE regarding osteoporosis … A

413
a. Involutional type 1 osteoporosis, predominantly trabecular bone loss in axial
skeleton
b. Postmenopausal osteoporosis, affects women within menopause, lasting 1 year
c. Involutional type 2 osteoporosis, affect only in men over age 65
d. Age associated osteoporosis predominantly cortical bone loss
e. Secondary (type 3) osteoporosis usually irreversible

88. A 21 year-old man is evaluated in your spinal cord injury clinic 12 months after a C2
complete spinal cord injury requiring full time mechanical ventilation. What will you
recommend? E
a. Frog breathing
b. Pursed lip breathing
c. Aggressive diaphragmatic strengthening exercise
d. Initiating a weaning protocol by slowly decreasing tidal volume
e. An eclectrodiagnostic study to evaluate for a phrenic nerve pacemaker

89. A 60 year-old women is seen in consultation by your rehabilitation team after elective
surgery. She has a new finding of 1/5 strength in her lower extremities, but retained
proprioception and vibratory sense. You make the diagnosis of … B
a. Posterior spinal cord syndrome
b. Anterior spinal cord syndrome
c. Brown Sequard syndrome
d. Central cord syndrome
e. Syringomyelia

90. You are evaluating a patient with a nerve palsy. During evaluation, the patient reveals a
positive Fromment’s sign. Which nerve is injured? E
a. Musculocutaneous
b. Axillaris
c. Median
d. Radial
e. Ulnar

414
91. When patients with traumatic brain injury (TBI) asking about returning to drive, you tell
them…D
a. Person with behavioural problems must wait at least 1 month after injury
b. Only an on-road evaluation is necessary to determine fitness for driving
c. Persons with TBI must wait at least 1 year post injury before resuming driving
d. Clinical assessment should include sensorimotor impairment vision and cognitive
function
e. Persons with brain injury should not drive and the physician must notify the state of
their status

92. What is the highest neurologic level on a person with complete spinal cord injury that
might be capable of unassisted independent transfer skills with or without equipment? C
a. C2
b. C4
c. C6
d. C8
e. Th1

93. In evaluating a 4 weeks-old infant for apparent paralysis of the left upper extremity, you
can not elicit biceps and triceps jerk, but sensation is intact. This finding is compatible with a
…A
a. Erb’s palsy
b. Klumpke’s type
c. Brachial plexitis
d. Peripheral neuropathy
e. Congenital hemiparesis

94. The most common method of exercise based on the neuro developmental approach for
cerebral palsy patient is … E
a. PNF
b. Voyta
c. Kabat
d. Phelps

415
e. Bobath

95. The following is the earliest treatment of CTEV : E


a. AFO
b. Plantar stop splint
c. Outflare orthopaedic shoes
d. Surgical soft tissue release
e. Stretching and serial plaster corrections

96. The Ortolani test will be positive in the following condition … D


a. Congenital lower limb deficiency
b. Slipped capital femoral epiphysis
c. Congenital hemisacralization
d. Congenital hip dislocation
e. Juvenile coxa plana

97. The major goal of phase I Cardiac Rehabilitation is to … D


a. Stimulate the development of coronary collolateral
b. Stimulate of myocardial exercise tolerance
c. To achieve cardiovascular fitness
d. Prevent deconditioning
e. Increase cardiac output

98. The following is specific considerations when prescribe exercise for individuals with
hypertension … D
a. β-blocker may also increase the predisposition to hyperglicemia
b. β-blocker may increase submaximal and maximal exercise capacity
c. pasient with DBP 80 mmHg, before exercise first being evaluated by their physician
d. anti hypertensive medications may lead to sudden excessive reductions in post
exercise blood pressure
e. for individuals with episodes of ischemia during exercise, the intensity should be set
≤10 bpm above the ischemic threshold
416
99. for chronic lung disease various walking test with sub maximal exercise have all the
benefit below, EXCEPT … C
a. To detect exercise intolerance
b. To evaluate functional capacity
c. To detect limitation in lung function
d. To know and detect functional limitation
e. To evaluate reconditioning exercise programme

100. The following is specific consideration when prescribe exercise for patients with
diabetes mellitus … A
a. Resistance training should be encouraged in the absence of contraindications,
retinopathy and recent treatments using laser surgery
b. For individual taking insulin, changing insulin timing, increase insulin dose, decrease
carbohydrate consumption
c. Common symptom hypoglicemia include polyuria, fatigue and acetone breath
d. Hypoglicemia may delayed and occur up to 2 hour post exercise
e. Dehydration a common occurrence of hypoglicemia

101. The following condition(s) is/are common foot abnormalities in patients with
rheumatoid arthritis : E
1. Hammer toe
2. Posterior tibialis tendon tendinitis
3. Metatarsalgia
4. Decreased mediasi longitudinal arch

102. Third stage of shoulder hand syndrome consists of the following clinical signs … C
1. Burning pain
2. Shoulder “frozen”
3. Swelling increases
4. Patchy osteoporosis

103. Postoperative pain patients with amputee may caused by … E


417
1. Oedema
2. Neuroma
3. Bony spur
4. Scar contracture

104. Neurogenic claudication is/are charascteristic with : B


1. Pain progressed from proximal to distal
2. Skin/trophic changes present
3. Pain decreased with walking up hill
4. Leg/foot pulses decreased

105. The following is/are the sign(s) of autonomic dysreflexia in a T4 Paraplegia ASIA A : A
1. Founding headache
2. Nasal congestion
3. Hypertension
4. Tachycardia

106. Stroke rehabilitation program consists of : E


1. Retraining the damaged nervous system and preventing or overcoming “learned
disuse”
2. Ensuring that appropriate aids are provided and used properly
3. Teaching new adaptive method
4. Preventing complications

107. The common behavioural and personality changes seen following Traumatic Brain
Injury is/are: E
1. Attention deficit
2. Memory deficit
3. Spatial dysfunction
4. Visual perceptual deficit

108. The purpose of bracing in CP patient is/are : A


418
1. To control movement that interfere functions
2. To prevent/correct contractures
3. To promote better balance
4. To immobilize the patient

109. Congenital muscular torticollis caused by : C


1. Hemivertebrae
2. Deformation from intrauterine positioning
3. Ipsilateral plagiocephaly
4. Fibrotic contracture of the sternocleidomastoid

110. Communication disorder in cerebral palsy : E


1. Occurs in 40% of children with CP
2. Especially in quadriplegia and extra pyramidal type
3. Sometimes causes misdiagnosis of mental retardation
4. Begin speech therapy early and continue throughout childhood

111. Among the following, which activity might NOT stressful for the patient with cardiac
disease ?A
1. Leg raises
2. Watching TV
3. Walking (2.0 mph)
4. Bridging exercise on a mat

112. The purpose of occupational therapy for the patient with the cardiac disease as follow :
B
1. To get relaxation
2. To prevent heart attack
3. Energy conservation
4. To get more early recovery

113. The aim(s) of chest physical therapy in asthmatic patient is/are : E


419
1. To relieve bronchospasm
2. To aids removal of secrets
3. To coordinate respiratory movements
4. To assist relaxation and gain control of breathing

114. When is invasive pain management procedures should be considered ? E


1. Pain become intractable
2. Pain not relieved by increasing dose of opiates
3. These medications lead to overwhelming side effects
4. Pain responds poorly to opiate therapy and non opiate therapy

115. Goals of rehabilitation approach in Parkinson’s disease is/are : C


1. Improve rigidity
2. Improve posture
3. Improve bradykinesia
4. Improve ADL function

116. The need for BMD measurement is indicated for perimenopausal women in order to : E
1. Evaluate patients with risk factors of osteoporotic fracture
2. Evaluate patient with metabolic bone disease
3. Evaluate medications that affect the skeleton
4. Monitor the efficacy of treatment

117. A terms player with lateral epicondylitis. Which equipment modification is/are suitable
for this athlete to support recovery? D
1. Stiffer racquet
2. Larger racquet head
3. Increased string tension
4. Larger handle diameter

118. An example of closed kinetic chain exercise of the quadriceps is : D

420
1. Knee extension on a stacked-weight machine
2. Knee extension on an isokinetic machine
3. An isometric quadriceps set
4. A partial squat

119. When will athlete with biceps tendinitis can return to sports? A
1. The athlete is pain free
2. There is no limitation of ROM
3. Six weeks when there is no surgery
4. Two months after surgical intervention

120. The following muscle(s) is/are the primary move for internal rotation at glenohumeral
joint : A
1. Pectoralis major
2. Latissimus dorsi
3. Subscapularis
4. Teres minor

121. The following statement(s) is/are regarding of the ankle mortis : A


1. The talus function as a hinge joint
2. Consist of tibia, fibula and talus
3. Its motion is 20˚ dorsi-flexion
4. Consist of talus and calcaneus

122. Some of the gait characteristic of normal aging include : E


1. Relatively move time is spent in stance phase
2. Slower walking speed
3. Shorter stride length
4. Wider base of gait

123. The following statement(s) is/are TRUE regarding the term of metabolic equivalent to
describe exercise intensity : E
421
1. One MET represents an energy expenditure of approximately 1.2 kcal/minute for
70 kg person
2. One MET is equivalent to the amount of energy expended during one minute rest
3. The absolute energy expenditure during exercise depends on the body size
4. One MET is considered equivalent to a VO2 of 3.5 mL/kg/minute

124. The following statement(s) is/are TRUE regarding metabolic responses to exercise : A
1. The Crebs cycle can be considered a primer for oxidative phosphorylation
2. Oxydation of carbohydrates results in a total of 36 ATPs per unit glucose
substrate
3. The energy to perform short term high intensity exercise comes primarily from
anaerobic pathways
4. During prolonged exercise of low moderate intensity the energy source shift from
carbohydrate to protein as a primary energy substrate

125. The pathophysiology chronic illness caused by active tobacco use is/are : A
1. Increases erythropoiesis, thereby increasing blood viscocity
2. COHb limits the oxygen carrying capacity of Hb
3. Causes the release of catecholamines
4. Decrease homocysteine levels

126. Specific function(s) of upper limb movement that must be considered in orthotic
prescription include : E
1. Reach
2. Carry
3. Release
4. Prehension pattern

127. Ankle foot orthoses with plantar stop : A braddom 255


1. It is used to control plantar spasticity
2. The plantar stop is commonly set at neutral position
3. Produces a flexion moment at the knee during heel strike
4. Produces a extension moment at the knee during heel-strike
422
128. Above knee suction socket generally is NOT prescribed for an older patient for the
following reason(s) : C
1. It is heavier than an above-knee prosthesis with hip joint and pelvic band
2. It requires more training of stump musculature to maintain the suction
3. It prevents desired rotation of the hip joint for an older patient
4. It is more difficult and energy consuming to put up

129. The physiologic effect(s) of ultrasound include as the following : E


1. Increase the extensibility of collagen tissue
2. Decrease accumulation of edema
3. Reduce muscle spasm
4. Increase blood flow

130. The following is/are the physiologic effect(s) of cold therapy : C


1. Increase nerve conduction velocity
2. Decrease accumulation of edema
3. Increase muscle excitability
4. Decrease metabolism

131. The following statement(s) is/are TRUE regarding biofeedback : E


1. Biofeedback has no specific medical contraindication
2. It is be prescribed if patients have potential voluntary control
3. It is be prescribed if patients are well-motivated and cooperative
4. It can be used for muscle recruitment in patients with peripheral nerve injury

132. The prescription of exercise for the patient with simple reccurent low back pain should
achieve at least the following : E
1. Improved low back flexibility
2. Improved posture with minimized lumbar lordosis
3. Improved body mechanics in all activities and exercise
4. Improved strength of abdominal and hip extensor muscle
423
133. The prescription of exercise for the patient with simple reccurent low back pain should
achieve at least the following : D
1. Improved low back flexibility
2. Improved posture with minimized lumbar lordosis
3. Improved body mechanics in all activities and exercise
4. Improved strength of abdominal and hip extensor muscle

134. The following is/are the treatment options of retention or the failure to empty : A
1. Valsava’s and Crede’s maneuver
2. Clean intermittent catheterization
3. Intravesical prostaglandin
4. Spincterotomy incision

135. The following statement is/are the clinical sign of spondylolisthesis : E


1. A defect of pars interarticularis vertebra  spondilolisis ?
2. The posterior elements of the vertebra remain in normal position
3. On oblique view : a Scotty dog appearance is separated from the neck 
spondilolisis ?
4. Vertebral body and transverse processes slip anteriorly on the vertebral body
below

136. The following statement is/are TRUE regarding transitional vertebrae : C


1. Lumbarization of the first sacral vertebrae decreases the level arm of the lumbar spine
2. Unilateral sacralization of a sacral vertebra may produces Bertolotti’s syndrome
3. Lumbarization causes less stress on the lumbosacral vertebrae
4. Unilateral sacralization often cause herniation of the disc

137. A C7 quadriplegic patient should NOT expected to perform the following activities
independently : C
1. Sliding board transfer
2. Ambulate with crutches

424
3. Wheelchair to car transfer
4. Floor to wheelchair transfer

138. The following is/are the sign(s) and symptom(s) may occur in Parkinson disease : E
1. Tremor at rest
2. Micrographia
3. Drooling
4. Dementia

139. The following is/are the cause(s) of peripheral neuropathic pain : B


1. Diabetic
2. Poliomyelitis
3. Chemotherapy
4. Vitamin A deficiency

140. Some physical findings below must be assessed in patients with neurogenic bladder : E
1. Sacral sensation
2. Cognitive impairments are an important test
3. Hand function for SCI patients who are the perform self catheterization
4. Bulbocavernosus reflex to test in the integrity of the pundendal nerve and S2-S4
segments

141. The following factor(s) determining ambulation potential in individuals with


myelomeningocele : A
Braddom 1087, cucur 808
1. Obesity
2. Cognitive function
3. Musculoskeletal complication
4. Hamstring and tibialis posterior function

142. The following is/are the key question(s) to ask the parent to assist determination of
needs for comprehensive promotor evaluating : E
425
1. Absence of weight gain 2-3 months at >2 year of age
2. Feeding time > 30 minutes
3. Have respiratory problem
4. Gurgle voice

143. Hip displacement is cerebral palsy may caused by : A braddom 1066, 1056
1. Asymmetrical activity of muscles surrounding the hip
2. Decreasing of weight bearing
3. Motor delayed development
4. GMFCS 1

144. The potential benefit(s) of exercise training program in severe COPD patients is/are : D
1. Improve FVC
2. Increased FEV1
3. Increased FEV1/FVC
4. Increase exercise tolerance

145. Management options for COPD patients including : A


1. O2 supplement to maintain SaO2 >90%
2. Controlled breathing techniques
3. Energy and work conservation
4. Use of antiviral agents

146. The benefit of pursed lip breathing in COPD patients as follow : B


1. Preventing air tapping
2. Decreased tidal volume
3. Facilitation of relaxation
4. Increased work of breathing

147. The following is/are the cause(s) of cancer-related fatigue : A


1. Malnutrition and dehydration
426
2. Iatrogenic medications
3. Cognitive failure
4. Hypocalcemia

148. PM&R program in Fibromyalgia including the following : A


1. TENS
2. Amitriptyline
3. Patient education
4. Trigger point injection

149. Medial stress of injuries of the elbow in throwing athletes, includes : A


1. Ulnar nerve traction
2. Flexor muscle strain or tear
3. Avulsion of medial epicondyle
4. Olecranon osteophyte formation

150. The prevention of illness and injury in sport depends : E


1. Equipment
2. Health control
3. Rules and facilities
4. Appropriate clothing

427
NATIONAL BOARD EXAMINATION
(FEB 2018 – REMID)

1. A 45 year old woman presents with pain in the region of the Achilles Tendon. This is
the third such episode over the past 2 years. The Achilles tendon appears swollen and warm.
Appropriate initial management would include… E
a. Cortison injection of the achiles sheath
b. Given SWD for reducing swollen and warm
c. Immobilization in an ankle plantar-flexed position
d. Prescription of a custom-molded AFO to reduce pronation
e. Use of a small heel lift with initiation of gastroc-soleus stretches as tolerated

2. Pain around the mediaspect of the knee joint very often caused by inflammation of the
pes anserius. The conjoin tendon of the following muscles… C
a. Gracilis-Sartorius- Biceps Femoris
b. Gracilis-Sartorius-Vastus Medialis
c. Semitendinosis-Sartorius-Gracilis
d. Sartorius-Gracilis-Vastus Medialis
e. Semimembranosus- Sartorius-Gracilis

3. The following is the initial treatment for acute osteoarthritis, EXCEPT: C


a. Knee orthosis
b. Icing to relieve joint symptoms
c. Given USD to reduce inflammation
d. Medication to reduce inflammation and pain
e. Immobilization of the joint to reduce inflammation and joint pain

4. For pain to be consider chronic it must have been present for at least… B
a. 1-2 months
b. 3-6 months
c. 7-9 months
d. 10-12 months
e. 13-16 months

428
5. In thoracic outlet syndrome if the adson’s test is positive, this suggested to the following
condition… E
a. Hyperthropy of the sternocleidomastoideus muscle
b. Hyperthropy of thescalenus posterior muscle
c. Hyperthropy of thepectoralis mayor muscle
d. Cracture of the clavicle
e. Cervical rib

6. A 40 year old man, 4 weeks after traumatic brain injury with left hemiplegia. He no
develops severe spasticity, particularly affecting the left shoulder and arm muscles. Which one
of the following treatments is NOT validated? C
a. Injection of botulinum a toxin in the spastic muscle
b. Alcohol injection in the spastic muscle motor points
c. Low level laser therapy
d. Gaba-type medication
e. Icing

7. In the physical examination found left hemineglect. The following sign can be present,
EXCEPT… A
a. Aphasia
b. Denial of his hemiplegia
c. Mistakes in the localization of sounds
d. The absence of the left part of his drawing
e. A decrease of the visual field at the opposite side of the lesion

8. What is the estimated rate of nerve regeneration? C


a. 0,1 mm/day
b. 1 mm/week
c. 1mm/day
d. 1cm/week
e. 1cm/day

9. The following statement is NOT the characteristic sign of conus medullaris syndrome D
a. Perineal sensory deficit
429
b. Sphinter dysfunction
c. Loss of anal reflex
d. Pyramidal sign
e. Flaccid sign

10. The gold standard for diagnosis of duchenne muscular dysthrophy is… C
a. To check blood creatine kinase level
b. To check isoenzim CKMB
c. To have muscle biopsy
d. MRI of leg muscles
e. To perform EMG

11. A child with C5 ASIA a spinal cord injury should eventually become independent in
which activity D
a. Intermittent catheterization
b. Transfer to level surface
c. Standing
d. Feeding
e. Bathing

12. The period when intense motor learning and basic language development occur at E
a. Birth to six months
b. Six months to 1 year
c. Birth to one year of age
d. Birth to two year of age
e. Birth to three year of age

13. One of your 4 year-old patient exchibits. The following characteristic: distress over
minor changes join environment, echolalia, lack of awareness of the existence of feeling in
others, non participation in simple games. The most likely diagnostic is.. A
a. Autism
b. Cerebral palsy
c. Mental retardation
d. Hearing impairment
430
e. Minimal brain damage

14. The main motor characteristic of CP ataxia is A


a. Disturbance of balance
b. Writing movement
c. Jerky movement
d. Spasticity
e. Rigidity

15. The statement below is NOT characteristic of Cerebral palsy… D


a. The underlying neurologic lesion must be static
b. The sign of cerebral palsy is a disorder of posture
c. The sign of cerebral palsy is a disorder of movement
d. The process which cause the cerebral palsy is still active
e. It is caused by non progressive injury which affects the immature brain

16. Communication is very important in human and takes many forms especially in
children. The appropriate skill of communication in a child 2 years – old is… D
a. A look
b. A gesture
c. The first word
d. Two-words phrases
e. One or two sentence

17. A prerequisite to glossopharyngeal breathing is… B


a. Patent tracheostomy
b. Good tongue strength
c. Good abdominal
d. Good intercostals externus strength
e. Need some diaphragmatic activity to assist

18. An individual with emphysema would… A

431
a. Show decreased flow on forced expiration largely due to loss of elastic recoil of the
lung
b. Have a smaller (more negative) pleural pressure than normal at TI.C
c. Have increased bronchial smooth muscle tone
d. Have an elevated FEV1 FVC ratio
e. Have an elevated FEV1

19. The following condition is NOT a contraindication for exercise program to a diabetic
patient with cardiovascular complications… C
a. Ketosis
b. Blood sugar <180mg%
c. Blood sugar 180-250mg%
d. Blood sugar >300mg%
e. Blood sugar < 400mg%

20. How might the patient with obstructive pulmonary disease help overcome air trapping?
C
a. Slowly inhale until the lungs are fully expanded
b. Forcefully inhale for as long as possible
c. Exhale with the lips purses
d. Forcefully exhale
e. Breathing control

21. Component of comprehensive pulmonary rehabilitation include… E


a. increase aerobic capacity
b. increase of symptoms, increase pulmonary capacity test
c. muscle strength exercise, repair of pulmonary radiography examination
d. endurance exercises, intervention of psychological an life style, increase respiratory
function test
e. exercise training, nutritional therapy, education for management of psychological
and life style, promotion for long term adherence

22. Exercise programming in patient PTCA have more benefit to… A


a. Increase aerobic capacity

432
b. Increase myocardical oxygen demand
c. Decrease ejection fraction of the left ventricle
d. Decrease cardiac output response to submaximal exercise
e. Decrease stroke volume response to submaximal exercise

23. The three main sites of fractured associated with bone loss are… A
a. Spine, hip, and wrist
b. Spine, hip, and knee
c. Spine, hip, and ankle
d. Spine, hip, and elbow
e. Spine, wrist and ankle

24. Which medication is associated with osteoporosis? C


a. Hydroclorothiazide
b. Cournadin
c. Phenytoin
d. Diltiazem
e. NSAID

25. Which statement is TRUE regarding calcitonin? E


a. It is a first line therapy for osteoporosis prevention and treatment
b. Its concomitant use with calcium is contraindicated
c. It is available in subcutan injection
d. It is available in an oral form
e. It has analgesic properties

26. What is the most common cause of dementia in geriatric population? D


a. Toxic-metabolic dementia
b. Lewy body dementia
c. Vascular dementia
d. Alzheimer disease
e. Parkinson

433
27. This type of activity and exercise is NOT recommended in osteoporotic patient to
prevent developing compression fractures of the spine? C
a. A prevent falls
b. Back exercise
c. Flexion exercise for the spine
d. Keep heavy object close to chest when lifting
e. Flex hips and knees to bend down to lift objects

28. A female volleyball athlete has been diagnosed with a grade 2 quadriceps strain after
lunging for a ball. The athlete trainer has determined that the rectus femoris involved and there
is loss of 45 degrees of knee flexion while lying prone. What do you recommend to help the
athlete return to play? A
a. Hamstring stretch
b. Quadriceps stretch
c. Hip adductor stretch
d. Hip abductor stretch
e. Iliotibial band stretch

29. Which is the most important risk factor for an ankle sprain? E
a. Flat foot
b. Rear foot valgus
c. Shirt achiles tendon
d. Generalised joint laxity
e. A history of a previous sprain

30. Which statement is true according patellofemoral pain syndrome ? E


a. Pes anserinus pain
b. The pain often be felt while running or jogging
c. Also reffered to as posteromedial knee pain syndrome
d. Should always be associated with laxity of ligament around the knee
e. Is characterized by pain at front or middle edge of the knee or under the knee cap

434
A young basketball athlete, aged 18 years old, injured his right ankle while playing basketball
in a competition 1 week ago. His x-ray films showed no fracture. His physician diagnosed the
injury as lateral ankle sprain. Visual Analog Scale score has decrease in intensity from 8/10 to
6/10, but the pain increases with weight bearing and certain demonstrated movements. No
warmth of the skin of anterolateral aspect of the right ankle, slight ecchymosis and swelling are
noted. His anterior drawer test is positive, and his talar tilt is negative (number 31 until 33)
31. What is the classification of the injury due to its severity, and what stage of healing is this
patient in ? C
A. Grade I ankle sprain and inflammatory phase
B. Grade I ankle sprain and proliferative phase
C. Grade II ankle sprain and inflammatory phase
D. Grade II ankle sprain and proliferative phase
E. Grade III ankle sprain and inflammatory phase

32. In this case, what structure has been injured? A


A. Anterior talofibular ligament
B. Posterior talofibular ligament
C. Anterior tibiofibular ligament
D. Calcaneofibular ligament
E. Syndesmotic ligament

33. What programs are suited for this case? E


A. Resteind fixation
B. Stretching exercise
C. Endurance exercise
D. Proprioceptive and balance exercise
E. Weight bearing as tolerated and ROM exercise

34. The myoneuronal junction main neurotransmitter is D


A. Dopamine
B. Serotonin
C. Substance P
D. Acetyl choline
E. N-methyl-D-Aspartic Acid (NMDA)

435
35. Factors involved in the formation of contractures include D
A. a increase in the number of sarcomeres
B. a change from type I to type II muscle fibers
C. inadequate cross-linkage of collagen molecules
D. disturbed balance between synthesis and degradation of collagen
E. a decrease in the number of mitochondria

36. In a resting healthy man, the ejection fraction is approximately ? E


A. 0,1
B. 0,2
C. 0,3
D. 0,4
E. 0,5

37. The annulospiral ending is the primary afferent fiber coming from the A
A. Nuclear bag
B. Nuclear chain
C. Intrafusal fibers
D. Extrafusal fibers
E. Myonural junction

38. A 34 year old man has complete foot drop for 15 days, EMG showed fibrillation potentials
and reduced pattern of anterior tibial peroneus longus, extensor halucis longus and lumbosacral
paraspinal muscle but not in the gastrocnemius, Nerve conduction velocity of peroneal nerve is
normal. The interpretation is D
A. L2 radiculopathy
B. L3 radiculopathy
C. L4 radiculopathy
D. L5 radiculopathy
E. S1 radiculopathy

39. The vital capacity is the sum of : E


A. Inspiratory reserve volume plus tidal volume
B. Inspiratory capacity and functional residual capacity
436
C. Inspiratory capacity, tidal volume and expiratory reserve volume
D. Inspiratory capacity, tidal volume and functional residual volume
E. Inspiratory reserve volume, tidal volume and expiratory reserve volume

40. Which of the following describes “scapulo humeral rhythm’? A


A. 2/3 glenohumeral and 1/3 scapulothoracic
B. 1/3 glenohumeral and 2/3 scapulothoracic
C. 1/2 glenohumeral and 1/2 scapulothoracic
D. 1/4 glenohumeral and 3/4 scapulothoracic
E. 3/4 glenohumeral and 1/4 scapulothoracic

41. The most function of gluteus maximus during gait when A


A. Loading response
B. Terminal stance
C. Pre swing
D. Pre swing
E. Mid stance

42. The normal degree of femoral neck inclination in adult is : C


A. 75 degrees
B. 100 degrees
C. 125 degrees
D. 150 degrees
E. 175 degrees

43. One of these muscles in NOT a biarticular muscle : E


A. Iliopsoas
B. Quadricep
C. Hamstring
D. Biceps brachii
E. Gluteus maximus

437
44. In pronator teres syndrome all of the following muscles could be affected, EXCEPT A
A. Pronator teres
B. Pronator quadratus
C. Flexor pollicis longus
D. Flexor digitorum sublimis
E. Flexor digitorum profundus

45. The most active muscles at heel strikes of ipsilateral limb is D


A. Sartorius
B. Rectus femoris
C. Gluteus minimus
D. Gluteus maximus
E. Semimembranosus

46. Types of heat transfer via medium such as the movement of air of water is B
A. Evaporation
B. Convection
C. Conduction
D. Radiation
E. Electrical

47. The intensity of ultrasound prescription for tendonitis/bursitis is C


A. 01,-0,2 W/cm2
B. 0,3-0,4 W/cm2
C. 0,5-2,0 W/cm2
D. 2,5-3,0 W/cm2
E. 3,5-4,0 W/cm2

48. Patients with diagnosis Bell’s palsy of left side, six days after onset. Modalities that be
given were SWD of area retro auricular. Effect that expected is A
A. Decreased of stylomastoideus foramen edema
B. Increased of left facial muscle strength
C. Make facial massage more easy
438
D. Reduced facial asymmetry
E. Reduced Facial pain

49. Mrs V 30 year old diagnosed with cervical pain, radiating to the right arm and hand,
radiologic examination revised narrowing of foramen intervertebralis and paravertebra area C5
and C6. The initial treatment consist of... A
A. Diathermy, TENS, Cervical traction, and Neck Calliet exercise
B. Diathermy, TENS, and Neck Calliet exercise
C. Diathermy, TENS, and Low level laser therapy
D. Diathermy, TENS, and Cervical traction
E. Diathermy, TENS

50. A patient presents to your office with knee pain from a flare of rheumatoid arthritis. She has
a moderate effusion and warmth at her knee, the therapist wants to use ultrasound to her knee
for treatment. The consideration about using USD B
A. It help with pain
B. It is contraindicated
C. It is accelerates healing
D. It is increase flexibility
E. It will help reduce inflammation

51. Which therapeutic modality delivers medication to the side of pathology by promoting the
movement of charged particles through the skin under an imposed electrical field ? B
A. TENS
B. Iontophoresis
C. Phonophoresis
D. Low energy laser
E. Ultraviolet radiation

52. The function of medial T strap in AFO is to B


A. Pes equines control
B. Pes valgus control
C. Pes planus control
D. Pes varus control

439
E. CTEV control

53. Which one of the following is appropriate with Milwaukee orthosis ? E


A. Usually use for apex below Th 10 vertebrae
B. It is technically simple and do not require the service of a skilled person
C. Correction more effective for curve more than 45o during growth period
D. Less ventilation and more chest restriction than most other types of support
E. It should be used in conjunction with an exercise program both in and out of the
orthosis

54. Which used to treat back pain cause by L3 vertebrae fracture, the most important
contribution of thoracolumbosacral orthosis is made by the following statements, EXCEPT B
A. Prevent flexion movement
B. Increased intra-abdominal pressure
C. Effective fixation of the orthosis to the chest
D. Immobilization by the use of a three point system
E. Painful stimuli that provide reminders to change or maintain posture

55. The muscles required for ambulation using crutches are E


A. Triceps and biceps
B. Shoulder depressors and biceps
C. Biceps, triceps and wrist flexions
D. Shoulder depressors, biceps, wrist extensors
E. Shoulder depressors, triceps, wrist extensors, and finger flexors

56. Orthosis for Scoliosis which designed for correction of lumbar, thoracolumbar and low
thoracic curve spices at or below T10 is D
A. CTLSO
B. Milwauke Orthosis
C. The Miami Orthosis
D. The Boston Orthosis
E. The underarm Plastic Body Jacket

440
57. A 60 year old women with right medial knee pain has genu varum deformity that is
observed while standing and walking. What shoe modification can help her pain ? C
A. Rocker bottom
B. Medial wedge
C. Lateral wedge
D. Arch support
E. Shoe lift

58. Delay Onset of Muscle Soreness (DOMS) usually develop after D


A. Isotonic exercise
B. Isometric exercise
C. Isokinetic exercise
D. Eccentric exercise
E. Concentric exercise

59. Performing a seated leg exercise using quadriceps bench is an example of B


A. Closed kinetic chain exercise
B. Open kinetic chain exercise
C. Mixed kinetic chain exercise
D. Static kinetic chain exercise
E. Isokinetic exercise

60. Massage does NOT exerts its therapeutic effect of the following mechanism: C
A. Increased of muscle strength
B. Improved soft tissue blood flow
C. Stimulation of local opioid receptors
D. Mobilization of edema within soft tissue
E. Cutaneous stimulation of the autonomic nervous system

61. Conditioning exercise program is NOT result in the following mechanism : A


A. Decrease in metabolic activity of muscle
B. Increase in muscle strength
C. Increase in venous return
441
D. Increase stroke volume
E. Decrease in pulse rate

62. A 31 year old women was admitted with Guillian Barre Syndrome. Speech therapy is
priority decision because of C
A. Dependent for activities of daily living
B. Inability in communicate effectively
C. Difficulty in swallowing
D. Sputum retention
E. Anxiety

63. massage techniques used in sport medicine for the purpose of relieving soft tissue from the
abnormal grip of tight fascia is : A
A. Friction
B. Kneading
C. Efflorage
D. Vibration
E. Tappotement

64. The following below is NOT cause of the chronic ankle instability : B
A. Ligament laxity
B. Achilles tendinitis
C. Recurrent ankle sprain
D. Peroneal muscle weakness
E. Ankle proprioceptive deficit

65. Conservative rehabilitation for rotator cuff tear on recovery phase (up to 6 months) is to D
A. Reduced pain
B. Increased flexibility
C. Reduced inflammation
D. Upper extremity range of motion and proprioception
E. Reestablish nonpainful and scapulohumeral range of motion

442
66. Special problem associated with humeral shaft fracture is B
A. Ulnar nerve injury
B. Radial nerve injury
C. Axillar nerve injury
D. Medianus nerve injury
E. Musculocutaneous nerve injury

67. Test used to assess the contraction of the tensor fascia lata is A
A. Ober test
B. Fabere test
C. Thomas test
D. Trendelenberg test
E. Contra Patrick test

68. Treatment for rotator cuff tendonitis include all of the following. EXCEPT A
A. Immobilization
B. Restoration of proprioception
C. Restoration of range of motion
D. Control of pain and inflammation
E. Normalization of strength and dynamic muscle control

69. The rehabilitation program for spondylolisthesis include C


A. Should not restrict heavy lifting activity
B. Surgical treatment for grades 1 spondylolisthesis
C. Back flexion exercise program is more effective
D. Back extension exercise program is more effective
E. Anterior pelvic tilt exercise program is more effective

70. Extrasceletal manifestations of ankylosing spondylitis include A


A. Achilles enthesupathy
B. Tricuspid regurgitation
C. Nail pitting

443
D. Cataracts
E. Colitis

71. The examination for Achilles tendon rupture is E


A. Linel’s test
B. Suction sign
C. Talar tilt test
D. Homan’s test
E. Thomson’s sign

72. After a mastectomy, full active assisted shoulder ROM exercise should be done E
A. Post surgery day 1
B. Post surgery day 2
C. Post surgery day 3
D. Post surgery day 7
E. After removal of surgical drain

73. The patient may complain of numbness, paresthesias, or dysesthesis radiating to the first,
second, third, and lateral fourth digits. Symptoms may exacerbated during sleep and relieved
with wrist shaking. This condition found in B
A. Tunnel Syndrome
B. Carpal Tunnel syndrome
C. Pronator teres syndrome
D. Saturday night palsy
E. Guyon syndrome

74. A disorder of neuromuscular transmission due to an autoimmune response against. Ach


receptor post sinaptic membrane, associated with thymic disorder or thymic tumor with the
clinical presentation are proximal fatique and weakness is B
A. Lambert Latin syndrome
B. Myasthenic syndrome
C. Myasthenia gravis
D. periodic paralysis
E. botulism
444
75. Individuals with SCI are considered to be at risk for autonomic dysreflexia and orthostatic
hypotension if lesion on... A
A. T6 or above
B. T7
C. T8
D. T9
E. T10

76. Repetition is preserved in A


A. Transcortical sensory aphasia
B. Wernicke aphasia
C. Anomic aphasia
D. Global aphasia
E. Broca aphasia

77. A 27-year old women complaints of pain in the shoulder and suprascapular region 1 month
after undergoing modified radial neck dissection for carcinoma of the thyroid. Examination
shows decreased sensation to light touch over the superior aspect of the right trapezius and
scapular winging with resisted shoulder abduction. Which of the following nerve was most
likely transected during the procedure? B
A. Axillary
B. Long thoracic
C. Suprascapular
D. Dorsal scapular
E. Spinal accessory

78. The most cranial nerve involved in Miller Fisher syndrome (variant of GBS) is D
A. Facialis nerve
B. Abducent nerve
C. Trigeminal nerve
D. Oculomotor nerve
E. Olfactorious nerve

445
79. A 50-year-old women with secondary progressive multiple sclerosis, mild cognitive deficits
and severe lower-limb spasticity. What most effective treatment for this patient? C
A. Range of motion exercise
B. Warm water exercise
C. Intrathecal baclofen
D. Diazepam injection
E. Tizanidin

80. What is the mechanism of action of an intramuscular botulinum type A toxin in reducing
spasticity ? A
A. Blocks the release of presynaptic acetylcholine release at the neuromuscular
junction and end plate
B. Inhibits afferent fibers in the dorsal root of the spinal nerve
C. Competitively blocks the acetylcholine receptor
D. Destroys the neuromuscular junction
E. Acts as a GABA agonist

81. A 50-year-old man who had living independently suffers a left frontotemporal infarction.
Neurologic examination reveals non fluent aphasia and right hemiparesis, wheelchair
dependent ambulation. After w weeks on the rehabilitation clinic, he is noted to be progressing
very poorly from function viewpoint despite improving neurologic examination. The most
likely reason for poor progress is... E
A. Progressive dementia
B. Recurrent infarction
C. Subdural hematoma
D. Seizure disorder
E. Depression

82. A 14 year-old boy with haemophilia has had recurrent hemarthrosis of the knee in the past,
but now has full range of motion and equal sized quadriceps muscle measurement. His parents
ask advice regarding appropriate athletic activities. You advice for sports but advice against A
A. Competitive basketball
B. Competitive swimming
C. Competitive golf
D. Rowing
E. Biking
446
83. Protective and postural reflexes (advanced postural reactions) is C
A. Plantar grasp
B. Foot hand placement
C. Head and body righting
D. Symmetric tonic neck reflex
E. Asymmetric tonic neck reflex

84. A 9-year-old girl with C5 ASIA A spinal cord injury sustained 2 year ago is evaluated for
upper extremity splinting. Which statement regarding this scenario is TRUE ? D
A. Short hand splint should be prescribed to strengthen wrist extensor muscles
B. A resting hand splint should be prescribed for daytime use to preserve function
C. A long short hand splint be prescribed to strengthen wrist extensor muscles
D. A mobile arm support or balanced forearm orthosis could be prescribed to make
self-feeding possible
E. Wrist extension splint would be contraindicated for daytime use because it would
interfere with function

85. The child has chief complaint of bowlegs that are gradually progressing in severity and not
improving spontaneously. The deformity is called B
A. Clubfoot
B. Tibia vara
C. Knock knees
D. Genu valgum
E. Genu recurvatum

86. What is seen when the Landau reflex is elicited and when does it appear? : B
A. In the vertical position, the baby will support weight on his feet. Appears at 3-4 months
of age
B. In ventral suspension, baby will extend their hand & lower extremities. Appears 4-
5 months of age
C. In supine position, the baby will extend the arm and leg on the side that the head is
turned toward. Appears 5-6 months of age
D. In sitting position, the baby will extend the arm and hand to catch himself and prevent
falling to one side. Appears at 5-7 months of age

447
E. In standing position, the baby will extend the arm and hand to catch himself and prevent
falling to one side. Appears at 5-7 months old age

87. The parents of a 4-year-old boy with Duchenne Muscular Dystrophy (DMD) ask your
opinion about the future of their son. Which of the following statements is NOT correct? B
A. DMD typically becomes clinically evident at approximately 3 to 5 year of age
B. Weakness will beginning in the shoulder girdle muscles and later in the pelvis
C. Early difficulties noted are clumsiness, poor walking and frequent falls
D. Death is usually due to respiratory insufficiency
E. Intellect may be affected

88. Exercises for cardiovascular conditioning should involve the following components
EXCEPT A
A. Isometric training
B. Circuit training
C. Swimming
D. Jogging
E. Cycling

89. Exercise for diabetic patients is beneficial because B


A. Increase blood glucose level
B. Increase insulin sensitivity
C. Increase insulin resistance
D. Decrease HDL
E. Increase LDL

90. Physical activities advised to patients with CHF functional class III (NYHA) is : B
A. 1-2 METs
B. 3-4 METs
C. 5-6 METs
D. 7-8 METs
E. 9-10 METs

91. The respiratory function of a patient with ankylosing spondylitis characterized by : E


448
A. Decrease in diaphragmatic excursion
B. Increase in diaphragmatic excursion
C. Obstructive respiratory dysfunction
D. Increase use of accessory muscles
E. Diminished chest expansion

92. Which of the following is the common sources of afferent stimulation that result in
autonomic hyperreflexia : B
A. Bladder irritability
B. Rectal distension
C. Skin irritability
D. Stomachache
E. Headache

93. which physiological change occurs in the cardiovascular system with aging ? D
A. Increased FO2
B. Increased stroke volume
C. Increased resting heart rate
D. Decreased ejection fraction
E. Increased resting cardiac output

94. This is NOT the characteristic of geriatric patient : D


A. Multipathologic
B. Balance disturbance
C. Non specific clinical symptoms
D. No changes of functional status
E. Limited physiologic reserved capacity

95. Which of the following is the most important lifestyle modification for prevention of
osteoporosis ? D
A. Eating a diet high in protein
B. Eating a diet high phosphorus
C. Increasing the intake the alcohol

449
D. Avoiding cigarette smoking and high intake of caffeine
E. Minimizing the use of non-steroidal anti-inflammatory medications

96. The bisphosphonates alendronate and risedronate. D


A. Have excellent gut absorption
B. Have no gastrointestinal side effects
C. Act primarily by stimulating bone formation
D. Have been shown to reduce vertebral and nonvertebral fractures
E. Are no more effective on fracture reduction than hormone-replacement therapy,
selective estrogen receptor modulators and calcitonin

97. In older patients undergoing a rehabilitation program after a hip fracture, benefit from
weight bearing exercise can: C
A. Improve pain control
B. Improve gait pattern
C. Improve walking velocity
D. Decrease risk of prosthetic failure
E. Decrease incidence of hip dislocation

98. Which bony wrist injury related to forceful extension of the hand is commonly seen in
gymnasts? D
A. Ulnar fracture
B. Radius fracture
C. Lunate fracture
D. Scaphoid fracture
E. Hamatum fracture

99. The 30 year-old football player who present with knee pain, immediate swelling and
inability to bear weight after a fall, should undergo... A
A. Ice, elevation, compression and crutches as indicated
B. Ice, elevation, return to football playing the next day
C. Heat, NSAID, Ace wrap, return to football playing the next day
D. Cold compresses, acetaminophen, 5 days off from football playing
E. Heat wrap, knee immobilization, advantage weight bearing as tolerated
450
100. Competitive female athletes are at high risk of stress fracture if they have the female
athlete triad of... A
A. Amenorrhea, anorexia, osteoporosis
B. Amenorrhea, bulimia, poor flexibility
C. Bulimia, hypocalcemia, poor flexibility
D. Amenorrhea, hypocalcemia, osteoporosis
E. Amenorrhea, hypercalcemia, osteoporosis

ANSWER:
A: if numbers 1,2,3 are corrected
B: if numbers 1,3 are corrected
C: if numbers 2,4 are corrected
D: if only numbers 4 is correct
E: if all numbers are correct

101. Physical examination of someone with biceps tendinitis will typically reveal... D
1. absent biceps tendon reflex
2. decreased ROM of the elbow
3. tenderness over the lateral aspect of the arm
4. significant palpable tenderness at the bicipital groove

102. the characteristic(s) of Rheumatoid arthritis is/are... A


1. evidence of inflammatory by history, examination and laboratory test
2. symmetric, polyarticular, inflammatory arthritis
3. multiple joint involvement
4. history suggest mechanical pain

103. The following is/are the etiology of genu valgum. There is/are... B
1. foot pronation
2. femoral anteversion
3. compression medial aspect of the knee joint
4. shortened medial knee joint structure
451
104. A girl patient with idiopathic scoliosis the following are the signs that the Cobb’s angle
might be increasing rapidity... A
1. her pubic hairs are growing
2. her breast are growing bigger
3. she has her menarche a month ago
4. the Risser’s sign in reaching to grade 4

105. The inflammation of the thumb extensor pollicis brevis and abductor pollicis longus is
known as... B
1. de Quervain’s syndrome
2. carpal tunnel syndrome
3. tenosynovitis of the thumb
4. trigger finger of the thumb

106. Indication(s) of using orthotic in rheumatoid arthritis is/are... A


1. decrease joint motion-stabilization
2. decrease pain and inflammation
3. reduced weight trough joint
4. stop the deformities

107. Adhesive capsulitis is/are... A


1. associated with diabetes mellitus, inflammatory arthritis, prolonged
immobilization
2. more common in woman than men
3. usually an idiopathic condition
4. divided into 2 stage

108. Which following criteria(s) must be present for at least 3 weeks for diagnosis of
Rheumatoid arthritis? A
1. swelling in three or more joint areas
2. symmetric joint swelling
3. swelling in hand joint
4. rheumatoid nodule

452
109. A 50 year-old woman who reportedly had a right anterior cerebral artery stroke. In your
evaluation. You expect to find... C
1. distal greater than proximal upper-extremity weakness
2. lower limb greater than upper limb weakness
3. upper and lower facial weakness
4. language is typically not disturbed

110. A C7 quadriplegic patient should be expected to performs the activities below


independently, which are/is... A
1. sitting pivot transfer
2. sliding board transfer
3. wheel-chair to car transfer
4. floor to wheel-chair transfer

111. The goal of medical treatment on Parkinson’s disease is/are... C


1. decreased dopamine action
2. increased dopamine action
3. increased cholinergic effect
4. decreased cholinergic effect

112. Claw hand deformity and loss of sensibility on first until fifth finger in Morbus Hansen
disease caused by lesion on... B
1. ulnaris nerve
2. radialis nerve
3. medianus nerve
4. musculocutaneus nerve

113. The most common behavioral and personality changes seen following traumatic brain
injury is/are... E
1. attention deficit rigidity
2. visual perceptual deficit
3. spatial dysfunction
4. memory deficit

453
114. The damage to the superior branch of the facial nerve in leprosy will increase the chances
of ... A
1. blindness
2. exposure keratitis
3. damage to the cornea
4. sensory loss of the face

115. The following is/are the sign(s) of the autonomic dysreflexia in a T4 paraplegia ASIA A:
A
1. Pounding headache
2. nasal congestion
3. hypertension
4. tachycardia

116. The following is/are the clinical characteristic of Down syndrome: E


1. the head usually larger than normal because of overdevelopment of facial bone
and muscle
2. the toes are usually short and in the majority there is wide space between the first
and second toes
3. ligamentum laxity causing deficiency in grip strength and ankle strength that
develop in school age
4. most significant musculoskeletal impairment are owing to hypotonia and
ligamentum laxity caused by deformity of the joint

117. Characteristic(s) of normal locomotor development is/are B


1. wide to narrow base
2. lower limb more function initially
3. proximal before distal
4. extension pattern initially than flexion followed by isolated movement

118. The following are the characteristics of Duchenne’s Muscular Dystrophy E


1. The disease affects male
2. it is transmitted by X linked mode of inheritance
3. Measurement of serum creatinine kinase is the most reliable test
4. the child’s mother, his sister and maternal aunts are potential carriers
454
119. You are examining a child of 7 to 8 months of age. Persistence of which of the following
primitive reflexes or postural responses would you be concern about? D
1. placing
2. landau
3. parachute
4. asymmetric tonic neck

120. Consideration for bracing in children with Spinal muscular atrophy type II rare E
1. light weight braces
2. to provide functional
3. psychological benefit
4. fitting for this brace is best done under 2 year of age

121. A patient 5 years old was diagnosed Spastic Cerebral Palsy Diplegic type e.c prenatal
come to the Rehabilitation clinic with complaints could not walk and stand straight, both legs
crossed and tiptoe in standing position. what assessment is/are needed to determine spasticity of
the hip? E
1. Phelps Gracilis Test
2. Duncan Elly Test
3. Thomas Test
4. Tardieu Test

122. Man 55 year old, weight 85 kg, height 165 cm. He got myocard infarct 1 month ago. the
fasting glucose level 250, total cholesterol 400, no hypertension. The factor(s) that should be
considered to prescribe cardiac rehabilitation program is/are... C
1. avoid all of exercise
2. assessed the stress test
3. limitation of daily activity
4. reduce the risk factors for second cardiac disease attack

123. Indication for exercise test termination in Cardiac Rehabilitation is/are... E


1. moderately severe angina (+3 to +4)
2. sign of poor perfusion (palor, cyanosis)
3. Central nervous system symptoms (dizziness)
4. drop in SBP > 10 mmHg from resting value
455
124. Improve aerobic capacity and increased oxygen transport also result from exercising
is/are... C
1. decreased cardiac output
2. decreased resting heart rate
3. decreased ejection fraction
4. decreased systolic blood pressure

125. Goal(s) of pulmonary rehabilitation program in the patient with chronic obstructive
pulmonary disease (COPD) is/are... A
1. facilitating drainage of excessive bronchial secretion
2. improving the efficiency of the breathing patterns
3. overall physical reconditioning and well being
4. reversing the result of pulmonary function test

126. For chronic lung disease various walking test with sub maximal exercise have the benefit
below... A
1. to detect exercise intolerance
2. to detect functional limitation
3. to evaluate reconditioning program
4. to detect limitation in lung function

127. Goal(s) of the physical therapy program in the patient with chronic obstructive airway
disease is/are include... A
1. overall physical reconditioning
2. improving the efficiency of the breathing patterns
3. facilitating drainage of excessive bronchial secretions
4. reverse the pulmonary function with specific breathing exercise

128. The following assessment is/are to asses gait and balance: B


1. Romberg test
2. 6 minute walking test
3. the Get up and Go test
4. 12 minute walking test

456
129. Older adults are the least physically active group, the essential for healthy aging to
maintained regular physical activities is/are from of exercise type of... E
1. Balance exercise
2. Flexibility exercise
3. Endurance exercise
4. Muscle strengthening exercise

130. The following assessment should be done for geriatric patient with falls risk factor: E
1. Visual system
2. Polypharmacy
3. Foots problems
4. Environment factors

131. Pattern of aging on function strength and endurance is to... A


1. increases in fat and connective tissue within older muscle
2. reduction in the number of functioning motor units
3. reduce of type II fast-twitch muscle fiber
4. increased of type I slow-twitch muscle
132. What is/are the treatment for progress proprioception, coordination and agility during the
chronic phase of soft tissue healing in sport injury rehabilitation? B
1. balance activities
2. isotonic strengthening exercise
3. surface modification
4. isokinetics strengthening exercise

133. How does the anterior cruciate ligament (ACL) injury become torn? B
1. deceleration of the leg via quadriceps contraction combined with valgus and
external rotation forces upon a slightly flexed knee
2. sudden external rotation of hyperflexed knee
3. sudden hyperextension of the knee
4. direct blows to the knee

134. Medial stress of injuries of the elbow in throwing athletes, include(s)... A


1. ulnar nerve traction
457
2. avulsion of medial epicondyle
3. flexor muscle strain or tear
4. olecranon osteophyte formation

A basketball athlete had injured after he jump and landed on his right foot in the internally
rotate position. He heard “pop” and felt painful on his right knee. he ceased from this game
because he cannot walk and got an edema suddenly.
135. What is possible diagnoses for his condition? C
1. quadriceps muscle rupture
2. ACL rupture
3. PCL rupture
4. Meniscal torn

136. Where is/are the ground reaction force vector located at mid stance? B
1. anterior to ankle
2. posterior to ankle
3. anterior to knee
4. posterior to knee

137. Plantar flexor muscles is inactive by the time of... D


1. foot flat
2. heel strike
3. mid stance
4. toe off

138. Hamstring function as a... E


1. protectors of posterior structures from over stretching in combined movement of
hip flexion and knee extension
2. good extensors, rotators and weak adductors of the hip
3. rotators of the knee with the knee flexion
4. good flexor of the knee

139. The difference between type I in contrast to type II B skeletal muscle fibers is/are... B
1. type I are predominantly aerobic
458
2. type I are predominantly anaerobic
3. type IIB are predominantly anaerobic
4. type IIB fibers are high hemoglobin content

140. The following statement(s) about type I muscle fiber is/are correct : A
1. contain a large number of mitochondria and myoglobin
2. dominant in much of marathon runner
3. also called as slow twitch fibers
4. generate larger force

141. The following is/are the characteristic(s) of fibrillation waves... A


1. have very small amplitude
2. have very short duration
3. the rhythm is irregular
4. diving bomber sound

142. A variety of therapeutic gain(s) by electrical stimulation a muscle contraction is/are... B


1. muscle strengthening
2. recovery of neuropraxia
3. muscle pump contraction
4. prevents from Wallerian degeneration process

143. A 53-year-old woman with chronic venous disease has lower extremity edema, skin
hyperpigmentation, dermatitis, and venous varicosities. She does not have any ulceration. The
mainstay of treatment for this patient is/are... D
1. intermittent external pneumatic compression
2. hydrotherapy with warm water (100 degree F)
3. topical steroid cream
4. gradient elastic stocking

144. The effect(s) of ultra violet radiating is/are... B


1. photochemical effects
2. relief of muscle spasm

459
3. have biological effects
4. increases the pain threshold

145. Functionally, a transmetatarsal amputation is superior to a Lisfranc’s (tarsometatarsal)


amputation, because it provide(s)... E
1. an intact tibialis anterior insertion to oppose the pull of the triceps surae
2. the stump that is easier to fit
3. a broader base of support
4. a longer lever arm

146. Phantom pain... C


1. is an expression of wish fulfillment
2. does not occur with congenital absence of a limb
3. indicates an emotional maladjustment
4. is accentuated and perpetuated by delayed wound healing

147. The indication(s) for using of an ankle-foot-orthosis (AFO) to improve gait is/are... E
1. weak push-off at late stance
2. mediolateral instability at the ankle
3. passive plantarflexion in swing phase
4. foot drop at heel strike due to weak ankle dorsiflexor

148. Which activity(ies) is/are included in energy conservation method below? C


1. work with antigravity assisting
2. sit to work when possible
3. rest after fatigue
4. plan ahead

NATIONAL BOARD EXAMINATION JULI 2018


PHYSICAL MEDICINE AND REHABILITATION
460
CHOOSE THE MOST APPROPIATE ANSWER

1. Female, 38 y.o with knee pain, from physical examination founds swollen at left knee, warm
palpable and pain at anterior knee with VAS 6-7, what will you do as a physiatrist?
A. Do PRICE and HARM
B. Send to radiologist for x-ray
C. Give modalities treatment (heat)
D. Give some resistance exc and TENS
E. Do PRICE and give some analgesic

2. The clinical symptoms of degenerative joint disease of the shoulder are below,
EXCEPT…….
A. Pain may be nocturnal and relieved by rest
B. Tenderness on palpation on the anterior and posterior aspects of the shoulder
C. Limitation and pain on active and passive range of motion which affecting their ADLs
D. Manual muscle testing (MMT) may or may not be affected depending on the severity of
the disease
E. Pain more commonly occurred in external rotation of the shoulder, but may also
be seen in adduction motion

3. The level of transhumeral amputation which preferred for optimum prosthetic restoration
is….
A. Long arm residual limb (7-10 cm from the distal humeral condyle)
B. Short arm residual limb (one-third of bone length)
C. Very long arm residual limb
D. Very short arm residual limb
E. Medium arm residual limb
4. What is common “red flags” that identify the 10-20% of cases less likely to improve well?
A. Obesity
B. Osteoporosis
C. High education
D. Diabetes Mellitus

461
E. Presence of significant emotional
5. Brain plasticity represents the capability of the damaged brain to repair itself. Plasticity does
NOT influenced by……
A. Complexity of stimulation
B. Location of the lesion
C. Repetition of tasks
D. Environment
E. Motivation
6. Group of motor speech disorders characteristized by slow, weak, imprecise, or unccordinated
movements of speech musculature is called……
A. Apraxia
B. Aphasia
C. Stuttering
D. Dysarthria
E. Dysphonia
7. A glove and stocking pattern of sensory disturbance usually develops with disease in…..
A. Peripheral nerve
B.The spinal cord
C. The brainstem
D. The thalamus
E. The cortex
8. In TBI subjects, the FIM (Functional Independence Measure) is INADEQUATE in which of
the following area, that is………
A. Neuropsychological
B. Sphincter control
C. Locomotion
D. Mobility
E. Self care
9. In stroke patient, there are various function problems such as in below, EXCEPT…..
A. Neglect of the hemiplegic side
B. Missing autonomic reactions
C. have selective movement

D. Lack of balance

462
E. Sensory deficit

10. Infantile form of spinal muscular atrophy, is generally……..


A. Have underlying pathology of degeneration of posterior horn cells in the spinal cord
(ant horn cell)
B. Have high amplitude, long duration potensial on EMG examination
C. 30% of cases have normal nerve conduction velocity
D. Transmitted by autosomal dominant gene (autosomal resesif  cucurulo 824)
E. Hyperreflexia of physiological reflex

11. The indicator for good prognosis for ambulation in Cerebral Palsy children is…….
A. Children with spastic diparesis
B. Dependent sitting occurs by age 2
C. The absence of postural reactions at age 2 years
D. Ability to crawl on hands and knees by 1.5-2.5 years
E. Persistence of three or more primitive reflexes at 18-20 months

12. A 12 years old child has spina bifida. The L2 and L3 neurological level are spared. The
motor strength below L3 is fair. Which rehabilitation program is NOT suitable for this patient?
A. Bladder management
B. Bowel management
C. Gait training
D. KAFO
E. AFO

13. A 10 years old boy complaint frequent falls. He used to walk independently but recently
showed waddling gait. He is able to whistle. He has no problem studying at school. His
electrodiagnostic evaluation showed normal SNAP, low amplitude CMAP, fibrillation and
PSW in spontaneous activity, MUAPs low amplitude and area, and full screen fill in maximal
contraction. What is the diagnosis of this patient?
A. Fascioscapulohumeral Dystrophy
B. Becker Muscular Atrophy
C. Spinal Muscular Atrophy
D. Guillain Barre Syndrome
E. Polimyositis
463
14. A 5 years old girl with Cerebral Palsy diplegic. Her gait showed tightness on hip flexor and
hamstring, quadriceps weakness and excessive ankle dorsoflexion. What kind of gait she has?
A. Trendelenburg gait
B. Stiffed knee gait
C. Scissoring gait
D. Crouch gait
E. Jump gait

15. A 14 year old boy presents with a 3 months history of knee pain. The pain is most marked
in the area of the tibial tubercles bilaterally and it increases with activity. Physical examination
is unremarkable. What is the likely diagnosis?
A. Osgood-Schlatter disease
B. Patelofemoral arthritis
C. Rheumatoid arthritis
D. Patellar tendinitis
E. Jumper knee

16. Which schematic diagram illustrating idealized shapes of spirograms on spirometry


examination that suggest found in patient with Obstructive lung disease?

17. The indication for long term oxygen therapy in patient with COPd if…………
A. PH> 7.45
B. PaO2< 60 mmHg

464
C. PaO2< 55 mmHg
D. PaO2> 45 mmHg
E. PaO2> 50 mmHg

18. What is the expected outcome of effective exercise training in a pulmonary rehabilitation
program for patients with chronic obstructive pulmonary disease (COPD)?
A. Improved radiologic status of the lung fields
B. Slowed loss of forced expiratory volume
C. Reduced need for supplemental oxygen
D. Make lung anatomy became normal
E. Reduced dyspnea

19. A patient with restrictive lung disease and mild scoliosis from Duchenne muscular
dystrophy (DMD) develops nocturnal hypoventilation. What is the initial treatment?
A. Breathing control
B. Incentive spirometry every four hours
C. Nocturnal supplement allow-flow oxygen
D. Tracheostomy and nocturnal mechanical ventilation with positive pressure
E. Nocturnal noninvasive positive airway pressure and short periods of daytime
hyperinsufflation

20. Which finding correlates with a high risk of ischemic ulceration?


A. Serum albumin 3.2 gm/dl
B. Ankle Brachial Index of 0.40 (0.9-1.30)
C. Hemoglobin less than 12 g/dL
D. Stenotic lesion of the superficial femoral artery
E. Transcutaneous oxygen (TcPO2) of 50 mmHg

21. Adding consecutive quantities of air into lungs exercise is called…….


A. Air stacking exercise
B. Effective coughing exercise
C. Controlled breathing exercise
D. Segmental breathing exercise
E. Diaphragmatic breating exercise
465
22. Which cardiorespiratory outcome would you expect from an elderly person participating in
an aerobic program?
A. Improvement in VO2 max
B. No change in stroke volume
C. No reduction in systemic vascular resistance
D. No change in tolerance during activities of daily living
E. A decrease in VO2 max but an increase in walking speed

23. The most appropriate exercise training for elderly patients with peripheral artery
disease……..
A. Walking in intensity, progression from 50% of peak exercise
B. Intermittent walking to near minimal claudication pain
C. Isotonic strengthening exercise of lower extremity
D. Swimming one hour
E. Bicycling 30 minutes

24. The most risk factor for falls in the elderly is………
A. Poor balance
B. Hip deformity
C. Knee weakness
D. Ankle weakness
E. Use antidepressants

25. An elderly woman is being discharged from the acute rehabilitation unit after a stroke that
resulted in right hemiparesis. When ordered by a physician and medically justified, which
device is covered under medicare?
A. Front wheeled walker
B. Spasticity medications
C. Observation activity daily living
D. Dressing assistance in the morning
E. Transportation for medical appoinments

466
26. Dizziness and unsteadiness are frequent symptoms with aging. It is may cause by the
following factors, EXCEPT………
A. Increased the threshold of cutaneous sense
B. Decreased the threshold of joint position sense
C. Decreased the ability of CNS to perform sensory motor integration
D. Decreased the ability of musculoskeletal system to performed sensory motor
E. Increased the threshold of for detecting movement in the metatarsophalangangeal joint

27. What is the most common diagnosis in young female gymnasts with chronic back pain?
A. Spondylitis
B. Spondylosis
C. Spondylolysis
D. Spondylolysthesis
E. Retrospondylolisthesis

28. What is the advantage of using closed kinetic chain exercises during the early stages of an
anterior cruciate ligament (ACL) rehabilitation program?
A. Improved targeting of the vastus medialis
B. Limited knee extensor synergy activity
C. Decreased shear forces across the joint
D. Decrease oedema of the joint
E. Improved terminal stretch

29. A 23 yo soccer player athlete presents knee pain after injury 2 days ago. The injury is
impact to the front of the tibia with the knee flexed. Physical examination shows a positive “sag
sign”. Which structure is injured?
A. Medial collateral ligament
B. Anterior cruciate ligament
C. Posterior cruciate ligament
D. Medial meniscus
E. Popliteal muscle

30. One of the criteria for return to sport for patient with ACL after reconstruction is……..
A. Quadriceps bilateral comparison (90% or greater)
467
B. Acceleration rate at 90% of quadriceps peak toque
C. Quadriceps torque body weight ratio (75% or greater)
D. Functional hop test (80% or greater of contralateral side)
E. Hamstrings quadriceps (>60 % for males, > 70% for females)

31. Medical rehabilitation program of ankle sprain rehabilitative phase in 6 weeks post-injury
is……
A. Aquatic exercise
B. Motor relearning
C. Strengthening exercise
D. Aerobic exercise using static bicycle
E. Balance exercise using balance board

32. Nerve conduction is slow for the patient with…………


A. Myopathy
B. Upper motor neuron lesion
C. Axonal sensory neuropathy
D. Primary axonal degeneration
E. Demyelinated peripheral nerve

33. The structure below is NOT a members of the motor unit


A. Skeletal muscle
B. Motor end plate
C. Neuromuscular junction
D. Neuron of the anterior horn of spinal cord
E. Neuron of the dorsal horn of the spinal cord

34. The strong, protective, connective tissue surrounding bundles or fascicles of myelinated and
unmyelinated nerve fibers is……
A. Endoneurium
B. Perineurium
C. Epineurium
D. Cell body

468
E. Axon
35. Which muscle fibers transmit poorly localized, dull, visceral pain to the spinal cord?
A. Unmyelinated C
B. Myelinated beta
C. Myelinated A-delta
D. Myelinated A-alpha
E. Unmyelinated gamma

36. conduction velocity is….


A. The number of nerve fibers activated
B. The maximum voltage difference between two points
C. The time between the initiation of the electrical stimulus and the beginning of saltory
conduction
D. The speed an impulse travels along a nerve and primarily dependent on the
integrity of the myelin sheath
E. The chemical transmission of the signal across the neuromuscular junction to initiate a
single fiber action potensial

37. The structure that was a sense of change in the skeletal muscle tension is………
A. Troponin
B. Tropomyosin
C. Muscle spindle
D. Golgi tendon organ
E. Myotendineal junction

A male of 29 years old with post op right total hip replacement due to hip OA, on day 2 during
rehabilitation program he complain of sudden painful of the right hip and can not move his
right hip. X-ray photo shows a dislocation. The revised of hip arthroplasty was done. The day
one after the second surgery, he complain of pain until he can not move and follow
rehabilitation program. From x-ray foto it was found that he suffered a dislocation. The third
surgery was done and day 1 after the third surgery, he was sent back to Rehabilitation
Department (Question number 38 and 39)
38. What is the correct exercise to be done?
A. Avoid knee ROM exercise
B. Lying down for 4-6 weeks
C. Proper positioning exercise
469
D. Gradually sitting up position
E. Walking exercise with NWB using bilateral crutches

39. The ortopedics ask for a brace, what kind of brace you would like to give?
A. HKAFO with ischial bearing to reduce weight bearing
B. Hip and knee orthoses to avoid hip and knee motion
C. A hemispica brace to avoid hip motion
D. KAFO to help in walking activity
E. Unloader knee orthoses

40. A 50 year old man with type 2 DM presents for evaluation of a swollen ankle. An
examination demonstrates diminish pinprick sensation in a stocking pattern, absent ankle jerks,
and a warm, erythrmatous right ankle with a mild effusion. What is the most likely diagnosis
relating to the right ankle?
A. Charcot joint
B. Gout arthritis
C. Septic arthritis
D. Avascular necrosis
E. Rheumatoid arthritis

41. A 30 years old man presents with low back, knee and ankle pain of two months duration.
He also has had burning on urination. Physical examination reveals a rash over the palms and
soles. Sacroiliitis is present on radiolographs. The clinical diagnosis is………
A. Gonococcal arthritis
B. Psoriatic arthropathy
C. Ankylosing spondylitis
D. Generalized osteoarthritis
E. Reactive arthritis (Reiter disease)

42. What is the most common cause of lumbar stenosis?


A. Spondylosis
B. Disc herniation
C. Spondylolisthesis
D. Degenerative disk disease

470
E. Vertebral compression fractures
43. The following statement is the sign of calcaneovalgus foot
A. Excessive dorsoflexion at the ankle with eversion of the foot
B. Adduction of the forefoot, with the heel in normal position
C. Usually associated contracture of toe extensors
D. Equines or plantar flexion at the ankle
E. Usually seen in muscle dystrophia

44. “Waddling” gait may cause by………


A. Contralateral gluteus medius weakness
B. Ipsilateral gluteus medius weakness
C. Bilateral gluteus medius weakness
D. Adductor muscle contracture
E. Hip pain

45. Knee flexion mechanism involve rotation of tibia upon femur. Its mechanism is…..
A. Active due to muscle contraction
B. Active due to muscle relaxation
C. Passive due to cruciate ligament tightness and loose
D. Passive due to collateral ligament tightness and loose
E. Passive due to anatomical configuration of the articular surface

46. Extending the wrist from full flexion automatically flexes the DIP joint about joint about
PIP 5 degrees, and the MCP joint about 35 degrees is called……….
A. Tenodesis
B. Movement of wrist
C. Opposition of the thumb
D. Screw home mechanism
E. Extensor hood mechanism

47. The reason why supraspinatus most vulnerable to excessive wear is……
A. Creates mechanical advantage for 1:10
B. Deltoid muscle do not shares the demand for supraspinatus
471
C. Has an internal moment arm for shoulder abduction about 1 cm
D. High forces generated in short period could cuse a tear to muscle tendon
E. Assist deltoid during abduction and provides dynamic and static stability to
glenohumeral joint

48. the following statement is the reason why cervical vertebra very prone to injury
A. Cervical vertebra connected to the head that have a bigger size and heavier weight
compare with cervical vertebra
B. The area of the corpus on cervical vertebra is smallest compare to the other vertebra
C. Cervical vertebra have specific movement that we call “YES and no motion”
D. The joint on the cervical vertebra have 3 degree freedom of mpotion
E. Cervical vertebra is the end of “open kinematic chain”

49. A first time stroke patient after two months of onset show good recovery. The patients
begin to activate muscles selectively outside the flexor and extensor synergies were
characteristic for Brunsstrom stages of motor recory in stage…………
A. 1
B. 2
C. 3
D. 4
E. 5

50. A 21 years old female, unmarried, paraplegia was referred as inpatient rehabilitation ward
due to T5 burst fracture. Stabilization with PPSW was done and now was medically stable.
Physical examination: vital signs and general health status were within normal limits. What
should you examine when you have to determine whether the patient has a complete or an
incomplete lesion?
A. Anal sensation and voluntary anal sphincter contraction
B. Anocutaneal and bulbocavernous reflexes
C. Myotome and bulbocavernous reflexes
D. Physiological reflexes
E. Pathological reflexes

51. You are evaluating a patient with hand problem. During evaluation, the patient shows a
positive Froment’s sign. Which nerve is injured?
A. Ulnar
472
B. Radial
C. Median
D. Ulnar and radial
E. Ulnar and median

52. Which type of aphasia is characterized by nonfluency, an intact ability to repeat, and intact
comprehension?
A. Broca
B. Anomic
C. Wernicke
D. Transcortical motor
E. Transcortical sensory

53. The temperature and pain sense loss usually develops with disease in…..
A. Posterior columns of spinal cord
B. Posterior horn of spinal cord
C. Anterior horn of spinal cord
D. Lateral horn of spinal cord
E. Cerebellum

54. The following statements in the therapeutic application of cryotherapy, EXCEPT in……..
A. An acute of musculoskeletal diseases
B. Chronic polyarthritis
C. Joint contracture
D. Sport injuries
E. spasticity

55. Massage exerts its therapeutic effects through all the of following mechanism, EXCEPT….
A. Decreased perspiration and secretion of cebaceous glands
B. Cutaneous stimulation of the autonomic nervous sytem
C. Mobilization of edema within soft tissue
D. Improved soft tissue blood flow
E. Release of endogenous opiates
473
56. One of the condition below are NOT the contraindication on using traction
A. Vertebrobasilar atherosclerosis disease
B. Ligamentous instability
C. Metastatic bone disease
D. Radicular pain
E. Osteoporosis

57. The general contraindications of electrotherapy include the following, EXCEPT….


A. Stimulation over carotid sinus
B. Stimulation across the heart
C. Thrombophlebitis
D. Dermal wounds
E. Malignancy

58. Which statement is NOT TRUE regarding ultrasound diathermy?


A. Should not be used in laminectomized spine
B. Must be moved constantly to avoid endothelial damage
C. Must be used with caution in the presence of metal implants
D. Should not used in the presence of methylmethacrylate cement
E. Phonophoresis uses direct current to drive medications through the skin ?

59. the following condition is contraindication for lumbal traction. There is….
A. Ischias syndrome
B. Lumbar disc herniation
C. Postural low back pain
D. Restrictive lung disease
E. Paralumbal muscle spasme

60. A boy with CP spastic quadriplegic with scissoring position of lower extremities. From
evaluation showed difficulty to fastly move the hip into abduction with hip on extension
position. The fast movement can occur smoothly with hip on flexion position. Which muscle
was injected for this program with the botulinum Toxin A type?
A. Iliopsoas muscle
474
B. Rectus femoris muscle
C. Adductor longus muscle
D. Medial hamstring muscle
E. Adductor magnus muscle

61. A toddler still unable to sit independently. She had history of jaundice and showed slow
writhing involuntary movements. She was diagnosed as Cerebral Palsy. What kind of CP this
child has?
A. Athetoid type
B. Chorea type
C. Spastic type
D. Mixed type
E. Ataxic type

62. This reflex is best elicited by a sudden dropping of the baby’s head in relation to its trunk
A. Vertical suspension
B. Extensor thrust
C. Righting reflex
D. Landau reflex
E. Moro reflex

63. Spina bifida is considered a primary failure of neurulation and the spinal cord defect
associated with the anatomic level of the lesion and neurologic level of functional involvement.
A boy suffered from spina bifida L4-L5 segment, the preserved muscle function is…….
A. Iliopsoas muscles
B. Hamstring muscles
C. Quadriceps muscles
D. Foot intrinsic muscles
E. Gluteus maximus muscle

Children two years of age have stereotypic and ritualistic behavior with impairment in social
interaction, communication and imaginative play. (Question number 64 & 65)
64. The diagnose is………
A. Fragile x syndrome

475
B. Mental retardation
C. Down syndrome
D. ADHD
E. Autism

65. That above case have also clinical features such….


A. Flat feet
B. In attention
C. Cognitive impairment
D. Executive dysfunction
E. A lack of eyes contact and difficulty playing with others

66. During ambulation using a transfemoral prosthesis, a patient raises his entire body with
plantar flexion of the sound foot during the swing phase of the prosthesis extremity. To
decrease the vaulting, what should you do?
A. Lengthen the prosthesis
B. Add a kneeextension aid
C. Tighten the socket suspension
D. Set the knee joint to assist flexion
E. Set the prosthesis foot in a few more degress of plantar flexion

67. a patient with osteoarthritis present with pain in the carpometacarpal joint of the thumb.
What is the most appropriate splint?
A. Ring
B. Thumb spica
C. Resting hand
D. Spring coil flexion assist
E. Spring coil extension assist

68. The proper length for a cane should be measured with the elbow in which position?
A. Full extension
B. 20 degrees of flexion
C. 45 degrees of flexion
D. 60 degrees of flexion
476
E. 90 degrees of flexion

69. In an amputee with a PTB prosthesis and SACH foot, incordinately delayed knee flexion at
heel strike may due to…….
A. Poor hip extension
B. Anterior socket tilt
C. Too soft a heel cushion
D. Excessive dorsoflexion of the foot
E. Discomfort over the anterior the tibia

70. A plastic ankle foot orthosis with a rigid ankle is indicated in patients with……..
A. Weak or absent dorsoflexors without severe weakness of the plantar flexors
B. Weak or absent dorsoflexors and plantar flexors
C. Weak or absent knee extensors
D. Absent to moderate spasticity
E. Plantar flexion contractures

71.Which of the following components of a KAFO would stabilize the knee, while applying
force to pressure tolerant tissue in the best way?
A. A knee cap strap
B. A single suprapatellar strap
C. A single patellar tendon strap
D. Combined suprapatellar and patellar tendon straps
E. A lower thigh band closure in combination with a calf band closure

72. Following surgery a patient is asked to perform bottle blowing exercises to maintain
pulmonary function. Which of the following is the most common side effect of the bottle
blowing exercise?
A. AV Block
B. Atrial fibrillation
C. Rupture of the trachea
D. Pulmonary embolism
E. Premature ventricular beats

477
73. A 56 yo female suffered from obesity (BMI 29) and hypertension (BP 135/85). What the
best exercise regimen for this woman?
A. Continue walking exercise while blood pressure exceeds 220/110, because the exercise
will decreased the blood pressure
B. Arm curl with dumbbell for biceps strengthening, with intensity 80% of 1 RM, 8
repetitions, 3 bouts/session, 2-3x/week
C. Leg extension exercise with quadriceps bench, with intensity 80% of 1 RM, 8
repetitions, 3 bouts/session, 2-3x/week
D. Fast walking exercise, with intensity 40-70% of heart rate reserve, 30-60 minutes/day,
3-5x/week
E. Static cycle exercise, with intensity 40-70% of heart rate reserve, 30-60
minutes/day, 3-5x/week

74. The following is causes of restrictive pulmonary disease, EXCEPT…….


A. Guillain Barre Syndrome
B. Chronic bronchitis
C. Myasthenia gravis
D. Pleural effusion
E. Obesity

75. Patient cardiac disease results in slight limitation on physical activity. They are comfortable
at rest. Ordinary physical activity results in fatique, palpation, dyspnea, or angina pain. Patient
can perform to completion any activity requiring ≥ 5 METs, but cannot perform to completion
of activities requiring ≥ 7 METs. Sexual intercourse to completion without interruption, walk at
4 mph on level ground. The Cardiac Functional Classification is……………
A. NYHA Class I
B. NYHA Class II
C. NYHA Class III
D. NYHA Class IV
E. NYHA Class V

76. Man, 70 yo, came to emergency with short of breath since two days before. He was heavy
smoker for 20 years and has been stopped 10 years ago because of short of breath and
recurrently hospitalized 2 years recently, history of past illness: no diabetic, no heart disease.
After acute state was overcome, he was reffered to pulmonary ward and consulted to
rehabilitation department. What the proper program for this patient?
A. Gradual mobilization
B. Breathing control and relaxation
478
C. Relaxation technique, gradual mobilization
D. Breathing control and removal technique secretion
E. Breathing control, pursed lip breathing, when the short breath relieve progress to
diaphragmatic breathing, removal technique secretion if needed, gradual
mobilization

77. Woman, 24 yo consulted from surgery ward with post ASD closure day two after
extubation. The hemodynamic was stable. In this condition, the prescription of rehabilitation
program is…………..
A. Deep breathing, secretion removal by effective coughing
B. Deep breathing, effective coughing, strengthening exercise
C. Gradual sitting with support if tolerable by the patient progress to active sitting, ,
strengthening exercise, effective coughing, active range of motion, ankle pumping
D. Gradual sitting with support if tolerable by the patient pogress to active sitting,
breathing exercise, effective coughing, active range of motion, ankle pumping
E. Gradual sitting with support if tolerable by the patient pogress to active sitting,
strengthening exercise, breathing exercise, effective coughing, active range of motion,
ankle pumping

78. The conservative treatment of scoliosis is bracing and specific exercise. The following are
the factors that influence the result in brace treatment, EXCEPT……..
A. Compliance to treatment
B. Dosage of wearing
C. Quality of bracing
D. The Risser sign
E. Family history

79. The following statement is NOT TRUE regarding strengthening exercise


A. Circuit weight training are carried out in a specific sequence using a variety of exercise,
could include 8 to 10 RMs, with a 30 second to 1 minute rest period between each bout
B. Brief repetitive isometric exercise (BRIME) regimen uses 5 to 10 brief but maximum
isometric contractions performed against resistance 5 days per week
C. The rule of tens by Daves uses 10 sets of 10 repetitions of 10 second contraction every
10 degrees in the range of motion
D. Isokinetic strengthening exercise is a dynamic exercise performed with variety
angular joint velocity
E. Regressive resistive exercise is the reverse of the DeLorme technique

479
80. Which of the following statements is TRUE regarding endurance exercise?
A. Anaerobic endurance exercise are performed to deplete the creatine system
B. A rating of 12 to 13 on the 20 point scale corresponds to about 85% of maximum HR
C. Aerobic endurance training uses small groups of muscles to increase the O2
comsumption
D. Aerobic training for longer durations and at lower intensities for 4 to 5 days per
week may be prescribed to maximize the weight –controlling effect
E. When the MET equivalent is used for exercise prescription, it is advisable to begin
exercise with an initial target range of 70 to 80% of the maximal MET level

81. Brief, explosive maneuvers that consist of an eccentric muscle contraction followed
immediately by a concentric contraction is called………….
A. Ballistic exercise
B. Plyometric exercise
C. Proprioception exercise
D. Passive stretching exercise
E. Neuromuscular fascilitation exercise

82. Muscle hypertrophy occurs at how many weeks of resistance training?


A. 4-5 weeks
B. 5-6 weeks
C. 6-7 weeks
D. 7-8 weeks
E. 8-9 weeks

83. The following statement is TRUE regarding clinical (bedside) swallowing studies (CSS)
A. Patient must have an intact cough reflex
B. It is capable of detecting “silent” aspiration
C. It is reliable as videofluoroscopic swallowing studies
D. This test is the gold standar in determining the safety of oral feeding
E. To reduced the risk of aspiration, the patient must upright for at least 5 minutes after the
CSS

84. Gait in elderly is characterized by……….

480
A. Decreased velocity
B. Longer stride length
C. Broader base of support
D. Increased single limb support
E. Not associated with falls and the fear of falling

85. Sarcopenia in elderly may also affect respiratory muscle strength, that would lead to……
A. Decreased residual volume and functional capacity
B. Decreased lung compliance and thoracicn wall mobility
C. Increased residual volume and functional residual capacity
D. Increased lung compliance and decreased thoracic wall mobility
E. Preserved lung compliance and decreased thoracic wall mobility

86. After Alzheimer disease, the most frequent dementia in the elderly is secondary to……..
A. Drug toxicity
B. Multiple infarcts
C. Metabolic causes
D. Subdural hematoma
E. Occult hydrocephalus

87. A 75 yo man with a history of hypertension, type II diabetes mellitus and mild sensory
neuropathy who is currently a nonsmoker wishes to begin an exercise program. What would be
the safest initial modality?
A. Walking with no competition
B. Treadmill walking at 1.5 mph on level surface
C. Stair stopper for 15 minutes without resistance
D. Lifting 1 to 3 pound weights overhead while seated
E. Machine based quadriceps extensions at 80% maximum weight

88. What is the most common cause of knee pain in runners?


A. Baker cyst
B. Discoid meniscus
C. Hamstring tendinitis

481
D. Iliotibial friction syndrome
E. Patellofemoral pain syndrome
89. A 17 yo football players sutained a noncontact knee injury while planting his leg to make a
cut. He heard a pop and felt his knee buckle. What is the most sensitive clinical test to establish
the diagnosis?
A. Thomas
B. Lachman
C. McMurray
D. Anterior drawer
E. Posterior drawer

90. The following statement is TRUE about plyometric exercise


A. It is have no risk to injury
B. Functional type of exercise
C. The principle is slow stretching
D. Is useful to many neurologic impairment
E. Exercise using this principle incluiding balance training

91. Ruptur of the profundus tendon (flexor digitorum) which most commonly due to trauma as
seen un athlete (foot ball, wrestling) is said…………..
A. Bennet’s fracture
B. Rolando’s fracture
C. Boxer’s fracture
D. Mallet finger
E. Jersey finger

92. A common term for patella tendinopathy is……..


A. Cho-Pat strap
B. Jumper’s knee
C. Patella Tendon Rupture
D. Osgood Schlater disease
E. Sinding-Larsen-Johansson disease

482
93. According to the standart of the American Spinal Injury Association, what is the key area
for testing pin prick sensation to determine the sensory level after T4 spinal cord injury?
A. Umbilicus
B. Acromion
C. Nipple line
D. Apex of axilla
E. Lateral aspect of the elbow

94. The following statement about Rheumatoid Arthritis is TRUE


A. Female: male ratio 1:3
B. Affects the synovial membrane of multiple joints
C. The joints mostly affected are shoulders and hips
D. The cause of “piano key sign” is the ulnar head “floats up” anteriorly in the wrist
E. Atlanto axial subluxation is best detected on x ray when a lateral view of the neck in
extension

95. The following test is the special test for tear of supraspinatus tendon
A. Hawkins impingement test
B. Neer impingement test
C. Apley’s scratch test
D. Yergasson test
E. Drop arm test

96. Which nerve is most commonly injured with traumatic anterior shoulder dislocation?
A. Radial
B. Axillary
C. Long thoracic
D. Thoracodorsalis
E. Musculocutaneous

A 56 yo right handed secretary presents to the emergency room with the sudden onset of
instability to speak and right hand clumsiness. Her symptoms began abruptly in the morning
while returning from the bathroom. Physical examination found the patient is alert. BP 150/90
mmHg; P 110/min; the rhytm is irregular. She is able to follow simple command and appears to
comprehend conversational speech, but her response are effotful and limited to simple one
483
word answers. She is unable to read aloud and can only write her name with difficulty. The
laboratory studies reveal a normal hematocrit, normal electrolytes and glucose. Her
electrocardiogram shows atrial fibrillation with a ventricular rate 90 to 120/min. a head
computed tomography (CT scan) obtained that evening is unremarkable. Within 2 days, her
speech has become more fluent, although she continues to have subtle word finding difficulties.
(Questions number 97&98)

97. What would you characterize the patient’s difficulty in communication?


A. A non fluent motor aphasia
B. A fluent sensory aphasia
C. Conduction aphasia
D. Global aphasia
E. Apraxia verbal

98. Why is her CT scan normal?


A. The infarction is too small
B. The infarction can only be seen in brain MRI
C. Because it is TIA
D. The cause of speech problem is not in the brain
E. The time interval from symptom onset to the CT scan is relatively short (less than
24 hours)

99. Exercise programs for patients with Parkinson’s disease have been shown to……..
A. Reduce the frequency of falling
B. Reduce tremor more than bradykinesia
C. Have no effect on activities of daily living
D. Produce a short term improvement in ambulation ability
E. Improve motor function primarily in more severely involved patients

100. A 45 yo man presents with a several month history of weakness in his lower and upper
extremities. On examinitaion, in addition to weakness in multiple muscles groups, he
demonstrates atrophy, hyperreflexia, spasticity of the legs, and bilateral Babinski sign.
Fasciculation in multiple muscles are also noted. His sensation to pain, temperature, and joint
position sense appear intact. What is his most likely diagnosis?
A. Amyotrophic lateral sclerosis (ALS)
B. Anterior spinal artery syndrome
484
C. Brown sequard syndrome
D. Vitamin B 12 deficiency
E. Central cord
ANSWER: A: IF NUMBER 1,2,3 ARE CORRECT
B: IF NUMBER 1 AND 3 ARE CORRECT
C: IF NUMBER 2 AND 4 ARE CORRECT
D: IF ONLY NUMBER 4 IS CORRECT
E: IF ALL NUMBER ARE CORRECT

101. “Scottie dog” appereance can be shown in what position (s) vertebral x-ray plane?
1. Antero posterior
2. Pstero anterior
3. Lateral
4. Oblique

102. Besides rotator cuff disease, what is another differential diagnostic possibility must be
considered in evaluating shoulder pain
1. Suprascapular nerve entrapment
2. Cervical facet syndrome
3. Cervical radiculitis
4. Shoulder instability

103. What are the most common types of bursitis around the hip
1. Ischial (tailor’s or weaver’s bottom)
2. Iliopsoas (iliopectineal)
3. Greater trochanteric
4. Supragluteal

104. The following statements is/are the major criteria in determining amputation level in the
vascular limb
1. Palpable pulses at the next more proximal joint have clinical significance only
when present, and their presence is a very positive indication of the likehood of healing at
any given level

485
2. The degree of loss sensory is significance in the diabetic patient whose ischemic.
Process is frequently accompanied by peripheral neuropathy
3. Skin temperature is a representative measure of collateral circulation
4. Dependent rubor indicates marginal viability of the skin
105. The following symptom/s is/are the most common in cauda equina syndrome, there
is/are……….
1. Burning ache in legs
2. Bladder dysfunction
3. Flaccid paraplegia in legs
4. Anesthesia in lower extremities and perineum

106. Vertebrobasilar system involvement may present with any combination of the following
sign and symptoms such as…….
1. Abnormalities of motor function often bilaterally
2. Ipsilateral cranial nerve function
3. Symptom of vertigo
4. Cognitive deficits

107. Tricyclic antidepressant are useful for patients having…. A/E


1. Chronic low back pain
2. Migraine headache
3. Tension headache
4. Neuropathic pain

108. Which type a aphasia have fluent characteristics? C


1. Broca’s
2. Anomic
3. Global
4. Conduction

109. The high risk factor(s) of neural tube defect (NTD) is/are the following E
1. Mother who taking valproic acid
2. Taking antiretroviral medication
3. Failure of neurulation
486
4. Maternal diabetes

110. The characteristic of Trisomy 21 is/are……….. A


1. Increases distance between 1st and 2nd finger
2. Clindactyly of 5th finger
3. Mongoloid slant of eyes
4. Polydactyly

111. The characteristic of clubfoot is/are……… B


1. Metatarsal 1 is more in plantar flexion
2. Inversion of hindfoot
3. Hindfoot equines
4. Midfoot varus

112. Indication for six minutes walk test is/are……. E


1. Functional status assessment of peripheral vascular disease
2. Pretreatment and posttreatment comparisons
3. Predictor of mortality and morbidity
4. Pulmonary rehabilitation evaluation

113. The effects of pursed lip breathing is/are……… C


1. Increase respiratory rate
2. Decrease respiratory rate
3. Decrease tidal volume
4. Increase tidal volume

114. Patient with congestive heart failure who is participated in cardiac rehabilitation program
will improve which parameter(s)? D
1. Diastolic Ventricular filling
2. Sydtolic ejection Friction
3. Oxygen saturation
4. VO2max

487
115. The following is/are the rehabilitation treatment of vertigo and dizziness in the elderly A
1. Modification of life style and home environment
2. Correct the internal representation of posture
3. Improve ankle, hip and neck mobility
4. Sedative drugs

116. Implementation of Comprehension Geriatric Assesment (CGA) include……. E


1. Identification of patient resource and strength
2. Evaluation in patient’s multiple problems
3. Multidisciplinary evaluation
4. Coordinated care plan

117. elderly patient with hypoglycemia may experiencing……….. E


1. Tachycardia
2. Diaphoresis
3. Lightheadedness
4. Visual disturbances

118. Which of the following is/are the component of the management postmastectomy
lymphedema? A
1. Manual lymphatic drainage
2. External compression
3. Surgical management
4. Loop diuretics

119. Stress testing to assess balance for athelete is/are……….. D


1. Sergeant Jump
2. Ruler Drop Test
3. Edgren Side Step Test
4. SEBT (Star Excursion Balance Test)

A 24 yo baseball player suffered from right shoulder pain after technical drill training session,
2 weeeks ago. The pain became worsen especially when he made upper head throwing

488
movements, usually occurred after 50 repetitions. He also felt fatique more easily. The pain was
on the anterolateral right shoulder, so crepitation (Question no 120-122)
120. From the history, what the possible anatomical structure/s could be injured? B
1. Bursa
2. Cartilage
3. Tendon
4. Ligament

121. If near, Hawkins and lift off test positive in this case, what specific structure(s) will be
involved? B
1. Supraspinatus tendon
2. Infraspinatus tendon
3. Subscapular tendon
4. Teres minor tendon

122. From shouder USG, the one of findings was a partial thickness tear of supraspinatus
tendon. What is the most proper modality shoud be given? D
1. Electrostimulation
2. Ultrasound diathermy
3. Short wave diathermy
4. Low Level LASER Therapy

123. Which of the following is/are NOT typical characteristic of motor unit action potensial
observed during electromyography of patients with myopathy? C
1. Short duration
2. Long duration
3. Low amplitude
4. High amplitude

124. The following is/are the typical characteristics of motor unit action potensial observed
during electromyography of patients with motor neuron disease A
1. Reduced motor unit recruitment
2. High amplitude
3. Long duration

489
4. Low amplitude

125. The step of examination in rehabilitation management for the child with Muscular
Dystrophy include……….. E
1. History with family concern
2. Assistive and adaptive device
3. Aerobic capacity and endurance
4. Community and school/play integration

126. A 6 yo girl with spastic quadriparesis needs a new wheelchair prescription. Patient factors
that influence the prescription include of the following(s) A
1. Presence of lower limb spasticity
2. Degree of trunk control present
3. Patient’s weight and height
4. Presence of Moro reflex

127. The following statement(s) is/are regarding idiopathic scoliosis B


1. Genetic factor is a sex-linked trait with incomplete penetrance and variable
expressivity
2. A sex-linked trait can be transmitted by a father, to either a son and daughter
3. The curves have strong tendency to progress rapidly during the adolescent growth
spurt
4. Juvenile idiophatic scoliosis occurs between ages 2 and 8

128. Tenodesis action is………. E


1. Extending the wrist from full flexion automatically flexes the DIP joint about 20
degress, the PIP joint about 50 degrees, and the MCP joint about 35 degrees
2. Position of the wrist significantly alters the length and subsequent passive tension
in extrinsic flexors muscles
3. Polyarticular muscle stretching across one joint generates a passive movement at
other joints
4. Having an important role in tetraplegic patient

129. Steppage gait may caused by…… A


1. Severely weak dorsiflexors

490
2. Plantarflexor spasticity
3. Equinus deformity
4. Weak quadriceps

130. Which of the following are suggested order of muscle tests on sidelying? A
1. Hip adductor
2. Gluteus medius
3. Gluteus minimus
4. Gluteus maximus

131. You are in the duty of one patient with diagnosis SCI C5 AIS A. the nurse call you to
inform that your patient feels very ill and cold sweating, the blood pressure was 170/100 mm
Hg. HR 60/min, respiration 22/min, temperature 37.5°C. your analysis is the patient suspected
having…….. A
1. Urinary infection
2. Bladder dysfunction
3. Autonomic dysreflexia
4. Blood hypertension

132. The physical medicine and rehabilitation programs for people with leprosy is/are……E
1. Oil massage
2. Active exercise
3. Soaking the feet in the water
4. Protection of the hand and feet

133. According to ICF (International Classification of Functioning, Disability and Health) in


stroke patient, the following is (are) include in the loss of body functions and structures
dimension. A
1. Shoulder hand syndrome
2. Cognitive impairment
3. Visuospasial neglect
4. Difficulty in ADL

134. Laser therapy has been proven effective in the treatment of……. D

491
1. Osteomalacia
2. Joint contracture
3. Rheumatoid arthritis
4. Myofascial syndrome

135. The following is/are the contraindication of short wave diathermy E


1. Urinary bladder stimulators
2. Moist wound dressing
3. Electrophrenic pacers
4. Contact lenses

136. The efeectiveness of cervical traction is dependent on many factors, including the amount
of force apphed. Other factors include………. E
1. Angle of pull
2. Position of the patient
3. Position of the cervical spine
4. Constant or intermitten traction

137. A deficiency of pulmonary surfactant would………. D


1. Decrease the change in intrapleural pressure required to achieve a given tidal volume
2. Increase functional residual capacity (FRC)
3. Decrease surface tension in the alveoli
4. Decrease lung compliance

138. What would you recommend to your patient with insensate feet? D
1. Walking barefoot
2. Vitamin C in large doses
3. Use of hiking boots for long walks
4. Restriction of duration of walking time

139. Which of the equipment(s) we needed for 6 minutes walking test? E


1. Area which is needed for walk minimal 30 meter
2. Emergency equipment
492
3. Finger oximetry
4. Stop watch

140. The prehensive function of a myoelectric hand stimulates which grip pattern(s)? D
1. Power
2. Lateral
3. Spherical
4. Three jaw chuck

141. The following is/are adverse effect of spinal bracing A


1. Excessive spinal motion above and below brace
2. Dysphagia and difficulty chewing
3. Axial muscle wasting
4. Posture erect

142. Which of the statement(s) is/are the correct statement about splinting? A
1. Early implementation of splint will influence on collagen orientation of developing
fibers in wound healing process
2. The indications are soft tissue of skin graft protection, anti deformity, positioning
and tissue lengthening
3. Splinting is essential to prevent and treat joint contractures and deformities
4. An optimal splint wearing schedule should not be monitored and modified

143. The clinical feature(s) of myofascial pain include the following statement(s) E
1. Continuous, dull, deep aching pain
2. Pain relief by inactivating trigger area
3. Pressure on hands in muscles reproduces pain
4. Local muscle twitch produced by trigger point stimulation

144. The red flag sign(s) in the low back pain patients include…….. E
1. Decrease of body weight significantly
2. Pain profound at night time
3. Age more than 50 yo

493
4. Fever

145. Joint protection technique in patients with osteoarthritis include the following principle(s)
E
1. Use appropriate assistive devices
2. Avoid activities that hurt affected joints
3. Avoid staying in one position for too long
4. Use largest and strongest joint and muscles

146. In the later disease of rheumatoid arthritis, the specific articular manifestation include E
1. Button hole deformity of fingers
2. Swan neck deformity of fingers
3. Z deformity of the thumb
4. Finger drop

147. the popular program for progressive resistance exercise are………. A


1. Oxford
2. De Lorme
3. DAPRE method
4. Karvonen method

148. Which equipment(s) can be provided in ROM exercise when we want to reducing the
effect of friction and gravity of the moving limb? D
1. Suspension
2. Shoulder wheel
3. Overhead pulley
4. Skate and powder board

149. Which adaptive equipment(s) can be provided when deficit in upper extremity have
limitation to control efficiency and safety of function? E
1. Swivel fork and spoons
2. Plate quards or specialized dishes
3. Weighed or large handled cups and utensils
4. Velcro or zipper closures instead of buttons may improve dressing
494
150. About Frenkel’s exercises for ataxic conditions, how to apply this exercise? E
1. Design primarily for coordination
2. They are not intended for strengthening
3. Commands should be given in an even, slow, monotonous voice
4. Ability of patients to interpret deep muscle and joint sensibility with eye closed
NATIONAL BOARD EXAMINATION
PHYSICAL MEDICINE AND REHABILITATION
SEPTEMBER 7, 2018
CHOOSE THE MOST APPROPIATE ANSWER

1. What is the primary treatment for acute stenosing flexor tenosynovitis? C


A. Injection into the tendon sheath
B. Repetitive gripping exercise
C. Resting and immobilization
D. Ultrasound therapy
E. Surgical release

2. The following in non-articular manifestation of rheumatoid arthritis, EXCEPT…….


A. Anemia
B. Scleritis
C. Alopecia
D. Vasculitis
E. Subcutaneous nodule

3. All of the following statement are true regarding the carpal tunnel, EXCEPT……..
A. Is a narrow tunnel
B. Covered by the transverse carpal ligament
C. The base is made by the metacarpal bones
D. Osteoarthritis will narrow more the carpal tunnel
E. The median nerve is the only nerve passing through this tunnel

495
4. What kind of test for assess the tensor fascia latae or iliotibial band tightness?
A. Ober test
B. Ely’s test
C. Thomast test
D. Patrick’s test
E. Trendelenburg’s test

5. The following is medical rehabilitation program after total cemented hip replacement (THR)
A. Prevent hip abduction
B. Prevent hip flexion above 100o
C. Ambulation training after 4 weeks
D. Early ambulation, weight bearing as tolerated with walker
E. Ambulation NWB (non weight bearing) with double cruthes

6. Brain plasticity is NOT influenced by…….


A. Complexity of stimulation
B. Location of the lesion
C. Repetition of tasks
D. Environment
E. Motivation

7. The temperature and pain sense loss usually develops with disease in…….
A. Posterior columns of spinal cord
B. Posterior horn of spinal cord
C. Anterior horn of spinal cord
D. Lateral horn of spinal cord
E. Cerebellum

8. The presence of ataxia suggest damage to which area of the brain?


A. Vestibular nucleus
B. Temporal lobe
C. Cortex cerebri

496
D. Cerebellum
E. Thalamus

9. Conduction aphasia is characterised by………


A. Abnormal comprehension
B. Impair written language
C. Impair naming object
D. Impaired repetition
E. Fluent speech

10. A 62 year-old woman came to you with a complaint of problems with her arm and leg. She
states that for the past few months she has noticed that her right arm feels like it is shaking
whenever she is sitting quitely. In addition for the past few weeks, she feels that her right leg
drags a bit whenever she walks. She also feels that she is moving more slowly than she used to.
Examination confirms a tremor in her right hand that is present primarily at rest. There is also a
constant resistance to passive stretch of the flexor muscles at the elbow. When she walks, she
takes small shuffling steps, particularly with her right leg. Her facial expression and manner of
speaking are subdued or depressed.
The most likely site of lesion resulting in these sign and symptoms is………….
A. The tegmentum on the rostral midbrain on the right
B. The right cerebellum, posterior lobe
C. The left cerebellum, anterior lobe
D. The substantia nigra on the right
E. The substantia nigra on the left

11. A child with C5 ASIA a spinal cord injury should eventually become independent in which
activity?
A. Intermittent catheterization
B. Transfer to level surface
C. Standing
D. Feeding
E. Bathing

12. An 12 year-old boy came to you with hypotonia, had difficulty in climbing stairs, running,
and frequent falls. What is the most likely diagnosis?
A. Fascioscapulohumeral muscular dystrophy
497
B. Limb-girdle muscular dystrophy
C. Spinal muscular atrophy Type I
D. Duchenne muscular dystrophy
E. Becker muscular dystrophy

13. The growth retardation of the medial and posterior part of the proximal tibial physis and
epiphysis due to abnormal weight bearing stress and compression force on knees, is known
as…….
A. Genu varum
B. Genu valgus
C. Blount’s disease
D. Ricket’s disease
E. Ollier’s multiple enchondromatosis

14. Cardinal clinical sign in school age Duchenne’s Muscular Dystrophy is……….
A. Sailor gait
B. Waddling gait
C. Stepage gait
D. Wide base gait
E. Trendelenburg gait

15. The characteristic of normal gross motor develoment in 3 year-old child is……
A. Legs externally rotated
B. Immature balance
C. A faster cadence
D. Runs smoothly
E. Wide base

16. The method of exercise based on neuro developmental approach for CP patient is…….
A. Kabat method
B. Voyta method
C. Phelps method
D. Deaver method

498
E. Bobath method

17. Guillain Barre Syndrome related deaths usually occur in ventilator dependent patients due
to commplications such as……
A. Cramp like pain this is may be worst at night
B. Deconditioning due to prolong bedrest
C. Low blood pressure
D. Rapid heart rate
E. Sepsis

18. Postural drainage has been used as a form of chest physical therapy to enhance the flow of
out of airways. However, we should avoid this modality in patients…….
A. With sputum production greater than 30 ml per day
B. Using inhalant bronchodilator
C. Who have difficulty coughing
D. Using nebulizer therapy
E. With pulmonary edema

19. The following is exercise prescription for elderly patients with peripheral artery disease.
A. Type of exercise is intermittent walking to near minimal claudiation pain
B. The intensity of exercise start from 70% of HRR
C. The exercise progression start from 15 min
D. Type of exercise using bicycle
E. Competitive sport

20. How usually patient with chronic respiratory disease deal with fatigue diaphragm?
A. Deep breathing slowly
B. Avoiding rapid shallow breathing
C. Prefer the breath in supine position
D. Continue doing activity daily work
E. Recruit all the accessory breathing muscles

21. The following is NOT the goal for pulmonary rehabilitation…..

499
A. to control exacerbation
B. to improve quality of life
C. to stop disease progression
D. to relieve clinical symptoms
E. to enhance functional capacity

22. The activity of walking 2 mph requires the following amount of METs.
A. 2 METs
B. 4 METs
C. 6 METs
D. 8 METs
E. 10 METs

23. Postmenopausal women should take calcium daily….


A. 500 mg
B. 1000 mg
C. 1200 mg
D. 1500 mg
E. 2000 mg

24. Which wheelchair modification is best for a 60 year-old woman with a short right
transfemoral amputation and a left transtibial amputation who has good upper extremity
function?
A. Chair with the rear axle moved 3-5 cm posteriorly
B. A tilt-in-space wheelchair
C. Reclining wheelchair
D. Standar wheelchair
E. Electric wheelchair

25. The major cause(s) of falls in the elderly is………………..


A. weakness, balance or joint problem
B. not appropriate the type of footwear
C. not appropriate use of walking aids

500
D. hearing problems
E. poor lighting

26. Which of the following is true regarding falls in the geriatric population?
A. Most falls are considered accidental, rather than related to underlying disease or functional
impairments
B. The timed “Get up & Go” test is an appropriate way to assess someone who presents with
frequent falls
C. Dynamic balance training activities such as Tai Chi have no effect on falls or fear of falling
D. Fall risk assessment should evaluate both intrinsic and extrinsic risk factors
E. Fall sometime no effect for balance

27. This is NOT the characteristic of geriatric patient.


A. Multipathologic
B. Balance disturbalance
C. Non specific clinical symptoms
D. No changes of functional status
E. Limited physiologic reserved capacity

28. The phase of return to sport focused on the component of…..


A. Strengthening all muscles
B. Stretching exercise
C. Sport specific skill
D. Balance training
E. ROM exercise

29. Which of the following biomechanical factors in NOT a risk for recurrent stress fracture in
runner athlete?
A. Excessive supination of the foot
B. Leg lenght discrepancy
C. Forefoot valgus
D. Knee varus
E. Flat foot

501
30. A water skiing athlete present shoulder pain after injured in the competition. Physical
examination showed anterior shoulder dislocation. Which nerve is most commonly injured with
traumatic anterior shoulder dislocations?
A. Radial
B. Median
C. Ulnaris
D. Axillary
E. Musculocutaneous
A young basketball athlete, aged 18 years old, injured his right ankle while playing basketball
in a comptetition 1 week ago. His x-ray films showed no fracture. His physician diagnosed the
injury as lateral ankle sprain. Visual analogue scale score has decrease in intensity from 8/10 to
6/10, but the pain increase with weight bearing and certain demonstrated movements. No
warmth of the skin of anterolateral aspect of the right ankle, slight ecchymosis and swelling are
noted. His anterior drawer test is positive, and his talar tilt is negative (number 31 until number
33)
31. What is the classification of the injury due to it’s severity, and what stage of healing is this
patient in?
A. Grade I ankle sprain and inflammatory phase
B. Grade I ankle sprain and proliferative phase
C. Grade II ankle sprain and inflammatory phase
D. Grade II ankle sprain and proliferative phase
E. Grade III ankle sprain and inflammatory phase

32. In this case, what structure has been injured ?


A. Anterior talofibular ligament
B. Posterior talofibular ligament
C. Anterior tibiofibular ligament
D. Calcaneofibular ligament
E. Syndesmotic ligament

33. What programs are suited for this case ?


A. Rest and fixation
B. Stretching exercise
C. Endurance exercise
D. Proprioceptive and balance exercise
E. Weight bearing as tolerated and ROM exercise
502
34. The myoneuronal junction main neurotransmitter is …….
A. Dopamine
B. Serotonine
C. Substance P
D. Acetylcholine
E. N-Methyl-D-Aspartic Acid (NMDA)

35. Factores involved in the formation of contractures include ……..


A. A increase in the number of sarcomeres
B. A change from type I to type II muscle fibers
C. Inadequate cross-linkage of collagen molecules
D. Disturbed balance between synthesis and degradation of collagen
E. A decrease in the number of mitochondria present in muscle fibers

36. All of the following may be seen in myasthenia gravis, EXCEPT ……..
A. Normal motor nerve conduction studies
B. Occasional positive waves and fibrillation
C. A decrement at low rates of repetitive stimulation
D. Large-amplitude, long duration, polyphasic motor units
E. A variable response following high rates of repetitive stimulation

37. The annulospiral ending is the primary afferent fiber coming from the ……..
A. Nuclear bag
B. Nuclear chain
C. Intrafusal fiber
D. Extrafusal fibers
E. Myoneural junction

38. A 54 year old man has complete foot drop for 15 days. EMG showed fibrillation potentials
and reduced interference pattern of anterior tibial, peroneus longus, extensor hallucis longus
and lumbosacral paraspinal muscles but not in the gastrocnemius. Nerve conduction velocity of
peroneal nerve is normal. The interpretation is :
A. L2 radiculopathy
503
B. L3 radiculopathy
C. L4 radiculopathy
D. L5 radiculopathy
E. S1 radiculopathy

39. The most characteristic of hemiplegic gait is ……….


A. Flexion of the knee in swing phase
B. Adduction of the hip in swing phase
C. Abduction of the hip in swing phase
D. Dorsi flexion of the ankle swing phase
E. Hip circumduction in swing phase

40. The following activity is the negative Mechanical Advantage less than one (MA<1).
A. Kids player tool like see-saw
B. Use a short lever arm when we stretch a joint
C. Use the cane in the opposite with the OA limb
D. Using extended holder while carry on a heavy bag
E. Pick up heavy think from the floor with squatting position and hold something as close as
possible to our body

41. The muscle that hast the important role in push-off is ……


A. Iliopsoas
B. Hamstring
C. Quadriceps
D. Triceps surae
E. Tibialis anterior

42. Which of the following describes “scapulo humeral rhythm”?


A. 2/3 glenohumeral and 1/3 scapulothoracic
B. 1/3 glenohumeral and 2/3 scapulothoracic
C. 1/2 glenohumeral and 1/2 scapulothoracic
D. 1/4 glenohumeral and 3/4 scapulothoracic
E. 3/4 glenohumeral and 1/4 scapulothoracic
504
43. The most function of gluteus maximus during gait when…………
A. Loading response
B. Terminal stance
C. Mid stance
D. Pre swing
E. Pre swing
44. The normal degree of femoral neck inclination in adult is……..
A. 75 degrees
B. 100 degrees
C. 125 degrees
D. 150 degrees
E. 175 degrees

45. What is the most proper modality should be given in a partial thickness tear of
supraspinatus tendon?
A. TENS
B. Electrostimulation
C. Ultrasound diathermy
D. Short wave diathermy
E. Low level LASER therapy

46. The depth that laser energy penetrates into the body is related to…….
A. Total watts
B. Duty cycle
C. Total joules
D. Output frequency
E. Treatment duration

47. In what condition would heat therapy be indicated?


A. Hemophilic hemarthrosis
B. Immature scar tissue
C. Impaired circulation
505
D. Acute inflammation
E. Chronic tendinitis

48. Which condition is NOT contraindication to applied ultrasound diathermy?


A. Using cardiac pacemakers
B. Nerve compression injury
C. Over pregnant uterus
D. Malignancy
E. Open wound

49. Which is NO contraindication for the use of electrical currents?


A. Edema
B. Pregnancy
C. Unstable arrythmia
D. Placement of electrodes over carotid sinus
E. Areas where venous or arterial thrombosis

50. Patients with diagnosis Bell’s Palsy of left side, six days after onset. Modalities that be
given were SWD of area retro auricular. Effect that expected is………
A. Decreased of stylomastoideus foraminal edema
B. Increased of left facial muscle strenght
C. Make facial massage more easy
D. Reduced facial assymmetry
E. Reduced facial pain

51. Which of the following muscles is the lateral rotator of the hip joint?
A. M. Tensor fascia latae
B. M. Gluetus minimus
C. M. Gluteus medius
D. M. Psoas major
E. M. Piriformis

52. What is the main predictor for risk of fractures in a person with osteoporosis?
506
A. Onset
B. Recent falls
C. Low body weight
D. Low physical activity
E. History of prior fractures

53. A 30 year-old girl with rheumatoid arthritis complains of neck pain with occipital
headaches. MRI demonstrates atlantoaxial subluxation. The posterior atlanto-odontal interval is
20 mm. Your initial treatment at this condition is………….
A. Passive ROM exercise
B. Fusion of the cervical spine
C. Cervical traction using 0.5 kg weight
D. Immobilization with philadelphia collar
E. Botulinum toxin injection of the paracervical muscles

54. Spurling maneuver,


A. Assess for upper motor neuron involvement
B. Applies traction on the nerve root
C. Is not sensitive for radiculopathy
D. Reproduce radicular symptom
E. Distract the neuroforamen

55. The following are the contributing factors in cancer-related fatigue, EXCEPT ………
A. Pain
B. Anemia
C. Radiation related
D. Light physical activity
E. Chemoteraphy related

56. When a patient with recent onset of hemiplegia shows progressive motor recovery to
Brunnstrom stage 3, what do you expect?
A. Decreasing tone and increasing isolated voluntary movements
B. Unchanged tone and increasing isolated voluntary movements
C. Decreasing tone and increasing synergistic movements
507
D. Increasing tone and increasing synergistic movements
E. Increasing tone and decreasing synergistic movements

57. A male 23 years old suffered traffic accident 3 months ago. He has stabilized fracture at
vertebrae Th. XII. The motor score of L2 and 3 are good, L4 trace, and below L4 are zero. And
double to incrase the walking ability, what should you do?
A. Give double AFO plus bilateral axillary crutches
B. Body weight-supported partial training
C. Strenghthening the impaired muscles
D. Functional electrical stimulation
E. Give double AFO

58. For severe generalized spasticity in persons with SCI, you may consider……..
A. Diazepam
B. ROM exercise
C. Botulinum toxin
D. Baclofen intrathecally
E. Stretching of spastic muscles

59. Finger extension and shoulder abduction within 72 hours after stroke predicts functional
recovery of the hemiplegic arm at……….
A. 1 month
B. 3 months
C. 6 months
D. 8 months
E. 12 months

60. A good initial treatment plan for neuropathic pain secondary to spinal stenosis would
include…….
A. Gabapentin
B. Acetaminophen
C. Neurotropic drug
D. Narcotic pain medication
E. Nonsteroid anti-inflammation
508
61. A 6 year-old boy with hemophilia has suffered a recurrent hemarthrosis of the knee. After
providing adequate factor VIII replacement, the most important therapy in the first 48 hours
is……..
A. Immobilization
B. Active assistive ROM exercise
C. Icing and passive ROM exercise
D. Ambulation with full weight bearing
E. Ambulation with partial weight bearing

62. Physical examination findings that are consistent with hypotonia in a term infant include all
of the following, EXCEPT……..
A. Head lag when pulled to sitting position
B. Decrease spontaneous movements
C. Frog leg position of the legs
D. Episthotonus
E. Weak suck

63. A child 3 years-old has cilinical sign: asymmetric face, shorting of sternocleidomastoideus
muscles without tumor, deficit of head rotation > 300. The appropriate management is………..
A. Deep heating
B. Stretching exercise
C. Asymmetric collar
D. Refer to orthopedic surgeon
E. Stimulation of the head movement

64. Which following statement below is correct about spastic diplegia?


A. Good prognosis of walking
B. Significant mental retardation
C. Significant oromotor disfunction
D. Involves all four extremities symmetrically
E. Involves upper extremities more than lower extremities

509
65. The maneuver to detect hip dislocation in infant, begin with hip flexion 90 degress, then hip
abduction, and hip examiner’s index finger pushes up gently on the greater trochanter is
called…….
A. Barlow maneuver
B. Galleazi maneuver
C. Ortolani maneuver
D. Hip click maneuver
E. Hip reduction maneuver
66. Intense motor learning and basic language development occur at………
A. Birth to sixth month
B. Birth to tenth month
C. Birth to one year of age
D. Birth to two years of age
E. Birth to three years of age

67. The respiratory problem in patient with Ankylosing Spondylitis is…….


A. Diminished chest expansion
B. Increased airways obstruction
C. Moderate dyspnea on exercise
D. Increased use of accessory muscle
E. Decreased in diaphragmatic excursion

68. Which of the following risk factors in most associated with a higher incidence of Deep Vein
Thrombosis (DVT) in adult with stroke?
A. Male sex
B. Female sex
C. Increased age
D. Smoking history
E. Muscle paralysis

69. A 60 year-old cancer patien with an acute deep vein thrombosis (DVT) in the right upper
limb, secondary to a long-standing central venous catheter is referred to you. What is the
precaution for this patient?
A. No precaution

510
B. Bed rest for 10-12 days to allow for clot maturation
C. Begin resistive exercise 12-24 hours after anticoagulant therapy
D. Exagerated upper extremity exercise on the affected side, but ambulation is not
restricted
E. No activity restriction, since upper limb DVTs have a low likelihood of causing a pulmonary
embolus

70. The clinical sign of COPD patients as below.


A. Exertional dyspneu often occurs when the FEV1 is less than 1500 ml
B. Arterial blood gases may normal during exercise
C. FEV1/FVC is more than 70% predicted normal
D. TLC are generally decreased
E. Residual volume decreased

71. Which of the following statements regarding arm versus leg exercise is true?
A. Target heart rate for leg exercise should be decreased by 10 bpm
B. Target heart rate for leg exercise should be increased by 10 bpm
C. Target heart rate should not be used as a guide for arm exercise
D. Target heart rate should not be used as a guide for leg exercise
E. Higher VO2 should be expected in arm exercise

72. Myocardial oxygen consumption is best estimated from……


A. Rate-pressure product (systolic BPxHR)
B. Cardiac output
C. Stroke volume
D. Heart rate
E. VO2 max

73. Which of the following is the most important lifestyle modification for prevention of
osteoporosis?
A. Eating a diet high in protein
B. Eating a diet high phosphorus
C. Increasing the intake of alcohol
D. Avoiding cigarette smoking and high intake of caffeine
511
E. Minimizing the use of non-steroidal anti-inflammatory medications

74. What is the most common cause of dementia in geriatric population


A. Toxic-metabolic dementia
B. Lewy body dementia
C. Vascular dementia
D. Alzheimer disease
E. Parkinson

75. This type of activity and exercise is NOT recommended in osteoporotic patient to prevent
developing compression fracture of the spine.
A. Back exercise
B. Balance exercise
C. Flexion exercise for the spine
D. Keep heavy objects close to chest when lifting
E. Flex hips and knees to bend down to lift objects

76. Which medication is associated with osteoporosis


A. Hydrochlorothiazide
B. Coumadin
C. Phenytoin
D. Diltiazem
E. NSAID

77. the condition of nerve injury that cause motor and sensory abnormalities in which the
myelin is impaired but the axon remain intact is called………..
A. Neurotmesis
B. Neuropraxia
C. Demyelination
D. Axonal degeneration
E. Wallerian degeneration

78. The 30 year-old football player who present with knee pain, immediate swelling and
inability to bear weight after a fall, should undergo……
512
A. Ice, elevation, compression, and cruthes as indicated
B. Ice, elevation, return to football playing the next dat
C. Heat, NSAID, ace wrap, return to football playing the next day
D. Cold compresses, acetaminophen, 5 days off from football playing
E. Heat wrap, knee immobilization, advantage weight bearing as tolerated

79. Competitive female athlete are at high risk of stress fractures if they have the female ahtlete
triad of……..
A. Amenorrhea, anorexia, osteoporosis
B. Amenorrhea, bulimia, poor flexibility
C. Bulimia, hypocalemia, poor flexibility
D. Amenorrhea, hypocalemia, osteoporosis
E. Amenorrhea, hypercalemia, osteoporosis

80. Which is the most important risk factor for an ankle sprain?
A. Flat foot
B. Rear foot valgus
C. Short achilles tendon
D. Generalized joint laxity
E. A history of a previous sprain

81. An abducted gait in an above-knee amputee may due to…….


A. Alignment of the prosthetic foot too close to the midline
B. An adducted hip joint on the pelvic band suspension
C. Adduction contracture of the hip
D. Too short of socket lateral wall
E. Too short prosthetic lenght

82. For short and very short transradial amputation level, the statement below is TRUE.
A. Limit elbow flexion strenght and elbow range of motion
B. Optimal for body powered prosthetic restoration
C. Major advantage due to cosmetic problems
D. Limit abduction shoulder range of motion
513
E. Ideal for elbow suspension

83. The right side about Chopart amputation is………


A. At the tarso-metatarsal junction
B. At the talo-navicular, cuboid, talus, and calcaneus amputation
C. A horizontal calcaneal amputation, all tarsals removed except calcaneus/talus
D. At midtarsal, talo-navicular, calcaneo-cuboid amputation, talus and calcaneus bones
remain
E. A vertical calcaneal amputation, the lower articular surfaces ot the tibia/fibula are sawn
through

84. The function of medial T strap in AFO is to……..


A. Pes equinus control
B. Pes valgus control
C. Pes planus control
D. Pes varus control
E. CTEV control

85. Which one of the following is appropriate with Milwaukee orthosis?


A. Usually use for apex below Th10 vertebrae
B. It is technically simple and do not require the service of a skilled person
C. Correction more effective for curve more than 450 during growth period
D. Less ventilation and more chest restriction than most other types of support
E. It should be used in conjunction with an exercise program both in and out of the
orthosis

86. The muscles required for ambulation using crutches are……


A. Triceps and biceps
B. Shoulder depressors and biceps
C. Biceps, triceps, and wrist flexions
D. Shoulder depressor, biceps, wrist extensors
E. Shoulder depressor, triceps, wrist extensors, and finger flexors

87. The following is true about DeLorme technique, EXCEPT:


514
A. Used to strengthen the muscle
B. Is also called progressive resistive exercise
C. The subject is tested to determine the “10 repetition maximum”
D. The weights would progressively increase from week to week
E. The person lift at various percentages of the 10 RM, starting at 100%

88. The best example of an open kinetic chain exercise of the quadriceps is……..
A. A stair climbing machine
B. A treadmill inclined to 7 degress
C. Leg ergocycle without resistance
D. A wind-resistance exercise bicycle
E. An isokinetic knee extension machine

89. Which approach of motor coordination and skill exercises uses resistance during the spiral
and diagonal movement patterns with the goal of fascilitating “irradiation” of impulses to other
parts of the body associated with the primary movement?
A. Bobath
B. Brunnstrom
C. Rood appoach
D. Carr and Sheperd approach
E. Propioceptive Nerve Stimulation

90. The formula for strengthening exercise is……..


A. Low resistance, high repitition
B. High resistance, high repetition
C. High resistance, fewer repetition
D. Low resistance, fewer repetition
E. Moderate resistance, high repetition

91. The following is basic requirement for a stroke patient to joint group therapy in occupation
therapy?
A. Marked abnormal movement
B. Who lack of concentration
C. Have good sitting balance
515
D. Have marked spasticity
E. Severe receptive aphasia

92. Delayed Onset of Muscle Soreness (DOMS) usually develop after……


A. Isotonic exercise
B. Isometric exercise

C. Isokinetic exercise
D. Eccentric exercise
E. Concentric exercise

93. A 45-year-old woman presents with pain in the region of the Achilles tendon. This is the
third such episode over the past 2 years. The achilles tendon appears swollen and warm.
Appropriate initial management would include……
A. Cortison injection of the Achilles sheath
B. Given SWD for reducing swollen and warm
C. Immobilization in an ankle plantar-flexed position
D. Prescription of a custom molded AFO to reduce pronation
E. Use of a small heel lift with initiation of gastroc-soleus stretches as tolerated

94. Pain around the medial aspect ot the knee joint very often caused by inflammation of the
pes anserinus. This conjoint tendon of the following muscles……
A. Gracilis-sartorius-bicep femoris
B. Gracilis-sartorius-vastus medialis
C. Semitendinosis-sartorius-gracilis
D. Sartorius-gracilis-vastus medialis
E. Semimembranosus-sartorius-gracilis

95. The following is the initial treatment for acute osteoarthritis, EXCEPT……
A. Knee orthosis
B. Icing to relieve joint symptoms
C. Given USD to reduce inflammation
D. Medication to reduce inflammation and pain
E. Immobilization of the joint to reduce inflammation and joint pain
516
96. In thoracic outlet syndrome, if the adson test is positive, this suggested to the following
condition…..
A. Hypertrophy of the sternecleidomastoideus muscle
B. Hypertrophy of the scalenus posterior muscle
C. Hypertrophy of the pectoralis mayor muscle
D. Fracture of the clavicle
E. Cervical rib

97. In the physical examination found left hemineglect. The following sign can be present,
EXCEPT…..
A. Aphasia
B. Denial of his hemiplegia
C. Mistakes in the localization of sounds
D. The absence of the left part of his drawings
E. A decrease of the visual field at the opposite side of the lesion

98. What is the estimated rate of nerve regeneration?


A. 0.1 mm/day
B. 1 mm/week
C. 1 mm/day
D. 1 cm/week
E. 1 cm/day

99. The following statement is NOT the characteristic signs of conus medullaris syndrome.
A. Perineal sensory deficit
B. Sphincter dysfunction
C. Loss of anal reflex
D. Pyramidal sign
E. Flaccid sign

100. A disorder of neuromuscular transmission due to an autoimmune response against Ach


receptors on the postsynaptic membrane, associated with thymic disorder or thymic tumor with
the cllinic presentation are proximal fatigue and weakness is……..
517
A. Lambert-Eaton Syndrome
B. Myasthenic Syndrome
C. Myasthenia Gravis
D. Priodic paralysis
E. Botulism

101. What is/are the indication of cryotherapy?


1. Reduction of pain and joint stiff
2. Decreasing spasticity
3. Acute arthritis
4. Myositis

102. The following statement(s) is/are TRUE. A


1. The mechanism of heat transfer of paraffin bath in conduction
2. The mechanism of heat transfer of whirpool is convection
3. The mechanism of heat transfer of SWD is conversion
4. The mechanism of heat transfer of USD is radiation

103. The effect(s) of ultraviolet radiating is/are……


1. Photochemical effects
2. Relief of muscle spasm
3. Have biological effects
4. Increases the pain threshold

104. Postoperative goal(s) of preprosthetic management include……


1. Independent mobility
2. Preparation of residual limb for prosthetic fitting
3. Independence in self-care and activities of daily living
4. Maintaining ROM, especially in the remaining proximal joints of the amputated
extremity

105. When positioning one or more joints with an orthoses, take into consideration of………
1. Joints biomechanics
518
2. The pathology
3. Tissue tension
4. Function

106. The indication(s) for using of an ankle-foot orthosis (AFO) to improve gait is/are…….
1. Weak push-off at late stance
2. Mediolateral instability at the ankle
3. Passive plantar flexion in swing phase
4. Foot drop at heel strike due to weak ankle dorsiflexor

107. This statement is/are the contraindication for prenatal exercise.


1. At risk for premature labour
2. Severe hypertensive disease
3. Poor obstetric history
4. No prenatal care

108. The following technique is/are used for treatment lymphedema.


1. Compression garment
2. Elevation
3. Massage
4. Icing

109. The purpose(s) of an initial comprehensive assessment of sensory integrative problem


is/are…….
1. To identify the impact of sensory processing and praxis problems
2. To provide information to assiste with treatment planning
3. To identify specific sensory integrative problems
4. To identify the behavior problem

110. The following statement is/are TRUE about shoulder impingement syndrome.
1. Most common in athletes and labors who repeatedly shoulder abduction more than 900
2. Repeated injury of tissue within subacromion space
3. The most common painful disorder of shoulder
519
4. Pain aggravated by passive abduction 60-1200

111. The characteristics of Hernia Nucleus Pulposus is/are ………


1. Increased pain by forward bending and sitting
2. Tingling sensation or numbness on the leg
3. Coughing increase the referred pain
4. Using corset is advices able

112. The following is/are the purpose of assessment patient with low back pain.
1. To determine the patient who have mechanical or structural causes
2. To differentiate pain due to systemic disease or other disease
3. To give proper rehabilitation intervention
4. To choose the orthoses

113. The stages of frozen shoulder are ………..


1. Painful stage
2. Frozen stage
3. Thawing
4. Resolution stage

114. The following statement is/are TRUE about Electrodiagnosis examination.


1. Able to localize a peripheral nerve lesion
2. Able to establish diagnosis of peripheral nervous system disease
3. Able to make early detection of possible peripheral nerve lesion
4. Able to establish diagnosis of central nervous system disease

115. The role of golgi tendon organs in voluntary motor activity is / are ……….
1. Detects relative muslce tension
2. In parallel with them muscle fibers
3. Has dynamic and static responses
4. Increase in muscle lenght during passive movement

520
116. The following statement(s) about type 1 muscles fiber is/are correct.
1. Contain a large number of mitochondria and myoglobin
2. Dominant in much of marathon runner
3. Also called as slow twitch fibers
4. Generate larger force

117. The following is/are true about hip joint


1. Is a ball-and-socket joint
2. Hip flexion at least 120 degrees
3. The use of a brace on the leg can alter the magnitude of the hip joint reaction force
4. The magnitude of the hip joint reaction force is influenced by the ratio of the abductor
muscle force and gravitational force lever arm

118. The following muscle is/are the primary movers at scapulothoracic interface
1. Subscapularis
2. Latissimus dorsii
3. Infraspinatus
4. Serratus anterior

119. Where is/are the ground reaction force vector located at mid stance?
1. Anterior to ankle
2. Posterior to ankle
3. Anterior to knee
4. Posterior to knee

120. The autonomic dysreflexia symptom(s) and clinical sign(s) include…….


1. Cutaneous vasodilation
2. Pounding headache
3. Reflex bradycardia
4. Hypotension

121. Common behavioral and personality changes seen in traumatic brain injury person is / are
…..

521
1. Memory deficit
2. Attention deficit
3. Spatial dysfunction
4. Behavioral disorders

122. Vertebrobasilar artery occlusions are cause a wide variety of cranial nerve, cerebellar, and
brainstem deficits. There is / are …….
1. Dysphagia and dysarthria
2. Facial hypesthesia, syncope and ataxia
3. Vertigo, nystagmus, diplopia, visual field deficits
4. Loss of pain and temperature sensation occurs on the ipsilateral face

123. Complication associated with autonomic dysreflexia could include ………


1. Severe hypertension
2. Cerebral bleeding
3. Retinal bleeding
4. Tachycardia

124. The reflexes important for assessing central nervous system developmental is / are as
follows.
1. Grasping reflexes
2. Protective reflexes
3. Vertical suspension
4. Neck righting reflex

125. The manifest signs of upper motor neuron involvement spastic children may include
………
1. Hyperreflexia
2. Crossed adduction reflex
3. Persistent primitive reflexes
4. Abnormal Babinski response after 2 years of age

126. The characteristic of cerebral palsy are as follow :


1. The underlying neurologic lesion must be static
522
2. The process which cause the cerebral palsy is still active
3. The sign of cerebral palsy is a disorder of movement and posture
4. It is caused by progressive injury which affects the mature brain

127. Essential component of Comprehensive Pulmonary Rehabilitation is/are……..


1. Nutritional management
2. Aerobic and respiratory muscle training
3. Behavior management, relaxation and education
4. Clearance of secretions, positioning and breathing exercise

128. The goals of Cardiac Rehabilitation Program is/are………..


1. To maximize exercise tolerance and activity daily living (ADL)
2. To utilize energy conservation and work simplification
3. To maximize cardiovascular capacity and fitness
4. To improve quality of life

129. Basic component for exercise training in COPD is/are…….


1. Respiratory muscle exercise
2. Upper extremity exercise
3. Aerobic exercise
4. Balance exercise

130. Hamstring function as a …………


1. Protectors of posterior structures from overstretching in combined movement of hup
flexion and knee extension
2. Good extensors, rotators, and weak adductors of the hip
3. Rotators of the knee with the knee flexion
4. Good flexor of the knee

131. Which of the following statements is TRUE regarding pain in the old person ?
1. Pain can commonly be located to a single site.
2. Elderly person do not feel pain as much as younger people
3. Pain leads to less functional impairment in the elderly than in the young people
523
4. Approximately one third of elderly person have chronic joint pain and arthritis

132. The following is / are the indication for measurement of bone density ……..
1. Adults taking medications such as phenytoin
2. To monitor treatment effect on osteoporosis
3. Women discontinuing estrogen therapy
4. Women aged 35

133. In prescribing pain treatment for elderly patients, the following is / are the proper
approach?
1. Use high-dose, short-acting narcotics initially to get the pain control
2. Avoid use of acetaminophen, due to liver toxicity
3. Start with aspirin, which is safe and effective
4. Start low and go slow for all medications

134. What is the most common cause of knee pain in runners?


1. Baker cyst
2. Diskoid meniscus
3. Hamstring tendinitis
4. Patellofemoral pain syndrome

135. Factors which considered by PMR doctors to design Rehabilitation Program for patient
post reconstruction following revision ACL is / are ………..
1. Graft selection
2. Surgical techniques
3. Physical performance factors
4. Mentally of the patient

136. What is / are the treatment for progress proprioception, coordination and agility during the
chronic phase of soft tissue healing in sport injury rehabilitation ?
1. Balance activities
2. Isotonic strengthening exercise
3. Surface modification
4. Isokinetics strengthening exercise
524
137. Medial stress of injuries of the elbow in throwing athletes, include(s) ……….
1. Ulnar nerve traction
2. Avulsion of medial epicondyle
3. Flexor muscle strain or tear
4. Olecranon osteophyte formation
138. “Scottie dog” appearance can be shown in what position(s) vertebral x-ray plane?
1. Antero posterior
2. Postero anterior
3. Lateral
4. Oblique

139. What is / are the correct statement for knee OA?


1. Increased varus in stance phase and valgus in swing phase
2. Decreased walking velocity, stride length, and cadence
3. Decreased peak flexion during stance and swing phase
4. Increased double support time and total stance phase

140. Patient with acute neck sprain patients usually is treated with ……..
1. Tricyclic antidepressant
2. Neck soft collar
3. Stretching exercise
4. Cold therapy

141. Human obesity is / are associated with an increased risk of knee OA specifically at
……….
1. Patello tibial compartment
2. Patello femoral compartment
3. Lateral tibio femoral compartment
4. Medial tibio femoral compartment

142. The following is / are true about method in stroke rehabilitation.


1. Functional Electrical Stimulation (FES) of the paretic limb enhances plasticity
525
2. The principle of constraint induce movement therapy to avoid the learned non use of
the paretic side
3. The principle of mirror therapy is that movement of the affected limb can be
stimulated via visual cues originating from the opposite side of the body
4. Proprioceptive Neuromuscular Facilitation (PNF) emphasized synergistic patterns of
movement that develop during recovery

143. The severity of traumatic brain injury based on ………


1. The severity of cognitive disturbance
2. Post-traumatic amnesia
3. The severity of motoric weakness
4. Glasgow Coma Scale

144. These are the rehabilitation programs for leprosy …………


1. Oil massage
2. Active exercise
3. Soaking the feet in water
4. Protective of the hands and feet

145. The characteristic spastic dyplegia of cerebral palsy are as follow : A


1. 18 months implied poor prognosis for ambulation
2. Children who did not sit by 4 years have not achieved ambulation
3. Sitting by 2 years was a good predictive sign of eventual ambulation
4. Children with diplegia have not adequate movement control of the arm and hands

146. Cardinal Clinical signs for SMA Type I Werdnig Hoffman Disease include …….. B
1. Weak cry
2. Tremor of hands
3. Frog posture
4. Weakness of legs, predominantly proximal

147. Rehabilitation of the child with Congenital Brachial Plexus Palsies is / are ……….. B
1. Avoid stretching of the shoulder capsule
2. Vigorous range of motion exercise
526
3. Gradually range of motion exercise
4. Elbow pronation positioning

148. Which physiologic factor(s) in elderly exacerbate(s) orthostatic hypotension? D


1. Decrease in arterial stiffness
2. Decreased creatinin clearance
3. Decreased peripheral resistance
4. Decreased baroreceptor response

149. What is the purpose of a warm-up period? B


1. To prevent soft tissue injury
2. Gradual reduction of cardiac work
3. Promotes continuous dissipation of heat
4. Redistribution of blood from muscle to internal organs

150. The following below is / are the condition causing in ventilatory impairment.
1. Spinal muscular atrophy
2. Muscular dystrophia
3. Kyphoscolliosis
4. Poliomyelitis

527
INDONESIAN COLLEGE
OF
PHYSICAL MEDICINE AND REHABILITATION

NATIONAL BOARD EXAMINATION

NOVEMBER 30, 2018 


528
NATIONAL BOARD EXAMINATION PHYSICAL MEDICINE AND
REHABILITATION
NOVEMBER 30, 2018

CHOOSE THE MOST APPROPIATE ANSWER

1. One of the early Rehabilitation Program of the burn patient is positioning. Which one
of the statement below that is NOT correct?
A. Body parts should be positioned as to maintain burned tissue in their stretch
B. Typically limbs should be positioned in extension-abduction alignment
C. Hanging over the edge of the table may compressed the radial nerve
D. Positioning is maintained using splints, pillow, and/or foam wedges
E. Positioning at patient comfortable to reduce pain

2. What is the' best initial knee imaging study on a patient with suspected Osgood-
Schlatter disease?
A. Magnetic resonance imaging
B. Computed tomography scan
C. Postero-anterior radiograph
D. Skyline view radiograph
E. Lateral x-ray

3. A 45-year-old long-distance runner with no history of knee injury wishes to reduce the
risk of degenerative disease of the knees. Your recommendation will be .
A. using oyster shell calcium
B. taking diclofenac 50 mg twice a day
C. discontinuing long-distance running
D. taking glucosamine and chondroitin sulfate
E. practicing stretching and quadriceps strengthening
529
4. The following statement is TRUE about ostearthritis (OA).
A. Osteoarthritis is a systemic bone disorder
B. Genetic predisposition is not the risk of OA
C. The prevalence of OA is higher in men than women
D. Characterized by non progressive articular cartilage damage
E. Involving cartilage, subchondral bone, menisci and periarticular soft tissues

5. The one of the following is NOT of the prevention programs for Rotator Cuff
tendinopathy.
A. Maintain the dynamic control ratio of shoulder external and internal rotators
B. Suggestion to make proper training periodization
C. Hotpack before and after training
D. Warming up and cooling down
E. Facilitate proper resting period

6. One of the following type aphasia is non-fluent. There is .


A. transcortical sensory
B. transcortical motor
C. conduction
D. Wernicke's
E. anomia

7. A glove and stocking pattern of sensory disturbance usually develops with disease
in....
A. peripheral nerve
B. the spinal cord
C. the brainstem
D. the thalamus
E. the cortex

8. The following reflex make possible the stabilization of the visual field. That
A. stretch reflexes

530
B. cervicospioal reflexes
C. vestibulocollic reflexes
D. vestibulospinal reflexes
E. vestibulo-ocular reflexes

9. Which following condition is NOT the abnormal involuntary movement ?


A. Tics
B. Ataxia
C. Chorea
D. Athetosis
E. .Hemibalism

10. A 40-year-old man. 4 weeks after traumatic brain injury with left
hcmiplcgia, He no develops severe spasticity, particularly affecting the left shoulder and
arm muscles. Which one of the following treatments is NOT validated?
A. Injection of Botulinurn A toxin in the spastic muscles
B. Alcohol injection in the spastic muscle motor points
C. Low level laser therapy
D. Gaba-type medication
E. Icing

11. A boy suffered from spina bifida L4-L5 segment, will have preserved muscle
function in......
A. iliopsoas muscles
B. hamstring muscles
C. quadriceps muscles
D. foot intrinsic muscles
E. gluteus maximus muscle

12. Partial absence of a limb with preservation of hand in children is called ..


A. amelia
B. adactyly
C. meromelia
D. hemimelia
531
E. phocomelia

13. A maneuver used to determine a dislocated hip in children, with the leg in a flexed
and adducted position, examiner push the femur posteriorly with the thumb is a maneuver
used to determine a dislocated hip in children. This test is called.
A. Ortholani step 2
B. Ortholani test
C. Galeazzi test
D. Barlow test
E. Allis test

14. A 9 year-old boy with primary muscle disease is being evaluated for toe-walking-
The reason of toe-walking is .....
A. hamstring hyperextensibility
B. weakness of plantiflexors
C. quadriceps weakness
D. hip flexor weakness
E. poor proprioception

15. Significant and clinically meaningful improvements from pulmonary rehabilitation is


A. Improved health status
B. Decreased of re-hospitalization
C. Back to normal condition like healthy person
D. Improved anatomical status of respiratory system
E. Decrease in dyspnea and increase functional capacity

16. The vital capacity is the sum total of the ......


A. tidal volume plus residual volume
B. inspiratory capacity and functional residual capacity
C. inspiratory capacity, tidal volume and expiratory reserve volume
D. inspiratory reserve volume,tidal volume and expiratory reserve volume
E. inspiratory reserve volume, functional residual capacity and tidal volume

17. The activity of jogging (5mph) requires the following amount of METs.
532
A. 3 METs
B. 5 METs
C. 8 METs
D. 10 METs
E. 12 METs

18. Which of the following is the correct criteria for night time ventilator assistance in the
patient with COPD?
A. Pa02 75 mmHg
B. PaC02 50 mmHg
C. FEVl 50% predicted
D. Respiratory rate 20/min
E. Maximal inspiratory force 1 OOcmH20

19. Treatment goals of arthritic conditions in older people include ...


A. relief from fear, fatigue, stiffness, and pain
B. improving cardio respiratory endurance
C. induce of the inflammatory process
D. identification of deformity
E. maximizing mobility

Woman a 69 year- old, with a medical diagnosis of post bilateral TKR osteoarthritis. She is
widowed and living alone in a third-floor apartement with elevator access. Yesterday,
she underwent elective surgery for bilateral total-knee arthroplasties. Now no pain in her both
knees, but there is still stiff on her both knees. Her medical history includes emphysema,
myocardial infarct 2 years ago, moderate obesity and hypertension. She lives independently
but has a maximum walking tolerance of one-half block when using a cane for support.
(question number 20 and 21).

20. The short-term goals is......


A. dependent with walker or cane
B. independent ambulation l00 m with walker
C. immobility in bed and if transfer with cane
D. active knee range of motion > 10 to 70 degrees
E. exercise in bed and up in chair more than 5 hours per day

533
21. The long-term goals is.....
A. ambulation>l OOm, rest breaks as needed allowing for baseline compromised
cardiopulmonary status
B. independent ambulation within building complex, no assistive device
C. resume all previous social activities with friends and family
D. return to independent swimming for community access
E. return to preoperative vocational routine

22. A young female gymnastic athlete suffered with chronic back pain. What is the most
common diagnosis in this athlete?
A. Spondylitis
B. Spondylosis
C. Spondylolysis
D. Spondylolisthesis
E. Spondyloarthropaties

23. Injury to the rotator cuff muscles and tendons of the shoulder is often occurs in?
A. Tennis player
B. Bowling player
C. Discus thrower
D. Boxing athletes
E. Rowing athletes

A young basketball athlete, aged 18 years old, injured his right ankle while playing basketball
in a competition 1 week ago. His x-ray films showed no fracture. His physician diagnosed
the injury as lateral ankle sprain. Visual Analog Scale score has decrease in intensity from 8/10
to 6/10, but the pain increases with weight bearing and certain demonstrated movements. No
wannth of the skin of anterolateral aspect of the right ankle, slight ecchymosis and swelling arc
noted. His anterior drawer test is positive, and his talar tilt is negative (number 24 until number
26)

24. What is the classification of the injury due to its severity, and what stage of healing is
this patient in?
A. Grade I ankle sprain and inflammatory phase
B. Grade Iankle sprain and proliferative phase
534
C. Grade II ankle sprain and inflammatory phase
D. Grade II ankle sprain and proliferative phase
E. Grade III ankle sprain and inflammatory phase

25. In this case, what structure has been injured?


A. Anterior talofibular ligament
B. Posterior talofibular ligament
C. Anterior tibiofibular ligament
D. Calcaneofibular ligament
E. Syndesmotic ligament

26. What programs are suited for this case?


A. Rest and fixation
B. Stretching exercise
C. Endurance exercise
D. Proprioceptive and balance exercise
E. Weight bearing as tolerated and ROM exercise

27. During aerobic exercise, blood flow remains relatively constant within .
A. the skin
B. the heart
C. the brain
D. the kidneys
E. the skeletal muscle

28. The most common clinical application of the H-reflex is to test .


A. Guillain Barre Syndrome
B. carpal Tunnel Syndrome
C. cervical radiculopathy
D. Sl-radiculopathy
E. polyneuropathy

535
29. Electrodiagnostic findings typical of a variety myopathic include :
A. Giant potential
B. Myotonic discharges
C. Complex repetitive discharges
D. Increased frequency of end-plate spikes
E. Decreased duration of motor unit potentials

30. Degree of freedom in human joint is highly dependent on .


A. bony enlargement
B. bony articulation
C. skeletal muscle
D. gravity
E. nerve

31. Bouncing or exaggerated plantar flexion on midstance through toe off can caused by
.
A. hallus rigidus
B. metatarsalgia
C. Achilles tendon rupture
D. gastroc-soleus weakness
E. Achilles tendon contracture

32. In pronator teres syndrome all of the following muscles could be affected, EXCEPT....
A. pronator teres
B. pronator quadrates
C. flexor pollicis longus
D. flexor digitorum sublirnis
E. flexor digitorum profundus

33. The effects of cryotherapy is related to .


A. desensitivity of pain receptor
B. slowing of cell metabolism
C. systemic vasoconstriction
536
D. increased metabolic rate
E. localized vasodilatation

34. You are setting up an electrical stimulation unit to control pain through the gate-
control theory of pain modulation. The correct parameters for this are:
A. High pulse rate, long pulse duration, short treatment duration, motor level stimulation
B. Low pulse rate, short pulse duration, short treatment duration, sensory level
stimulation
C. High pulse rate, short pulse duration, moderate treatment duration, motor level
stimulation
D. High pulse rate, short pulse duration, long treatment duration, sensory
level stimulation
E. High pulse rate, moderate pulse duration, long treatment duration, noxious level
stimulation

35. What is the thermal effect of cold therapy?


A. Provide massage
B. Fascilitation of exercise
C. Mechanical debridement
D. Peripheral vasoconstriction due to cold
E. Decreases the tendency of blood pooling

36. The common "knuckle-bender orthosis" is used to....... .


A. A.stretch flexion contracture at the DIP joint
B. stretch flexion contracture at the PIP joint
C. stretch flexion contracture at the MCP joint
D. stretch extension contracture at the DIP joint
E. stretch extension contracture at the MCP joint

37. You are evaluating a female with rheumatoid arthritis who complains of bilateral foot
pain while standing and walking. She has moderate hallux valgus, claw toes,
subluxation and depression of the metatarso-phalangeal joints, as well as pes
planovalgus. You would prescribe.....
A. rigid molded plastic AFOs

537
B. bivalve molded maximum surface-bearing plastic AFOs
C. thermo-adjustable shoes (which contain Plastizote insoles)
D. double upright ankle-foot orthoses (AFOs) with double-action ankle
E. extra-depth shoes of deer skin with wide high toe boxes and custom-molded PVC.
petite insole(s)
38. At the final orthotic evaluation of AFO, you must be sure that the proximal
compomnent AFO do not impinge on the.
A. plantar muscle origin
B. posterior tibial nerve
C. popliteal artery
D. peroneal nerve
E. pes anserinus

39. The following is basic requirement for a stroke patient to joint group therapy in
occupational therapy.
A. Marked abnormal movement
B. Who lack of concentration
C. Have good sitting balance
D. Have marked spasticity
E. Severe receptive aphasia

40. The method frequently use on non-cooperative CP patients which bases treatment
on activation of postural development and equilibrium reaction to guide normal
development is..... .
A. Bobath Methode
B. Deaver method
C. Vojta method
D. Phelp methode
E. PNF methode

41. These are the effect of treadmill training in Parkinson patients EXCEPT .
A. It reduces tremor
B. It decreased rigidity
C. It decreased fall risk

538
D. It improves quality of life
E. It improves gait impairment

42. The Scoliosis Research Society ( SRS ) has defined a scoliosis as...
A. any curve which is greater or equal to 10 degrees with or without a
rotatory component in frontal plane
B. any curve which is greater or equal to 15 degrees with or without a rotatory
component in frontal plane
C. any curve which is greater or equal to 15 degress with or without a rotatory
component in sagital plane
D. any curve which is great or equal to 10 degress with a rotatory component in frontal
plane
E. any curve which is great or equal to 15 degress with a rotatory component in frontal
plane

43. A 25-year-old man comes to your office for evaluation of low back pain. As part of
the physical examination, you mark a point at the L5 vertebral body and another point midline
10 cm above. You ask him to flex forward maximally while keeping his knees extended and
measure the distance between the two points. This distance is 11 cm. You suspect he may
have what diagnosis?
A. Scheuermann disease
B. Lumbar herniated disc
C. Ankylosing spondylitis
D. Spondilitis tuberculosis
E. Lumbar spondylolisthesis

44. With adhesive capsulitis of the shoulder, the physical examination is most consistent
with limitations in.
A. active range of motion only
B. passive range of motion only
C. both active and passive range of motion
D. reduced strength and loss of active range of motion
E. no loss of range of motion but reduced strength of the muscles of the shoulder girdle

539
45. A 40 year-old woman presents with the acute onset of unilateral pes planus, swelling
and tenderness just distal to the medial malleolus and right ankle pain. On examination there is
unilateral hindfoot valgus and forefoot abduction. Your diagnosis is......
A. plantar fasciitis
B. navicular fracture
C. acute deltoid ligament strain
D. rupture of the Achilles tendon
E. posterior tibial is ligament rupture

46. Which of the following is the correct description of Yergason's test?


A. The examiner provides resistance against pronation of the forearm with the elbow in
extension
B. The examiner provides resistance against supination of the forearm with the elbow in
extension
C. The examiner provides resistance against supination of the forearm with the
elbow flexed at 90 degree
D. The examiner provides resistance against pronation of the forearm with the elbow
flexed at 90 degree
E. The examiner provides resistance against flexion of the elbow at 90 degree and the
forearm in complete supination

47. A 45 yo man with Multiple Sclerosis comes to the Neurology clime complaining of
urinary incontinence He indicites that he experiences increased urgency and frequency of
urination. The most likely urodynamic finding in this patient is ......
A. a spastic bladder
B. an atonic bladder
C. urge incontinence
D. stress incontinence
E. overflow incontinence

48. Lesion in the temporal lobe of the brain may result in .


A. primary progressive aphasia
B. expressive aphasia
C. receptive aphasia
D. anomic aphasia
E. global aphasia
540
49. The following statements are the pathophysiology and signs of Parkinson,
EXCEPT....
A. decreases of neuron dopaminergic
B. Lewy bodies in substansia nigra
C. paralysis agitans
D. resting tremor
E. bradykinesia
50. Contralateral hemiplegia which leg relatively more spared than hand and face
indicated that the lesion of stroke located in
A. middle cerebral artery upper division
B. middle cerebral artery lower division
C. middle cerebral artery main stem
D. posterior cerebral artery
E. anterior cerebral artery

51. Traumatic SCI with neurological deficit such as ipsilateral flaccid paralysis
(motor loss) at the level of the lesion, ipsilateral loss of all sensory modalities
at the level of lesion, ipsilatral loss of position sense and vibration below the lesion
and contralateral loss of pain and temperature below the lesion is indicated as .....
A. central cord syndrome·
B. anterior cord syndrome
C. posterior cord syndrome
D. Brown-Sequard syndrome
E. conus medullaris syndrome

52. A child with spastic hemiparesis is observed to have excessive plantar flexion and
inversion of the ankle during gait training, impairing his safety. An initial therapy to
improve his ambulation is..... .
A. ice packs to the gastrocnemius muscle for 30 minutes prior to ambulation
B. stimulation of the tibialis anterior at 3-5 Hz for 1 hour a day
C. an AFO with anterior channel spring
D. phenol block of the peroneal nerve
E. an AFO with medial T-strap

541
53. Acquired subluxation or dislocation of the hips in spastic cerebral palsy is usually due to
muscular imbalance and pull of the.....
A. knee extensor and hip abductors
B. hip flexors and tensor fascia lata
C. hip flexors and hip adductors
D. hip extensor and hip abductos
E. knee flexor and hip abductors

54. A 13-year-old girl is found to have scoliosis on routine physical examination. Spine
radiographs demonstrate a Cobb angle of25 degree. The optimal treatment is..... .
A. radiologic follow-up every 4 to 6 months
B. surgical instrumentation and bony fusion
C. use Milwaukee brace for 23 hours with exercise
D. lateral surface electrical stimulation with bracing for 6 hours a day
E. lateral surface electrical stimulation with trunk-strengthening exercise

55. You are examining a child of 7 to 8 months of age. Persistence of which one of the
following primitive reflexes or postural responses would you be concerned about?
A. Landau
B. Placing
C. Parachute
D. Plantar grasp
E. Asymmetric tonic neck

56. For chronic lung disease various walking test with sub maximal exercise have all the
benefit below, EXCEPT .
A. to detect exercise intolerance
B. to detect limitation in lung function
C. to know and detect functional limitation
D. to evaluate reconditioning exercise program
E. to prescribe the intensity of exercise program

57. Regarding exercise in congestive heart failure, which of the following is true?
A. Rapid hemodynamic changes may occur during warm-water aquatic therapy
542
B. Exercise heart rate should be at 80% to estimated maximum heart rate
C. Warm-up and cool-down periodes should be shortened
D. Strengthening should be done isommetrically
E. Blood pressure monitoring is not necessary

58. The following condition is NOT a contraindication for exercise program prescribed to
a diabetic patient with cardiovascular disease.
A. Blood sugar : 180-250 mg %
B. Blood sugar : > 300 mg %
C. Blood sugar : < 80 mg %
D. Acidosis
E. Ketosis

59. Among the following. which activity might be especially stress full. for the patient with
cardiac disease?
A. Leg raises
B. Outdoor walking
C. stretching exercise
D. Bridging exercise on a mat
E. Lower extremity cycle ergometry

60. Which statement is TRUE regarding calcitonin? D / E


A. It is a first line therapy for osteoporosis prevention and treatment
B. Its concomitant use with calcium is contraindicated
C. It is available in subcutan injection
D. It is available in an oral form
E. It has analgesic properties

61. The bisphosphonates alendronate and risedronate .


A. have excellent gut absorption
B. have no gastrointestinal side effects
C. act primarily by stimulating bone formation
D. have been shown to reduce vertebral and nonvertebral fractures

543
E. are no more effective on fracture reduction than hormone-replacement therapy,
selective estrogen receptor modulators and calcitonin

62. A 85-year-old man paraplegic who develops swelling and erythema in the left
lower extremity. Non invasive venous studies reveals a deep venous thrombosis extending into
the thigh. He receive therapeutic anticoagulation with intravenous heparin. You would be
give treatment program that include .....
A. Bed rest
B. Cold pack
C. Superficial heating
D. Ankle stretching exercise
E. Intermittent pneumatic compression devices

63. Which bony wrist injury related to forceful extension of the hand is commonly
seen in gymnasts?
A. Ulnar fracture
B. Radius fracture
C. Lunate fracture
D. Scapboid fracture
E. Hamatum fracture

64. One of the common low back injuries among Golfers is disc herniation. The
mechanism of the incidence is.
A. Overuse
B. Muscle cramp
C. Muscle imbalance
D. Rotation with axial loading
E. Axial load with forward flexion

65. This disorder is usually seen in preadolescent athletes who participate in activities
such as jumping or running. This disorders a result of recurring micro trauma from the
quadriceps contracting.
A. Patella fracture
B. Patellar tendinosis
C. Tibial plateu fracture

544
D. Pes anserinus bursitis
E. Osgood-Schlatter disease

66. A 18 yo basketball player suffered from chronic ankle instability. The statements
below is right, EXCEPT .....
A. Mechanical ankle instability can be caused by anterior talofibular ligament laxity
B. Chronic ankle instability consist of mechanical and functional ankle instability
C. This condition needs structural and comprehensive rehabilitation program
D. Functional ankle instability reflects the proprioceptive incompetence
E. This condition always needs surgery approach

67. The annulospiral ending is the primary afferent fiber coming from the.....
A. nuclear bag
B. nuclear chain
C. intrafusal fibers
D. extrafusal fibers
E. myoneural junction

68. A 54 years old man has complete foot drop for 15 days. EMG showed fibrillation
potentials & reduced mterference pattern of anterior tibial, peroneus longus, extensor halucis
longus and lumbosacral paraspinal muscles but not in the gastrocnemius. Nerve conduction
velocity of peroneal nerve is normal. The interpretation is .
A. L2 radiculopathy
B. L3 radiculopathy
C. L4 radiculopathy
D. L5 radiculopathy
E. S1 radiculopathy

69. The vital capacity is the sum of


A. inspiratory reserve volume plus tidal volume
B. inspiratory capacity and functional residual capacity
C. inspiratory capacity, tidal volume and expiratory reserve volume
D. inspiratory capacity, tidal volume and functional residual volume
E. inspiratory reserve volume, tidal volume and expiratory reserve volume
545
70. The most active muscles at heel strikes of the ipsilateral limb is......
A. sartorius
B. rectus femoris
C. gluteus minimus
D. gluteus maximus
E. semimembranosus

71. What the role of the gluteus maximus in providing spine stability?
A. It controls stance phase
B. It primarily functions as a hip flexor
C. It controls repetitive lumbar rotation_
D. It controls acceleration and deceleration
E. It seves as a primary muscle to fascilitate lumbar flexion

72. The following muscle is classified as monoarticular type.


A. Soleus
B. Gastrocnemius
C. Flexor carpi radialis
D. Flexor pollicis longus
E. Flexor digitorum superficialis

73. Which condition is indication for cervical traction?


A. Vertebrobasilar arterial insufficiency
B. Atherosclerosis of vertebral arteries
C. Thoracic Outlet Syndrome
D. Cervical root syndrome
E. Cervical HNP

74. Types of heat transfer via medium such as the movement of air or water is ....
A. evaporation
B. convection

546
C. conduction
D. radiation
E. electrical

75. The intensity of ultrasound prescription for tendonitis / bursitis is ......


A. 0.1-0.2 W/cm2
B. 0.3-0.4 W/ cm2
C. 0,5-2.0 W/cm2
D. 2.5-3.0 W/cm2
E. 3.5-4.0 W/cm2

76. A patient with DM has painful, swollen ankle, worst with weight bearing, X-ray shows
fragmentation of the articular surface, synovial fluid is clear. The choice of orthosis is....
A. AFO
B. KAFO
C. HKAFO
D. PTB orthosis
E. Orthopedic shoe

77. Which of the following is TRUE regarding the skeletal design of the lower extremity
prosthesis?
A. Endoskeletal more rugged
B. Exoskeletal is less durable
C. Endoskeletal tends to weigh less
D. Endoskeletal tends to require less maintenance
E. Exoskeletal is easily adjusted after fabrication

78. Spinal orthotic for maximum immobilization of the spine is .


A. Jewett brace
B. Taylor brace
C. Cowhom brace
D. plastic body jacket
E. Knight-Taylor brace

547
79. In a dynamic exercise, a lengthening of the muscle fiber is called .
A. Concentric contraction
B. Isokinetic contraction
C. Isometric contraction
D. Eccentric contration
E. Isotonic contraction

80. Performing a seated leg exercise using quadricep bench is an example of .


A. closed kinetic chain exercise
B. open kinetic chain exercise
C. mixed kinetic chain exercise
D. static kinetic chain exercise
E. isokinetic exercise

81. Massage does NOT exerts its therapeutic effect of the following mechanism.
A. Increased of muscle strength
B. Improved soft tissue blood flow
C. Stimulation oflocal opioid receptors
D. Mobilization of edema within soft tissue
E. Cutaneous stimulation of the autonomic nervous system.

82. The most commonly employed diagnostic maneuvers for Thoracic Outlet
Syndrome are all the following EXCEPT.....
A. Adson test
B. Neer's maneuver
C. hyperabduction test
D. shoulder depression test
E. three minutes arm elevation

83. A 35 year-old female hairdresser has complained of right shoulder and arm pain
lasting 4 weeks. The pain is located in the anterior and lateral shoulder region, radiating
laterally toward the elbow, with feeling of arm heaviness , and numbness in the right hands.

548
The pain occurs primarily while working and is relieved when lowering the arm. The
most likely diagnosis is
A. adhesive capsulitis
B. impingement syndrome
C. thoracic outlet syndrome
D. suprascapular nerve lesion
E. anterior shoulder instability

84. In Thoracic Outlet Syndrome, if the Adson test is positive, this suggested to the
following condition ....
A. hyperthropy of the stemocleidomastoideus muscle
B. hypertropy of the scalenus posterior muscle
C. hyperthropy of the pectoralis mayor muscle
D. cracture of the clavicle
E. Cervical rib

85. Conservative rehabilitation for rotator cuff tear on recovery phase (up to 6 months) is
to.....
A. reduced pain
B. increased flexibility
C. reduced inflammation
D. improve upper extremity range of motion and propioception
E. reestablish nonpainful and scapulohumeral range of motion

86. Which of the following is NOT the characteristic of radial nerve lesion?
A. Weakness of extensor carpi radialis
B. Weakness of extensor carpi ulnaris
C. Weakness of finger extensor
D. Drophand
E. Ape hand

87. Mrs A,69-year old suddenly has trouble in knowing how for or near an object is to her
body
and also very poor in decision making. It is possible that she suffers.....
549
A. right brain lesion
B. cerebellar lesion
C. mid brain lesion
D. left brain lesion
E. pons lesion

88. Predictors of hand movement and function after stroke, if within one month the
following is found .....
A. arm pronation
B. arm supination
C. lower arm spasticity
D. paresthesia of upper arm
E. Shoulder shrug and synergistic hand movements

89. Which of the following physical exam findings would be most consistent with a C6
radiculopathy?
A. Medial brachial sensory loss
B. Decreased wrist extensor
C. Decreased biceps reflex
D. Deltoid weakness
E. Triceps weakness

90. Prognosis for ambulation is a recurring issue when treating children with cerebral palsy.
A finding that has been documented to predict independent ambulation in cerebral palsy is .....
A. cruising by age 24 months
B. pulling to stand by age 24 months
C. sitting unsupported by age 24 months
D. kneeling supported by age 15 months
E. rolling supine to prone by age 15 months

91. The following are the characteristics of Duchene's Muscular Dystrophy,EXCEPT.....


A. EMG shows high amplitude, long duration MUAP with decrease recruitment
pattern on effort
B. the child's mother, his sisters and maternal aunts are potential carriers
550
C. measurement of serum creatinin kinase is is the most reliable test
D. it is transmitted by x-linked recessive mode in inheritance
E. the disease affects males

92. The child has chief complaint of bowlegs that are gradually progressing in severity and
not improving spontaneously. The deformity is called..... .
A. clubfoot
B. tibia vara
C. knock knees
D. geou valgum
E. genu recurvatum

93. The parents of a 4-year-old boy with Duchenne Muscular Dystrophy (DMD) ask your
opinion about the future of their son. Which of the following statements is NOT correct?
A. DMD typically becomes clinically evident at approximately 3 to 5 years of age
B. Weakness will beginning in the shoulder girdle muscles and later in the pelvis
C. Early difficulties noted are clumsiness, poor walking and frequent falls
D. Death is usually due to respiratory insufficiency
E. Intellect may be affected

94. During mild to moderate exercise, pulmonary ventilation increases primarily as a result
of increased .....
A. tidal volume
B. vital capacity
C. residual volume
D. total lung capacity
E. breathing frequency

95. From ECG examination, where is the localization of anterior myocardial infarction ?
A. Q waves in leads I, aVL, V5, V6
B. Q waves in leads VI, V2, V3, V4
C. Q waves in leads II, III, aVF
D. Q waves in leads I, UI, aVL
E. Tall Rwaves in leads Vl,V2
551
96. Mr.A with angina exhibits symptoms and 1 mm down-sloping ST segmen depression at
a HR of 129 bpm on his exercise test. His target HR should be set at. .

A. 90-99 bpm
B. 100-108 bpm
C. 109-119 bpm
D. 120-129 bpm
E. 130-139 bpm

97. A prerequisite to glossopharyngeal breathing is .


A. patent tracheostomy
B. good tongue strength
C. good abdominal support
D. good intercostals extemus strength
E. need some diaphragmatic activity to assist

98. In older patients undergoing a rehabilitation program after a hip fracture, benefit from
weight-bearing exercises can .
A. improve pain control
B. improve gait pattern
C. improve walking velocity
D. decrease risk of prosthetic failure
E. decrease incidence of hip dislocation

99. The three main sites of fracture associated with bone loss are .
A. spine, hip and wrist
B. spine, hip and knee
C. spine, hip and ankle
D. spine, hip and elbow
E. spine, wrist, and ankle

100. Functional assessment of instability in the older faller is .

552
A. vision.
B. strength.
C. feedback
D. flexibility
E. standing reach

ANSWER:
A. : IF NUMBERS 1, 2, 3ARE CORRECT
B. : IF NUMBERS 1, 3 ARE CORRECT
C. : IF NUMBERS 2,4 ARE CORRECT
D. : IF ONLY NUMBER 4 IS CORRECT
E. : IF ALL NUMBERS ARE CORRECT

101. In the case of low back pain, when we suspect Pager's disease, the laboratory test that
must be order to conform the diagnosis is/are D .
1. CPK
2. amilase
3. rheumatoid factor
4. alkaline phosphatase

102. Red flags oflow back pain is I are as follow (s). C


1. Not appropiate signs & symptoms
2. Systemic steroid
3. Low education
4. Saddle anastesia

103. Management of chronic back pain associated with osteoporotic vertebral fracture
should include E
1. a program of strengthening paravertebal, abdominal and gluteal muscles
2. relief of stress on the spine through use of proper body mechanic
3. safe program to improve flexibilitiy and balance
4. spinal orthosis

553
104. Rehabilitation management of osteoporosis patients depend on E .
1. the risk factors for osteoporosis
2. the degree of frailty and risk to fall
3. accurate determination of the degrees of bone loss
4. the capacity for participation in AD Ls and safe exercise

105. These are the rehabilitation programs for leprosy. E


1. Oil massage
2. Active exercise
3. Soaking the feet in water
4. Protection of the hands and feet

106. The mode of action(s) of diazcpam to reduce muscle spasticity is /arc through the
following mechanism(s). D
1. To stimulate P substance
2. To stimulate Glutamate
3. To stimulate Aspartate
4. To stimulate GABA

107. Rehabilitation treatment for Parkinson' s disease is/are ........ E


1. gait training, including speed and step length
2. fine motor training. such as writing
3. coordination exercise
4. endurance exercise

108. Important factor(s) that influence walking ability after stroke include..... E
1. balance
2. coordination
3. muscle strength
4. cognitive function

109. Rehabilitation of the Child with Congenital Brachial Plexus Palsies is I are C .
1. vigorous range of motion exercises
554
2. avoid stretching of the shoulder capsule
3. elbow pronation positioning
4. gradually range of motion exercises

110. Club foot consists of associated deformities. There is/are B .


1. forefoot varus
2. forefoot valgus
3. equinus or plantar flexion
4. valgus or eversion deformity of the heel

111. The key principle(s) in choosing an orthoses for children is/are A .


1. Reduce abnormal postures and tones
2. Enhance functional independence
3. Enhance normal movements
4. Heavy of the orthoses

112. What kind of exercise can be done as early rehabilitation approach(es) in ICU setting? E
1. Neuromuscular electrical stimulation
2. Stimulation of oromotor
3. Chest physical therapy
4. Self care training

113. The objective(s) of giving pursed-lips breathing is/are E .


1. introduce the patient of bow to control respiration
2. to decrease the respiratory rate
3. to increase tidal volume
4. to increase oxygenation

114. Diabetic angiopathy involves A .


1. large and small arteries
2. usually bilateral involvement
3. more commonly affects small arteries

555
4. more commonly affect aortoiliac arteries

115. Gait factors associated with falling in the elderly is/are ..... B

1. increase stride in length


2. increase stride in speed
3. increase time of double support phase
4. increase ofann swing during walking

116. The charactistic changes postur in elderly is/are ........ A


1. slighly flexed of knees and hips
2. increasing thoracic kyphosis
3. forward head posture
4. narrowed base of support

117. The benefit(s) of physical activities in the elderly include(s) the following. A
1. Slowing of disease progression
2. Promotion of the person's health
3. Prolongation of functional independence
4. Prevention of aortic aneurysm rupture

118. In prescribing pain treatment for elderly patient, it is best to use the following
approach( es): B
1. Start low and go slow for all medicine
2. Stan with aspirin, which is safe and effective
3. Keep on guard against the side effects
4. Use high dose, short-acting narcotics initially to get the pain under control

119. Medial stress of injuries of the elbow in throwing athletes, include(s) A .


1. ulnar nerve traction
2. avulsion of medial epicondyle
3. flexor muscle strain or tear
4. olecranon osteophyte formation
556
120. How does the anterior cruciate ligament (ACL) injury become torn ? B
1. Deceleration of the leg via quadriceps contraction combined with valgus and
external rotation forces upon a slightly flexed knee.
2. Sudden external rotation ofhyperllexed knee
3. Sudden hyperextension of the knee
4. Direct blows to the ankle

A basketball athlete had injured after be jump and landed on his right foot in the internally
rotate position. He heard "pop" and felt painful on his right knee. He ceased from this game
because he cannot walk and got an edema suddenly.
121. What is possible diagnoses for his condition? C
1. Quadriceps muscle rupture
2. ACL rupture
3. PCL rupture
4. Meniscal torn

122. What is /are special test will positive for his condition? C
1. Homan test
2. Mc Murray test
3. Sagging knee
4. Anterior Drawer test

123. Plantar flexor muscles is inactive by the time of ..... D


1. Mid stance
2. Heel strike
3. Foot flat
4. Toe off

124. A lesion at medial side of the Sciatic nerve will lead to weakness of the following
muscle(s)A
1. Semimembranosus
2. Semitendinosus
3. Gastrocnemius
557
4. Biceps femoris

125. Hamstring function as a ..... E


1. protectors of posterior structures from over stretching in combined movement of
hip flexion and knee extension
2. good extensors, rotators and weak add.uctorsof the hip
3. rotators of the knee with the knee flexion
4. good flexor of the knee

126. The difference between type I in contrast to type IIB skeletal muscle fibers is/are.... B
1. type I are predominantly aerobic
2. type I are predominantly anaerobic
3. type IIB are predominantly anaerobic
4. type IIB fibers are high hemoglobin content

127. The following is/are the characteristic (s) fibrillation waves. A


1. Have very small amplitude
2. Have very short duration
3. The rbytm is irregular
4. Diving bomber sound

128. With the EMG machine we can get the following result. A
1. The severity of pathology
2. The level I location of motor unit pathology
3. The motor and sensory conduction velocities
4. The etiology of motor unit abnormalities

129. The following statement(s) is/are TRUE about shortwave diathermy.....A


1. Inductive applicators produce more heat in deeper tissue
2. Capacitive plates produce more heat in the skin and superficial tissue
3. Inductive coil applicators produce the most heat in tissues that have high electrical
conductivity
4. To avoid burns during application, the patient's skin must be kept moist by wrapping
with wet towels
558
130. A variety of therapeutic gain(s) by electrical stimulation a muscle contraction
is/are .....A
1. muscle strengthening
2. recovery of neuropraxia
3. muscle pump contraction
4. prevents from Wallerian degeneration process

131. A 53-year-old woman with chronic venous disease has lower extremity edema,
skin hyperpigmentation, dermatitis, and venous varicosities. She does not have any
ulceration. The mainstay of treatment for this patient is /are ..... D
1. intermittent external pneumatic compression
2. hydrotherapy with warm water( 100 degree F)
3. topical steroid cream
4. gradient elastic stocking

132. functionally, a transmetatarsal amputation is superior to a Lisfranc's (tarsometatarsal)


amputation, because is provide(s) A
1. an intact tibialis anterior insertion to oppose the putl of the triceps surae
2. a stump that is easier to fit
3. a broader base of support
4. a longer lever arm

133. Phantom pain ..... C


1. is an expression of wish fulfillment
2. does not occur with congenital absence of a limb
3. indicates an emotional maladjustment
4. is accentuated and perpetuated by delayed wound healing

134. The manifestation of circumduction gait of amputee who is wearing a lower limb
prosthesis usually caused by ..... A
1. socket too small, the residual limb cannot enter fully
2. inadequate suspension, socket slips down during swing
3. amputee is reluctant to flex the knee during swing because of poor balance

559
4. amputee does not bother to flex the knee because the prosthesis is too short

135. Which activity(ies) is/are included in energy conservation method below? C


1. Work with antigravity assisting
2. Sit to work when possible
3. Rest after fatigue
4. Plan ahead

136. About Frenkel's exercises for ataxic conditions ..... E


1. design primarily for coordination
2. they are not intended for strengthening
3. commands should be given in an even, slow, monotonous voice
4. ability of patients to interpret deep muscle and joint sensibility may be checked by
having the patient perform the exercises with eye closed

137. The purpose(s) of an initial comprehensive assessment of sensory integrative problem


is/are A
1. to identify the impact of sensory processing and praxis problems
2. to provide information to assist with treatment planning
3. to identify specific sensory integrative problems
4. to identify the behavior problem

138. The following is/are the risk factor(s) for osteoporosis fractures. A
1. Personal history of low impact fracture
2. Inadequate physical activity
3. Current low BMD
4. High body weight

139. What is/are the indication(s) for osteoporosis investigation? E


1. Prolonged treatment with cyclosporine
2. Premature menopause (before age 40)
3. Chronic renal disease
4. Hiperparatbyroid

560
140. Joint protection techniques in patients with osteoarthritis include the following
principle(s)E

1. Use appropriate assistive devices


2. Avoid activities that hurt affected joints
3. Avoid staying in one position for too long
4. Use largest and strongest joint and muscles

141. About Rheumatoid Arthritis . E


1. Is an auto immune disease
2. Erotion of cartilages
3. Chronic synovities
4. Erotion of bones

142. A 20 year-old C 7 quadriplegic patient should be expected to perform following


activities independently by ..... A
1. sitting pivot transfer
2. sliding board transfer
3. wheelchair- to- car transfer
4. floor to wheelchair transfer

143. Expectation for ADL function by C6 injury is/are E .


1. feed, dress upper body with setup.
2. dressing lower body possible
3. forward weight shifts
4. side to side weight shifts

144. The blindness occurred in leprosy patients is/are caused by the involvement of the
following nerve(s). B
1. Optic nerve
2. Trigeminal nerve
3. Occulomotor nerve
4. Facial nerve
561
145. The most useful developmental milestones is/are C ..

1. social interaction (5 years)


2. gross motor: walking (12 months)
3. fine motor: pincer grasp (l 8 months)
4. speech: 10 words/body parts (18 months)

146. The following is /are the indicator(s) that child bad good prognosis for future walking.
B
1. Head control at the age 9 months
2. Moro response should be present at the age 12 months
3. Sitting balance at the age 2 years
4. Asymmetric tonic neck reflex should be present at the age 12 months

147. A 14-year-old boy with hemophilia has bad recurrent hemarthrosis of the knee in the
past, but presently has full ROM and normal quadriceps muscle strength. His parents ask
advice regarding appropriate sport activities. You strongly support an active life style, but
advise AGAINST ..... D
1. cycling with moderate speed
2. walking with moderate speed
3. competitive swimming
4. competitive basketball

148. The goals of Cardiac Rehabilitation program is/are ..... E


1. to improve quality of life
2. to maximize cardiovascular capacity and fitness
3. to utilize energy conservation and work simplification
4. to maximize exercise tolerance and activities of daily living ( ADL)

149. The absolute contraindication for entry inpatient and outpatient exercise testing
is/are..... E
1. unstable angina
2. thrombophlebitis

562
3. moderate to severe aortic stenosis
4. uncompensated congestive heart failure.

150. Man, 55 year old, weight 85 kg, height 165 cm. He got myocard infarct 1 month ago.
The fasting glucose level 250, total cholesterol 400, no hypertension. The factor(s) that
should be considered to prescribe cardiac rehabilitation program is/are ..... C
1. avoid all of exercise
2. assessed the stress test
3. limitation of daily activity
4. reduce the risk factor for 2nd cardiac disease attack

563
MARET 2019

1. A 38-year-old with rheumatoid arthritis of neck pain with occipital head … of the
cervical spine is obtained which demonstrates atlantoaxial subluxation. Your … this time
before you recommend surgical treatment is … B
a. Use Thomas collar
b. Use of a Hallo brace
c. Use Philadelphia collar
d. Treatment with TENS
e. Botulinum toxin injections of the trapezii
2. Which nerve is easily getting injured on humerus fracture? C
a. Ulnaris
b. Radialis
c. Axillaris
d. Medianus
e. Musculocutaneus

3. A 50-year-old man with type 2 diabetes mellitus presents for evaluation of a swollen
ankle … examination demonstrates diminished pinprick sensation in a stocking pattern, absent
… jerks, and a warm, erythematous right ankle with a mild effusion. What is the most …
diagnosis relating to the right ankle? A
a. Charcot joint
b. Septic arthritis
c. Avascular necrosis
d. Rheumatoid arthritis
e. Ankylosing ankle joint

4. 24 year man develops insidious of chronic low back pain with morning stiffness. A
diagnosis of ankylosing spondylitis requires …
a. a positive HLA B27 test
b. a positive gaenslen’s test
c. an elevated erythrocyte sedimentation rate
d. ophthalmologic confirmation of acute anterior uveitis
e. radiographic demonstration of sacroiliac joint abnormality

564
5. Boutenierre deformity in a Rheumatoid hand is typically located in the following joint.
a. PIPs
b. MCPs
c. Ulnarcarpal
d. Radiocarpal
e. Carpometacarpals

6. Physical examination of spasticity, you find there is a slight increqse in muscle tone, ,,,
a catch, followed by minimal resistance throughout the reminder (<50%) of the …, you will
grade this patient with modified Ashworth Scale as …
a. 1
b. 1+
c. 2
d. 3
e. 4

7. Inability to perform “OK” sign is the characteristics of …


a. Pronator syndrome
b. Carpal tunnel syndrome
c. Radial neuropathy syndrome
d. Anterior interosseous syndrome
e. Posterior interosseous syndrome

8. What kind of exercise beneficial to improve decrease of laryngeal elevation during


swallowing?
a. Shaker exercise
b. Masako maneuver
c. Mendelsohn maneuver
d. Effortful swallow maneuver
e. Supraglottic swallow maneuver

9. Where is the lesion site in ataxic dysarthria, as found in Friedreich’s ataxia ?


a. Cerebellum
565
b. Multiple sites
c. Basal ganglia
d. Extrapyramidal system
e. Bilateral upper motor neuron

10. Which finding is associated with a more favourable prognosis in Amyotrophic Lateral
Sclerosis (ALS) ? B
a. Female sex
b. Young age and male sex
c. Bulbar involvement presentation
d. Short time period from symptoms to diagnosis
e. Predominance of lower motor neuron findings on EMG studies

11. The following statements are true regarding therapy for cerebral palsy EXCEPT …
a. Hippotherapy users horse as a tool in physical therapy
b. Strengthening exercise would increase spasticity and abnormal reflexes
c. Tone reduction after Botulinum toxin injections lead can last 3-6 months
d. Constraint-induced therapy techniques to improve upper extremity in hemiplegia
e. The Bobath treatment is based on normalizing movement patterns and inhibiting
abnormal reflexes

12. The period which intense motor learning and basic language development occur is at …
a. birth to sixth month
b. birth to tenth month
c. birth to one year of age
d. birth to two years of age
e. birth to three years of age

13. What is the strongest single predictor of mortality in adults with pediatric onset
disabilities?
a. Inability to walk
b. Feeding problems
c. Intellectual disability
d. Head circumference
566
e. Presence of epilepsy

14. The trainer for People With Disability (PWD) in CBR area is ?
a. Family
b. Cadres
c. Community
d. Formal leader
e. Informal leader

15. When treating patients with cerebral palsy, consider … A


a. Vocational rehabilitation for job training and placement
b. KAFO for all ambulatory patients to improve gait pattern
c. Spinal bracing to prevent progressive deformity of scoliosis
d. Limited strengthening as it has been shown to increase spasticity
e. AFOs only in nonambulatory patients for positioning and contracture prevention

16. The common etiologies for cerebral aplsy include all the following, EXCEPT … E
a. Prematurity
b. Cerebral ischemia
c. Hyperbilirubinemia
d. Cerebral hypoxemia
e. Vitamin C deficiency

17. What is the expected outcome of effective exercise training in a pulmonary


rehabilitation … for patients with chronic obstructive pulmonary disease (COPD)?
a. Reduced dyspnea
b. Reduced need for supplemental oxygen
c. Slowed loss of forced expiratory volume
d. Improved radiologic status of the lungs fields
e. Reverse the process of pulmonary anatomy to the normal condition

18. Which exercise has the greatest effect on increasing blood pressure?
a. Cycling
567
b. Marathon
c. Swimming
d. Weight lifting
e. Tai chi stretching

19. Physical activities advised to patients with CHF functional class III (NYHA) is…
a. 1-2 METs
b. 3 – 4 METs
c. 5 – 6 METs
d. 7 – 8 METs
e. 9 – 10 METs

20. Exercise for diabetic patients is beneficial because … B


a. Increase blood glucose level
b. Increase insulin sensitivity
c. Increase insulin resistance
d. Decrease HDL
e. Increase LDL

21. What is the most common cause of death in patients with ventilator-dependent
tetraparesis … who have survived the first 24 hours? E
a. Pressure ulcer infection
b. Pulmonary embolism
c. Pneumothorax
d. Renal failure
e. Pneumonia

22. Assuming the use of prostheses, which lower extremity amputation requires the greatest
… energy expenditure for ambulation?
a. Below knee
b. Hemipelvectomy
c. Knee disarticulation
d. Bilateral above knee

568
e. Unilateral hip disarticulation

23. Which cardiorespiratory outcome would you expect from an elderly person participating
aerobic program?
a. No change in stroke volume and a reduction in systemic vascular resistance
b. A decrease in VO2 max but an increase in walking speed
c. No change in tolerance during activities of daily living
d. No change in thorax expansion
e. Improvement in VO2max

24. Aging in neurologic aspect makes …


a. Shifting of brain activity from posterior to anterior region
b. Frontal gray matter gain
c. Increased brain volume
d. Increased blood flow
e. Cortical thicking

25. Anorexia of aging is meaning …


a. Increased energy intake
b. Increased stomach acid production
c. Hyperchlorhydria lead to bacterial overgrowth
d. Decreased appetite caused by decreased smell and taste sensation
e. Decreased gastric compliance causing late satiety and shortened postprandial satiety

26. A 65-year-old woman with breast cancer has a solitary metastatic lesion at the left rib
…. Among the following, the first-line pain medication in this setting is …C
a. Ibuprofen
b. Amitriptyline
c. Acetaminophen
d. Morphin sulfate
e. Local corticosteroid injection
27. After mastectomy, full assisted shoulder ROM exercises should be initiated … E
a. At 24 hours

569
b. At three days
c. At one week
d. To patient tolerance
e. After removal of the surgical drains

28. The following are the true statements about ankle sprain, EXCEPT … D
a. The most common injured ligament is anterior talofibular ligament
b. The provocative tests are anterior drawer test and talar tilt test
c. The mechanism of injury is inversion on a plantar flexed foot
d. The mechanism of injury is eversion on a plantar flexed foot
e. History of rolling over the ankle

29. The following statement is one of the basic principle of rehabilitation protocol after
anterior cruciate ligament (ACL) injury
a. Appropriate use of open/losed kinetic chain exercise, avoiding early open chain
exercise that may shear or tear the weak immature ACL graft
b. Exercise for lower extremity usually done to gain muscular endurance rather than
flexibility and strength
c. Control of swelling and pain is not the important intervention in initial rehabilitation
program
d. Initiation of quadriceps and hamstring activity can be postponed

30. A 22-year-old female gymnast presents to your clinic after a patellar dislocation
during… She wastreated in the emergency room with reduction of the patella and …
Radiographs and MRI of the knee are negative fracture or evidence of … lesions. You choose
to treat her with immobilization for 2 weeks and then begin therapy. The most appropriate
therapy recommendation is to focus on improving
a. Strength of the iliopsoas
b. Strength of the hamstring
c. Flexibility of the biceps femoris
d. Strength of the vastus medialis
e. Flexibility of gastrocnemius-soleus complex

31. Following disorder is usually seen in preadolescent athletes who participate in activities
jumping or running. The disorder is a result of recurring microtrauma from the quadriceps
contracting (cucur 752)

570
a. Pes anserinus
b. Ligamen sprain
c. Pes anserinus bursitis
d. Tibial plateau fracture
e. Osgood schlatter disease

32. A 22-year-old female gymnast presents to your clinic after a patellar dislocation during
practice. She was treated in the emergency room with reduction of the patella and
immobilization. Radiographs and MRI of the knee are negative fracture or evidence of
osteochondral lesions. You choose to treat her with immobilization for 2 weeks and then begin
physical therapy. The most appropriate therapy recommendation is to focus on improving …
a. Strength of the iliopsoas
b. Strength of the hamstring
c. Flexibility of the biceps femoris
d. Strength of the vastus medialis
e. Flexibility of gastrocnemius-soleus complex

33. The smallest unit of muscle contraction is … B


a. Motor unit
b. Sarcomere
c. Sarcolemma
d. Actin, myosin
e. Intrafusal fibers

34. What is the most common abnormal electrodiagnostic finding in lumbar spinal stenosis?
C
a. Normal H-reflex
b. Abnormal sensory nerve conduction
c. Decreased chronodispersion of F-wave testing
d. Presence of fasciculation and cramp potentials
e. Fibrillation potentials in multiple and bilateral myotomes

35. Which information is useful in the electrodiagnostic evaluation of a 9-year-old child


with spinal cord injury who has no radiologic abnormality but has sensory abnormailities? B
a. Repetitive stimulation recording
571
b. Somatosensory evoked potential
c. Sensory nerve conduction study
d. Motor nerve conduction study
e. Magnetic evoked potential

36. Typical median nerve conduction changes in patient with carpal tunnel syndrome show
… cucur 408
a. Prolonged sensory distal latency
b. Prolonged motor distal latency
c. Low amplitude of actin and myosin
d. Abnormal F waves
e. Abnormal H waves

37. All of the following factors influence the strength of muscle contraction, EXCEPT …
a. The amount of ATP
b. The muscle volume
c. The amount of actin and myosin
d. The number of motor units which are active
e. The starting point/position in which muscle start to contract

38. Skeletal muscle tipe I fibers are predominantly … A cucur 343 dan 350
a. Red fibers
b. White fibers
c. Are an aerobic
d. For weight lifting
e. Are rapid contracting fibers

39. Skeletal snapping hip syndrome is caused … cucur 223


a. A tight gluteus maximus snapping over the greater trochanter
b. A tight iliopsoas tendon snapping over the lesser trochanter
c. An acetabular labral tear or lose body in the hip joint
d. A tight gluteus medius over the lesser trochanter
e. A tight iliotibial band over the greater trochanter
572
40. Maximal flexion of the knee in the gait cycle is reached … cucur 473
a. In mid stance
b. In heel strike
c. In the middle of swing phase
d. Immediately after the heel strike
e. During acceleration in swing phase

41. The weakest spot in the posterior lumbar region which is not strong … longitudinal
ligament is …
a. L1-2
b. L2-3
c. L3-4
d. L4-5
e. L5-S1

42. Complete loss of strength of gluteus maximus is compensated by the patient … phase of
the gait cycle by …
a. Increase plantar flexion
b. Maintaining knee flexion
c. Hyperextension of the spine
d. External tibial rotation at heel strike
e. Increased activity in the rectus femoris

43. Concentric activation of the iliopsoas muscle is essential to which phase of the gait
cycle?
a. Preswing
b. Midstance
c. Heel strike
d. Terminal stance
e. Terminal swing

44. The pretibial muscle group is most active at what phase of gait cycle?
a. At toe off
573
b. At mid swing phase
c. At late swing phase
d. At mid stance phase
e. Immediately following heel strike

45. Methods of treating acute inflammatory tendinitis include all of the following, EXCEPT

a. Use of ultrasonic heating
b. Application of cold packs
c. Splinting of the involved tendon
d. Local steroid and anesthetic injection
e. Administration of oral anti-inflammatory agents

46. The following is NOT used for lymphedema


a. Compression garmen
b. Superficial heating
c. Elevation
d. Stroking
e. Massage

47. A therapeutic modality theorized to exert its main effect by the “gate control theory” of
pain is …
a. USD
b. TENS
c. Contrast bath
d. Laser therapy
e. Magnetotherapy

48. When standing upright in water with surface level at the level of papilla mammae, the
relative body weight is …
a. 5% weight bearing
b. 15% weight bearing
c. 25% weight bearing
d. 35% weight bearing
574
e. 45% weight bearing

49. What is a contraindication for superficial heat?


a. Superficial thrombophlebitis
b. Joint replacement
c. Muscle spasme
d. Sensory deficit
e. Hematoma

50. Which modality most effectively treats the chronic periarticular tissue of the hip joint?
a. Hot packs
b. Infra red radiation
c. Shortwave diathermy
d. Microwave diathermy
e. Ultrasound diathermy

51. In a person with a transfemoral amputation, an abnormal prosthetic gait with lateral
trunk bending forward the involved side in midstance most commonly occurs with hip … D
a. Extension contracture
b. Adductor weakness
c. Flexion contracture
d. Abductor weakness
e. Eversion weakness

52. A boy, 6-year-old with Prader-Willi syndrome, associated hypotonia, has hypermpbile
flat feet requiring treatment. These are best managed by … A
a. Double-upright short leg brace and medial T strap
b. Molded plastic shoe inserts for the talus support
c. Orthopaedic surgery for a triple arthrodesis
d. High-top shoes with Thomas heel
e. Soft plastizole shoe insert

575
53. The best rational choice of wheelchair for a C8 tetraplegia ASIA A is … E
a. Regular wheelchair with solid tyre
b. Hand-operated electrical wheelchair
c. Mouth-operated electrical wheelchair
d. Regular wheelchair with aired/inflated tyre
e. Regular wheelchair with vertical bars on its handrims

54. A patient with DM has painful, swollen ankle, that become worst with weight bearing.
X-ray shows fragmentation of the articular surface, synovial fluid is clear. The choice of
orthosis isE
a. AFO
b. HKAFO
c. Diabetic shoe
d. KAFO with medial pad
e. Patellar tendon bearing orthosis

55. The prehensile function of a myoelectric hand simulates …


a. Three-jaw chuck
b. Spherical
c. Lateral
d. Power
e. Pinch

56. The proper length for a cane should be measured with the elbow in which position?
a. Full extension
b. 20 degrees of flexion
c. 45 degrees of flexion
d. 60 degrees of flexion
e. 90 degrees of flexion

57. The proprioceptive Neuromuscular Facilitation (PNF) is a neurophysiologic therapeutic


… approach for CP, developed by …
a. Fay
b. Rood
576
c. Bobath
d. Brunnstrom
e. Kabat and Knott

58. Parkinson disease patients may improve in walking by the following exercise
a. Voyta
b. Knott
c. Kabat
d. Bobath
e. Frenkel

59. Diabetic patient with peripheral neuropathy SHOULD NOT perform the following
exercise
a. Jogging
b. Rowing
c. Bicycling
d. Swimming
e. Light weight lifting

60. Which condition would qualify as an impairment according to the International


Classification of Function (ICF)?
a. Loss of hearing due to occupational exposure
b. Inability to wotk due to chronic illness
c. Inability to walk one block
d. Supported by insurance
e. Age

61. The best example of an open kinetic chain exercise of the quadriceps is …
a. An isokinetic knee extension machine
b. A wind-resistance exercise bicycle
c. A treadmill inclined to 7 degrees
d. A stair climbing machine
e. Isometric exercise

577
62. For a patient with dysphagia following stroke, the chin-tuck maneuver is effective
because it decreases …
A. Esophageal sphincter excursion
B. Pharyngeal peristalsis
C. Esophageal peristaltic
D. Pharyngeal pressure
E. Airway opening

63. Para Articular Heterotropic Ossification in SCI patient most commonly occurs at the
following joint
a. Hip
b. Knee
c. Ankle
d. Elbow
e. Shoulder

64. a common source of shoulder pain in shoulder impingement syndrome is…


a. bursitis
b. osteoarthritis
c. short head of biceps
d. long head of biceps
e. the rotator cuff tendon

65. a 35 year old package delivery truck driver was present with the insidious onset of right
buttock and posterior thigh pain that aggravated by sitting and using foot pedals. Examination
was reneal external rotation of the right thigh shorthening, but no weakness or sensory or reflex
loss. The treatment should include…
a. pelvic traction
b. a lumbosacral corset
c. a right shoe insert and heel lift
d. infrared to gluteal muscles
e. stretching of the buttocks and hip muscles

578
66. when burn wound is around the perineum and both lower extremities, the preferred
position are the following, except…
a. hip abduction
b. kne extension
c. ankle dorsiflexion
d. pillow under the knee
e. hip in neutral position

67. the following are the characteristics of duchene’s muscular dystrophy, except…
a. EMG shows high amplitude, long duration MUAP with decrease recruitment
pattern on effort
b. The child’s mother, his sisters and maternal aunts are potential carriers
c. Measurement of serum creatinine kinase is the most is the most reliable test
d. It is transmitted by x linked recessive mode in inheritance
e. The disease affects males

68. The following muscles are usually affected by a spiral groove lesion, except for the…
a. Triceps
b. Brachioradialis
c. Extensor carpi radialis
d. Extensor carpi ulnaris
e. Extensor digitorum communis

69. Anosognosia commonly occurs in patients with stroke. The following is the clinical sign

a. Lack of recognition of illness
b. Loss of two-point discrimination
c. Inability to orient objects in three dimension
d. Inability to distinguish colored visual stimuli
e. Increased risk of seizures from olfactory stimulation

70. The best treatment of spasticity in SCI is …


a. Botox im
b. Baclofen iv
579
c. Baclofen oral
d. Diazepam iv
e. Baclofen intrathecal

71. The following nerve is a branch of the posterior cord of the brachial plexus
a. Long scapular nerve
b. Subscapular nerve
c. Suprascapular nerve
d. Levator scapular nerve
e. Musculocutaneus nerve

72. Pronator Teres syndrome is caused by entrapment of the following nerve


a. Ulnar nerve
b. Radial nerve  kunci
c. Median nerve
d. Anterior interosseous nerve
e. Posterior interosseous nerve

73. A 7-day-old infant is referred to you … history of traumatic delivery, and the physical
examination … and below the right ear. Clinically, the child has a moderately severe right …
nerve palsy. You advise the parents that …
a. Protection of the eye is unnecessary
b. Weekly electrodiagnostic studies are necessary
c. Spontaneous recovery occurs in the majority of cases
d. Surgical decompression may be helpful at 1 month of age
e. Electrical stimulation of the right fascial muscles speeds recovery

74. A birth defect that arises after neurulation (neural tube closure) rather than during
neurulation is
a. Anencephaly
b. Sacral lipoma
c. Meningocele
d. Cranioarachischisis
e. Spina bifida cystic
580
75. There are several methods of therapy in the treatment of cerebral palsy. The rood method …
a. Consist of passive exercise designed to replicate phylogenetic progression and enlist a two-
person team to use set patterns for several hours per day
b. Focuses on altering abnormal posture, reducing or increasing tone, and developing
movement patterns in keeping a normal developing sequences
c. Emphasize stimulation techniques that fascilitate or inhibit specific motor … and is
aimed at teaching awareness or normal patterns
d. Emphasizes extensive bracing to maintain a correct erect posture
e. Paertains to learning disabilities and sensory integration

76. In the management of scoliosis in a nonambulatory child with spastic cerebral palsy …
a. Location of the most inferior force of a three-point system at a greater distance from
superior forces decreases effectiveness of its leverage
b. Three-point forces schemes are much more effective at stabilizing multijoint than single-
joint deformities
c. A seating system is more likely to correct a progressive spinal deformity than is a suitable
spinal orthosis
d. Lateral tilting of the pelvis in the direction of the convexity of the curve is …
e. pelvic orientation is usually under voluntary control

77. the primary focus of rehabilitation for a 2 year old child newly diagnosed with cystic
fibrosis should be…
a. airway secretions management
b. general physical conditioning
c. respiratory muscle exercise
d. respiratory muscle rest
e. anxiety reduction

78. The following are true regarding developmental milestone EXCEPT


a. Walking : 12 months
b. babbling : 12 months
c. Pincer grasp : 10-12 months
d. Speech body part : 18 months
e. Social skill interaction : 4 years

581
79. Persons with COPD typically experience exertional dyspneu when FEV1 become less than
a. 1500 ml
b. 1800 ml
c. 2000 ml
d. 2200 ml
e. 2500 ml

80. For a 60-year-old man who has had an uncomplicated myocardial infarction, exercise
training of 3 months duration improves all of the following during exercise, EXCEPT B
a. Time to onset of ischemic changes on ECG
b. Amplitude of ST segmen depression
c. Duration of exercise blood pressure
d. Blood pressure
e. Heart rate

81. Patient with COPD improved exercise tolerance is primarily due to …


a. Improved efficiency in peripheral muscle O2 utilization
b. Increased resting arterial partial pressure of O2
c. Strengthening of the diaphragm
d. Increased minute ventilation
e. Increased FEV1

82. postural drainage has been used as a form of chest physical therapy to enhance the flow of
mucus of airways. However, one should avoid this modality in patients…
a. with pulmonary edema
b. using nebulizer therapy
c. who have difficulty coughing
d. using inhalant bronchodilators
e. with sputum production greater than 30 ml per day

83. A 53-year-old woman with chronic venous disease has lower extremity edema, skin
hyperpigmentation, dermatitis, and venous varicosities. She does not have any ulcerations.
Mainstay of treatment for this patient is …
a. Topical steroid cream
582
b. Gradient elastic stockings
c. Hydrotherapy with warm water
d. Intermittent external pneumatic compression
e. Elevation of the lower extremities as much as possible

84. A 45-year-old man with post myocardial infarction, has been complicated by congestive
heart failure. He has been reffered to your inpatient acute cardiac rehabilitation program. He is
anxious about his condition and initially refuses to participate. You explain to him that the
major goal of acute cardiac rehabilitation is to
a. Accelerate myocardial healing
b. Evaluate the myocardial exercise tolerance
c. Prevent muscular and neurovascular deconditioning
d. Stimulate the development of coronary collaterals
e. Begin to conditioning process to improve myocardial efficiency

85. A 69-year-old woman wishes to reduce her risk of compression fractures and is interested
in taking alendronate. You inform her that…
a. Alendronate increases BMD by increasing osteoblastic activity
b. Gastrointestinal absorption of alendronate is facilitated by calcium
c. Although fracture risk is reduced with the use of alendronate, BMD unaffected
d. Use of alendronate by women with existing vertebral fractures decrease further
fractures
e. The most common side effects associated with use of alendronate
cardiovascular and neurologic systems

86. Postmenopausal women should ingest how many miligrams of calcium daily?
a. 500
b. 1000
c. 1200
d. 1500
e. 2000

87. After Alzheimer disease, the most frequent dementia in the elderly is secondary to …
a. Sepsis
b. Drug toxicity
583
c. Multiple infarcts
d. Subdural hematoma
e. Occult hydrocephalus

88. Which exercise has the greates effect on bone formation?


a. Yoga
b. Cycling
c. Swimming
d. Weight lifting
e. Tai Chi stretching

89. A 29-year-old cross-country runner experiences new-onset pain in the region of the first
metatarsal. Exam reveals tenderness medial to the first metatarsal head worsened by passive
great to extension. The cause of her sesamoid injury include of the following, EXCEPT
a. Pes planovalgus
b. Early hallux rigidus
c. Excessive pronation
d. Excessive supination
e. Gastroc-soleus tightness

90. A patient complains of pain inferior to the anteromedial surface of the knee, especially
while climbing stairs. On examination, there is tenderness to palpation over the proximal
anteromedial tibia. What is the most likely diagnosis?
a. Osteoarthritis
b. Patellar tendinitis
c. Prepatellar bursitis
d. Pes anserinus bursitis
e. Iliotibial band syndrome

91. 20-year-old football player reports anterior shoulder pain during a game. He … game, but
radiographs after the game revealed a type 2 acromioclaviculare joint sprain. How is a type 2
acromioclavicular injury defined?
a. Dislocation of coracoclavicular joint
b. Acromioclavcular and coracoclavicular ligaments are both intact
c. Acromioclavicular and coracoclavicular ligaments are both disrupted
584
d. Acromioclavicular liogament is disrupted, but the coracoclavicular ligament is intact
e. Acromioclavicular ligament is intact and the coracoclavicular ligament

92. A 55-year-old runner presents with pain in the region of the Achilles tendon. This the third
such episode over the past 4 years. The Achilles tendon appears swollen. Appropiate initial
management would include …
a. Surgery
b. Use AFO
c. Cortisone injection of the Achilles tendon sheath
d. Use of a small heel lift with initiation of gastro-soleus stretches as tolerated
e. Immobilization in an ankle plantar-flexed position with limited weight bearaing

93. A 27-year-old manual labourer was present with a-6-month history of right shoulder pain.
He has a past medical history of a right shoulder dislocation after a water-skiing accident 3
years ago. On physical examination, he had normal strength and sensation with symmetric
reflexes. The shoulder apprehension test was positive. Impingement tests and the O’Brien
active compression test were negative. Which diagnosis was most consistent with this
presentation?
a. Rotator cuff tendinitis
b. Glenohumeral osteoarthritis
c. Dislocation of biceps tendon
d. Anterior-inferior labrum tear
e. Superior labral anterior to posterior (SLAP)

94. A 33-year-old cab driver was involved in a rear-end motor vehicle crash and hit his knee
against the dashboard. He presents with a six-week history of knee pain, a positive posterior
drawer sign, and difficulty walking down inclines. What is the most important muscle group to
strengthen?
a. Hip flexors
b. Knee flexors
c. Hip adductors
d. Hip extensors
e. Knee extensors

95. A patient recently fell onto her outstretched hand with the wrist dorsiflexed and radially
deviated. Physical examination is suggestive of a scaphoid fracture. Radiograph of the wrist
and hand, including special scaphoid views, are negative. The appropriate initial treatment is …
585
a. Observation
b. Kinesiotaping
c. A cast or splint
d. An elastic bandage
e. Operative intervention

96. Mallet finger deformity results from rupture of the …


a. Extensor tendon insertion
b. Distal collateral ligaments
c. Extensor indicis muscle
d. Flexor profundus tendon
e. Flexor indicis muscle

97. Which clinical disorder is most likely to produce detrusor hypereflexia?


a. Poliomyelitis
b. Tabes dorsalis
c. Multiple sclerosis
d. Muscular dystrophy
e. Amyotropic Lateral Sclerosis

98. Chronic radicular pain in patients with a spinal cord injury (SCI) is most effectively treated
with
a. Analgesic
b. Anticonvulsants
c. Muscle relaxants
d. Narcotic analgesia
e. Nonsteroid anti-inflammatory drugs

99. In an older adult who previously had polio, new weakness is most consisistent with
postpolio syndrome when it occurs …
a. In proximal muscle groups of both the upper and lower extremities
b. In distal muscle groups of both the upper and lower extremities
c. In the muscles most severely involved in the initial illness

586
d. After recent weight gain
e. After recent disuse

100. To reduce flexion hypertonicity at the elbow following a stroke, the muscles to be consider
for botulinum toxin injection include the biceps brachii, brachialis, and…
a. Pectoralis major
b. Coracobrachialis
c. Brachioradiais
d. Teres mayor
e. Teres minor

101. The following statement(s) abpout primary bone healing is/are TRUE E
1. Occurs 2 weeks after injury
2. Intramedullary vascularity has to be intact
3. Requires direct contact depends on the part of bone’s cortex
4. The healing process depends on resorption of osteoclast followed by osteoblast in
order to form new bone

102. The prescription of exercise for the patient with simple recurrent low back pain should
achive at least the following E
1. Improved low back flexibility
2. Improved posture with minimized lumbar lordosis
3. Improved body mechanics in all activity and exercise
4. Improved strength of abdominal and hip extensor muscle

103. What is the rationale of giving quadricpes strengthening exercise in a patient with
osteoarthritis of the knee joint? D
1. To strengthen the joint tissue
2. To achieve a better level system
3. To increases the force acting on the joint
4. To decrease compressive forces on the joint

104. The principle(s) of burn rehabilitation is/are................... A


1. Lubrication
587
2. Hydrotherapy
3. Splints are useful for passive positioning
4. USD using stationary transducer with intensity > 3 watt/cm2

105. The proprioceptive disturbances would be found in the following disease(s) A


1. Stroke
2. Tabes dorsalis
3. Brown-sequard syndrome
4. Funiculus anterior lesions

106. Preventive measures to minimize falls for stroke patient is/are........... A


1. Timed voiding
2. Balance training
3. Cognitive remediation
4. Maximizes use of sdatives

107. Autonomic dysreflexia caused by noxious stimuli below the injury level such as.............
C
1. Cervical muscle spasm
2. Labor and delivery
3. Severe headache
4. Ingrown to nail

108. What is meralgia parasthetica (ME) E


1. ME is mononeuropathy of the lateral femoral cutaneous nerve
2. The symptom of ME is anterolateral thigh numness and pain
3. ME is usually idiopathic and associated with obesity, pregnancy, and diabetes
4. The symptom can be exacerbated by walking or extension of the hip and by
wearing tight clothing or undergarments

109. The following is/ are the factor(s) affecting morbidity and mortality in
meningomyelocele E
1. Renal failure
2. Hydromelia
588
3. Tethered cord
4. Hydrocephalus
110. In toeing gait, it is consider normal development because D
1. It is originate from internal tibia torsion in children 3 year old
2. It is originate from femoral torsion in toddler 2-3 years old
3. It is have significant of birth history
4. Generally reduce with age

111. A disorder(s) that would result in a leg length discrepancy in children by shortening the
extremity would be..... A
1. Poliomyelitis
2. Leg-Perthes disease
3. A growth plate injury
4. A vascular malformation

112. Methods for prescribing exercise intensity using heart rate (HR) is/are the following C
1. Target HR : HRR x % intensity desired
2. Target HR : HRmax x % intensity desired
3. Target HR : ((HRR-HRrest) x % intensity desired) + HR rest
4. Target HR : ((Hrmax-Hrrest) x % intensity desired) + HR rest

113. What is/are the major goal(s) of phase II cardiac rehabilitation A


1. To achieve an early return to work
2. To achive cardiovascular conditioning
3. To achive control of modifiable risk factors
4. To increasing of heart rate for a given exercise or workload

114. The following is/are the contraindication(s) for entry into patient or outpatient exercise
program C
1. Resting systolic blood pressure 140mmHg
2. Orthopedic problems that prohibit exercise
3. Resting diastolic : BP 90 mmHg
4. Uncontrolled sinus tachycardia

589
115. The general principles of exercise for osteoporosis include the ……….. E
1. Principle of specificity
2. Principle of progression
3. Principle of reversibility
4. Principle of initial values

116. What is/are the indication(s) for measurement of bone density A


1. Woman discontinuing estrogen therapy
2. Adults taking cyclosporine
3. Men aged 70 and older
4. Women aged 40

117. To prevent risk of fracture of patient with osteoporosis, what home modifications
should be considered? E
1. Stair rails
2. Improved lighting
3. Easy access to emergency phone
4. Nonskid tape of different colours on the outer edges of steps

118. While evaluating an elderly woman with severe arthritic pain, what is/are seconday
consequence of her pain that should also treated? C
1. Dementia
2. Clinical depression
3. Acute delirium
4. Cognitive disturbance

119. A recreational tennis player had right elbow pain since 3 weeks ago. The pain increased
gradually with the time, especially after she played tennis, an extended the wirst against
resistance on dorsal aspect of the hand. What is/are the proper management(s) for this particular
patient? E
1. Avoid wrist extension
2. Check and adjust the frip of racquet
3. Wrist extensor eccentric strengthening exercise

590
4. Ultrasound diathery on lateral epicondylus area

120. In sport activities, tendons are most vulnerable to injury when B


1. Tension is applied quickly and sustained without adequate warm up
2. Tension is applied quicky and stopped rapidly
3. Tension is applied obliquely direction
4. Tension is applied as a tensile foreces

121. A 20 yo male athlete suffered right knee pain and swollen after pivoting activity during
jumping drill exercise. After 2 weeks pain and sweeling reduced, but there was athropy on his
right thigh. He still giving way and locking while do pivoting and kicking activities on the right
knee and also still felt pain on right lateral right knee joint line during landing from jumping
activity. What the possible anatomical structure/s could be injured? A
1. Ligament(s)
2. Meniscus
3. Cartilage
4. Tendon

122. The following factor(s) contribute to the development of injuries in runners E


1. Poor footwear
2. Training errors
3. Poor flexibility
4. Poor training surface

123. Basic funtion of the circulatory system include............... E


1. Delivery of nutrient
2. Transportation of oxygen
3. Conveyance of hormones
4. Removal of waste products

124. The following statement is/are TRUE about electrodiagnosis examination A


1. Able to localize a peripheral nerve lession
2. Able to establish diagnosis of peripheral nervous system disease
3. Able to make early detection of possible peripheral nerve lession
591
4. Able to establish clinical diagnosis of central nervous system disease

125. The role of golgi tendon organs in voluntary motor activity is/are................. B
1. Detect relative muscle tension
2. In parallel with muscle fibers
3. Has dynamic and static responses
4. Increase in muscle length during passive movement

126. The following is/are the possible cause(s) of genu recurvatum at the stance phase of gait
A
1. Quadriceps weakness
2. Quadriceps spasticity
3. Plantar flexion spasticity
4. Dorsiflexor contracture

127. About ankle mortis, the following statement(s) is/are TRUE A


1. In motion is 20 degree dorsiflexion
2. The talus function as hinge joint
3. Consist of tibia, fibula , and talus
4. Consist of talus and calcaneus

128. The two major “crutch walking” muscles of the shoulder is/are... C
1. Teres minor
2. Latissimus dorsi
3. Pectoralis Major
4. Lower pectoralis major

129. The following statement(s) is/are TRUE about phsycial modality A


1. The mechanism of heat transfer of paraffin bath is conduction
2. The mechanism of heat transfer of whirpool is convection
3. The mechanism of heat transfer of SWD is conversion
4. The mechanism of heat transfer of USD is radiation

592
130. The absolute contraindication(s) for intraarticular and extraarticular corticosteroid
injection is/are...... E
1. Osteochondroal fracture
2. Adjacent osteomyelitis
3. Clotting disorder
4. Acute injury

131. Laser therapy has been proven effective in the treatment of.................. D
1. Osteomalacia
2. Joint contracture
3. Rheumatoid arthritis
4. Myofascial syndrome

132. Which condition(s) has/have not the possibility for successful ambulation using
HKAFO brace and billateral axillary crutches? A
1. T6 Paraplegia AIS/Frankel A
2. T8 Paraplegia AIS/Frankel A
3. T10 Paraplegia AIS/Frankel A
4. T12 Paraplegia AIS/Frankel A

133. In below knee-amputees, excess knee flexion on the prosthetic side during heel strike
may cause by...... B
1. Heel cushion too stiff
2. Foot set with insufficient plantar flexion
3. Socket set too far anterior over foot
4. Insufficient flexion provided within the socket

134. Above knee suction socket generally NOT prescribed for an older patient for the
following reason(s) C
1. It prevents desired rotation of the hip
2. It is more difficult and energy consuming to put on
3. It is heavier and above-knee prosthesis with hip joint and pelvic band
4. It requires more training of the stump muscluature to maintain the suction

593
135. Goal(s) of Bobath’s Neurodevelopmental treatment to Cerebral Palsy is/are to... B
1. Normalize tone
2. Improve motor coordination
3. Inhibit primitive refleks
4. Activate postural responses (stability)
136. What is/are the potential effects of corticosteroid injection? A
1. Glucocorticoid-induced osteoporosis
2. Tendon and ligament rupture
3. Steroid-induce arthropathy
4. Subcutaneous hyperthophy
137. The following is/are the benefit(s) of regular physical activity C
1. Decreased exercise threshold for the onset of disease signs or symptoms
2. Increased capillary density in skeletal muscle
3. Decreased exercise threshold fot the accumulation of lactate
4. Increased maximal oxygen uptake
138. A 45 years old woman presents chronic pain in the region of the Achilles tendon. The
Achilles tendon appears swollen. Appropriate initial treatment(s) would include.......... A
1. Boutonierre
2. Piano-key sign
3. Ulnar deviation
4. DIP joint subluxation
139. A 45 years old woman presents chronic pain in the region of the Achilles tendon. The
Achilles tendon appears swollen. Appropriate initial treatment(s) would include.......... D
1. Cortison injection of the gastrocnemius muscle
2. Cortison injection of the Achilles tendon sheath
3. Immobilization in ankle plantar-flexed position with limited weight bearing
4. Use of small heel lift with initiation of gastroc-soleus stretches as tolerated

140. A Patient complain of knee pain after falling of her flexed knee. Physical examination
shows a positive “sag sign”. Which ligament(s) is/are injured? D
1. Medial collateral
2. Lateral collateral
3. Anterior cruciate
4. Posterior cruciate
594
141. Rheumatoid foot problem(s) needing intervention such as.............. E
1. Hallux valgus
2. Cocked-up toes
3. Reduced subtalar motion
4. Subluxed metatarsal phalangeal heads

142. Good prognosis of motor recovery after stroke in upper limb is/are.......... D
1. Severe proximal flaccidity
2. Complete arm paralysis at onset
3. Partial return of grasp strength by 8 weeks
4. If the patient shows minimal motor recovery of the hand by 4 weeks

143. Lumbar epidural steroid injection show(s) improvement in the following E


1. Mobility
2. Leg pain
3. Return to work
4. Short-term pain

144. Treatment of acquired dysphagia in a 67-year old man with a left hemiparesis might
include........ D
1. Upright feeding and head turning to the nonaffected side
2. Changing diet to one of softer and thinner consistency
3. Use of long-term thermal stimulation, up roght feeding
4. Side chin tucking and head turning to the affected

145. A birth defect that arises during neurulation (neural tube closure) rather than after
neurulation is / are…. B
1. Spina bifida cystica
2. Sacral lipoma
3. Meningomyelocele
4. Teratoma

595
146. Persistance of which of the following primitive reflex(es) or postural response(s) would
you be heighten suspicion of normal development A
1. Persistance Landau reflex beyond 2 years of age
2. Persistance Parachute reflex beyond 2 years of age
3. Persistance placing reflex beyond 7-8 months of age
4. Persistance asymmetric tonic neck reflex beyond 6 months of age

147. Early intervention will prevent the need for a surgical procedure in congenital mu
torticolis. If there is no eraly intervention, the potential complication(s) include............ E
1. Craniofascial asymmetry
2. Intermittent head tilt
3. Visual disturbance
4. Scoiliosis

148. Pulmonary rehabilitation programs should include which of the following component(s)
E
1. Patient education
2. Physical reconditioning
3. Breathing techniques and exercises
4. Smoking cessation and related counseling

149. AACVPR paramater for in patient cardiac rehabilitation daily ambulation E


1. Normal cardiac rhytm and ECG for previous 48 hours
2. Stable or falling creatinine kinase and troponin values
3. No new or recurrent chest pain in previous 48 hours
4. No indication of decompensated heart failure

150. In COPD patient, adverse response to exercise leading to exercise is/are........... A


1. Oxyhemoglobin saturation < 90%
2. Severe dyspnea during exercise
3. Heart rate > 120 bpm
4. PVC 1/minute

596
NATIONAL BOARD EXAMINATION
PHYSICAL MEDICINE AND REHABILITATION
JUNE 14, 2019

1. The most common cause of significant knee hemaarthrosis after trauma injury is...
a. Antrerior cruciate ligament rupture
b. Lateral collateral ligament tear
c. Osteochondral fracture
d. Patelar dislocation
e. Meniscal tear

2. The 25 years old man has had heel pain for 1month. He runs approximately 40 km
weekly. The pain is most severe when he first stands up and walks after sitting or lying down.
Palpation of the medial plantar surface of the heel increase the pain. He walk with a limp.
Which of the following is the most likely cause of the pain?
a. Heel spur
b. Achilles tendinosis
c. Impingement of the posterior sacral nerve root
d. Inflamation of the origin of the plantar fascia
e. Articular cartilage degeneration of the calcaneo cuboid joint

3. 24 year old marathon runner report lateral knee pain after hill training. Examination
reveals, no effusion, and results of ober test are positive.what is the most likely diagnosis?
a. Biceps tendinitis
b. Lateral meniscal tear
c. Popliteus tenosynovitis
d. Peroneal nerve entrapment
e. Iliotibial band friction syndrome

4. The risk of foot ulceration in diabetic patient is increasing by all of the following,
except...
a. Metatarsal subluxation and fat pad migration
b. Increased mobility of the subtalar joint
c. A history of previous foot ulceration

597
d. The presence of plantar foot callus
e. The lost of vibration sensation

5. Provocative test of acromioclavicular joint impingiment is..


a. Cross-chest test
b. Relocation test
c. Apprehesion test
d. Anterior drawer test
e. Anterior load and shift test

6. An infarct in the lower division of the middle cerebral artery division would be
associated with which type of aphasia...
a. Broca
b. Global
c. wernicke
d. transcortical motoric
e. transcortical sensoric

7. Which of the following of the physical exam findings would be most consistent with C6
radiculopathy?
a. Decreased brachioradialis reflex
b. Medial brachial sensory loss
c. Decreased biceps refleks
d. Deltoid weakness
e. Triceps weakness

8. Which type of stroke typically has the best outcome?


a. Hemoragic pontine
b. Internal capsule lacunar
c. Embolic left cortical infarction
d. Rupture of anterior cerebral artery aneurism
e. Ischemic occlusion of the right internal carotid artery

598
9. Treatment of acquired dysphagia in a 67 year old man with a left hemiparesis might
include...
a. Chin tucking and reclining feeding
b. Chin tucking and head turning to the affected side
c. Changing diet to one of softer and thinner consistency
d. Upright feeding and head turning to the nonaffected side
e. Use of the longterm thermal stimulation and upright feeding

10. Which of the following risk factors is the most associated with a higher incidence of
DVT in adults with a stroke?
a. Male sex
b. Female sex
c. Increased age
d. Smoking history
e. Increased motor weakness

11. A child 3 years old has clinical sign: asymetric face, shortening of
sternocleidomastoideus muscle without tumor, deficit of head rotation >30. The appropriate
management is...
a. Deep heating
b. Stretching exercise
c. Asymmetric collar
d. Refer to orthopedic division
e. Stimulation of the head movement

12. Children of tetralogy of fallot with assume the squating positionto relieve exercise
induced dyspneu. This will....
a. Decrease heart rate
b. Increase respiratory capacity
c. Decrease pulmonary artery pressure
d. Increase peripheral vascular resistance and there by decreases right to the left
shunt
e. Reduce the energy requirement of the activity by lowering the center gravity

599
13. A child with a congenital transverse radial limb deficiency should have in initial
prosthesis. What developmental stage?
a. At the time of first sitting independently
b. At the time of starting kinder garden
c. At soon as posible after birth
d. At the time of initially walking
e. At the time of initially standing

14. CTEV is a deformity of the foot with characteristic of...


a. Disorder of lateral part of the foot
b. Disorder of medial part of the foot
c. Disorder of anterior part of the foot
d. Disorder of posterior part of the foot
e. Disorder of posterior, medial, anterior part of the foot

15. The term fitness must include all of the component below..
a. Cardiorespiratory endurance, muscle strength, BMI and FEV1
b. Cardiorespiratory endurance, muscle strength and endurance
c. Cardiorespiratory endurance, flexibility, BMI, muscle strength and endurance
d. Cardiorespiratory endurance, muscle strength , lean body mass, flexibility
e. Cardiorespiratory endurance

16. For pulmonary patients, clinical exercise testing have all benefit below, except..
a. To determine function of the lung
b. To monitor the result of the rehabilitation program
c. To prescribe the intensity of the exercise programe
d. To rule out the presence of the significant cardiac disease
e. To rule out the presence exercise – induced bronchospasm

17. A patient with restrictive lung disease and mild scoliosis from DMD develops
nocturnalhypoventilation. What is the initial treatment?
a. Intubation
b. Incentive spirometry every four hours

600
c. Nocturnal supplemental low – flow oxygen
d. Tracheostomy and nocturnal mechanical ventilation with possitive presure
e. Nocturnal noninvasive positive airway presure and short period daytime
18. Which cardiopulmonary parameter should improve in patient with CHF who is
participating In a cardiac rehabilitation program.?
a. VO2 max
b. Pulmonary blood flow
c. Systolic blood presure
d. Systolic ejection fraction
e. Diastolic ventricular filling

19. The following are contraindicated physical activities in the osteoporotic patients,
except...
a. Golf swing
b. Running
c. Jumping
d. Walking
e. jogging

20. The following are the physiologic changes in aging process, except...
a. Lower flexibility
b. Higher vital capacity
c. Lower reaction time
d. Higher residual volume
e. Higher resting blood pressure

21. Which the physiologic factor in the eldery exacerbates orthostatis?


a. Decreased in arterial stiffness
b. Decreased creatinine clearence
c. Decreased peripheral resistance
d. Decreased baroreceptor response
e. Decreased lower limb muscle strength

601
22. In prescribing pain treatment for eldery patients, it is best to use which approach?
a. Start with NSAID
b. Start low and go slow for all medications
c. Start with aspirin, which is safe and effective
d. Avoid use of acetaminophen, due to liver toxicity
e. Use high-dose, shortacting narcotics initially to get the pain under control

23. The following are the risk factors for developing NSAID-related gastropathy, except....
a. Previous history of uncomplicated ulcer
b. Concurrent use of the corticosteroid
c. High-dose NSAID therapy
d. Concurrent use of aspirin
e. Age under 40 years

24. What is the common diagnosis in young female gymnasts with chronic back pain?
a. Spondylitis
b. Spondylosis
c. Spondylolisis
d. Spondyloarthrosis
e. spondylolisthesis

25. One week after the patien began a resistance/strengthening program, you note a 15%
increase in the force of their maximum voluntary contraction. This is most likely secondary to a
change in the...
a. Number of muscle fiber present
b. Motor unit recruitment pattern
c. Neuromuscular junction
d. Number of myofibrils
e. Fiber density

26. A 17 year old football player sustained a noncontact knee injury while planting his leg
to make a cut. He heard a pop and felt his knee buckle. What is the most sensitive clinical test
to establish the diagnosis?
a. Anterior drawer
602
b. Posterior drawer
c. Varus test
d. Mc murray
e. Lachman
27. Which equipment modification is suitable for a tennis player with lateral epicondylitis ?
a. Stiffer racquet
b. Larger racquet head
c. Larger racquet diameter
d. Increased string tension
e. Decreased string tension

28. What is the most common mechanism for achilles tendon rupture ?
a. Passive dorsiflexion
b. Eccentric dorsiflexion
c. Passive plantar flexion
d. Concentric dorsiflexion
e. Eccentric plantar flexion

29. Which electrodiagnosis finding is associated with a poor prognosis in GBS ?


a. Compound MUAP less than 10% normal
b. Significant delayed conduction velocity
c. Absence of late response F or H wave
d. Large fibrilation potentials
e. Nascent potential

30. During an electrophysiological examination, somatosensory evoked potentials evaluate


the integrity of the...
a. Fasciculus gracilis and fasciculus cuneatus
b. Tractus spinothalamicus ventralis
c. Tractus corticospinalis ventralis
d. Tractus corticospinalis lateralis
e. Tractus pyramidalis

603
31. The primary neurotransmitter of afferent pain neurons in the spinal cord is appear to
be...
a. GABA
b. Endorphin
c. Enkephalin
d. Substance P
e. Beta-lipoprotein

32. The following joint is classified trochoginglimus joint


a. Hip Joint
b. Wrist Joint
c. Knee Joint
d. Ankle Joint
e. Shoulder Joint

33. When you see a patient everytime he is walking always holding his thigh during the
stance phase, most probably he has lesions of the following nerve...
a. Tibial
b. Sciatic
c. Femoral
d. Common peroneal
e. Tibial and peroneal

34. The following are barriers to motor recovery after stroke, except...
a. Lack of voluntary movement within 2 weeks
b. Lack of movement out of synergy patterns
c. Flaccidity within 1 week
d. Severe spasticity
e. Sensory deficits

35. Biofeedback has been effective for treating stroke patients with...
a. Spasticity
b. Visual neglect

604
c. Impaired balance
d. Impaired hand function
e. Oropharyngeal incoordination

36. Heat Therapy through convection method could be generated by the following
modality..
a. USD
b. Infra red
c. Cold laser
d. Whirpool
e. Ultraviolet treatment\

37. Which finding is a relative contraindication to cryotherapy ?


a. Pain
b. Edema
c. Acute hematoma
d. Impaired sensation
e. Acute inflammation

38. A patient with DM has painful, swollen ankle, worst with weight bearing, X-ray shows
fragmentation of the articular surface, synovial fluid is clear. The choice or orthosis is...
a. AFO
b. KAFO
c. HKAFO
d. PTB orthosis
e. Orthopedic shoes

39. In PTB socket the patient’s stump in held in how many degree knee flexion?
a. 15 degree
b. 20 degree
c. 25 degree
d. 30 degree
e. 35 degree

605
40. The immediate post-surgical fitting with pylon leg is especially directed to....
a. Shape a good stump
b. Accelerated the stump healing
c. Avoid the development of phantom pain
d. Accelerate the cessation of phantom sensation
e. Promote early return of proprioceptive function

41. Specific therapeutic exercise for scoliosis are all the following, except...
a. Done in and out of the brace
b. Stretching exercise of the convex side
c. EDLF exercise is done in the kneeling position
d. Muscle strengthening exercise of the convex side
e. Abdominal muscle strengthening exercise is included

42. The following are guideliness for exercise during pregnancy, except...
a. Avoid supine exercise after the first trismester
b. Stretching exercise is preferable to intermittent activity
c. Stop exercising when fatique and avoid exhaustive exercise
d. Pregnancy should maintain adequate intake for basal, exercise and pregnancy needs
e. Many physiologic and morphologic changes of pregnancy persist 12 weeks
postpartum

43. The following is the guidelines for exercise for persons with HIV infections.
a. Aerobic exercises that produce a negative energy balance is contraindicated
b. Aerobic exercises for person with established clinical weight loss is indicated
c. Aerobic exercises for person with a lack cardiopulmonary reserve is indicated
d. Person with asymptomatic HIV seropositive are restricted activity and competition
e. Person symptoms prior to development of AIDS are encouraged to exercise and
competition

44. The patients lies supine with examiner holding the knee in 15 degrees of flexion. The
femur is stabilized with one hand while pressure is applied to the posterior aspect of the

606
proximal tibia is an attempt to translate it anteriorly. A positive test occurs when excessive
anterior translation of tibia occurs with a soft endpoint. This test named is...
a. Lachman test
b. Mc murray test
c. Posterior drawer test
d. Patellofemoral grind test
e. Valgus and varus test

45. The following is NOT the proper program for acute back pain in a patient with
osteoporotic vertebra fracture.
a. Avoid strengthening exercises for 2 months
b. Provide a brace to support the spine
c. Avoid ADL training
d. Treat constipation
e. Relieve the pain

46. In patient with chronic lateral epicondylitis, the important consideration in therapy woul
be on....
a. Infrared
b. Immediate icing
c. Injection of corticosteroids
d. Use appropriate wrist strap
e. Strengthening and stretching of the wrist extensors

47. Using computer mouse that is too far to side or to high on the table will lead to...
a. Anterior shoulder subluxation
b. Thoracic outlet syndrome
c. Rotator cuff tendinitis
d. Bicipitalis tendinitis
e. Adhesive capsulitis

48. What is the most reliable physical signs of MTPS ?


a. A rope-like nodular in the muscle and pattern of reffered pain
b. Focal tenderness and pain recofnation
607
c. Vasomotor and temperature changes
d. Limited of ROM
e. Muscle spasme

49. The doloreaux is caused by pathology of the following cranial nerve...


a. II
b. III
c. IV
d. V
e. VI

50. Patient with pronator teres syndrome will have the following clinical signs, EXCEPT...
a. Intrinsic muscle atrophy
b. Pain in the lateral dorsal forearm
c. Normal strength of the pronator teres muscle
d. Weakness of the median innervated hand intrinsic
e. Decrease sensation in the first three digits and over the thenar area

51. A patient with LBP of six weeks duration has mild left calf atrophy diminished plantar
flexion strength and absent achilles reflex. Electrodiagnostic testing shows an absent H reflex;
normal insertional activity in all muscle tested; moderately large amplitude, mildly polyphasic
motor units in the left medial gastronemius, the lateral hamstring and the gluteus maximus; and
normal motor units elsewhere, what is the diagnosis?
a. Ischias syndrome
b. Acute SI radiculopathy
c. Chronic SI adiculopathy
d. Lumbosacral polyradiculopathy
e. Chronic tibial neuropathy in the thigh

52. A 25 year-old restrained driver involved in a head on motor vehicle crash. Two weeks
later the individual complains of pain and paresthesias in the thigh. No loss of muscle strength
is noted on examination. But there is a decrease in sensation in the symptomatic thigh. Which
neuropathy is the most likely etiology for this patient’s complain?
a. Lateral femoral cutaneous
b. Ilioinguinal

608
c. Saphenous
d. Obturator
e. sciatic

53. The most common abnormal urodynamic finding in stroke survivors is...
a. The absence of urethral sphincter activity
b. Detrusor-sphincter dyssynergia
c. Low bladder compliance
d. Detrusor hyperreflexia
e. Detrusor hyporeflexia

54. The determinant(s) of walking prognosis in CP patients is...


a. Rolling supine to prone by age 15 months
b. Persistance of primitive reflexes
c. Presence of protective reflexes
d. Presence of spinal reflexes
e. Age of sitting ability

55. The presence of 4 years old boy with DMD ask your opinion about future of their son.
Which of the following statement is NOT correct?
a. DMD typically becomes clinicaly evidence at approximately 3 to 5 years of ago
b. Weakness will begining in the shoulder girdle muscles and later in the pelvic
c. Early difficulties noted are clumsiness, poor walking and frequent falls
d. Death is usually due to respiratory insufficiency
e. Intellect may be affected

56. Leg length discrepancy in children can result from several disorders. A disorder that
would result in a discrepancy by lengthening the extremity would be...
a. A vascular malformation
b. A growth plate injury
c. An overridding fracture
d. Leg – perthes disease
e. poliomyelitis

609
57. Of the common orthopedic procedures performed for contractures and spaticity in
spastic diplegia, the procedure with the highest incidence pf recurrence of deformity post
operatively is...
a. Adductor longus and gracilis myotomy with anterior branch obturator neurectomy
b. Achilles tendon lengthening
c. Hamstring lengthening
d. Iliopsoas recession
e. Gracilis sling

58. Spinal muscular athropy type II is chronic in nature, despite their muscle weakness, this
child has the following clinical sign..
a. The weakness only affecting lower extremities
b. Not able to achieve independent sitting
c. Hip dislocation is not uncommon
d. Can not walk with bracing
e. Scoliosis is uncommon

59. Which cardiorespiratory outcome would you expect from an eldery person participating
in an aerobic program?
a. No change in stroke volume and a reduction in systemic vascular resistance
b. A decrease in VO2max but an increase in walking speed
c. No change in tolerance during activities of daily living
d. Improvement in blood pressure
e. Improvement in VO2 max

60. Among supported-ventilation techniques, the most practical way to allow adequate
oxygenation is..
a. An intermittent abdominal presure ventilator
b. Intermittent positive – presure ventilation
c. A wrap-style ventilator
d. A rocking bed
e. A chest shell

610
61. A 50 year-old man with mild systemic hypertension has a marked increase in blood
pressure. When he is performing his weight-lifting exercise. During active muscle contraction
you recommended...
a. Shallow rapid breathing
b. Nasal breathing
c. Closed glottis
d. Exhalation
e. Inhalation
62. Exercise stress testing may be expected to disclose all the following EXCEPT..
a. Myocardial ischaemia
b. Coronary artery plaques
c. Ventricular ectopic activity
d. Left ventricular dysfunction
e. Chronotropic incompetence

63. The american geriatrics society’s goal of diabetic control in eldery includes...
a. Tight hypoglycemic control
b. Tight HBA1C below 7%
c. Fasting plasma glucose below 120mg/dL
d. Reduction of cardiovascular complications
e. Reduction of neuromuscular complications

64. Which clinical assesment tool is NOT used to evaluate functional mobility skills the
geriatric population?
a. Katz index
b. Barthel index
c. FIM instrument
d. Six minute walk test
e. TUG test

65. Which level is the most common site of SCI in eldery patient?
a. Cervical
b. Upper thoracic
c. Lower thoracic
611
d. upper lumbosacral
e. lower lumbosacral

66. the use of opiod analgesics for chronic pain...


a. is contraindicated
b. demonstrated a threshold but not a celling effect
c. may allowed only by intramuscular administration
d. is most effective when given on as-needed closing regimen
e. may be no more effective than nonsteroid anti-inflammatory drugs

67. a 43 years old running presents with right-sides hip pain. On examination, you note
tenderness to palpation over the greater trochanter on the affected side. No focal neurologic
deficits are noted and no other areas of tenderness are elicited. A key part of your treatment
plan should include strengthening of the hip...
a. internal rotators
b. external rotators
c. extensors
d. abductors
e. adductors

68. what is the most common cause of the knee pain runners?
a. Baker cyst
b. Osteoarthritis
c. Diskoid meniscus
d. hamstring tendinitis
e. patellofemoral pain syndrome

69. what is the advantages of using closed kinetic chain exercices during the early stages an
ACL rehabilitation program?
a. Improve terminal stretch
b. Minimal patellar compression
c. Decreased shear forces across the joint
d. Limited knee extensors synergist activity
e. Improved targeting of the vastus medialis
612
70. Ankle sprains are ubiquitous in sports and are the most common athletic injury. The
most commonly injury for this disorder is the...
a. Achilles tendon
b. Calcaneo fibular ligament
c. Anterior talofibular ligament
d. The posterior talofibular ligament (kunci)
e. The strong medial deltoid ligament
 C : SALAH

71. Nerve conduction velocity is slow in...


a. Myopathy
b. Axonal sensory neuropathy
c. UMN lession
d. Primary axonal degeneration
e. Demyelinated peripheral nerve

72. The structure below is NOT a member of the motor unit...


a. Skeletal muscle
b. Motor end plate
c. Neuromuscular junction
d. Neuron of the dorsal horn of the spinal cord
e. Neuron of the anterior horn of the spinal cord (kunci)
 D : SALAH

73. The myoneural junction main neurotransmitter is...


a. N-Methyl-D-Aspartate acid
b. Acethyl choline
c. Substance P
d. Serotonine
e. Dopamine

613
74. Expect at very slow speeds of walking the knee flexes soon after heel strike to
approximately...
a. 5 degrees
b. 15 degrees
c. 25 degrees
d. 35 degrees
e. 45 degrees

75. In a person with a transfemoral amputation, an abnormal prosthetic gait with lateral
trunk bending toward the involved side in modstance most commonly occurs with hip...
a. Extension contractures
b. Abductors contractures
c. Flexion contractures
d. Abductor weakness
e. Adductor weakness

76. In nondisabled subjects walking at self-selected normal speeds, the normal distribution
of time spent in double support is approximately . . . . . .
a. 10%
b. 20%
c. 30%
d. 40%
e. 50%

77. Which modality should precede shoulder stretching in a patient with adhesive
capsulitis?
a. Massage
b. Ice packs
c. Ultrasound
d. Laser Therapy
e. Electrical stimulation

78. What is the most important factor to consider when injecting myofascial trigger points?
a. Type of medication injected
614
b. Localization of trigger points
c. Frequency of injection
d. muscle spasm severity
e. size of needle

79. Methods of treating acute inflamatory tendinitis include the foloowing, EXCEPT . . .
a. use of ultrasonic heating
b. application of cold packs
c. splinting of the involved tendon
d. local steroid and anesthetic injections
e. administration of oral anti-inflamatory agents

80. Part of shoe which hold firmly the calcaneous is the


a. counter
b. quarter
c. shank
d. vamp
e. heel

81. A lumbrical bar on a wrist-hand orthosis . . . . .


a. is primarily used in radial nerve injuries
b. is used to replace the absent of MCP flexion
c. is positioned on the volar aspect of the distal phalanges
d. is positioned on the volar aspect of the proximal phalanges
e. is used to prevent MCP hyperextension with the finger extension
(juni 2013 nomor 44 jawaban E)

82. In a 35-year old patient with Charcot-Marie-Tooth (CMT) disease, orthotic management
will most likely consist of . . . .
a. custom-molded, longitudinal arch supports
b. a hip-knee-ankle-foot orthosis
c. a knee-ankle-foot orthosis
d. an ankle-foot orthosis
615
e. a knee cage brace

83. The neurodevelopmental training technique in stroke management . . . .


a. uses a cuff shoulder sling
b. fascilitates diagonal pattern
c. facilitates tone on the spastic hemiplegic side
d. attempts to inhibit tone in the spastic hemiplegic extremity
e. requires restricting voluntary movement of the unaffected extremity
84. The functional Independence Measure (FIM) assesses . . .
a. prognosis
b. impairmant
c. activity limitations
d. environment factors
e. participation restricitions

85. An example of closed kinetic chain exercise for the quadriceps is . . . .


a. partial squat
b. Walking on a treadmill
c. An isometric quadriceps set
d. knee extension on a stacked-weight machine
e. Knee extension on an isokinetic machine (eg, Cybex)

86. A patient experiences pain and decrease motion in the rught shoulder following a
radical neck dissection for laryngeal carcinoma. On examination, you note atrophy and
weakness of the ipsilateral trapezius. Passive range of motion (ROM) is normal,; however,
active ROM shows decreased shoulder abduction and decreassed scapular rotation. Your
therapy orders should emphasize strenghtening exercise for the . . . . .
a. deltoid
b. rhomboids
c. infraspinatus
d. pectoralis major
e. latisimus dorsi

616
87. A 45-year-old woman presents with a three-month history of nonradicular low back
pain but no history of trauma. X-rays show degenerative changes and grade 1 spondylolisthesis
at L4-5. What is the best recommendation?
a. Deep heating
b. Bed rest for two weeks
c. A polyproylene body jacket
d. Abdominal muscle strengthening
e. Isokinetic back extension strengthening

88. A 28 year old man develops insidious onset of chronic low back pain with morning
stiffness. A diagnosis of ankylosing spondylitis requires . . . .
a. radiographic demonstartion of sacroilliac joint abnormality
b. Ophthalmologi confirmation od acute anterior uveitis
c. an elevated erythrocyte sedimentation rate
d. a positive HLA-B27 test
e. CT-scan

89. de Quervain disease is a tenosynovitis of the . . . . .


a. extensor pollicis longus and flexor pollicis longus
b. abductor pollicis longus and extensor pollicis brevis
c. adductor pollicis brevis and extensor pollicis brevis
d. flexor pollicis longus and adductor pollicis longus
e. extensor pollicis longus and extensor pollicis brevis

90. When a patient with recent onset of hemiplegia shows progressive motor recovery from
brunnstrom stage 1 to brunnstrom stage 3, what do you expect?
a. decreasing tone and increasing synergic movements
b. increasing tone and increasing synergic movements
c. decreasing tone and decreasing synergic movements
d. decreasing tone and increasing isolated voluntary movements
e. Unchanged tone and increasing isolated voluntary movements

91. In a patient with an acute spinal injury (SCI), intermittent cathetetization should be
instituted . . . . .

617
a. within 24 hours of injury
b. within 48 hours of injury
c. when spinal shock is resolved
d. when reguler, controlled fluid intake is occuring
e. when the patient can perform self-catheterization

92. Which nerve is most commonly injured with traumatic anterior shoulder dislocation?
a. Ulnar
b. Radial
c. Axillary
d. Long thoracic
e. Thoracodorsal

93. What is the best initial knee imaging study on a patient with suspected Osgood-Sclatter
disease?
a. Magnetic resonance imaging
b. computed tomography scan
c. skyline view radiograph
d. Anterior radiograph
e. Lateral radiograph

94. There are several methods of therapy in treatment of cerebral palsy. The bobath
method . .
a. consist of passive exercise designed to replicates phylogenetic progression and enlists a two-
person team to use set patterns for several hours per day
b. focuses on altering abnormal posture, reducing or increasing tone, and developing
movement patterns in keeping a noral developing sequences.
c. emphasizes stimulation techniques that fascilitate or inhibit specific motor respone, and is
aimed at teaching awareness or normal patterns
d. emphasizes extensive bracing to maintain a correct erect psotures
e. paertains to learning disabilities an sensory integration

95. Persistance of which one of the following primitive reflexes or postural responses
should you be heighten suspicion of abnormal development?
a. persistance asymmetric tonic neck reflex beyond 6 months of age
618
b. peristance the plantar grasp beyond 7-8 months of age
c. persistance placing reflex beyond 7-8 months of age
d. persistance parachute reflex beyond 2 years of age
e. persistance Landau reflex beyond 2 years of age

96. The long-term consequences of the high bladder outlet resistance in children with
myelomeningocele include . . . .
a. flaccid detrusor
b. spastic detrussor
c. ureteral dilatation
d. retrograde ejaculation
e. increased bladder compliance

97. In a child with Duchenne muscular distrophy (DMD), which muscle group has the most
significant weakness at the time of presentation?
a. proximal and distal lower extremity
b. proximal upper extremities
c. proximal lower extremities
d. distal upper extremities
e. distal lower extremities

98. A 68 year old man with severe COPD is considering the benefits of starting an exercise
training program. You can inform him that he would most likely experience an . . . . .
a. increase FEV1
b. improved vital capacity
c. decreased resting heart rate
d. increased exercise tolerance
e. increased maximum voluntary ventilation

99. The most sensitive diagnostic test for early respiratory muscle weakness in patients with
neuromuscular disease is . . . .
a. FEV1
b. total lung capcity
c. arterial blood gases
619
d. minute ventilation volume
e. maximum inspiratory and expiratory pressures

100. A 59-year-old man with COPD is underlying exercise training in your pulmonary
rehabilitation program . His resting arterial blood gases are pH 7,43 PO2 67, pCO2 40. During
ambulation training, he complains of fatigue and dyspnea. His pulse is 115 and regular, lungs
are clear, ear oximetry reveals 80% O2 saturation. For next exercise session you would . . . .
a. reduce the level of exercise
b. start psotural drainage and percission
c. add O2 via nasal cannula during walking
d. add digoxin to slow the heart rate and prevent arrhythmias
e. initiate incentive spirometry to improve diaphragmatic endurance

ANSWER :
A : IF NUMBERS 1,2,3 ARE CORRECT
B : IF NUMBERS 1,3 ARE CORRECT
C : IF NUMBERS 2,4 ARE CORRECT
D : IF ONLY NUMBER IS CORRECT
E : IF ALL NUMBERS ARE CORRECT

101. The rehabilitation programs (s) pf a patient with AMP include . . . .


1. Do not flex the hip joint more than 90 degrees
2. Weight bearing can be done after 3 months
3. Weight bearing can be done immediately
4. Do not abduct the hip joint
Jawaban : B

102. Preoperative exercise (s) for total hip replacement include . . . .


1. Progressive resistive exercise for the triceps, latissimus dorsi and other crutch
walking muscle
2. Foot-ankle exercise to diminish venous statis to prevent the occurence of
thrombophlebitis
3. Breathing exercise, including instruction on proper cough technique and postural
dranaige
4. Isometric exercise for quadriceps, gluteus medius and gluteus maximus
620
Jawaban : E

103. Compared with conventional residual limb care for a patient with a transtibial
amputation, the immediate postoperative rigid dressing technique . . . .
1. Provides less effective desensitization
2. Provides less effective pain control
3. Stimulates good stump forming
4. Promote wound healing
Jawaban : D
104. Which exercise has the greatest effect on bone formation ?
1. Cycling
2. Weight lifting
3. Swimming
4. Leg press exercise
Jawaban : C

105. The functional outcome of complete L3 paraplegia ia/are . . .


1. Independent in bowel and bladder routines
2. Independent in wheelchair propulsion
3. Independent in pressure relief
4. Standing without orthosis
Jawaban : A

106. Differential diagnosis of delirium in patients with intracerebral tumors is/are .


1. intratumoral hemorrhage
2. obstructive hydrocephalus
3. reactive pertumoral edema
4. medication : sedative–hypnotic
Jawaban : E

107. a person who had a lacunar infarct in the posterior limb of the internal capsule would
likely present with . . .
a. dysarthria-clumsy hand syndrome

621
b. pure sensory stroke
c. ataxic hemiparesis
d. pure motor stroke
Jawaban : D

108. therapeutic exercise for patients with parkinson disease includes . . .


1. training the individual not to focus on foot posotion
2. shortening step length to prevent freezing
3. teaching the individual to turn en bloc
4. using verbal and visual cueing
Jawaban : D

109. juvenile rheumatoid arthitis is associated with . . .


1. a high incidence of iridocyclitis
2. spondylitis of the thoracic spine
3. a positive antinuclear antibody test
4. lymphadenopathy and hepatosplenomegaly
Jawaban : B

110. arthrogryposis multiplex congenital has/have the following clinical sign(s)


1. the joint are relaitvely normal, but fixed due to surrounding soft tissue contracture
2. muscle is usually hypertrophic ratther than atrophic
3. usually intellectually normal
4. usually myopathic in origin
Jawaban : B

111. in the case of low back pain, when we suspect Paget’s disease, the laboratory test tat
must be order to confirm the diagnosis is/are . . .
1. CPK
2. Amilase
3. Rheumatoid factor
4. Alkaline phosphatase
Jawaban : D
622
112. In post myocardial infarction, adverse response to exercise leading to exercise
discontinuation is/are . . . .
1. 2nd and 3rd degree AV block
2. Increase in systolic blooed pressure (SBP) >30mmHg during exercise with increasing
workload
3. Diastolic blood pressure (DBP) ≥ 110 mmHg
4. No significance ventricular or atrial arrhythmias with or without associated
sign/symtom
5. Jawaban : B
113. The following is/are special consideration(s) for patient with diabetes mellitus who
undergo exercise training.
1. Common symptoms associated with hypoglycemia in individual taking insulin or
oral hypoglycemic agent
2. Retinal detachment in individual with diabetes mellitus and retinopathy
3. Hyperglycemia for individual with type I diabetes melittus
4. Dehydration resulting for polyuria
Jawaban : E

114. The foci of intervention(s) during phase I cardiac rehabilitation include . . .


1. Modifiable risk factor reduction strategy
2. Alleviation of anxitety and depression
3. Early ambulation and ADL training
4. Exercise with moderate intensity
Jawaban : A

115. Strategy(ies) and devices(s) to reduce vertebral compression forces for patients with
osteoporosis include . . . .
1. Reposotioning telephones closer to spare trunk flexion
2. Long-handled reacher
3. Rotating platform
4. Wheeled carts
Jawaban : E

116. The following statements(s) is/are TRUE regarding pharmacology in osteoporosis


623
1. Parathroid hormone is catabolic agent
2. Bisphosponates inhibit bone resorption
3. Calcitonin directly inhibits osteoblast activity
4. Raloxifene (SERM) activating certain estrogen pathway
Jawaban : C

117. The following is/are the factors(s) which can help predict who will sustain in a hip
fracture
1. Maternal hip fracture
2. Lower bone density
3. Hyperthroidism
4. Obesity
Jawaban : A

118. A good initial treatment plan(s) for neurogenic claudication secondary to spinal stenosis
would include . . . .
1. Narcotic pain medication
2. Lumbar bracing
3. Acetaminophen
4. Gabapentin
Jawaban : D

119. A 20 yo male athlete suffered right knee pain and swollen after pivoting activity during
jumping drill exercise. After 2 weeks pain and swelling reduced, but there waas atrophy on his
right thigh. He still felt giving wat and licking while do pivoting and kicking activities on the
right knee. If Lachman, anterior drawer and pivot shift test were positive in this case, what
specific ligamen(s) will be involved?
1. Posterior cruciate
2. Lateral collateral
3. Medial collateral
4. Anterior criciate
Jawaban : D

120. A 18 yo basketball player suffered from chronic ankle instability. What is/are the
RIGHT statement(s) for this condition?

624
1. Mechanical ankle instability can be caused by anterior talofibular ligament laxity
2. Chronic ankle instability consist of mechanical and fuctional ankle instability
3. Functional ankle instability reflects the proprioceptive incompetence
4. This condition needs simple rehabilitation program
Jawaban : A

121. A 20 year-old football player has repeated episodes of heat cramps during summer
training sessions. A deficiency of what electrolyte is/are responsible for heat cramps?
1. Magnesium
2. Potasium
3. Chloride
4. Sodium
Jawaban : D

122. Problems (s) frequently associated with the pronated foot in runners is/are . . . .
1. Plantar fascitis
2. Achilles tendinitis
3. Posterior tibial tendinitis
4. Patellofemoral pain syndrome
Jawaban : E

123. Electrodiagnostic techniques which most useful for identifying Mystenia Gravis
is/are . . . .
1. Needle EMG
2. Single fiber EMG
3. F-waves and H-reflexes
4. Repetitive nerve stimulation
Jawaban : C

124. The following statement(s) is/are correct regarding the motor unit . . .
1. Its electrical activity is called MUAP
2. The targeted system of EMG examintation
3. The number of its muscle fibers depends on the function of the particular muscle

625
4. Consisting of one motor cell in the anterior horn of the medulla spinalis and all of
its muscle fibers innerveted
Jawaban : E

125. Muscle endurance can be increased through gains in . . . .


1. Muscle strength
2. Circulatory function
3. Metabolic efficiency
4. Muscular hypertrophy
Jawaban : A
126. The following is/are true about hip joint
1. Is a ball-and-socket joint
2. Hip flexion at least 120 degrees
3. The use of a brace on the leg can alter the magnitude of the hip joint reaction force
4. The magnitude of the hip joint reaction force is influenced by the ratio of the
abductor muscle force and gravitational force lever arm
Jawaban : E

127. The following muscle is/are the primary movers at scapulothoracic interface.
1. Subscapularis
2. Latissimus dorsi
3. Infraspinatus
4. Seratus anterior
Jawaban : C

128. Plantar flexor muscle are inactive by the time of . . . .


1. Mid stance
2. Heel strike
3. Foot flat
4. Toe off
Jawaban : D

129. The following is the appropiate use of contrast baths :

626
1. The firt resevoir usually warm (43°C) and the other cool (16°C)
2. The firt reservoir usually warm (38°C) and the other cool (10°C)
3. Soaking duration often about 4 minutes in the warm bath and 1-2 minutes in the
cool bath
4. Soaking duration often about 10 minutes in the warm bath and 4-5 minutes in the cool
bath
Jawaban : B

130. The clinical application of functional electrical stimulation (FES) in rehabilitation


is/are . . . .
1. Improvement in ROM
2. Restoration of functional movement
3. Augmentation of arterial, venous, and lymphatic blood flow
4. Fascilitation and relearning of voluntary and functional motor control
Jawaban : E

131. The following is/are TRUE about paraffin bath . . .


1. It is maintaned at about 52 degrees celcius
2. The indication is contractures particularly in the hands
3. Paraffin baths consist of 1:7 mixtures of mineral oil and paraffin
4. Three forms application of paraffin bath : dipping, immersion and uses a brush to
paint parrafin onto portions of the body
Jawaban : E

132. The following is(are) the advantage of SACH foot


1. Very simple
2. Quite cheap
3. Easy maintenance
4. Good ankle movements
Jawaban : A

A rigid dressing may be attached to a pylon and foot to allow ambulation soon after surgery.
The combination of rigid dressing, pylon and foot is called an immediate postoperative
prosthesis (IPOP). (Number 133-134)
133. The advantage(s) of IPOP is/are . . . .
627
1. Improved healing time
2. Protection of the surgical site
3. Reduction of contracture development
4. Control and shaping of the residual limb
Jawaban : E

134. The contraindication(s) to the use of an IPOP include . . .


1. Excessive postoperative edema
2. History of slow healing
3. Extreme obesity
4. Diabetic patients
Jawaban : A

135. Reflex effects of massage is/are . . .


1. Decrease in pain
2. General relaxation
3. Increased persipiration
4. Decrease muscle tightness
Jawaban : A

136. The following statement(s) is/are TRUE regarding glucocorticoid. .


1. Glucocorticoid exert their action by blocking prostaglandin and leukotriene
synthesis
2. The action is decrease synovial fluid complement
3. Decrease number of mast cell in synovium
4. Increase neutrophil number
Jawaban : A

137. The following is/are used for treatment lympedema . . .


1. Compression garment
2. Elevation
3. Massage
4. Icing
628
Jawaban : A

138. Test(s) to evaluate shoulder impingement syndrome is/are . . .


1. Speed test
2. Neer’s sign
3. Empty can test
4. Hawkins test
Jawaban : C

139. What is bankart lesion?


1. Compression of brachial plexus and /or subcalvian vessels as they exit between the
superior shoulder girdle and first rib
2. Injury to the glenoid labrum and biceps tendon (long head)
3. Compression fracture of the posterior humeral head
4. Tear or avulsion of anterior glenoid labrum
Jawaban : D

140. The following statement is/are the rehabilitation goal(s) of after rotator cuff repair
1. Maintain integrity of the repair rotator cuff. Never overstress healing tissue
2. Reestablished full passive ROM as quickly and safely as possible
3. Improved external rotation muscular strength. Reestablished muscular balance
4. Restore patient’s functional use of shoulder but do so gradually
Jawaban : E

141. A 13 year old girl is found to have scoliosis on demonstrate a Cobb angle routine
physical examination. Spine radiographs demonstrate Cobb angle of 25 degree. The optimal
treatment is . . .
1. Lateral surface electrical stimulation with trunk-strengthening exercise
2. Lateral surface electrical stimulation with bracing for 6 hourse a day
3. Surgical instrumentation and bony fusion procedures
4. Use of a Milwaukee brace for 23 hourse a day
Jawaban : D

142. The following is/are the clinical predictor(s) of aspiration found on bedside exam. . .
629
1. Presence of dysarthria
2. Ineffective tongue control
3. Altered cough mechanism
4. Vocal “wetness” when talking
Jawaban : E

143. The following is/are the late onset complication(s) after stroke
1. Seizures
2. Spasticity
3. Depression
4. Dysphagia
Jawaban : E

144. The following statement is/are TRUE regarding β-blocker


1. May cause hypoglycemia
2. May adverserly affect thermoregulatory function
3. Reduce submaximal and maximal exercise capacity
4. May lead to sudden reductions in post-exercise blood pressure
Jawaban : D

145. For children with cerebral palsy, tone-reducing orthoses should be used on the lower
extremities when the child . . . .
1. Begins to pull to a stand on toes
2. Stands flat but walk on the toes
3. Is non ambulatory but tends the feet in plantar flexion
4. Is non ambulatory but tends to keep the legs in extension
Jawaban : E

146. The symptoms comparison of pyramidal and extra pyramidal cerebral palsy is/are . . .
1. Contracture in pyramidal type is usually late
2. The tone of the extra pyramidal type often decreased
3. Involuntary movement is often present in pyramidal type
4. Clasp knife phenomenon is often occures in pyramidal type
630
Jawaban : D

147. Congenital muscular torticollis should be treated conservatively, but an appropiate


evaluation is needed to know the differential diagnosis. The following is/are the differential
diagnosis for congenital muscular torticollis.
1. Congenital absence of one of the cervical muscles.
2. Benign paroxysmal torticillis
3. Klippel-Feil syndrome
4. Hemivertebra
Jawaban : E
148. The following is/are the goal(s) of the rehabilitation prescription for COPD patients
1. Improve endurance
2. Optimize oxygen utilization
3. Increase independent functioning
4. Optimize airways secreation elimination
Jawaban : E

149. The following is/are the component(s) of pulmonary rehabilitation for COPD patient . . .
1. Assess baseline 6-minute walk
2. Train in energy and work conservation
3. Assess and advices modification nutritional intake
4. Titrate O2 flow to maintain Sp.O2 >99% during exercise
Jawaban : A

150. Nutritional supplements necessary for COPD patients because inappropriate nutrition
can . . .
1. Impair surfactant synthesis
2. Impair respiratory muscle function
3. Lead to hypercapnic respiratory failure
4. Difficulty in weaning from mechanical ventilation
Jawaban : E

631
Ujian September 2019
CHOOSE THE MOST APPROPRIATE ANSWER

1. The following is TRUE about Myofascial Pain Syndrome (MPS), EXCEPT .........
a. Is one of the most common case of musculoskeletal pain
b. Is characterized by myofascial trigger points (TrPs) in skeletal muscle
c. The tender areas are generalized , diffuse, involving bilateral and symmetrical
muscle
d. TrP is defined as a hyperirritable spot associated with a hypersensitive palpable nodule in
a taut band
e. Electromyographic studies show that the trigger point within the taut band show
distinctive spontaneous electrical activity
2. Anterior knee pain syndrome is associated with the following signs, EXCEPT ........
a. Positive anterior drawer test
b. Tight lateral thigh structures
c. Atrophy of quadriceps
d. Q angle more than 15
e. Lateral patellar tilt
3. Which maneuver is used to identify sacroiliac joint pathology?
a. Ober
b. Thomas
c. Shcoeber
d. Lasegue
e. Gaenslen
4. A 45-year-old has been hospitalized for an acute exacerbation of rheumatoid arthritis. In
order to prevent additional loss of strength you would prescribe .........
a. Isotonic exercises
b. Isometric exercises
c. Isokinetic exercises
d. Stretching exercises
e. Passive ROM exercises

5. Ulnar deviation of the fingers in rheumatoid arthritis has many causes if the following
factors, EXCEPT .........
a. Synovitis and capsulitis in the MCP joint
b. Extensive tendosynovitides in the flexor tendons
c. The contraction of the long flexor tendons of the fourth and fifth fingers
d. Any external stress by movements performed in the course of one’s daily life
e. Sliding down from the back of the metacarpal capitlum in the direction of the ulna

6. A 27-year-old complain of pain in the shoulder an suprascapular region 1 month after


undergoing modified radical neck dissection for carcinoma of the thyroid. Examination

632
shows decreased sensation to light touch over the superior aspect of the right trapezius and
scapular winging with resisted shoulder abduction. Which of the following nerve was the
most likely transected during the procedure?
a. Subclavius
b. Long Thoracic
c. Suprascapular
d. Dorsal Scapular
e. Spinal accessory

7. The patient may complain of numbness, paresthesia or dysesthesis radiating to the


first, second, third an lateral fourth digits. Symptoms may exacerbated during sleep
and relieved with wrist shaking. This condition found in ..........
a. Cubital Tunnel syndrome
b. Carpal Tunnel Syndrome
c. Pronator Teres Syndrome
d. Cervical Root Syndrome
e. Guyon Syndrome
8. Which type of aphasia would you expect to see in a patient who is noted to have
fluent speech, with signs of comprehension, but unable to repeat?
a. Broca aphasia
b. Anomic aphasia
c. Conduction aphasia
d. Transcortical motor aphasia
e. Transcortical sensory aphasia
9. A 60-year-old man who had a left cortical stroke 6 months ago wishes to improve
his hand function. He has good cognition. Sensation was only mildly decreased to
light touch. Muscle strength of shoulder flexor 4/5, elbow flexor 3/5, elbow
extensor 3/5, wrist extensor 3/5, finger flexor 2/5, finger extensor 2/5. Which
technique is most likely to result in functional improvement in this patient?
a. ROM exercises
b. Constraint-induced movement
c. Electrical stimulation to finger flexors
d. Propioceptive neuromuscular fascilitation
e. Electromyographic biofeedback to wrist extensor muscle
10. The correct statement for stage 4 of Brunnstroms recovery in stroke patient is …
a. more complex movement combinations are learned as the basic synergies
lose their dominance over motor acts
b. basic synergy patterns appear, minimal voluntary movements may be
present
c. patient gains voluntary control over synergies, increase in spasticity
d. synergy patterns still predominate, decrease in spasticity
e. increase in spasticity and synergy patterns

633
11. All of the following statements are correct regarding cerebral palsy, EXCEPT…
a. children with CP who do not gain independent sitting balance by 4 years of
age have a poor prognosis for motor skill development
b. the neurologic deficit is stable, but clinical manifestations vary as the
nervous system matures
c. sitting by the age of 4 years is a good prognostic sign for walking
d. children with GMFCS V had limited trunk control
e. spastic type of CP prone to develop contracture
12. The exercises regimen for children with myopathy include …
a. anaerobic exercise
b. eccentric exercises
c. concentric exercises
d. maximal aerobic activity
e. high resistance exercises
13. Child with a congenital transverse radial limb deficiency should have in initial
prosthesis fit at what developmental stage?
a. At the time of first sitting independently
b. At the time of starting kinder garden
c. At the time of initially walking
d. At soon as possible after birth
e. At 3-year-old
14. The following drugs for reduce spasticity can cause sedation, EXCEPT …
a. Baclofen
b. Diazepam
c. Clonidine
d. Tozanidine
e. Dantrolene sodium
15. The one of the following characteristic spastic diplegia of cerebral palsy is NOT
the appropriate statement.
a. Persistent obligatory infantile reflex activity beyond 18 months implied
poor prognosis for ambulation
b. Children with spastic diplegia have not adequate movement control of
the arms and hands.
c. Children with spastic diplegia who did not sit by 4 year have not achieved
ambulation
d. Sitting by 2 year was a good predictive sign of eventual ambulation
e. Children with spastic diplegia have good prognosis for ambulation
16. What type of Cerebral Palsy (CP) with unsteady gait, dysmetria, intention tremor in
upper extremities and truncal titubation?
a. ataxia type
b. mixed type
c. spastic type
d. athetoid type
e. hypotonic type

634
17. Dyspnea, tachycardia and sudden decrease of SaO 2 to less than 90% in phase I
cardiac rehabilitation may cause by the following, EXCEPT …
a. Atelectasis
b. Pneumonia
c. Emphysematous
d. Pulmonary embolism
e. Exacerbation of CHF
18. For the post-stroke patient with history of coronary artery disease and fibrillation,
an aerobic exercise session should be terminated if …
a. Heart rate does not increases
b. Heart rate increases to 20% over baseline
c. Respiratory rate increases 10% over baseline
d. Diastolic blood pressure increases to 120 mmHg
e. Systolic blood pressure increases to 160 mmHg
19. The 6MWT is one of the clinical exercise testing, has been used for the following,
EXCEPT …
a. Provide the mechanism of exercise limitation
b. Monitor response to therapeutic intervention
c. Preoperative and postoperative evaluation
d. Assessment of functional capacity
e. Predict morbidity and mortality
20. What exercises are recommended for persons with non-progressive mild to
moderate restrictive lung disease (VC of 60% predicted)?
a. Inspiratory resistive exercise
b. Glossopharyngeal breathing
c. Rapid shallow breathing
d. Diaphragmatic breathing
e. Pursed lip breathing
21. The major goal of phase II cardiac rehabilitation is …
a. Assessment of cardiovascular function
b. To achieve cardiovascular conditioning
c. Alleviation of anxiety and depression
d. ADL independency
e. Early ambulation
22. During vigorous exercise, while participating in a stress test, it is noted that the T
wave increases by 10%. No other changes are noted. How should this be
interpreted?
a. Definitely abnormal – highly suspicious but not absolutely diagnostic of
ischemic heart disease
b. Mildly abnormal – would recommend that maximum effort stress test be
performed
c. Definitely abnormal and diagnostic for ischemic heart disease
d. Definitely abnormal and diagnostic for myocardial infarction
e. Normal

635
23. The following factors is the body function and structures of ICF core set of risk
factors for falls in acute rehabilitation settings, EXCEPT …
a. ADL dependency
b. Vestibular function
c. Propioceptive function
d. Blood pressure function
e. Visuospatial perception
24. Global decline in cognitive capacity in a full conscious geriatric patients is
commonly caused by …
a. Anxiety
b. Delirium
c. Dementia
d. Major Depression
e. Couping behavior
25. Functional assessment of instability in the older faller is …
a. vision
b. vestibular
c. proprioception
d. Standing reach test
e. activities at time of fall
26. A 45-year-old woman had a modified radical mastectomy with axillary dissection
for breast cancer and now has severe lymphedema. Evaluation reveals no evidence
of recurrent tumor, infection, or venous thrombosis. To maintain edema control,
you recommend …
a. diuretic medication
b. Codman’s exercises
c. anti-inflammatory medication
d. low-dose antibiotic prophylaxis
e. use of a graded compression garment
27. How does the anterior cruciate ligament (ACL) injury become torn?
a. Deceleration of the leg via quadriceps contraction combined with
valgus and external rotation forces upon a slightly flexed knee.
b. Sudden external rotation of hyperflexed knee
c. Sudden internal rotation of hyperextend knee
d. Sudden hyperflexion of the knee
e. Direct blows to the ankle
28. The following statement is TRUE about of posterior interosseous nerve syndrome.
a. Pain reproduction with resisted wrist extension while elbow extended
b. Pain with resisted long finger extension is a specific finding
c. Cause by nerve compression at the arcade of Frohse
d. Weakness of digital flexor can occur
e. X-rays are usually helpful

636
A young basketball athlete, 8-year-old, injured his right ankle while playing basketball in a
competition 1 week ago. His x-ray films showed no fracture. His physician diagnosed the injury
as lateral ankle sprain. Visual Analog Scale has decrease in intensity from 6/10 to 4/10, but the
pain increases with weight bearing and certain demonstrated movements. Physical examination
reveals no warmth of the skin of anterolateral aspect of the right ankle, slight ecchymosis and
swelling are noted. His anterior drawer test and talar test is negative (number 29 and 30).

29. What is the most likely diagnosis?


a. Grade I ankle sprain, a minor tear with instability
b. Grade I ankle sprain, a minor tear with no instability
c. Grade II ankle sprain, a partial tear with instability
d. Grade II ankle sprain, a partial tear with no instability
e. Grade III ankle sprain, a complete tear with gross instability
30. In this case, what structure has been injured?
a. Anterior talofibular ligament
b. Posterior talofibular ligament
c. Calcaneofibular ligament
d. Syndesmotic ligament
e. Tibiotalar ligament
31. Surface electrodes for recording antidromic sural nerve conduction studies are best
placed at …
a. posterior to the medial malleolus
b. posterior to the lateral malleolus
c. anterior to the medial malleolus
d. anterior to the lateral malleolus
e. posterior gastrocnemius
32. The following statement about type 1 muscles fiber is NOT correct.
a. Contain a large number of mitochondria and myoglobin
b. Dominant in much of marathon runner
c. Also called as slow twitch fibers
d. Predominantly aerobic
e. Generate larger force
33. The primary muscle of inspiration is …
a. sternocleidomastoid
b. rectus abdominalis
c. pectoralis major
d. diaphragm
e. trapezius
34. During exercise, there is an increase of physiological response in a person’s. That is

a. stroke volume
b. residual volume
c. diastolic pressure
d. total peripheral resistance
637
e. pulmonary arterial resistance
35. A pulmonary embolism which may occur in a prolonged immobilization patient
mostly due to …
a. development of atherosclerosis
b. development of atrial fibrillation
c. decreasing the total lung capacity
d. decreasing of the lung vital capacity
e. development of deep venous thrombosis
36. The primary afferent fibers coming from the nuclear chain of the muscle spindle
have the following conduction velocity …
a. <40 m/sec
b. 40-60 m/sec
c. 61-70 m/sec
d. 71-80 m/sec
e. 81-90 m/sec
37. The subtalar joint of the foot is an articulation between the …
a. Tibia and talus bone
b. Fibula and talus bone
c. Calcaneus and talus bone
d. Talus and navicular bone
e. Calcaneus and cuboid bone
38. Atlanto-odontoid is classified as …
a. Gynglimus joint
b. Trochoid joint
c. Ellipsoidal joint
d. Condyloid joint
e. Ball and socket joint
39. The real leg length is the distance between …
a. Umbilicus to medial malleolus
b. Umbilicus to lateral malleolus
c. Femoral neck to lateral malleolus
d. Greater trochanter to lateral malleolus
e. Anterior superior iliac spine to medial malleolus
40. The most function of gluteus maximus during gait when …
a. loading response
b. terminal stance
c. double support
d. mid stance
e. pre swing
41. What is the characteristics of swan neck deformity?
a. Flexion of MCP and DIP, hyperextention of PIP
b. Flexion of MCP, hyperextention of PIP and DIP
c. Extention of MCP, flexion of DIP and PIP
d. Extention of MCP and DIP, flexion PIP

638
e. Extention of MCP, PIP and DIP

42. In the normal gait pattern, the ankle is held in a neutral position at …
a. toe off
b. foot flat
c. heel strike
d. mid stance
e. mid swing
43. Produces deep heating through the conversion of radiowave electromagnetic energy
to thermal energy is …
a. Radial shockwave therapy
b. Ultrasound diathermy
c. Shortwave diathermy
d. Microwave diathermy
e. Ultraviolet radiation
44. Which one of the following CPM (Continuous Passive Motion) is contraindicated?
a. Burn patients
b. Capsulotomies
c. Unstable fracture
d. Total knee replacement
e. Fractures with open reduction internal fixation
45. The good conductor for electric current is …
a. ligament
b. tendon
c. muscle
d. nerve
e. bone
46. When treating supraspinatus tendonitis with USD, which shoulder position is most
beneficial?
a. Arm abducted and internally rotated
b. Arm abducted and externally rotated
c. Arm adducted and externally rotated
d. Arm adducted and internally rotated
e. Position of comfort
47. A 50-year-old woman with secondary progressive multiple sclerosis has mild
cognitive deficits and sever lower-limb spasticity. What the most effective
treatment for this patient?
a. Range of motion exercise
b. Warm water exercise
c. Intrathecal baclofen
d. Diazepam injection
e. Cold therapy
48. The volume ratio between paraffin and mineral oil in paraffin bath treatment is …
639
a. 1:7
b. 1:6
c. 1:5
d. 1:4
e. 1:3
49. The following are the causes of circumduction in patient with lower limb
prosthesis, EXCEPT …
a. socket so small
b. abduction contracture
c. foot in plantar flexion
d. inadequate suspension
e. insufficient knee flexion during swing
50. In a hemiplegic, the function of a posterior stop in a double upright AFO with dual
action adjustable ankle joint and extended steel shank is to …
a. improve knee stability at heel strike when the posterior stop is set in 5
degrees of dorsiflexor rather than plantar flexion
b. cause the ground reaction force line to fall posterior to the knee joint in mid
stance
c. assist in control of moderate to severe plantar flexor spasticity
d. assist weak plantar flexors
e. assist weak hamstring
51. A logical approach to the initial management of phantom pain 1 week
postoperatively is …
a. TENS
b. Epidural steroid injection
c. Local ice pact treatment
d. Lumbar sympathetic block
e. Surgical neuroma excision
52. When should upper extremity prosthesis fitting be initiated in the adult?
a. When residual limb strength is full
b. When the patient requests a prosthesis
c. Within the first month after amputation
d. When residual limb volume has stabilized
e. When there is no phantom pain sensation
53. The following are the primary goals of treatment during preprosthetic phase after
lower limb amputation in the dysvascular patient, EXCEPT …
a. promote healing
b. prevent mobility
c. control pain adequately
d. prevent hip and knee contracture
e. adjust to changes in body image
54. The following is NOT TRUE regarding the function of upper limb splints.
a. To strengthening the muscles
640
b. To prevent contractures
c. To rest the body part
d. To correct deformity
e. To promote activity
55. The language characteristic in Boston Classification of Aphasia which fluent
neologistic language, poor comprehension and intact repetition is …
a. tanscortical sensory type
b. transcortical motor type
c. conduction type
d. Broca’s type
e. global type
56. A 70-year-old woman complains of right knee pain and stiffness. On physical
examination, she has a genu varum deformity with unstable joint. A exercise
training prescription should include …
a. isokinetic hamstring strengthening
b. isokinetic quadriceps strengthening
c. isometric gastrocsoleus strengthening
d. open kinetic chain quadriceps strengthening
e. closed kinetic chain quadriceps strengthening
57. You are seeing a 56-year-old male patient in consultation 3 days after severe stroke.
He is medically stable and has flaccid hemiplegia with poor sitting balance. He is
sitting up in a chair for 2 hours twice daily and has just started bedside physical
therapy and occupational therapy. You would recommend …
a. Strength the flaccid arm
b. Improve with less intensive physical therapy.
c. Focusing on sitting balance, followed by transfer
d. Start aggressive physical and occupational therapy
e. Start electrical stimulation to the flaccid arm to enhance neurologic
recovery.
58. The most common method of exercise based on the neurodevelopmental approach
for cerebral palsy patient is the …
a. Rood
b. Voyta
c. Kabat
d. Phelps
e. Bobath
59. These are the effects of treadmill training in Parkinson patients EXCEPT …
a. it reduces tremor
b. it decrease fall risk
c. it increase endurance
d. it improves quality of life
e. it improves gait impairment
60. The following approach to normalize tone and active purposeful movement and
postural response using sensory motor stimuli is called …

641
a. Rood approach
b. Bobath approach
c. Movement therapy
d. Brunnstrom approach
e. Kabat and Voss approach
61. Yergason test is a test to detect inflammation of the following muscle.
a. Medial head of the triceps
b. Long head of the triceps
c. Short head of the triceps
d. Long head of the biceps
e. Short head of the biceps
62. Early symptoms of ankylosing spondylitis in a young adult is …
a. upper cervical pain
b. lower cervical pain
c. upper thoracal pain
d. lower thoracal pain
e. low back pain
63. The following is TRUE about plantar fasciitis.
a. Is common in patients whose occupation requires long period of sitting
b. There is deep tenderness of the posterolateral aspect of the calcaneus
c. The condition is more prevalent in people with supinated feet
d. Use of a accommodation shoes to improve the pain
e. The treatment is stretching of the heel cord
64. A common source of shoulder pain in shoulder impingement syndrome is …
a. Bursitis
b. Osteoarthritis
c. short head of biceps
d. long head of biceps
e. the rotator cuff tendon
65. The following below is NOT cause of the chronic ankle instability.
a. Ligamen laxity
b. Achilles tendinitis
c. Recurrent ankle sprain
d. Peroneal muscle weakness
e. Ankle proprioceptive deficit
66. The following statement is NOT the characteristic signs of conus medullaris
syndrome …
a. Perineal sensory deficit
b. Sphinter dysfunction
c. Loss of anal reflex
d. Pyramidal sign
e. Incontinentia
67. The mechanism of diazepam to reduce the muscle spasticity is by …
a. Stimulating the GABA-mediated inhibition in CNS

642
b. Inhibiting glutamate and aspartate in CNS
c. Inhibiting the extrafusal fiber
d. Inhibiting the pyramidal tract
e. Inhibiting the motor cortex
68. Parkinsonian feature which impaired automatic motor task performance is …
a. bradykinesia
b. hypotonia
c. spasticity
d. weakness
e. dystonia
69. The nerve most commonly injured with posterior or posterolateral approach for
total hip replacement is the …
a. superior gluteal
b. obturator
c. peroneus
d. femoral
e. sciatic
70. When patients with post traumatic brain injury (TBI) ask about returning to driving,
you tell them that …
a. all persons with post TBI can drive
b. persons with brain injury should not drive
c. only an on-road evaluation is necessary to determine fitness for driving
d. persons with TBI must wait at least 1 year post-injury before resuming
driving
e. clinical assessment should include sensorimotor impairment, vision,
and cognitive function
71. Management of PesPlanus grade 1 at age less than 1 years old is done by …
a. observation
b. Thomas heel
c. arches support
d. boot type shoe
e. Chukka type shoes
72. Congenital muscular torticollis caused by …
a. hemivertebra
b. ipsilateralplagiocephaly
c. deformation from intrauterine positioning
d. fibrotic contracture of the paraspinal muscle
e. fibrotic contracture of the upper trapezius muscle
73. A 3-year-old girl with Juvenile Rheumatoid Arthritis (JRA) has involvement
limited to the right knee and left ankle. Her referring physician noted a positive
antinuclear antibody test. You explain to her mother that type of JRA is associated
with …
a. diurnal temperature variations
b. a high incidence of iridocyclitis

643
c. spondylitis of the thoracic spine
d. lymphadenopathy and hepatosplenomegaly
e. severe erosive changes and residual disability

74. The rehabilitation medicine program for children with delayed development
include the following programs, EXCEPT …
a. Cognitive and behavioral development stimulation
b. Gross motor development stimulation
c. Speech development stimulation
d. Prevocational counseling
e. Fine motor stimulation
75. A baby of 3 months old shows a symmetrical weakness, proximal muscles more
than distal. Abdominal breathing, no sensory deficits. The strong possibility of
diagnosis is:

A. Poliomyelitis
B. Guillan bare syndrome
C. Spinal muscular atrophy
D. Duchene muscular dystrophy
E. Transverse lesion of the spinal cord

76. The child has chief complaint of bowlegs that are gradually progressing in severity
and not improving spontaneously. The deformity is called …
a. clubfoot
b. tibia vara
c. knock knees
d. genu recurvatum
e. anteversion deformity
77. All the following conditions are related to obesity, EXCEPT …
a. osteoporosis
b. hypertension
c. pulmonary hypotension
d. Type II diabetes mellitus
e. decrease tolerance to heat
78. The statement below is TRUE for prescribe exercise training for patient with
coronary vascular disease.
a. Risk stratify the patient using the risk stratification schema (the AHA and
the AACVPR) is not recommended
b. Symptom-limited exercise testing prior to participation is strongly
recommended
c. For resistance exercise the intensity 25-30 repetitions per set to fatigue
d. Exercise training is based on comorbidities only
e. Calisthenic is modalities for aerobic exercise
79. The following is an absolute contraindications to exercise tolerance testing.
644
a. Unstable angina
b. Controlled diabetes mellitus
c. FEV1> 60% of predicted value
d. Chronic compensated heart failure
e. Resting blood pressure 140/90 mmHG

80. The following is TRUE regarding glossopharyngeal breathing (GPB) or frog


breathing.
a. The patient take a deep breathing and the augment it by projecting
boluses of air into the lungs by pumping action of the glottis, then the
glottis closes with each gulp.
b. A GPB as an invasive method for emergent support of ventilation
c. One breath usually consists of 6-8 gulps of 500 ml air each.
d. A tracheostomy include us of GPB
e. GPB rate of 2-4 breaths/minute
81. The following respiratory therapy of COPD patient is NOT TRUE.
a. Oxygen therapy should be used for patients with PaO2> 60 mmHg
b. Titrate O2 flows to maintain SaO2> 90% during exercise
c. Inhaler use to prevent tongue deposition of medication
d. Respiratory muscle resistive exercise training
e. Diaphragmatic and pursed lip breathing
82. The following statements is NOT TRUE regarding the purpose of resistance
training for patient with cardiac disease.
a. Improve self-confidence
b. Improve muscular strength
c. Decrease muscular endurance
d. Decrease cardiac demands of muscular work during daily activity
e. Prevent and treat osteoporosis, type 2 diabetes mellitus and obesity
83. Which of the following would most optimally be included in an initial exercise
program for a healthy elderly person with moderate degenerative osteoarthritis?
a. Tennis
b. Treadmill
c. Volleyball
d. Step aerobic
e. Stationary cycling
84. What is the most common cause of dementia in geriatric population?
a. Toxic – Metabolic dementia
b. Traumatic brain injury
c. Lewy body dementia
d. Vascular dementia
e. Alzheimer disease
85. Which of the following is most likely to enable you to determine the reason for a
fall in an older patient?
645
a. Muscle strength examination
b. A completed neurologic examination
c. Magnetic resonance imaging of the brain
d. Determination of the patient’s functional status
e. A careful history of the circumstances surrounding the fall

86. In aging people, which lung function increases?


a. Forced expiratory volume in 1 second
b. Forced vital capacity
c. Total lung capacity
d. Residual volume
e. Vital capacity
87. A male patient, 30 years old, no history of regular sport activities, comes with a
chief complaint of pain on the right knee joint around the patellar tendon insertion
after play football. This worst when he runs or climbs the stairs. The most probable
diagnosis is …
a. Osgood – Schlatter disease
b. Chondromalacia patella
c. Ligamental sprain
d. Meniscal tear
e. ACL tear
88. To prevent sport injury especially for lower extremity, should be strengthen
following core muscles. That are …
a. abdominal muscle, hip extensor, knee extensor
b. abdominal muscle, hip internal rotator, hip extensor
c. abdominal muscle, hip internal rotator, hip extensor, knee extensor
d. abdominal muscle, hip abductor, back extensor, hip external rotator
e. abdominal muscle, hip external rotator, back extensor, knee extensor
89. The following statements are TRUE regarding heel pain syndrome, EXCEPT …
a. This overuse phenomenon is commonly seen in runners
b. The most common site of heel pain is the medial calcaneal tuberosity
c. This results from repetitive heel impact along with hyperflexion of the
toes
d. Microscopic tissue degeneration can occur and a calcaneal spur maybe seen
e. The medial calcaneal tuberosity is the origin of the plantar fascia, flexor
digitorum brevis, quadrates plantae, and abductor hallucis muscles
90. The following statement is NOT TRUE regarding medial epicondylitis (golfer
elbow)
a. A painful tendinitis of the common flexor-pronator origins, adjacent to the
medial epicondyle
b. Sports often associated with this injury include golf, racquet and bowling,
c. Chronic overload results in inflammation, microtears and calcifications
d. The origin of extensor carpi radialis brevis is most often involved
646
e. Resisted wrist flexion and pronation are painful
91. The most common cause of significant knee hemarthrosis after traumatic injury is

a. anterior cruciate ligament rupture
b. lateral collateral ligament tear
c. osteochondral fracture
d. patellar dislocation
e. meniscal tear
92. A 26 year old mother of 18-month-old twins presents with a complaint of pain in
her right shoulder, which started several weeks ago. The pain has progressed
significantly, and she finds it hard to perform activities that involve reaching over
her head. Physical examination abnormalities include pain in anterior aspect of the
shoulder with palpation as well as pain with resisted elbow flexion and supination
of the arm. What is the most likely diagnosis for this patient?
a. Osteoarthritis
b. Bursitis calcarea
c. Bicipital tendinitis
d. Incomplete rotator cuff tear
e. Degenerative disease of the acromioclavicular joint.
93. The facet joint most often affected in osteoarthropathy of the cervical spine is …
a. C1-2
b. C2-3
c. C4-5
d. C5-6
e. C7-T1
94. In an advance carpal tunnel syndrome, the following muscles will be atrophied.
a. 2nd lumbricalis
b. Opponens pollicis
c. Abductor pollicis brevis
d. deeper part of flexor pollicis brevis
e. Superficial part of flexor pollicis brevis
95. The presence of a Scalenus Anticus Syndrome can be detected through the
following sequence of maneuvers.
a. Deep inspiration – turn the head in circumduction
b. Deep exhalation – turn the head foreward and lateralward
c. Deep inspiration – turn the head foreward and lateralward
d. Deep inspiration – turn the head backward and lateralward
e. Deep exhalation – turn the head backward and lateralward
96. Shoulder pain are common after stroke episode. The following statement is TRUE
regarding shoulder pain after stroke.
a. Is usually due bursitis calcarea
b. Is usually due to reflex sympathetic dystrophy
c. Is usually due to central pain/thalamic syndrome
d. Is usually due to subluxation the head of humerus

647
e. Requires injection with steroids for effective management
97. A stroke patient shows signs of weakness and sensory deficits especially in he right
lower extremity, while the right upper extremity shows minimal disturbance. There
are also slight cognitive disturbance and aphasia. This condition most probability
affecting the following artery.
a. Middle cerebral artery
b. Anterior cerebral artery
c. Posterior cerebral artery
d. Superior cerebral artery
e. Posterior inferior cerebellar artery

98. All of the following are correct regarding the bowel training in SCI patient,
EXCEPT …
a. to give fiber-rich diet
b. choose the stimulating environment
c. give stool softener prior to he training
d. consider the patient’s pre-injury bowel habit
e. consider the gastro-colic reflex when starting the training
99. A 42-year-old factory worker you have diagnosed and treated with carpal tunnel
syndrome. The most important objective testing that confirms your diagnosis
include the one of the following test.
a. Nerve conduction studies and electromyography
b. Tinel’s test at the wrist
c. Sensory examination
d. X-rays of the wrist
e. Prayer test
100. Muscle weakness in the following conditions are LMN type, EXCEPT …
a. GBS
b. DMD
c. Polio
d. Diabetic
e. Multiple sclerosis
101. When prescribe shoes for patient with rheumatoid arthritis, you should consider…….B
1. Rubber and buffer for better contact and additional shock absorption
2. Sufficiently wide heel, between 10 cm high
3. Broad forefoot section
4. Hard forefoot sole
102. Secondary Frozen shoulder results from a known precipitating events such as……E
1. Cervical spondylosis
2. Tendonitis of rotator cuff
3. Fractures of shoulder region
4. Degenerative arthritis of glenohumeral joint
103. NSAIDs must be used with caution in the elderly because have adverse drug reaction such
as…..E
1. Gastrointestinal ulceration
648
2. Inhibition of platelet aggregation
3. Inhibition of antidiuretic hormone
4. Increased renal reabsorption of chloride and K +
104. The following is/are TRUE regarding pyriformis syndromes….A
1. Examination reveals buttock tenderness, pain over the pyriformis fossa
2. Pain with sustained hip flexion, adduction and internal rotation
3. Pain is exacerbated by sitting and athletic activity
4. Initial treatment with pelvic rotation
105. A girl patient with idiopathic scoliosis the following is(are) the sign(s) that Cobb’s angle might
be increasing rapidity…..A
1. Her pubic hairs are growing
2. Her breast are growing bigger
3. She has menarche a month ago
4. The Risser’s sign in reaching to grade 4
106. The following statements is/are the characteristic of the spondylolisthesis….E
1. Is the gradual displacement of one vertebra on another vertebra
2. On the oblique x-ray may observe the “Scotty dog”
3. Produce pain with extension and hamstring tightness
4. A stress fracture through the pars interarticularis
107. Flaccid dysarthria common find in which disease(s) below…..A
1. Bell’s Palsy
2. Mystenia gravis
3. Guillain Barre Syndrome
4. Bilateral cortical stroke
108. The common complication in individuals with SCI is/are…….E
1. Neurogenic bladder/bowel
2. Deep venous thrombosis
3. Pressure ulcers
4. Pneumonia
109. What is post-polio syndrome? E
1. A confirmed history of paralytic poliomyelitis
2. Partial to fairly complete neurologic and functional recovery
3. A period to neurologic and functional stability of at least 15 years duration
4. Onset of 2 or more of the following health problems since achieving period of stability
110. The risk factor(s) for disability after stroke is/are……E
1. Global aphasia
2. Bilateral lesions
3. Impaired cognition
4. Sensory and visual deficits
111. The following is/are action of phenol injection for spaticity management….A
1. Side effect is risk of destruction to surrounding tissues
2. Effect on motor power is minimal weakening
3. Side effects is risk of dysesthesias
4. The duration of effect is 15 days
112. A stroke involving the vertebra-basilar system may give the following manifestatiton(s). B
1. Ataxia
2. Cortical blindness
3. Hemiplegia alternans facialis
4. Unhibited neurogenic bladder
113. The following is/are the characteristic(s) of Duchenne’s Muscular Dystrophy. E
1. The disease affects males
2. It is transmitted by X linked mode of inheritance
649
3. Measurement of serum creatinine kinase is the most reliable test
4. The child’s mother, his sisters and maternal aunts are potential carriers
114. The following is/are TRUE about Legg-Calve-Perthes disease….E
1. Predispose to pain and late degeneration
2. Is classified as one of the ostechondrosis
3. Is a disorder affecting the capital femoral epyphysis
4. Is one of the most common hip disorders of childhood
115. The following is/are the secondary complication(s) of children with Brachial plexus injury.E
1. Torticollis
2. The affected arm is shorter
3. Lack of awareness of the arm
4. Muscle atrophy and joint contracture
116. In Klumpke’s type of brachial plexus injury the following muscle is/are spared. D
1. Flexor pollicis longus
2. Extensor pollicis brevis
3. Flexor digitorum profundus
4. Extensor carpi radialis longus
117. A 15-year-old boy has Juvenile rheumatoid arthritis, manifested hip and knee contracture. The
following appropriate treatment is/are…….A
1. Prone position in bed
2. Stretching exercise twice daily
3. Low-impact and low intensity endurance exercise
4. Ultrasound diathermy treatment of both hips and knees
118. The following statement(s) is/are the special precautions that should be applied when designing
exercise for children….E
1. Children with the certain illnesses may require adjusments in their exercise prescription
2. Children may experience a higher incidense of damage the ephiphyseal growth plate
3. In a hot environment, children show less heat dissipitation than adult
4. In a cold environment, increasing their risk of hypothermia
119. Man, 55 year old, weight 85 kg, height 165 cm. He got myocard infarct 1 month ago. The
fasting glucose level 250, total cholesterol 400, no hypertension. Factor(s) that should be
considered to prescribe cardiac rehabilitation program is/are…….C
1. Avoid all of exercise
2. Assessed the stress test
3. Limitation of daily activity
4. Reduce the risk factor for 2nd cardiac disease attack
120. Which of the following statement(s) about static/isometric exercise is/are accurate for patient
with heart disease?....D
1. It is inappropriate for all patients with heart disease
2. It results in reduced systolic blood pressure
3. It result in reduced pressure on the heart
4. It is results in increased heart rate
121. The benefit(s) of pulmonary rehabilitation is/are………..A
1. Improve in the quality of life
2. Improve in exercise tolerance
3. Reduction in number of hospitalization
4. Able to complete simple activities with increasing heart rate more than 120 bpm
122. The American Geriatrics Society’s goal of diabetic control in the elderly include(s)…..D
1. Tight glycemic control
2. Hemoglobin A1c below 7%
3. Fasting plasma glucose below 120 mg/dl
4. Reduction of cardiovascular complication
650
123. Improve aerobic capacity and increased oxygen transport is the effect of exercise in cardiac
patient. This will result(s) from….
1. Decreased cardiac output
2. Decreased resting heart rate
3. Decreased ejection fraction
4. Decreased systolic blood pressure
124. The aim(s) of chest physical therapy in asthmatic patient is/are to…..E
1. Relieve bronchospasm
2. Aids removal of secrets
3. Coordinate respiratory movements
4. Assist relaxation and gain control of breathing
125. Risk of falling in cancer patients is/are due to……
1. Hemiplegia secondary to a brain tumor
2. Chemotherapy-related neuropathy
3. Steroid-induced myopathy
4. Orthostatic hypotension
126. Older adults are the least physically active group, the essential for healthy aging to maintained
regular physical activities is/are from of exercise type…..
1. Balance exercise
2. Flexibility exercise
3. Endurance exercise
4. Muscle strengthening exercise
127. For patient with osteoporosis, the following of exercise must be AVOID.
1. Exercise frequency three to five days per week
2. Weight bearing aerobic activities
3. Exercise duration 30-60 minutes
4. High intensity resistance exercise
128. Which of the following of patient(s) would be appropriate for osteoporosis investigation?
1. Women over age 65
2. Men with hypogonadism
3. Person with hyperparathyroidism
4. Women with menopause before age 40
129. The following is/are the musculoskeletal system sign(s) in SLE
1. Myalgia
2. Painful joints
3. Muscle weakness
4. Jaccoud-like arthropathy
A basketball athlete had injury after he jump and landed on his right foot in the internally rotate
position. He heard “pop” and felt painful on his right knee. He ceased from this game because he cannot
walk and got an edema suddenly (number 130 and 131).
130. What is/are possible diagnose for this condition? C
1. Quadriceps muscle rupture
2. ACL rupture
3. PCL rupture
4. Meniscal torn
131. What is/are special test(s) positive for his condition? C
1. Pattelar tilt test
2. Mc murray test
3. Posterior drawer test
4. Anterior drawer test

651
132. What rehabilitation program(s) is/are suited for case after one week grade I ankle sprain?
1. Cryotherapy
2. Rest and fixation
3. Ankle stretching exercise
4. Weight bearing and Rom exercise as tolerated
133. Patellofemoral pain in athlete is caused by sport(s) such as………
1. Soccer
2. Cycling
3. Running
4. Basketball
134. What is Biker’s palsy?
1. An entrapment or pressure neuropathy of the ulnar nerve on the ulnar side within
Guyon’s canal
2. Caused by direct pressure of the hand with standar handlebars
3. Methods of prevention include wearing padded bicycle gloves
4. The flexor carpi ulnaris muscle is intack
135. With the EMG machine we can get the following result(s), that is/are…..A
1. The severity of pathology
2. The level / location of motor unit pathology
3. The motor and sensory conduction velocities
4. The etiology of motor unit abnormalities
136. The change(s) that result from stress deprivation from disuse or mobilization of the joint
is/are……
1. Atrophy of cartilage
2. Adhesion between synovial fold
3. Declines collagen mass by approximately 10%
4. Excess deposition of restricting connective tissue in muscle
137. Sypmtoms(s) of tear the anterior talofibular ligaments is/are……
1. Pain when ankle joint is loaded or moved
2. Positive anterior drawer test in total tear of ligament
3. Swelling and tenderness in front of the lateral malleolus
4. Increased supination compared with the undamaged ankle joint
138. The following is/are the synovial joint(s) in the shoulder girdle
1. Acromioclavicular joint
2. Sternoclavicular joint
3. Glenohumeral joint
4. Scapulothoracic joint
139. To stabilize the shoulder against the upper reaction from a cane, patient need the good function
of these muscle. B
1. Pectoralis major
2. Pectoralis minor
3. Latissimus dorsi
4. Subclavius
140. The following muscle(s) is/are classified as the extrinsic muscle of the hand.
1. Extensor digiti minimi
2. Flexor digiti minimi
3. Palmaris longus
4. Brachioradialis
141. Transcranial magnetic stimulation
1. It can depolarize neurons and create evoked respons or change neuronal plasticity
2. It can study the cavity of motor cortices via electromagmetic induction
3. It can denote interhemispheric differences in corticospinal output
652
4. Is one of invasive method
142. The effects of ultra violet radiating is/are…..
1. Photochemical effects
2. Relief of muscle spasm
3. Have a biological effects
4. Increases the pain threshold
143. The characteristic reflection of ultrasound is/are…..
1. Occurs mainly at interfaces of tissues with different acoustic impedances
2. Is great at the interfaces of soft tissue and metallic implants
3. Is minimal at the interfaces of layers of soft tissue
4. Is great at the interface of soft tissue and bone
144. A 53-year-old woman with chronic venous disease has lower extremity edema, skin
hyperpigmentation, dermatitis, and venous varicosities. She does not have any ulceration. The
mainstay of treatment for this patient is/are……..
1. Intermittent external pneumatic compression
2. Hydrotherapy with arm water (100 degree F)
3. Topical steroid cream
4. Gradient elastic stocking
145. In checking out lower extremity prosthesis attention is/are directed to…….
1. Comfort
2. Stability
3. Aligment
4. Appearance
146. The choice of upper extremity spilints for a flexible claw hand is/are………
1. Cock up splint
2. Resting splint
3. Ulnar deviation splint
4. Knuckle-bender splint
147. Functionally, a transmetatarsal amputation is superior to a Lisfranc’s (tarsometatarsal)
amputation, because it provide(s)……….
1. An intact tibialis anterior insertion to oppose the pull of the triceps surae
2. A stump that is easier to fit
3. A broader base of support
4. A longer lever arm
148. The following is/are the positive effect of singing in group music therapy.
1. Improve mood
2. Improve social participation
4. Improve respiratory function
5. In quadriplegia, would increase recruitment of accessory respiratory muscle
149. The benefit(s) of regular exercise training in the elderly is/are………
1. Promotion of person’s health
2. Slowing of disease progression
3. Prolongation of functional independence
4. Prevention of aortic aneurysm and mitral valve dysfunction
150. The following is/are the swallowing compensation(s) to sustain laryngeal elevation during
swallow to increase airway protection.
1. Neck flexion
2. Neck rotation
3. Supraglotic swallow
4. Mendelsohn manuever

653
SOAL UTULOK 20 MARET 2020

1. Patient may demontrate a abnormal “OK” sign or have difficulty forming a first because of
an inability to approximate the thumb and index finger is clinical presentation?
a. Posterior interosseius nerve syndrome
b. Tardy ulnar nerve palsy
c. Cubitan tunnel syndrome
d. Anterior interosseus nerve syndrome
e. Pronator teres syndrome

2. Nerve control for coordinating bladder contraction and sphincter externa relaxation in
the voiding phase is?
a. Cerebral coretex
b. Pontin micturition center
c. Sacral micturition center
d. Pudendal nerve
654
e. Sympathetic nerve

3. The following included mononeuropathy multiplex is?


a. Guilla barre syndrome
b. Baxter neuropathy
c. Diabetic
d. Leprosy
e. Saturday night palsy

4. Type of muscular dystrophy that muscle usually affected first are hips and shoulders,
then progresses to the arms and legs, through progression is slow. Usually begin in the
teen or early adult years and autosomal recessive. This type is?
a. Limb girdle muscular dystrophy
b. Facio scapulohumeral muscular dystrophy
c. Distal muscular dystrophy
d. Emery dreifuss muscular dystrophy
e. Myotonic dystrophy

5. The cerebellum consists of three functionally distrinct parts with different roles
concerned primarilly with subconscious control of motor activity. One of them is
spinocerebellum. The following function of spinocerebellum is?
a. Maintaining balance
b. Control eye movements
c. Enhances muscle tone and coordinates skilled voluntary movements
d. Planning voluntary activity
e. Initiating voluntary activity

6. Nerve and muscle cells do not come into direct contact at a neuromuscular junction. The
space or clift, between these two structures is too large for electrical transmission of
an impulse between them. Just as at neural synapse, a chemical messenger carries the
signal between terminal button and the muscle fiber. The neurotransmitter is?
a. AchE

655
b. K
c. Na
d. Ca
e. Ach

7. Desending tracts which function as a facilitation of muscle tone is?


a. Lateral reticulospinal tract
b. Medial reticulospinal tract
c. Lateral corticospinal tract
d. Ventral spinocereberal tract
e. Lateral spinothalamic tract

656
8. Quadrilateral transfemoral socket?
a. Narrow mediolateral
b. The ischial tuberosity is contained inside the socket providing a bony lock
betweens ischium and greater trochanter
c. There are reliefs for abductor longus, hamstring, greater trochanter, gluteus
maximus and rectus femoris
d. Weight bearing is consentrated in the medial aspect of the the ischium and the
ischial ramus
e. Narrow mediolateral design was developed to provide a more normal anatomic
alignment of the femur insight the socket

9. During exercise, there is an increase in a person’s ?


a. Stroke volume
b. Diastolic pressure
c. Venous compliance
d. Pulmonary arterial resistance
e. Total peripheral resistance

10. Which muscle laterally rotates, extends and flexes the hip?
a. Glutens maximums
b. Glutens medius
c. Glutens minimums
d. Quadratus lumborum
e. Quadriceps femoris

11. The following statement is not typical characteristic of MUAP observed during EMG
test of patient with myopathy
a. Early recruitment
b. Short duraction
c. High amplitude
d. Low amplitude
e. None of the above

657
12. The H-reflex test of the median nerve recorded at the flexor carpi radial muscle can
be used to detected?
a. C5-C6 radiculopathy
b. C6-C7 radiculopathy
c. C7-T1 radiculopathy
d. C5 radiculopathy
e. C6 radiculopathy

13. The presence of nascent potential in EMG examination shows the following process
of?
a. Good prognosis
b. Poor prognosis
c. Reinnervation
d. Partial denervation
e. Total denervation

14. The MUAP, observed during electromyography examination of patient with motor
neuron disease EXECPT :
a. Long duration
b. High amplitude
c. Reduced motor unit recruitment
d. Low amplitude
e. Is correct none of the above

15. Vital capacity and residual capacity are …


a. Tidal volume
b. Residual volume
c. Total lung capacity
d. Functional capacity
e. Functional residual capacity

658
16. Based on a comprehensive patient assessment, the goals of long-term management
for the patient with chronic bronchitis may include the following is …
a. Reduce ventilation
b. Reduce alveolar ventilation
c. Reduce secretion clearance
d. Reduce the work breathing
e. Reduce perfusion matching and gas exchange

17. Physiological changes when changing from the upright to the supine position is …
a. Increased vital capacity
b. Increased residual volume
c. Increased airway resistance
d. Increased total lung capacity
e. Increased arterial oxygen levels

18. The termal effects of ultrasound diathermy :


a. Increase tissue temperature, decrease metabolic rate, decrease pain, increase
circulation
b. Increase tissue temperature, decrease pain, increase circulation, increase
metabolic rate
c. Increase tissue temperature, decrease metabolic rate, decrease circulation,
decrease pain
d. Increase tissue temperature, increase metabolic rate, decrease circulation,
decrease pain
e. Increase tissue temperature, increase metabolic rate, increase circulation, decrease
soft tissue extensibility

19. Where is the lesion site in ataxic dysarthria, as found in Friedreich’s ataxia?
a. Cerebellum
b. Multiple sites

659
c. Basal ganglia
d. Extrapyramidal system
e. Bilateral upper motor neuron

20. An infract in the lower division of the left middle cerebral artery division would be
associated with which type of aphasia?
a. Broca
b. Global
c. Wernicke
d. Transcortical motoric
e. Transcortical sensoric

21. Which type of stroke has the best outcome?


a. Haemorrhagic pontine
b. Internal capsular lacunar
c. Embolic left cortical infraction
d. Rupture of anterior cerebral artery aneurysm
e. Ischemic occlusion of the right internal carotid artery

22. Treatment of acquired dysphagia in 67-yo man with a left hemiparesis might include :
a. Chin tucking and reclining feeding
b. Chin tucking and head turning to the affected side
c. Changing diet to one of softer and thinner consistency
d. Upright feeding and head turning to the unaffected side
e. Use of long-term thermal stimulation and upright feeding

23. Using a computer mouse that is too far to the side or too high in the table will lead
to :
a. Anterior shoulder subluxation
b. Thoracic outlet syndrome
c. Rotator cuff tendinitis
d. Bicipitalis tendinitis

660
e. Adhesive capsulitis

24. A 35-yo package-delivery truck driver as present with the insidious onset of right
buttock and posterior thigh pain that aggravated by sitting and using foot pedals.
Examination reveal external rotation of the right thigh shortening, but no weakness or
sensory or motoric loss. Treatment should include :
a. Pelvic rotation
b. A lumbosacral corset
c. A right shoe insert and heel lift
d. Infrared to the gluteal muscle
e. Stretching of the buttocks and hip muscles

25. When burn wound is around the perineum and both lower extremities, the prone
position at the following, EXCEPT :
a. Hip adduction
b. Knee extension
c. Ankle dorsiflexion
d. Pillow under the knees
e. Hip in neutral position

26. The following are the characteristic of Duchene’s Muscular Dystrophy, EXCEPT :
a. EMG shows high amplitude, long duration MUAP with decrease recruitment on
effort

27. What is the strongest single predictor of mortality in adults with pediatrics
onset disabilities?
a. Inability to walk
b. Feeding problems
c. Intellectual disability
d. Head circumference
e. Presence of epilepsy

661
28. The maneuver to detect hip dislocation in infant, where begin with hip flexed 900,
then hip is abducted and the examiner’s index finger gently pinched up on the
greater trochanter, is called :
a. Barlow maneuver
b. Galleazi maneuver
c. Ortolani maneuver
d. Hip click maneuver
e. Hip reduction maneuver

29. A 13 y.o girl is found to have scoliosis on routine physical examination.


Spine radiographis demonstrate a Cobb angle of 250. The optimal treatment is :
A. Internal surface electrical stimulation with trunk strengthening exercise
B. Internal surface electrical stimulation with bracing for 6 hours a day
C. Radiologic follow-up every 4 to 6 months
D. Surgical instrumentation and bony fusion
E. Use Milwaukee brace for 23 hours

30. A child 3 y.o has clinical sign, asymetrics face, shorthening of


strenocleidomastoideus muscle without tumor, deficit of head rotation > 300. The
appropriate management :
A. Deep heating
B. Stretching exercise
C. Asymmetris collar
D. Refer to orthopedic surgeon
E. Stimulation of the head movement

31. Pronator teres syndrome is caused by entrapment of the following nerve :


A. Ulnar nerve
B. Radial nerve
C. Median nerve
D. Anterior interosseus nerve
E. Posterior interosseus nerve

662
32. The best treatment of spasticity in SCI is :
A. Botox i.m
B. Bacloven i.v
C. Baclofen oral
D. Diazepam i.v
E. Baclofen intrathecal

33. Which symptom is characteristic of vascular claudication rather than neurogenic


claudication?
A. Spasm and pain hamstring muscle
B. Calf pain is evoked with uphill walking
C. Inclined treadmill walking decrease the pain
D. Thigh cramping is noted with prolonged standing
E. A change in position is needed to decrease pain

34. You got refferal a 60 y.o cancer patient with an acute deep vein thrombosis (DVT) in
the right upper limb , secondary to a long-standing central venous catheter. What
therapy restriction would you recommend for the patient?
A. No restriction
B. Bed rest for 10-12 days to allow for clot maturation
C. Begin restrictive exercise 12-24 hours after the patient is therapeutic on an
anticoagulant
D. Limit therapy to ambulation, balance and ADL training if anticoagulant is
medically contraindicated
E. No activity restriction, since upper limb DVT’s have a low likelihood of causing
a pulmonary embolus

35. A 6 y.o boy with hemophilia has suffered a recurrent hemarthrosis of the knee. After
providing adequate factor VIII replacement, the most important therapy in the first 48
hours is
A. Immobilization

663
B. Active assistive ROM exercise
C. Icing and passive ROM exercise
D. Ambulation with full weight bearing
E. Ambulation with partial weight bearing

36. Children with tetralogy of Fallot with assume the squatting position to relieve
exercise induced dyspnea :
A. Decrease heart rate
B. Increase inspiratory capacity
C. Decrease pulmonary artery pressure
D. Increase peripheral vascular resistance and there by decreases right to left shunt
E. Reduces the energy requirement of the acitivity by lowering the center of gravity

37. Acquired subluxation or dislocation of the hips in sastic cerebral palsy is usually
due to muscular imbalance and pull of the :
A. Knee extensor and hip abduction
B. Hip flexors and tensor fascia lata
C. Hip flexors and hip adductors
D. Hip extensor and hip abductors
E. Knee flexors and hip abductors

38. The following are the contraindicated physical activities in the osteoporotic
patients EXCEPT :
A. Golf swing
B. Running
C. Jumping
D. Walking
E. Jogging

39. A 29 y.o cross-country runner experience new-onset pain in the region of the first
metatarsal
Exam reveals tenderness to the first metatarsal head worsened by passive great toe

664
extension. The cause of her sesamoid injury include of the following, EXCEPT :
A. Pes planovalgus
B. Early hallux rigidus
C. Excessive pronation
D. Excessive supination
E. Gastric-soleus tightness

40. A 20 y.o football player reports anterior shoulder pain during a game. He completes
the game, but radiographs after the game revealed a type 2 acromioclavicular joint
sprain. How is a type 2 acromioclavicular joint injury defused? cc pg 193
A. Dislocation of coracoclavicular joint
B. Acromioclavicular and coracoclavicular ligament are both intact
C. Acromioclavicular and coracoclavicular ligament are both disrupted
D. Acromioclavicular ligament is disrupted, and coracoclavicular ligament is intact
E. Acromioclavicular ligament is intact, and coracoclavicular ligament is disrupted

41. A 35 y.o runner presents with pain in the region of the Achiles tendon. This the third
such episode over the past 4 years. The Achilles tendon appears swollen.
Appropriate initial management would include :
A. Surgery
B. Cortisone injection of the Achilles tendon
C. Cortisone injection of the Achilles tendon sheath
D. Use of a small heel lift with initiation of gastric -soleus stretches as tolerated
E. Immobilization in an ankle plantar-plexed position with limited weight bearing

42. The American Geriatrics Society’s goal of diabetic control in elderly includes:
A. Tight hypoglycemic control
B. Tight hemoglobin A1c below 7%
C. Fasting plasma glucose below 120mg/dl
D. Reduction of cardiovascular complication
E. Reduction of neuromuscular complications

665
43. In prescribing pain treatment for elderly patients, it is best to use which approach?
A. Start with NSAID
B. Start low and go slow for all medications
C. Start with aspirin, which is safe and effective
D. Avoid use of acetaminophen, due to liver toxicity
E. Use high dose, short acting narcotics initially to get pain under control

44. A 69 y.o woman wishes to reduce her risk of compressionfractures and is interested
in taking alendronate. You inform her that:
A. Alendronate increases BMD by increasing osteoblast activity
B. Gastrointestinal absorpsion of alendronate is facilitate by calcium
C. Although fracture risk is reduced with the useof alendronate, BMDis relatively
unaffected
D. Use of alendronate by women with existing vertebral fractures decreases the risk
of further fractures
E. The most common side effects associated with the use alendronate involve
the cardiovascular and neurologic systems

45. Which electrodiagnostic finding is associated with a poor prognosis in


Guillain-Barre Syndrome?
A. Compound MUAP less than 10% normal
B. Significant delayed conduction velocity
C. Absence of late responses F or H wave cc pg 432
D. Large fibrillation potentials
E. Nascent potential

46. The primary neurotransmitter of afferent pain neuron in the spinal cord appears to
be:
A. GABA
B. Endorphin
C. Enkephalin
D. Substance P

666
E. Beta lipoprotein

47. This disorder is usually seen in preadolescent athletes who participates in activities
such as jumping or running, the disorder is a result of recurring microtrauma from
the quadriceps contracting:
A. Pes anserinus
B. Ligament sprain
C. Pes anserinus bursitis
D. Tibial plateu fractures
E. Osgood-schlatter disease

48. The gluteus maximus gait:


A. The individual shifts the trunk over the affected side during stance phase
B. The trunk quickly shifts posteriorly at heel strike
C. Excessive hyperextension of the knee
D. The body leans over the left leg during that leg’s stance phase
E. The right side of the pelvis drops when the right leg leaves the ground and begin
swing phase

49. What will you find in an examination when a newborn baby has an erb’s paralysis?
A. The affected side of upper extremity is not active, moro (-), grasp reflex (-)
B. BIlateral of upper extremity are active, moro (-), grasp reflex (-)
C. The affected side of upper extremity is not active, moro (+), grasp reflex (-)
D. BIlateral of upper extremity are active, moro (+), grasp reflex (+)
E. The affected side of upper extremity is not active, moro (-), grasp reflex (+)

50. One week after the patien began a resistance/strengthening program, you note a 15%
increase in the force of their maximum voluntary contraction. This is most likely
secondary to a change in the...
a. Number of muscle fiber present
b. Motor unit recruitment pattern
c. Neuromuscular junction

667
d. Number of myofibrils
e. Fiber density

51. ........ The goals of long-term management for the patient with chronic bronchitis mya
include the following:
A. Reduce ventilation
B. Reduce airway secretion
C. Reduce secretion clearance
D. Reduce the work of breathing
E. Reduce perfusion matching and gas exchange

52. There is submaximal cardiorespiratory fitness testing:


A. Harvard step test
B. Cycle ergometer test
C. Treadmill test
D. Timed up and go test
E. Astrand test

53. Other changes in the cardiopulmonary system that result from exercise:
A. Decrease vital capacity
B. Decrease systolic blood pressure
C. Increased resting heart rate
D. Increased oxygen saturation
E. Increased cardiopulmonary efficiency

54. Which of the following parameters most closely correlates with oxygen consumption:
A. Oral temperature
B. Rectal temperature
C. Pulse rate
D. Respiratory rate
E. Blood pressure

668
55. Maximal heart rate that recommended for cardiac insufficient patients during
exercise:
A. 40-50% HR max
B. 50-60% HR max
C. 60-70% HR max
D. 70-85% HR max
E. >85% HR max

56. Hyperextended knee + complete foot contact is given what orthosis:


A. Hinged + ventral AFO
B. Rigid + dorsal AFO
C. Hinged + dorsal AFO
D. Rigid + ventral AFO

57. The primitive reflex(es) that coresponding with apedal reflextive development are :
A. Flexor with drawl, extensor thrust and optical righting
B. Crossed extention, ATNR and neck righting
C. Amphion reaction, flexor with drawl and extensor trust
D. Amphion reaction, crossed extention and flexor with drawl
E. ATNR, flexor with drawl and extensor trust

58. Oral motion skills in infant aged 4-6 months :


A. Phasic bite and cup drinking
B. Phasic bite and munch chew pattern
C. Phasic bite and rotary chew pattern
D. Phasic bite and dissociation of tounge blade and tongue tip
E. Phasic bite and some lip closure during chewing

59. Typical gait abnormalities on spastic diplegia cerebral patsy :


A. Scissoring gait pattern, hips flexed and adducted, knee flexed with valgus and
ankle in equinus
B. Scissoring gait pattern, hips flexed and adducted, hip hiking or hip circumduction

669
and upper extremity posturing
C. Scissoring gait pattern, hips flexed and adducted, ankle dorsoflexion, and
supinated foot in stance phase
D. Scissoring gait pattern, hips flexed and adducted, hip hiking or circumduction and
ankle in equinus
E. Scissoring gait pattern, hips flexed and adducted, knee flexed with valgus and
overactive posterior tibialis

60. A 4 year old boy is brought into your office because his mother has noticed that he
has difficulty getting up from a seated position on the floor while playing with his
toys. On physical exam, there is increased gastronemius calf circumference
bilaterally. You think the child may have dystrophic myopathy. The maneuver the
child performs to assist him in standing is caused by :
A. Proximal leg weakness
B. Distal leg weakness
C. Proximal arm weakness
D. Distal arm weakness
E. Proximal arm and distal leg weakness

61. Body-powered terminal devices (whether hooks or hands) in upper extremity


prosthetics can be voluntary-opening (VO) or voluntary-closing (VC) types. When
we compare both types, VO is :
a. Most common and practical type
b. More physiological function
c. Heavier
d. Less durable
e. Maximum prehensile force is determined by individual strength

62. A patient came with neck pain caused by traumatic accident on wheel. The
examination reveals soft tissue injury and cervical ligamentous sprain. You prescribe
a:
a. Soft cervical collar

670
b. Thomas collar / hard cervical collar
c. Philladelphia collar
d. Sterno-occipital mandibular immobilizer (SOMI)
e. Minerva cervico-thoracic orthosis

63. Charcot’s joint is a chronic progressive degenerative arthropathy with loss of


proprioception and pain sensation, leading to instability and joint destruction,
secondary to a:
a. Bone destruction
b. Metabolic disease
c. Sensory neuropathy
d. Amyloidosis
e. Sarcoidosis

64. Charcot’s joint characterized by, EXCEPT:


a. Osteophytes
b. Bony fragments
c. Subluxation
d. Periartikular debris
e. Proprioceptive disorder

65. Glenoid fossa articulates with ....... of the humeral head.


a. 30%
b. 40%
c. 50%
d. 60%
e. 70%

66. A 46 years old woman complaining about pain on her forefoot. The pain is a sharp
shooting forefoot pain radiating to the third and fourth digit. Dysesthesias and
numbness are present. From the examination: inflamation signs (-), tenderness in
the interdigit web space (+), Foot compression squeeze metatarsal heads together

671
is (+) painfull. this condition is caused by
a. Plantar fasciitis
b. Metatarsal osteoarthritis
c. Bunions
d. Neuropatic disorder
e. Morton’s neuroma

67. The programme you consider to such condition, EXCEPT:


a. Shoes with a soft sole and wide toe box
b. Accommodative padding: Metatarsal pad
c. Injection: Corticosteroid and lidocaine may be diagnostic and therapeutic
d. Referred to surgeon if indicated
e. Reffered to radiologic department to have radiograph imaging of the foot

68. 23 years old runner presents with the gradual onset of pain along the posteromedial
border of the tibia. Pain may decrease with exercise and then become more apparent
after the completion of the activity and last until the next morning. From the physical
examination: tenderness on palpation along the entire length of medial tibial border.
Plain films showed no abnormality. USG MSK imaging showed focal hyperechoic
elevation of the periosteum wuth irregularity over distal tibia and increased flow on
Doppler interrogation. You assumed the condition as:
a. Shin’s splint
b. Runner’s knee
c. Stress fracture
d. Acute compartment syndrome of the leg
e. Chronic exertional compartment syndrome

69. You treated the patient with, EXCEPT:


a. Resting, icing and stretching were done initially
b. Orthotics was prohibited in this condition as it will weakened lower limb muscles
c. Return to activity should be gradual and occurs when the patient is pain free for
several days

672
d. Training should start at 50% of preinjury level for intensity and distance
e. In training, soft level surfaces should be used initially

70. The Scoliosis research society define scoliosis as:


A. Any curve which is great or equal to 10 degrees with a rotatory component in
frontal plane
B. Any curve which is great or equal to 15 degrees with a rotatory component in
frontal plane
C. Any curve which is great or equal to 10 degrees with or without a rotatory
component in frontal plane
D. Any curve which is great or equal to 15 degrees with or without a rotatory
component in frontal plane
E. . Any curve which is great or equal to 15 degrees with or without a rotatory
component in sagital plane

71. The following included therapeutic heat, EXCEPT:


A. Radiant heating agent
B. SWD
C. US Diathermy
D. Ultraviolet radiation
E. Phonophoresis

72. Assessment of cognition in the elderly is done by assessment:


A. MMSE
B. Mild cognitive impairment
C. Geriatric Depression Scale
D. Geriatric Depression Scale and MMSE
E. Mild cognitive impairment and MMSE

73. Dimension of care in geriatri (kepotong T_T)


a. The safety of home environment and public environment
b. The safety of home environment and the adequacy of the patient’s access to

673
needed personal service and medical services
c. the adequacy of the patient’s access to needed personal service and medical
services
d. The safety of home environment and medical services
e. The safety of home environment and the adequacy of the patient’s access to
needed personal service

74. Goals of care in elderly patients are:


A. always, social and functional goals assume priority over health-related goals
B. Regaining a previous health status, attending a family event
C. completing ADLs with help and choosing an appropriate decision-maker
D. Regaining a previous health status and choosing an appropriate decision-maker
E. completing ADLs without help and choosing an appropriate decision-maker

75. The stretch reflex or myotactic reflex is an activity of:


A. Motor unit
B. Muscle spindle
C. Musculotendinous junction
D. Myoglobin
E. Muscle fiber

76. Wallenberg syndrome occurs due to occlusion of the following


1. Vertebral arteries
2. Posterior inferior cerebellar artery A
3. Superior lateral medullary artery
4. Posterior cerebral artery

77. Brain plasticity represents the capability of the damaged brain to repair itself by
means of morphologic and physiologic responses. Plasticity is influenced by
1. Complexity of stimulation
2. Environment E
3. Repetition of task

674
4. Motivation

78. Vesicoureteral reflux occurs in 17-25% of SCI patients. Predisposition to


vesicouretral reflux is high bladder pressure during fillingand voiding phase. The
following is included during filling phase:
1. Detrusor over-activity
2. Detrusor spinchter dyssynergia C
3. Low bladder compliance
4. High voiding pressure

79. The following are innervation charactheristics type I muscle fiber


1. Smaller cell body
2. Thinner diameter axon A
3. Slower twitch muscle fiber
4. Higher innervation ratio

80. Muscular receptors that provide proprioceptive information is/are:


1. Ruffini C
2. Golgi tendon organ
3. Paciniform
4. Muscle spindle

81. Pharyngeal phase of swallowing requires A


1. Soft palate elevation and velopharyngeal closure
2. Laryngeal elevation, with forward movement of the hyoid bone and folding of the
epiglottis
3. Adduction of the ventricular and true vocal fold
4. Cricopharyngeal muscle contraction

82. The function of upper limb movement that must be considered in the orthotic
prescription is/ are:
1. Reach

675
2. Carry E
3. Prehension pattern
4. Release

83. What requirement must the patient meet in order to use an AFO effectively?
1. Knee extension strength of 3/5 E
2. Stable limb size without fluctuating edema for use of a plastic AFO
3. Skin pressure tolerance and patient compliance with skin checks
4. Hip flexion strength of 3/5

84. What are the indication for use of an AFO to improve a patient’s gait?
1. Weak push-off at late stance phase E
2. “Foot drop” at heel strike due to weak ankle dorsiflexors
3. “Foot drop” passive plantarflexion in swing phase
4. Mediolateral instability at the ankle

85. In quite standing, the ground reaction force (GRF) are located in :
1. Posterior to knee
2. Anterior to ankle C
3. Anterior to hip
4. Posterior to hip

86. The probable cause (s) of steppage gait is (are) :


1. Plantarflexors spasticity
2. Equiaus deformity A
3. Severely weak dorsiflexor
4. Weak plantarflexors

87. Pelvic tilt exercise is of ten prescribed for LBP patient, because it produces :
1. Abdominal muscle strengthening A
2. Reflex inhibition of back musculature
3. Stretching of low back structures to prevent fixed excessive of lumbar lordosis

676
4. Permanent reduction of lumbar lordosis

88. William flexion exercise prescribed for patient, because it produces :


1. Abdominal muscle stretching
2. Hamstring stretching A
3. Gluteal muscle strengthening
4. Multifidius muscle strengthening

89. Which of the following exercise is/are indicated for coordination training :
1. De lonue exercise
2. Codman’s exercise B
3. Frenkel exercise
4. Klapp exercise

90. Which are emphasized to increase muscle strength?


1. More Repetition
2. More Resistance C
3. Low Ressitance
4. Fewer repetition

91. Nerve conduction studies finding in a preganglionic lesion of branchial plexophaty is


(are) :
1. Abnormal SNAP
2. Normal SNAP C
3. Normal CMAP
4. No motor respons

92. The purpose of exercise testing may to be assess any of the following :
1. Functional work capacity E
2. Possible presence and extend of coronary disease
3. Prognosis
4. Effect of therapeutic interventions

677
93. The Dosage of laser dependent on :
1. Output of the laser in mW A
2. The Time of exposure in second
3. The beam surface area of the laser in cm2
4. Output of the laser in watt

94. The type of electrical current :


1. Direct current
2. Alternating current A
3. Interferential current
4. Pulsed current or pulsative current

95. The effect (s) of cryotherapy :


1. Increase nerve ( sensory and motor) conduction velocity D
2. Decrease pain threshold
3. Increase rate of blood flow
4. Reduce muscle spasm by interrupting the pain –spasm-pain cycle

96. The following criteria usually indicate structural scoliosis :


1. Rotation in thoracic vertebrae A
2. Rib hump
3. While bending forward , spinal column deviation is not correted from the midline
4. Difference in the length of the lower limbs

97. In genu varum (bow legs) :


1. Charaterized by a grater than usual distance between the knees in an erect
position
2. Weight distribution on the tibia surfaces is imbalanced with the lateral aspect of
the knee bearing the main load
A
3. Often result of inverted foot position

678
4. The effectiveness of exercise in many cases of genu varus is good

98. Weakness in the shoulder girdle may effect ;


1. Faulty posture C
2. Difficulty in gross motor functioning such as shooting a ball at a basket
3. Functional difficulties in fine motor skill and in manipulative actions employing
the wrist and fingers in proximal – distal coordination pattern in motor
development
4. Difficulty of action such as writing, cutting and regulating the precise amounts of
strength applied in such action

99. Possible causes of toe- out :


1. Tibial torsion
2. Shortening of internal hip rotators B
3. Imbalance of hip joint
4. Torsion angle of hip bone > 12ᵒ

100. Patient with foot drop can caused by


1. Common peroneal
2. Diabetes diffuse polyneuropathy A
3. L4-L5 radiculopathy
4. Lumbar plexopathy

679
SOAL UTULOK 14 FEBRUARI 2020

1. Patient with weakness involving face, arm and leg, no sensory deficit, aphasia or
parietal sign can occur with lesion location in?
a. Anterior limb of internal capsule
b. Posterior limb of internal capsule
c. Junction of internal capsule and thalamus
d. Ventro-lateral thalamus
e. Thalamocortical projection

2. Patient may demontrate a abnormal “OK” sign or have difficulty forming a first
because of an inability to approximate the thumb and index finger is clinical
presentation?
a. Posterior interosseius nerve syndrome
b. Tardy ulnar nerve palsy
c. Cubitan tunnel syndrome
d. Anterior interosseus nerve syndrome
e. Pronator teres syndrome

3. Nerve control for coordinating bladder contraction and sphincter externa relaxation
in the voiding phase is?
a. Cerebral coretex
b. Pontin micturition center
c. Sacral micturition center
d. Pudendal nerve
e. Sympathetic nerve

4. The following included mononeuropathy multiplex is?


a. Guilla barre syndrome
b. Baxter neuropathy
c. Diabetic
d. Leprosy

680
e. Saturday night palsy

5. Type of muscular dystrophy that muscle usually affected first are hips and
shoulders, then progresses to the arms and legs, through progression is slow.
Usually begin in the teen or early adult years and autosomal recessive. This type
is?
a. Limb girdle muscular dystrophy
b. Facio scapulohumeral muscular dystrophy
c. Distal muscular dystrophy
d. Emery dreifuss muscular dystrophy
e. Myotonic dystrophy

6. The cerebellum consists of three functionally distrinct parts with different roles
concerned primarilly with subconscious control of motor activity. One of them is
spinocerebellum. The following function of spinocerebellum is?
a. Maintaining balance
b. Control eye movements
c. Enhances muscle tone and coordinates skilled voluntary movements
d. Planning voluntary activity
e. Initiating voluntary activity

7. Nerve and muscle cells do not come into direct contact at a neuromuscular
junction. The space or clift, between these two structures is too large for electrical
transmission of an impulse between them. Just as at neural synapse, a chemical
messenger carries the signal between terminal button and the muscle fiber. The
neurotransmitter is?
a. AchE
b. K
c. Na
d. Ca
e. Ach

681
8. Desending tracts which function as a facilitation of muscle tone is?
a. Lateral reticulospinal tract
b. Medial reticulospinal tract
c. Lateral corticospinal tract
d. Ventral spinocereberal tract
e. Lateral spinothalamic tract

9. Quadrilateral transfemoral socket?


a. Narrow mediolateral
b. The ischial tuberosity is contained inside the socket providing a bony lock
betweens ischium and greater trochanter
c. There are reliefs for abductor longus, hamstring, greater trochanter, gluteus
maximus and rectur femoris
d. Weight bearing is consentrated in the medial aspect of the the ischium and the
ischial ramus
e. Narrow mediolateral design was developed to provide a more normal anatomic
alignment of the femur insight the socket

10. During exercise, there is an increase in a person’s ?


a. Stroke volume
b. Diastolic pressure
c. Venous compliance
d. Pulmonary arterial resistance
e. Total peripheral resistance

11. An exercise stress test to rule out ischemic heart disease is positive if?
a. The systolic bood pressure rises
b. The ST segment of the ECG is depressed
c. The heart rate fails to increase
d. A diastolic murmur is heard
e. The diastolic blood pressure decreases

682
12. During aerobic exercise, blood flow remains relatively constant within?
a. The skin
b. The heart
c. The brain
d. The skeletal muscles
e. The kidneys

13. Which of the following conditions causes pulse pressure to increase?


a. Tachycardia
b. Hypertension
c. Hemorrhage
d. Aortic stenosis
e. Heart failure

14. An ectopic extrasystole caused by a ventricular focus is characterized by?


a. Interuption of the reguler SA node discharge
b. Retrograde conduction of the action potential to the atria
c. A skipped ventricular contraction
d. A skipped atrial contraction
e. A larger than normal force of contraction

15. Which muscle laterally rotates, extends and flexes the hip?
a. Glutens maximums
b. Glutens medius
c. Glutens minimums
d. Quadratus lumborum
e. Quadriceps femoris

16. The most active muscles at heel strikes of the ipsilateral limb is?
a. Sartorius
b. Semimembranosus
c. Rectus femoris

683
d. Gluteus minimus
e. Gluteus maximus

17. Motion of the scapula along the chest wall occurs through the action of the
muscles, EXCEPT?
a. Trapezius
b. Pectoralis minor
c. Pectoralis major
d. The rhomboids
e. None of the above is correct

18. The following statement is not typical characteristic of MUAP observed during
EMG test of patient with myopathy
a. Early recruitment
b. Short duraction
c. High amplitude
d. Low amplitude
e. None of the above

19. The H-reflex test of the median nerve recorded at the flexor carpi radial muscle
can be used to detected?
a. C5-C6 radiculopathy
b. C6-C7 radiculopathy
c. C7-T1 radiculopathy
d. C5 radiculopathy
e. C6 radiculopathy

20. The presence of nascent potential in EMG examination shows the following
process of?
a. Good prognosis
b. Poor prognosis
c. Reinnervation

684
d. Partial denervation
e. Total denervation

21. The MUAP, observed during electromyography examination of patient with


motor neuron disease EXECPT :
a. Long duration
b. High amplitude
c. Reduced motor unit recruitment
d. Low amplitude
e. Is correct none of the above

22. Vital capacity and residual capacity are …


a. Tidal volume
b. Residual volume
c. Total lung capacity
d. Functional capacity
e. Functional residual capacity

23. Based on a comprehensive patient assessment, the goals of long-term


management for the patient with chronic bronchitis may include the following is

a. Reduce ventilation
b. Reduce alveolar ventilation
c. Reduce secretion clearance
d. Reduce the work breathing
e. Reduce perfusion matching and gas exchange

24. Physiological changes when changing from the upright to the supine position is

a. Increased vital capacity
b. Increased residual volume
c. Increased airway resistance

685
d. Increased total lung capacity
e. Increased arterial oxygen levels

25. The termal effects of ultrasound diathermy :


a. Increase tissue temperature, decrease metabolic rate, decrease pain, increase
circulation
b. Increase tissue temperature, decrease pain, increase circulation, increase
metabolic rate
c. Increase tissue temperature, decrease metabolic rate, decrease circulation,
decrease pain
d. Increase tissue temperature, increase metabolic rate, decrease circulation,
decrease pain
e. Increase tissue temperature, increase metabolic rate, increase circulation, decrease
soft tissue extensibility

26. Comparing the effect of flexion and extension exercise on spine, it was
demonstrated that :
a. Patient with osteoporosis who performed back extension exercise had a
considerably lower rate of fracture than those who performed spinal flexion
exercise or no exercise
b. Patient with osteoporosis who performed spinal flexion exercise had a
considerably lower rate of fracture than those who performed back extension
exercise
c. There is no difference result for back extension exercise and flexion exercise
d. Patient doesn’t need exercise in osteoporosis because of risk of fracture
e. Weakness in abdominal muscles adds to the problem of poor posture and
protruded abdomen so the patients need flexion back exercise

27. Where is the lesion site in ataxic dysarthria, as found in Friedreich’s ataxia?
a. Cerebellum
b. Multiple sites
c. Basal ganglia

686
d. Extrapyramidal system
e. Bilateral upper motor neuron

28. An infract in the lower division of the left middle cerebral artery division would
be associated with which type of aphasia?
a. Broca
b. Global
c. Wernicke
d. Transcortical motoric
e. Transcortical sensoric

29. The following muscles are usually affected by a spinal groove lesion, EXCEPT
for the :
a. Triceps
b. Brachioradialis
c. Extensor carpi radialis
d. Extensor carpi ulnaris
e. Extensor digitorum communis

30. Which type of stroke has the best outcome?


a. Haemorrhagic pontine
b. Internal capsular lacunar
c. Embolic left cortical infraction
d. Rupture of anterior cerebral artery aneurysm
e. Ischemic occlusion of the right internal carotid artery

31. Treatment of acquired dysphagia in 67-yo man with a left hemiparesis might include :
a. Chin tucking and reclining feeding
b. Chin tucking and head turning to the affected side
c. Changing diet to one of softer and thinner consistency
d. Upright feeding and head turning to the unaffected side
e. Use of long-term thermal stimulation and upright feeding

687
32. Using a computer mouse that is too far to the side or too high in the table will lead
to :
a. Anterior shoulder subluxation
b. Thoracic outlet syndrome
c. Rotator cuff tendinitis
d. Bicipitalis tendinitis
e. Adhesive capsulitis

33. A 35-yo package-delivery truck driver as present with the insidious onset of right
buttock and posterior thigh pain that aggravated by sitting and using foot pedals.
Examination reveal external rotation of the right thigh shortening, but no weakness or
sensory or motoric loss. Treatment should include :
a. Pelvic rotation
b. A lumbosacral corset
c. A right shoe insert and heel lift
d. Infrared to the gluteal muscle
e. Stretching of the buttocks and hip muscles

34. When burn wound is around the perineum and both lower extremities, the prone
position at the following, EXCEPT :
a. Hip adduction
b. Knee extension
c. Ankle dorsiflexion
d. Pillow under the knees
e. Hip in neutral position

35. The most commonly employed diagnostic maneuvers for TOS are, EXCEPT :
a. Adson test
b. Yergason test
c. Hyperabduction test
d. Shoulder depression test

688
e. Three minutes arm elevation

36. The following are the characteristic of Duchene’s Muscular Dystrophy, EXCEPT :
a. EMG shows high amplitude, long duration MUAP with decrease recruitment on
effort
Opsi b – e kepotong

37. What is the strongest single predictor of mortality in adults with pediatrics onset
disabilities?
a. Inability to walk
b. Feeding problems
c. Intellectual disability
d. Head circumference
e. Presence of epilepsy

38. The maneuver to detect hip dislocation in infant, where begin with hip flexed 900,
then hip is abducted and the examiner’s index finger gently pinched up on the greater
trochanter, is called :
a. Barlow maneuver
b. Galleazi maneuver
c. Ortolani maneuver
d. Hip click maneuver
e. Hip reduction maneuver

39. Which statement below is correct about spastic diplegia?


a. Involves all four extremities
b. Have good walking prognosis
c. Significant mental retardation
d. Significant oromotor dysfunction
e. Involves upper extremities more than lower

40. The period which intense motor learning and basic language development occur is

689
at :
a. Birth to sixth months
b. Birth to tenth months
c. Birth to one year of age
d. Birth to two year of age
e. Birth to three year of age

41. 41. A 13 y.o girl is found to have scoliosis on routine physical examination. Spine
radiographis demonstrate a Cobb angle of 250. The optimal treatment is :
A. Internal surface electrical stimulation with trunk strengthening exercise
B. Internal surface electrical stimulation with bracing for 6 hours a day
C. Radiologic follow-up every 4 to 6 months
D. Surgical instrumentation and bony fusion
E. Use Milwaukee brace for 23 hours

42. A child 3 y.o has clinical sign, asymetrics face, shorthening of


strenocleidomastoideus muscle without tumor, deficit of head rotation > 300. The
appropriate management :
A. Deep heating
B. Stretching exercise
C. Asymmetris collar
D. Refer to orthopedic surgeon
E. Stimulation of the head movement

43. The following nerve is a branch of the posterior cord of the brachial plexus :
A. Long scapular nerve
B. Subscapular nerve
C. Suprascapular nerve
D. Levator scapular nerve
E. Musculocutaneous nerve

690
44. Pronator teres syndrome is caused by entrapment of the following nerve :
A. Ulnar nerve
B. Radial nerve
C. Median nerve
D. Anterior interosseus nerve
E. Posterior interosseus nerve

45. The best treatment of spasticity in SCI is :


A. Botox i.m
B. Bacloven i.v
C. Baclofen oral
D. Diazepam i.v
E. Baclofen intrathecal

46. Which symptom is characteristic of vascular claudication rather than neurogenic


claudication?
A. Spam and pain hamstring muscle
B. Calf pain is evoked with uphill walking
C. Inclined treadmill walking decrease the pain
D. Thigh cramping is noted with prolonged standing
E. A change in position is needed to decrease pain

47. The clinical sign of COPD patients as below :


A. Exertional dyspnea often occurs when the FEV, is less than 1500 ml
B. Arterial blood gases may normal during exercise
C. FEV, FVC is more than 70% predicted normal
D. TLC are generally decreased
E. Residual volume decreased

48. You got refferal a 60 y.o cancer patient with an acute deep vein thrombosis (DVT) in
the right upper limb , secondary to a long-standing central venous catheter. What
therapy restriction would you recommend for the patient?

691
A. No restriction
B. Bed rest for 10-12 days to allow for clot maturation
C. Begin restrictive exercise 12-24 hours after the patient is therapeutic on an
anticoagulant
D. Limit therapy to ambulation, balance and ADL training if anticoagulant is
medically contraindicated
E. No activity restriction, since upper limb DVT’s have a low likelihood of causing a
pulmonary embolus

49. A 6 y.o boy with hemophilia has suffered a recurrent hemarthrosis of the knee. After
providing adequate factor VIII replacement, the most important therapy in the first 48
hours is :
A. Immobilization
B. Active assistive ROM exercise
C. Icing and passive ROM exercise
D. Ambulation with full weight bearing
E. Ambulation with partial weight bearing

50. Children with tetralogy of Fallot with assume the squatting position to relieve
exercise induced dyspnea :
A. Decrease heart rate
B. Increase inspiratory capacity
C. Decrease pulmonary artery pressure
D. Increase peripheral vascular resistance and there by decreases right to left shunt
E. Reduces the energy requirement of the acitivity by lowering the center of gravity

51. A child with s congenital transverse radial limb deficiency should have in initiation
prosthesis fit at what developmental age?
A. At the time of first sitting independently
B. At the time of starting kindergarten
C. As soon as possible after birth
D. At the time of initially walking

692
E. At the time od initially standing

52. Acquired subluxation or dislocation of the hips in sastic cerebral palsy is usually due
to muscular imbalance and pull of the :
A. Knee extensor and hip abduction
B. Hip flexors and tensor fascia lata
C. Hip flexors and hip adductors
D. Hip extensor and hip abductors
E. Knee flexors and hip abductors

53. Physical examination findings that are consistent with hypotension in aterm infant
include all of the following EXCEPT :
A. Head lag when pulled to sitting position
B. Decrease spontaneous movements
C. Frog leg position of the legs
D. Episthotonus
E. Weak suck

54. A 65 y.o woman with breast cancer has a solitary metastatic lesion at the left rib eage.
Among the following, the first-line pain medication in this setting in :
A. Ibuprofen
B. Amitriptyline
C. Acetaminophen
D. Morphin sulfate
E. Local corticosteroid injection

55. The following are the contraindicated physical activities in the osteoporotic patients
EXCEPT :
A. Golf swing
B. Running
C. Jumping
D. Walking

693
E. Jogging

56. The following are the physiologic changes in the aging prosess, EXCEPT :
A. Lower flexibility
B. Higher vital capacity
C. Lower reaction time
D. Higher residual volume
E. Higher resting blood pressure

57. For patient with Congestive Heart Disease, which of the following statements in
accurate?
A. Most of the important from regular exercise is within the myocardian
B. Exercise capacity is improved because of peripheral adaptations
C. These patient can never expect improved physical fitness
D. Strengthening exercise is prescribed for these patients
E. Complete bed rest is prescribed for these patients

58. A 29 y.o cross-country runner experience new-onset pain in the region of the first
metatarsal. Exam reveals tenderness to the first metatarsal head worsened by passive
great toe extension. The cause of her sesamoid injury include of the following,
EXCEPT :
A. Pes planovalgus
B. Early hallux rigidus
C. Excessive pronation
D. Excessive supination
E. Gastric-soleus tightness

59. A 20 y.o football player reports anterior shoulder pain during a game. He completes
the game, but radiographs after the game revealed a type 2 acromioclavicular joint
sprain. How is a type 2 acromioclavicular joint injury defused?
A. Dislocation of coracoclavicular joint
B. Acromioclavicular and coracoclavicular ligament are both intact

694
C. Acromioclavicular and coracoclavicular ligament are both disrupted
D. Acromioclavicular ligament is disrupted, and coracoclavicular ligament is intact
E. Acromioclavicular ligament is intact, and coracoclavicular ligament is disrupted

60. A 35 y.o runner presents with pain in the region of the Achiles tendon. This the third
such episode over the past 4 years. The Achilles tendon appears swollen. Appropriate
initial management would include :
A. Surgery
B. Cortisone injection of the Achilles tendon
C. Cortisone injection of the Achilles tendon sheath
D. Use of a small heel lift with initiation of gastric -soleus stretches as tolerated
E. Immobilization in an ankle plantar-plexed position with limited weight bearing

61. The American Geriatrics Society’s goal of diabetic control in elderly includes:
A. Tight hypoglycemic control
B. Tight hemoglobin A1c below 7%
C. Fasting plasma glucose below 120mg/dl
D. Reduction of cardiovascular complication
E. Reduction of neuromuscular complications

62. In prescribing pain treatment for elderly patients, it is best to use which approach?
A. Start with NSAID
B. Start low and go slow for all medications
C. Start with aspirin, which is safe and effective
D. Avoid use of acetaminophen, due to liver toxicity
E. Use high dose, short acting narcotics initially to get pain under control

63. A 69 y.o woman wishes to reduce her risk of compressionfractures and is interested
in taking alendronate. You inform her that:
A. Alendronate increases BMD by increasing osteoblast activity
B. Gastrointestinal absorpsion of alendronate is facilitate by calcium
C. Although fracture risk is reduced with the useof alendronate, BMDis relatively

695
unaffected
D. Use of alendronate by women with existing vertebral fractures decreases the risk
of further fractures
E. The most common side effects associated with the use alendronate involve the
cardiovascular and neurologic systems

64. Which electrodiagnostic finding is associated with a poor prognosis in Guillain-Barre


Syndrome?
A. Compound MUAP less than 10% normal
B. Significant delayed conduction velocity
C. Absence of late responses F or H wave
D. Large fibrillation potentials
E. Nascent potential

65. Typical median nerve conduction changes in patients with mild carpal tunnel
syndrome show:
A. Prolonged sensory distal latency
B. Prolonged motor distal latency
C. Low amplitude motor response
D. Abnormal F wave
E. Abnormal H wave

66. The primary neurotransmitter of afferent pain neuron in the spinal cord appears to be:
A. GABA
B. Endorphin
C. Enkephalin
D. Substance P
E. Beta lipoprotein

67. This disorder is usually seen in preadolescent athletes who participates in activities
such as jumping or running, the disorder is a result of recurring microtrauma from the
quadriceps contracting:

696
A. Pes anserinus
B. Ligament sprain
C. Pes anserinus bursitis
D. Tibial plateu fractures
E. Osgood-schlatter disease

68. The gluteus maximus gait:


A. The individual shifts the trunk over the affected side during stance phase
B. The trunk quickly shifts posteriorly at heel strike
C. Excessive hyperextension of the knee
D. The body leans over the left leg during that leg’s stance phase
E. The right side of the pelvis drops when the right leg leaves the ground and begin
swing phase

69. Eccentric contraction:


A. Muscle attachments move farther apart, it’s an acceleration activity
B. Muscle attachments move closer together, it,s an acceleration activity
C. Muscle attachments move farther apart, movement is usually occurring against gravity
D. Muscle attachments move farther apart, the contraction is used with a
decelaration activity
E. Muscle attachments move closer together, the contraction is used with a deceleration
activity

70. Acupunture like TENS:


A. TENS with long duration, high amplitude also called high rate TENS
B. TENS with short duration, high amplitude also called low rate TENS
C. TENS with long duration, high amplitude pulse also called low rate TENS
D. TENS with short duration, short amplitude also called high rate TENS

71. Wallenberg syndrome occurs due to occlusion of the following


1. Vertebral arteries
2. Posterior inferior cerebellar artery

697
3. Superior lateral medullary artery
4. Posterior cerebral artery

72. Brain plasticity represents the capability of the damaged brain to repair itself by
means of morphologic and physiologic responses. Plasticity is influenced by
1. Complexity of stimulation
2. Environment
3. Repetition of task
4. Motivation

73. Patient with foot drop can caused by


1. Common peroneal
2. Diabetes diffuse polyneuropathy
3. L4-L5 radiculopathy
4. Lumbar plexopathy

74. Vesicoureteral reflux occurs in 17-25% of SCI patients. Predisposition to


vesicouretral reflux is high bladder pressure during fillingand voiding phase. The
following is included during filling phase:
1. Detrusor over-activity
2. Detrusor spinchter dyssynergia
3. Low bladder compliance
4. High voiding pressure

75. The following are innervation charactheristics type I muscle fiber


1. Smaller cell body
2. Thinner diameter axon
3. Slower twitch muscle fiber
4. Higher innervation ratio

76. Muscular receptors that provide proprioceptive information is/are:


1. Ruffini

698
2. Golgi tendon organ
3. Paciniform
4. Muscle spindle

77. Pharyngeal phase of swallowing requires


1. Soft palate elevation and velopharyngeal closure
2. Laryngeal elevation, with forward movement of the hyoid bone and folding of the
epiglottis
3. Adduction of the ventricular and true vocal fold
4. Cricopharyngeal muscle contraction

78. The function of upper limb movement that must be considered in the orthotic
prescription is/are:
1. Reach
2. Carry
3. Prehension pattern
4. Release

79. What requirement must the patient meet in order to use an AFO effectively?
1. Knee extension strength of 3/5
2. Stable limb size without fluctuating edema for use of a plastic AFO
3. Skin pressure tolerance and patient compliance with skin checks
4. Hip flexion strength of 3/5

80. What are the indication for use of an AFO to improve a patient’s gait?
1. Weak push-off at late stance phase
2. “Foot drop” at heel strike due to weak ankle dorsiflexors
3. “Foot drop” passive plantarflexion in swing phase
4. Mediolateral instability at the ankle

81. In quite standing, the ground reaction force (GRF) are located in :
1. Posterior to knee

699
2. Anterior to ankle
3. Anterior to hip
4. Posterior to hip

82. The probable cause (s) of steppage gait is (are) :


1. Plantarflexors spasticity
2. Equiaus deformity
3. Severely weak dorsiflexor
4. Weak plantarflexors

83. The prime mover (s) of the eversion ankle :


1. Plantaris
2. Peroneus longus
3. Peroneus tertius
4. Peroneus brevis

84. Pelvic tilt exercise is of ten prescribed for LBP patient, because it produces :
1. Abdominal muscle strengthening
2. Reflex inhibition of back musculature
3. Stretching of low back structures to prevent fixed excessive of lumbar
lordosis
4. Permanent reduction of lumbar lordosis

85. William flexion exercise prescribed for patient, because it produces :


1. Abdominal muscle stretching
2. Hamstring stretching
3. Gluteal muscle strengthening
4. Multifidius muscle strengthening

86. Which of the following exercise is/are indicated for coordination training :
1. De lonue exercise
2. Codman’s exercise

700
3. Frenkel exercise
4. Klapp exercise

87. Which are emphasized to increase muscle strength?


1. More Repetition
2. More Resistance
3. Low Ressitance
4. Fewer repetition

88. A patient was involved in a motorcycle accident, and it is suspected that he may
have axillary nerve palsy. Which muscle (s) to be checked electrophy sologically ?
1. Rhomboid
2. Teres minor
3. Teres mayor
4. Deltoid

89. Nerve conduction studies finding in a preganglionic lesion of branchial plexophaty is


(are) :
1. Abnormal SNAP
2. Normal SNAP
3. Normal CMAP
4. No motor respons

90. The purpose of exercise testing may to be assess any of the following :
1. Functional work capacity
2. Possible presence and extend of coronary disease
3. Prognosis
4. Effect of therapeutic interventions

91. A variety of walk test are available is/are :


1. Time based tests
2. Fixed-distance test

701
3. Velocity determined walk test
4. Controlled – pacing incremental test

92. The Dosage of laser dependent on :


1. Output of the laser in mW
2. The Time of exposure in second
3. The beam surface area of the laser in cm2
4. Output of the laser in watt

93. The type of electrical current :


1. Direct current
2. Alternating current
3. Interferential current
4. Pulsed current or pulsative current

94. The effect (s) of cryotherapy :


1. Increase nerve ( sensory and motor) conduction velocity
2. Decrease pain threshold
3. Increase rate of blood flow
4. Reduce muscle spasm by interrupting the pain –spasm-pain cycle

95. The following criteria usually indicate structural scoliosis :


1. Rotation in thoracic vertebrae
2. Rib hump
3. While bending forward , spinal column deviation is not correted from the
midline
4. Difference in the length of the lower limbs

96. In genu varum (bow legs) :


1. Charaterized by a grater than usual distance between the knees in an erect
position
2. Weight distribution on the tibia surfaces is imbalanced with the lateral aspect

702
of the knee bearing the main load
3. Often result of inverted foot position
4. The effectiveness of exercise in many cases of genu varus is good

97. Weakness in the shoulder girdle may effect ;


1. Faulty posture
2. Difficulty in gross motor functioning such as shooting a ball at a basket
3. Functional difficulties in fine motor skill and in manipulative actions employing
the wrist and fingers in proximal – distal coordination pattern in motor development
4. Difficulty of action such as writing, cutting and regulating the precise
amounts of strength applied in such action

98. Possible causes of toe- out :


1. Tibial torsion
2. Shortening of internal hip rotators
3. Imbalance of hip joint
4. Torsion angle of hip bone > 12ᵒ

99. Possible cause of lordosis :


1. Shortening of the muscles that tilt the pelvis anteriorly
2. Weakness of the muscles that tilt the pelvis posteriorly
3. Shortening of the ligament and the fascia covering the posterior surface of the
waist
4. Injury of the lumbar vertebrae cause spondylolisthesis

100. In hip and knee Osteoarthritis , strategies aimed at reducing loads on joint include :
1. Weight loss
2. Cushioned shoes
3, Walking aids
4. Activity modification.

703
UTULOK MARET 2019

24. The volume of gas that remains in the lung sat the end of a maximum expiration is....
A. Tidal volume
B. Residual volume
C. Total lung capacity
D. Functional capacity
E. Functional residual capacity

25. Patients are comfortable at rest,but ordinary physical activity results is dyspnea,
fatique, palpitations, pain,or angina, include NYHA....
A. I
B. II
C. III
D. IV
E. V

26. Vital capacity and residual capacity are....


A. Tidal volume
B. Residual volume
C. Total lung capacity
D. Functional capacity
E. Functional residual capacity

27. Based on a comprehensive patient asessment, the goals of long-term management for
the patient with chronic bronchitis may include the following is....
A. Reduce ventilation
B. Reduce alveolar ventilation
C. Reduce secretion clearance
D. Reduce the work breathing
E. Reduce perfusion matching and gas exchange

28. Nerve control for coordinating bladder contraction and sphincter externarel axation
in the voiding phase is....
A. Cerebral coretex
B. Pontin micturition center
C. Sacral micturition center
D. Pudendal nerve
E. Stmpathetic nerve

29. The process of propagating acurrent from on enode to another is known as....
A. Nodes of Ranvier
B. Saltatory conduction
C. Conduction block
D. Ortodromic
E. Antidromic

704
30. Grade II osteoarthritis based on kelgreen & lawrance :
A. No radiographic features of are present
B. Definite osteophytes and possible joint space narrowing on
anteroposterior weight-bearing radiograph
C. Doubtful joint space narrowing and possible ostephyticlipping
D. Multiple osteophytes,definite joint space narrowing, sclerosis, possible bony
deformity
E. Large osteophytes, severe sclerosis, marked joint space narrowing and definite
bony deformity

31. The primitive reflex(es) that corresponding with a pedal reflextive development are:
A. Flexor withdrawl,extensor thrust and optical righting
B. Crossed extention, ATNR and neck righting
C. Amphion reaction,flexor withdrawl and extensor trust
D. Amphion reaction,crossed extention and flexor withdrawl
E. ATNR, flexor withdrawl and extensor trust

32. Type of muscular dystrophy that muscles usually affected first are hips and
shoulders, then progresses to the arms and legs,though progression is slow. Usually
begin in the teen or early adult years and autosomal recessive.This type is....
A. Limb girdle muscular dystrophy
B. Facioscapulohumeral muscular dystrophy
C. Distal muscular dystrohy
D. Emerydreifuss muscular dystrphy
E. Myotonic dystrophy

33. Gross motor function classification system (GMFCS) level III:


A. Walks without restrictions limitation in more advanced gross motion skills
B. Walks without assistive devices: limitation walking outdoors and in the
community
C. Self mobility with limitation : children are transported or use power mobility
outdoors and in the community
D. Walks with assistive mobility devices : limitations walking outdoors
and in the community
E. Self mobility is severely limited even with the use of assistive technology

34. Typical gait abnormalities on spastic diplegia cerebral palsy:


A. Scissoring gait pattern,hips flexed and adducted, knee flexed with valgus
and ankle in equinus
B. Scissoring gait pattern,hips flexed and adducted,hip hiking or hip circumduction
and upper extremity posturing
C. Scissoring gait pattern, hips flexed and adducted, ankle dorsoflexion, and
supinated foot instance phase
D. Scissoring gait pattern, hips flexed and adducted ,hip hiking or circumduction
and ankle in equinus
E. Scissoring gait pattern, hips flexed and adducted, knee flexed with valgus and
overactive posterior tibialis

705
35. The cardinal sign of down syndrome:
A. Hypotonus,joint laxity,simian crease,obesity,clynodactily
B. Hypotonus,joint laxity,simian crease,obesity,polydactily
C. Hypotonus,joint laxity,simian crease,clynodactily,knee valgus
D. Hypotonus,joint laxity,simian crease,obesity,saddle nose
E. Hypotonus,joint laxity, simiancrease,clynodactily, saddle nose

36. Oral motion skills in infant aged 4-6 months:


A. Phasic bite and cup drinking
B. Phasic bite and munch chew pattern
C. Phasic bite and rotary chew pattern
D. Phasic bite and dissociation of tounge blade and tongue tip
E. Phasic bite and some lip closure during chewing

37. The first energy store house tapped at the onset of contractile activity is....
A. Glycolysis
B. Creatine phosphate
C. Oxidative phosphorylation
D. Lactate
E. Fat

38. Fata 60-year old man without any significant past medical history present to your
outpatient office with asymmetrica trophy, weakness and fasciculations. He also
complains of some difficulty swallowing his meals and complains of a strained and
strangled quality in his speech. He describes normal bowel and bladder function.
Which of the following is most likely hi diagnosis?
a. Amyotrophic lateral sclerosis (ALS)
b. Spinal muscle atrophy III
c. Primary lateral sclerosis (PLS)
d. Poliomyelitis
e. Spinal muscle trophy IV

39. A 4 year old boy is brought into your office because his mother has noticed that he
has difficulty getting up from a seated position on the floor while playing with his
toys. On physical exam, there is increased gastronemius calf circumference
bilaterally.You think the child may have dystrophic myopathy.The maneuver the child
performs to assist him in standing is caused by:
A. Proximal leg weakness
B. Distal leg weakness
C. Proximal arm weakness
D. Distal arm weakness
E. Proximal arm and distal leg weakness

40. A patient present to the emergency department with ptosis, anhidrosis and miosis on
the left side of his face. He has difficulty walking and recently fell to his left side. On
706
physical exam, there is decrease in pain and temperature sensation on the left side of
his face and on the right side of his body.There is no muscle weakness. He most likely
has:
A. Lateral medullary syndrome
B. Medial medullary syndrome
C. Benedikt syndrome
D. Weber syndrome
E. Pusher syndrome

41. Which type of aphasia would you expect to see inpatient who is noted to have
fluent speech,with signs of comprehension,but unable to repeat?
A. Anomicaphasia
B. Transcortical sensory aphasia
C. Conduction aphasia
D. Transcortical motor aphasia
E. Broca aphasia
42. Locked-insyndrome (tetraparesis in a completely conscious patient with only the
ability to move eyes vertically and blink) is suggestive of an occlusion in which
artery(ies) ?
A. Vertebral artery
B. Posteriorinferiorcerebralartery
C. Posteriorcerebralartery
D. Bilateral basilar arteries
E. Vertebral and basilar arteries

43. You are performing an EMG/NC Soft upper extremities to evaluate for carpal
tunnel syndrome and notice that the sensory and motor, median and ulnar latencies
are delayed. Your next steps should be:
A. Test radial sensory nerve
B. Needle testing of bilateral abductor pollicis brevis (APB) muscles
C. Test tibial motor nerve
D. Test the sural nerve
E. Test the peroneal motor nerve

44. How does limb temperature cooling affect electrodiagnostic f indings ?


A. No change in conduction velocity,decreased amplitude
B. No change in conduction velocity,no change in amplitude
C. Decreased conduction velocity, increased ampitude
D. Increased conduction velocity,decreased amplitude
E. Decreased conduction velocity,decreased amplitude

45. Axonal damage (with Wallerian degeneration) would present ith:


A. Decreased compound motor action potential (CMAP) amplitude with
proximal stimulation and distal stimulation
B. Decreased CMAP amplitude with proximal stimulation but not distal stimulation
C. Decreased CMAP amplitude distally but not proximality
D. Slowing of conduction velocity across the lesion

707
E. Slowing of conduction velocity distal to the lesion

46.In a normal adult,from what muscle can an H-reflex be obtained


A. Hamstring
B. Flexor carpiradialis
C. Biceps
D. Extensordigitorum
E. Extensorcarpi radialis

47.When is it most appropriate to perform F-waves?


A. For the evaluation of radiculopathy
B. For the evaluation of peroneal neuropathy at the fibular head
C. For the evaluation of possible acute inflammatory demyelinating
polyneuropathy (AIDP)
D. For the evaluation of peripheral neuropathy
E. For the evaluation of polyneuropathy

48. An 83 year old man with COPD, Ischaemic Heart disease and multiple falls. He
attends Emergency Dept following of all;he has a cough productive of green
sputum, a temperature of 37.4 and pulse 100 beats per minute,saturations 90% on
room air, BP138/78. He lives alone and has no home care. The patient tells you he
wants to go home; his daughter (who lives 120 miles away)is distressed and says he
cannot manage at home and must be admited.Which is the most appropriate
management plan? (ga tau jawabannya apax D)
A. Admit to an acute un it for medical treatment : he has evidence of an acute illness
B. Admit to an Intermediatecaebed :he needs a place of safety to assess his needs
C. Home with antibiotics and analgesia:his wishes over ride his daughters concerns
D. Home with early follow up in the falls clinic and referral to the community
based COPD team
E. Multidisciplinary assessment ED to assess his needs

49. The gold standards frame work for palliative care encourages doctors to ask the
question “Would you be surprised if this patient died in the next 6 months?”If the
doctor is unsure it suggest looking for clinical end of life and diseaserelated
indicators.Regarding dementia the diseaserelated indicators do NOT include:
A. Onset of dementia five or more years ago
B. Recurrent febrile episodes or infections
C. Unable to dress,walk or eat without assistance
D. Urinary and faecal incontinence
E. Worsening eating problems,needing puréed/soft dictor supplements

50. An 89 year old woman with vascular dementia is admitted with facial bruising and
suspected head injury following afall at home. She lived with her husband and
daughter (are tired social worker with lasting power of at torney for her parents) an
disdependent for all activities of daily living. Her devoted husband came in every
day to help feed her. However the ward sister observed the patient's husband slap her
on the face when she refused toe at the hospital food he offered.The next most
708
appropriate step should be:(ga tau jawabannya apa x D)
A. An Independent Mental Capacity Advocate should be consulted
B. A Protection of Older Vulnerable Adult investigation should be instigated
C. The husband's behavior should be immediately challenged by the ward doctor
D. The patient's daughters hould be called and advised of what occurred
E. The polices hould be called to investigate an alleged witnesse dassault

51. An 86 year old woman was found on the bathroom floor by one of her careers. An
ambulance was called and she was sent to the local hospital Emergency Department
(ED) for an assessment. Her past medical history included hypertension, recurrent all
sand memory problems". A comprehensive history and physical examination
demonstrated no acute organic illness or injury. The patient was independently
mobile with her zimmer frame in ED which her daughter confirmed was her usual
level of mobility.The patient had decided that she wanted to go home but the nurse
incharge of ED had concerns about her safety on returning home,particularly in light
of her memory problems and falls.The patient was becoming more agitated at not
being allowed to go home. The next most appropriate test pin her management
should be:
A. CT brain scan
B. Daughter's views as certained
C. Formal Cognitive Assessment
D. Physiotherapy assessment
E. Urgent Deprivation of Liberty Safe guarding (DOLS) application

52. When defining frailty using the Fried model the following criteria should be
considered:
A. Polypharmacy
B. Sensory impairment (sight or hearing)
C. Timed get up and go test
D. Two or more falls within the last year
E. Unintentional weight loss

53. An 69 year old male with type 2 diabetes and high BMI.Presents with 24 hour
history of persistent numbness affecting hisright side of the body including face. He
is now complaining of pain affecting his right arm.On examination he has no focal
motor weakness. What is the most likely location of pathology?
A. Brainstem
B. Left frontal lobe
C. Left thalamus
D. Peripheral neuropathy
E. Spinal cord

709
54. An 80 year old woman was admitted to hospital with breathlessness. Past medical
history of hypertension,COPD and myocardial infarction. Inpatient echocardiography
showed left ventricular systolic dysfunction with ejection fraction (EF) of 36% with
no significant valvular lesions. The patient responded well to acute heart failure
treatment.On discharge which treatment strategy is most relevant int his case?
A. Beta blocker, ACE inhibitor and Mineral corticoid Receptor Agent(MRA)
B. Beta blocker, an Angiotensin receptor blocker/Neprilysin inhibitor and MRA
C. Beta-blocker should be avoided because of her COPD.
D. MRA should be avoided as EF is more than 35%
E. Referral to Cardiologist for device therapy

55. When assessed in the community hospital,she was noted to have rheumatoid arthritis
deformities involving her hands.She was able to stand and transfer with the help of 1
person but was unsteady on herf eet.She was very keen to return home as soon as
feasible. Which walking aid is most likely to be of benefit at this stage?
A. Gutterframe
B. Rollator frame
C. Tripod walking stick
D. Walking crutches
E. Zimmer frame

56. An 86year old woman is inpatient stroke rehabilitation.She has a urinary catheter insitu.
Staff notice her urine is cloudy and send asample for urine culture. The patient is
clinically well and apyrexial fevers. Urine culture:significant growth of E coli
Which action is most appropriate ? B or E
A. Commence course of trimethoprim
B. Observe
C. Remove urinary catheter
D. Repeat the urine culture
E. Replace urinary catheter

57. An 80 year old man reports deteriorating mobility over the last 6 months.Both
knees are painful when hewalks and that this limits his mobility. Physical
examination reveals Heberden's nodes and there is significant crepitus present at
both knee joints. His BMI is calculated as 42kg/m².Which of the following
measures would be most appropriate ?
A. Acupuncture
B. Chondroitin supplements
C. Glucosamine supplements
D. Regular oral ibuprofen
E. Regular oral paracetamol

710
58. School competition 2 weeks ago,he accidentally bumped his fingers to a coming
ball.There was no open wound, however he felt the tip of his right middle finger was
painful and swollen.He complains that his right middle finger was not straight as it
used tobe. From the physical examination: the finger can be actively moved, yet the
finger tip rest as 40°flexion due to lack of active DIP extension.What is your
diagnosis?
A. Boutonniere deformity
B. Swan neck finger
C. Mallet finger
D. Jones fracture
E. Haglund deformities

59. From the above case (no.35). The finding of radiologic imaging of the right middle
finger a non displaced bony avulsion of distal phalanx. From the ultrasound
examination : oedem of the ligament and terminal extensor tendon distal to DIP joint.
Based on Doyle's classification, such injury can be classified as....
A. Type I
B. Type II
C. Type III
D. Type IV
E. Type V

60. In Basket ball Finger,the statement below is one of splinting indication, EXCEPT...
A. Maintain free movement of thePIPjoint
B. Correct the DIP joint in extension position
C. Avoid hyperextension of DIP joint
D. Prevent the finger from swan neck deformity
E. Protect other fingers from such injury

61. Kleirnet and Duran-Houser protocols are approach methods for repaired
tendon rehabilitation which performed...technique.
A. Immobilization
B. Early active mobilization
C. Late active rehabilitation
D. Early passive mobilization
E. Late passive rehabilitation

62. The Silesianbelt is most of ten used as an auxillary (backup) for traditional
suction suspension syatems.The problem with choosing the Silesianbelt as the
sole means of suspension is...
A.Instability to control residual limb rotation within the socket
B. Cannot be used for long residual limbs
C. Heavy and expensive
D. Difficult to do and off
E. Good in vigorous walking but not in running
63. Absolut contraindication in exercise tolerance testing, except...
A. Angina unstable
B. Dissecti Aneurisma aorta
711
C. Acute DVT
D. Resting BP>180/110mmHg
E. AV block gr II-III

64. A 60 years old patients with CAD in phaseII, stable condition with no
contraindication in excercise training,HR max of 160 and resting HR of 60
bpm.How many target HR from karvonen method to exercise prescription for this
patient?
A. 110-125bpm
B. 140-155bpm
C. 120-135bpm
D. 100-115bpm
E. 130-145 bpm

65. These need to be done in post operative chest therapy, except ....
A. Most therapy program start post-op day I
B. Diaphragmatic and Pursed lip breathing are used to assist the ventilator
C. Secretion management techniques include postural drainage,vibration,and
percussion
D. Vibration is preferred post operatively
E. These treatment are contraindicated inpatients with cardiacorhemodynamic
instability or in cases of pneumothorax

66. Lesions at the cervical and thorasic level can impair control over regional blood flow
during excercise,causing:
A. Enhance venous return and cardiac output
B. Venous blood pooling in the legs and consequently reducing cardiac preload
C. Limit max heart rate to110 to130beats per minute
D. Stroke volume and cardiac output given oxygen uptake tend to increase
E. Sympathetic nervous control over vasomotor and so do motor responses of the
insensate skin

67. Prehension describes the ability of the fingers and thumb to hold or seize objects with
hand. A form ofprehension that DOES NOT involve the thumb is:
A. Power grip
B. Precision grip
C. Hook grip
D. Power (key) pinch
E. Precision pinch

68. Muscle that, if shortened, predispose a person to slouched sitting posture with
posterior tilted pelvis are:
A. Hamstring
B. Quadricep
C. Iliopsoas
D. Semi spinalis
E. Abdominal

712
69. A muscle known as corset muscle which increase intraabdominal pressure
abdstabilizing lumbar region through attachment to thoracolumbarfascia is:
A. Rectus abdominis’
B. Obliquus externus abdominis
C. Obliquus internus abdominis
D. Transversus abdominis
E. Quadratus Lumborum

70. A

71. What type of injury does this mechanism


describe?
A. Fracture of tarsal bone
B. Rupture of Achilles tendon
C. Ankle Sprain
D. Gastrocnemius muscle strain
E. Syndesmosis sprain

72. What examination will give a positive


result?
A. Anterior drawer test
B. Talar tilt test
C. Percussion test
D. External rotation test
E. Thompson test

73. A 3-minute step test is one of the several method to estimate maximal oxygen
consumption from sub maximal exercise. It is targeted for:
A. General non lower extremity othopaedic and non obese patients
B. Active and acute conditions individuals
C. Peripheral vascular disease and elderly
D. Balance problem patients and obesity
E. Ambulatory persons with chronic conditions
713
74. Thesyme’s prosthesis is indicated
for:
A. Hip disarticulation
B. Above knee amputation
C. Through knee disarticulation
D. Below knee amputation
E. Ankle and partial foot disarticulation

75. What is posterior and anterior stops on an AFO ?


A. posterior stop limits plantar flexion;ananterior stop limits dorsi flexion
following . mid-stance
B. A posterior stop limits dorsiflexion; an anterior stop also limits dorsi flexion
following mid- stance
C. A posterior stop limits dorsiflexion; ananterior stop limits plantarflexion
following mid- stance
D. A posterior stop limits plantarflexion; ananterior stoplimits dorsiflexion following
push-off
E. A posterior stop limits plantarflexion; ananterior stop also limits plantar flexion
following mid-stance

76. Quadrilateral transfemoral socket:


A. Narrow mediolateral
B. The ischial tuberosity is contained inside the socket providing a bony lock
between ischium and greater trochanter.
C. There are reliefs for abductor longus,hamstring,greater trochanter,gluteus
maximus and rectus femoris
D. Weight bearing is consentrated in the medial aspect of the ischium and the ischial
ramus
E. Narrow mediolateral design was developed to provide a more normal anatomic
alignment of the femur insight the socket

77. What is the purpose of the terminal device?


A. To provide reach
B.Toprovide prehension
C. To provide carry
D. To provide release
E. To provide carry and release

78. Boston brace is also known as:


A. Milwaukee brace
B. Jewett brace
C. Cervico thoraco lumbosacral orthosis
D.Thoracolumboscral orthosis
E. Knee brace

79. The following is indication for Milwaukee brace except : B atau E

714
A. With cobb’sangle 50– 60 degrees
B. In geriatric patients
C. Notforhighthoraco/cervicalcurves
D.With lumbosacral curves
E. Not for patient with neurological complication

80. A variety of walk test are available is / are : E


1. Time – based test
2. Fixed – distance tests
3. Velocity – determined walk test
4. Controlled – pacing incremental tests

81. Clinical manifestations of congestive heart failure is / are : E


1. Dyspnea
2. Orthopnea
3. Tachypnea
4. Paroxysmal nocturnal dyspnea

82. Wallenberg syndrome occurs due to occlusion of the following : (A)


1. Vertebral arteries
2. Posterior inferior cerebellar artery
3. Superior ateral medullary artery
4. Posterior cerebral artery

83. Sensory processing is essensial for child growth. The components of processing is
(are) : E
1. Registration
2. Modulation
3. Discrimination
4. Organisation

84. The characteristic of motion development : B


1. The extensor system develops before the flexor system
2. The flexor system develops before the extensor system

715
3. The proximal stability followd by the development of distal mobility
4. The distal stability followed by the development of proximal mobility

85. This component(s) affects the speech ability : E


1. Sensory awareness, perceptual processing
2. Neuromusculosceletal
3. Cognitive
4. Cardiorespiration system

86. The risk factor of developing iliotibial band : E


1. Excessive running distance
2. Leg length discrepancy
3. Weakness of hip abductor muscles
4. Pes cavus

87. Beside suspends the prosthesis, the benefit of a pelvic belt and hip joimt on above knee
prosthesis is : A
1. Control rotation of the residual limb within the socket
2. Increase medial lateral stability of the residual limb within the socket
3. Can be used for those with short residual limbs
4. Comfortable in use while walking and sitting

88. A transfemoral amputation patients with above knee prosthese creates an active lumbar
lordosis during stance phase. The extensive trunk extensions is caused by the following
:E
1. Weak hip extensors and abdominal muscles
2. A flexion contracture that cannot be accommodated proshetically
3. Insufficient initial socket flexion leads the patient to extend the lumbar spine
4. A prosthesis that is too long

89. Clinical manifestations of congestive heart failure : E


1. Dyspnea

716
2. Orthopnea
3. Tachypnea
4. Paroxysmal nocturnal dyspnea

90. The goals of long-term management of the patient with valvular hearth disesase
include the following : E
1. Reduce the work of the heart
2. Maximize the patient’s quality of life
3. Efficiency of oxygen transport
4. Optimize physical endurance

91. The Purpose of exercise testing may be to assess any of the following : E
1. Functional work capacity
2. Possible presence and extent of coronary disease
3. Prognosis
4. Effects of therapeutic interventions

92. Indication for exercise test termination in cardiac rehabilitation are: E


1. Moderately severe angina (+3 to +4)
2. Sign of poor perfusion (palor, cyanosis)
3. Central nervous system symtoms (dizziness)
4. Drop in ABP > 10 mmHg from resting value

93. The dosage of laser dependent on : E


1. Output of the laser in mW
2. The time of exposure in seconds
3. The beam surface area of the laser in cm2
4. Output of the laser in Watt

94. The effect(s) of cryotheaphy : D


1. Increase nerve (sensory and motor) conduction velocity

717
2. Decrease pain threshold
3. Increase rate of blood flow
4. Reduce muscle spasm by interrupting the pain-spasm-pain cycle

95. In hip & knee osteoarthritis, strategies aimed at reducing loads on joint include : E
1. Weight loss
2. Cushioned shoes
3. Walking aids
4. Activity modification

96. The following are characteritic for the injuries to the menisci of knee joint : E
1. A sudden or forceful twisting motion on a planned foot is the most common
mechanism of injury for acute meniscy tears
2. Sometimes there is clicking within the knee
3. Physical examination include medial or lateral joint line tenderness & effusion
4. Mc Murray test is positive

97. Suggested rehabilitation on Bone Mineral Density T Score reduction to -1 to -2,5 SD


(osteopenia) : B
1. Patient education, preventive intervention
2. Pain management
3. Back strengthening exercises
4. No limit load lifting

98. The following is / are red flags of low back pain : A


1. Unintentional weight lose
2. Structural deformity
3. Difficulty with micturition
4. Emotions such as stress and anxiety

99. To prescrible an Ankle Foot Orthoses ( AFO ) the followings are considered : E
1. Metal or plastic AFO can be used effectively to control ankle motion

718
2. Metal AFO are relatively contra indicated in children because the weight of the
brace can cause external tibial rotation
3. AFOs should provide mediolateral stability as a safety feature
4. AFOs can not stabilize the knee during gait

100. The most common physiatric AFO prescription include : A


1. Foot drop
2. Lumbar spinal cord injury
3. Plantar spacity
4. Weakness of quadriceps muscle

101. The advantages of split hooks in terminal devices of upper limb prothesis is / are : E
1. Basic grasp function
2. Effisiency of use
3. Better ability of user to see what is being held
4. Ability to grape small objects

102. The objectives of upper limb orthotic applications are as follows : A


1. Protection
2. Correction
3. Assitance with function
4. Strengthen the muscle

103. Longterm use of lumbosacral orthoses can cause as follows : E


1. Increase in motion in the segment above or below that are controlled by the
orthoses
2. Muscle atrophy
3. Osteoporosis
4. Psychological dependence

104. Important rehabilitation goal for patient with burns include the following : E
1. Minimizing the occurrence and adverse consequence of hypertrophic scar

719
formation
2. Preserving motor skill function
3. Minimizing contracture formation and maintaning full joint range of motion
4. Improving strength and endurance

105. Muscle that need strengthening in preparation for crutch walking : E


1. Latissimus dorsi
2. Triceps
3. Pectoralis major
4. Hip extensors

106. Shoe modifications and foot orthoses in sport are widely prescribed with the primary
goal of : B
1. Altering patterns of movement
2. Improving performance skills
3. Lower extremity joint alignment
4. Overcoming injuries

107. Brain plasticity represents the capability of the damaged brain to repaired itself by
means of morphologic and physiologic response. Palsticity is influenced by : (E)
1. Complexity of stimulation
2. Environment
3. Repetition of task
4. Motivation

108. Patient with foot drop can caused by … A


1. Common peroneal
2. Diabetes diffus polyneuropathy
3. L4 – L5 Radiculopathy
4. Lumbar plexopathy

720
109. The following is exitatory neurotransmitters / mediators …(C)
1. GABA
2. Glutamat
3. Serotonin
4. Aspartate

110. The mechanical benefit of intermitten traction to the structures surrounding vertebrae
is : B
1. Relieve pressure on the dura and blood vessel
2. Reduce circulatory congestion
3. Relieve pressure on nerve roots in the intervertebral foramina
4. Decrease the concentration of noxious chemical irritants

111. In checking out flower extremity prothesis attention is directed to : A


1. Comfort and stability
2. Alignment
3. Appearance
4. Workmanship
112. The function of upper limb movement that must be considered in the orthotic
prescripstion is/are : E
1. Reach
2. Carry
3. Prehension pattern
4. Release

113. Pharyngeal phase of swallowing requires …. (A)


1. Soft palatic elevation and velopharyngeal closure
2. Laryngeal elevation, with forward movement of the hyoid bone and folding of the
epiglottis
3. Adduction of the ventricular and true vocal fole
4. Cricopharyngeal muscle contraction

721
114. What is post – polio syndrome ? A
1. A confirmed history of paralytic poliomyelitis
2. Partial to fairly complete neurologic and functional recovery
3. A period of neurologic and functional stability of at least 15 years duration
4. Onset of 2 or more of the following helath problems since achieving period of stability

115. Part of william flexion back exercise for patient with low back pain : A
1. Abdominal muscle strengthening
2. Hamstring strecthing
3. Gluteal muscle strengthening
4. Multifidius muscle strengthening

116. A way to reduce the amount of force required of the back extensor muscles while
lifting is : A
1. Reduce the speed of lifting
2. Reduce the magnitude of the external load
3. Reduce the length of the external moment arm
4. Reduce the length of the internal moment arm

117. Prime move muscle that make the scapula moving up : C


1. M Infraspinatus
2. M trapezius
3. M Rhomboideus
4. M Supraspinatus

118. The effectiveness of cervical traction is dependent on many factors, including the
amount of force applied othe factor include : A
1. Position of the cervical spine
2. Angle of pull
3. Position of the patient
4. Constant or intermitten traction

722
119. The dosage of laser dependent on : E
1. Output of the laser in MW
2. The time of exposure in second
3. The beam surface area of the laser in cm2-
4. output of the laser in Watt

120. The effect(s) of cryotheraphy : D


1. Increase nerve (sensory and motor) conduction velocity
2. Decrease pain threshold
3. Increase rate of blood flow
4. Reduce muscle spasm by interrupting the pain – spasm – pain cycle

121. In order to decrease the development of ligament and tendon sprains and strains and
chronic joint pain symtoms among weight lifters during weight training, it is
imperative to notice this following item : E
1. Proper form and tchnique
2. Constantly changing the exercise performed for each body part
3. Appropriate rest for each body part
4. Position order in training begin with supine, followed by squat, sit and stand
Naration : one of the overuse injuries due to posture among cyclists is perineal numbness
(number 99-100)

122. The injury might be due to : B


1. Pudendal nerve compression
2. Sacral root neuropathy
3. Penile arterial insuffciency
4. Too tight underwear

123. The recommended strategy to resolves the condition is : B

723
1. Frequent changing to a different saddle design
2. Put a soft and gentle foam underneath saddle sheath
3. Frequent standing during long workout
4. Wearing a proper and loosen underwear
CEDERA OLAHRAGA

1. The following are the true statements about ankle sprain, EXCEPT … D
a. The most common injured ligament is anterior talofibular ligament
b. The provocative tests are anterior drawer test and talar tilt test
c. The mechanism of injury is inversion on a plantar flexed foot
d. The mechanism of injury is eversion on a plantar flexed foot
e. History of rolling over the ankle

2. The following statement is one of the basic principle of rehabilitation protocol after
anterior cruciate ligament (ACL) injury
a. Appropriate use of open/losed kinetic chain exercise, avoiding early open chain
exercise that may shear or tear the weak immature ACL graft
b. Exercise for lower extremity usually done to gain muscular endurance rather than
flexibility and strength
c. Control of swelling and pain is not the important intervention in initial rehabilitation
program
d. Initiation of quadriceps and hamstring activity can be postponed

3. 24 year old marathon runner report lateral knee pain after hill training. Examination
reveals, no effusion, and results of ober test are positive.what is the most likely diagnosis?
a. Biceps tendinitis
b. Lateral meniscal tear
c. Popliteus tenosynovitis
d. Peroneal nerve entrapment
e. Iliotibial band friction syndrome

4. A 22-year-old female gymnast presents to your clinic after a patellar dislocation during
practice. She was treated in the emergency room with reduction of the patella and
immobilization. Radiographs and MRI of the knee are negative fracture or evidence of
osteochondral lesions. You choose to treat her with immobilization for 2 weeks and then

724
begin physical therapy. The most appropriate therapy recommendation is to focus on
improving …
a. Strength of the iliopsoas
b. Strength of the hamstring
c. Flexibility of the biceps femoris
d. Strength of the vastus medialis
e. Flexibility of gastrocnemius-soleus complex

5. A Patient complain of knee pain after falling of her flexed knee. Physical examination
shows a positive “sag sign”. Which ligament(s) is/are injured? D
a. Medial collateral
b. Lateral collateral
c. Anterior cruciate
d. Posterior cruciate
e. Meniscus

725
KINESIOLOGY

1. The following joint is classified trochoginglimus joint


a. Hip Joint
b. Wrist Joint
c. Knee Joint
d. Ankle Joint
e. Shoulder Joint

2. Atlanto-odontoid is classified as …
A. Gynglimus joint
B. Trochoid joint
C. Ellipsoidal joint
D. Condyloid joint
E. Ball and socket joint

3. The real leg length is the distance between …


A. Umbilicus to medial malleolus
B. Umbilicus to lateral malleolus
C. Femoral neck to lateral malleolus
D. Greater trochanter to lateral malleolus
E. Anterior superior iliac spine to medial malleolus

4. The following statement about type 1 muscles fiber is NOT correct.


A. Contain a large number of mitochondria and myoglobin
B. Dominant in much of marathon runner

726
C. Also called as slow twitch fibers
D. Predominantly aerobic
E. Generate larger force

5. In nondisabled subjects walking at self-selected normal speeds, the normal distribution of


time spent in double support is approximately . . . . . .
a. 10%
b. 20%
c. 30%
d. 40%
e. 50%

727
GERIATRI

1. Aging in neurologic aspect makes …


a. Shifting of brain activity from posterior to anterior region
b. Frontal gray matter gain
c. Increased brain volume
d. Increased blood flow
e. Cortical thicking

2. Anorexia of aging is meaning …


a. Increased energy intake
b. Increased stomach acid production
c. Hyperchlorhydria lead to bacterial overgrowth
d. Decreased appetite caused by decreased smell and taste sensation
e. Decreased gastric compliance causing late satiety and shortened postprandial satiety

3. A 65-year-old woman with breast cancer has a solitary metastatic lesion at the left rib ….
Among the following, the first-line pain medication in this setting is …
a. Ibuprofen
b. Amitriptyline
c. Acetaminophen
d. Morphin sulfate
e. Local corticosteroid injection

4. A 69-year-old woman wishes to reduce her risk of compression fractures and is interested
in taking alendronate. You inform her that…
a. Alendronate increases BMD by increasing osteoblastic activity

728
b. Gastrointestinal absorption of alendronate is facilitated by calcium
c. Although fracture risk is reduced with the use of alendronate, BMD unaffected
d. Use of alendronate by women with existing vertebral fractures decrease further
fractures
e. The most common side effects associated with use of alendronate cardiovascular and
neurologic systems
5. Postmenopausal women should ingest how many miligrams of calcium daily?
a. 500
b. 1000
c. 1200
d. 1500
e. 2000

6. After Alzheimer disease, the most frequent dementia in the elderly is secondary to …
a. Sepsis
b. Drug toxicity
c. Multiple infarcts
d. Subdural hematoma
e. Occult hydrocephalus

7. The following are contraindicated physical activities in the osteoporotic patients, except...
a. Golf swing
b. Running
c. Jumping
d. Walking
e. Jogging

8. The following are the physiologic changes in aging process, EXCEPT


a. Lower flexibility
b. Higher vital capacity
c. Lower reaction time
d. Higher residual volume
e. Higher resting blood pressure
729
9. Which the physiologic factor in the eldery exacerbates orthostatis?
a. Decreased in arterial stiffness
b. Decreased creatinine clearence
c. Decreased peripheral resistance
d. Decreased baroreceptor response
e. Decreased lower limb muscle strength

10. In prescribing pain treatment for eldery patients, it is best to use which approach?
a. Start with NSAID
b. Start low and go slow for all medications
c. Start with aspirin, which is safe and effective
d. Avoid use of acetaminophen, due to liver toxicity
e. Use high-dose, shortacting narcotics initially to get the pain under control

730
MODALITAS

1. Methods of treating acute inflammatory tendinitis include all of the following, EXCEPT
a. Use of ultrasonic heating
b. Application of cold packs
c. Splinting of the involved tendon
d. Local steroid and anesthetic injection
e. Administration of oral anti-inflammatory agents

2. The following is NOT used for lymphedema


a. Compression garmen
b. Superficial heating
c. Elevation
d. Stroking
e. Massage

3. A therapeutic modality theorized to exert its main effect by the “gate control theory” of
pain is …
a. USD
b. TENS
c. Contrast bath
d. Laser therapy
e. Magnetotherapy

4. When standing upright in water with surface level at the level of papilla mammae, the
relative body weight is …
a. 5% weight bearing
b. 15% weight bearing

731
c. 25% weight bearing
d. 35% weight bearing
e. 45% weight bearing

5. What is a contraindication for superficial heat?


a. Superficial thrombophlebitis
b. Joint replacement
c. Muscle spasme
d. Sensory deficit
e. Hematoma

6. Which modality most effectively treats the chronic periarticular tissue of the hip joint?
a. Hot packs
b. Infra red radiation
c. Shortwave diathermy
d. Microwave diathermy
e. Ultrasound diathermy

7. Heat Therapy through convection method could be generated by the following modality..
a. USD
b. Infra red
c. Cold laser
d. Whirpool
e. Ultraviolet treatment

8. Which finding is a relative contraindication to cryotherapy ?


a. Pain
b. Edema
c. Acute hematoma
d. Impaired sensation
e. Acute inflammation

732
9. Which modality should precede shoulder stretching in a patient with adhesive capsulitis?
a. Massage
b. Ice packs
c. Ultrasound
d. Laser Therapy
e. Electrical stimulation
10. Phonophoresis involves:
A. The use of ultrasound to facilitate transdermal mitigation of topically
administered Medications
B. The use of an imposed electric fi eld to mitigate charged particles across biologic
membranes
C. The use of ultraviolet radiation to mitigate charged particles across biologic
membranes
D. The use of ultraviolet radiation to facilitate transdermal mitigation of topically
administered medications
E. None of the above

733
Muskuloskeletal

1. Which ligament is most commonly injured in lateral ankle sprains?


a. Calcaneofibular ligament
b. Anterior talofibular ligament
c. Tibionavicular ligament
d. Posterior talofibular ligament
e. Calcaneocuboid ligament

2. Which bone is most commonly affected in a wrist fracture?


a. Lunate
b. Capitate
c. Distal radius
d. Scaphoid
e. Trapezoid

3. Lateral epicondylitis most commonly affects which muscle or muscles?


a. Extensor carpi radialis brevis and extensor digitorum communis
b. Pronator teres, flexor carpi radialis, palmaris longus, flexor carpi ulnaris, and flexor
digi-torum superficialis
c. Triceps muscle
d. Biceps tendon
e. Brachioradialis

4. Which muscle is the most powerful forearm supinator?


a. Supinator muscle
734
b. Pronator teres muscle
c. Biceps brachii muscle
d. Brachioradialis muscle
e. Pronator quadratus

5. The differential diagnosis of trigger finger includes all of the following except:
a. Dupuytren’s disease
b. Ganglion of the tendon sheath
c. Rheumatoid arthritis
d. Phalange fracture
e. Tendinitis

6. The true leg length should be measured between:


a. Greater trochanter and lateral malleolus
b. Umbilicus and lateral malleolus
c. Anterior superior iliac spine (ASIS) and medial malleolus
d. Anterior inferior iliac spine and medial malleolus
e. Anterior superior iliac spine (ASIS) and lateral malleolus

7. Adhesive capsulitis or frozen shoulder:


a. Results from thickening and contraction of the capsule around the
glenohumeral joint
b. Is more commonly seen in middle-aged men
c. Has risk factors including psoriasis
d. Is more frequently noted in colder climates
e. Caused by trauma

8. In sports, which knee ligament is the most commonly injured?


a. Anterior cruciate ligament (ACL)
b. Posterior cruciate ligament (PCL)
c. Lateral collateral ligament (LCL)
d. Medial collateral ligament (MCL)
e. Posterior talofibular ligamen
735
9. Which statement is not true regarding plantar fasciitis?
a. Increased tension on the plantar fascia leads to chronic inflammation
b. Heel spurs may contribute to its etiology
c. A tight Achilles tendon is frequently associated with plantar fasciitis
d. Night plantar flexion splints are not indicated
e. The treatment is stretching the heelcord
10. Which provocative test is useful in detecting rotator cuff impingement?
a. Drop arm test
b. O’Brien test
c. Apley scarf test
d. Neer’s test
e. Cross arm test

11. What is the most common cause of adhesive capsulitis?


a. Diabetes
b. Female gender
c. Hypothyroidism
d. Idiopathic
e. Trauma

12. Identify the final treatment phase of sports rehabilitation:


a. Resolving pain and inflammation
b. Restoring range of motion (ROM)
c. Strengthening
d. Sports/task-specific activities
e. Endurance

13. The diagnosis of aseptic noninflammatory olecranon bursitis:


a. Is based on plain radiographs, demonstrating an olecranon spur in all cases
b. Is made by X ray
c. Requires aspiration of bursal fluid in all cases

736
d. Is usually straightforward and based on characteristic appearance on physical
examination
e. Is made only with MRI

14. The rotator cuff muscles include all of the following except:
a. Teres minor
b. Supraspinatus
c. Rhomboids
d. Infraspinatus
e. Subscapularis

15. What is the Adson’s test used for?


a. To detect thoracic outlet syndrome
b. To check for adequate blood perfusion to the hand
c. To detect anterior instability of the shoulder joint
d. To detect symptoms of carpal tunnel syndrome (CTS)
e. To detect impingement

737
Pediatri

1. The most common site of spinal cord injury (SCI) in children is:
a. Thoracic
b. Lumbar
c. Cervical
d. Sacral
e. Lumbosacral

2. An 8-year-old with a diagnosis of Duchenne muscular dystrophy (DMD) is referred for


exercise. All of the following are allowed except:
a. Swimming
b. Daily walking
c. Weight lifting
d. Playing Wii
e. Playing card

3. Each of the following is true of tibia vara except?


a. Bowing of the proximal tibia is a result of abnormal function of the medial portion
of the proximal tibial growth plate
b. This disease is found most commonly in obese children who walk at 9 to 10 months
c. It is more common in Whites than other racial groups
d. Treatment is usually surgical, involving osteotomy of the proximal tibia and fi bula
e. Blount’s disease

4. Which of the following is the major factor limiting ambulation in Duchenne muscular
dystrophy (DMD)?
738
a. Joint contracture
b. Weakness
c. Scoliosis
d. Restrictive lung disease
e. Osteoporosis

5. Neural tube defects occur between ______ days of gestation:


a. 7 and 18
b. 18 and 30
c. 31 and 46
d. 47 and 65
e. More than 2 months\

6. The most common type of skull fracture after a head injury is:
a. Depressed
b. Linear
c. Comminuted
d. Open
e. Closed

7. Most children with spinal muscular atrophy (SMA) have:


a. Mental retardation
b. A single crease across their palm
c. Epilepsy
d. High cognitive function
e. Hearing loss

8. 4-year-old boy presents with a history of diffi culty climbing stairs, falling, waddling
gait, and large calf muscles. You suspect Duchenne muscular dystrophy (DMD). Of the
following, the easiest and best confi rmatory diagnosis test is:
a. Serum level of creatine kinase (CK)
b. Polymerase chain reaction (PCR) genetic test
c. Muscle biopsy

739
d. Simple blood test
e. Family history

9. In a child diagnosed with cerebral palsy (CP), independent sitting by what age is a good
prognostic indicator for ambulation?
a. 6 months
b. 12 months
c. 24 months
d. 36 months
e. 4 years

10. The occurrence of contractures in Duchenne muscular dystrophy (DMD) appears to be


directly related to which of the following?
a. Poor nutrition
b. Obesity
c. Prolonged static positioning of the limb
d. Muscular atrophy
e. Malnutrition

11. In which type of spinal muscular atrophy (SMA) can most of the patients sit but not
walk?
a. Type I
b. Type II
c. Type III
d. Type IV
e. Type I and II

12. Around what age should the fi rst prosthetic fi tting for a unilateral defi ciency occur?
a. 1 to 2 months
b. 3 to 4 months
c. 6 to 7 months
d. When the child begins to walk
e. When the child is able to navigate stairs

740
13. Which of the following is not a type of dyskinetic cerebral palsy (CP)?
a. Dystonia
b. Ataxia
c. Athetosis
d. Hemiballismus
e. Opistotonus
14. The most common identifi able risk factor for childhood ischemic stroke is:
a. Hematological disorders
b. Congenital heart disease
c. Central nervous system infection
d. Vasculitis
e. Trauma

15. Which is the most common type of cerebral palsy (CP)?


a. Spastic quadriplegia
b. Dyskinetic CP
c. Spastic diplegia
d. Mixed CP
e. Ataxic

741
Neuromuskular

1.Of the following choices, which is not a major problem affecting activities of daily living
(ADLs) reported by multiple sclerosis (MS) patients?
a. Sensory disturbance
b. Fatigue
c. Balance difficulties
d. Weakness
e. Muscle strength

2. Which of the following statements regarding Duchenne muscular dystrophy is true?


a. Usually diagnosed by age 5 years
b. X-linked recessive condition
c. Patients generally lose the ability to ambulate at 8 to 12 years of age
d. All of the above
e. Most common occure in women

3. In myasthenia gravis (MG), early monitoring of which of the following can help
prognosticate?
a. Exertion-related fatigue
b. Spirometry
c. Response to edrophonium
d. Onset of diplopia
e.

4. Symptoms of botulism present how soon after spore ingestion?


a. 1 hour

742
b. 2 to 4 hours
c. 1 day
d. 1 month
e. 3 months

5. Which condition is characterized by onset before age 20 years, gait ataxia, and
progressive paralysis?
a. Guillain-Barré syndrome (GBS)
b. Spinal muscular atrophy (SMA)
c. Friedreich’s ataxia
d. Becker’s muscular dystrophy (BMD)
e. Duchene muscular dystrophy (DMD)

6. What is the leading cause of mortality in children with neuromuscular diseases?


a. Neurologic complications
b. Renal complications
c. Pulmonary complications
d. Cardiac complications
e. Liver complications

7. Which of the following disorders is exacerbated with rest?


a. Lambert-Eaton myasthenic syndrome (LEMS)
b. Botulism
c. Myasthenia gravis (MG)
d. Amyotrophic lateral sclerosis
e. Multiple sclerosis

8. A 16-year-old male presents to your offi ce with concerns that recently he uses his hands
and arms to “walk” up his own body from a squatting position. He states that he was
otherwise independent with standing and walking and has been doing well as a student and
plans on attending college. Which lower motor neuron disease does he most likely have?
a. Spinal muscular atrophy (SMA) type I
b. SMA type II
743
c. SMA type III
d. Multiple sclerosis
e. Amyotrophic lateral sclerosis (ALS)

9. Of the several patterns of multiple sclerosis, which is most common?


a. Secondary progressive
b. Progressive remitting
c. Progressive-relapsing
d. Relapsing-remitting
e. Primary progressive

10. All of the following symptoms are seen in multiple sclerosis (MS) except:
a. Impairment of deep sensation, proprioception
b. Scanning speech
c. Impaired convergence
d. Bowel incontinence
e. Bladder incontinence

11. What is a requirement for the diagnosis of botulism?


a. Muscle biopsy
b. Botulinum toxin found in stool or blood
c. Chest x-ray
d. MRI
e. USG

12. Which of the following can trigger Guillain-Barré syndrome (GBS)?


a. Bacterial pneumonia
b. Gastrointestinal viral infection
c. Blood transfusions
d. Syphillis
e. Blandder disfunction

744
13. Which of the following is not a sign/symptom of Parkinson’s disease?
a. Impaired vibration or position sense
b. Resting tremor
c. Slowing of movements
d. Tremor superimposed on muscular rigidity
e. Bradykinesia
14. A patient presents to the emergency department with ptosis, anhidrosis, and miosis on
the left side of his face. He has difficulty walking and recently fell to his left side. On
physical exam, there is decrease in pain and temperature sensation on the left side of his
face and on the right side of his body. There is no muscle weakness. He most likely has:
a. Lateral medullary syndrome
b. Medial medullary syndrome
c. Benedikt’s syndrome
d. Weber’s syndrome
e. Central cord syndrome

15. Which of the following medications used for spasticity has the least amount of sedation
and cognitive impairment?
a. Diazepam
b. Dantrolene sodium
c. Baclofen
d. Clonidine

745
ORTOTIK-PROSTETIK

1. What is the “safe” position of the hand (to prevent claw deformity)?
A. Wrist extended, metacarpophalangeal (MCP) joint fl exed, proximal
interphalangeal (PIP) joint extended, distal interphalangeal (DIP) joint extended
B. Wrist flexed, MCP fl exed, PIP flexed, DIP fl exed
C. Wrist extended, MCP extended, PIP extended, DIP extended
D. Wrist flexed, MCP extended, PIP flexed, DIP flexed
E. None of the above

2. This device is used for cubital tunnel syndrome:


A. Long arm splint
B. Flail arm splint
C. Resting splint
D. Elbow flexion splint
E. Wrist flexion splint

3. Which of these devices assists with plantar flexion?


A) Anterior stop
B) Posterior stop
C) Anterior spring
D) Posterior spring
E) Posterior leaf spring

4. Which orthosis is useful for anterior cruciate ligament (ACL) tears or postop repair?
A) Cho-Pat strap
B) Swedish knee cage
746
C) Craig-Scott orthosis
D) Lenox Hill derotation orthosis
E) Infrapatellar strap

5. Where does most of cervical rotation occur?


A) C1 to C2
B) C2 to C3
C) C3 to C4
D) C4 to C5
E) C6 to C7

6. Which of these thoracolumbosacral orthoses (TLSO) cannot be used on an unstable


thoracolumbar spine fracture?
A) Jewett brace
B) Cruciform anterior spinal hyperextension (CASH) brace
C) Molded plastic TLSO
D) Taylor’s brace
E) None of the above

747
KARDIORESPIRASI

1. Dyspnea, tachycardia and sudden decrease of SaO2 to less than 90% in phase I cardiac
rehabilitation may cause by the following, EXCEPT …
A. Atelectasis
B. Pneumonia
C. Emphysematous
D. Pulmonary embolism
E. Exacerbation of CHF

2. For the post-stroke patient with history of coronary artery disease and fibrillation, an
aerobic exercise session should be terminated if …
A. Heart rate does not increases
B. Heart rate increases to 20% over baseline
C. Respiratory rate increases 10% over baseline
D. Diastolic blood pressure increases to 120 mmHg
E. Systolic blood pressure increases to 160 mmHg

3. The 6MWT is one of the clinical exercise testing, has been used for the following,
EXCEPT …
A. Provide the mechanism of exercise limitation
B. Monitor response to therapeutic intervention
C. Preoperative and postoperative evaluation
D. Assessment of functional capacity
E. Predict morbidity and mortality

4. What exercises are recommended for persons with non-progressive mild to moderate
restrictive lung disease (VC of 60% predicted)?
748
A. Inspiratory resistive exercise
B. Glossopharyngeal breathing
C. Rapid shallow breathing
D. Diaphragmatic breathing
E. Pursed lip breathing

5. The major goal of phase II cardiac rehabilitation is …


A. Assessment of cardiovascular function
B. To achieve cardiovascular conditioning
C. Alleviation of anxiety and depression
D. ADL independency
E. Early ambulation

6. During vigorous exercise, while participating in a stress test, it is noted that the T wave
increases by 10%. No other changes are noted. How should this be interpreted?
A. Definitely abnormal – highly suspicious but not absolutely diagnostic of ischemic
heart disease
B. Mildly abnormal – would recommend that maximum effort stress test be performed
C. Definitely abnormal and diagnostic for ischemic heart disease
D. Definitely abnormal and diagnostic for myocardial infarction
E. Normal

7. Physical activities advised to patients with CHF functional class III (NYHA) is…
A. 1-2 METs
B. 3 – 4 METs
C. 5 – 6 METs
D. 7 – 8 METs
E. 9 – 10 METs

8. During an acute episode of dyspnea in chronic obstructive pulmonary disease (COPD)


patients, which breathing technique may help to reduce symptoms and the work of
breathing?
A) Controlled cough
B) Huffing
749
C) Pursed-lip breathing
D) Breath holds
E) None of the Above

9. Which of the following is most useful in diagnosing obstructive lung disease?


A) Maximal static expiratory pressure (PE max)
B) Diffusing capacity for carbon dioxide
C) Forced vital capacity (FVC)
D) volume of air forcibly exhaled in the first second of forced exhalation (FEV1)
E) Ratio of the forced expiratory volume in 1 second to FVC (FEV1/FVC)

10. What is “air shifting”?


A. A technique to decrease microatelectasis
B. A technique to promote secretion drainage
C. A technique to ventilate the apical lung fields
D. A technique to reduce respiratory rate in patients with dyspnea
E. None of the above

750
EMG

1. In myopathies, the motor unit action potentials (MUAPs) may demonstrate all of the
following except:
A) Low amplitude
B) Moderate amplitude
C) Long duration
D) Polyphasicity
E) Early recruitment

2. On needle electromyographic (EMG) testing, normal muscles at rest:


A) Are electrically silent
B) Sounds like raindrops on tin roof
C) Will spontaneously discharge potentials with an initial negative deflection
D) Will spontaneously discharge potentials with an initial positive deflection
E) Will discharge potentials only if the muscle belly is tapped

3. To diagnose a conduction block with electrodiagnostic testing, what percentage decrease


in compound motor action potential amplitude should be noted from the proximal to the
distal segment?
A) 20%
B) 35%
C) 50%
D) 75%
E) 100%

4. All of the following muscles include innervation from the L4 nerve root except:
A) Tibialis anterior
751
B) Gluteus maximus
C) Sartorius
D) Adductor magnus
E) Adductor longus

5. What muscle does the long thoracic nerve innervate?


A) Supraspinatus
B) Rhomboid
C) Trapezius
D) Infraspinatus
E) Serratus anterior

6. Which of these statements regarding the H-reflex is not true?


A) This reflex is elicited with submaximal stimulation
B) As the intensity of the stimulation is subtly increased from the peak H-reflex
amplitude, there is a gradual drop in H-reflex amplitude with a concomitant increase
in M-wave amplitude
C) The H-reflex is often used to assess for S1 radiculopathy (with pickup over the
gastrocnemius-soleus group), but can also be used to assess for C6/C7
radiculopathy with pickup over the flexor carpi radialis
D) Side-to-side differences in H-reflex latencies of greater than 1.5 msec are suggestive
of S1 radiculopathies
E) Side-to-side differences in H-reflex latencies of greater than 1.0 msec are
suggestive of S1 radiculopathies

7. Axonal damage (with Wallerian degeneration) would present with:


A) Decreased compound motor action potential (CMAP) amplitude with proximal
stimulation and distal stimulation
B) Decreased CMAP amplitude with proximal stimulation but not distal stimulation
C) Decreased CMAP amplitude distally but not proximally
D) Slowing of conduction velocity across the lesion
E) Slowing of conduction velocity distal to the lesion

752
8. All of the following muscles are innervated by the posterior cord of the brachial plexus
except:
A) Triceps
B) Deltoid
C) Biceps
D) Brachioradialis
E) Anconeus

9. If a patient has tarsal tunnel syndrome, one would likely find which of the following on
nerve conduction studies?
A) Increased latency of the sural nerve at the ankle
B) Increased latency of the tibial nerve at the ankle
C) Decreased latency of the tibial nerve at the ankle
D) Decreased conduction velocity of the tibial nerve
E) Decreased conduction velocity of the peroneal (fibular) nerve

10. During electrodiagnostic testing, how can you tell if an accessory peroneal nerve is
present?
A) There is decreased compound motor action potential (CMAP) amplitude when
the peroneal nerve is stimulated at the ankle, and normal CMAP amplitude
with stimulation at the fibular head
B) There is decreased CMAP amplitude when the peroneal nerve is stimulated at the
fibular head, and normal CMAP amplitude with stimulation at the ankle
C) There is unusually slowed conduction velocity in the peroneal nerve
D) There is unusually fast conduction velocity in the peroneal nerve
E) None of the above

753
THERA EXC/TX WICARA/OT

1. Specific therapeutic exercise for scoliosis are all the following, except...
a. Done in and out of the brace
b. Stretching exercise of the convex side
c. EDLF exercise is done in the kneeling position
d. Muscle strengthening exercise of the convex side
e. Abdominal muscle strengthening exercise is included

2. The following are guideliness for exercise during pregnancy, except...


a. Avoid supine exercise after the first trismester
b. Stretching exercise is preferable to intermittent activity
c. Stop exercising when fatique and avoid exhaustive exercise
d. Pregnancy should maintain adequate intake for basal, exercise and pregnancy needs
e. Many physiologic and morphologic changes of pregnancy persist 12 weeks
postpartum

3. The following is the guidelines for exercise for persons with HIV infections.
a. Aerobic exercises that produce a negative energy balance is contraindicated
b. Aerobic exercises for person with established clinical weight loss is indicated
c. Aerobic exercises for person with a lack cardiopulmonary reserve is indicated
d. Person with asymptomatic HIV seropositive are restricted activity and competition
e. Person symptoms prior to development of AIDS are encouraged to exercise and
competition

754
4. Dysphagia may be due to:
a. Neurological dysfunction
b. Cancer
c. Aging
d. All of the above
e. None of the above

5. Treatment of acquired dysphagia in a 67 year old man with a left hemiparesis might
include...
a. Chin tucking and reclining feeding
b. Chin tucking and head turning to the affected side
c. Changing diet to one of softer and thinner consistency
d. Upright feeding and head turning to the nonaffected side
e. Use of the longterm thermal stimulation and upright feeding

6. An infarct in the lower division of the middle cerebral artery division would be
associated with which type of aphasia...
a. Broca
b. Global
c. Wernicke
d. transcortical motoric
e. transcortical sensoric

7. Which type of aphasia would you expect to see in a patient who is noted to have fluent
speech, with signs of comprehension, but unable to repeat?
A. Broca aphasia
B. Anomic aphasia
C. Conduction aphasia
D. Transcortical motor aphasia
E. Transcortical sensory aphasia

8. Using computer mouse that is too far to side or to high on the table will lead to...
a. Anterior shoulder subluxation
b. Thoracic outlet syndrome
755
c. Rotator cuff tendinitis
d. Bicipitalis tendinitis
e. Adhesive capsulitis

9. A 70-year-old woman complains of right knee pain and stiffness. On physical


examination, she has a genu varum deformity with unstable joint. A exercise training
prescription should include …
A. isokinetic hamstring strengthening
B. isokinetic quadriceps strengthening
C. isometric gastrocsoleus strengthening
D. open kinetic chain quadriceps strengthening
E. closed kinetic chain quadriceps strengthening

10. You are seeing a 56-year-old male patient in consultation 3 days after severe stroke. He
is medically stable and has flaccid hemiplegia with poor sitting balance. He is sitting up in a
chair for 2 hours twice daily and has just started bedside physical therapy and occupational
therapy. You would recommend …
A. Strength the flaccid arm
B. Improve with less intensive physical therapy.
C. Focusing on sitting balance, followed by transfer
D. Start aggressive physical and occupational therapy
E. Start electrical stimulation to the flaccid arm to enhance neurologic recovery.

756
UTULNAS MARET 2020

1. Which is the correct statement od Predictors of motor recovery in stroke patient?


A. Poor prognosis if there is severe distal spasticity
B. Poor prognosis if there is prolonged “flaccidity” period
C. Poor prognosis with no measurable grasp strength by two weeks
D. Good prognosis of recovery for useful hand function if there is complete arm paralysis
at onset
E. Full or good recovery if the patient shows some motor recovery of the hand by eight
weeks (two months)

2. What type of urinary dysfunction mostly occurs after traumatic brain injury?
A. Urinary tract infection
B. Urinary urgency/ frequency
C. Overactive bladder (OAB)
D. Detrusor sphincter dyssinergiamale (DSD)
E. Uninhibited bladder

3. Patient, male, 45 years old came to your clinic with stroke in sub acute phase. As you do
the examination, you found that he can move his hand to reach the stuff in front of him
with synergy pattern still predominating, there is decrease spasticity. Based on
Brunnstrom stage, which stage has the patient achieved?
A. Stage 2
B. Stage 3
C. Stage 4
D. Stage 5
E. Stage 6

4. One of the post stroke complication is Complex Regional Pain Syndrome type 1. Which
one is correct pathonegesis of this complication on stroke condition?
757
A. Abnormal adrenergic sensitivity develops in injured nociceptors, and circulating or
locally secreted parasympathetic neurotransmitter trigger that painful afferent activity.
B. Cutaneous injury activated nociceptor fibers that promote the peripheral pain Signalling
system that will evoke the pain.
C. High treshold mechanoreseptor input develops capacity to evoke pain.
D. Sympathetic responses are unknown but may be related to central dysregulation of
nociceptive impulses.
E. With time, afferent sympathetic fibers develop capacity to activate nociceptor fibers.

5. Post polio syndrome is a diagnosis of exclusions. The hallmark of post polio syndrome is:
A. Unusual fatigue.
B. Cold intolerance.
C. New weakness.
D. History of falls.
E. Myalgia.

6. A 26 year old woman with complete T6 paraplegia has managed her bladder since the
injury using intermittent self catheterization. Three months after spinal cord injury she
develods leakage. What pharmacological agents is likely to be most useful in controlling
this?
A. Ephedrine (noradrenergic)
B. Bethanecol (cholinergic)
C. Prazosine (alpha symmpathetic bladder)
D. Oxybutinin (anticholinergic)
E. Baclofen (gamma aminobutyric acid)

7. Autonomic control of the stomach and upper intestine:


A. Sympathetic innervation to the stomach, small intestine, and proximal colon is supplied
by the vagal nerve.
B. The recto colic reflex refers to the increase colonic activity in the first 10 minutes after
meal.
C. The gastro colic respons or gastro colic reflex refers to increases colonic activity in the
first 10 minutes after meal.
D. Enteric nervous system responsible for reflex circuits that coordinated motor pattern,
digestive activity, and giant migratory contraction.
E. Neural control of stomach and upper intestine are an extremely organized and
integrated mechanism that involve central nervous system.

8. A 42 years old male, diagnosed with Traumatic Spinal Cord Injury AIS A with neurological
level T8. He is complaining urine leakage during the day. The most possible bladder
dysfunction problem of this patient in voiding phase is:
A. Areflexive bladder.
758
B. Neurogenic detrussor underactivity.
C. Neurogenic acontractile detrusor.
D. Normal detrusor function.
E. Detrusor sphincter dysinergia (DSD).

9. A 34 years old man with SCI AIS A T7 due to transverse myelitis since 6 months ago. He
is using self CIC (clean intermittent catheterization) and complaining of increasing
spasticity and appearance of urine leakage between periods of catheterization. You should
be aware that the patient could have:
A. Pneumonia.
B. Urinary tract infection.
C. Autonomic dysreflexia.
D. Heterotropic ossificans.
E. Vesicourethral reflex.
10. Clinical signs of Parkinson’s disease are:
A. Resting tremor, spasticity, sensory deficit.
B. Resting tremor, contracture, bradykinesia.
C. Spasticity, weakness, dysphonia.
D. Dysphagia, dysphonia, rigidity.
E. Resting tremor, bradykinesia, rigidity.

11. A 21 years old female, single, paraplegia was referring as inpatient rehabilitation ward
due to T5 burst fracture. Stabilization with PPSW was done and now was medically stable.
On physical examination was found that vital signs and general health status were within
normal limits. What should you examine when you have to determine wheter the patient
has a compete or incomplete lesion?
A. Anal sensation and voluntary anal sphincter contraction.
B. Anocutaneal and bulbocavernous reflex.
C. Myotome and dermatome.
D. Physiological reflexes.
E. Pathological reflexes.

12. Mr. S 45 years old was admitted due to weakness on both legs, a days before he fell
from tree. He felt discomfort on his lower abdominal and could not urinate. He aldo
vomits every time he got drink or ate. In the Emergency Room he was consulted to
Digestive Surgeon. He was also consulted to an Urologist due to haematuria after
indwelling catheterization was inserted. Which statement not correct?
A. This patient could have ileus paraliticus and need to be prepared for operation.
B. This patient needed radiological findings 3 abdominal positions to diagnose his
abdominal problem.
C. This patient would need USG of the abdomen to diagnose what happened in his
abdominal.
759
D. CT scan of abdomen was not better from USG to find the problem.
E. This patient did not any supportive examination to diagnose hid abdominal problem.

13. Ms. G, 28 year old, was consulted to rehabilitation due to paralysis after a car accident.
The neurological examination came as follow:
Sensory: C2 – C6: 2/2
C7: 1/1
C8 – S4-5: 0/0
Motor: C5: 5/5
C6: 5/ not tested (severe wrist pain)
C7: 3/2
C8: 0/0
T1: 0/0
L2 – S1: 0/0
VAC: no/ DAP: no
Based on AIS classification what id the functional diagnosis of this patient:
e. SCI AIS A with sensory level C6 bilaterally, motor level C5 bilaterally, neurological
level C5, ZPP: sensory C6, motor C7.
f. SCI AIS A with sensory level C6 bilaterally, motor level C6 bilaterally, neurological
level C6.
g. SCI AIS unable to determine, with sensory level C5 bilaterally, motor level C5
bilaterally, neurological level C5, ZPP: sensory C6, motor C7.
h. SCI AIS A with sensory level C6 bilaterally, motor level C7 right, unable to
determine on the left side, neurological level unable to determine.
i. SCI AIS A with sensory level C6 bilaterally, motor level C7 right, unable to
determine on the left side, neurological level unable to determine, ZPP: sensory
C7, motor C7.

14. Which condition is characterized by onset before age 20 years, gait ataxia, and
progressive paralysis?
A. Guillane-Barre Syndrome (GBS).
B. Spinal Muscular Atrophy (SMA).
C. Friedreich Ataxia.
D. Becker’s Muscular Dystrophy (BMD).
E. Duchene Muscular Dystrophy (DMD).

15. Which of the following disorder is the weakness often relieved temporarily after
exercise or exertion?
A. Lambert-Eaton Myasthenic Syndrome (LEMS).
B. Botulism.
C. Myasthenia Gravis (MG).
D. Amyotrophic Lateral Sclerosis (ALS).
760
E. Multiple Sclerosis.

16. The root syndrome that result from hernia nucleus pulposus between vertebra
lumbalis 4 and 5 is:
A. Root L3.
B. Root L4.
C. Root L5.
D. Root S1.
E. Root S2.

17. A patient seen in the occupational unit is told to draw a clock. After examining his
illustration, you note that all the numbers of the clock are drew and squeezed to the right
side. You suspected that this patient has a:
A. Homonymus hemianopsia.
B. Bitemporal hemianopsia.
C. Quadrantanopia.
D. Hemi spatial neglect.
E. Cortical blindness.

18. If an individual has inability to initiate speech and respond to speech with anything
other than simple word, this individual has:
A. Non fluent aphasia.
B. Fluent aphasia.
C. Anomic aphasia.
D. Transcortical aphasia.
E. Disruptive aphasia.

19. Tic doloreaux is caused by pathology of which cranial nerver?


A. II.
B. III.
C. IV.
D. V.
E. VI.

20. An infact in the lower division of the left middle cerebral artery division ….. associated
with which type of aphasia?
A. Broca.
B. Global.
C. Wernicke.
D. Transcortical motoric.
E. Transcortical sensoric.

761
21. Which finding is associated with a more favourable prognosis in Amyotrophic Lateral
Sclerosis (ALS)?
A. Female sex.
B. Young age.
C. Bulbar involvement presentation.
D. Short time period from symptoms to diagnosis.
E. Predominance of lower motor neuron findings on EMG studies.

22. A 21 year old man is evaluated in your spinal cord injury clinic 12 months after C2
complete spinal cord injury requiring full tie mechanical ventilation. You recommed….
A. Aggressive frog breathing.
B. Aggressive diaphragmatic strenghtening exercise.
C. Aggressive breathing control exercise for his power wheelchair.
D. Initiating a weaning protocol by slowly decreasing tidal volume.
E. An electro diagnostic study to evaluate for a phrenic nerve pacemaker.

23. Which of following statement about Rheumatoid Arthritis (RA) is true?


A. Female : male ratio = 1 : 3.
B. Affects the synovial membrane of multiple joints.
C. The joints mostly affected are shoulder and hip.
D. DIP subluxation is the hand deformity in the late stage of RA.
E. Systemic inflammation is not the risk factor for osteoporosis in RA.

24. The following is true about Osteoarthrtitis (OA):


A. OA is systemic bone disorder.
B. Genetic predisposition is not the risk of OA.
C. The prevalence of OA is higher in men than woman.
D. Characterized by non progressive articular cartilage ….
E. Knee OA is heterogenous disease, involving cartilage,……, bone, menisci, and
periarticular soft tissue.

25. Medical rehabilitation after Total Knee Replacement:


A. Isotonic quadriceps exercise in the first week after surgery.
B. The goal after 2 weeks is 0-90o ROM of the affected bone.
C. Early mobilization is not recommended.
D. The use of CPM until 4 weeks.
E. Gait training after 3 weeks.

26. Which statement is correct concerning patient with osteoarthitis?


A. Exercise contributes to pain reduction.
B. Reduced aerobic capacity is difficult to reserve.
C. Knee orthosis and walking aids are not effective to reduce pain.
762
D. High intensity progressive ressistance exercise causes the disease to progress.
E. Group exercise programs are more effective than individual programs in reducing
disability.

27. Return to sport phase focused to the component:


A. Stregthening all muscles.
B. Stretching exercise.
C. Sport specific skills.
D. Balance training.
E. ROM exercise.

28. The increase in strenght notes 1 week after beginning progressive resistance exercise
in a specific group of mucles is most likely due to:
A. Increased muscle protein.
B. Increased recruitment of motor unit.
C. Increase in muscle oxidative capacity.
D. Conversion of fast to slow twitch fibers.
E. Decreased activity of Golgi tendon organs.

29. The muscle that has the important role in push off is:
A. Iliopsoas.
B. Hamstring.
C. Quadriceps.
D. Triceps surae.
E. Tibialis anterior.

30. What is the primary treatment for stenosing flexor tenosynovitis?


A. Resting and immobilization.
B. Surgical release.
C. Injection into the tendon sheath.
D. Repetitive gripping exercise.
E. Ultrasound therapy.

31. A 40 year old woman present with history of progressive pain over the proximal medial
aspect of tibia. The pain is said to be increased with running up hills. The most likely
diagnosis is:
A. Pes anserinus bursitis.
B. Prepatellar bursitis.
C. Patellar tendon synovitis.
D. Tibial collateral ligament strain.
E. Cartilage damage to the medial aspect to knee.

763
32. Sacroilitis is always seen in:
A. Reiter’s syndrome.
B. Intestinal arthropathy.
C. Psoriatic arthropathy.
D. Ankylosing spondylitis.
E. Juvenile chronic arthropathy.

33. Which statement is correct regarding the iliotibial band friction syndrome?
A. Excessive supination secondary to soft sole on shoe sole in causative.
B. The most severe pain is directly over medial joint line of the knee.
C. Treatment consist of stretching and massage of the ITB.
D. The ober test has not been found.
E. McMurray test has been found.

34. Which one is not a goal of acute phase of Rehabilitation in sport injuries?
A. Retard muscle atrophy.
B. Reduce pain and inflammation.
C. Create conditions for tissue healing.
D. Reestablish full range of motion of the around the injury.
E. Establish neuromuscular control of the joint in neutral position.

35. Which one is not the risk and etilogical risk factor of pressure ulcers?
A. Fecal and urinary incontinence.
B. Decrease skin temperature.
C. Pressure, shear, and frictions.
D. Hypoalbuminemia.
E. Elderly.

36. A 25 year old man presents to you with 2 days of knee pain and swelling after falling
off the ladder. The swelling began immediately after the fall. His neurologis examination is
normal. He is unable to fully extend or flex the knee because of the pain and swelling. He
is ambulating with antalgic gait limp. Your recommendation includes:
A. Icing, NSAID drug, knee immobilization, crutches, activity modification, X rays,
recheck in 5 days.
B. Icing, narcotics, TENS, refferal to an Orthopaedic Surgeon.
C. NSAID drug, X ray, return to work, recheck in 6 weeks.
D. Icing, crutches, X ray, recheck in 2 weeks.
E. Refer to an Orthopaedic Surgeon.

37. An 18 year old student sustained multiple injuries, including a proximal left radius
fracture, in an automobile accident. He is unable to extend the digits of his left hand. You

764
would expect to find fibrillation potentials in which of the following muscles if radial nerve
was injured near the fracture:
A. Triceps, supinator, extensor indicis.
B. Anconeus, brachioradialis, extensor digitorum communis.
C. Brachioradialis, extensor carpi radialis longus, extensor indicis.
D. Anconeus, extensor digitorum communis, extensor pollicis longus.
E. Extensor digitorum communis, extensor indicis, extensor pollicis longus.

38. Which of the following statement is not true regarding diabetic foot care?
A. In the presence of foot deformities, the physician should consider custom shoe insert
and or footwear to distribute pressure evenly over the foot.
B. It is advisable that patient minimizes or eliminates weight bearing during the healing
period of skin infection.
C. Patient should be encouraged to routinely visit podiatrist …. And general foot care.
D. Tennis or running shoes should be avoided in diabetic patient.
E. In the presence of sensory disturbance SWD is contraindicated.

39. De quervain’s tenosynovitis affects which two tendons?


A. Abductor pollicis brevis and extensor pollicis brevis.
B. Abductor pollicis brevis and extensor pollicis longus.
C. Abductor pollicis brevis and abductor….
D. Abductor pollicis longus and extensor pollicis brevis.
E. Abductor pollicis longus and extensor carpi radialis.

40. Which statement below is correct for patella femoral pain syndrome?
A. Osteoarthritis is one of the etiological factor.
B. The pain often to be felt while running or jogging.
C. Also reffered to as posteromedial knee pain syndrome.
D. Should always be associated with laxity of ligament around the knee.
E. Is characterized by pain at the front or middle edge of the knee or under the knee cap.

41. A national football player twist his knee when attempts to tackle an oncoming player.
Examination reveals no medial lateral laxity or joint line tenderness. The posterior drawer
test and pivot shift result are negative, but the lachman test result is positive. What is the
most likely diagnosis?
A. MCL injury.
B. LCL injury.
C. ACL injury.
D. PCL injury.
E. PCL MCL injury.

42. Which following statement is correct about tennis elbow in the athlete?
765
A. Return to sport after the patient is pain free and involved extremity’s strength 50% of
that on the contralateral side.
B. Tenderness is localized to 2-5 cm proximal and anterior to the lateral epicondyle.
C. Symptoms are reproduced with the wrist resisted flexion while elbow is extended.
D. Acute on chronic pain is suggestive of frank rupture of the extensor origin.
E. The extensor carpi ulnaris is most often involved.

43. 24 year old marathon runner reports lateral knee pain after hill training. Examination
reveals no effusion, and result of Ober test are positive. What is the most likely diagnosis?
A. Biceps tendinitis.
B. Lateral meniscal tear.
C. Popliteus tenosynovitis.
D. Peroneal nerve entrapment.
E. Iliotibial band friction syndrome.

44. Which following statement regarding ankle sprain is correct?


A. Grade III lateral ankle sprain is there is a complete tear……
B. In the plantar flexed position, the ATFL is under tension….
C. Treatment in chronic phase of lateral ankle sprain is strengthening gastrocnemius
muscles.
D. Medial ankle sprain is the most common injury, than lateral one, because deltoid
ligament is weak.
E. Return to sport patient with lateral ankle sprain when pain and swelling are minimal and
the gastrocnemius are strong.

45. A martial arts player complaint of back knee pain after kicks his opponent. Examination
of knee demonstrates pain during varus stres test. What is the most likely diagnosis?
A. Medial collateral ligament injury.
B. Lateral collateral ligament injury.
C. Posterior cruciate ligament injury.
D. Anterior cruciate ligament injury.
E. Strain popliteus muscle.

46. The muscle that is inactive at the time of initial contact in the ….. is:
A. Gluteus maximus.
B. Iliopsoas.
C. Quadriceps.
D. Hamstring.
E. Pretibial muscles.

47. Which of the following muscle is not an erector spinae group?


A. Iliocostalis thoracis.
766
B. Longissimus thoracis.
C. Spinalis thoracis.
D. Iliocostalis cervicis.
E. Semispinalis cervicis.

48. The factor that cannot stimulate the bone healing process is:
A. Optimal axial loading.
B. Short period immobilzation.
C. NSAID.
D. Ultrasound Diathermy modalities can be used in sub acute phase.
E. Exersice beyond the range of pain.

49. The unbalanced force that gives rise to an active against an inidividu ….. object when
the individual makes or modifies the movement, …..
A. Force motions.
B. Force time.
C. Coordination continuum.
D. Optimal projection.
E. Spin or rotation.

50. At the structural end of the neuromuscular control process are the functional units of
the control of muscle called:
A. Motor unit.
B. Muscle spindle.
C. Musculotendinous junction.
D. Myoglobin.
E. Muscle fiber.

51. A patient with subtalar joint axis less then 45 degrees in the sagittal plane will tend to
have:
A. Abduction that is smaller than rearfoot or hindfoot.
B. Less pronation of eversion.
C. Excessive supination of eversion.
D. Excessive pronation of eversion.
E. A high foot arch.

52. Contrast bath is not indicated in:


A. Rhematoid arthritis.
B. Reflex sympathetic dystrophy.
C. Arteriosclerosis endarteritis.
D. Joint sprain.
E. Muscular strains.
767
53. Which is not therapeutic heat?
A. Radiant heating heat.
B. SWD.
C. US diathermy.
D. Ultraviolet radiation.
E. Phonophoresis.

54. The typical sequences of sensation in response to cryotherapy is:


A. Intense cold – burning – aching – analgesia – numbness.
B. Intense cold – aching – burning – analgesia – numbness.
C. Intense cold – analgesia – aching – burning – numbness.
D. Intense cold – numbness – burning – aching – analgesia.
E. Intense cold – burning – aching – numbness – analgesia.

55. A balanced forearm orthosis (BFO) is shoulder elbow wrist hand orthosis (SEWHO). The
device can be helpful to support the forearm and arm against gravity and allows patients
with weak shoulder and elbow muscles to move the horizontally and flex the elbow to
bring the hand to the mouth. One that is requirement for prescribing the device is:
A. Some residual muscle strength (MMT at least poor …..grade 2).
B. Preserved ROM of the shoulder and elbow joint.
C. Lack of coordination of elbow flexion.
D. Adequate trunk stability and balance.
E. Adequate endurance in sitting position.

56. A patient came with neck pain caused by traumatic accident on wheel. The
examination reveals soft tissue injury and cervical ligamentous sprain. Your prescribe is:
A. Soft cervical collar.
B. Thomas colar/ hard cervical collar.
C. Philadelphia collar.
D. Sterno occipital mandibular immobilizer (SOMI).
E. Minerva cervico thoracic orthosis.

57. To prescribe an Ankle Foot Orthoses (AFO), which of the following is not a
consderation?
A. Metal or plastic AFO can be used effectively to control ankle motion.
B. Metal AFO are relatively contra indicated in children because the weight of the brace
can cause external tibial rotation.
C. AFO should provide mediolateral stability as a safety feature.
D. AFO cannot stabilize the knee during gait.
E. AFO may help ankle dorsiflexion.

768
58. A systematic approach to collecting patient data that allows the practitioner to
evaluate the frail older adult health status and functional impairment in multiple area or
domains is:
A. Holistic geriatric assesment.
B. Frailty assesment.
C. Functional geriatric assesment.
D. Comprehensive geriatric assesment.
E. Holistic mobility assesment.

59. Which of the following is not include to general health of geriatric assesment?
A. Physical medical condition.
B. Mental health condition.
C. Functioning.
D. Social circumstances and environment.
E. Formal job description.

60. Functional status in the elderly is influenced by:


A. Body function condition and social support.
B. Spcial support and cognitive.
C. Spiritually and personal factors.
D. Personal factors and affective.
E. Affective and body structure condition.

61. Help elderly patient become independent as maximum as possible, and to prevent
from losing the ability to function is:
A. Short term goals.
B. Immediate goals.
C. Longterm goals.
D. Care plan.
E. Discharge planning.

62. Basic comprehension of hierarchy of physical function in the elderly are:


A. Strength and endurance.
B. Balance and activity daily living.
C. Flexibility and activity daily living.
D. Coordination gross motor and specific physical movements.
E. Coordination fine motor and gross motor.

63. Assesment of cognition in elderly commonly used is/are:


A. Mini mental state examination.
B. Mild cognitive impairement.
C. Geriatric depression scale.
769
D. Geriatric depression scale and mini mental state examination.
E. Mini mental state examination and Mild cognitive impairement.

64. Assesment of prognosis in cases of elderly rehabilitation includes:


A. ad vitam and ad functionam.
B. ad vitam and ad sanationam.
C. ad vitam, ad sanationam and ad functionam.
D. ad sanationam and ad functionam.
E. dubia ad bonam and dubia ad malam.

65. In discharge planning the elderly requires several meetings, including:


A. Management meeting, geriatric meeting, family meeting, case conference.
B. Geriatric meeting, family meeting, case conference.
C. Geriatric meeting, case conference.
D. Geriatric meeting, family meeting.
E. Management meeting, geriatric meeting, family meeting.

66. Living situation in elderly encompasses two dimension:


A. The safety of the home environment and public environment.
B. The safety of the home environment and the adequacy of the patient’s access to
needed personal and medical services.
C. The adequacy of the patient’s access to needed personal and medical services.
D. The safety of the home environment and medical services.
E. The safety of the home environment and the adequacy of the patient’s access to
needed personal services.

67. Goals of care in elderly patients are:


A. Always social and functional goals assume priority over health related goals.
B. Regaining a previous health status, attending a future family event.
C. Completing ADLs without help and choosing an appropiate decision maker.
D. Regaining previous healt status and choosing an appropiate decision maker.
E. Completing ADLs without help and choosing an appropiate decision maker.

68. Children 2 years of age have stereotypic and ritualistic behaviour with impairment in
social interactions, communications, and imaginative play, are:
A. Down syndrome.
B. ADHD.
C. Autism.
D. Fragile X Syndrome.
E. Mental retardation.

770
69. This reflex is the best elicited by sudden dropping of the baby’s head in relation to its
trunk.
A. Righting reflex.
B. Landau reflex.
C. Moro reflex.
D. Vertical suspension.
E. Extensor thrust.

70. Lumbar lordosis in Duchene muscular dystrophy is:


A. Compesation of hip extensor weakness.
B. Compesation of hip flexor weakness.
C. Compesation of hip adductor weakness.
D. Compesation of hip abductor weakness.
E. Compesation of hip endorotator weakness.

71. Primary foot contact in duchene muscular dystrophy is:


A. Compesation of hip extensor weakness.
B. Compesation of knee extensor weakness.
C. Compesation of ankle plantar flexor weakness.
D. Compesation of hip flexor weakness.
E. Compesation of knee flexor weakness.

72. About becker muscular dystrophy:


A. Autosomal recessive inheritance.
B. The disorder is on gene Xp21.
C. Has the same incidence as Duchene muscular dystrophy.
D. BMD patients have distribution of weakness similar to those with distal spinal muscular
atrophy.
E. Creatine phosphokinase usually normal.

73. Motor development of a child with spina bifida….


A. Children with high lumbar lesion pull to stand and cruise near the expected age.
B. Lower lumbar lesion often requires devices, such as a reciprocal gait orthose.
C. Children with mid lumbar lesion can usually sit with some delay and increased
lordosis.
D. The level of neurologic impairment did not influence the expectations for function
outcome and prognosis.
E. Children with lumbar lesion require assistive for passive standing, usually started at 12
to 18 months.

74. Secondary deformity that does not happen in children with torticollis such as:
A. Contrallateral occipital flattening.
771
B. Flattening of ipsilateral face.
C. Scoliosis cervical.
D. Arachnodactyly.
E. Plagiocephaly.

75. Brachial plexus injury in children include:


A. Never caused by delivery the baby.
B. Violent upward pull the shoulder, causing damage to the upper trunk.
C. Horner’s syndrome can be associated with injury of the T8 and T1 roots.
D. Erb’s paralysis typically result from a fall onto a hyper abducted shoulder.
E. Sudden traction to the neck, causing injury to the lower trunk of the brachial plexus.

76. Musculoskeletal assesment in the children is considered abnormal if:


A. Full term infant lack of 25o of elbow flexion.
B. Full term infant lack of 30o of hip extension.
C. Full term infant lack of 30o of elbow extension.
D. Preterm infant lack of 45o of hip extension.
E. Preterm infant with lower muscle tone.

77. Lower limb examination may demonstrate ROM & bony abnormality, but some the
consider normal in development, such as:
A. Bow leg in children less than 2 years old.
B. Flat feet physiologis in 5 years old.
C. Toeing in with tibial torsion in toddler.
D. Children who often W sitters walk with patella to be medially deviated.
E. Metatarsal are medially deviated in 5 years old.

78. Child’s gait pattern change and progress until 7 years old. It is considered abnormal if:
A. High guard position of the arm at 2 years old.
B. Foot flat pattern at 1 years old.
C. Stride length decrease as they grow.
D. Cadence decreace as they grow.
E. Base of support narrowing.

79. In patient with myelomeningocele at the level of lesion L3-4 should:


A. have iliopsoas function.
B. have quadriceps function.
C. have hamstring function.
D. have extensor hip function.
E. walk independently without forearm crutch.

80. Juvenille Rheumatoid Arthritis is….


A. Joint inflammation less than 6 weeks.
B. In children more than 16 years old.
C. Any joint can be affected and may limit the mobility.
D. It is not possible accompanied by eye problem.

772
E. All patients have positive rheumatoid factors.

81. The method frequently used on CP which bases treatment on activation of postural
development and equilibrium reaction to guide normal development is:
A. Bobath method.
B. Deaver method.
C. Votja method.
D. Phelp method.
E. PNF method.

82. The statement that is not associated with the role of exercise in obesity children:
A. Prevention of becoming overweight and obese.
B. Reducing the risk of obesity in adulthood.
C. Engagement in physical activity.
D. Increasing of active behaviour.
E. Protect against chronic disease.

83. Which of the following types of CP is prone to develop contractures?


A. Ataxic.
B. Spastic.
C. Flaccid.
D. Athetoid.
E. Choreiform.

84. Protective and postural reflexes (advanced postural reactions) is…


A. Plantar grasp.
B. Foot hand replacement.
C. Head and body righting.
D. Symmetric tonic neck reflex.
E. Asymmetric tonic neck reflex.

85. Bowlegs in children is caused by growth retardation of the medial and posterior part of
the proximal tibial epiphysis due to abnormal weight bearing stress and compression force
on the knees. This disease is known as…
A. Genu varum.
B. Blount disease.
C. Rickett’s disease.
D. Ollier multiple enchondromatosis.
E. Arthrogyposis multiplex congenital.

86. A 4 years old boy was brought into your officel because his mother has noticed that he
has difficulty getting up from a seated position on the floor while playing with his toys. On
physical exam, there was increased gastrocnemius calf circumference bilaterally. You think
the child may have dystrophic myopathy. The maneuver the child performs to assist him in
standing was caused by…
A. Distal leg weakness.
B. Distal arm weakness.
C. Proximal leg weakness.
773
D. Proximal arm weakness.
E. Spinal muscle weakness.

87. Early treatment infant with diagnosis CTEV is…


A. Ankle foot orthosis.
B. Dennis Brown splint.
C. Outflare orthopedic shoes.
D. Surgery for soft tissue release.
E. Stretching and serial plaster correction.

88. Based on acquisition of speech and language skills, in what age does the child have the
ability to first word emerge and responds to simple requests?
A. 3 month.
B. 6 month.
C. 9 month.
D. 12 month.
E. 15 month.

89. Crouched gait in CP is caused by…


A. abductor weakness.
B. increased adductor tone.
C. increased iliopsoas tone.
D. spasticity of the gastrocnemius muscles.
E. hamstring, psoas, rectus femoris, and gastrocnemius contractur.

90. Not a correct statement for Haemophillia is…


A. After repeated hamarthrosis, irritating haemosiderin deposits occurs in the synovial
lining cells, the synovium becomes largely replaced by fibrous tissue.
B. Most patients with haemophillia in whom joint problems develop have factor VIII or IX
deficiency.
C. The prevalence of joint contracture in the severe form of the disease between 50-95%.
D. Treatment for acute hamarthrosis with ROM and deep heating.
E. The knees, elbows, and ankles are the joints most often affected.

91. Which one the absolute contraindication of cardiac rehabilitation for patient with
chronic heart disease?
A. Blood glucose 150-200 mg/dl.
B. Hemoglobin 10-12 g/dl.
C. Pulmonary hypertension.
D. Diastolic pressure >115 mmHg.
E. Systolic pressure > 140 mmHg.

92. What is the stratification risk for the patient that feels angina and short of breathness
when he plays soccer and echocardiography examination finding the risk ejection is 45%?
A. Low risk.
B. Moderate risk.
C. Low to moderate risk.
D. Unstratified risk.
774
E. High risk.

93. Which of the following conditions is not a potential contraindication for entry into a
cardiac exercise programme?
A. Unstable angina.
B. Resting diastolic blood pressure >100 mmHg.
C. Aortic stenosis.
D. Active pericarditis.
E. Well controlled diabetes.

94. The following is the most correction exercise prescription for patient with chronic
heart failure…
A. Supervised, aerobic exercise, duration can be started from 30 minutes, increased
gradually, continuous intensity 70-90% VO2max 3-5 times per week.
B. Supervised, aerobic exercise, duration can be started from 10 minutes, increased
gradually, continuous intensity 50-70% VO2max 3-5 times per week.
C. Ground walking exercise activity 5 mph, start with 15 minutes duration, 3-5 times per
week.
D. Supervised, competitive sport, intensity start with 60% VO2max 3-5 times per week.
E. Aerobic ergocycle exercise, gradual intensity as tolerated 60 minutes 3-5 times per
week.

95. The most correct step of cardiac rehabilitation for patient with chronic heart disease
is…
A. Giving exercise, medication evaluation, muscle strength assesment, functional capacity
assesment, psychological assesment, enviroment assesment.
B. Patient risk stratification assesment, functional capacity assesment, activity and
vocational evaluation, psychological profile assesment, medication evaluation, exercise
program, evaluation of cardiac rehabilitation program.
C. Functional capacity assesment, medication evaluation, psychosocial assesment, giving
exercise, enviroment assesment.
D. Medication evaluation, muscle strength assesment, muscle endurance assesment,
exercise program, psychological profile assesment.
E. Functional capacity assesment, patient risk stratification assesment, medication
evaluation, evaluation of cardiac rehabilitation program.

96. Studies have shown that the risk reduction with cardiac rehabilitation after a
Myocardial Infaction (MI) is…
A. 1%
B. 5%
C. 10%
D. 15%
E. >20%

97. What is the expected outcome of effective exercise training in pulmonary


rehabilitation program for patient with chronic obstructive pulmonary disease?
A. Reduced need for supplemental oxygen.
B. Reduced dyspnea.
775
C. Improved radiological status of the lung fields.
D. Slowed loss of forced expiratory volume.
E. Improved lung anatomy.

98. During an exercise tolerance test, what percent the maximum heart rate is the usual
target?
A. 65
B. 75
C. 85
D. 95
E. 60

99. A patient with restrictive lung dsease and mild scoliosis from Duchene Muscular
Dystrophy (DMD) develops noctural hypoventilation. What is the initial treatment?
A. Incentive spirometry every four hours.
B. Treacheostomy and nocturnal mechanical ventilation with positive pressure.
C. Nocturnal supplemental low flow oxygen.
D. Nocturnal non invasive positive airway pressure and short periods of daytime
hyperinsufflation.
E. Breathing control.

100. Which cardiorespiratory outcome would you expect from an elderly person
participating in an aerobic program?
A. A decrease in VO1max but an increase in walking speeds.
B. No change in tolerance during activities of daily living.
C. Improvement of VO2max.
D. No change in stroke volume and reduction in systemic vascular resistance.
E. Improvement of Quality of Life.

101. Other changes in the cardiopulmonary system those result from exercise:
A. Decrease vital capacity.
B. Decrease sistolic blood pressure.
C. Increased resting heart rate.
D. Increased oxygen saturation.
E. Increased cardiopulmonary efficiency.

102. The volume of gas that remains in the lung at the end of maximum expiration is…
A. Tidal volume.
B. Residual volume.
C. Total lung capacity.
D. Functional capacity.
E. Functional residual capacity.

103. Patient are comfortable at rest, but ordinary physical activity results is dyspnea,
fatigue, palpitation, pain, or angina, include NYHA…
A. I
B. II
C. III
776
D. IV
E. V

104. Based on comprehensive patient assesment, the goals of longterm management for
patient with chronic bronchitis may include the following…
A. Reduce ventilation
B. Reduce alveolar ventilation
C. Reduce secretion clearance
D. reduce the work of breathing
E. Reduce perfusion matching and gas exchange

105. The best predictor of mortality of COPD and its responds well to intervention such as
lung volume reduction is…
A. Force expiratory volume in 1 second.
B. BODE index.
C. Degree of hypoxaemia.
D. Presence of hypercapnia.
E. Presence of hypercarbia.

106. Which of the following parameters most closely corelates with oxygen consumption?
A. Oral temperature.
B. Rectal temperature.
C. Pulse rate.
D. Respiratory rate.
E. Blood pressure.

107. Maximal heart rate that recommended for cardiac insufficient patient during exercise
is…
A. 40 to 50 percent of maximal heart rate.
B. 50 to 60 percent of maximal heart rate.
C. 60 to 70 percent of maximal heart rate.
D. 70 to 85 percent of maximal heart rate.
E. More than 85 percent of maximal heart rate.

108. What is the intrinsic factor that does not contribute to the development of tendinitis?
A. Age.
B. Genetic predisposition.
C. Poor training technique.
D. Muscle imbalance.
E. Anatomic malaligment.

109. Characteristic of closed kinetic chain exercise, except:


A. Distal end fixed.
B. Squatting.
C. More shear stress.
D. Stimulate the propioceptive system.
E. Leg press.

777
110. What muscle does not need strengthening in preparation for crutch walking?
A. Triceps muscle.
B. Latissimus dorsi muscle.
C. Pectoralis major muscle.
D. Quadriceps muscle.
E. Serratus anterior muscle.

111. Contraindication of diathermy:


A. Allergic to cold.
B. Epilepsy.
C. Malignancy.
D. Spasticity.
E. Joint contracture.

112. When to start late post operative and early exercise phase in the rehabilitation of
anterior cruciate ligament reconstruction?
A. Days 1-14.
B. Weeks 2-6.
C. Weeks 6-12.
D. Month 3-6.
E. Month 6-9.

113. The incorrect statement of splinting to prevent contracture in burn management is:
A. Static splinting is used mainly to counteract the deforming position.
B. Splints can provide the stress needed to orient the collagen being deposited during the
early stages of wound healing as well as maintain joint aligment, or to increase ROM.
C. Serial casting can result in flatter, softer and more supple tissue.
D. Splint can prevent contracture without exercise.
E. Splint are placed avoiding compression to the burnt area.

114. You notice that your patient has varus deformity at the ankle when he walks. What
adjustments to his shoe can you suggest so that his gait is more stable?
A. Cushioned heel.
B. Medial heel flare.
C. Lateral heel flare.
D. Heel lift.
E. Insole.

115. Which ankle foot orthoses (AFO) can be used for someone with a foot drop and weak
quadriceps?
A. AFO with posterior foot stop.
B. Solid AFO.
C. AFO with posterior stop with knee Klenzak joint.
D. Solid AFO with anterior and posterior stop.
E. Leaf spring AFO.

116. A patient present with wrist pain. He has pain when gripping a cup handle. You notice
a positive Finkelstein’s test. What brace would you recommend for this patient?
778
A. Volar wrist splint.
B. Radial nerve palsy splint.
C. Resting hand splint.
D. Thumb spica splint.
E. Knuckle Bender.

117. In a person with transfemoral amputation, an abnormal prosthetic gait with lateral
trunk bending toward the involved side in midstance most commonly occurs with hip:
A. Adductor weakness.
B. Abductor weakness.
C. Flexion contracture.
D. Extension contracture.
E. Internal rotator weakness.

118. The prehensile function of a myoelectric hand stimulate which grip pattern?
A. Lateral.
B. Power.
C. Spherical.
D. Three jaw chuck.
E. Pinching.

119. What is the test to check for contraction of the iliotibial band?
A. Thomson test.
B. Painful arc test.
C. Ober test.
D. Yegarson test.
E. Thomas test.

120. Assuming the use of prostheses, which lower extermity amputation, requires the
greatest energy expenditure for ambulation?
A. Unilateral Syme.
B. Hemipelvectomy.
C. Bilateral above knee.
D. Unilateral hip disarticulation.
E. Below knee plus above knee.

121. Which of the following best defines an eccentric contractions?


A. Muscle lengthening contraction.
B. Muscle shortening again resistance.
C. Muscle shortening without resistance.
D. Muscle does not change length during contraction.
E. A contraction generating little force.

122. An example of closed kinetic chain exercise for the quadriceps is:
A. A partial squat.
B. An isometric quadricep set.
C. Knee extension on a stacked weight machine.
D. Knee extension on an isokinetic machine (eg, Cybex).
779
E. Jumping jack exercise.

123. Which exercise has the greatest effect on bone formation?


A. Cycling.
B. Swimming.
C. Weight lifting.
D. Tai Chi Stretching.
E. Yoga.

124. The following are occupational therapy exercise that can best be used in patient with
frozen shoulder:
A. Tic tac toe board.
B. Macrame.
C. LOTCA.
D. Hammer board.
E. Arm skate board.

125. In thoracic outlet syndrome, if the adson’s test is positive, this suggested to the cause
of following condition:
A. Hyperthophy of the splenicus cervicis muscle.
B. Hypertrophy of the M. scalenus posterior.
C. Hyperthophy of the M. pectoralis mayor.
D. Fracture of the clavicle.
E. Cervical rib.

126. A 25 years old man has had heel pain for 1 month. He runs approximately 40 km
weekly. The pain is most severe when he first stands up and walk after sitting or lying
down. Palpation of the medial plantar surface of the heel increase the pain. He walks with
a limp. Which of this following is the most likely cause of the pain?
A. Heel spur.
B. Achilles tendinosis.
C. Impingement of the posterior sacral nerve root.
D. Inflammation of the origin of plantar fascoa.
E. Articular cartilage degeneration of the calcaneo cuboid joint.

127. Which following statement is correct regarding the degenerative joint disease of the
cervical spine?
A. Radicular pain is very rare.
B. Vertigo and visual disturbance may occur.
C. Ankylosis between the vertebrae never occur.
D. Anterior spurs are the most common cause of symptoms.
E. Spontaneous remission and exacerbations are uncommon.

128. When a disk protrudes, it usually occurs in posterolaterally direction, because…


A. The annulus is thicker posteriorly.
B. The longitudinal ligament supports the spine in bending position.
C. The posterolateral aspect is not covered by the longitudinal ligament.

780
D. The forces of weight bearing in upright position are more on the posterior side of
vertebral body.
E. The forces of weight bearing in bending position are more on the posterior side of
vertebral body.

129. A person with unilateral metatarsalgia would be likely to have the following gait
abnormality:
A. Decreased heel strike.
B. Subtalar and genu varus.
C. Increase double support.
D. Ipsilateral hip flexion and abduction.
E. Increased step length on the contralateral side.

130. A patient ambulates with a trendelenburg gait. You suspect an injury to the…
A. Sciatic nerve.
B. Femoral nerve.
C. Obturator nerve.
D. Inferior gluteal nerve.
E. Superior gluteal nerve.

131. An upper brachial plexus lesion do not affect what movement?


A. Wrist flexion.
B. Elbow flexion.
C. Elbow extension.
D. Shoulder abduction.
E. Shoulder exorotation.

132. Word findings difficulty without other serious linguistic deficits, means…
A. Anomia.
B. Global aphasia.
C. Broca’s aphasia.
D. Wernicke’s aphasia.
E. Stuttering.

133. A 72 years old man presented with bilateral asymmetrical weakness of hand grip. On
examination there was lost of muscle bulnk on the volar aspect of the forearms and
impaired flexion on the distal interphalangeal joints of the finger. Impaired function of
which muscle is chiefly contributing to the weakness seen?
A. Flexor carpi ulnaris.
B. Flexor carpi radialis.
C. Flexor pollicis longus.
D. Flexor digitorum profundus.
E. Flexor digitorum superficialis.

134. Individual with SCI are consdered to be at risk for autonomic dysreflexia and
orthostatic hypotension if lesion on…
A. T6 or above.
B. T7 or above.
781
C. T8 or above.
D. T12 or above.
E. T12 or below.

135. The most cranial nerve involved in Miller Fisher Syndrome (variant of GBS) is…
A. Facialis nerve.
B. Abducent nerve.
C. Trigeminal nerve.
D. Occulomotor nerve.
E. Hypoglossus nerve.

136. You are evaluating a patient with a nerve palsy. During evaluation, the patient reveals
a positive Froment’s sign. Which nerve in injured?
A. Brachioradialis.
B. Axillaris.
C. Median.
D. Radial.
E. Ulnar.

137. A 35 years old male present with 3 weeks history of right side interscapular pain,
which began approximately 48 hours after he painted a ceiling. The pain occasionally
radiates into the right upper extremity, and is associated with paresthesia in the index and
middle fingers on the right hand. Base don this history alone, the most likely diagnosis is…
A. Fibromyalgia.
B. Brachial plexitis.
C. Cervical radiculopathy.
D. Carpal Tunnel Syndrome.
E. Thoracic Outlet Syndrome.

138. You are about to examine a child who sustained a fracture during a fall from swing
set. The serious complication following supracondylar fracture of the humerus in a child
is/are:
A. Gunstock deformity.
B. Tardy ulnar nerve palsy.
C. Non union of the fracture.
D. Volkmann’s ischaemic contracture.
E. Arm length discrepancy.

139. After a peripheral nerve lesion with axonotmesis nerve regrowth usually occurs at a
speed of:
A. 3-5 mm/day.
B. 1-3 mm/day.
C. 0.1-0.3 mm/day.
D. less than 0.1 mm/day.
E. more than 5 mm/day.

140. Which one is true according Nerve Conduction Studies (NCS)?


A. Insertion Activity or intrinsic electrical activity are the first to record in NCS.
782
B. NCS cannot be perfomed in cranial nerves, NCS should only be perfomed in peripheral
nerves.
C. NCS can be performed in three nerves types which is autonomic, motor, and sensory
nerve conduction testing.
D. NCS are done by pricking the needle through the skin and muscle then start stimulating
it through electrical impulses.
E. In NCS, you get direct information about the muscle and indirect information about the
nerves that supply the muscle that being test.

141. How many electrode (s) used in NCS and EMG?


A. One electrode which is active electrode.
B. Two electrodes which is active and ground electrode.
C. Two electrodes which is active and reference electrode.
D. Three electrodes which is active, ground, and polarized electrode.
E. Three electrodes which is active, ground, and reference electrode.

142. Which one is abnormal spontaneous potential that are generated specific at the level
of the muscle fiber?
A. cramps.
B. tremors.
C. fasciculations.
D. positive sharp waves.
E. myokymic discharge.

143. If patient has tarsal tunnel syndrome, one would likely to find which of the following
on nerve conduction studies?
A. Increased latency of the sural nerve at the ankle.
B. Increased latency of the tibial nerve at the ankle.
C. Decreased conduction velocity of the tibial nerve.
D. Decreased conduction velocity of the sural nerve.
E. Decreased conduction velocity of the peroneal (fibular) nerve.

144. The motor unit action potential below was taken using a monopolar needle in the
quadriceps muscle:

What can you determine about the amplitude of the motor unit?
e. Normal
f. Increased latency
g. Decreased amplitude

783
h. Increased amplitude
i. Mixed increased and decreased amplitude

145. On needle EMG testing, the muscle that would not be affected in a lesion to the
posterior cord is:
A. deltoid.
B. triceps.
C. pronator teres.
D. extensor carpi ulnaris.
E. extensor indicis propius.

146. What indicator does not have to be measures to diagnosed frailty syndrome?
A. dysphagia.
B. visual impairment.
C. fatigue.
D. gait speed.
E. weakness.

147. Exercise and activity modification prescription in type 2 DM with retinopathy


complication is…
A. High intensity aerobic exercise.
B. Resistance physical activity.
C. Head down activity.
D. Low intensity exercise.
E. Jumping exercise.

148. Which condition below is not a contraindication to the manipulation of the cervical
spine?
A. Osteoporosis.
B. Herniated cervical disk.
C. Acute cervical soft tissue injury.
D. Spasm of the paracervical muscle.
E. Fracture of the cervical vertebra.

149. What is the most reliable physical sign of Myofascial Trigger Point Syndrome?
A. A rope like nodular in the muscle and pattern of reffered pain.
B. Focal tenderness and pain recognition.
C. Vasomotor and temperature changes.
D. Limited of ROM.
E. Muscle spasm.

150. Para articular Heterotrophic Ossification in SCI patient most commonly occurs at the
following joint:
A. hip.
B. knee.
C. ankle.
D. elbow.
E. shoulder
784
NATIONAL BOARD EXAMINATION INDONESIAN PHYSICAL MEDICINE AND
REHABILITATION

JULY 3, 2020

CHOOSE THE MOST APPROPRIATE ANSWER

1. What type of urinary dysfunction mostly occurs after Traumatic Brain Injury?
A. Urinary tract infection
B. Urinary urgency/frequency
C. Overactive bladder (OAB)
785
D. Detrusor sphincter dyssynergia (DSD)
E. Uninhibited Bladder

2. One of the post stroke complications is Complex Regional Pain Syndrome (CRPS)
type I. Which one is the correct pathogenesis of this complication on Stroke condition?
A. Abnormal adrenergic sensitivity develops in injured nociceptors, and circulating
or locally secreted parasympathetic neurotransmitters
trigger the painful afferent activity
B. Cutaneous injury activates nociceptor fibers that promotes the peripheral pain-
signaling system that will evoke the pain
C. High-threshold mechanoreceptor input develops capacity to evoke pain
D. Sympathetic responses are unknown but may be related to central dysregulation of
nociceptive impulses
E. With time, afferent sympathetic fibers develop capacity to activate nociceptor
fibers

3. Patient, Male, 45 years’ old came to your clinic with stroke in sub-acute phase. On
the examination you found as he moved his hand to reach the stuff in front of him, the
synergy patterns were still predominating, but the spasticity is decreased. Based on
Brunnstrom stage, which stage has the patient achieved?
a. Stage 2
b. Stage 3
c. Stage 4
d. Stage 5
e. Stage 6

4. Post-polio syndrome is a diagnosis of exclusion. The hallmark of post-polio


syndrome is
A. Unusual fatigue
B. Cold intolerance
C. New weakness
D. History of falls
E. Myalgia

5. A 26-year-old woman with complete T6 paraplegia has managed her bladder since
the injury using intermittent self-catheterization. Three months after spinal cord injury she
develops leakage. What pharmacological agent is likely to be most useful in controlling
this?
a. Ephedrine (noradrenergic)
b. Bethanecol (cholinergic)
c. Prazosine (alpha-sympathetic blocker)
d. Oxybutinin (anticholinergic)
e. Baclofen (gamma aminobutyric acid)

6. A stroke patient shows signs of weakness and sensory deficits especially in the
Right Lower Extremity, while the Right Upper Extremity shows minimal disturbance. There
are also slight mental disturbance and aphasia. This condition most probably affecting the
following artery:
786
a. Posterior cerebral artery
b. Middle cerebral artery
c. Anterior cerebral artery
d. Superior cerebellar artery
e. Inferior cerebellar artery

7. Which is the correct statement of predictors of motor recovery in stroke patient?


a. Poor prognosis if there is severe distal spasticity
b. Poor prognosis if there is prolonged “flaccidity” period
c. Poor prognosis with measurable grasp strength by two weeks
d. Full or good recovery if the patient shows some motor recovery of the hand by
eight weeks (two months)
e. Good prognosis of recovery for useful hand function if there is complete arm
paralysis at onset

8. A 42 years old male, diagnosed with Traumatic Spinal Cord Injury AIS A with
neurological level T8. He is complaining about urine leakage during the day. The most
possible bladder dysfunction problem of this patient in voiding phase is:
a. Areflexive bladder
b. Neurogenic detrusor underactivity
c. Neurogenic acontractile detrusor
d. Normal detrusor function
e. Detrusor sphincter dyssynergia (DSD)

9. A 34 years old man with SCI AIS A T7 due to Transverse Myelitis since 6 months
ago. He is using self-CIC (Clean Intermittent Catheterization) and complaining of increasing
spasticity and appearance of urine leakage between the periods of catheterization. You
should be aware that the patient could have
a. Increasing fluid intake
b. Urinary tract infection
c. Autonomic dysreflexia
d. Heterotropic ossification
e. Vesicoureteral Reflux

10. Clinical signs of Parkinson’s disease are:


a. Resting tremor, spasticity, sensory deficit
b. Resting tremor, contracture, bradykinesia
c. Spasticity, weakness, dysphonia
d. Dysphagia, dysphonia, rigidity
e. Resting tremor, bradykinesia, rigidity

11. One that is not to be emphasized in the evaluation for rehabilitation of a person
with Parkinson disease is to
a. Assess the degree of rigidity and bradykinesia using the UPDRS to see how these
symptoms interfere with ADL
b. Analyze equilibrium by tandem walking
c. Assess fine motor task such as writing
d. Measure and record restriction in joint mobility
787
e. Assess the resting tremor

12. A 21 year-old female, single, paraplegia was referred as inpatient in the


rehabilitation ward due to T5 burst fracture. Stabilization was done and now was medically
stable. On physical examination it was found that vital signs and general health status
were within normal limits. What should you examine when you have to determine
whether the patient has a complete or incomplete lesion?
a. Anal sensation and voluntary anal sphincter contraction
b. Anocutaneal and bulbocavernous reflexes
c. Myotome and dermatome
d. Physiological reflexes
e. Pathological reflexes

13. Ms G, 28 years old was consulted to rehabilitation due to paralysis after a car
accident. The neurological examination came as follows:
Sensory: C2 – C6 = 2/2
C7 = 1/1
C8 – S4-5 = 0/0
Motor Score: C5 = 5/5
C6 = 5/Not Tested (severe wrist pain). C7 = 3/2
C8 = 0/0
T1 = 0/0
L2 - S1 = 0/0
VAC = No and DAP = No

Based on AIS Classification what is the functional diagnosis of this patient?


a. SCI AIS A with Sensory level C6 bilaterally, Motor level C5 bilaterally, Neurological
level C5, ZPP: Sensory C6, Motor C7.
b. SCI AIS A with Sensory level C6 bilaterally, Motor level C6 bilaterally, Neurological
level C6.
c. SCI AIS unable to determine with Sensory level C5 bilaterally, Motor level C5
bilaterally, Neurological Level C5, ZPP Sensory C6, Motor C7.
d. SCI AIS A with Sensory level C6 bilaterally, Motor level C7 right, unable to
determine on the left side, Neurological level unable to determine.
e. SCI AIS A with Sensory level C6 bilaterally, Motor level C7 right, unable to
determine on the left side, Neurological level unable to determine, ZPP: Sensory C7,
Motor C7.

14. Which condition is characterized by onset before age 20 years, gait ataxia, and
progressive paralysis?
A. Guillain-Barré syndrome (GBS)
B. Spinal muscular atrophy (SMA)
C. Friedreich’s ataxia
D. Becker’s muscular dystrophy (BMD)
E. Duchene muscular dystrophy (DMD)

15. Which of the following disorder is the weakness often relieved temporarily after
exercise or exertion?
788
A. Lambert-Eaton myasthenic syndrome (LEMS)
B. Botulism Intoxication
C. Myasthenia gravis (MG)
D. Amyotrophic lateral sclerosis
E. Multiple sclerosis

16. Which of following statement is not true about Sciatic Pain Syndrome?
a. Sciatic pain usually results from one or more nerves in the lumbar spine being
compressed or irritated
B. The symptoms may radiate into other parts of the body such as in the buttocks,
legs and feet
C. A disc prolapse (herniation) is almost always the cause of Sciatic Pain Syndrome in
geriatric patients
D. Surgery is only necessary in a small percentage of patients
E. The strategies to manage the pain include cognitive behavioral therapy and
mindfulness-based programs.

17. A patient seen in the occupational unit is told to draw a clock. After examining his
illustration, you note that all the numbers of the clock are drew and squeezed to the right
side. You suspected that this patient has a:
A. Homonymous Hemianopsia
B. Bitemporal Hemianopsia
C. Quadrantanopia
D. Visuospatial neglect
E. Cortical blindness

18. If an individual has inability to initiate speech and respond to speech with anything
other than simple word, this individual has:
A. Non fluent aphasia
B. Fluent aphasia
C. Anomic aphasia
D. Transcortical aphasia
E. Disruptive aphasia

19. Tic doloreaux is caused by pathology of which cranial nerve?


A. II
B. III
C. IV
D. V
E. VI

20. An infarct in the lower division of the left middle cerebral artery division would be
associated with which type of aphasia?
A. Broca
B. Global
C. Wernicke
D. Transcortical motoric
E. Transcortical sensoric
789
21. Which finding is associated with a more favourable prognosis in Amyotrophic
Lateral Sclerosis (ALS)?
A. Female sex
B. Young age
C. Bulbar involvement presentation
D. Short time period from symptoms to diagnosis
E. Predominance of lower motor neuron findings on EMG studies

22. A 21-year-old man is evaluated in your spinal cord injury clinic 12 months after C2
complete spinal cord injury requiring full-time mechanical ventilation. Your
recommendation is
a. Aggressive frog breathing exercise
b. Aggressive diaphragmatic strengthening exercise
c. Aggressive breathing control exercise
d. Initiating a weaning protocol by slowly decreasing tidal volume
e. An electro-diagnostic study to evaluate for a phrenic nerve pacemaker

23. Which of the following statement about Rheumatoid Arthritis (RA) is true?
A. Female:male ratio = 1 : 3
B. Affects the synovial membrane of multiple joints
C. The joints mostly affected are shoulders and hip
D. DIP subluxation is the hand deformity in the late stage of RA
E. Systemic inflammation is not the risk factor for osteoporosis in RA

24. The following is true about ostearthritis (OA):


A. Osteoarthritis is a systemic bone disorder
B. Genetic predisposition is not the risk of OA
C. The prevalence of OA is higher in men than women
D. Characterized by non progressive articular cartilage damage
E. Knee OA is heterogeneous disease, involving cartilage, subchondral bone,
menisci and periarticular soft tissues.

25. Medical rehabilitation after Total Knee Replacement:


A. Isotonic quadriceps exercise in the first week after surgery
B. The goal after 2 WEEks is 0-90° ROM of the affected knee
C. Early mobilization is not recommended
D. The use of CPM until 4 weeks
E. Gait training after 3 weeks

26. Which statement is correct concerning patient with osteoarthritis?


A. Exercise contributes to pain reduction
B. Reduced aerobic capacity is difficult to reserve
C. Knee orthosis and walking aids are not effective to reduce pain
D. High intensity progressive resistance exercises causes the disease to progress
E. Group exercise programs are more effective than individual programs in reducing
disability

790
27. Return to sport phase focused to the component:
A. Strengthening all muscles
B. Stretching exercise
C. Sport specific skill
D. Balance training
E. ROM exercise

28. The increase in strength noted 1 WEEk after beginning progressive resistance
exercises in a specific group of muscles is most likely due to:
A. Increased muscle protein
B. Increased recruitment of motor units
C. Increase in muscle oxidative capacity
D. Conversion of fast to slow twitch fiber
E. Decreased activity of Golgi tendon organs

29. The muscle that has the important role in push-off is


A. Iliopsoas
B. Hamstring
C. Quadriceps
D. Triceps surae
E. Tibialis anterior

30. Which of the following treatment is more effective considering of the fast recovery
for stenosing flexor tenosynovitis?
a. Resting and immobilization
b. Microwave Diathermy
c. Injection into the tendon sheath
d. Repetitive gripping exercise
e. Ultrasound therapy

31. A 40-year-old woman presents with history of progressive pain over the proximal
medial aspects of tibia. The pain is said to be increased with running up hills. The most
likely diagnosis is:
A. Pes anserinus bursitis
B. Prepatellar bursitis
C. Patellar tendon synovitis
D. Tibial-collateral ligament strain
E. Cartilage damage to the medial aspect of the knee

32. Sacroiliitis is always seen in:


a. Reiter’s syndrome
b. Intestinal arthropathy
c. Psoriatic arthropathy
d. Ankylosing spondylitis
e. Juvenile chronic arthropathy

33. Which statement is correct regarding the iliotibial band friction syndrome?
A. Excessive supination secondary to soft sole on shoe sole is causative
791
B. The most severe pain is directly over medial joint line of the knee
C. Treatment consists of stretching and massage of the ITB
D. The Ober test has not been found
E. Mc Murray test has been found

34. Which one is not a goal of the acute phase of Rehabilitation in sport injuries?
a. Retard muscle atrophy
b. Reduce pain and inflammation
c. Create conditions for tissue healing
d. Reestablish full range of motion of the joint around the injury
e. Establish neuromuscular control of the joint in neutral positions

35. Which one is not the risk factor of pressure ulcers?


A. Fecal and urinary incontinence
B. Decrease skin temperature
C. Pressure, shear and friction
D. Hypoalbuminemia
E. Elderly

36. A 25-year-old man presents to you with 2 days of knee pain and swelling after
falling off the ladder. The swelling began immediately after the fall. His neurologic
examination is normal. He is unable to fully extend or flex the knee because of pain and
swelling. He is ambulating with antalgic gait limp. Your recommendations include:
a. Icing, NSAID drug, knee immobilization, crutches, activity modification, x- rays,
recheck in 5 days
b. Icing, narcotics, TENS, referral to an Orthopedic Surgeon
c. NSAID drugs, x-rays, return to work, recheck in 6 weeks
d. Icing, crutches, x-rays, recheck in 2 WEEks
e. Refer to an Orthopedic Surgeon

37. An 18-year-old student sustained multiple injuries, including a proximal left radius
fracture, in an automobile accident. He is unable to extend the digits of his left hand. You
would expect to find fibrillation potentials in which of the following muscles if radial nerve
was injured near the fracture.
A. Triceps, supinator, extensor indicis
B. Anconeus, brachioradialis, extensor digitorum communis
C. Brachioradialis, extensor carpi radialis longus, extensor indicis
D. Anconeus, extensor digitorum communis, extensor pollicis longus
E. Extensor digitorum communis, extensor indicis, extensor pollicis longus

38. Which of the following statement is not true regarding diabetic foot care?
a. In the presence of foot deformities, the physician should consider customs shoe
insert and or footwear to distribute pressure evenly over the foot
b. It is advisable that patient minimizes or eliminates weight bearing during the
healing period of skin infections.
c. Patients should be encouraged to routinely visit a podiatrist for nail and general
foot care
d. Tennis or running shoes should be avoided in diabetic patients
792
e. In the presence of sensory disturbance SWD is contraindicated

39. De Quervain’s tenosynovitis affects which two tendons?


A. Abductor pollicis brevis and extensor pollicis brevis
B. Abductor pollicis brevis and extensor pollicis longus
C. Abductor pollicis brevis and abductor pollicis longus
D. Abductor pollicis longus and extensor pollicis brevis
E. Abductor pollicis longus, extensor carpi radialis

40. Which statement below is correct for a patella-femoral pain syndrome?


A. Osteoarthritis is one of the etiological factor
B. The pain often be felt while running or jogging
C. Also referred to as posteromedial knee pain syndrome
D. Should always be associated with laxity of ligament around the knee
E. Is characterized by pain at the front or middle edge of the knee or under the knee
cap

41. A national football player twists his knee when he attempts to tackle an oncoming
player. Examination reveals no medial lateral laxity or joint line tenderness. The posterior
drawer tests and pivot shift results are negative, but the Lachman test’s result is positive.
What is the most likely diagnosis?
A. MCL injury
B. LCL injury
C. ACL injury
D. PCL injury
E. PCL-MCL injury

42. Which following statement is correct about tennis elbow in the athlete?
A. Return to sports after the patient is pain free and the involved extremity’s strength
50% of that on the contralateral side
B. Tenderness is localized to 2 to 5 cm proximal and anterior to the lateral epicondyle.
C. Symptoms are reproduced with wrist resisted flexion while elbow is extended
D. Acute-on-chronic pain is suggestive of a frank rupture of the extensor origin
E. The extensor carpi ulnaris is most often involved

43. 24-year-old marathon runner reports lateral knee pain after hill training.
Examination reveals no effusion; and Ober’s test is positive. What is the most likely
diagnosis?
A. Biceps tendinitis
B. Lateral meniscal tear
C. Popliteus tenosynovitis
D. Peroneal nerve entrapment
E. Iliotibial band friction syndrome

44. Which of following statement regarding ankle sprain is not correct?


A. The most common type of ankle sprain is an inversion injury or lateral ankle sprain
B. Grade 3 lateral ankle sprain means if there is a complete tear or rupture of the
ligaments that caused instability
793
C. The immediate treatment’s goal for acute ankle sprain is to decrease pain and to
prevent limitation of joint of motion
D. All activities that might increase swelling in the first 24 hours should be avoided
E. In mild injury, range of motion and strengthening exercises could begin within
48-72 hours

45. A martial arts player complaints of back knee pain after kicking his opponent.
Examination of the knee demonstrates pain during varus stress test. What is the most
likely diagnosis?
a. Medial collateral ligament injury
b. Lateral collateral ligament injury
c. Posterior cruciate ligament injury
d. Anterior cruciate ligament injury
e. Strain of popliteus muscle

46. Which following statements about muscles activities during gait cycle is true?
A. During stance phase gluteal muscles, tensor fascia lata and erectors spinae act as
stabilizer
B. During stance phase only gluteus maximus muscle act as stabilizer
C. During stance phase only gluteus maximus and medius muscles act as stabilizer
D. During stance phase gluteal muscles and erector spinae act as stabilizer
E. During stance phase gluteal muscles and tensor fascia lata act as stabilizer

47. Which of the following muscle is not an erector spinae group?


A. Iliocostalis thoracis
B. Longissimus thoracis
C. Spinalis thoracis
D. Iliocostalis cervicis
E. Semispinalis thoracis

48. Which of following treatments is not true for enhancing bone healing process after
fracture?
A. Short period of immobilization
B. Ultrasound treatment in subacute phase
C. Electrical stimulation
D. Axial loading or gradually weight bearing after several weeks
E. Exercise beyond the range of pain

49. The Unbalanced force that gives rise to an action against an individual's body or
object when the individual makes or modifies the movement, is called
A. force motion
B. force time
C. coordination continuum
D. optimal projection
E. spin or rotation

794
50. At the structural end of the neuromuscular control process are the functional units
of the control of muscles called
A. motor unit
B. muscle spindle
C. musculotendinous junction
D. myoglobin
E. muscle fiber

51. Which of following statements of the use of contrast bath is not true?
A. to reduce swelling around injuries
B. to improve muscle recovery following exercise
C. to enhance bone healing process
D. to decrease pain and control inflammation
E. to improve mobility

52. One that is not a physiological effect of electrical stimulation is........


A. response is not always elicited
B. if adequate, the stimulus produces a contraction of muscle directly
C. As the duration of stimulus increases, the strength of the stimulus increase
D. Accommodation to a change in rate of stimulus is better in nerve than muscle
E. Will not cause increase in muscle strength

53. Which one is not therapeutic heat?


A. Radiant heating agent
B. SWD
C. US Diathermy
D. Ultraviolet radiation
E. Phonoporesis

54. The typical sequence of sensations in response to cryo-therapy is:


A. Intense cold – burning – aching – analgesia – numbness
B. Intense cold – aching – burning – analgesia – numbness
C. Intense cold – analgesia– aching – burning – numbness
D. Intense cold – numbness – burning – aching – analgesia
E. Intense cold – burning – aching – numbness – analgesia

55. A Balanced forearm orthosis (BFO) is a shoulder-elbow-wrist-hand orthosis


(SEWHO). The device can be helpful to support the forearm and arm against gravity and
allows patients with weak shoulder and elbow muscles to move the arm
horizontally and flex the elbow to bring the hand to the mouth. One that is not a
requirement for prescribing the device is:
A. Some residual muscle strength (MMT at least poor or grade 2)
B. Preserved ROM of the shoulder and elbow joints
C. Lack of coordination of elbow flexion
D. Adequate trunk stability and balance
E. Adequate endurance in a sitting position

56. One that is not the indication for use of an AFO to improve a patient’s gait, is…..
795
A. ”Foot drop” at heal strike due to weak ankle dorsiflexors
B. ”Foot drop” passive plantar flexion in swing phase
C. Mediolateral instability at the ankle
D. Weak push-off at late stance phase
E. “Knee buckling” at beginning of stance phase

57. A patient came with neck pain caused by traumatic accident on wheel. The
examination reveals soft tissue injury and cervical ligamentous sprain. You prescribe a:
A. Soft cervical collar
B. Thomas collar / hard cervical collar
C. Philladelphia collar
D. Sterno-occipital mandibular immobilizer (SOMI)
E. Minerva cervico-thoracic orthosis

58. To prescribe an Ankle Foot Orthoses (AFO), which of the followings is not a
consideration?
A. Metal or plastic AFO can be used effectively to control ankle motio
B. Metal AFO are relatively contra indicated in children because the weight of the
brace can cause external tibial rotation
C. AFOs should provide mediolateral stability as a safety feature
D. AFOs can not stabilize the knee during gait
E. AFOs may help ankle dorsiflexion

59. A systematic approach to collecting patient data that allows the practitioner to
evaluate the frail older adult health status and functional impairments in multiple areas or
domains is
A. holistic geriatric assessment
B. frailty assessment
C. functional geriatric assessment
D. comprehensive geriatric assessment
E. holistic mobility assessment

60. Which of the following is not included to general health of geriatric assessment?
A. Physical medical condition
B. Mental health condition
C. Functioning
D. Social circumstances and environment
E. Formal Job Description

61. Help elderly patient become independent as maximum as possible, and to prevent
from losing the ability to function is
A. short term goals
B. immediate goals
C. long term goals
D. care plan
E. discharge planning

796
62. Basic components of hierarchy of physical function in the elderly are
A. strength and endurance
B. balance and activity daily living
C. flexibility and activity daily living
D. coordination gross motor and specific physical movements
E. coordination line motor and gross motor

63. Which of following tests are commonly used for assessment of cognitive function in
elderly?
A. Mini Mental State Examination
B. Mild Cognitive Impairment
C. Geriatric Depression Scale
D. Geriatric Depression Scale and Mini Mental State Examination
E. Mini Mental State Examination and Mild Cognitive Impairment

64. Assessment of prognosis in cases of elderly rehabilitation include


A. ad vitam and ad functionam
B. ad vitam and ad sanationam
C. ad vitam, ad sanationam, ad functionam
D. ad sanationam and ad functionam
E. dubia ad bonam dan dubia ad malam

65. In discharge planning in the elderly requires several meetings, including


A. management meeting, geriatric meeting, family meeting, case conference
B. geriatric meeting, family meeting, case conference
C. geriatric meeting, case conference
D. geriatric meeting, family meeting
E. management meeting, geriatric meeting, family meeting

66. Living situation in elderly covers two dimensions:


A. The safety of the home environment and public environment
B. The safety of the home environment and the adequacy of the patient’s
accessibility to the needed personal and medical services
C. The adequacy of the patient’s accessibility to the needed personal and medical
services
D. The safety of the home environment and medical services
E. The safety of the home environment and the adequacy of the patient’s accessibility
to the needed personal service

67. The goals of care in elderly patient are:


A. Social and functional goals are priority over the health related goal
B. Regain the previous health status, and can attend the family events
C. Independent in ADL and can choose the appropriate decision
D. Regain the previous health status and can choose the appropriate decision
E. Completing ADLs without help

68. Children two years of age have stereotypic and ritualistic behavior with impairment
in social interaction, communication and imaginative play, are:
797
A. Down syndrome
B. ADHD
C. Autism
D. Fragile x syndrome
E. Mental retardation

69. This reflex is best elicited by a sudden dropping of the baby’s head in relation to its
trunk.
A. Righting Reflex
B. Landau Reflex
C. Moro Reflex
D. Vertical Suspension
E. Extensor thrust

70. Lumbar lordosis in Duchene muscular dystrophy is ……..


A. Compensation of hip extensor weakness
B. Compensation of hip flexor weakness
C. Compensation of hip adductor weakness
D. Compensation of hip abductor weakness
E. Compensation of hip endorotator weakness

71. Primary forefoot contact in Duchenne muscular dystrophy is…….


A. Compensation of hip extensor weakness
B. Compensation of knee extensor weakness
C. Compensation of ankle plantar flexion weakness
D. Compensation of hip flexor weakness
E. Compensation of knee flexor weakness

72. Which statement about Becker Muscular Dystrophy is true?


A. It is an Autosomal recessive inheritance
B. The disorder is on gene Xp21
C. Has the same incidence as Duchenne muscular dystrophy
D. Becker Muscular Dystrophy patients have distribution of weakness similar to
those with distal spinal muscular atrophy
E. Creatine phoshokinase is usually normal

73. Which statement about motor development of a child with spina bifida is true?
A. Children with high lumbar lesions pull to stand and cruise near the expected age
B. Lower lumbar lesions often require devices, such as a reciprocal gait orthosis
C. Children with mid-lumbar lesions can usually sit with some delay and increased
lordosis
D. The level of neurologic impairment did not influence the expectations for
functional outcome and prognosis
E. Children with lumbar lesions require assistive device for passive standing, usually
started at 12 to 18 months

74. Which secondary deformity does not happen in children with torticollis?
a. Contralateral occipital flattening
798
b. Flattening of ipsilateral face
c. Scoliosis cervical
d. Arachnoidactyly
e. Plagiocephaly

75. Which of the following statement about Brachial plexus injury in children is true?
A. is never caused by delivery process of the baby
B. Violent upward pull of the shoulder, causing damage to the upper trunk
C. Horner’s syndrome can be associated with injury of the C8 and T1 roots
D. Erb’s paralysis typically result from a fall onto a hyper-abducted shoulder
E. Sudden traction to the neck, causing injury to the lower trunk of the brachial plexus

76. A variety of problems may cause language and speech delays in children. Which of
following is sign of delayed development?
A. Baby of 4 months begins to babbling and try to imitate sounds
B. Baby of 9 months start to call “mama” to his/her mother
C. Baby of 9 months can show back and forth sharing sounds, smile or facial
expressions
D. Child of 1 year can point to objects or pictures and follow simple instructions
E. Child of 2 years can speak sentence with at least 2-3 words

77. Considered to the normal development of lower limb in children, which of


following finding is ROM & bony abnormality?
A. Bow leg in children less than 2 years old
B. Flat feet physiological in 5 years old
C. Toeing in with tibial torsion in toddler
D. W sitters children, who walk with patella to be medially deviated
E. Metatarsal deviation medially in 5 years old

78. Child’s gait pattern change and progress until 7 years old. It is considered abnormal
if :
A. High guard position of the arm at 2 years old
B. Foot flat pattern at 1 year old
C. Stride length decreases as they grow
D. Cadence decreases as they grow
E. Base of support narrowing

79. In patient with myelomeningocele at the level of lesion L3-4 should


A. have iliopsoas function
B. have quadriceps function
C. have hamstring function
D. have extensor hip function
E. walk independently without forearm crutches

80. Juvenile Rheumatoid Arthritis is....


A. Joint inflammation less than 6 weeks
B. In children more than 16 year-old
C. Any joint can be affected and may limit the mobility
799
D. It is not possible to be accompanied by eye problems
E. All patients have positive Rheumatoid factor

81. The method frequently used on CP which bases treatment on activation of postural
development and equilibrium reaction to guide normal development is :
A. Bobath Method
B. Deaver method
C. Vojta method
D. Phelp method
E. PNF method

82. The statement that is not associated with the role of exercise in obesity children :
A. Prevention of becoming overweight and obese
B. Reducing the risk of obesity in adulthood
C. Engagement in physical activity
D. Increasing of active behaviors
E. Protect against chronic disease

83. Which of the following types of CP is prone to develop contractures?


A. Ataxic
B. Spastic
C. Flaccid
D. Athetoid
E. Choreiform

84. Protective and postural reflexes (advanced postural reactions) is …………….


A. plantar grasp
B. foot hand placement
C. head and body righting
D. symmetric tonic neck reflex
E. asymmetric tonic neck reflex

85. Bowleg in children is caused by growth retardation of the medial and posterior part
of the proximal tibial epiphysis due to abnormal weight bearing stress and compression
force on the knees. This disease is known as ………..
a. Genu varum
b. Blount’s disease
c. Ricket’s disease
d. Ollier’s multiple enchondromatosis
e. Arthrogyposis multiplex congenital

86. A 4-year-old boy was brought into your office because his mother has noticed that
he has difficulty getting up from a seated position on the floor while playing with his toys.
On physical exam, there was increased gastrocnemius calf circumference bilaterally. You
think the child may have dystrophic myopathy. The maneuver the child performs to assist
him in standing was caused by……………..
A. distal leg weakness
b. distal arm weakness
800
c. proximal leg weakness
d. proximal arm weakness
e. spinal muscle weakness

87. Early treatment for infant with diagnosis CTEV is…………….


a. Ankle foot orthosis
b. Dennis Brown splint
c. Outflare orthopedic shoes
d. Surgery for soft tissue release
e. Stretching and serial plaster correction

88. Based on acquisition of speech and language skills, in what age does the child have
the ability to first words emerge and responds to simple requests ?
a. 3 months
b. 6 months
c. 9 months
d. 12 months
e. 15 months

89. Crouched gait in Cerebral Palsy patients , is caused by……………


a. abductor weakness
b. increased adductor tone
c. increased iliopsoas tone
d. spasticity of the gastrocnemius muscles
e. hamstring, psoas, rectus femoris and gastrocnemius contracture

90. Not a correct statement for Hemophilia is ........


A. After repeated hemarthosis, irritating hemosiderin deposits occurs in the synovial
lining cells, the synovium becomes largely replaced by fibrous
tissue
B. Most patients with hemophilia in whom joint problems develop have factor VIII or
IX deficiency
C. The prevalence of joint contracture in the severe from the disease between 50-95%
D. Treatment for acute hemarthosis using ROM and deep heating
E. The knees, elbows and ankles are the joints most often affected

91. Which is the absolute contraindication of Cardiac Exercise Training for patient with
chronic heart disease?
A. Blood glucose 150 - 200 mg/dl
B. Hemoglobin 10 – 12 g/dl
C. Pulmonary Hypertension
D. Diastolic pressure > 115 mmHg
E. Systolic preassure > 140 mmHg

92. What is the stratification risk for the patient that feels angina and shortness of
breath when he plays soccer and the echocardiography examination finding of ejection
fraction is 45% ?
A. Low risk
801
B. Moderate risk
C. Low to moderate risk
D. Unstratified risk
E. High risk

93. Which of the following condition is not a potential contraindication for entry into a
cardiac exercise programme?
A. Unstable angina
B. Resting diastolic blood pressure > 100 mmHg
C. Aortic stenosis
D. Active pericarditis
E. Well-controlled diabetes

94. Which of following is the most correct exercise prescription for patient with
chronic heart failure?
A. Supervised, aerobic exercise, duration can be started from 30 minutes
increased gradually, continuous intensity 70 - 90% VO2Max 3–5 times per week
B. Supervised, aerobic exercise, duration can be started from 10 minutes increase
gradually, continuous intensity 50 - 70% VO2Max, 3–5 times per week
C. Ground walking exercise activity 5 mph, start with 15 minutes duration, 3-5 times
per week
D. Supervised, competitive sport, intensity start with 60% VO2Max 3–5 times per
week
E. Aerobic ergocycle exercise, gradual intensity as tolerated 60 minutes 3–5 times per
week

95. Which of the following in the most correct step of cardiac rehabilitation for patient
with chronic heart disease?
A. Giving exercise, medication evaluation, muscle strength assessment,
functional capacity assessment, psychosocial assessment, environment
assessment.
B. Patient risk stratification assessment, functional capacity assessment, activity and
vocational evaluation, psychological profile assessment, medication evaluation, exercise
program, evaluation of cardiac rehab program.
C. Functional capacity assessment, medication evaluation, psychosocial assessment,
giving exercise, environment assessment.
D. Medication, evaluation, muscle strength assessment, muscle endurance
assessment, exercise program, psychological profile assessment.
E. Functional capacity assessment, Patient risk stratification assessment, medication,
evaluation, evaluation of cardiac rehabilitation program.

96. Studies have shown that the risk reduction with cardiac rehabilitation after a
Myocardial Infarction (MI) is ......
A. 1%
B. 5%
C. 10 %
D. 15 %
E. >20 %
802
97. What is the expected outcome of effective exercise training in a pulmonary
rehabilitation program for patients with chronic obstructive pulmonary disease (COPD)?
A. Reduced need for supplemental oxygen
B. Reduced dyspnea
C. Improved radiologic status of the lung fields
D. Slowed loss of forced expiratory volume
E. Improved lung anatomy

98. During an exercise tolerance test, what percentage of the maximum heart rate is
the usual target?
A. 60
B. 65
C. 75
D. 85
E. 95

99. A patient with restrictive lung disease and mild scoliosis from Duchenne muscular
dystrophy (DMD) develops nocturnal hypoventilation. What is the initial treatment?
A. Incentive spirometry every four hours
B. Tracheostomy and nocturnal mechanical ventilation with positive pressure
C. Nocturnal supplemental low-flow oxygen
D. Nocturnal noninvasive positive airway pressure and short periods of daytime
hyperinsufflation
E. Breathing control

100. Which cardiorespiratory outcome would you expect from an elderly person
participating in an aerobic program?
a. A decrease in VO2MAx but an increase in walking speeds
b. No change in tolerance during activities of daily living
c. Improvement in VO2MAx
d. No change in stroke volume and a reduction in systemic vascular resistance
e. Improvement of Quality of Life

101. Other changes in the cardiopulmonary system those results from exercise:
A. Decrease vital capacity
B. Decreased systolic blood pressure
C. Increased resting heart rate
D. Increased oxygen saturation
E. Increased cardiopulmonary efficiency

102. The volume of gas that remains in the lungs at the end of a maximum expiration
is……..
A. Tidal volume
B. Residual volume
C. Total lung capacity
D. Functional capacity
E. Functional residual capacity
803
103. When patients are comfortable at rest, but ordinary physical activity results in
dyspnea, fatique, palpitations, pain, or angina, according to NYHA is stage ….
A. I
B. II
C. III
D. IV
E. V

104. Based on a comprehensive patient assessment, the goals of long-term


management for the patient with chronic bronchitis may include the following is ….
A. Reduce ventilation
B. Reduce alveolar ventilation
C. Reduce secretion clearance
D. Reduce the work breathing
E. Reduce perfusion matching and gas exchange

105. The best predictor of mortality of COPD and it responds well to intervention such
as lung volume reduction is
A. Force expiratory volume in 1 second
B. BODE Index
C. Degree of hypoxemia
D. Presence hypercapnia
E. Presence hypercarbia

106. Which of the following parameters most closely correlates with oxygen
consumption?
A. Oral temperature
B. Rectal temperature
C. Pulse rate
D. Respiratory rate
E. Blood pressure

107. For patients with mild cardiac insufficiency, which percentage of maximal heart
rate could be used for exercise under closed monitoring with wireless ECG?
A. 40 to 50 percent of maximal heart rate
B. 50 to 60 percent of maximal heart rate
C. 60 to 70 percent of maximal heart rate
D. 70 to 85 percent of maximal heart rate
E. More than 85 percent of maximal heart rate

108. What is the Intrinsic factor that does not contribute to the development of
tendinitis?
A. Age
B. Genetic predisposition
C. Poor training technique
D. Muscle imbalance
E. Anatomic malaligment
804
109. Not a characteristic of closed kinetic chain exercise is……
A. Distal end fixed
B. Squatting
C. More shear stress
D. Stimulate the proprioceptive system
E. Leg press

110. What Muscle does not need strengthening in preparation for crutch walking ?
A. Triceps muscle
B. Latissimus dorsi muscle
C. Pectoralis major muscle
D. Quadriceps muscle
E. Serratus anterior muscle

111. Contraindication of diathermy is …..


A. Allergic to cold
B. Epilepsy
C. Malignancy
D. Spasticity
E. Joint contracture

112. After Anterior Cruciate Ligament Reconstruction, when is commonly the time when
the patient can bear the full body weight in walking?
A. One day after surgery
B. One week after surgery
C. Two weeks after surgery
D. Three weeks after surgery
E. Four weeks after surgery

112 a. A soccer professional athlete came to the Sport Injury Rehabilitation for diagnostic
testing of quadriceps muscle strength post Arthroscopy for Anterior Cruciate Ligament
injury. When is the exact time to do the diagnostic testing ?
A. 1st week
B. 4th week
C. 7th week
D. 10th week
E. 13th week

113. Which is the incorrect statement of splinting to prevent contracture in burn


management?
A. Static splinting is used mainly to counteract the deforming position
B. Splints can provide the stress needed to orient the collagen being deposited during
the early stages of wound healing as well as maintain joint alignment, or
to increase ROM
C. Serial casting can result in flatter, softer and more supple tissue
D. Splints can prevent contracture without exercise
805
E. Splints are placed avoiding compression to the burnt area

114. You notice that your patient has a varus deformity at the ankle when he walks.
What adjustments to his shoe can you suggest so that his gait is more stable?
A. Cushioned heel
B. Medial heel flare
C. Lateral heel flare
D. Heel lift
E. Insole

115. The best ankle-foot orthosis (AFO) to be used for someone with a foot drop but has
a weak quadriceps, is ........
A. AFO with posterior foot stop
B. Solid AFO
C. AFO with posterior stop with a Klenzak joint
D. Solid AFO with anterior and posterior stops
E. Leaf Spring AFO

116. A patient presents with wrist pain. He has pain when gripping a cup handle. You
notice a positive Finkelstein’s test. What brace would you recommend for this patient?
A. Volar wrist splint
B. Radial nerve palsy splint
C. Resting hand splint
D. Thumb spica splint
E. Knuckle Bender

117. A person with transfemoral amputation has an abnormal prosthetic gait of the
trunk swaying laterally toward the involved side during midstance. This occurs most
commonly due to the hip ......
A. adductor weakness
B. abductor weakness
C. flexion contracture
D. extension contracture
E. internal rotator weakness

118. In which prehensile hand function could a myoelectric prosthesis hand be used?
A. Lateral
B. Power
C. Spherical
D. Three-jaw chuck
E. Pinching

119. What is the test to check for contraction of the iliotibial band?
A. Thomson’s test
B. Painful arc test
C. Ober test
D. Yergason’s test
E. Thomas test
806
120. Which lower extremity amputation requires the greatest energy expenditure for
ambulation using prosthesis?
a. Unilateral syme
b. Hemipelvectomy
c. Bilateral above knee
d. Unilateral hip disarticulation
e. Above knee and below knee on the other side

121. Which of the following best defines an eccentric contraction?


A. Muscle lengthening contraction
B. Muscle shortening again resistance
C. Muscle shortening without resistance
D. Muscle does not change length during contraction
E. A contraction generating little force

122. An example of closed kinetic chain exercise for the quadriceps is ……


A. A partial squat
B. An isometric quadriceps set
C. Knee extension on a stacked-weight machine
D. Knee extension on an isokinetic machine (eg, Cybex)
E. Jumping jack exercise

123. Which exercise has the greatest effect on bone formation?


A. Cycling
B. Swimming
C. Weight lifting
D. Tai Chi stretching
E. Yoga

124. Which of following exercise in occupational therapy cannot be used for improving
range of motion of frozen shoulder?
A. Tictactoe board
B. Macrame
C. Hammer board
D. Nine hole peg board
E. Arm skate board

125. The Adson test was positive in patient with Thoracic Outlet Syndrome. This finding
suggested to the cause of which condition?
A. Hypertrophy of the splenius cervicis muscle
B. Hypertrophy of the m scalenus posterior
C. Hyperthropy of the m pectoralis mayor
D. Fracture of the clavicle
E. Cervical rib

807
126. A 25-year old man has had heel pain for 1 month. He runs approximately 40 km
weekly. The pain is most severe when he first stands up and walks after sitting or lying
down. Palpation of the medial plantar surface of the heel increases the pain. He walks with
a limp. Which of the following is the most likely cause of the pain?
A. Heel spur
B. Achilles tendinosis
C. Impingement of the posterior sacral nerve root.
D. Inflammation of the origin of the plantar fascia.
E. Articular cartilage degeneration of the calcaneo cuboid joint

127. Which following statement is correct regarding the degenerative joint disease of
the cervical spine?
A. Radicular pain is very rare
B. Vertigo and visual disturbance may occur
C. Ankylosis between the vertebrae never occurs
D. Anterior spurs are the most common cause of symptoms
E. Spontaneous remission and exacerbations are uncommon
128. When a disk protrudes, it usually occurs in the posterolaterally direction,
because…………
A. the annulus is thicker posteriorly
B. the longitudinal ligaments support the spine in bending position
C. the posterolateral aspect is not covered by the longitudinal ligaments
D. the forces of weight bearing in upright position are more on the posterior side of
the vertebral body
E. the forces of weight bearing in bending position are more on the posterior side of
the vertebral body

129. Which of the following is not the treatment for patient with metatarsalgia?
A. Metatarsal pad in short term
B. Some inflammation reducing modalities such laser therapy
C. Toe waving-lift spread reach exercise
D. Ankle rocker retraining
E. Hip and knee muscles strengthening exercises

130. A patient ambulates with a Trendelenburg gait. You suspect an injury to the ..
A. sciatic nerve
B. femoral nerve
C. obturator nerve
D. inferior gluteal nerve
E. superior gluteal nerve

131. An upper brachial plexus lesion does not affect what movement ?
A. wrist flexion
B. elbow flexion
C. elbow extension
D. shoulder abduction
E. shoulder exorotation

808
132. Word finding difficulty without other serious linguistic deficits, means …………..
A. Anomia
B. Global aphasia
C. Broca’s aphasia
D. Wernicke’s aphasia
E. Stuttering

133. A72-year-old man presented with bilateral asymmetrical weakness of hand grip.
On examination there was lost of muscle bulk on the volar aspect of the forearms and
impaired flexion of the distal interphalangeal joints of the fingers. Impaired function of
which muscle is chiefly contributing to the weakness seen?
A. Flexor carpi ulnaris
B. Flexor carpi radialis
C. Flexor pollicis longus
D. Flexor digitorum profundus
E. Flexor digitorum superficialis

134. Individuals with SCI are considered to be at risk for autonomic dysreflexia and
orthostatic hypotension if lesion on …………..
A. T4 or above
B. T6 or above
C. T8 and above
D. T12 and above
E. T12 and below

135. The most cranial nerve involved in Miller Fisher syndrome ( variant of GBS ) is
A. facialis nerve
B. abducent nerve
C. trigeminal nerve
D. occulomotor nerve
E. hypoglossus nerve

136. You are evaluating a patient with a nerve palsy. During evaluation, the patient
reveals a positive Froment’s sign. Which nerve is injured ?
A. Brachioradialis
B. Axillaris
C. Median
D. Radial
E. Ulnar

137. A 35-year-old male presents with a 3-week history of right-sided interscapular pain,
which began approximately 48 hours after she painted a ceiling. The pain occasionally
radiates into the right upper extremity, and is associated with paresthesias in the index
and middle fingers of the right hand. Based on this history alone, the most likely diagnosis
is …………..
A. Fibromyalgia
B. Brachial plexitis
C. Cervical radiculopathy
809
D. Carpal Tunnel Syndrome
E. Thoracic Oulet Syndrome

138. You are about to examine a child who sustained a fracture during a fall from a
swing set. The serious complication following supracondylar fracture of the humerus in a
child is/are :
A. Gunstock deformity
B. Tardy ulnar nerve palsy
C. Nonunion of the fracture
D. Volkmann’s ischemic contracture
E. Arm length discrepancy

139. After a peripheral nerve lesion with axonotmesis nerve regrowth usually occurs at
a speed of :
A. 3-5 mm/day
B. 1-3 mm/day
C. 0.1-0.3 mm/day
D. less than 0.1 mm/day
E. more than 5 mm/day
140. Which one is true according to the Nerve Conduction Studies (NCS)?
A. insertion Activity or intrinsic electrical activity are the first to record in NCS
B. NCS cannot be performed in cranial nerves. NCS should only be performed in
peripheral nerves.
C. NCS can be performed in three nerve types which is autonomic, motor and
sensory nerve conduction testing
D. NCS are done only by pricking the needle through the skin and muscle then start
stimulating it through electrical impulses
E. In NCS, you get direct information about the muscle and indirect information
about the nerves that supply the muscle that is being tested

141. How many electrode (s) used in Nerve Conduction Studies (NCS) and
Elecromyography (EMG)?
A. One electrode which is: active electrode
B. Two electrodes which is: active and ground electrode
C. Two electrodes which is: active and reference electrode
D. Three electrodes which is: active, ground and polarized electrode
E. Three electrodes which is: active, ground and reference electrode

142. Which one is abnormal spontaneous potentials that are generated specific at the
level of the muscle fiber?
A. Cramps
B. Tremors
C. Fasciculation
D. Positive sharp waves
E. Myokymic discharges

143. If a patient has tarsal tunnel syndrome, one would likely find which of the following
on nerve conduction studies?
810
A. Increased latency of the sural nerve at the ankle
B. Increased latency of the tibial nerve at the ankle
C. Decreased conduction velocity of the tibial nerve
D. Decreased conduction velocity of the sural nerve
E. Decreased conduction velocity of the peroneal (fibular) nerve

144. The motor unit action potential below was taken using a monopolar needle in the
quadriceps muscle.

What can you determine about the amplitude of the motor unit?
A. Normal
B. Increased latency
C. Decreased amplitude
D. Increased amplitude
E. Mixed increased and decreased amplitude

145. On needle electromyographic (EMG) testing, the muscle that would not be affected
in a lesion to the posterior cord is ……
A. Deltoid
B. Triceps
C. Pronator teres
D. Extensor carpi ulnaris
E. Extensor indicis proprius

146. Which indicator does not have to be measured to diagnose frailty syndrome?
A. Dysphagia
B. Visual impairment
C. Fatigue
D. Gait speed
E. Weakness

147. Which exercise and activities modification should be prescribed for patient with
Diabetes Mellitus type 2 and retinopathy complication?
A. high-intensity aerobic exercises
B. resistance physical activity
C. head-down activities
D. low intensity exercise
E. jumping activities

148. Which condition below is not a contraindication to the manipulation of the cervical
spine?
A. Osteoporosis
811
B. Herniated cervical disc
C. Acute cervical soft tissue injury
D. Spasm of the paracervical muscles
E. Fracture of the cervical vertebra

149. What is the most reliable physical sign of Myofascial Trigger Point Syndrome?
A. A rope-like nodular in the muscle and pattern of referred pain
B. Focal tenderness and pain recognition
C. Vasomotor and temperature changes
D. Limited of ROM
E. Muscle spasm

150. Para Articular Heterotropic Ossification in SCI patient most commonly occurs at the
following joint:
A. Hip
B. Knee
C. Ankle
D. Elbow
E. Shoulder

NATIONAL BOARD EXAMINATION INDONESIAN PHYSICAL MEDICINE AND


REHABILITATION
DECEMBER, 2020

CHOOSE THE MOST APPROPRIATE ANSWER

1. When intermittent catheterization in acute spinal cord injury (SCI) patient should
be started?
A. within 24 hours of injury
B. when regular, controlled fluid intake is occurring
C. when spinal shock is resolved
D. when the patient can perform self-catheterization
E. when the caregiver is determined

2. One week after onset of Bell palsy, what is the most reliable electro-diagnostic
parameter for predicting ultimate recovery?
A. Quantity of fibrillation potentials present in the denervated muscles
B. Recruitment abnormalities in the affected muscles
C. Blink reflex latencies
D. Side-to-side comparison of motor evoked amplitudes
E. Amplitude of the CMAP of the affected side

3. A 65-year-old female patient is evaluated after the acute onset of diplopia with
drooping eyelids. Examination is significant for ptosis and a large unreactive pupil on the
left. Her left eye rests in an abducted position with intorsion. She has intention tremor on
the right and an ataxic gait. Where is the area of brain lesion?
A. Left midbrain
B. Right midbrain
812
C. Left pons
D. Right pons
E. Cerebral peduncle

4. A 22-year-old male with C6 ASIA B tetraplegia secondary to a motor vehicle


accident 2 months ago is undergoing inpatient rehabilitation. His bladder is managed with
a suprapubic catheter and he is on a daily bowel program using digital stimulation and a
bisacodyl suppository. While resting supine in bed one evening, he suddenly develops a
pounding headache. His blood pressure is found to be 180/100 mmHg and his heart rate is
56. His face is flushed. What is the first step in the initial management of this patient?
A. Sit him up and loosen any restrictive clothing
B. Give him nifedipine 10 mg tablet
C. Apply ½ inch of nitro paste to his anterior chest wall
D. Flush his suprapubic catheter.
E. Using a well lubricated finger, check his lower rectum for fecal impaction

5. When a patient with recent onset of hemiplegia shows progressive motor recovery
from Brunnstrom Stage 1 to Brunnstrom Stage 3, what do you expect?
A. Decreasing tone and increasing isolated voluntary movements
B. Unchanged tone and increasing isolated voluntary movements
C. Decreasing tone and increasing synergistic movements
D. Increasing tone and increasing synergistic movements
E. Decreasing tone and decreasing synergistic movements

6. A patient presents with severe knee stiffness due to osteoarthritis. Which of the
following statement is true when considering injection of the joint?
A. Injection of the knee with intra-articular corticosteroids has been shown to provide
relief up to 12 months after injection
B. Tendon rupture and nerve atrophy are common complications of knee injection
C. Severe juxta-articular osteoporosis is a contraindication to intra-articular steroid
injections
D. Knee injection using the superomedial approach should be performed with the
patient’s knee flexed to 90 degrees
E. Using the superomedial approach, the needle is inserted medial to the patellar
tendon 1 cm above the tibial plateau

7. Which muscle that usually gets impact related to tennis elbow?


A. Extensor carpi radialis muscles
B. Extensor carpi ulnaris muscles
C. Extensor digitorum muscles
D. Pronator muscles
E. Supinator muscles

8. A 55-year-old woman presents with a three-month history of non-radicular low


back pain and has no history of trauma. X-rays show degenerative changes and grade 1
spondylolisthesis at L4-5. What is the best recommendation?
A. Bed rest for two weeks
B. Abdominal muscle strengthening
813
C. Isokinetic back extension strengthening
D. A polypropylene body jacket
E. TLS corset

9. What is the most common pathology underlying rotator cuff disorder in the non-
athlete?
A. Deltoid atrophy
B. Subacromial impingement
C. Acromioclavicular arthritis
D. Bicipital tendon disorder
E. Partial rupture of glenohumerale ligaments

10. In a boutonniere deformity, the proximal interphalangeal (PIP) joint is in …….


A. extension and the distal interphalangeal joint is in hyperflexion
B. extension and the distal interphalangeal joint is in hyperextension
C. flexion and the distal interphalangeal joint is in hyperflexion
D. flexion and the distal interphalangeal joint is in hyperextension
E. proximal and distal interphalangeal joint are in flexion

11. What is a contraindication for superficial heat?


A. Sensory deficit
B. Spasticity
C. Joint contracture
D. Hematoma
E. Superficial thrombophlebitis

12. The proper length for a cane should be measured with the elbow in which
position?
A. Full extension
B. 20 degrees of flexion
C. 45 degrees of flexion
D. 60 degrees of flexion
E. 90 degrees of flexion

13. A patient with osteoarthritis presents with pain in the carpometacarpal joint of the
thumb. What is the most appropriate splint?
A. Ring
B. Resting hand
C. Spring coil extension assist
D. Thumb spica
E. Cock up

14. During ambulation using a trans-femoral prosthesis, a patient raises his entire body
(vaults) with plantar flexion of the sound foot during the swing phase of the prosthetic
extremity. To decrease the vaulting, you should……….
A. tighten the socket suspension
B. lengthen the prosthesis
C. shorten the prosthesis
814
D. add a knee extension aid
E. set the prosthetic foot in a few more degrees of plantar flexion

15. Which of the following is commonly seen in a child with cerebral palsy?
A. Tight Quadriceps muscle
B. Tight Hamstrings muscle
C. Tight Tibialis Anterior muscle
D. Tight Gastrocnemius-Soleus complex
E. Tight Extensor Digitorum Longus muscle

16. Management of children Pes Planus grade 1 at age of one year is............
A. Serial Taping
B. Arches support
C. Thomas heel shoes
D. Chukka type shoes
E. Observation

17. In a child with Duchenne Muscular Dystrophy (DMD), which muscle group has the
most significant weakness at the time of presentation?
A. Distal upper extremities
B. Distal lower extremities
C. Proximal upper extremities
D. Proximal lower extremities
E. Trunk muscles

18. A 55 year-old man sustains an uncomplicated myocardial infarction. During the last
part of the hospital stay, he is doing all his own self-care, including showering and shaving,
without symptoms. Upon discharge to his home, he should…
A. be allowed to do stationary cycling with moderate-to-vigorous effort
B. be permitted activities of no higher than 2 METS
C. abstain from sexual activity
D. not be permitted walking at 2 miles per hour
E. be encouraged to perform activities of 3 to 4 METS

19. Chronic bronchitis in the absence of emphysema is characteristic by …………….


A. Loss of alveolar walls
B. Loss of radial traction around extra alveolar blood vessels
C. Hypertrophy of bronchial mucous glands
D. Reduced lung elastic recoil
E. Under-secretion of mucus by goblet cells.

20. For patient with Congestive Heart Disease, which of the following statements is
accurate?
A. Exercise capacity is improved because of peripheral adaptations
B. Exercise capacity is improved because of increase end-systolic volume
C. Most of the important resulting from regular exercise is within the myocardium
D. These patient can never expect improved physical fitness
E. Complete bed rest is prescribed for these patients
815
21. Which is physiologic factor in the elderly exacerbates orthostatic hypotension?
A. Decreased heart rate
B. Decreased peripheral resistance
C. Decreased baroreceptor response
D. Decreased creatinine clearance
E. Decrease in arterial stiffness

22. Weakness of which of the following is consistent with C7 nerve root radiculopathy?
A. Shoulder abduction and elbow flexion
B. Elbow extension and wrist flexion
C. Elbow flexion and wrist extension
D. Finger flexion and interossei muscles
E. Interossei muscles

23. In an older adult who previously had polio, new weakness is most consistent with
post- polio syndrome when it occurs
A. after recent disuse
B. in proximal muscle groups of both the upper and lower extremities
C. after recent weight gain
D. in the muscles most severely involved in the initial illness
E. after new gastrointestinal infection

24. A 34-year-old patient presents to the ICU with a traumatic brain injury after a
motor vehicle collision 1 day ago. The patient is unconscious however she had a normal CT
scan of her brain. The most likely cause is :
A. Arterial vasospasm
B. Diffuse axonal injury
C. Epidural hemorrhage
D. Subdural hemorrhage
E. Cerebral contusion

25. A patient with low back pain of six weeks’ duration has mild left calf atrophy,
diminished plantar flexion strength, and absent Achilles reflex. Electro-diagnostic testing
shows an absent H reflex; normal insertional activity in all muscles tested; moderately
large amplitude, mildly polyphasic motor units in the left medial gastrocnemius, the lateral
hamstring, and the gluteus maximus; and normal motor units elsewhere. What is the
diagnosis?
A. Acute S1 radiculopathy
B. Old tibial neuropathy in the thigh
C. Old S1 radiculopathy
D. Lumbosacral polyradiculopathy
E. L4-L5 radiculopathy

26. The optimal postoperative dressing for a trans-tibial amputee is a :


A. Elastic bandage
B. Shrinker dressing
C. Semirigid dressing
816
D. Rigid dressing
E. Rigid removable dressing

27. A 59-year-old woman sustained a stroke one year ago. She now has good voluntary
control of the affected arm, with elbow and shoulder strength at 4/5, wrist extension at
2/5, finger extension at 2/5, and fair grip. Tone is minimally increased throughout the arm
and sensation is intact. What is the most effective rehabilitation’s intervention for
improving her hand function?
A. Restraining the unaffected hand during activities of daily living
B. Electromyographic biofeedback for the wrist extensors
C. Alternating hot and cold stimulation to the affected hand
D. Intensive training of the unaffected hand in one-handed techniques
E. Strengthening exercise especially for all flexor muscles

28. A 50-year-old man with type 2 diabetes mellitus presents for evaluation of a
swollen ankle. An examination demonstrates diminished pinprick sensation in a stocking
pattern, absent ankle jerks, and a warm, erythematous right ankle with a mild effusion.
What is the most likely diagnosis relating to the right ankle?
A. Charcot joint
B. Rheumatoid arthritis
C. Septic arthritis
D. Avascular necrosis
E. Osteoarthritis

29. Spondylosis is common nonspecific term used to describe:


A. Inflammation of one or more vertebral joints
B. Inflammation of the vertebra, including the joints and entheses
C. Slippage of one of the vertebrae forward onto the bone directly beneath it.
D. Hypertrophic changes of the end plates and facet joints.
E. Defect of the pars interarticularis, or the isthmus between the superior and inferior
articular facets

30. A 38-year-old man sustained extensive third degree burns over both arms and
open fracture of the left tibia after a fiery motorcycle accident. Three weeks later, he is still
comatose but is now medically and physiologically stable. Therapy for his arms should
consist of which of the following?
A. An aggressive passive range of motion program to minimize contractures
B. Active assisted motion adjusted to the condition
C. Continuous resting splint to minimize heterotopic ossification
D. Splinting in 30 degrees of flexion to minimize soft tissue tension on the flexor side.
E. Splinting in 90 degrees of flexion to minimize soft tissue tension on the flexor side

31. The best example of an open kinetic chain exercise of the quadriceps is…….
A. a stair-climbing machine
B. a wind-resistance exercise bicycle
C. an isokinetic knee extension machine
D. a treadmill inclined to 7 degrees
E. sit to stand exercise
817
32. A patient complains of knee pain after falling on her flexed knee. Physical
examination shows a positive “posterior sag sign.” Which ligament is injured?
A. Medial collateral
B. Lateral collateral
C. Anterior cruciate
D. Posterior cruciate
E. Anterior and posterior cruciate

33. A 30-year-old male patient presents to you with a history of right upper extremity
below elbow amputation from a blast injury at work. He was previously an engineer on an
oil rig. He would like to eventually return to work. Which of the following is a reason a
myoelectric device is most appropriate for this patient?
A. A myoelectric device is typically lighter than a body-powered prosthesis
B. A myoelectric device provides a better ability to perform behind the back and
overhead activities with the prosthetic while maintaining mechanical strength
C. A myoelectric device has better function in wet or dirty conditions compared to a
body-powered prosthesis
D. A myoelectric device can be used for a longer period of time by the amputee in
comparison to a body-powered prosthesis
E. A myoelectric device is relatively cheap

34. Which condition would qualify as impairment according to the International


Classification of Functioning Disability and Health (ICF)?
A. Short stature
B. Loss of hearing due to occupational exposure
C. Inability to work due to chronic illness
D. Inability to walk one block
E. Inability to communicate with others

35. Which modality should precede shoulder stretching in a patient with adhesive
capsulitis?
A. Massage
B. Microwave Diathermy
C. Ultrasound
D. TENS
E. Low level laser

36. One week after a patient began a resistance-strengthening program, you note a
15% increase in the force of their maximum voluntary contraction. This is most likely
secondary to a change in the
A. motor unit recruitment pattern
B. number of myofibrils
C. number of muscle fibers present
D. fiber density
E. myoneural junction

37. The Functional Independence Measure (FIM) assesses


818
A. impairment
B. activity limitations
C. participation restrictions
D. prognosis
E. therapy program

38. What is the function of the ilio-femoral ligament?


A. Limit extension, abduction, and external rotation of the hip
B. Limit extension, adduction, and internal rotation of the hip
C. Limit extension and external rotation of the hip
D. Limit flexion, abduction, and external rotation of the hip
E. Limit flexion, adduction, and internal rotation of the hip

39. All of the following are true about piriformis syndrome except:
A. The sciatic nerve may be involved because in some individuals the nerve runs
through the piriformis muscle fibers
B. Pain may be in lateral buttock, posterior hip and thigh, sciatica-like pain
C. Pain with flexion, abduction, and external rotation
D. Pain may be exacerbated by walking up stairs or prolonged sitting
E. Radiologic tests such as MRIs may be required to rule out other causes of sciatic
nerve compression

40. Which ligament is the weakest of the ankle ligaments?


A. Anterior talofibular ligament
B. Posterior talofibular ligament
C. Calcaneo-fibular ligament
D. Deltoid ligament
E. Tibiofibular ligament

41. A tibial nerve injury is characterized by………


A. Weakness of knee flexion
B. Weakness of knee extension
C. Weakness of ankle dorsiflexion
D. Weakness of foot eversion and dorsiflexion
E. Weak foot inversion and plantar flexion

42. Which following tendon involved in De Quervain’s tenosynovitis?


A. Extensor pollicis longus (EPL) and flexor digitorum superficialis (FDS)
B. Abductor pollicis brevis (APB) and flexor digitorum profundus (FDP)
C. Extensor pollicis longus (EPL) and extensor pollicis brevis (EPB)
D. Extensor pollicis brevis (EPB) and abductor pollicis longus (APL)
E. Flexor carpi radialis (FCR) and palmaris longus (PL)

43. What type of T-strap is used for someone with a valgus ankle deformity?
A. Superior T-strap
B. Posterior T-strap
C. Posteriolateral T-strap
D. Lateral T-strap
819
E. Medial T-strap

44. A 25 years-old man came with lower neck pain after suffering a car accident. On
the examination you reveal a soft tissue injury and cervical ligamentous sprain. Which
orthotic you choose for this patient?
A. Soft cervical collar
B. Hard cervical collar
C. Philladelphia collar
D. Sterno-occipital mandibular immobilizer (SOMI)
E. Minerva brace

45. A newborn is holding his head with his chin rotated toward the left and the ear
approximating the right shoulder. Which muscle is primarily implicated?
A. Left cervical paraspinal
B. Right cervical paraspinal
C. Left sternocleidomastoid
D. Right sternocleidomastoid
E. Right splenius capitis

46. A 5 months old baby with Spina Bifida presents with this lower extremity
deformity. What is the abnormality of his feet?

A. Equinovarus
B. Equinovalgus
C. Vertical talus
D. Talipes cavus
E. Calcaneovalgus

47. The Ortolani test will be positive in the following condition…..


A. Congenital lower limb deficiency
B. Congenital hemivertebrae
C. Congenital hemi-sacralization
D. Congenital hip dislocation
E. Congenital hydrocephalus

48. At what developmental stage should a child with a congenital transverse radial
limb deficiency have an initial prosthesis fitting?
A. As soon as possible after birth
B. At the time of initial rolling
C. At the time of first sitting independently
D. At the time of initial walking
E. At the time of starting kinder garden
820
49. Which of the following complications below is the most common risk factor for
cerebral palsy (CP)?
A. Premature birth
B. TORCH congenital infections
C. History of maternal seizures
D. Maternal bleeding
E. Age of the mother

50. A 3 month-old girl has a congenital torticollis, with the chin turn to the left and the
head tilts significantly to the right. On examination you found no palpable fibrosis of the
neck muscles. What is the next you should do?
A. Prescribe semi-rigid cervical collar
B. Instruct the parents to stretch gentle the left sternocleidomastoideus muscles
C. Prescribe Dantrolene sodium to control muscular spasms
D. Examine the hips to rule out an associated hip dysplasia
E. Advice surgical consultation for operative intervention

51. Which of the following statement bellow is not of the beneficial effect of exercise
training in cardiac rehabilitation?
A. Lowering resting blood pressure
B. Reducing plasma glucose
C. Increasing LDL cholesterol
D. Increasing HDL cholesterol
E. Increasing peripheral resistance

52. Which option describes the pulmonary dysfunction seen in patients suffering from
Duchenne Muscular Dystrophy (DMD)?
A. Decreased lung compliance and increased atelectasis secondary to loss of
surfactant
B. Restrictive lung disease due to weakness of the diaphragm, chest, and abdominal
walls
C. Hyperventilation with signs of respiratory alkalosis due to hyper-reactive
diaphragm
D. Impaired innervation of the phrenic nerve
E. Stiffness of the shoulder causes the restriction in lung expansion

53. A patient with a recent traumatic brain injury (TBI) developed right lower extremity
spasticity and knee flexion contracture which has been treated unsuccessfully with
Ultrasound Diathermy and stretching. What is the next step in the management of this
patient?
A. Short Wave Diathermy
B. Electrical stimulation
C. Hydro Pool Therapy
D. Serial casting
E. Surgical tendon lengthening

821
54. Which cardiopulmonary parameter should improve in a patient with congestive
heart failure who is participating in a cardiac rehabilitation program?
A. VO2max
B. Systolic ejection fraction
C. Diastolic ventricular filling
D. Pulmonary blood flow
E. Dilatation of vascular

55. Goal for children with developing language in syntax area is to increase ………………
A. intelligibility
B. production of closed syllables
C. consonant repertoire
D. sentence length
E. vocabulary size

56. Which of the following statement about lung disorders is true?


A. Restrictive lung disease sometimes a primary ventilatory impairment
B. Restrictive lung disease always primary oxygenation impairment
C. Obstructive lung disease always a primary ventilatory impairment
D. Obstructive lung disease sometimes primary oxygenation impairment
E. Chronic obstructive lung disease can be oxygenation and ventilatory impairment

57. Cardiac output is defined as a product of which of the following components?


A. Heart rate and stroke volume
B. Heart rate and aerobic capacity
C. Stroke volume and oxygen consumption
D. Ejection fraction and aerobic capacity
E. Myocardial oxygen capacity and heart rate

58. What is the most likely goal of inpatient rehabilitation for cancer palliative care
patient?
A. Contracture management
B. Transfer training
C. Endurance training
D. Bladder training
E. Caregiver training

59. Which of following condition did not limit cancer’s patient doing their exercise
program?
A. Cachexia
B. Bone metastasis
C. Fluctuating periods of sickness
D. Low risk of microbial infection
E. Cancer-related fatigue

60. Chin-tuck maneuver is safe and effective for a patient with dysphagia following
stroke, because it ………
A. increases pharyngeal peristalsis
822
B. decreases esophageal sphincter closure
C. decreases pharyngeal pressure
D. decreases airway opening
E. helps the pharyngeal closure

61. Which type of aphasia is characterized by non-fluency, an intact ability to repeat,


and intact comprehension?
A. Wernicke
B. Broca
C. Conduction
D. Transcortical motor
E. Transcortical sensory

62. Which of the following assessment is not to be emphasized in the evaluation for
rehabilitation of a person with Parkinson disease?
A. the degree of rigidity and bradykinesia using the UPDRS to see how these
symptoms interfere with ADL
B. analyze equilibrium by tandem walking
C. fine motor task such as writing
D. restriction in joint mobility
E. phonation and articulation of the speech

63. A 38-year-old secretary has a two-year history of severe right radial wrist pain,
which is worse with pinching activities and wrist ulnar deviation. Job modification,
extensive physical therapy, and multiple cortisone shots have failed to provide much relief.
What is the best recommendation?
A. Wrist orthotic in resting position
B. Suggestion to rest the right hand from heavy activities
C. Referral to a pain intervention program
D. Wrist tendon lengthening surgery
E. Surgical decompression of the first extensor compartment

64. When poor initiation is noted during rehabilitation of a patient with a traumatic
brain injury (TBI), which region of the brain is the suspected the location of injury?
A. Frontal
B. Temporal
C. Parietal
D. Occipital
E. Cerebellum

65. Which of the following is the most common etiology of reversible cognitive
decline?
A. Trauma of the brain
B. Vitamin B12 deficiency
C. Depression
D. Hypothyroidism
E. Normal pressure Hydrocephalus

823
66. Which of the following has the poorest prognosis of nerve recovery?
A. Axonotmesis
B. Conduction block
C. Demyelination
D. Neurapraxia
E. Neurotmesis

67. Which provocative test evaluates both the hip and sacroiliac joints?
A. Straight leg raise Test or Lasègue
B. Patrick Test or FABER
C. Trendelenburg Test
D. Thomas Test with resistance applied
E. Ober’s Test

68. A 6-years old boy with hemophilia has suffered a recurrent hemarthrosis of the
knee. After providing adequate factor VIII replacement, the most important therapy in the
first 48 hours is…….
A. Immobilization
B. Active assistive ROM exercise
C. Icing and passive ROM exercise
D. Ambulation with full weight bearing
E. Ambulation with partial weight bearing

69. What is the most common cause of knee pain in runners?


A. Diskoid meniscus
B. Hamstring tendinitis
C. Baker cyst
D. Partial rupture of PCL
E. Patellofemoral pain syndrome
70. A 15 year old boy who actively competes in basketball and soccer presents with a
complaint of knee pain exacerbated with activity. There is no history of trauma. The
patient is afebrile with normal vital signs. On examination, there is tenderness at the
anterior tibial tubercle with swelling of the proximal tibia, pain with resisted knee
extension, but no instability is noted and Lachman’s test is negative. Examination is
otherwise normal. Which of the following is the etiology of the patient’s likely disorder?
A. Strain of the anterior cruciate ligament
B. Referred pain from the hip
C. Repetitive contraction of the quadriceps
D. Degenerative joint
E. Hematogenous spread of infection

71. Which of the following exercises involve the least amount of force on the low back
and are therefore used in the early stages of lumbar rehabilitation?
A. Leg raises
B. Sit ups
C. Curl ups
D. Pin on plank
E. Lying prone and extending the spine while extending arms and legs
824
72. Myodesis is sometime necessary during amputation surgery. Which of following
statements best describe the definition of myodesis and when it is indicated?
A. when opposing muscles are sutured separately to opposing ends of the end of the
cut bone; performed when patient has sufficient blood supply
B. when muscles and fasciae are sutured directly to bone via drill holes; performed
when bone has sufficient blood supply
C. when opposing muscles are sutured to each other and the periosteum at the end
of cut bone; performed when patient has severe anemia
D. when muscles are left with the cut ends unanchored and the residual limb skin is
sutured tightly over the end of cut bone; performed when patient has
uncontrolled diabetes
E. when muscles are sutured to opposing fasciae at the end of cut bone; performed in
crushed injuries

73. A method to apply topical medications through the skin with a coupling medium
using ultrasound in treating soft tissue injury and inflammation is called ……….
A. Diathermy
B. Iontophoresis
C. Phonophoresis
D. Faradisation
E. Ultrasonic bubbling

74. Which recreational activity is not suggestive following a total knee arthroplasty?
A. Dancing
B. Double Tennis
C. Jogging
D. Cycling
E. Swimming
75. Which of the following muscles are not the rotator cuff muscles?
A. Teres minor
B. Supraspinatus
C. Infraspinatus
D. Rhomboids
E. Subcapularis

76. Which trunk(s) of the brachial plexus contribute(s) to the radial nerve?
A. Upper and lower trunks
B. Upper and middle trunks
C. Upper, middle, and lower trunks
D. Lower trunk
E. Lower and middle trunks

77. What is the proper position of the hand splint to prevent claw deformity?
A. Wrist extended, metacarpophalangeal (MCP) joint flexed, proximal
interphalangeal (PIP) joint extended, distal interphalangeal (DIP) joint extended
B. Wrist flexed, MCP flexed, PIP flexed, DIP flexed
C. Wrist extended, MCP extended, PIP extended, DIP extended
825
D. Wrist flexed, MCP extended, PIP flexed, DIP flexed
E. Wrist extended, MCP flexed, PIP flexed and DIP flexed

78. A 75 years old man suffered an unstable T12-L1 spine fracture. Due to his
comorbidities a surgery is not possible. Which type of brace you choose for this patient?
A. Halo Brace
B. Jewett Brace
C. Minerva Brace
D. Taylor’s brace
E. Knight Brace

79. A child should be able to maintain his or her head in the midline position by what
age?
A. 2 months
B. 3 months
C. 4 months
D. 5 months
E. 6 months

80. Which lower limit of normal innervation will the rate of hip dislocation be the
highest in a child with myelomeningocele?
A. T-12 or higher
B. L-1
C. L-2
D. L-4
E. L-5

81. A 4-year-old boy presents with a history of difficulty climbing stairs, falling,
waddling gait, and large calf muscles. You suspect a Duchenne Muscular Dystrophy (DMD).
What is the easiest and best confirmatory diagnosis test for this patient?
A. Serum level of creatine kinase (CK)
B. Polymerase chain reaction (PCR) genetic test
C. Muscle biopsy
D. Endocrine profile
E. Family history

82. Good prognostic indications for ambulation in children with cerebral palsy (CP)
include all of the following except ……
A. Independent sitting by 2 years of age
B. Fewer than three primitive reflexes by 18 months of age
C. Unilateral spastic cerebral palsy and IQ > 50.
D. Hemiplegic CP
E. Term birth

83. With the asymmetric tonic neck reflex (ATNR), lateral rotation of the head on the
trunk produces which of the following?
A. Flexion in the upper and the lower limbs on the nasal side, and extension of both
limbs on the occipital side
826
B. Flexion in the upper and the lower limbs on the nasal side, and flexion of both
limbs on the occipital side
C. Extension in the upper and the lower limbs on the nasal side, and flexion of both
limbs on the occipital side
D. Extension in the upper and the lower limbs on the nasal side, and extension of both
limbs on the occipital side
E. Extension in the upper and the lower limbs on the nasal side, and neutral
anatomic position of both limbs on the occipital side

84. Which of the following activities you don’t expect from three years old child to be
able to complete?
A. Up and down stairs with hands on rail
B. Jumps clearing ground and lands on feet together
C. Walks down stairs alternating feet
D. Pedals tricycle
E. Walking around

85. Which of the following is a physiologic measure expressing the energy cost of
physical activities?
A. Metabolic oxygen consumption
B. Mean exercise training
C. Measure of exercise tolerance
D. Metabolic equivalent of task (MET)
E. Mean basal metabolic rate

86. Which of the following is not a proven therapeutic benefit of cardiac rehabilitation
after a myocardial infarction (MI)?
A. Improved left ventricular (LV) function
B. Increased resting cardiac output
C. Increased maximum cardiac output
D. Decreased rate of recurrent MI
E. Decreased mortality

87. On the basis of metabolic equivalent of a task, a patient having which of the
following metabolic equivalent (MET) levels should not return to employment after cardiac
rehabilitation?
A. MET 7 to 8
B. MET 6 to 7
C. MET 5 to 6
D. MET 4 to 5
E. MET 2 to 3

88. An 80-year-old man presents for his routine annual physical examination. His wife
notes that he has become much less active in the prior six months. You decided to assess
his degree of frailty with the Physical Frailty Phenotype. Which of the following are
included in this tool to assess frailty?
A. A history of chronic lung disease
B. Decreased grip strength
827
C. Inability to rise from the chair five times without use of arms
D. A “yes” response to the question “Do you have difficulty walking one block?”
E. A “no” response to the question “Do you feel full of energy?”

89. During an acute episode of dyspnea in chronic obstructive pulmonary disease


(COPD) patients, which breathing technique may help to reduce symptoms and the work
of breathing?
A. Huffing
B. Controlled cough
C. Breath holds
D. Pursed-lip breathing
E. Segmental breathing

90. Which of the following is a useful tool in measuring exertion during a physical
activity?
A. Wong-Baker scale
B. Borg scale
C. Ranchos Los Amigos scale
D. Disability rating scale
E. Fugl Meyer Scale

91. Which of the following condition below is not absolute indication for terminating
exercise testing?
A. Dislodged ECG leads
B. Subject’s desire to stop
C. Onset of 2nd or 3rd degree AV block
D. Increased systolic blood pressure until 180 mmHg
E. Increased diastolic blood pressure more than 120 mmHg

92. Which test best assesses the magnitude of functional impairment in pulmonary
disease?
A. Chest radiography
B. Arterial blood gas measurements (ABG)
C. Pulmonary function tests (PFT)
D. Ventilation perfusion (V/Q) lung scan
E. Incentive Spirometer

93. Which of following statement about Six Minutes’ Walk Test is true?
A. 6 MWT is a test that can aid in assessing the functional capacity of patients with
cardiopulmonary disease
B. 6 MWT is tested by retro-walking reciprocally along a 30 meter lap without cheer
within 6 minutes.
C. The outcome of this test is lap time measured in minutes.
D. Lap distance is estimated from body weight and height adjusted to gender,
measured in meter.
E. 6 MWT is heavily carried out, hard tolerated by patients with COPD, and its safety
and reliability have not been assessed.

828
94. Which one is not related with the leg discomfort as a cause of daily activity
limitation in chronic cardiorespiratory problem?
A. The origin is due to a number of histological and metabolic alternations manifested
in peripheral muscles
B. Shift from type I oxidative to type II glycolytic muscle fibers
C. Decrease in aerobic enzyme activity and mitochondrial volume density
D. Decrease in muscle capillary density
E. All fiber types are remain across the peripheral muscles

95. Which of the following is true regarding falls in the geriatric population?
A. Most falls are considered accidental, rather than related to underlying diseases or
functional impairments
B. The timed “Get up & Go” test is the appropriate way to assess someone who
presents with frequent falls
C. Dynamic balance training activities such as Tai Chi have no effect on fear of fall
D. Risk of fall assessment should be evaluate for both intrinsic and extrinsic risk
factors
E. Joint problem is the most common cause of fall in the elderly

96. Axonal damage with Wallerian degeneration would present with:


A. Decreased CMAP amplitude with proximal stimulation but not distal stimulation
B. Decreased CMAP amplitude distally but not proximally
C. Decreased compound motor action potential (CMAP) amplitude with proximal
stimulation and distal stimulation
D. Slowing of conduction velocity across the lesion
E. Slowing of conduction velocity distal to the lesion

97. Which of the following condition is not likely cause of swallowing difficulty?
A. Nerve lesion
B. Tumor
C. Aging
D. Psychological
E. Gastrointestinal disorders

98. Neuroplasticity is a concept that refers to all of the following except:

A. The potential ability of the central nervous system (CNS) to modify its structural
and functional organization
B. Partial recovery is possible long after sustaining a brain injury
C. The brain remains capable of changing in response to experience and injury
D. Insult or injury to the CNS is permanent and functional ability cannot be altered
with any type of intervention
E. The healthy area of the brain is capable of taking over the functions of the injured
part of the brain.

99. A patient with gangrene of the left distal foot has the following segmental
pressures as follow: Ankle brachial index 0.50; Lower calf brachial index 0.60; Upper calf

829
0.90; Lower thigh 1.00; Upper thigh 1.10. What level of amputation is recommended for
optimal healing and ambulation?
A. Syme
B. Transtibial
C. Knee disarticulation
D. Transfemoral
E. Hip disarticulation

100. Which of the following statements regarding constraint-induced movement


therapy (CIMT) is true?
A. It requires constraint of the affected extremity
B. It utilizes a passive non intensive approach
C. It aims to increase the use of the unaffected extremity
D. It must be trained in the non-distracted room
E. It is based on principles of repeated practice and intense activity on the affected
side

101. A 43-year-old runner presents with right-sided hip pain. On examination, you note
tenderness to palpation over the greater trochanter on the affected side. No focal
neurologic deficits are noted and no other areas of tenderness are elicited. A key part of
your treatment plan should include strengthening of the hip ……………
A. flexor
B. extensors
C. internal rotators
D. abductors
E. adductors

102. Which statement is not true regarding plantar fasciitis?


A. Increased tension on the plantar fascia leads to chronic inflammation
B. Heel spurs may contribute to its etiology
C. A tight Achilles tendon is frequently associated with plantar fasciitis
D. Night plantar flexion splints are not indicated
E. The pain is usually the worst with the first few steps after awakening

103. Which following initial therapeutic exercise regimen should be included for a burn
patient, who has not undergone a skin graft?
A. Passive range of motion toward the contractile force of the scar
B. Passive range of motion against the contractile force of the scar
C. Active assisted range of motion toward the contractile force of the scar
D. Active range of motion against the contractile force of the healing scar
E. Active range of motion toward the contractile force of the scar

104. As a physiatrist you know that dystrophin deficient muscle is very susceptible to
exercise- induced muscle injury. The best way to begin patients with Duchenne Muscular
Dystrophy (DMD) on an exercise program is …..
A. To allow the boys to play as hard and as long as they want, stopping only when
they are too tired to play any more

830
B. To have the boys exercise in a playful manner but with the supervision of a physical
therapist
C. To have the boys participate in the standard school physical program
D. To start with home exercise 45 minute 3 times a week
E. The exercise program is not a good idea, as it could make the disease progress
faster

105. Neural strategies of functional improvement after central nervous system injury
include all of the following except:
A. Restoration
B. Remediation
C. Redacting
D. Recruitment
E. Retraining

106. Not included in the stages of fracture healing is …………..


A. Hematoma formation
B. Bone deformation
C. Fibro-cartilaginous callus formation
D. Bony callus formation
E. Bone remodeling

107. A high Q angle of the patella is often associated with …………


A. a low-riding patella (patella infra)
B. a high-riding patella (patella alta)
C. varus deformities of the knee
D. osteonecrosis of the tibial plateau
E. recurrent subluxation of the patella

108. Which of following is not correct about Extracorporeal Shock Wave Therapy
(ESWT)?
A. involves delivery of shock waves to injured soft tissue
B. promote neovascularization at the tendon-bone junction
C. amplify growth factor and protein synthesis to stimulate collagen synthesis and
tissue remodeling
D. an invasive treatment
E. stimulate proliferation of tenocytes and osteoprogenitor differentiation

109. Which of musculoskeletal effects can occur after aquatic rehabilitation?


A. Mean stroke volume increases 35% on average
B. Increase work of breathing 60%
C. Decrease heart rate after 40 minute immersion
D. Potassium excretion also increases with immersion
E. Hydrostatic forces add an additional circulatory drive to remove edema, muscle
lactate and other metabolic end products

110. Which of following condition is Intermittent Compression Pumps may not be used?
A. Amputations
831
B. Traumatic edema
C. Lymphedema post mastectomy
D. Arterial insufficiency
E. Venous insufficiency

111. Which crutches gait describes two crutches in contact with the floor and moving
both limbs past the crutches?
A. Two-point gait
B. Three-point gait
C. Four-point gait
D. Swing-through gait
E. Swing-to gait

112. After Total Hip Replacement with lateral approach, you instruct the patient to
avoid which of following movement?
A. abduction greater than 20 degrees
B. adduction beyond neutral
C. flexion greater than 45 degrees
D. flexion less than 45 degrees
E. external rotation beyond neutral

113. Which anatomic structure should be used as a reference point in order to


determine the proper backrest height for a standard wheelchair?
A. C7 Vertebrae level
B. T1 Vertebrae level
C. T4 Vertebrae level
D. Posterior 12th rib
E. Scapula

114. What will be the result of a heel cushion of a prosthesis that is too soft?
A. Excessive knee flexion
B. Insufficient knee flexion
C. Excessive knee extension
D. Insufficient knee extension
E. Excessive ankle dorsi-flexion

115. Children with cerebral palsy should be started to have therapy…..


A. When they begin to show signs of walking
B. When they display limitations in fine motor skills
C. Only after the diagnosis is confirmed
D. Even if the diagnosis is not established and abnormal muscle tone exists
E. When they start to understand a sentence

116. In infants, at what level does the spinal cord terminate?


A. T10
B. T12
C. L2
D. L4
832
E. L5

117. Children with cerebral palsy (CP) are at risk for fractures and osteoporosis because
of all of the following except:
A. Immobility
B. Antiepileptic medication
C. Chronic respiratory infection
D. Always play in the isolated and closed room
E. Malnutrition with calcium and vitamin D deficiency

118. Common presentations of myopathy in infants and children include all of the
following
except:
A. Hypotonia
B. Delayed motor milestones
C. Feeding problem
D. Language problem
E. Abnormality of gait

119. A 12-year-old girl with limb girdle muscular dystrophy is having trouble keeping up
in school. She cannot ambulate fast enough to get to her next class on time. She is having
difficulty writing her examination papers and finishing on time. The ultimate cause of the
majority of her clinical problems is:
A. Skeletal muscle weakness
B. Spasticity
C. Cardiomyopathy
D. Joint contracture
E. Decreased cardiorespiratory endurance

120. Which of following statement is the most proper for effect of exercise on COPD
patient?
A. There is an normal mechanism of respiratory system in COPD
B. There is no limited respiratory capacity
C. The ability to increase ventilation in agreement to the demand of oxygen supply
for exercise due to breathless as the main complaint.
D. The peripheral muscular dysfunction and muscular fatigue also cannot limit the
exercise capacity leading to reduced exercise tolerance
E. In the fact, the training program applied to the patients is not able to increase
exercise tolerance

121. For a patient with CAD, good left ventricular function, and no history of myocardial
infarction, exercise intensity should approximately be
A. to anginal threshold
B. 5 bpm (beats per minute) below anginal threshold
C. 10 bpm below anginal threshold
D. 15 bpm below anginal threshold
E. 20 bpm below anginal threshold

833
122. Which of the statements below is true regarding component of exercise
prescription?
A. The selection of mode not necessarily based on the desired outcomes, focusing on
exercises that are most likely to sustain participation and enjoyment
B. Intensity is the relative physiologic difficulty of the exercise
C. Intensity and duration of exercise does not interact and are not inversely related
D. Progression (overload) of exercise sometimes is not necessary
E. Duration or time is the length of warm up, conditioning, cool down, and
stretching phases

123. The amount of gas moving in and out of the lungs during resting respiration is
called:
A. Tidal volume
B. Vital capacity (VC)
C. Total lung capacity (TLC)
D. Forced vital capacity (FVC)
E. Forced expiratory volume in 1 second (FEV1)

124. What is the cause of airflow limitation in emphysema?


A. Narrowed airway caliber
B. Neuromuscular weakness of the chest wall
C. Loss of elastic recoil and decreased air tethering
D. Upper airway obstruction
E. Tightness of the chest muscles

125. During pediatric neuro-rehabilitation, all of the following are common focuses of
rehabilitation except:
A. Tone abnormalities and spasticity
B. Preventing deep vein thrombosis
C. Preventing heterotopic ossification
D. Chronic subdural hematoma
E. Preventing secondary deformities

126. By what age should the palmar grasp reflex disappear?


A. 2 months
B. 4 months
C. 6 months
D. 12 months
E. 18 months

127. What is the benefit of partial body weight supported gait training?
A. Ambulation training for patients who have sufficient strength and balance
B. Enhances development of compensatory gait strategies
C. Provides earlier weight-bearing to increase strength and increase spasticity
D. Allows for the simulation of task-specific walking movements
E. Assist certain impaired functions and ease movements of the trunk

834
128. Which of the following statements below is true regarding maximal versus sub
maximal exercise testing?
A. Maximal tests require participants to exercise to the point of volitional fatigue
B. Sub-maximal tests require participants to exercise to the point of fatigue
C. Result of sub-maximal exercise testing can’t be used to prescribe an exercise
D. Result of sub-maximal exercise testing can’t be used to predict VO2max
E. Examples of maximal exercise testing are 6-minute walking test and 12-meter walk
test

129. A 16-year-old football player suffers a T10 fracture. He is paraplegic and has no
pain or temperature sensation below T10. However, his proprioception, light touch,
pinprick, and vibration are intact. These findings are suggestive of:
A. Anterior spinal cord syndrome
B. Posterior spinal cord syndrome
C. Brown-Sequard syndrome
D. Conus medullaris syndrome
E. Central cord syndrome

130. A 22 years old patient sustained a spinal cord injury after fell from a tree. On
physical examination you found as follow:
Motor key muscles: C5—5/5; C6—5/5; C7—3/3; C8—2/2; T1—2/2; L1—1/1; L2—1/1;
L3—1/1; L4—0/0; L5—0/0; Voluntary anal contraction was weak.
Sensory pinprick and light touch to the armpit were intact; impaired (1) from armpit to
rectum with pinprick and light touch sparing at S4 to S5. Anocutaneal and bulbocavernous
reflexes positive. What is the functional diagnosis of this patient?
A. T1 AIS C
B. C6 AIS C
C. C6 AIS B
D. C7 AIS C
E. C7 AIS D

131. Neuroplasticity is not positively influenced by:


A. Stimulated Environment
B. Repetition of tasks
C. Motivation
D. Compensation
E. Demand or need

132. Which type of aphasia would you expect to see in a patient who is noted to have
fluent speech, with signs of comprehension, but unable to repeat?
A. Anomic aphasia
B. Conduction aphasia
C. Transcortical sensory aphasia
D. Transcortical motor aphasia
E. Wernicke’s aphasia

133. Unawareness of illness in patients with spatial neglect is called…..


A. Asomatognosia
835
B. Anosodiaphoria
C. Anosognosia
D. Apraxia
E. Aprosodia

134. According to the American Spinal Injury Association (ASIA) International Standards
for Neurological Classification of Spinal Cord Injury (ISNCSCI), testing of a key muscle
should begin in which grade position?
A. 1
B. 2
C. 3
D. 4
E. 5

135. Restrictive lung disease is common in severe myopathies and is due primarily to:
A. Recurrent pneumonia
B. Lung Mycosis
C. Obesity
D. Respiratory muscle weakness
E. Intrinsic lung damage

136. The Wee-FIM (pediatric functional independence measure), functional assessment


in the pediatric population may be measured beginning at what age?
A. 3 months
B. 6 months
C. 12 months
D. 24 months
E. 36 months

137. Myofascial pain syndrome is characterized by…


A. Tender points
B. Trigger points
C. Widespread tenderness
D. No change in muscle tension
E. Muscle spasm

138. Which of the following is not one of the six determinants of gait?
A. Pelvic tilt
B. Knee and hip flexion
C. Lateral displacement of the pelvis
D. Knee extension
E. Foot/ankle motion

139. What should be closed monitored by rehabilitation physician for


myelomeningocele (MMC) children as they move to adulthood?
A. Rechecks as needed
B. Avoidance of activity
C. Insistence on employment
836
D. Arrange marriage
E. Anticipation and prevention of life-threatening events

140. A 75-year-old woman, who sustained a stroke with right hemiplegia and severe
receptive aphasia three months ago, is being evaluated for electromyography and force
biofeedback after developing improvements in voluntary motor control. You advise
against this therapy because of her……..
A. age
B. aphasia
C. duration of stroke
D. shoulder subluxation
E. walking prognosis is poor

141. All of the following measurement is necessary for maintaining the skin integrity in
stroke patients except:
A. Frequent turning and positioning
B. Protection from moisture
C. Decreasing patient mobility
D. Maintenance of adequate nutrition and hydration
E. Use moisturizing cream and skin protectants

142. A 25-year-old woman presents with distal weakness, dysarthria, and dysphagia
with facial weakness. Physical examination reveals normal sensation, areflexia, and diffuse
weakness. What is the earliest electro-diagnostic finding expected in this patient?
A. Abnormal late responses
B. Prolonged distal onset motor latencies
C. Absent sural sensory response
D. Low-amplitude CAMP
E. High-amplitude CMAP

143. A 70-year-old woman is referred for rehabilitation following a recent left total hip
replacement surgery due to neck femur fracture. Operation was conducted through lateral
approach. To prevent hip dislocation, you instruct the patient to avoid following activity
except....................
A. Side lying on the affected side
B. cross the legs at the knees
C. reach down to pull up blankets when lying in bed
D. bring the knee up higher than the hip
E. pick up something on the floor while sitting

144. A 55-year-old retired athlete jumper with diabetes complains of pain on


ambulation due to degenerative joint disease in his right hip. You prescribe a single cane
to
A. improve balance and trunk support
B. increase the base of support, thus increasing stability and balance
C. compensate for gluteus medius weakness
D. decrease weight-bearing forces across the arthritic hip joint
E. help in sensory feedback or environmental scanning
837
145. Which of the following statements is true regarding the use of continuous passive
motion (CPM) following total knee arthroplasty?
A. The use of CPM has been associated with a decreased incidence of deep vein
thrombosis
B. The use of CPM has not demonstrated any difference in clinical outcomes at 1 year
following surgery
C. The use of CPM prevents the incidence of knee flexion contracture
D. The use of CPM increases analgesic use in patients who used CPM following TKA
E. The use of CPM shorten the hospital length of stay

146. Rehabilitation in patients with patella-femoral syndrome is focused on


strengthening of which muscle?
A. Biceps femoris
B. Rectus femoris
C. Vastus lateralis
D. Vastus medialis obliquus
E. Vastus intermedius

147. During gait evaluation of a patient, you observe a right genu recurvatum during the
stance phase. Which of the following could be the cause?
A. Right-ankle dorsiflexion contracture
B. Right-ankle plantar flexion contracture
C. Right hamstring spasticity
D. Right hamstring weakness
E. Right gluteus medius weakness

148. Which of the following statements about nutritional issues of CP Children is not
true?
A. Under nutrition is a goal in the management of cerebral palsy
B. Triceps skin fold helps determine nutritional status in cerebral palsy
C. Poor nutritional intake is associated with decreased community participation
D. Poor swallow, reflux, and constipation contribute to poor nutrition in cerebral palsy
E. Low copper and manganese levels can cause weak bones, neurological function
issues, growth problems, and increase a child’s risk of infection.

149. What will happen, when parents of children with cerebral palsy active involved in
the therapy?
A. Children are more dependent
B. Therapy is less important for function
C. Compliance at home is improved
D. Leisure time is reduced
E. Children become whiny

838
150. A 73 year-old woman was referred to your clinic with rheumatoid arthritis on both
hands. Which of the following is not a part of rehabilitation intervention?
A. Resting the involved joints
B. Heavy exercise of the involved joints
C. Joint protection instructions
D. Splinting regimens
E. Heat modalities

839

You might also like